Вы находитесь на странице: 1из 2327

Introduction to Methods of Applied Mathematics or Advanced Mathematical Methods for Scientists and Engineers

Sean Mauch March 19, 2003

Contents
Anti-Copyright Preface 0.1 Advice to Teachers . . . . 0.2 Acknowledgments . . . . 0.3 Warnings and Disclaimers 0.4 Suggested Use . . . . . . 0.5 About the Title . . . . . xxiv xxv xxv xxv xxvi xxvii xxvii

. . . . .

. . . . .

. . . . .

. . . . .

. . . . .

. . . . .

. . . . .

. . . . .

. . . . .

. . . . .

. . . . .

. . . . .

. . . . .

. . . . .

. . . . .

. . . . .

. . . . .

. . . . .

. . . . .

. . . . .

. . . . .

. . . . .

. . . . .

. . . . .

. . . . .

. . . . .

. . . . .

. . . . .

. . . . .

. . . . .

. . . . .

. . . . .

. . . . .

. . . . .

. . . . .

. . . . .

. . . . .

. . . . .

. . . . .

. . . . .

. . . . .

Algebra
and Functions Sets . . . . . . . . . . . . . . . . . Single Valued Functions . . . . . . . Inverses and Multi-Valued Functions . Transforming Equations . . . . . . . Exercises . . . . . . . . . . . . . . . Hints . . . . . . . . . . . . . . . . . Solutions . . . . . . . . . . . . . . . . . . . . . . . . . . . . . . . . . . . . . . . . . . . . . . . . . . . . . . . . . . . . . . . . . . . . . . . . . . . . . . . . . . . . . . . . . . . . . . . . . . . . . . . . . . . . . . . . . . . . . . . . . . . . . . . . . . . . . . . . . . . . . . . . . . . . . . . . . . . . . . . . . . . . . . . . . . . . . . . . . . . . . . . . . . . . . . . . . . . . . . . . . . . . . . . . . . . . . . . . . . . . . . . . . . . . . . . . . . . . . . . . . . . . . . . . . . . . . . . . . . . .

1
2 2 4 6 9 12 15 17

1 Sets 1.1 1.2 1.3 1.4 1.5 1.6 1.7

2 Vectors 2.1 Vectors . . . . . . . . . . . . . . . . . . 2.1.1 Scalars and Vectors . . . . . . . 2.1.2 The Kronecker Delta and Einstein 2.1.3 The Dot and Cross Product . . . 2.2 Sets of Vectors in n Dimensions . . . . . 2.3 Exercises . . . . . . . . . . . . . . . . . 2.4 Hints . . . . . . . . . . . . . . . . . . . 2.5 Solutions . . . . . . . . . . . . . . . . .

. . . . . . . . . . . . . . . . . . . . . . . . . . Summation Convention . . . . . . . . . . . . . . . . . . . . . . . . . . . . . . . . . . . . . . . . . . . . . . . . . . . . . . . . . . . . . . . . .

. . . . . . . .

. . . . . . . .

. . . . . . . .

. . . . . . . .

. . . . . . . .

. . . . . . . .

. . . . . . . .

. . . . . . . .

. . . . . . . .

. . . . . . . .

. . . . . . . .

. . . . . . . .

. . . . . . . .

. . . . . . . .

. . . . . . . .

. . . . . . . .

. . . . . . . .

. . . . . . . .

. . . . . . . .

. . . . . . . .

23 23 23 26 27 34 37 39 41

II

Calculus
. . . . . . . . . . . . . . . . . . . . . . . . . . . . . . . . . . . . . . . . . . Approximate . . . . . . . . . . . . . . . . . . . . . . . . . . . . . . . . . . . . . . . . . . . . . . . . . . . . . . . . . . . . . . . . . . . . . . . . . . . . . . . . . . . . . . . . . . . . . . . . . . . Functions. . . . . . . . . . . . . . . . . . . . . . . . . . . . . . . . . . . . . . . . . . . . . . . . . . . . . . . . . . . . . . . . . . . . . . . . . . . . . . . . . . . . . . . . . . . . . . . . . . . . . . . . . . . . . . . . . . . . . . . . . . . . . . . . . . . . . . . . . . . . . . . . . . . . . . . . . . . . . . . . . . . . . . . . . . . . . . . . . . . . . . . . . . . . . . . . . . . . . . . . . . . . . . . . . . . . . . . . . . . . . . . . . . . . . . . . . . . . . . . . . . . . . . . . . . . . . . . . . . . . . . . . . . . . . . . . . . . . . . . . . . . . . . . . . . . . . . . . . . . . . . . . . . . . . . . . . . . . . . . . . . . . . . .

48
49 49 54 57 62 64 67 69 74 76 82 82 82 83 85 85 85

3 Dierential Calculus 3.1 Limits of Functions . . . . . . . . . . . . . . . 3.2 Continuous Functions . . . . . . . . . . . . . 3.3 The Derivative . . . . . . . . . . . . . . . . . 3.4 Implicit Dierentiation . . . . . . . . . . . . . 3.5 Maxima and Minima . . . . . . . . . . . . . . 3.6 Mean Value Theorems . . . . . . . . . . . . . 3.6.1 Application: Using Taylors Theorem to 3.6.2 Application: Finite Dierence Schemes 3.7 LHospitals Rule . . . . . . . . . . . . . . . . 3.8 Exercises . . . . . . . . . . . . . . . . . . . . 3.8.1 Limits of Functions . . . . . . . . . . 3.8.2 Continuous Functions . . . . . . . . . 3.8.3 The Derivative . . . . . . . . . . . . . 3.8.4 Implicit Dierentiation . . . . . . . . . 3.8.5 Maxima and Minima . . . . . . . . . . 3.8.6 Mean Value Theorems . . . . . . . . .

ii

3.8.7 LHospitals Rule 3.9 Hints . . . . . . . . . . 3.10 Solutions . . . . . . . . 3.11 Quiz . . . . . . . . . . 3.12 Quiz Solutions . . . . .

. . . . .

. . . . .

. . . . .

. . . . .

. . . . .

. . . . .

. . . . .

. . . . .

. . . . .

. . . . .

. . . . .

. . . . .

. . . . .

. . . . .

. . . . .

. . . . .

. . . . .

. . . . .

. . . . .

. . . . .

. . . . .

. . . . .

. . . . .

. . . . .

. . . . .

. . . . .

. . . . .

. . . . .

. . . . .

. . . . .

. . . . .

. . . . .

. . . . .

. . . . .

. . . . .

. . . . .

. . . . .

. . . . .

. . . . .

. . . . .

. . . . .

. 86 . 88 . 93 . 113 . 114 116 116 122 122 123 125 127 127 130 134 134 134 136 136 137 138 141 150 151 154 154 155 163

4 Integral Calculus 4.1 The Indenite Integral . . . . . . . . . . . . . . 4.2 The Denite Integral . . . . . . . . . . . . . . . 4.2.1 Denition . . . . . . . . . . . . . . . . 4.2.2 Properties . . . . . . . . . . . . . . . . 4.3 The Fundamental Theorem of Integral Calculus . 4.4 Techniques of Integration . . . . . . . . . . . . 4.4.1 Partial Fractions . . . . . . . . . . . . . 4.5 Improper Integrals . . . . . . . . . . . . . . . . 4.6 Exercises . . . . . . . . . . . . . . . . . . . . . 4.6.1 The Indenite Integral . . . . . . . . . . 4.6.2 The Denite Integral . . . . . . . . . . . 4.6.3 The Fundamental Theorem of Integration 4.6.4 Techniques of Integration . . . . . . . . 4.6.5 Improper Integrals . . . . . . . . . . . . 4.7 Hints . . . . . . . . . . . . . . . . . . . . . . . 4.8 Solutions . . . . . . . . . . . . . . . . . . . . . 4.9 Quiz . . . . . . . . . . . . . . . . . . . . . . . 4.10 Quiz Solutions . . . . . . . . . . . . . . . . . .

. . . . . . . . . . . . . . . . . .

. . . . . . . . . . . . . . . . . .

. . . . . . . . . . . . . . . . . .

. . . . . . . . . . . . . . . . . .

. . . . . . . . . . . . . . . . . .

. . . . . . . . . . . . . . . . . .

. . . . . . . . . . . . . . . . . .

. . . . . . . . . . . . . . . . . .

. . . . . . . . . . . . . . . . . .

. . . . . . . . . . . . . . . . . .

. . . . . . . . . . . . . . . . . .

. . . . . . . . . . . . . . . . . .

. . . . . . . . . . . . . . . . . .

. . . . . . . . . . . . . . . . . .

. . . . . . . . . . . . . . . . . .

. . . . . . . . . . . . . . . . . .

. . . . . . . . . . . . . . . . . .

. . . . . . . . . . . . . . . . . .

. . . . . . . . . . . . . . . . . .

. . . . . . . . . . . . . . . . . .

. . . . . . . . . . . . . . . . . .

. . . . . . . . . . . . . . . . . .

. . . . . . . . . . . . . . . . . .

. . . . . . . . . . . . . . . . . .

. . . . . . . . . . . . . . . . . .

. . . . . . . . . . . . . . . . . .

. . . . . . . . . . . . . . . . . .

. . . . . . . . . . . . . . . . . .

. . . . . . . . . . . . . . . . . .

5 Vector Calculus 5.1 Vector Functions . . . . . . . . . . . . . . . . . . . . . . . . . . . . . . . . . . . . . . . . . . . . . . 5.2 Gradient, Divergence and Curl . . . . . . . . . . . . . . . . . . . . . . . . . . . . . . . . . . . . . . . 5.3 Exercises . . . . . . . . . . . . . . . . . . . . . . . . . . . . . . . . . . . . . . . . . . . . . . . . . .

iii

5.4 Hints . . . . . 5.5 Solutions . . . 5.6 Quiz . . . . . 5.7 Quiz Solutions

. . . .

. . . .

. . . .

. . . .

. . . .

. . . .

. . . .

. . . .

. . . .

. . . .

. . . .

. . . .

. . . .

. . . .

. . . .

. . . .

. . . .

. . . .

. . . .

. . . .

. . . .

. . . .

. . . .

. . . .

. . . .

. . . .

. . . .

. . . .

. . . .

. . . .

. . . .

. . . .

. . . .

. . . .

. . . .

. . . .

. . . .

. . . .

. . . .

. . . .

. . . .

. . . .

. . . .

. . . .

. . . .

. . . .

. . . .

166 168 177 178

III

Functions of a Complex Variable


. . . . . . . . . . . . . . . . . . . . . . . . . . . . . . . . . . . . . . . . . . . . . . . . . . . . . . . . . . . . . . . . . . . . . . . . . . . . . . . . . . . . . . . . . . . . . . . . . . . . . . . . . . . . . . . . . . . . . . . . . . . . . . . . . . . . . . . . . . . . . . . . . . . . . . . . . . . . . . . . . . . . . . . . . . . . . . . . . . . . . . . . . . . . . . . . . . . . . . . . . . . . . . . . . . . . . . . . . . . . . . . . . . . . . . . . . . . . . . . . . . . . . . . . . . . . . . . . . . . . . . . . . . . . . . . . . . . . . . . . . . . . . . . . . . . . . . . . . . . . . . . . . . . . . . . . . . . . . . . . . . . . . . . . . . . . . . . . . . . . . . . . . . . . . . . . . . . . . . . . . . . . . . . . . . . . . . . . . .

179
180 180 184 188 193 195 197 201 208 211 239 239 242 246 248 251 257 267 269 285 296

6 Complex Numbers 6.1 Complex Numbers . . . 6.2 The Complex Plane . . 6.3 Polar Form . . . . . . . 6.4 Arithmetic and Vectors 6.5 Integer Exponents . . . 6.6 Rational Exponents . . 6.7 Exercises . . . . . . . . 6.8 Hints . . . . . . . . . . 6.9 Solutions . . . . . . . .

7 Functions of a Complex Variable 7.1 Curves and Regions . . . . . . . . . . . . 7.2 The Point at Innity and the Stereographic 7.3 Cartesian and Modulus-Argument Form . . 7.4 Graphing Functions of a Complex Variable 7.5 Trigonometric Functions . . . . . . . . . . 7.6 Inverse Trigonometric Functions . . . . . . 7.7 Riemann Surfaces . . . . . . . . . . . . . 7.8 Branch Points . . . . . . . . . . . . . . . 7.9 Exercises . . . . . . . . . . . . . . . . . . 7.10 Hints . . . . . . . . . . . . . . . . . . . .

. . . . . . Projection . . . . . . . . . . . . . . . . . . . . . . . . . . . . . . . . . . . . . . . . . . . . . . . .

. . . . . . . . . .

. . . . . . . . . .

. . . . . . . . . .

. . . . . . . . . .

. . . . . . . . . .

. . . . . . . . . .

. . . . . . . . . .

. . . . . . . . . .

. . . . . . . . . .

. . . . . . . . . .

. . . . . . . . . .

. . . . . . . . . .

. . . . . . . . . .

. . . . . . . . . .

. . . . . . . . . .

. . . . . . . . . .

. . . . . . . . . .

. . . . . . . . . .

. . . . . . . . . .

. . . . . . . . . .

. . . . . . . . . .

. . . . . . . . . .

. . . . . . . . . .

. . . . . . . . . .

. . . . . . . . . .

. . . . . . . . . .

iv

7.11 Solutions . . . . . . . . . . . . . . . . . . . . . . . . . . . . . . . . . . . . . . . . . . . . . . . . . . 301 8 Analytic Functions 8.1 Complex Derivatives . . . . . . . . . . . . . . 8.2 Cauchy-Riemann Equations . . . . . . . . . . 8.3 Harmonic Functions . . . . . . . . . . . . . . 8.4 Singularities . . . . . . . . . . . . . . . . . . 8.4.1 Categorization of Singularities . . . . . 8.4.2 Isolated and Non-Isolated Singularities 8.5 Application: Potential Flow . . . . . . . . . . 8.6 Exercises . . . . . . . . . . . . . . . . . . . . 8.7 Hints . . . . . . . . . . . . . . . . . . . . . . 8.8 Solutions . . . . . . . . . . . . . . . . . . . . 359 359 366 371 376 376 380 382 387 395 398 436 436 439 441 446 449 453 455 456 461 461 463 466 467 469

. . . . . . . . . .

. . . . . . . . . .

. . . . . . . . . .

. . . . . . . . . .

. . . . . . . . . .

. . . . . . . . . .

. . . . . . . . . .

. . . . . . . . . .

. . . . . . . . . .

. . . . . . . . . .

. . . . . . . . . .

. . . . . . . . . .

. . . . . . . . . .

. . . . . . . . . .

. . . . . . . . . .

. . . . . . . . . .

. . . . . . . . . .

. . . . . . . . . .

. . . . . . . . . .

. . . . . . . . . .

. . . . . . . . . .

. . . . . . . . . .

. . . . . . . . . .

. . . . . . . . . .

. . . . . . . . . .

. . . . . . . . . .

. . . . . . . . . .

. . . . . . . . . .

. . . . . . . . . .

. . . . . . . . . .

9 Analytic Continuation 9.1 Analytic Continuation . . . . . . . . . . . . . . . . . . . . . . . . . . . . . 9.2 Analytic Continuation of Sums . . . . . . . . . . . . . . . . . . . . . . . . 9.3 Analytic Functions Dened in Terms of Real Variables . . . . . . . . . . . . 9.3.1 Polar Coordinates . . . . . . . . . . . . . . . . . . . . . . . . . . . 9.3.2 Analytic Functions Dened in Terms of Their Real or Imaginary Parts 9.4 Exercises . . . . . . . . . . . . . . . . . . . . . . . . . . . . . . . . . . . . 9.5 Hints . . . . . . . . . . . . . . . . . . . . . . . . . . . . . . . . . . . . . . 9.6 Solutions . . . . . . . . . . . . . . . . . . . . . . . . . . . . . . . . . . . . 10 Contour Integration and the Cauchy-Goursat Theorem 10.1 Line Integrals . . . . . . . . . . . . . . . . . . . . . . . 10.2 Contour Integrals . . . . . . . . . . . . . . . . . . . . . 10.2.1 Maximum Modulus Integral Bound . . . . . . . 10.3 The Cauchy-Goursat Theorem . . . . . . . . . . . . . . 10.4 Contour Deformation . . . . . . . . . . . . . . . . . .

. . . . . . . .

. . . . . . . .

. . . . . . . .

. . . . . . . .

. . . . . . . .

. . . . . . . .

. . . . . . . .

. . . . . . . .

. . . . . . . .

. . . . . . . .

. . . . . . . .

. . . . . . . .

. . . . . . . .

. . . . . . . .

. . . . .

. . . . .

. . . . .

. . . . .

. . . . .

. . . . .

. . . . .

. . . . .

. . . . .

. . . . .

. . . . .

. . . . .

. . . . .

. . . . .

. . . . .

. . . . .

. . . . .

. . . . .

. . . . .

. . . . .

. . . . .

. . . . .

. . . . .

. . . . .

. . . . .

10.5 Moreras Theorem. . . . . . . . . . . 10.6 Indenite Integrals . . . . . . . . . . 10.7 Fundamental Theorem of Calculus via 10.7.1 Line Integrals and Primitives . 10.7.2 Contour Integrals . . . . . . 10.8 Fundamental Theorem of Calculus via 10.9 Exercises . . . . . . . . . . . . . . . 10.10Hints . . . . . . . . . . . . . . . . . 10.11Solutions . . . . . . . . . . . . . . . 11 Cauchys Integral Formula 11.1 Cauchys Integral Formula 11.2 The Argument Theorem . 11.3 Rouches Theorem . . . . 11.4 Exercises . . . . . . . . . 11.5 Hints . . . . . . . . . . . 11.6 Solutions . . . . . . . . .

. . . . . . . . . . . . . . . . . . . . Primitives . . . . . . . . . . . . . . . . . . . . . . . . Complex Calculus . . . . . . . . . . . . . . . . . . . . . . . . . . . . . .

. . . . . . . . .

. . . . . . . . .

. . . . . . . . .

. . . . . . . . .

. . . . . . . . .

. . . . . . . . .

. . . . . . . . .

. . . . . . . . .

. . . . . . . . .

. . . . . . . . .

. . . . . . . . .

. . . . . . . . .

. . . . . . . . .

. . . . . . . . .

. . . . . . . . .

. . . . . . . . .

. . . . . . . . .

. . . . . . . . .

. . . . . . . . .

. . . . . . . . .

. . . . . . . . .

. . . . . . . . .

. . . . . . . . .

. . . . . . . . .

. . . . . . . . .

470 472 473 473 473 474 477 481 482 492 493 500 501 504 508 510 524 524 524 526 528 535 536 538 539 546 549 552

. . . . . .

. . . . . .

. . . . . .

. . . . . .

. . . . . .

. . . . . .

. . . . . .

. . . . . .

. . . . . .

. . . . . .

. . . . . .

. . . . . .

. . . . . .

. . . . . .

. . . . . .

. . . . . .

. . . . . .

. . . . . .

. . . . . .

. . . . . .

. . . . . .

. . . . . .

. . . . . .

. . . . . .

. . . . . .

. . . . . .

. . . . . .

. . . . . .

. . . . . .

. . . . . .

. . . . . .

. . . . . .

. . . . . .

. . . . . .

. . . . . .

. . . . . .

. . . . . .

. . . . . .

. . . . . .

. . . . . .

. . . . . .

12 Series and Convergence 12.1 Series of Constants . . . . . . . . . . . . . . . . . . . . 12.1.1 Denitions . . . . . . . . . . . . . . . . . . . . 12.1.2 Special Series . . . . . . . . . . . . . . . . . . 12.1.3 Convergence Tests . . . . . . . . . . . . . . . . 12.2 Uniform Convergence . . . . . . . . . . . . . . . . . . 12.2.1 Tests for Uniform Convergence . . . . . . . . . 12.2.2 Uniform Convergence and Continuous Functions. 12.3 Uniformly Convergent Power Series . . . . . . . . . . . 12.4 Integration and Dierentiation of Power Series . . . . . 12.5 Taylor Series . . . . . . . . . . . . . . . . . . . . . . . 12.5.1 Newtons Binomial Formula. . . . . . . . . . . .

. . . . . . . . . . .

. . . . . . . . . . .

. . . . . . . . . . .

. . . . . . . . . . .

. . . . . . . . . . .

. . . . . . . . . . .

. . . . . . . . . . .

. . . . . . . . . . .

. . . . . . . . . . .

. . . . . . . . . . .

. . . . . . . . . . .

. . . . . . . . . . .

. . . . . . . . . . .

. . . . . . . . . . .

. . . . . . . . . . .

. . . . . . . . . . .

. . . . . . . . . . .

. . . . . . . . . . .

. . . . . . . . . . .

. . . . . . . . . . .

. . . . . . . . . . .

. . . . . . . . . . .

. . . . . . . . . . .

. . . . . . . . . . .

. . . . . . . . . . .

vi

12.6 Laurent Series . . . . . . . . . . . . . . . . . . . . . 12.7 Exercises . . . . . . . . . . . . . . . . . . . . . . . . 12.7.1 Series of Constants . . . . . . . . . . . . . . 12.7.2 Uniform Convergence . . . . . . . . . . . . . 12.7.3 Uniformly Convergent Power Series . . . . . . 12.7.4 Integration and Dierentiation of Power Series 12.7.5 Taylor Series . . . . . . . . . . . . . . . . . . 12.7.6 Laurent Series . . . . . . . . . . . . . . . . . 12.8 Hints . . . . . . . . . . . . . . . . . . . . . . . . . . 12.9 Solutions . . . . . . . . . . . . . . . . . . . . . . . . 13 The Residue Theorem 13.1 The Residue Theorem . . . . . . . . . . . . 13.2 Cauchy Principal Value for Real Integrals . . 13.2.1 The Cauchy Principal Value . . . . . 13.3 Cauchy Principal Value for Contour Integrals 13.4 Integrals on the Real Axis . . . . . . . . . . 13.5 Fourier Integrals . . . . . . . . . . . . . . . 13.6 Fourier Cosine and Sine Integrals . . . . . . 13.7 Contour Integration and Branch Cuts . . . . 13.8 Exploiting Symmetry . . . . . . . . . . . . . 13.8.1 Wedge Contours . . . . . . . . . . . 13.8.2 Box Contours . . . . . . . . . . . . 13.9 Denite Integrals Involving Sine and Cosine . 13.10Innite Sums . . . . . . . . . . . . . . . . . 13.11Exercises . . . . . . . . . . . . . . . . . . . 13.12Hints . . . . . . . . . . . . . . . . . . . . . 13.13Solutions . . . . . . . . . . . . . . . . . . .

. . . . . . . . . .

. . . . . . . . . .

. . . . . . . . . .

. . . . . . . . . .

. . . . . . . . . .

. . . . . . . . . .

. . . . . . . . . .

. . . . . . . . . .

. . . . . . . . . .

. . . . . . . . . .

. . . . . . . . . .

. . . . . . . . . .

. . . . . . . . . .

. . . . . . . . . .

. . . . . . . . . .

. . . . . . . . . .

. . . . . . . . . .

. . . . . . . . . .

. . . . . . . . . .

. . . . . . . . . .

. . . . . . . . . .

. . . . . . . . . .

. . . . . . . . . .

. . . . . . . . . .

. . . . . . . . . .

. . . . . . . . . .

554 559 559 565 565 567 568 570 573 581 625 625 633 633 638 642 646 648 651 654 654 657 658 661 666 680 686

. . . . . . . . . . . . . . . .

. . . . . . . . . . . . . . . .

. . . . . . . . . . . . . . . .

. . . . . . . . . . . . . . . .

. . . . . . . . . . . . . . . .

. . . . . . . . . . . . . . . .

. . . . . . . . . . . . . . . .

. . . . . . . . . . . . . . . .

. . . . . . . . . . . . . . . .

. . . . . . . . . . . . . . . .

. . . . . . . . . . . . . . . .

. . . . . . . . . . . . . . . .

. . . . . . . . . . . . . . . .

. . . . . . . . . . . . . . . .

. . . . . . . . . . . . . . . .

. . . . . . . . . . . . . . . .

. . . . . . . . . . . . . . . .

. . . . . . . . . . . . . . . .

. . . . . . . . . . . . . . . .

. . . . . . . . . . . . . . . .

. . . . . . . . . . . . . . . .

. . . . . . . . . . . . . . . .

. . . . . . . . . . . . . . . .

. . . . . . . . . . . . . . . .

. . . . . . . . . . . . . . . .

. . . . . . . . . . . . . . . .

. . . . . . . . . . . . . . . .

. . . . . . . . . . . . . . . .

. . . . . . . . . . . . . . . .

. . . . . . . . . . . . . . . .

. . . . . . . . . . . . . . . .

vii

IV

Ordinary Dierential Equations


. . . . . . . . . . . . . . . . . . . . . . . . . . . . . . . . . . . . . . . . . . . . . . . . . . . . . . . . . . . . . . . . . . . . . . . . . . . . . . . . . . . . . . . . . . . . . . . . . . . . . . . . . . . . . . . . . . . . . . . . . . . . . . . . . . . . . . . . . . . . . . . . . . . . . . . . . . . . . . . . . . . . . . . . . . . . . . . . . . . . . . . . . . . . . . . . . . . . . . . . . . . . . . . . . . . . . . . . . . . . . . . . . . . . . . . . . . . . . . . . . . . . . . . . . . . . . . . . . . . . . . . . . . . . . . . . . . . . . . . . . . . . . . . . . . . . . . . . . . . . . . . . . . . . . . . . . . . . . . . . . . . . . . . . . . . . . . . . . . . . . . . . . . . . . . . . . . . . . . . . . . . . . . . . . . . . . . . . . . . . . . . . . . . . . . . . . . . . . . . . . . . . . . . . . . . . . . . . . . . . . . . . . . . . . . . . . . . . . . . . . . . . . . . . . . . . . . . . . . . . . . . . . . . . . . . . . . . . . . . . . . . . . . . . . . . . . . . . . . . . . . . . . . . . . . . . . . . . . . . . . . . . .

772
773 773 775 775 777 780 780 782 786 791 791 792 795 796 797 801 803 803 807 812 814 817 820 823 844 845

14 First Order Dierential Equations 14.1 Notation . . . . . . . . . . . . . . . . . . . . . . . . . . . . . 14.2 Example Problems . . . . . . . . . . . . . . . . . . . . . . . . 14.2.1 Growth and Decay . . . . . . . . . . . . . . . . . . . . 14.3 One Parameter Families of Functions . . . . . . . . . . . . . . 14.4 Integrable Forms . . . . . . . . . . . . . . . . . . . . . . . . . 14.4.1 Separable Equations . . . . . . . . . . . . . . . . . . . 14.4.2 Exact Equations . . . . . . . . . . . . . . . . . . . . . 14.4.3 Homogeneous Coecient Equations . . . . . . . . . . . 14.5 The First Order, Linear Dierential Equation . . . . . . . . . . 14.5.1 Homogeneous Equations . . . . . . . . . . . . . . . . . 14.5.2 Inhomogeneous Equations . . . . . . . . . . . . . . . . 14.5.3 Variation of Parameters. . . . . . . . . . . . . . . . . . 14.6 Initial Conditions . . . . . . . . . . . . . . . . . . . . . . . . . 14.6.1 Piecewise Continuous Coecients and Inhomogeneities . 14.7 Well-Posed Problems . . . . . . . . . . . . . . . . . . . . . . . 14.8 Equations in the Complex Plane . . . . . . . . . . . . . . . . . 14.8.1 Ordinary Points . . . . . . . . . . . . . . . . . . . . . 14.8.2 Regular Singular Points . . . . . . . . . . . . . . . . . 14.8.3 Irregular Singular Points . . . . . . . . . . . . . . . . . 14.8.4 The Point at Innity . . . . . . . . . . . . . . . . . . . 14.9 Additional Exercises . . . . . . . . . . . . . . . . . . . . . . . 14.10Hints . . . . . . . . . . . . . . . . . . . . . . . . . . . . . . . 14.11Solutions . . . . . . . . . . . . . . . . . . . . . . . . . . . . . 14.12Quiz . . . . . . . . . . . . . . . . . . . . . . . . . . . . . . . 14.13Quiz Solutions . . . . . . . . . . . . . . . . . . . . . . . . . .

viii

15 First Order Linear Systems of Dierential Equations 15.1 Introduction . . . . . . . . . . . . . . . . . . . . . . . . . . . . . 15.2 Using Eigenvalues and Eigenvectors to nd Homogeneous Solutions 15.3 Matrices and Jordan Canonical Form . . . . . . . . . . . . . . . . 15.4 Using the Matrix Exponential . . . . . . . . . . . . . . . . . . . . 15.5 Exercises . . . . . . . . . . . . . . . . . . . . . . . . . . . . . . . 15.6 Hints . . . . . . . . . . . . . . . . . . . . . . . . . . . . . . . . . 15.7 Solutions . . . . . . . . . . . . . . . . . . . . . . . . . . . . . . . 16 Theory of Linear Ordinary Dierential Equations 16.1 Exact Equations . . . . . . . . . . . . . . . . . . . . . . . . . . 16.2 Nature of Solutions . . . . . . . . . . . . . . . . . . . . . . . . 16.3 Transformation to a First Order System . . . . . . . . . . . . . . 16.4 The Wronskian . . . . . . . . . . . . . . . . . . . . . . . . . . . 16.4.1 Derivative of a Determinant. . . . . . . . . . . . . . . . 16.4.2 The Wronskian of a Set of Functions. . . . . . . . . . . . 16.4.3 The Wronskian of the Solutions to a Dierential Equation 16.5 Well-Posed Problems . . . . . . . . . . . . . . . . . . . . . . . . 16.6 The Fundamental Set of Solutions . . . . . . . . . . . . . . . . . 16.7 Adjoint Equations . . . . . . . . . . . . . . . . . . . . . . . . . 16.8 Additional Exercises . . . . . . . . . . . . . . . . . . . . . . . . 16.9 Hints . . . . . . . . . . . . . . . . . . . . . . . . . . . . . . . . 16.10Solutions . . . . . . . . . . . . . . . . . . . . . . . . . . . . . . 16.11Quiz . . . . . . . . . . . . . . . . . . . . . . . . . . . . . . . . 16.12Quiz Solutions . . . . . . . . . . . . . . . . . . . . . . . . . . . 17 Techniques for Linear Dierential 17.1 Constant Coecient Equations 17.1.1 Second Order Equations 17.1.2 Real-Valued Solutions .

. . . . . . .

. . . . . . .

. . . . . . .

. . . . . . .

. . . . . . .

. . . . . . .

. . . . . . .

. . . . . . .

. . . . . . .

. . . . . . .

. . . . . . .

. . . . . . .

. . . . . . .

. . . . . . .

. . . . . . .

. . . . . . .

. . . . . . .

. . . . . . .

. . . . . . .

847 847 848 853 861 866 871 873 901 901 902 906 907 907 908 910 912 914 917 920 921 923 929 930

. . . . . . . . . . . . . . .

. . . . . . . . . . . . . . .

. . . . . . . . . . . . . . .

. . . . . . . . . . . . . . .

. . . . . . . . . . . . . . .

. . . . . . . . . . . . . . .

. . . . . . . . . . . . . . .

. . . . . . . . . . . . . . .

. . . . . . . . . . . . . . .

. . . . . . . . . . . . . . .

. . . . . . . . . . . . . . .

. . . . . . . . . . . . . . .

. . . . . . . . . . . . . . .

. . . . . . . . . . . . . . .

. . . . . . . . . . . . . . .

. . . . . . . . . . . . . . .

. . . . . . . . . . . . . . .

. . . . . . . . . . . . . . .

. . . . . . . . . . . . . . .

. . . . . . . . . . . . . . .

Equations 931 . . . . . . . . . . . . . . . . . . . . . . . . . . . . . . . . . . . . . . 931 . . . . . . . . . . . . . . . . . . . . . . . . . . . . . . . . . . . . . . 932 . . . . . . . . . . . . . . . . . . . . . . . . . . . . . . . . . . . . . . 936

ix

17.2 17.3 17.4 17.5 17.6 17.7 17.8 17.9

17.1.3 Higher Order Equations . . . . . . . . Euler Equations . . . . . . . . . . . . . . . . 17.2.1 Real-Valued Solutions . . . . . . . . . Exact Equations . . . . . . . . . . . . . . . . Equations Without Explicit Dependence on y . Reduction of Order . . . . . . . . . . . . . . . *Reduction of Order and the Adjoint Equation Additional Exercises . . . . . . . . . . . . . . Hints . . . . . . . . . . . . . . . . . . . . . . Solutions . . . . . . . . . . . . . . . . . . . .

. . . . . . . . . .

. . . . . . . . . .

. . . . . . . . . .

. . . . . . . . . .

. . . . . . . . . .

. . . . . . . . . .

. . . . . . . . . .

. . . . . . . . . .

. . . . . . . . . .

. . . . . . . . . .

. . . . . . . . . .

. . . . . . . . . .

. . . . . . . . . .

. . . . . . . . . .

. . . . . . . . . .

. . . . . . . . . .

. . . . . . . . . .

. . . . . . . . . .

. . . . . . . . . .

. . . . . . . . . .

. . . . . . . . . .

. . . . . . . . . .

. . . . . . . . . .

. . . . . . . . . .

. . . . . . . . . .

. . . . . . . . . .

. . . . . . . . . .

. . . . . . . . . .

. . . . . . . . . .

. . . . . . . . . .

938 941 943 946 947 948 949 952 958 961 985 985 987 991 993 996 998 1001 1002 1005 1007

18 Techniques for Nonlinear Dierential Equations 18.1 Bernoulli Equations . . . . . . . . . . . . . . . . . . 18.2 Riccati Equations . . . . . . . . . . . . . . . . . . . 18.3 Exchanging the Dependent and Independent Variables 18.4 Autonomous Equations . . . . . . . . . . . . . . . . 18.5 *Equidimensional-in-x Equations . . . . . . . . . . . . 18.6 *Equidimensional-in-y Equations . . . . . . . . . . . . 18.7 *Scale-Invariant Equations . . . . . . . . . . . . . . . 18.8 Exercises . . . . . . . . . . . . . . . . . . . . . . . . 18.9 Hints . . . . . . . . . . . . . . . . . . . . . . . . . . 18.10Solutions . . . . . . . . . . . . . . . . . . . . . . . . 19 Transformations and Canonical Forms 19.1 The Constant Coecient Equation . . . . . . . . . 19.2 Normal Form . . . . . . . . . . . . . . . . . . . . . 19.2.1 Second Order Equations . . . . . . . . . . . 19.2.2 Higher Order Dierential Equations . . . . . 19.3 Transformations of the Independent Variable . . . . 19.3.1 Transformation to the form u + a(x) u = 0

. . . . . . . . . .

. . . . . . . . . .

. . . . . . . . . .

. . . . . . . . . .

. . . . . . . . . .

. . . . . . . . . .

. . . . . . . . . .

. . . . . . . . . .

. . . . . . . . . .

. . . . . . . . . .

. . . . . . . . . .

. . . . . . . . . .

. . . . . . . . . .

. . . . . . . . . .

. . . . . . . . . .

. . . . . . . . . .

. . . . . . . . . .

. . . . . . . . . .

. . . . . . . . . .

. . . . . . . . . .

. . . . . . . . . .

. . . . . . . . . .

. . . . . . . . . .

. . . . . . . . . .

. . . . . . . . . .

. . . . . . . . . .

. . . . . .

. . . . . .

. . . . . .

. . . . . .

. . . . . .

. . . . . .

. . . . . .

. . . . . .

. . . . . .

. . . . . .

. . . . . .

. . . . . .

. . . . . .

. . . . . .

. . . . . .

. . . . . .

. . . . . .

. . . . . .

. . . . . .

. . . . . .

. . . . . .

. . . . . .

. . . . . .

. . . . . .

. . . . . .

. . . . . .

1019 . 1019 . 1022 . 1022 . 1023 . 1025 . 1025

19.4

19.5 19.6 19.7 20 The 20.1 20.2 20.3 20.4 20.5 20.6 20.7

19.3.2 Transformation to a Constant Coecient Equation Integral Equations . . . . . . . . . . . . . . . . . . . . . 19.4.1 Initial Value Problems . . . . . . . . . . . . . . . 19.4.2 Boundary Value Problems . . . . . . . . . . . . . Exercises . . . . . . . . . . . . . . . . . . . . . . . . . . Hints . . . . . . . . . . . . . . . . . . . . . . . . . . . . Solutions . . . . . . . . . . . . . . . . . . . . . . . . . . Dirac Delta Function Derivative of the Heaviside Function The Delta Function as a Limit . . . . Higher Dimensions . . . . . . . . . . Non-Rectangular Coordinate Systems Exercises . . . . . . . . . . . . . . . Hints . . . . . . . . . . . . . . . . . Solutions . . . . . . . . . . . . . . .

. . . . . . .

. . . . . . .

. . . . . . .

. . . . . . .

. . . . . . .

. . . . . . .

. . . . . . .

. . . . . . .

. . . . . . .

. . . . . . .

. . . . . . .

. . . . . . .

. . . . . . .

. . . . . . .

. . . . . . .

. . . . . . .

. . . . . . .

. . . . . . .

. . . . . . .

. . . . . . .

. . . . . . .

. . . . . . .

. . . . . . .

. . . . . . .

1026 1028 1028 1030 1033 1035 1036

. . . . . . .

. . . . . . .

. . . . . . .

. . . . . . .

. . . . . . .

. . . . . . .

. . . . . . .

. . . . . . .

. . . . . . .

. . . . . . .

. . . . . . .

. . . . . . .

. . . . . . .

. . . . . . .

. . . . . . .

. . . . . . .

. . . . . . .

. . . . . . .

. . . . . . .

. . . . . . .

. . . . . . .

. . . . . . .

. . . . . . .

. . . . . . .

. . . . . . .

. . . . . . .

. . . . . . .

. . . . . . .

. . . . . . .

. . . . . . .

. . . . . . .

. . . . . . .

. . . . . . .

. . . . . . .

1042 . 1042 . 1044 . 1046 . 1047 . 1049 . 1051 . 1053 1060 . 1060 . 1062 . 1066 . 1066 . 1069 . 1072 . 1075 . 1075 . 1077 . 1078 . 1080 . 1083

21 Inhomogeneous Dierential Equations 21.1 Particular Solutions . . . . . . . . . . . . . . . . . . . . . . . . . . . . . . . . . . . 21.2 Method of Undetermined Coecients . . . . . . . . . . . . . . . . . . . . . . . . . . 21.3 Variation of Parameters . . . . . . . . . . . . . . . . . . . . . . . . . . . . . . . . . 21.3.1 Second Order Dierential Equations . . . . . . . . . . . . . . . . . . . . . . . 21.3.2 Higher Order Dierential Equations . . . . . . . . . . . . . . . . . . . . . . . 21.4 Piecewise Continuous Coecients and Inhomogeneities . . . . . . . . . . . . . . . . . 21.5 Inhomogeneous Boundary Conditions . . . . . . . . . . . . . . . . . . . . . . . . . . 21.5.1 Eliminating Inhomogeneous Boundary Conditions . . . . . . . . . . . . . . . . 21.5.2 Separating Inhomogeneous Equations and Inhomogeneous Boundary Conditions 21.5.3 Existence of Solutions of Problems with Inhomogeneous Boundary Conditions . 21.6 Green Functions for First Order Equations . . . . . . . . . . . . . . . . . . . . . . . 21.7 Green Functions for Second Order Equations . . . . . . . . . . . . . . . . . . . . . .

. . . . . . . . . . . .

. . . . . . . . . . . .

. . . . . . . . . . . .

. . . . . . . . . . . .

. . . . . . . . . . . .

. . . . . . . . . . . .

. . . . . . . . . . . .

. . . . . . . . . . . .

xi

21.7.1 Green Functions for Sturm-Liouville Problems . 21.7.2 Initial Value Problems . . . . . . . . . . . . . 21.7.3 Problems with Unmixed Boundary Conditions . 21.7.4 Problems with Mixed Boundary Conditions . . 21.8 Green Functions for Higher Order Problems . . . . . . 21.9 Fredholm Alternative Theorem . . . . . . . . . . . . 21.10Exercises . . . . . . . . . . . . . . . . . . . . . . . . 21.11Hints . . . . . . . . . . . . . . . . . . . . . . . . . . 21.12Solutions . . . . . . . . . . . . . . . . . . . . . . . . 21.13Quiz . . . . . . . . . . . . . . . . . . . . . . . . . . 21.14Quiz Solutions . . . . . . . . . . . . . . . . . . . . . 22 Dierence Equations 22.1 Introduction . . . . . . . . . . . . . . . . . . 22.2 Exact Equations . . . . . . . . . . . . . . . . 22.3 Homogeneous First Order . . . . . . . . . . . 22.4 Inhomogeneous First Order . . . . . . . . . . 22.5 Homogeneous Constant Coecient Equations . 22.6 Reduction of Order . . . . . . . . . . . . . . . 22.7 Exercises . . . . . . . . . . . . . . . . . . . . 22.8 Hints . . . . . . . . . . . . . . . . . . . . . . 22.9 Solutions . . . . . . . . . . . . . . . . . . . .

. . . . . . . . . . .

. . . . . . . . . . .

. . . . . . . . . . .

. . . . . . . . . . .

. . . . . . . . . . .

. . . . . . . . . . .

. . . . . . . . . . .

. . . . . . . . . . .

. . . . . . . . . . .

. . . . . . . . . . .

. . . . . . . . . . .

. . . . . . . . . . .

. . . . . . . . . . .

. . . . . . . . . . .

. . . . . . . . . . .

. . . . . . . . . . .

. . . . . . . . . . .

. . . . . . . . . . .

. . . . . . . . . . .

. . . . . . . . . . .

. . . . . . . . . . .

. . . . . . . . . . .

. . . . . . . . . . .

. . . . . . . . . . .

. . . . . . . . . . .

. . . . . . . . . . .

1093 1096 1099 1101 1105 1110 1118 1124 1127 1165 1166

. . . . . . . . .

. . . . . . . . .

. . . . . . . . .

. . . . . . . . .

. . . . . . . . .

. . . . . . . . .

. . . . . . . . .

. . . . . . . . .

. . . . . . . . .

. . . . . . . . .

. . . . . . . . .

. . . . . . . . .

. . . . . . . . .

. . . . . . . . .

. . . . . . . . .

. . . . . . . . .

. . . . . . . . .

. . . . . . . . .

. . . . . . . . .

. . . . . . . . .

. . . . . . . . .

. . . . . . . . .

. . . . . . . . .

. . . . . . . . .

. . . . . . . . .

. . . . . . . . .

. . . . . . . . .

. . . . . . . . .

. . . . . . . . .

1167 . 1167 . 1169 . 1170 . 1172 . 1175 . 1178 . 1180 . 1181 . 1182 1185 . 1185 . 1189 . 1199 . 1202 . 1204 . 1208

23 Series Solutions of Dierential Equations 23.1 Ordinary Points . . . . . . . . . . . . . . . . . . 23.1.1 Taylor Series Expansion for a Second Order 23.2 Regular Singular Points of Second Order Equations 23.2.1 Indicial Equation . . . . . . . . . . . . . . 23.2.2 The Case: Double Root . . . . . . . . . . 23.2.3 The Case: Roots Dier by an Integer . . .

. . . . . . . . . . . . Dierential Equation . . . . . . . . . . . . . . . . . . . . . . . . . . . . . . . . . . . . . . . . . . . . . . . .

. . . . . .

. . . . . .

. . . . . .

. . . . . .

. . . . . .

. . . . . .

. . . . . .

. . . . . .

. . . . . .

. . . . . .

. . . . . .

. . . . . .

. . . . . .

. . . . . .

. . . . . .

xii

23.3 23.4 23.5 23.6 23.7 23.8 23.9

Irregular Singular Points The Point at Innity . . Exercises . . . . . . . . Hints . . . . . . . . . . Solutions . . . . . . . . Quiz . . . . . . . . . . Quiz Solutions . . . . .

. . . . . . .

. . . . . . .

. . . . . . .

. . . . . . .

. . . . . . .

. . . . . . .

. . . . . . .

. . . . . . .

. . . . . . .

. . . . . . .

. . . . . . .

. . . . . . .

. . . . . . .

. . . . . . .

. . . . . . .

. . . . . . .

. . . . . . .

. . . . . . .

. . . . . . .

. . . . . . .

. . . . . . .

. . . . . . .

. . . . . . .

. . . . . . .

. . . . . . .

. . . . . . .

. . . . . . .

. . . . . . .

. . . . . . .

. . . . . . .

. . . . . . .

. . . . . . .

. . . . . . .

. . . . . . .

. . . . . . .

. . . . . . .

. . . . . . .

. . . . . . .

. . . . . . .

. . . . . . .

. . . . . . .

. . . . . . .

1218 1218 1221 1226 1227 1250 1251

24 Asymptotic Expansions 24.1 Asymptotic Relations . . . . . . . . . . . . . . 24.2 Leading Order Behavior of Dierential Equations 24.3 Integration by Parts . . . . . . . . . . . . . . . 24.4 Asymptotic Series . . . . . . . . . . . . . . . . 24.5 Asymptotic Expansions of Dierential Equations 24.5.1 The Parabolic Cylinder Equation. . . . . 25 Hilbert Spaces 25.1 Linear Spaces . . . . . . . . . . . 25.2 Inner Products . . . . . . . . . . . 25.3 Norms . . . . . . . . . . . . . . . 25.4 Linear Independence. . . . . . . . . 25.5 Orthogonality . . . . . . . . . . . 25.6 Gramm-Schmidt Orthogonalization 25.7 Orthonormal Function Expansion . 25.8 Sets Of Functions . . . . . . . . . 25.9 Least Squares Fit to a Function and 25.10Closure Relation . . . . . . . . . . 25.11Linear Operators . . . . . . . . . . 25.12Exercises . . . . . . . . . . . . . . 25.13Hints . . . . . . . . . . . . . . . .

. . . . . .

. . . . . .

. . . . . .

. . . . . .

. . . . . .

. . . . . .

. . . . . .

. . . . . .

. . . . . .

. . . . . .

. . . . . .

. . . . . .

. . . . . .

. . . . . .

. . . . . .

. . . . . .

. . . . . .

. . . . . .

. . . . . .

. . . . . .

. . . . . .

. . . . . .

. . . . . .

. . . . . .

. . . . . .

. . . . . .

. . . . . .

. . . . . .

1253 . 1253 . 1257 . 1266 . 1273 . 1274 . 1274 1280 . 1280 . 1282 . 1283 . 1285 . 1285 . 1286 . 1288 . 1290 . 1297 . 1300 . 1305 . 1306 . 1307

. . . . . . . . . . . . . . . . . . . . . . . . . . . . . . . . . . . . . . . . . . . . . . . . . . . . . . . . . . . . . . . . Completeness . . . . . . . . . . . . . . . . . . . . . . . . . . . . . . . .

. . . . . . . . . . . . .

. . . . . . . . . . . . .

. . . . . . . . . . . . .

. . . . . . . . . . . . .

. . . . . . . . . . . . .

. . . . . . . . . . . . .

. . . . . . . . . . . . .

. . . . . . . . . . . . .

. . . . . . . . . . . . .

. . . . . . . . . . . . .

. . . . . . . . . . . . .

. . . . . . . . . . . . .

. . . . . . . . . . . . .

. . . . . . . . . . . . .

. . . . . . . . . . . . .

. . . . . . . . . . . . .

. . . . . . . . . . . . .

. . . . . . . . . . . . .

. . . . . . . . . . . . .

. . . . . . . . . . . . .

. . . . . . . . . . . . .

. . . . . . . . . . . . .

. . . . . . . . . . . . .

. . . . . . . . . . . . .

. . . . . . . . . . . . .

. . . . . . . . . . . . .

. . . . . . . . . . . . .

xiii

25.14Solutions . . . . . . . . . . . . . . . . . . . . . . . . . . . . . . . . . . . . . . . . . . . . . . . . . . 1308 26 Self 26.1 26.2 26.3 26.4 26.5 Adjoint Linear Operators Adjoint Operators . . . . . Self-Adjoint Operators . . . Exercises . . . . . . . . . . Hints . . . . . . . . . . . . Solutions . . . . . . . . . . 1310 . 1310 . 1311 . 1314 . 1315 . 1316 1317 . 1317 . 1318 . 1321 . 1321 . 1326 . 1329 . 1330 . 1331 1333 . 1333 . 1336 . 1340 . 1344 . 1347 . 1348 . 1349 . 1352 . 1361 . 1361

. . . . .

. . . . .

. . . . .

. . . . .

. . . . .

. . . . .

. . . . .

. . . . .

. . . . .

. . . . .

. . . . .

. . . . .

. . . . .

. . . . .

. . . . .

. . . . .

. . . . .

. . . . .

. . . . .

. . . . .

. . . . .

. . . . .

. . . . .

. . . . .

. . . . .

. . . . .

. . . . .

. . . . .

. . . . .

. . . . .

. . . . .

. . . . .

. . . . .

. . . . .

. . . . .

. . . . .

. . . . .

. . . . .

. . . . .

27 Self-Adjoint Boundary Value Problems 27.1 Summary of Adjoint Operators . . . 27.2 Formally Self-Adjoint Operators . . . 27.3 Self-Adjoint Problems . . . . . . . . 27.4 Self-Adjoint Eigenvalue Problems . . 27.5 Inhomogeneous Equations . . . . . . 27.6 Exercises . . . . . . . . . . . . . . . 27.7 Hints . . . . . . . . . . . . . . . . . 27.8 Solutions . . . . . . . . . . . . . . .

. . . . . . . .

. . . . . . . .

. . . . . . . .

. . . . . . . .

. . . . . . . .

. . . . . . . .

. . . . . . . .

. . . . . . . .

. . . . . . . .

. . . . . . . .

. . . . . . . .

. . . . . . . .

. . . . . . . .

. . . . . . . .

. . . . . . . .

. . . . . . . .

. . . . . . . .

. . . . . . . .

. . . . . . . .

. . . . . . . .

. . . . . . . .

. . . . . . . .

. . . . . . . .

. . . . . . . .

. . . . . . . .

. . . . . . . .

. . . . . . . .

. . . . . . . .

. . . . . . . .

. . . . . . . .

. . . . . . . .

. . . . . . . .

. . . . . . . .

. . . . . . . .

28 Fourier Series 28.1 An Eigenvalue Problem. . . . . . . . . . . . . . . . . . 28.2 Fourier Series. . . . . . . . . . . . . . . . . . . . . . . 28.3 Least Squares Fit . . . . . . . . . . . . . . . . . . . . 28.4 Fourier Series for Functions Dened on Arbitrary Ranges 28.5 Fourier Cosine Series . . . . . . . . . . . . . . . . . . . 28.6 Fourier Sine Series . . . . . . . . . . . . . . . . . . . . 28.7 Complex Fourier Series and Parsevals Theorem . . . . . 28.8 Behavior of Fourier Coecients . . . . . . . . . . . . . 28.9 Gibbs Phenomenon . . . . . . . . . . . . . . . . . . . 28.10Integrating and Dierentiating Fourier Series . . . . . .

. . . . . . . . . .

. . . . . . . . . .

. . . . . . . . . .

. . . . . . . . . .

. . . . . . . . . .

. . . . . . . . . .

. . . . . . . . . .

. . . . . . . . . .

. . . . . . . . . .

. . . . . . . . . .

. . . . . . . . . .

. . . . . . . . . .

. . . . . . . . . .

. . . . . . . . . .

. . . . . . . . . .

. . . . . . . . . .

. . . . . . . . . .

. . . . . . . . . .

. . . . . . . . . .

. . . . . . . . . .

. . . . . . . . . .

. . . . . . . . . .

. . . . . . . . . .

. . . . . . . . . .

xiv

28.11Exercises . . . . . . . . . . . . . . . . . . . . . . . . . . . . . . . . . . . . . . . . . . . . . . . . . . 1366 28.12Hints . . . . . . . . . . . . . . . . . . . . . . . . . . . . . . . . . . . . . . . . . . . . . . . . . . . . 1374 28.13Solutions . . . . . . . . . . . . . . . . . . . . . . . . . . . . . . . . . . . . . . . . . . . . . . . . . . 1376 29 Regular Sturm-Liouville Problems 29.1 Derivation of the Sturm-Liouville Form . . . . . . . . . . . . 29.2 Properties of Regular Sturm-Liouville Problems . . . . . . . . 29.3 Solving Dierential Equations With Eigenfunction Expansions 29.4 Exercises . . . . . . . . . . . . . . . . . . . . . . . . . . . . 29.5 Hints . . . . . . . . . . . . . . . . . . . . . . . . . . . . . . 29.6 Solutions . . . . . . . . . . . . . . . . . . . . . . . . . . . . 30 Integrals and Convergence 30.1 Uniform Convergence of Integrals . . . 30.2 The Riemann-Lebesgue Lemma . . . . 30.3 Cauchy Principal Value . . . . . . . . 30.3.1 Integrals on an Innite Domain 30.3.2 Singular Functions . . . . . . . 1423 . 1423 . 1425 . 1436 . 1442 . 1446 . 1448 1473 . 1473 . 1474 . 1475 . 1475 . 1476 1478 . 1478 . 1480 . 1483 . 1488 . 1491 . 1493 . 1496 . 1498 . 1501 . 1508

. . . . . .

. . . . . .

. . . . . .

. . . . . .

. . . . . .

. . . . . .

. . . . . .

. . . . . .

. . . . . .

. . . . . .

. . . . . .

. . . . . .

. . . . . .

. . . . . .

. . . . . .

. . . . . .

. . . . . .

. . . . . .

. . . . . .

. . . . . .

. . . . . .

. . . . .

. . . . .

. . . . .

. . . . .

. . . . .

. . . . .

. . . . .

. . . . .

. . . . .

. . . . .

. . . . .

. . . . .

. . . . .

. . . . .

. . . . .

. . . . .

. . . . .

. . . . .

. . . . .

. . . . .

. . . . .

. . . . .

. . . . .

. . . . .

. . . . .

. . . . .

. . . . .

. . . . .

. . . . .

. . . . .

. . . . .

. . . . .

. . . . .

31 The Laplace Transform 31.1 The Laplace Transform . . . . . . . . . . . . . . . . 31.2 The Inverse Laplace Transform . . . . . . . . . . . . 31.2.1 f (s) with Poles . . . . . . . . . . . . . . . . 31.2.2 f (s) with Branch Points . . . . . . . . . . . . 31.2.3 Asymptotic Behavior of f (s) . . . . . . . . . 31.3 Properties of the Laplace Transform . . . . . . . . . . 31.4 Constant Coecient Dierential Equations . . . . . . 31.5 Systems of Constant Coecient Dierential Equations 31.6 Exercises . . . . . . . . . . . . . . . . . . . . . . . . 31.7 Hints . . . . . . . . . . . . . . . . . . . . . . . . . .

. . . . . . . . . .

. . . . . . . . . .

. . . . . . . . . .

. . . . . . . . . .

. . . . . . . . . .

. . . . . . . . . .

. . . . . . . . . .

. . . . . . . . . .

. . . . . . . . . .

. . . . . . . . . .

. . . . . . . . . .

. . . . . . . . . .

. . . . . . . . . .

. . . . . . . . . .

. . . . . . . . . .

. . . . . . . . . .

. . . . . . . . . .

. . . . . . . . . .

. . . . . . . . . .

. . . . . . . . . .

. . . . . . . . . .

. . . . . . . . . .

. . . . . . . . . .

. . . . . . . . . .

. . . . . . . . . .

xv

31.8 Solutions . . . . . . . . . . . . . . . . . . . . . . . . . . . . . . . . . . . . . . . . . . . . . . . . . . 1511 32 The Fourier Transform 32.1 Derivation from a Fourier Series . . . . . . . . . . . . . . . . . . . . . . . . . . . 32.2 The Fourier Transform . . . . . . . . . . . . . . . . . . . . . . . . . . . . . . . . 32.2.1 A Word of Caution . . . . . . . . . . . . . . . . . . . . . . . . . . . . . . 32.3 Evaluating Fourier Integrals . . . . . . . . . . . . . . . . . . . . . . . . . . . . . 32.3.1 Integrals that Converge . . . . . . . . . . . . . . . . . . . . . . . . . . . 32.3.2 Cauchy Principal Value and Integrals that are Not Absolutely Convergent. . 32.3.3 Analytic Continuation . . . . . . . . . . . . . . . . . . . . . . . . . . . . 32.4 Properties of the Fourier Transform . . . . . . . . . . . . . . . . . . . . . . . . . 32.4.1 Closure Relation. . . . . . . . . . . . . . . . . . . . . . . . . . . . . . . . 32.4.2 Fourier Transform of a Derivative. . . . . . . . . . . . . . . . . . . . . . . 32.4.3 Fourier Convolution Theorem. . . . . . . . . . . . . . . . . . . . . . . . . 32.4.4 Parsevals Theorem. . . . . . . . . . . . . . . . . . . . . . . . . . . . . . 32.4.5 Shift Property. . . . . . . . . . . . . . . . . . . . . . . . . . . . . . . . . 32.4.6 Fourier Transform of x f(x). . . . . . . . . . . . . . . . . . . . . . . . . . 32.5 Solving Dierential Equations with the Fourier Transform . . . . . . . . . . . . . 32.6 The Fourier Cosine and Sine Transform . . . . . . . . . . . . . . . . . . . . . . . 32.6.1 The Fourier Cosine Transform . . . . . . . . . . . . . . . . . . . . . . . . 32.6.2 The Fourier Sine Transform . . . . . . . . . . . . . . . . . . . . . . . . . 32.7 Properties of the Fourier Cosine and Sine Transform . . . . . . . . . . . . . . . . 32.7.1 Transforms of Derivatives . . . . . . . . . . . . . . . . . . . . . . . . . . 32.7.2 Convolution Theorems . . . . . . . . . . . . . . . . . . . . . . . . . . . . 32.7.3 Cosine and Sine Transform in Terms of the Fourier Transform . . . . . . . 32.8 Solving Dierential Equations with the Fourier Cosine and Sine Transforms . . . . 32.9 Exercises . . . . . . . . . . . . . . . . . . . . . . . . . . . . . . . . . . . . . . . 32.10Hints . . . . . . . . . . . . . . . . . . . . . . . . . . . . . . . . . . . . . . . . . 32.11Solutions . . . . . . . . . . . . . . . . . . . . . . . . . . . . . . . . . . . . . . . 1543 . 1543 . 1545 . 1548 . 1549 . 1549 . 1552 . 1554 . 1556 . 1556 . 1557 . 1559 . 1562 . 1564 . 1564 . 1565 . 1567 . 1567 . 1568 . 1569 . 1569 . 1571 . 1573 . 1574 . 1576 . 1583 . 1586

. . . . . . . . . . . . . . . . . . . . . . . . . .

. . . . . . . . . . . . . . . . . . . . . . . . . .

. . . . . . . . . . . . . . . . . . . . . . . . . .

. . . . . . . . . . . . . . . . . . . . . . . . . .

. . . . . . . . . . . . . . . . . . . . . . . . . .

. . . . . . . . . . . . . . . . . . . . . . . . . .

. . . . . . . . . . . . . . . . . . . . . . . . . .

. . . . . . . . . . . . . . . . . . . . . . . . . .

. . . . . . . . . . . . . . . . . . . . . . . . . .

. . . . . . . . . . . . . . . . . . . . . . . . . .

xvi

33 The 33.1 33.2 33.3 33.4 33.5 33.6 33.7 33.8

Gamma Function Eulers Formula . . . . Hankels Formula . . . . Gauss Formula . . . . . Weierstrass Formula . . Stirlings Approximation Exercises . . . . . . . . Hints . . . . . . . . . . Solutions . . . . . . . .

. . . . . . . .

. . . . . . . .

. . . . . . . .

. . . . . . . .

. . . . . . . .

. . . . . . . .

. . . . . . . .

. . . . . . . .

. . . . . . . .

. . . . . . . .

. . . . . . . .

. . . . . . . .

. . . . . . . .

. . . . . . . .

. . . . . . . .

. . . . . . . .

. . . . . . . .

. . . . . . . .

. . . . . . . .

. . . . . . . .

. . . . . . . .

. . . . . . . .

. . . . . . . .

. . . . . . . .

. . . . . . . .

. . . . . . . .

. . . . . . . .

. . . . . . . .

. . . . . . . .

. . . . . . . .

. . . . . . . .

. . . . . . . .

. . . . . . . .

. . . . . . . .

. . . . . . . .

. . . . . . . .

. . . . . . . .

. . . . . . . .

. . . . . . . .

. . . . . . . .

. . . . . . . .

1610 . 1610 . 1612 . 1614 . 1616 . 1618 . 1623 . 1624 . 1625 1627 . 1627 . 1628 . 1631 . 1633 . 1634 . 1635 . 1638 . 1641 . 1644 . 1645 . 1649 . 1651 . 1651 . 1655 . 1660 . 1662

34 Bessel Functions 34.1 Bessels Equation . . . . . . . . . . . . . . . . . . . . 34.2 Frobeneius Series Solution about z = 0 . . . . . . . . . 34.2.1 Behavior at Innity . . . . . . . . . . . . . . . . 34.3 Bessel Functions of the First Kind . . . . . . . . . . . . 34.3.1 The Bessel Function Satises Bessels Equation . 34.3.2 Series Expansion of the Bessel Function . . . . . 34.3.3 Bessel Functions of Non-Integer Order . . . . . 34.3.4 Recursion Formulas . . . . . . . . . . . . . . . 34.3.5 Bessel Functions of Half-Integer Order . . . . . 34.4 Neumann Expansions . . . . . . . . . . . . . . . . . . 34.5 Bessel Functions of the Second Kind . . . . . . . . . . 34.6 Hankel Functions . . . . . . . . . . . . . . . . . . . . . 34.7 The Modied Bessel Equation . . . . . . . . . . . . . . 34.8 Exercises . . . . . . . . . . . . . . . . . . . . . . . . . 34.9 Hints . . . . . . . . . . . . . . . . . . . . . . . . . . . 34.10Solutions . . . . . . . . . . . . . . . . . . . . . . . . .

. . . . . . . . . . . . . . . .

. . . . . . . . . . . . . . . .

. . . . . . . . . . . . . . . .

. . . . . . . . . . . . . . . .

. . . . . . . . . . . . . . . .

. . . . . . . . . . . . . . . .

. . . . . . . . . . . . . . . .

. . . . . . . . . . . . . . . .

. . . . . . . . . . . . . . . .

. . . . . . . . . . . . . . . .

. . . . . . . . . . . . . . . .

. . . . . . . . . . . . . . . .

. . . . . . . . . . . . . . . .

. . . . . . . . . . . . . . . .

. . . . . . . . . . . . . . . .

. . . . . . . . . . . . . . . .

. . . . . . . . . . . . . . . .

. . . . . . . . . . . . . . . .

. . . . . . . . . . . . . . . .

. . . . . . . . . . . . . . . .

. . . . . . . . . . . . . . . .

. . . . . . . . . . . . . . . .

. . . . . . . . . . . . . . . .

. . . . . . . . . . . . . . . .

xvii

Partial Dierential Equations

1685

35 Transforming 35.1 Exercises 35.2 Hints . . 35.3 Solutions

Equations 1686 . . . . . . . . . . . . . . . . . . . . . . . . . . . . . . . . . . . . . . . . . . . . . . . . . . 1687 . . . . . . . . . . . . . . . . . . . . . . . . . . . . . . . . . . . . . . . . . . . . . . . . . . 1688 . . . . . . . . . . . . . . . . . . . . . . . . . . . . . . . . . . . . . . . . . . . . . . . . . . 1689 1690 . 1690 . 1691 . 1696 . 1697 . 1699 . 1701 . 1702 . 1703 1709 . 1709 . 1709 . 1711 . 1714 . 1715 . 1718 . 1721 . 1723 . 1739 . 1744

36 Classication of Partial Dierential Equations 36.1 Classication of Second Order Quasi-Linear Equations 36.1.1 Hyperbolic Equations . . . . . . . . . . . . . 36.1.2 Parabolic equations . . . . . . . . . . . . . . 36.1.3 Elliptic Equations . . . . . . . . . . . . . . . 36.2 Equilibrium Solutions . . . . . . . . . . . . . . . . . 36.3 Exercises . . . . . . . . . . . . . . . . . . . . . . . . 36.4 Hints . . . . . . . . . . . . . . . . . . . . . . . . . . 36.5 Solutions . . . . . . . . . . . . . . . . . . . . . . . .

. . . . . . . .

. . . . . . . .

. . . . . . . .

. . . . . . . .

. . . . . . . .

. . . . . . . .

. . . . . . . .

. . . . . . . .

. . . . . . . .

. . . . . . . .

. . . . . . . .

. . . . . . . .

. . . . . . . .

. . . . . . . .

. . . . . . . .

. . . . . . . .

. . . . . . . .

. . . . . . . .

. . . . . . . .

. . . . . . . .

. . . . . . . .

. . . . . . . .

. . . . . . . .

. . . . . . . .

. . . . . . . .

37 Separation of Variables 37.1 Eigensolutions of Homogeneous Equations . . . . . . . . . . . . . 37.2 Homogeneous Equations with Homogeneous Boundary Conditions . 37.3 Time-Independent Sources and Boundary Conditions . . . . . . . . 37.4 Inhomogeneous Equations with Homogeneous Boundary Conditions 37.5 Inhomogeneous Boundary Conditions . . . . . . . . . . . . . . . . 37.6 The Wave Equation . . . . . . . . . . . . . . . . . . . . . . . . . 37.7 General Method . . . . . . . . . . . . . . . . . . . . . . . . . . . 37.8 Exercises . . . . . . . . . . . . . . . . . . . . . . . . . . . . . . . 37.9 Hints . . . . . . . . . . . . . . . . . . . . . . . . . . . . . . . . . 37.10Solutions . . . . . . . . . . . . . . . . . . . . . . . . . . . . . . .

. . . . . . . . . .

. . . . . . . . . .

. . . . . . . . . .

. . . . . . . . . .

. . . . . . . . . .

. . . . . . . . . .

. . . . . . . . . .

. . . . . . . . . .

. . . . . . . . . .

. . . . . . . . . .

. . . . . . . . . .

. . . . . . . . . .

. . . . . . . . . .

. . . . . . . . . .

. . . . . . . . . .

. . . . . . . . . .

. . . . . . . . . .

. . . . . . . . . .

xviii

38 Finite Transforms 1826 38.1 Exercises . . . . . . . . . . . . . . . . . . . . . . . . . . . . . . . . . . . . . . . . . . . . . . . . . . 1830 38.2 Hints . . . . . . . . . . . . . . . . . . . . . . . . . . . . . . . . . . . . . . . . . . . . . . . . . . . . 1831 38.3 Solutions . . . . . . . . . . . . . . . . . . . . . . . . . . . . . . . . . . . . . . . . . . . . . . . . . . 1832 39 The 39.1 39.2 39.3 Diusion Equation 1836 Exercises . . . . . . . . . . . . . . . . . . . . . . . . . . . . . . . . . . . . . . . . . . . . . . . . . . 1837 Hints . . . . . . . . . . . . . . . . . . . . . . . . . . . . . . . . . . . . . . . . . . . . . . . . . . . . 1839 Solutions . . . . . . . . . . . . . . . . . . . . . . . . . . . . . . . . . . . . . . . . . . . . . . . . . . 1840 1846 . 1846 . 1846 . 1847 . 1848 . 1851 . 1852

40 Laplaces Equation 40.1 Introduction . . . . . . . . . . 40.2 Fundamental Solution . . . . . 40.2.1 Two Dimensional Space 40.3 Exercises . . . . . . . . . . . . 40.4 Hints . . . . . . . . . . . . . . 40.5 Solutions . . . . . . . . . . . .

. . . . . .

. . . . . .

. . . . . .

. . . . . .

. . . . . .

. . . . . .

. . . . . .

. . . . . .

. . . . . .

. . . . . .

. . . . . .

. . . . . .

. . . . . .

. . . . . .

. . . . . .

. . . . . .

. . . . . .

. . . . . .

. . . . . .

. . . . . .

. . . . . .

. . . . . .

. . . . . .

. . . . . .

. . . . . .

. . . . . .

. . . . . .

. . . . . .

. . . . . .

. . . . . .

. . . . . .

. . . . . .

. . . . . .

. . . . . .

. . . . . .

. . . . . .

. . . . . .

41 Waves 1864 41.1 Exercises . . . . . . . . . . . . . . . . . . . . . . . . . . . . . . . . . . . . . . . . . . . . . . . . . . 1865 41.2 Hints . . . . . . . . . . . . . . . . . . . . . . . . . . . . . . . . . . . . . . . . . . . . . . . . . . . . 1871 41.3 Solutions . . . . . . . . . . . . . . . . . . . . . . . . . . . . . . . . . . . . . . . . . . . . . . . . . . 1873 42 Similarity Methods 42.1 Exercises . . . . . . . . . . . . . . . . . . . . . . . . . . . . . . . . . . . . . . . . . . . . . . . . . 42.2 Hints . . . . . . . . . . . . . . . . . . . . . . . . . . . . . . . . . . . . . . . . . . . . . . . . . . . 42.3 Solutions . . . . . . . . . . . . . . . . . . . . . . . . . . . . . . . . . . . . . . . . . . . . . . . . . 1893 . 1898 . 1899 . 1900

43 Method of Characteristics 1903 43.1 First Order Linear Equations . . . . . . . . . . . . . . . . . . . . . . . . . . . . . . . . . . . . . . . . 1903 43.2 First Order Quasi-Linear Equations . . . . . . . . . . . . . . . . . . . . . . . . . . . . . . . . . . . . 1904 xix

43.3 The Method of Characteristics and the Wave Equation 43.4 The Wave Equation for an Innite Domain . . . . . . 43.5 The Wave Equation for a Semi-Innite Domain . . . . 43.6 The Wave Equation for a Finite Domain . . . . . . . 43.7 Envelopes of Curves . . . . . . . . . . . . . . . . . . 43.8 Exercises . . . . . . . . . . . . . . . . . . . . . . . . 43.9 Hints . . . . . . . . . . . . . . . . . . . . . . . . . . 43.10Solutions . . . . . . . . . . . . . . . . . . . . . . . . 44 Transform Methods 44.1 Fourier Transform for Partial Dierential Equations 44.2 The Fourier Sine Transform . . . . . . . . . . . . 44.3 Fourier Transform . . . . . . . . . . . . . . . . . 44.4 Exercises . . . . . . . . . . . . . . . . . . . . . . 44.5 Hints . . . . . . . . . . . . . . . . . . . . . . . . 44.6 Solutions . . . . . . . . . . . . . . . . . . . . . .

. . . . . . . .

. . . . . . . .

. . . . . . . .

. . . . . . . .

. . . . . . . .

. . . . . . . .

. . . . . . . .

. . . . . . . .

. . . . . . . .

. . . . . . . .

. . . . . . . .

. . . . . . . .

. . . . . . . .

. . . . . . . .

. . . . . . . .

. . . . . . . .

. . . . . . . .

. . . . . . . .

. . . . . . . .

. . . . . . . .

. . . . . . . .

. . . . . . . .

. . . . . . . .

. . . . . . . .

. . . . . . . .

. . . . . . . .

1906 1907 1908 1910 1911 1914 1916 1917

. . . . . .

. . . . . .

. . . . . .

. . . . . .

. . . . . .

. . . . . .

. . . . . .

. . . . . .

. . . . . .

. . . . . .

. . . . . .

. . . . . .

. . . . . .

. . . . . .

. . . . . .

. . . . . .

. . . . . .

. . . . . .

. . . . . .

. . . . . .

. . . . . .

. . . . . .

. . . . . .

. . . . . .

. . . . . .

. . . . . .

. . . . . .

1924 . 1924 . 1926 . 1926 . 1928 . 1932 . 1934 1956 . 1956 . 1957 . 1959 . 1964 . 1966 . 1977 . 1980 2040 . 2041 . 2044 . 2045

45 Green Functions 45.1 Inhomogeneous Equations and Homogeneous Boundary Conditions 45.2 Homogeneous Equations and Inhomogeneous Boundary Conditions 45.3 Eigenfunction Expansions for Elliptic Equations . . . . . . . . . . . 45.4 The Method of Images . . . . . . . . . . . . . . . . . . . . . . . . 45.5 Exercises . . . . . . . . . . . . . . . . . . . . . . . . . . . . . . . 45.6 Hints . . . . . . . . . . . . . . . . . . . . . . . . . . . . . . . . . 45.7 Solutions . . . . . . . . . . . . . . . . . . . . . . . . . . . . . . .

. . . . . . .

. . . . . . .

. . . . . . .

. . . . . . .

. . . . . . .

. . . . . . .

. . . . . . .

. . . . . . .

. . . . . . .

. . . . . . .

. . . . . . .

. . . . . . .

. . . . . . .

. . . . . . .

. . . . . . .

. . . . . . .

. . . . . . .

. . . . . . .

46 Conformal Mapping 46.1 Exercises . . . . . . . . . . . . . . . . . . . . . . . . . . . . . . . . . . . . . . . . . . . . . . . . . 46.2 Hints . . . . . . . . . . . . . . . . . . . . . . . . . . . . . . . . . . . . . . . . . . . . . . . . . . . 46.3 Solutions . . . . . . . . . . . . . . . . . . . . . . . . . . . . . . . . . . . . . . . . . . . . . . . . .

xx

47 Non-Cartesian Coordinates 47.1 Spherical Coordinates . . . . . . . . . . . . . . . . . . . . . . . . . . . . . . . . . . . . . . . . . . 47.2 Laplaces Equation in a Disk . . . . . . . . . . . . . . . . . . . . . . . . . . . . . . . . . . . . . . . 47.3 Laplaces Equation in an Annulus . . . . . . . . . . . . . . . . . . . . . . . . . . . . . . . . . . . .

2057 . 2057 . 2058 . 2061

VI

Calculus of Variations

2065

48 Calculus of Variations 2066 48.1 Exercises . . . . . . . . . . . . . . . . . . . . . . . . . . . . . . . . . . . . . . . . . . . . . . . . . . 2067 48.2 Hints . . . . . . . . . . . . . . . . . . . . . . . . . . . . . . . . . . . . . . . . . . . . . . . . . . . . 2081 48.3 Solutions . . . . . . . . . . . . . . . . . . . . . . . . . . . . . . . . . . . . . . . . . . . . . . . . . . 2085

VII

Nonlinear Dierential Equations

2172

49 Nonlinear Ordinary Dierential Equations 2173 49.1 Exercises . . . . . . . . . . . . . . . . . . . . . . . . . . . . . . . . . . . . . . . . . . . . . . . . . . 2174 49.2 Hints . . . . . . . . . . . . . . . . . . . . . . . . . . . . . . . . . . . . . . . . . . . . . . . . . . . . 2179 49.3 Solutions . . . . . . . . . . . . . . . . . . . . . . . . . . . . . . . . . . . . . . . . . . . . . . . . . . 2180 50 Nonlinear Partial 50.1 Exercises . . 50.2 Hints . . . . 50.3 Solutions . . Dierential Equations . . . . . . . . . . . . . . . . . . . . . . . . . . . . . . . . . . . . . . . . . . . . . . . . . . . . . . . . . . . . . . . . . . . . . . . . . . . . . . . . . . . . . . . . . . . . . . . . . . . . . . . . . . . . . . . . . . . . . . . . . . . . . . . . . . . . . . . . . . . . . 2202 . 2203 . 2206 . 2207

VIII

Appendices

2226
2227

A Greek Letters

xxi

B Notation C Formulas from Complex Variables D Table of Derivatives E Table of Integrals F Denite Integrals G Table of Sums H Table of Taylor Series I

2229 2231 2234 2238 2242 2244 2247

Table of Laplace Transforms 2250 I.1 Properties of Laplace Transforms . . . . . . . . . . . . . . . . . . . . . . . . . . . . . . . . . . . . . 2250 I.2 Table of Laplace Transforms . . . . . . . . . . . . . . . . . . . . . . . . . . . . . . . . . . . . . . . . 2252 2256 2259 2260 2262 2264 2266 2268

J Table of Fourier Transforms K Table of Fourier Transforms in n Dimensions L Table of Fourier Cosine Transforms M Table of Fourier Sine Transforms N Table of Wronskians O Sturm-Liouville Eigenvalue Problems P Green Functions for Ordinary Dierential Equations

xxii

Q Trigonometric Identities 2271 Q.1 Circular Functions . . . . . . . . . . . . . . . . . . . . . . . . . . . . . . . . . . . . . . . . . . . . . 2271 Q.2 Hyperbolic Functions . . . . . . . . . . . . . . . . . . . . . . . . . . . . . . . . . . . . . . . . . . . . 2273 R Bessel Functions 2276 R.1 Denite Integrals . . . . . . . . . . . . . . . . . . . . . . . . . . . . . . . . . . . . . . . . . . . . . . 2276 S Formulas from Linear Algebra T Vector Analysis U Partial Fractions V Finite Math W Physics 2277 2278 2280 2284 2285

X Probability 2286 X.1 Independent Events . . . . . . . . . . . . . . . . . . . . . . . . . . . . . . . . . . . . . . . . . . . . 2286 X.2 Playing the Odds . . . . . . . . . . . . . . . . . . . . . . . . . . . . . . . . . . . . . . . . . . . . . . 2287 Y Economics Z Glossary 2288 2289

xxiii

Anti-Copyright
Anti-Copyright @ 1995-2001 by Mauch Publishing Company, un-Incorporated. No rights reserved. Any part of this publication may be reproduced, stored in a retrieval system, transmitted or desecrated without permission.

xxiv

Preface
During the summer before my nal undergraduate year at Caltech I set out to write a math text unlike any other, namely, one written by me. In that respect I have succeeded beautifully. Unfortunately, the text is neither complete nor polished. I have a Warnings and Disclaimers section below that is a little amusing, and an appendix on probability that I feel concisesly captures the essence of the subject. However, all the material in between is in some stage of development. I am currently working to improve and expand this text. This text is freely available from my web set. Currently Im at http://www.its.caltech.edu/sean. I post new versions a couple of times a year.

0.1

Advice to Teachers

If you have something worth saying, write it down.

0.2

Acknowledgments

I would like to thank Professor Saman for advising me on this project and the Caltech SURF program for providing the funding for me to write the rst edition of this book.

xxv

0.3

Warnings and Disclaimers

This book is a work in progress. It contains quite a few mistakes and typos. I would greatly appreciate your constructive criticism. You can reach me at sean@its.caltech.edu. Reading this book impairs your ability to drive a car or operate machinery. This book has been found to cause drowsiness in laboratory animals. This book contains twenty-three times the US RDA of ber. Caution: FLAMMABLE - Do not read while smoking or near a re. If infection, rash, or irritation develops, discontinue use and consult a physician. Warning: For external use only. Use only as directed. Intentional misuse by deliberately concentrating contents can be harmful or fatal. KEEP OUT OF REACH OF CHILDREN. In the unlikely event of a water landing do not use this book as a otation device. The material in this text is ction; any resemblance to real theorems, living or dead, is purely coincidental. This is by far the most amusing section of this book. Finding the typos and mistakes in this book is left as an exercise for the reader. (Eye ewes a spelling chequer from thyme too thyme, sew their should knot bee two many misspellings. Though I aint so sure the grammars too good.) The theorems and methods in this text are subject to change without notice. This is a chain book. If you do not make seven copies and distribute them to your friends within ten days of obtaining this text you will suer great misfortune and other nastiness. The surgeon general has determined that excessive studying is detrimental to your social life. xxvi

This text has been buered for your protection and ribbed for your pleasure. Stop reading this rubbish and get back to work!

0.4

Suggested Use

This text is well suited to the student, professional or lay-person. It makes a superb gift. This text has a boquet that is light and fruity, with some earthy undertones. It is ideal with dinner or as an apertif. Bon apetit!

0.5

About the Title

The title is only making light of naming conventions in the sciences and is not an insult to engineers. If you want to learn about some mathematical subject, look for books with Introduction or Elementary in the title. If it is an Intermediate text it will be incomprehensible. If it is Advanced then not only will it be incomprehensible, it will have low production qualities, i.e. a crappy typewriter font, no graphics and no examples. There is an exception to this rule: When the title also contains the word Scientists or Engineers the advanced book may be quite suitable for actually learning the material.

xxvii

Part I Algebra

Chapter 1 Sets and Functions


1.1 Sets

Denition. A set is a collection of objects. We call the objects, elements. A set is denoted by listing elements the between braces. For example: {e, , , 1} is the set of the integer 1, the pure imaginary number = 1 and the transcendental numbers e = 2.7182818 . . . and = 3.1415926 . . .. For elements of a set, we do not count multiplicities. We regard the set {1, 2, 2, 3, 3, 3} as identical to the set {1, 2, 3}. Order is not signicant in sets. The set {1, 2, 3} is equivalent to {3, 2, 1}. In enumerating the elements of a set, we use ellipses to indicate patterns. We denote the set of positive integers as {1, 2, 3, . . .}. We also denote sets with the notation {x|conditions on x} for sets that are more easily described than enumerated. This is read as the set of elements x such that . . . . x S is the notation for x is an element of the set S. To express the opposite we have x S for x is not an element of the set S. Examples. We have notations for denoting some of the commonly encountered sets. = {} is the empty set, the set containing no elements. Z = {. . . , 3, 2, 1, 0, 1, 2, 3 . . .} is the set of integers. (Z is for Zahlen, the German word for number.) 2

Q = {p/q|p, q Z, q = 0} is the set of rational numbers. (Q is for quotient.)

1 2

R = {x|x = a1 a2 an .b1 b2 } is the set of real numbers, i.e. the set of numbers with decimal expansions.

C = {a + b|a, b R, 2 = 1} is the set of complex numbers. is the square root of 1. (If you havent seen complex numbers before, dont dismay. Well cover them later.) Z+ , Q+ and R+ are the sets of positive integers, rationals and reals, respectively. For example, Z+ = {1, 2, 3, . . .}. Z0+ , Q0+ and R0+ are the sets of non-negative integers, rationals and reals, respectively. For example, Z0+ = {0, 1, 2, . . .}. (a . . . b) denotes an open interval on the real axis. (a . . . b) {x|x R, a < x < b} We use brackets to denote the closed interval. [a..b] {x|x R, a x b} The cardinality or order of a set S is denoted |S|. For nite sets, the cardinality is the number of elements in the set. The Cartesian product of two sets is the set of ordered pairs: X Y {(x, y)|x X, y Y }. The Cartesian product of n sets is the set of ordered n-tuples: X1 X2 Xn {(x1 , x2 , . . . , xn )|x1 X1 , x2 X2 , . . . , xn Xn }. Equality. Two sets S and T are equal if each element of S is an element of T and vice versa. This is denoted, S = T . Inequality is S = T , of course. S is a subset of T , S T , if every element of S is an element of T . S is a proper subset of T , S T , if S T and S = T . For example: The empty set is a subset of every set, S. The rational numbers are a proper subset of the real numbers, Q R.
Note that with this description, we enumerate each rational number an innite number of times. For example: 1/2 = 2/4 = 3/6 = (1)/(2) = . This does not pose a problem as we do not count multiplicities. 2 Guess what R is for.
1

Operations. The union of two sets, S T , is the set whose elements are in either of the two sets. The union of n sets, n Sj S1 S2 Sn j=1 is the set whose elements are in any of the sets Sj . The intersection of two sets, S T , is the set whose elements are in both of the two sets. In other words, the intersection of two sets in the set of elements that the two sets have in common. The intersection of n sets, n Sj S1 S2 Sn j=1 is the set whose elements are in all of the sets Sj . If two sets have no elements in common, S T = , then the sets are disjoint. If T S, then the dierence between S and T , S \ T , is the set of elements in S which are not in T . S \ T {x|x S, x T } The dierence of sets is also denoted S T . Properties. The following properties are easily veried from the above denitions. S = S, S = , S \ = S, S \ S = . Commutative. S T = T S, S T = T S. Associative. (S T ) U = S (T U ) = S T U , (S T ) U = S (T U ) = S T U . Distributive. S (T U ) = (S T ) (S U ), S (T U ) = (S T ) (S U ).

1.2

Single Valued Functions

Single-Valued Functions. A single-valued function or single-valued mapping is a mapping of the elements x X f into elements y Y . This is expressed as f : X Y or X Y . If such a function is well-dened, then for each x X there exists a unique element of y such that f (x) = y. The set X is the domain of the function, Y is the codomain, (not to be confused with the range, which we introduce shortly). To denote the value of a function on a 4

particular element we can use any of the notations: f (x) = y, f : x y or simply x y. f is the identity map on X if f (x) = x for all x X. Let f : X Y . The range or image of f is f (X) = {y|y = f (x) for some x X}. The range is a subset of the codomain. For each Z Y , the inverse image of Z is dened: f 1 (Z) {x X|f (x) = z for some z Z}. Examples. Finite polynomials, f (x) = n ak xk , ak R, and the exponential function, f (x) = ex , are examples of single k=0 valued functions which map real numbers to real numbers. The greatest integer function, f (x) = x , is a mapping from R to Z. x is dened as the greatest integer less than or equal to x. Likewise, the least integer function, f (x) = x , is the least integer greater than or equal to x. The -jectives. A function is injective if for each x1 = x2 , f (x1 ) = f (x2 ). In other words, distinct elements are mapped to distinct elements. f is surjective if for each y in the codomain, there is an x such that y = f (x). If a function is both injective and surjective, then it is bijective. A bijective function is also called a one-to-one mapping. Examples. The exponential function f (x) = ex , considered as a mapping from R to R+ , is bijective, (a one-to-one mapping). f (x) = x2 is a bijection from R+ to R+ . f is not injective from R to R+ . For each positive y in the range, there are two values of x such that y = x2 . f (x) = sin x is not injective from R to [1..1]. For each y [1..1] there exists an innite number of values of x such that y = sin x.

Injective

Surjective

Bijective

Figure 1.1: Depictions of Injective, Surjective and Bijective Functions

1.3

Inverses and Multi-Valued Functions

If y = f (x), then we can write x = f 1 (y) where f 1 is the inverse of f . If y = f (x) is a one-to-one function, then f 1 (y) is also a one-to-one function. In this case, x = f 1 (f (x)) = f (f 1 (x)) for values of x where both f (x) and f 1 (x) are dened. For example ln x, which maps R+ to R is the inverse of ex . x = eln x = ln(ex ) for all x R+ . (Note the x R+ ensures that ln x is dened.) If y = f (x) is a many-to-one function, then x = f 1 (y) is a one-to-many function. f 1 (y) is a multi-valued function. We have x = f (f 1 (x)) for values of x where f 1 (x) is dened, however x = f 1 (f (x)). There are diagrams showing one-to-one, many-to-one and one-to-many functions in Figure 1.2. Example 1.3.1 y = x2 , a many-to-one function has the inverse x = y 1/2 . For each positive y, there are two values of x such that x = y 1/2 . y = x2 and y = x1/2 are graphed in Figure 1.3.

We say that there are two branches of y = x1/2 : the positive and the negative branch. We denote the positive 1/2 branch as y = x; the negative branch is y = x. We call x the principal branch of x . Note that x is a 6

one-to-one

many-to-one

one-to-many

domain

range

domain

range

domain

range

Figure 1.2: Diagrams of One-To-One, Many-To-One and One-To-Many Functions

Figure 1.3: y = x2 and y = x1/2 one-to-one function. Finally, x = (x1/2 )2 since ( x)2 = x, but x = (x2 )1/2 since (x2 )1/2 = x. y = x is graphed in Figure 1.4. Now consider the many-to-one function y = sin x. The inverse is x = arcsin y. For each y [1..1] there are an innite number of values x such that x = arcsin y. In Figure 1.5 is a graph of y = sin x and a graph of a few branches of y = arcsin x. Example 1.3.2 arcsin x has an innite number of branches. We will denote the principal branch by Arcsin x which maps [1..1] to .. . Note that x = sin(arcsin x), but x = arcsin(sin x). y = Arcsin x in Figure 1.6. 2 2

Figure 1.4: y =

Figure 1.5: y = sin x and y = arcsin x

Figure 1.6: y = Arcsin x Example 1.3.3 Consider 11/3 . Since x3 is a one-to-one function, x1/3 is a single-valued function. (See Figure 1.7.) 11/3 = 1.

Figure 1.7: y = x3 and y = x1/3 Example 1.3.4 Consider arccos(1/2). cos x and a portion of arccos x are graphed in Figure 1.8. The equation cos x = 1/2 has the two solutions x = /3 in the range x (..]. We use the periodicity of the cosine, cos(x + 2) = cos x, to nd the remaining solutions. arccos(1/2) = {/3 + 2n}, n Z.

Figure 1.8: y = cos x and y = arccos x

1.4

Transforming Equations

Consider the equation g(x) = h(x) and the single-valued function f (x). A particular value of x is a solution of the equation if substituting that value into the equation results in an identity. In determining the solutions of an equation, 9

we often apply functions to each side of the equation in order to simplify its form. We apply the function to obtain a second equation, f (g(x)) = f (h(x)). If x = is a solution of the former equation, (let = g() = h()), then it is necessarily a solution of latter. This is because f (g()) = f (h()) reduces to the identity f () = f (). If f (x) is bijective, then the converse is true: any solution of the latter equation is a solution of the former equation. Suppose that x = is a solution of the latter, f (g()) = f (h()). That f (x) is a one-to-one mapping implies that g() = h(). Thus x = is a solution of the former equation. It is always safe to apply a one-to-one, (bijective), function to an equation, (provided it is dened for that domain). For example, we can apply f (x) = x3 or f (x) = ex , considered as mappings on R, to the equation x = 1. The equations x3 = 1 and ex = e each have the unique solution x = 1 for x R. In general, we must take care in applying functions to equations. If we apply a many-to-one function, we may 2 introduce spurious solutions. Applying f (x) = x2 to the equation x = results in x2 = 4 , which has the two solutions, 2 2 x = { }. Applying f (x) = sin x results in x2 = 4 , which has an innite number of solutions, x = { +2n | n Z}. 2 2 We do not generally apply a one-to-many, (multi-valued), function to both sides of an equation as this rarely is useful. Rather, we typically use the denition of the inverse function. Consider the equation sin2 x = 1. Applying the function f (x) = x1/2 to the equation would not get us anywhere. sin2 x
1/2

= 11/2

Since (sin2 x)1/2 = sin x, we cannot simplify the left side of the equation. Instead we could use the denition of f (x) = x1/2 as the inverse of the x2 function to obtain sin x = 11/2 = 1. Now note that we should not just apply arcsin to both sides of the equation as arcsin(sin x) = x. Instead we use the denition of arcsin as the inverse of sin. x = arcsin(1) 10

x = arcsin(1) has the solutions x = /2 + 2n and x = arcsin(1) has the solutions x = /2 + 2n. We enumerate the solutions. + n | n Z x= 2

11

1.5

Exercises

Exercise 1.1 The area of a circle is directly proportional to the square of its diameter. What is the constant of proportionality? Hint, Solution Exercise 1.2 Consider the equation x+1 x2 1 = 2 . y2 y 4 1. Why might one think that this is the equation of a line? 2. Graph the solutions of the equation to demonstrate that it is not the equation of a line. Hint, Solution Exercise 1.3 Consider the function of a real variable, f (x) = What is the domain and range of the function? Hint, Solution Exercise 1.4 The temperature measured in degrees Celsius 3 is linearly related to the temperature measured in degrees Fahrenheit 4 . Water freezes at 0 C = 32 F and boils at 100 C = 212 F . Write the temperature in degrees Celsius as a function of degrees Fahrenheit.
Originally, it was called degrees Centigrade. centi because there are 100 degrees between the two calibration points. It is now called degrees Celsius in honor of the inventor. 4 The Fahrenheit scale, named for Daniel Fahrenheit, was originally calibrated with the freezing point of salt-saturated water to be 0 . Later, the calibration points became the freezing point of water, 32 , and body temperature, 96 . With this method, there are 64 divisions between the calibration points. Finally, the upper calibration point was changed to the boiling point of water at 212 . This gave 180 divisions, (the number of degrees in a half circle), between the two calibration points.
3

x2

1 . +2

12

Hint, Solution Exercise 1.5 Consider the function graphed in Figure 1.9. Sketch graphs of f (x), f (x + 3), f (3 x) + 2, and f 1 (x). You may use the blank grids in Figure 1.10.

Figure 1.9: Graph of the function. Hint, Solution Exercise 1.6 A culture of bacteria grows at the rate of 10% per minute. At 6:00 pm there are 1 billion bacteria. How many bacteria are there at 7:00 pm? How many were there at 3:00 pm? Hint, Solution Exercise 1.7 The graph in Figure 1.11 shows an even function f (x) = p(x)/q(x) where p(x) and q(x) are rational quadratic polynomials. Give possible formulas for p(x) and q(x). Hint, Solution 13

Figure 1.10: Blank grids. Exercise 1.8 Find a polynomial of degree 100 which is zero only at x = 2, 1, and is non-negative. Hint, Solution

14

10

Figure 1.11: Plots of f (x) = p(x)/q(x).

1.6

Hints

Hint 1.1 area = constant diameter2 . Hint 1.2 A pair (x, y) is a solution of the equation if it make the equation an identity. Hint 1.3 The domain is the subset of R on which the function is dened. Hint 1.4 Find the slope and x-intercept of the line. Hint 1.5 The inverse of the function is the reection of the function across the line y = x. Hint 1.6 The formula for geometric growth/decay is x(t) = x0 rt , where r is the rate.

15

Hint 1.7 Note that p(x) and q(x) appear as a ratio, they are determined only up to a multiplicative constant. We may take the leading coecient of q(x) to be unity. p(x) ax2 + bx + c f (x) = = 2 q(x) x + x + Use the properties of the function to solve for the unknown parameters. Hint 1.8 Write the polynomial in factored form.

16

1.7

Solutions

Solution 1.1 area = radius2 area = diameter2 4 The constant of proportionality is . 4 Solution 1.2 1. If we multiply the equation by y 2 4 and divide by x + 1, we obtain the equation of a line. y+2=x1 2. We factor the quadratics on the right side of the equation. x+1 (x + 1)(x 1) = . y2 (y 2)(y + 2) We note that one or both sides of the equation are undened at y = 2 because of division by zero. There are no solutions for these two values of y and we assume from this point that y = 2. We multiply by (y 2)(y + 2). (x + 1)(y + 2) = (x + 1)(x 1) For x = 1, the equation becomes the identity 0 = 0. Now we consider x = 1. We divide by x + 1 to obtain the equation of a line. y+2=x1 y =x3 Now we collect the solutions we have found. {(1, y) : y = 2} {(x, x 3) : x = 1, 5} The solutions are depicted in Figure /refg not a line. 17

-6

-4

-2 -2

-4

-6

Figure 1.12: The solutions of

x+1 y2

x2 1 . y 2 4

Solution 1.3 The denominator is nonzero for all x R. Since we dont have any division by zero problems, the domain of the function is R. For x R, 1 2. 0< 2 x +2 Consider 1 . (1.1) y= 2 x +2 For any y (0 . . . 1/2], there is at least one value of x that satises Equation 1.1. x2 + 2 = x= Thus the range of the function is (0 . . . 1/2] 18 1 y

1 2 y

Solution 1.4 Let c denote degrees Celsius and f denote degrees Fahrenheit. The line passes through the points (f, c) = (32, 0) and (f, c) = (212, 100). The x-intercept is f = 32. We calculate the slope of the line. slope = The relationship between fahrenheit and celcius is 5 c = (f 32). 9 Solution 1.5 We plot the various transformations of f (x). Solution 1.6 The formula for geometric growth/decay is x(t) = x0 rt , where r is the rate. Let t = 0 coincide with 6:00 pm. We determine x0 . x(0) = 109 = x0 11 10
0

100 5 100 0 = = 212 32 180 9

= x0

x0 = 109 At 7:00 pm the number of bacteria is 10 At 3:00 pm the number of bacteria was 109 11 10
180 9

11 10

60

1160 3.04 1011 51 10

10189 35.4 11180

19

Figure 1.13: Graphs of f (x), f (x + 3), f (3 x) + 2, and f 1 (x). Solution 1.7 We write p(x) and q(x) as general quadratic polynomials. p(x) ax2 + bx + c f (x) = = q(x) x2 + x + We will use the properties of the function to solve for the unknown parameters. 20

Note that p(x) and q(x) appear as a ratio, they are determined only up to a multiplicative constant. We may take the leading coecient of q(x) to be unity. f (x) = p(x) ax2 + bx + c = 2 q(x) x + x +

f (x) has a second order zero at x = 0. This means that p(x) has a second order zero there and that = 0. f (x) = ax2 x2 + x +

We note that f (x) 2 as x . This determines the parameter a. lim f (x) = lim ax2 x x2 + x + 2ax = lim x 2x + 2a = lim x 2 =a 2x2 x2 + x +

f (x) =

Now we use the fact that f (x) is even to conclude that q(x) is even and thus = 0. f (x) = Finally, we use that f (1) = 1 to determine . f (x) = 2x2 x2 + 1 2x2 x2 +

21

Solution 1.8 Consider the polynomial p(x) = (x + 2)40 (x 1)30 (x )30 . It is of degree 100. Since the factors only vanish at x = 2, 1, , p(x) only vanishes there. Since factors are nonnegative, the polynomial is non-negative.

22

Chapter 2 Vectors
2.1
2.1.1

Vectors
Scalars and Vectors

A vector is a quantity having both a magnitude and a direction. Examples of vector quantities are velocity, force and position. One can represent a vector in n-dimensional space with an arrow whose initial point is at the origin, (Figure 2.1). The magnitude is the length of the vector. Typographically, variables representing vectors are often written in capital letters, bold face or with a vector over-line, A, a, a. The magnitude of a vector is denoted |a|. A scalar has only a magnitude. Examples of scalar quantities are mass, time and speed. Vector Algebra. Two vectors are equal if they have the same magnitude and direction. The negative of a vector, denoted a, is a vector of the same magnitude as a but in the opposite direction. We add two vectors a and b by placing the tail of b at the head of a and dening a + b to be the vector with tail at the origin and head at the head of b. (See Figure 2.2.) The dierence, a b, is dened as the sum of a and the negative of b, a + (b). The result of multiplying a by a scalar is a vector of magnitude || |a| with the same/opposite direction if is positive/negative. (See Figure 2.2.) 23

x
Figure 2.1: Graphical representation of a vector in three dimensions.

b a a+b -a

2a a

Figure 2.2: Vector arithmetic.

Here are the properties of adding vectors and multiplying them by a scalar. They are evident from geometric 24

considerations. a+b=b+a a = a commutative laws (a + b) + c = a + (b + c) (a) = ()a associative laws (a + b) = a + b ( + )a = a + a distributive laws Zero and Unit Vectors. The additive identity element for vectors is the zero vector or null vector. This is a vector of magnitude zero which is denoted as 0. A unit vector is a vector of magnitude one. If a is nonzero then a/|a| is a unit vector in the direction of a. Unit vectors are often denoted with a caret over-line, n. Rectangular Unit Vectors. In n dimensional Cartesian space, Rn , the unit vectors in the directions of the coordinates axes are e1 , . . . en . These are called the rectangular unit vectors. To cut down on subscripts, the unit vectors in three dimensional space are often denoted with i, j and k. (Figure 2.3).
z k j i x

Figure 2.3: Rectangular unit vectors.

25

Components of a Vector. Consider a vector a with tail at the origin and head having the Cartesian coordinates (a1 , . . . , an ). We can represent this vector as the sum of n rectangular component vectors, a = a1 e1 + + an en . (See Figure 2.4.) Another notation for the vector a is a1 , . . . , an . By the Pythagorean theorem, the magnitude of the vector a is |a| = a2 + + a2 . 1 n
z

a a3 k a1 i a2 j x y

Figure 2.4: Components of a vector.

2.1.2

The Kronecker Delta and Einstein Summation Convention


ij = 1 if i = j, 0 if i = j.

The Kronecker Delta tensor is dened

This notation will be useful in our work with vectors. Consider writing a vector in terms of its rectangular components. Instead of using ellipses: a = a1 e1 + + an en , we could write the expression as a sum: a = n ai ei . We can shorten this notation by leaving out the sum: a = ai ei , i=1 where it is understood that whenever an index is repeated in a term we sum over that index from 1 to n. This is the 26

Einstein summation convention. A repeated index is called a summation index or a dummy index. Other indices can take any value from 1 to n and are called free indices. Example 2.1.1 Consider the matrix equation: A x = b. We can write out the matrix and vectors explicitly. x1 b1 a11 a1n . . . = . .. . . . . . . . . . an1 ann xn bn This takes much less space when we use the summation convention. aij xj = bi Here j is a summation index and i is a free index.

2.1.3

The Dot and Cross Product


a b |a||b| cos ,

Dot Product. The dot product or scalar product of two vectors is dened, where is the angle from a to b. From this denition one can derive the following properties: a b = b a, commutative. (a b) = (a) b = a (b), associativity of scalar multiplication. a (b + c) = a b + a c, distributive. (See Exercise 2.1.) ei ej = ij . In three dimensions, this is i i = j j = k k = 1, i j = j k = k i = 0.

a b = ai bi a1 b1 + + an bn , dot product in terms of rectangular components. If a b = 0 then either a and b are orthogonal, (perpendicular), or one of a and b are zero. 27

The Angle Between Two Vectors. We can use the dot product to nd the angle between two vectors, a and b. From the denition of the dot product, a b = |a||b| cos . If the vectors are nonzero, then = arccos ab |a||b| .

Example 2.1.2 What is the angle between i and i + j? = arccos = arccos = . 4 i (i + j) |i||i + j| 1 2

Parametric Equation of a Line. Consider a line in Rn that passes through the point a and is parallel to the vector t, (tangent). A parametric equation of the line is x = a + ut, u R.

Implicit Equation of a Line In 2D. Consider a line in R2 that passes through the point a and is normal, (orthogonal, perpendicular), to the vector n. All the lines that are normal to n have the property that x n is a constant, where x is any point on the line. (See Figure 2.5.) x n = 0 is the line that is normal to n and passes through the origin. The line that is normal to n and passes through the point a is x n = a n. 28

x n=1 x n=0

x n= a n n a

x n=-1

Figure 2.5: Equation for a line. The normal to a line determines an orientation of the line. The normal points in the direction that is above the line. A point b is (above/on/below) the line if (b a) n is (positive/zero/negative). The signed distance of a point b from the line x n = a n is n (b a) . |n| Implicit Equation of a Hyperplane. A hyperplane in Rn is an n 1 dimensional sheet which passes through a given point and is normal to a given direction. In R3 we call this a plane. Consider a hyperplane that passes through the point a and is normal to the vector n. All the hyperplanes that are normal to n have the property that x n is a constant, where x is any point in the hyperplane. x n = 0 is the hyperplane that is normal to n and passes through the origin. The hyperplane that is normal to n and passes through the point a is x n = a n. 29

The normal determines an orientation of the hyperplane. The normal points in the direction that is above the hyperplane. A point b is (above/on/below) the hyperplane if (b a) n is (positive/zero/negative). The signed distance of a point b from the hyperplane x n = a n is n (b a) . |n| Right and Left-Handed Coordinate Systems. Consider a rectangular coordinate system in two dimensions. Angles are measured from the positive x axis in the direction of the positive y axis. There are two ways of labeling the axes. (See Figure 2.6.) In one the angle increases in the counterclockwise direction and in the other the angle increases in the clockwise direction. The former is the familiar Cartesian coordinate system.
y x

Figure 2.6: There are two ways of labeling the axes in two dimensions. There are also two ways of labeling the axes in a three-dimensional rectangular coordinate system. These are called right-handed and left-handed coordinate systems. See Figure 2.7. Any other labelling of the axes could be rotated into one of these congurations. The right-handed system is the one that is used by default. If you put your right thumb in the direction of the z axis in a right-handed coordinate system, then your ngers curl in the direction from the x axis to the y axis. Cross Product. The cross product or vector product is dened, a b = |a||b| sin n, where is the angle from a to b and n is a unit vector that is orthogonal to a and b and in the direction such that the ordered triple of vectors a, b and n form a right-handed system. 30

z k j i x y y j

z k i x

Figure 2.7: Right and left handed coordinate systems. You can visualize the direction of a b by applying the right hand rule. Curl the ngers of your right hand in the direction from a to b. Your thumb points in the direction of a b. Warning: Unless you are a lefty, get in the habit of putting down your pencil before applying the right hand rule. The dot and cross products behave a little dierently. First note that unlike the dot product, the cross product is not commutative. The magnitudes of a b and b a are the same, but their directions are opposite. (See Figure 2.8.) Let a b = |a||b| sin n and b a = |b||a| sin m. The angle from a to b is the same as the angle from b to a. Since {a, b, n} and {b, a, m} are right-handed systems, m points in the opposite direction as n. Since a b = b a we say that the cross product is anti-commutative. Next we note that since |a b| = |a||b| sin , the magnitude of a b is the area of the parallelogram dened by the two vectors. (See Figure 2.9.) The area of the triangle dened by two vectors is then 1 |a b|. 2 From the denition of the cross product, one can derive the following properties: 31

a b b

a b a
Figure 2.8: The cross product is anti-commutative.

b b sin a

b a

Figure 2.9: The parallelogram and the triangle dened by two vectors. a b = b a, anti-commutative. (a b) = (a) b = a (b), associativity of scalar multiplication. a (b + c) = a b + a c, distributive. (a b) c = a (b c). The cross product is not associative. i i = j j = k k = 0. 32

i j = k, j k = i, k i = j. i j k a b = (a2 b3 a3 b2 )i + (a3 b1 a1 b3 )j + (a1 b2 a2 b1 )k = a1 a2 a3 , b1 b2 b3 cross product in terms of rectangular components. If a b = 0 then either a and b are parallel or one of a or b is zero. Scalar Triple Product. Consider the volume of the parallelopiped dened by three vectors. (See Figure 2.10.) The area of the base is ||b||c| sin |, where is the angle between b and c. The height is |a| cos , where is the angle between b c and a. Thus the volume of the parallelopiped is |a||b||c| sin cos .

b c a b
Figure 2.10: The parallelopiped dened by three vectors. Note that |a (b c)| = |a (|b||c| sin n)| = ||a||b||c| sin cos | . 33

Thus |a (b c)| is the volume of the parallelopiped. a (b c) is the volume or the negative of the volume depending on whether {a, b, c} is a right or left-handed system. Note that parentheses are unnecessary in a b c. There is only one way to interpret the expression. If you did the dot product rst then you would be left with the cross product of a scalar and a vector which is meaningless. a b c is called the scalar triple product. Plane Dened by Three Points. Three points which are not collinear dene a plane. Consider a plane that passes through the three points a, b and c. One way of expressing that the point x lies in the plane is that the vectors x a, b a and c a are coplanar. (See Figure 2.11.) If the vectors are coplanar, then the parallelopiped dened by these three vectors will have zero volume. We can express this in an equation using the scalar triple product, (x a) (b a) (c a) = 0.

x c a b

Figure 2.11: Three points dene a plane.

2.2

Sets of Vectors in n Dimensions


x = (x1 , x2 , . . . , xn ), 34 y = (y1 , y2 , . . . , yn ).

Orthogonality. Consider two n-dimensional vectors

The inner product of these vectors can be dened


n

x|y x y =
i=1

xi yi .

The vectors are orthogonal if x y = 0. The norm of a vector is the length of the vector generalized to n dimensions. x = xx Consider a set of vectors {x1 , x2 , . . . , xm }. If each pair of vectors in the set is orthogonal, then the set is orthogonal. xi xj = 0 if i = j If in addition each vector in the set has norm 1, then the set is orthonormal. xi xj = ij = Here ij is known as the Kronecker delta function. Completeness. A set of n, n-dimensional vectors {x1 , x2 , . . . , xn } is complete if any n-dimensional vector can be written as a linear combination of the vectors in the set. That is, any vector y can be written
n

1 0

if i = j if i = j

y=
i=1

ci xi .

35

Taking the inner product of each side of this equation with xm ,


n

y xm =
i=1 n

ci xi

xm

=
i=1

ci xi xm

= cm xm xm y xm cm = xm 2 Thus y has the expansion


n

y=
i=1

y xi xi . xi 2

If in addition the set is orthonormal, then y=

(y xi )xi .
i=1

36

2.3

Exercises

The Dot and Cross Product


Exercise 2.1 Prove the distributive law for the dot product, a (b + c) = a b + a c. Hint, Solution Exercise 2.2 Prove that a b = ai bi a1 b1 + + an bn . Hint, Solution Exercise 2.3 What is the angle between the vectors i + j and i + 3j? Hint, Solution Exercise 2.4 Prove the distributive law for the cross product, a (b + c) = a b + a b. Hint, Solution Exercise 2.5 Show that i j k a b = a1 a2 a3 b1 b2 b3 Hint, Solution 37

Exercise 2.6 What is the area of the quadrilateral with vertices at (1, 1), (4, 2), (3, 7) and (2, 3)? Hint, Solution Exercise 2.7 What is the volume of the tetrahedron with vertices at (1, 1, 0), (3, 2, 1), (2, 4, 1) and (1, 2, 5)? Hint, Solution Exercise 2.8 What is the equation of the plane that passes through the points (1, 2, 3), (2, 3, 1) and (3, 1, 2)? What is the distance from the point (2, 3, 5) to the plane? Hint, Solution

38

2.4

Hints

The Dot and Cross Product


Hint 2.1 First prove the distributive law when the rst vector is of unit length, n (b + c) = n b + n c. Then all the quantities in the equation are projections onto the unit vector n and you can use geometry. Hint 2.2 First prove that the dot product of a rectangular unit vector with itself is one and the dot product of two distinct rectangular unit vectors is zero. Then write a and b in rectangular components and use the distributive law. Hint 2.3 Use a b = |a||b| cos . Hint 2.4 First consider the case that both b and c are orthogonal to a. Prove the distributive law in this case from geometric considerations. Next consider two arbitrary vectors a and b. We can write b = b + b where b is orthogonal to a and b is parallel to a. Show that a b = a b . Finally prove the distributive law for arbitrary b and c. Hint 2.5 Write the vectors in their rectangular components and use, i j = k, and, i i = j j = k k = 0. 39 j k = i, k i = j,

Hint 2.6 1 The quadrilateral is composed of two triangles. The area of a triangle dened by the two vectors a and b is 2 |a b|. Hint 2.7 Justify that the area of a tetrahedron determined by three vectors is one sixth the area of the parallelogram determined by those three vectors. The area of a parallelogram determined by three vectors is the magnitude of the scalar triple product of the vectors: a b c. Hint 2.8 The equation of a line that is orthogonal to a and passes through the point b is a x = a b. The distance of a point c from the plane is a (c b) |a|

40

2.5

Solutions

The Dot and Cross Product


Solution 2.1 First we prove the distributive law when the rst vector is of unit length, i.e., n (b + c) = n b + n c. (2.1)

From Figure 2.12 we see that the projection of the vector b + c onto n is equal to the sum of the projections b n and c n. Now we extend the result to the case when the rst vector has arbitrary length. We dene a = |a|n and multiply Equation 2.1 by the scalar, |a|. |a|n (b + c) = |a|n b + |a|n c a (b + c) = a b + a c. Solution 2.2 First note that ei ei = |ei ||ei | cos(0) = 1. Then note that that dot product of any two distinct rectangular unit vectors is zero because they are orthogonal. Now we write a and b in terms of their rectangular components and use the distributive law. a b = ai e i b j e j = ai b j e i e j = ai bj ij = ai b i Solution 2.3 Since a b = |a||b| cos , we have = arccos ab |a||b|

41

c b b+c nc nb n (b+c) n

Figure 2.12: The distributive law for the dot product. when a and b are nonzero. = arccos (i + j) (i + 3j) |i + j||i + 3j| = arccos 4 2 10 = arccos 2 5 5 0.463648

Solution 2.4 First consider the case that both b and c are orthogonal to a. b + c is the diagonal of the parallelogram dened by b and c, (see Figure 2.13). Since a is orthogonal to each of these vectors, taking the cross product of a with these vectors has the eect of rotating the vectors through /2 radians about a and multiplying their length by |a|. Note 42

that a (b + c) is the diagonal of the parallelogram dened by a b and a c. Thus we see that the distributive law holds when a is orthogonal to both b and c, a (b + c) = a b + a c.

a c

b+c

a b a (b+c)

Figure 2.13: The distributive law for the cross product. Now consider two arbitrary vectors a and b. We can write b = b + b where b is orthogonal to a and b is parallel to a, (see Figure 2.14). By the denition of the cross product, a b = |a||b| sin n. Note that |b | = |b| sin , 43

a b b
Figure 2.14: The vector b written as a sum of components orthogonal and parallel to a. and that a b is a vector in the same direction as a b. Thus we see that a b = |a||b| sin n = |a|(sin |b|)n = |a||b |n a b = a b . Now we are prepared to prove the distributive law for arbitrary b and c. a (b + c) = a (b + b + c + c ) = a ((b + c) + (b + c) ) = a ((b + c) ) = a b + a c =ab+ac a (b + c) = a b + a c

= |a||b | sin(/2)n

44

Solution 2.5 We know that i j = k, and that i i = j j = k k = 0. Now we write a and b in terms of their rectangular components and use the distributive law to expand the cross product. a b = (a1 i + a2 j + a3 k) (b1 i + b2 j + b3 k) = a1 i (b1 i + b2 j + b3 k) + a2 j (b1 i + b2 j + b3 k) + a3 k (b1 i + b2 j + b3 k) = a1 b2 k + a1 b3 (j) + a2 b1 (k) + a2 b3 i + a3 b1 j + a3 b2 (i) = (a2 b3 a3 b2 )i (a1 b3 a3 b1 )j + (a1 b2 a2 b1 )k Next we evaluate the determinant. i j k a a a a a a a1 a2 a3 = i 2 3 j 1 3 + k 1 2 b2 b3 b1 b3 b1 b2 b1 b2 b3 = (a2 b3 a3 b2 )i (a1 b3 a3 b1 )j + (a1 b2 a2 b1 )k Thus we see that, i j k a b = a1 a2 a3 b1 b2 b3 Solution 2.6 The area area of the quadrilateral is the area of two triangles. The rst triangle is dened by the vector from (1, 1) to (4, 2) and the vector from (1, 1) to (2, 3). The second triangle is dened by the vector from (3, 7) to (4, 2) and the vector from (3, 7) to (2, 3). (See Figure 2.15.) The area of a triangle dened by the two vectors a and b is 1 |a b|. 2 45 j k = i, k i = j,

The area of the quadrilateral is then, 1 1 1 1 |(3i + j) (i + 2j)| + |(i 5j) (i 4j)| = (5) + (19) = 12. 2 2 2 2
y

(3,7)

(2,3) (4,2) (1,1) x

Figure 2.15: Quadrilateral. Solution 2.7 The tetrahedron is determined by the three vectors with tail at (1, 1, 0) and heads at (3, 2, 1), (2, 4, 1) and (1, 2, 5). These are 2, 1, 1 , 1, 3, 1 and 0, 1, 5 . The area of the tetrahedron is one sixth the area of the parallelogram determined by these vectors. (This is because the area of a pyramid is 1 (base)(height). The base of the tetrahedron is 3 half the area of the parallelogram and the heights are the same. 1 1 = 1 ) Thus the area of a tetrahedron determined 23 6 by three vectors is 1 |a b c|. The area of the tetrahedron is 6 1 1 7 | 2, 1, 1 1, 3, 1 1, 2, 5 | = | 2, 1, 1 13, 4, 1 | = 6 6 2

46

Solution 2.8 The two vectors with tails at (1, 2, 3) and heads at (2, 3, 1) and (3, 1, 2) are parallel to the plane. Taking the cross product of these two vectors gives us a vector that is orthogonal to the plane. 1, 1, 2 2, 1, 1 = 3, 3, 3 We see that the plane is orthogonal to the vector 1, 1, 1 and passes through the point (1, 2, 3). The equation of the plane is 1, 1, 1 x, y, z = 1, 1, 1 1, 2, 3 , x + y + z = 6. Consider the vector with tail at (1, 2, 3) and head at (2, 3, 5). The magnitude of the dot product of this vector with the unit normal vector gives the distance from the plane. 1, 1, 1 4 4 3 1, 1, 2 = = | 1, 1, 1 | 3 3

47

Part II Calculus

48

Chapter 3 Dierential Calculus


3.1 Limits of Functions

Denition of a Limit. If the value of the function y(x) gets arbitrarily close to as x approaches the point , then we say that the limit of the function as x approaches is equal to . This is written: lim y(x) =

Now we make the notion of arbitrarily close precise. For any > 0 there exists a > 0 such that |y(x) | < for all 0 < |x | < . That is, there is an interval surrounding the point x = for which the function is within of . See Figure 3.1. Note that the interval surrounding x = is a deleted neighborhood, that is it does not contain the point x = . Thus the value of the function at x = need not be equal to for the limit to exist. Indeed the function need not even be dened at x = . To prove that a function has a limit at a point we rst bound |y(x) | in terms of for values of x satisfying 0 < |x | < . Denote this upper bound by u(). Then for an arbitrary > 0, we determine a > 0 such that the the upper bound u() and hence |y(x) | is less than . 49

y + + x

Figure 3.1: The neighborhood of x = such that |y(x) | < . Example 3.1.1 Show that
x1

lim x2 = 1. for all |x 1| < . First we

Consider any > 0. We need to show that there exists a > 0 such that |x2 1| < obtain a bound on |x2 1|. |x2 1| = |(x 1)(x + 1)| = |x 1||x + 1| < |x + 1| = |(x 1) + 2| < ( + 2) Now we choose a positive such that, ( + 2) = . We see that = 1 + 1,

is positive and satises the criterion that |x2 1| < for all 0 < |x 1| < . Thus the limit exists. 50

Example 3.1.2 Recall that the value of the function y() need not be equal to limx y(x) for the limit to exist. We show an example of this. Consider the function y(x) = 1 for x Z, 0 for x Z.

For what values of does limx y(x) exist? First consider Z. Then there exists an open neighborhood a < < b around such that y(x) is identically zero for x (a, b). Then trivially, limx y(x) = 0. Now consider Z. Consider any > 0. Then if |x | < 1 then |y(x) 0| = 0 < . Thus we see that limx y(x) = 0. Thus, regardless of the value of , limx y(x) = 0.

Left and Right Limits. With the notation limx+ y(x) we denote the right limit of y(x). This is the limit as x approaches from above. Mathematically: limx+ exists if for any > 0 there exists a > 0 such that |y(x) | < for all 0 < x < . The left limit limx y(x) is dened analogously. Example 3.1.3 Consider the function, approaches zero.
sin x , |x|

dened for x = 0. (See Figure 3.2.) The left and right limits exist as x sin x = 1, |x| lim lim sin x = 1 |x|

x0

lim +

x0

However the limit, sin x , x0 |x| does not exist.

Properties of Limits. Let lim f (x) and lim g(x) exist.


x x

51

Figure 3.2: Plot of sin(x)/|x|. lim (af (x) + bg(x)) = a lim f (x) + b lim g(x).
x x x

lim (f (x)g(x)) =
x

lim f (x)

lim g(x) .

lim

f (x) g(x)

limx f (x) if lim g(x) = 0. limx g(x) x

Example 3.1.4 We prove that if limx f (x) = and limx g(x) = exist then lim (f (x)g(x)) = lim f (x) lim g(x) .

Since the limit exists for f (x), we know that for all > 0 there exists > 0 such that |f (x) | < for |x | < . Likewise for g(x). We seek to show that for all > 0 there exists > 0 such that |f (x)g(x) | < for |x | < . We proceed by writing |f (x)g(x) |, in terms of |f (x) | and |g(x) |, which we know how to bound. |f (x)g(x) | = |f (x)(g(x) ) + (f (x) )| |f (x)||g(x) | + |f (x) ||| If we choose a such that |f (x)||g(x) | < /2 and |f (x) ||| < /2 then we will have the desired result: |f (x)g(x) | < . Trying to ensure that |f (x)||g(x) | < /2 is hard because of the |f (x)| factor. We will replace 52

that factor with a constant. We want to write |f (x) ||| < /2 as |f (x) | < /(2||), but this is problematic for the case = 0. We x these two problems and then proceed. We choose 1 such that |f (x) | < 1 for |x | < 1 . This gives us the desired form.

|f (x)g(x) | (|| + 1)|g(x) | + |f (x) |(|| + 1), for |x | < 1

Next we choose 2 such that |g(x)| < /(2(||+1)) for |x| < 2 and choose 3 such that |f (x)| < /(2(||+1)) for |x | < 3 . Let be the minimum of 1 , 2 and 3 .

|f (x)g(x) | (|| + 1)|g(x) | + |f (x) |(|| + 1) < |f (x)g(x) | < , for |x | <

+ , for |x | < 2

We conclude that the limit of a product is the product of the limits.

lim (f (x)g(x)) =

lim f (x)

lim g(x)

= .

53

Result 3.1.1 Denition of a Limit. The statement:


x

lim y(x) =

means that y(x) gets arbitrarily close to as x approaches . For any > 0 there exists a > 0 such that |y(x) | < for all x in the neighborhood 0 < |x | < . The left and right limits, lim y(x) = and lim+ y(x) =
x x

denote the limiting value as x approaches respectively from below and above. The neighborhoods are respectively < x < 0 and 0 < x < . Properties of Limits. Let lim u(x) and lim v(x) exist.
x x

lim (au(x) + bv(x)) = a lim u(x) + b lim v(x).


x x x

lim (u(x)v(x)) =
x

lim u(x)

lim v(x) .

lim

u(x) v(x)

limx u(x) if lim v(x) = 0. limx v(x) x

3.2

Continuous Functions

Denition of Continuity. A function y(x) is said to be continuous at x = if the value of the function is equal to its limit, that is, limx y(x) = y(). Note that this one condition is actually the three conditions: y() is 54

dened, limx y(x) exists and limx y(x) = y(). A function is continuous if it is continuous at each point in its domain. A function is continuous on the closed interval [a, b] if the function is continuous for each point x (a, b) and limxa+ y(x) = y(a) and limxb y(x) = y(b). Discontinuous Functions. If a function is not continuous at a point it is called discontinuous at that point. If limx y(x) exists but is not equal to y(), then the function has a removable discontinuity. It is thus named because we could dene a continuous function z(x) = y(x) for x = , limx y(x) for x = ,

to remove the discontinuity. If both the left and right limit of a function at a point exist, but are not equal, then the function has a jump discontinuity at that point. If either the left or right limit of a function does not exist, then the function is said to have an innite discontinuity at that point. Example 3.2.1
sin x x

has a removable discontinuity at x = 0. 0 H(x) = 1/2 1


1 x

The Heaviside function, for x < 0, for x = 0, for x > 0,

has a jump discontinuity at x = 0.

has an innite discontinuity at x = 0. See Figure 3.3.

Properties of Continuous Functions. Arithmetic. If u(x) and v(x) are continuous at x = then u(x) v(x) and u(x)v(x) are continuous at x = . is continuous at x = if v() = 0.
u(x) v(x)

Function Composition. If u(x) is continuous at x = and v(x) is continuous at x = = u() then u(v(x)) is continuous at x = . The composition of continuous functions is a continuous function. 55

Figure 3.3: A Removable discontinuity, a Jump Discontinuity and an Innite Discontinuity Boundedness. A function which is continuous on a closed interval is bounded in that closed interval. Nonzero in a Neighborhood. If y() = 0 then there exists a neighborhood ( , + ), that y(x) = 0 for x ( , + ). > 0 of the point such

Intermediate Value Theorem. Let u(x) be continuous on [a, b]. If u(a) u(b) then there exists [a, b] such that u() = . This is known as the intermediate value theorem. A corollary of this is that if u(a) and u(b) are of opposite sign then u(x) has at least one zero on the interval (a, b). Maxima and Minima. If u(x) is continuous on [a, b] then u(x) has a maximum and a minimum on [a, b]. That is, there is at least one point [a, b] such that u() u(x) for all x [a, b] and there is at least one point [a, b] such that u() u(x) for all x [a, b]. Piecewise Continuous Functions. A function is piecewise continuous on an interval if the function is bounded on the interval and the interval can be divided into a nite number of intervals on each of which the function is continuous. For example, the greatest integer function, x , is piecewise continuous. ( x is dened to the the greatest integer less than or equal to x.) See Figure 3.4 for graphs of two piecewise continuous functions. Uniform Continuity. Consider a function f (x) that is continuous on an interval. This means that for any point in the interval and any positive there exists a > 0 such that |f (x) f ()| < for all 0 < |x | < . In general, this value of depends on both and . If can be chosen so it is a function of alone and independent of then 56

Figure 3.4: Piecewise Continuous Functions the function is said to be uniformly continuous on the interval. A sucient condition for uniform continuity is that the function is continuous on a closed interval.

3.3

The Derivative

Consider a function y(x) on the interval (x . . . x + x) for some x > 0. We dene the increment y = y(x + x) y y(x). The average rate of change, (average velocity), of the function on the interval is x . The average rate of change is the slope of the secant line that passes through the points (x, y(x)) and (x + x, y(x + x)). See Figure 3.5.
y

y x x

Figure 3.5: The increments x and y. 57

If the slope of the secant line has a limit as x approaches zero then we call this slope the derivative or instantaneous dy rate of change of the function at the point x. We denote the derivative by dx , which is a nice notation as the derivative y is the limit of x as x 0. dy y(x + x) y(x) lim . dx x0 x
dy d x may approach zero from below or above. It is common to denote the derivative dx by dx y, y (x), y or Dy. A function is said to be dierentiable at a point if the derivative exists there. Note that dierentiability implies continuity, but not vice versa.

Example 3.3.1 Consider the derivative of y(x) = x2 at the point x = 1. y (1) lim y(1 + x) y(1) x0 x (1 + x)2 1 = lim x0 x = lim (2 + x)
x0

=2 Figure 3.6 shows the secant lines approaching the tangent line as x approaches zero from above and below. Example 3.3.2 We can compute the derivative of y(x) = x2 at an arbitrary point x. d (x + x)2 x2 x2 = lim x0 dx x = lim (2x + x)
x0

= 2x

58

4 3.5 3 2.5 2 1.5 1 0.5 0.5 1 1.5 2

4 3.5 3 2.5 2 1.5 1 0.5 0.5 1 1.5 2

Figure 3.6: Secant lines and the tangent to x2 at x = 1. Properties. Let u(x) and v(x) be dierentiable. Let a and b be derivatives are: d du dv (au + bv) = a +b dx dx dx du dv d (uv) = v+u dx dx dx du v u dv d u = dx 2 dx dx v v d a du (u ) = aua1 dx dx du dv d (u(v(x))) = = u (v(x))v (x) dx dv dx These can be proved by using the denition of dierentiation. 59 constants. Some fundamental properties of Linearity Product Rule Quotient Rule Power Rule Chain Rule

Example 3.3.3 Prove the quotient rule for derivatives.

d u = lim x0 dx v

u(x+x) v(x+x)

u(x) v(x)

x u(x + x)v(x) u(x)v(x + x) = lim x0 xv(x)v(x + x) u(x + x)v(x) u(x)v(x) u(x)v(x + x) + u(x)v(x) = lim x0 xv(x)v(x) (u(x + x) u(x))v(x) u(x)(v(x + x) v(x)) = lim x0 xv 2 (x) = = limx0
u(x+x)u(x) v(x) x

u(x) limx0 v 2 (x)

v(x+x)v(x) x

dv v du u dx dx v2

60

Trigonometric Functions. Some derivatives of trigonometric functions are: d sin x = cos x dx d cos x = sin x dx d 1 tan x = dx cos2 x d x e = ex dx d sinh x = cosh x dx d cosh x = sinh x dx d 1 tanh x = dx cosh2 x d 1 arcsin x = dx (1 x2 )1/2 d 1 arccos x = dx (1 x2 )1/2 d 1 arctan x = dx 1 + x2 d 1 ln x = dx x d 1 arcsinh x = 2 dx (x + 1)1/2 d 1 arccosh x = 2 dx (x 1)1/2 d 1 arctanh x = dx 1 x2

Example 3.3.4 We can evaluate the derivative of xx by using the identity ab = eb ln a . d x d x ln x e x = dx dx d = ex ln x (x ln x) dx 1 = xx (1 ln x + x ) x x = x (1 + ln x)

61

Inverse Functions. If we have a function y(x), we can consider x as a function of y, x(y). For example, if y(x) = 8x3 then x(y) = 3 y/2; if y(x) = x+2 then x(y) = 2y . The derivative of an inverse function is x+1 y1 d 1 x(y) = dy . dy dx Example 3.3.5 The inverse function of y(x) = ex is x(y) = ln y. We can obtain the derivative of the logarithm from the derivative of the exponential. The derivative of the exponential is dy = ex . dx Thus the derivative of the logarithm is 1 1 d d 1 ln y = x(y) = dy = x = . e y dy dy dx

3.4

Implicit Dierentiation

An explicitly dened function has the form y = f (x). A implicitly dened function has the form f (x, y) = 0. A few examples of implicit functions are x2 + y 2 1 = 0 and x + y + sin(xy) = 0. Often it is not possible to write an implicit equation in explicit form. This is true of the latter example above. One can calculate the derivative of y(x) in terms of x and y even when y(x) is dened by an implicit equation. Example 3.4.1 Consider the implicit equation x2 xy y 2 = 1. This implicit equation can be solved for the dependent variable. y(x) = 1 x 5x2 4 . 2 62

We can dierentiate this expression to obtain y = 1 2 1 5x 5x2 4 .

One can obtain the same result without rst solving for y. If we dierentiate the implicit equation, we obtain 2x y x We can solve this equation for
dy . dx

dy dy 2y = 0. dx dx

dy 2x y = dx x + 2y

We can dierentiate this expression to obtain the second derivative of y. d2 y (x + 2y)(2 y ) (2x y)(1 + 2y ) = dx2 (x + 2y)2 5(y xy ) = (x + 2y)2 Substitute in the expression for y . = Use the original implicit equation. = 10 (x + 2y)2 10(x2 xy y 2 ) (x + 2y)2

63

3.5

Maxima and Minima

A dierentiable function is increasing where f (x) > 0, decreasing where f (x) < 0 and stationary where f (x) = 0. A function f (x) has a relative maxima at a point x = if there exists a neighborhood around such that f (x) f () for x (x , x + ), > 0. The relative minima is dened analogously. Note that this denition does not require that the function be dierentiable, or even continuous. We refer to relative maxima and minima collectively are relative extrema. Relative Extrema and Stationary Points. If f (x) is dierentiable and f () is a relative extrema then x = is a stationary point, f () = 0. We can prove this using left and right limits. Assume that f () is a relative maxima. Then there is a neighborhood (x , x + ), > 0 for which f (x) f (). Since f (x) is dierentiable the derivative at x = , f ( + x) f () f () = lim , x0 x exists. This in turn means that the left and right limits exist and are equal. Since f (x) f () for < x < the left limit is non-positive, f ( + x) f () f () = lim 0. x0 x Since f (x) f () for < x < + the right limit is nonnegative, f () = lim +
x0

f ( + x) f () 0. x

Thus we have 0 f () 0 which implies that f () = 0. It is not true that all stationary points are relative extrema. That is, f () = 0 does not imply that x = is an extrema. Consider the function f (x) = x3 . x = 0 is a stationary point since f (x) = x2 , f (0) = 0. However, x = 0 is neither a relative maxima nor a relative minima. It is also not true that all relative extrema are stationary points. Consider the function f (x) = |x|. The point x = 0 is a relative minima, but the derivative at that point is undened. 64

First Derivative Test. Let f (x) be dierentiable and f () = 0. If f (x) changes sign from positive to negative as we pass through x = then the point is a relative maxima. If f (x) changes sign from negative to positive as we pass through x = then the point is a relative minima. If f (x) is not identically zero in a neighborhood of x = and it does not change sign as we pass through the point then x = is not a relative extrema. Example 3.5.1 Consider y = x2 and the point x = 0. The function is dierentiable. The derivative, y = 2x, vanishes at x = 0. Since y (x) is negative for x < 0 and positive for x > 0, the point x = 0 is a relative minima. See Figure 3.7. Example 3.5.2 Consider y = cos x and the point x = 0. The function is dierentiable. The derivative, y = sin x is positive for < x < 0 and negative for 0 < x < . Since the sign of y goes from positive to negative, x = 0 is a relative maxima. See Figure 3.7. Example 3.5.3 Consider y = x3 and the point x = 0. The function is dierentiable. The derivative, y = 3x2 is positive for x < 0 and positive for 0 < x. Since y is not identically zero and the sign of y does not change, x = 0 is not a relative extrema. See Figure 3.7.

Figure 3.7: Graphs of x2 , cos x and x3 .

65

Concavity. If the portion of a curve in some neighborhood of a point lies above the tangent line through that point, the curve is said to be concave upward. If it lies below the tangent it is concave downward. If a function is twice dierentiable then f (x) > 0 where it is concave upward and f (x) < 0 where it is concave downward. Note that f (x) > 0 is a sucient, but not a necessary condition for a curve to be concave upward at a point. A curve may be concave upward at a point where the second derivative vanishes. A point where the curve changes concavity is called a point of inection. At such a point the second derivative vanishes, f (x) = 0. For twice continuously dierentiable functions, f (x) = 0 is a necessary but not a sucient condition for an inection point. The second derivative may vanish at places which are not inection points. See Figure 3.8.

Figure 3.8: Concave Upward, Concave Downward and an Inection Point.

Second Derivative Test. Let f (x) be twice dierentiable and let x = be a stationary point, f () = 0. If f () < 0 then the point is a relative maxima. If f () > 0 then the point is a relative minima. If f () = 0 then the test fails. Example 3.5.4 Consider the function f (x) = cos x and the point x = 0. The derivatives of the function are f (x) = sin x, f (x) = cos x. The point x = 0 is a stationary point, f (0) = sin(0) = 0. Since the second derivative is negative there, f (0) = cos(0) = 1, the point is a relative maxima.

66

Example 3.5.5 Consider the function f (x) = x4 and the point x = 0. The derivatives of the function are f (x) = 4x3 , f (x) = 12x2 . The point x = 0 is a stationary point. Since the second derivative also vanishes at that point the second derivative test fails. One must use the rst derivative test to determine that x = 0 is a relative minima.

3.6

Mean Value Theorems

Rolles Theorem. If f (x) is continuous in [a, b], dierentiable in (a, b) and f (a) = f (b) = 0 then there exists a point (a, b) such that f () = 0. See Figure 3.9.

Figure 3.9: Rolles Theorem. To prove this we consider two cases. First we have the trivial case that f (x) 0. If f (x) is not identically zero then continuity implies that it must have a nonzero relative maxima or minima in (a, b). Let x = be one of these relative extrema. Since f (x) is dierentiable, x = must be a stationary point, f () = 0. Theorem of the Mean. If f (x) is continuous in [a, b] and dierentiable in (a, b) then there exists a point x = such that f (b) f (a) f () = . ba That is, there is a point where the instantaneous velocity is equal to the average velocity on the interval. 67

Figure 3.10: Theorem of the Mean. We prove this theorem by applying Rolles theorem. Consider the new function g(x) = f (x) f (a) f (b) f (a) (x a) ba

Note that g(a) = g(b) = 0, so it satises the conditions of Rolles theorem. There is a point x = such that g () = 0. We dierentiate the expression for g(x) and substitute in x = to obtain the result. g (x) = f (x) g () = f () f (b) f (a) ba

f (b) f (a) =0 ba f (b) f (a) f () = ba

Generalized Theorem of the Mean. If f (x) and g(x) are continuous in [a, b] and dierentiable in (a, b), then there exists a point x = such that f () f (b) f (a) = . g () g(b) g(a) We have assumed that g(a) = g(b) so that the denominator does not vanish and that f (x) and g (x) are not simultaneously zero which would produce an indeterminate form. Note that this theorem reduces to the regular theorem of the mean when g(x) = x. The proof of the theorem is similar to that for the theorem of the mean. 68

Taylors Theorem of the Mean. If f (x) is n + 1 times continuously dierentiable in (a, b) then there exists a point x = (a, b) such that (b a)2 (b a)n (n) (b a)n+1 (n+1) f (b) = f (a) + (b a)f (a) + f (a) + + f (a) + f (). 2! n! (n + 1)! For the case n = 0, the formula is f (b) = f (a) + (b a)f (), which is just a rearrangement of the terms in the theorem of the mean, f () = f (b) f (a) . ba (3.1)

3.6.1

Application: Using Taylors Theorem to Approximate Functions.

One can use Taylors theorem to approximate functions with polynomials. Consider an innitely dierentiable function f (x) and a point x = a. Substituting x for b into Equation 3.1 we obtain, f (x) = f (a) + (x a)f (a) + (x a)2 (x a)n (n) (x a)n+1 (n+1) f (a) + + f (a) + f (). 2! n! (n + 1)!

If the last term in the sum is small then we can approximate our function with an nth order polynomial. f (x) f (a) + (x a)f (a) + (x a)2 (x a)n (n) f (a) + + f (a) 2! n!

The last term in Equation 3.6.1 is called the remainder or the error term, Rn = (x a)n+1 (n+1) f (). (n + 1)! 69

Since the function is innitely dierentiable, f (n+1) () exists and is bounded. Therefore we note that the error must vanish as x 0 because of the (x a)n+1 factor. We therefore suspect that our approximation would be a good one if x is close to a. Also note that n! eventually grows faster than (x a)n , (x a)n = 0. n n! lim So if the derivative term, f (n+1) (), does not grow to quickly, the error for a certain value of x will get smaller with increasing n and the polynomial will become a better approximation of the function. (It is also possible that the derivative factor grows very quickly and the approximation gets worse with increasing n.) Example 3.6.1 Consider the function f (x) = ex . We want a polynomial approximation of this function near the point x = 0. Since the derivative of ex is ex , the value of all the derivatives at x = 0 is f (n) (0) = e0 = 1. Taylors theorem thus states that x2 x3 xn xn+1 ex = 1 + x + e, + + + + 2! 3! n! (n + 1)! for some (0, x). The rst few polynomial approximations of the exponent about the point x = 0 are f1 (x) = 1 f2 (x) = 1 + x x2 2 x2 x3 f4 (x) = 1 + x + + 2 6 f3 (x) = 1 + x + The four approximations are graphed in Figure 3.11. Note that for the range of x we are looking at, the approximations become more accurate as the number of terms increases.

70

2.5 2 1.5 1 0.5 -1 -0.5

0.5 1

2.5 2 1.5 1 0.5 -1 -0.5

0.5 1

2.5 2 1.5 1 0.5 -1 -0.5

0.5 1

2.5 2 1.5 1 0.5 -1 -0.5

0.5 1

Figure 3.11: Four Finite Taylor Series Approximations of ex Example 3.6.2 Consider the function f (x) = cos x. We want a polynomial approximation of this function near the point x = 0. The rst few derivatives of f are f (x) = cos x f (x) = sin x f (x) = cos x f (x) = sin x f (4) (x) = cos x Its easy to pick out the pattern here, f (n) (x) = (1)n/2 cos x for even n, (n+1)/2 (1) sin x for odd n.

Since cos(0) = 1 and sin(0) = 0 the n-term approximation of the cosine is, x2 x4 x6 x2(n1) x2n 2(n1) cos x = 1 + + + (1) + cos . 2! 4! 6! (2(n 1))! (2n)! Here are graphs of the one, two, three and four term approximations. 71

1 0.5 -3 -2 -1 -0.5 -1 1 2 3

1 0.5 -3 -2 -1 -0.5 -1 1 2 3

1 0.5 -3 -2 -1 -0.5 -1 1 2 3

1 0.5 -3 -2 -1 -0.5 -1 1 2 3

Figure 3.12: Taylor Series Approximations of cos x

Note that for the range of x we are looking at, the approximations become more accurate as the number of terms increases. Consider the ten term approximation of the cosine about x = 0, x2 x4 x18 x20 + + cos . 2! 4! 18! 20!

cos x = 1

Note that for any value of , | cos | 1. Therefore the absolute value of the error term satises, x20 |x|20 cos . 20! 20!

|R| =

x20 /20! is plotted in Figure 3.13. Note that the error is very small for x < 6, fairly small but non-negligible for x 7 and large for x > 8. The ten term approximation of the cosine, plotted below, behaves just we would predict. The error is very small until it becomes non-negligible at x 7 and large at x 8.

Example 3.6.3 Consider the function f (x) = ln x. We want a polynomial approximation of this function near the 72

1 0.8 0.6 0.4 0.2

10

Figure 3.13: Plot of x20 /20!.


1 0.5

-10

-5 -0.5 -1 -1.5 -2

10

Figure 3.14: Ten Term Taylor Series Approximation of cos x point x = 1. The rst few derivatives of f are f (x) = ln x 1 f (x) = x 1 f (x) = 2 x 2 f (x) = 3 x 3 f (4) (x) = 4 x 73

The derivatives evaluated at x = 1 are f (0) = 0, By Taylors theorem of the mean we have, ln x = (x 1) (x 1)n (x 1)n+1 1 (x 1)2 (x 1)3 (x 1)4 + + + (1)n1 + (1)n . 2 3 4 n n + 1 n+1 f (n) (0) = (1)n1 (n 1)!, for n 1.

Below are plots of the 2, 4, 10 and 50 term approximations.


2 1 -1 0.5 1 1.5 2 2.5 3 -2 -3 -4 -5 -6 2 1 -1 0.5 1 1.5 2 2.5 3 -2 -3 -4 -5 -6 2 1 -1 0.5 1 1.5 2 2.5 3 -2 -3 -4 -5 -6 2 1 -1 0.5 1 1.5 2 2.5 3 -2 -3 -4 -5 -6

Figure 3.15: The 2, 4, 10 and 50 Term Approximations of ln x Note that the approximation gets better on the interval (0, 2) and worse outside this interval as the number of terms increases. The Taylor series converges to ln x only on this interval.

3.6.2

Application: Finite Dierence Schemes

Example 3.6.4 Suppose you sample a function at the discrete points nx, n Z. In Figure 3.16 we sample the function f (x) = sin x on the interval [4, 4] with x = 1/4 and plot the data points. We wish to approximate the derivative of the function on the grid points using only the value of the function on those discrete points. From the denition of the derivative, one is lead to the formula f (x) f (x + x) f (x) . x 74 (3.2)

0.5

-4

-2

-0.5

-1

Figure 3.16: Sampling of sin x Taylors theorem states that f (x + x) = f (x) + xf (x) + x2 f (). 2

Substituting this expression into our formula for approximating the derivative we obtain f (x) + xf (x) + x f () f (x) f (x + x) f (x) x 2 = = f (x) + f (). x x 2 Thus we see that the error in our approximation of the rst derivative is x f (). Since the error has a linear factor 2 of x, we call this a rst order accurate method. Equation 3.2 is called the forward dierence scheme for calculating the rst derivative. Figure 3.17 shows a plot of the value of this scheme for the function f (x) = sin x and x = 1/4. The rst derivative of the function f (x) = cos x is shown for comparison. Another scheme for approximating the rst derivative is the centered dierence scheme, f (x) f (x + x) f (x x) . 2x 75
2

0.5

-4

-2

-0.5

-1

Figure 3.17: The Forward Dierence Scheme Approximation of the Derivative Expanding the numerator using Taylors theorem, f (x + x) f (x x) 2x f (x) + xf (x) + = = f (x) +

x2 f 2

(x) +

x3 f 6

() f (x) + xf (x) 2x

x2 f 2

(x) +

x3 f 6

()

x2 (f () + f ()). 12

The error in the approximation is quadratic in x. Therefore this is a second order accurate scheme. Below is a plot of the derivative of the function and the value of this scheme for the function f (x) = sin x and x = 1/4. Notice how the centered dierence scheme gives a better approximation of the derivative than the forward dierence scheme.

3.7

LHospitals Rule

Some singularities are easy to diagnose. Consider the function cos x at the point x = 0. The function evaluates x 1 to 0 and is thus discontinuous at that point. Since the numerator and denominator are continuous functions and the 76

0.5

-4

-2

-0.5

-1

Figure 3.18: Centered Dierence Scheme Approximation of the Derivative denominator vanishes while the numerator does not, the left and right limits as x 0 do not exist. Thus the function has an innite discontinuity at the point x = 0. More generally, a function which is composed of continuous functions and evaluates to a at a point where a = 0 must have an innite discontinuity there. 0
sin x Other singularities require more analysis to diagnose. Consider the functions sin x , sin x and 1cos x at the point x = 0. x |x| All three functions evaluate to 0 at that point, but have dierent kinds of singularities. The rst has a removable 0 discontinuity, the second has a nite discontinuity and the third has an innite discontinuity. See Figure 3.19.

Figure 3.19: The functions

sin x sin x , |x| x

and

sin x . 1cos x

77

An expression that evaluates to 0 , , 0 , , 1 , 00 or 0 is called an indeterminate. A function f (x) which 0 is indeterminate at the point x = is singular at that point. The singularity may be a removable discontinuity, a nite discontinuity or an innite discontinuity depending on the behavior of the function around that point. If limx f (x) exists, then the function has a removable discontinuity. If the limit does not exist, but the left and right limits do exist, then the function has a nite discontinuity. If either the left or right limit does not exist then the function has an innite discontinuity.

LHospitals Rule. Let f (x) and g(x) be dierentiable and f () = g() = 0. Further, let g(x) be nonzero in a deleted neighborhood of x = , (g(x) = 0 for x 0 < |x | < ). Then f (x) f (x) = lim . x g(x) x g (x) lim To prove this, we note that f () = g() = 0 and apply the generalized theorem of the mean. Note that f (x) f (x) f () f () = = g(x) g(x) g() g () for some between and x. Thus f (x) f () f (x) = lim = lim x g(x) g () x g (x) lim provided that the limits exist. LHospitals Rule is also applicable when both functions tend to innity instead of zero or when the limit point, , is at innity. It is also valid for one-sided limits. 0 LHospitals rule is directly applicable to the indeterminate forms 0 and . Example 3.7.1 Consider the three functions
sin x sin x , |x| x

and

sin x 1cos x

at the point x = 0.

sin x cos x = lim =1 x0 x x0 1 lim 78

Thus

sin x x

has a removable discontinuity at x = 0. lim + sin x sin x = lim =1 x0+ x |x|

x0

x0

lim

sin x sin x = lim = 1 x0 x |x|

Thus

sin x |x|

has a nite discontinuity at x = 0. cos x 1 sin x = lim = = x0 1 cos x x0 sin x 0 lim

Thus

sin x 1cos x

has an innite discontinuity at x = 0.

Example 3.7.2 Let a and d be nonzero. ax2 + bx + c 2ax + b = lim 2 + ex + f x dx x 2dx + e 2a = lim x 2d a = d lim Example 3.7.3 Consider

cos x 1 . x0 x sin x 0 This limit is an indeterminate of the form 0 . Applying LHospitals rule we see that limit is equal to lim sin x . x0 x cos x + sin x lim 79

This limit is again an indeterminate of the form 0 . We apply LHospitals rule again. 0 cos x 1 = x sin x + 2 cos x 2 1 Thus the value of the original limit is 2 . We could also obtain this result by expanding the functions in Taylor series.
x0

lim

1 x + x 1 2 24 cos x 1 lim = lim x5 x3 x0 x sin x x0 x x + 120 6 = =


x4 24 lim 2 x4 + x6 x0 x 6 120 1 x2 2 + 24 lim 2 4 x0 1 x + x 6 120

x + 2

1 2 We can apply LHospitals Rule to the indeterminate forms 0 and by rewriting the expression in a dierent form, (perhaps putting the expression over a common denominator). If at rst you dont succeed, try, try again. You may have to apply LHospitals rule several times to evaluate a limit. = Example 3.7.4
x0

lim cot x

1 x

x cos x sin x x sin x cos x x sin x cos x = lim x0 sin x + x cos x x sin x = lim x0 sin x + x cos x x cos x sin x = lim x0 cos x + cos x x sin x =0 = lim
x0

80

You can apply LHospitals rule to the indeterminate forms 1 , 00 or 0 by taking the logarithm of the expression. Example 3.7.5 Consider the limit,
x0

lim xx ,

which gives us the indeterminate form 00 . The logarithm of the expression is ln(xx ) = x ln x. As x 0 we now have the indeterminate form 0 . By rewriting the expression, we can apply LHospitals rule. ln x 1/x = lim x0 1/x x0 1/x2 = lim (x) lim
x0

=0 Thus the original limit is


x0

lim xx = e0 = 1.

81

3.8
3.8.1

Exercises
Limits of Functions
1 x

Exercise 3.1 Does


x0

lim sin

exist? Hint, Solution Exercise 3.2 Does


x0

lim x sin

1 x

exist? Hint, Solution

3.8.2

Continuous Functions

Exercise 3.3 Is the function sin(1/x) continuous in the open interval (0, 1)? Is there a value of a such that the function dened by f (x) = is continuous on the closed interval [0, 1]? Hint, Solution Exercise 3.4 Is the function sin(1/x) uniformly continuous in the open interval (0, 1)? Hint, Solution 82 sin(1/x) for x = 0, a for x = 0

Exercise 3.5 Are the functions x and Hint, Solution

1 x

uniformly continuous on the interval (0, 1)?

Exercise 3.6 Prove that a function which is continuous on a closed interval is uniformly continuous on that interval. Hint, Solution Exercise 3.7 Prove or disprove each of the following. 1. If limn an = L then limn a2 = L2 . n 2. If limn a2 = L2 then limn an = L. n 3. If an > 0 for all n > 200, and limn an = L, then L > 0. 4. If f : R R is continuous and limx f (x) = L, then for n Z, limn f (n) = L. 5. If f : R R is continuous and limn f (n) = L, then for x R, limx f (x) = L. Hint, Solution

3.8.3

The Derivative

Exercise 3.8 (mathematica/calculus/dierential/denition.nb) Use the denition of dierentiation to prove the following identities where f (x) and g(x) are dierentiable functions and n is a positive integer. 1. 2. 3.
d (xn ) dx

= nxn1 ,

(I suggest that you use Newtons binomial formula.)

d (f (x)g(x)) dx d (sin x) dx

dg = f dx + g df dx

= cos x. (Youll need to use some trig identities.) 83

4.

d (f (g(x))) dx

= f (g(x))g (x)

Hint, Solution Exercise 3.9 Use the denition of dierentiation to determine if the following functions dierentiable at x = 0. 1. f (x) = x|x| 2. f (x) = Hint, Solution Exercise 3.10 (mathematica/calculus/dierential/rules.nb) Find the rst derivatives of the following: a. x sin(cos x) b. f (cos(g(x))) c.
1 f (ln x)
x

1 + |x|

d. xx

e. |x| sin |x| Hint, Solution Exercise 3.11 (mathematica/calculus/dierential/rules.nb) Using d d 1 sin x = cos x and tan x = dx dx cos2 x nd the derivatives of arcsin x and arctan x. Hint, Solution 84

3.8.4

Implicit Dierentiation

Exercise 3.12 (mathematica/calculus/dierential/implicit.nb) Find y (x), given that x2 + y 2 = 1. What is y (1/2)? Hint, Solution Exercise 3.13 (mathematica/calculus/dierential/implicit.nb) Find y (x) and y (x), given that x2 xy + y 2 = 3. Hint, Solution

3.8.5

Maxima and Minima

Exercise 3.14 (mathematica/calculus/dierential/maxima.nb) Identify any maxima and minima of the following functions. a. f (x) = x(12 2x)2 . b. f (x) = (x 2)2/3 . Hint, Solution Exercise 3.15 (mathematica/calculus/dierential/maxima.nb) A cylindrical container with a circular base and an open top is to hold 64 cm3 . Find its dimensions so that the surface area of the cup is a minimum. Hint, Solution

3.8.6

Mean Value Theorems

Exercise 3.16 Prove the generalized theorem of the mean. If f (x) and g(x) are continuous in [a, b] and dierentiable in (a, b), then there exists a point x = such that f () f (b) f (a) = . g () g(b) g(a) 85

Assume that g(a) = g(b) so that the denominator does not vanish and that f (x) and g (x) are not simultaneously zero which would produce an indeterminate form. Hint, Solution Exercise 3.17 (mathematica/calculus/dierential/taylor.nb) Find a polynomial approximation of sin x on the interval [1, 1] that has a maximum error of terms that you need to. Prove the error bound. Use your polynomial to approximate sin 1. Hint, Solution
1 . 1000

Dont use any more

Exercise 3.18 (mathematica/calculus/dierential/taylor.nb) You use the formula f (x+x)2f (x)+f (xx) to approximate f (x). What is the error in this approximation? x2 Hint, Solution Exercise 3.19 The formulas f (x+x)f (x) and f (x+x)f (xx) are rst and second order accurate schemes for approximating the rst x 2x derivative f (x). Find a couple other schemes that have successively higher orders of accuracy. Would these higher order schemes actually give a better approximation of f (x)? Remember that x is small, but not innitesimal. Hint, Solution

3.8.7

LHospitals Rule

Exercise 3.20 (mathematica/calculus/dierential/lhospitals.nb) Evaluate the following limits. a. limx0


xsin x x3 1 x

b. limx0 csc x c. limx+ 1 +

1 x x

1 d. limx0 csc2 x x2 . (First evaluate using LHospitals rule then using a Taylor series expansion. You will nd that the latter method is more convenient.)

86

Hint, Solution Exercise 3.21 (mathematica/calculus/dierential/lhospitals.nb) Evaluate the following limits, a lim 1 + lim xa/x , x x x where a and b are constants. Hint, Solution

bx

87

3.9

Hints

Hint 3.1 Apply the , denition of a limit. Hint 3.2 Set y = 1/x. Consider limy . Hint 3.3 The composition of continuous functions is continuous. Apply the denition of continuity and look at the point x = 0. Hint 3.4 Note that for x1 =
1 (n1/2)

and x2 =

1 (n+1/2)

where n Z we have | sin(1/x1 ) sin(1/x2 )| = 2.

Hint 3.5 Note that the function x + x is a decreasing function of x and an increasing function of for positive x and . Bound this function for xed . Consider any positive and . For what values of x is 1 1 > . x x+ Hint 3.6 Let the function f (x) be continuous on a closed interval. Consider the function e(x, ) = sup |f () f (x)|.
|x|<

Bound e(x, ) with a function of alone. Hint 3.7 CONTINUE 1. If limn an = L then limn a2 = L2 . n 88

2. If limn a2 = L2 then limn an = L. n 3. If an > 0 for all n > 200, and limn an = L, then L > 0. 4. If f : R R is continuous and limx f (x) = L, then for n Z, limn f (n) = L. 5. If f : R R is continuous and limn f (n) = L, then for x R, limx f (x) = L. Hint 3.8 a. Newtons binomial formula is
n

(a + b) =
k=0

n nk k n(n 1) n2 2 a b = an + an1 b + a b + + nabn1 + bn . k 2 n k n! . (n k)!k!

Recall that the binomial coecient is

b. Note that

d f (x + x)g(x + x) f (x)g(x) (f (x)g(x)) = lim x0 dx x f (x + x) f (x) g(x + x) g(x) + f (x) lim . x0 x x

and g(x)f (x) + f (x)g (x) = g(x) lim Fill in the blank. c. First prove that
x0

lim

sin = 1.

and
0

lim

cos 1 = 0. 89

d. Let u = g(x). Consider a nonzero increment x, which induces the increments u and f . By denition, f = f (u + u) f (u), and f, u 0 as x 0. If u = 0 then we have = f df 0 as u 0. u du u = g(x + x) g(x),

If u = 0 for some values of x then f also vanishes and we dene = 0 for theses values. In either case, y = Continue from here. Hint 3.9 df u + u. du

Hint 3.10 a. Use the product rule and the chain rule. b. Use the chain rule. c. Use the quotient rule and the chain rule. d. Use the identity ab = eb ln a . e. For x > 0, the expression is x sin x; for x < 0, the expression is (x) sin(x) = x sin x. Do both cases. Hint 3.11 Use that x (y) = 1/y (x) and the identities cos x = (1 sin2 x)1/2 and cos(arctan x) =
1 . (1+x2 )1/2

90

Hint 3.12 Dierentiating the equation x2 + [y(x)]2 = 1 yields 2x + 2y(x)y (x) = 0. Solve this equation for y (x) and write y(x) in terms of x. Hint 3.13 Dierentiate the equation and solve for y (x) in terms of x and y(x). Dierentiate the expression for y (x) to obtain y (x). Youll use that x2 xy(x) + [y(x)]2 = 3 Hint 3.14 a. Use the second derivative test. b. The function is not dierentiable at the point x = 2 so you cant use a derivative test at that point. Hint 3.15 Let r be the radius and h the height of the cylinder. The volume of the cup is r2 h = 64. The radius and height are 64 related by h = r2 . The surface area of the cup is f (r) = r2 + 2rh = r2 + 128 . Use the second derivative test to r nd the minimum of f (r). Hint 3.16 The proof is analogous to the proof of the theorem of the mean. Hint 3.17 The rst few terms in the Taylor series of sin(x) about x = 0 are sin(x) = x x3 x5 x7 x9 + + + . 6 120 5040 362880

When determining the error, use the fact that | cos x0 | 1 and |xn | 1 for x [1, 1]. 91

Hint 3.18 The terms in the approximation have the Taylor series, x2 x3 x4 f (x) + f (x) + f (x1 ), 2 6 24 x2 x3 x4 f (x x) = f (x) xf (x) + f (x) f (x) + f (x2 ), 2 6 24 f (x + x) = f (x) + xf (x) + where x x1 x + x and x x x2 x. Hint 3.19 Hint 3.20 a. Apply LHospitals rule three times. b. You can write the expression as x sin x . x sin x

c. Find the limit of the logarithm of the expression. d. It takes four successive applications of LHospitals rule to evaluate the limit. For the Taylor series expansion method, csc2 x x2 sin2 x 1 x2 (x x3 /6 + O(x5 ))2 = 2 2 = x2 x2 (x + O(x3 ))2 x sin x

Hint 3.21 To evaluate the limits use the identity ab = eb ln a and then apply LHospitals rule.

92

3.10

Solutions

Solution 3.1 Note that in any open neighborhood of zero, (, ), the function sin(1/x) takes on all values in the interval [1, 1]. Thus if we choose a positive such that < 1 then there is no value of for which | sin(1/x) | < for all x ( , ). Thus the limit does not exist. Solution 3.2 We make the change of variables y = 1/x and consider y . We use that sin(y) is bounded. lim x sin 1 x = lim 1 sin(y) = 0 y

x0

Solution 3.3 1 Since x is continuous in the interval (0, 1) and the function sin(x) is continuous everywhere, the composition sin(1/x) is continuous in the interval (0, 1). Since limx0 sin(1/x) does not exist, there is no way of dening sin(1/x) at x = 0 to produce a function that is continuous in [0, 1]. Solution 3.4 1 1 Note that for x1 = (n1/2) and x2 = (n+1/2) where n Z we have | sin(1/x1 ) sin(1/x2 )| = 2. Thus for any 0 < < 2 there is no value of > 0 such that | sin(1/x1 ) sin(1/x2 )| < for all x1 , x2 (0, 1) and |x1 x2 | < . Thus sin(1/x) is not uniformly continuous in the open interval (0, 1). Solution 3.5 First consider the function x. Note that the function x + x is a decreasing function of x and an increasing function of for positive x and . Thus for any xed , the maximum value of x + x is bounded by . Therefore on the interval (0, 1), a sucient condition for | x | < is |x | < 2 . The function x is uniformly continuous on the interval (0, 1). Consider any positive and . Note that 1 1 > x x+ 93

for x< Thus there is no value of such that 1 2 2 + 4 .

1 1 < x

for all |x | < . The function

1 x

is not uniformly continuous on the interval (0, 1).

Solution 3.6 Let the function f (x) be continuous on a closed interval. Consider the function e(x, ) = sup |f () f (x)|.
|x|<

Since f (x) is continuous, e(x, ) is a continuous function of x on the same closed interval. Since continuous functions on closed intervals are bounded, there is a continuous, increasing function () satisfying, e(x, ) (), for all x in the closed interval. Since () is continuous and increasing, it has an inverse ( ). Now note that |f (x) f ()| < for all x and in the closed interval satisfying |x | < ( ). Thus the function is uniformly continuous in the closed interval. Solution 3.7 1. The statement
n

lim an = L

is equivalent to > 0, N s.t. n > N |an L| < . We want to show that > 0, M s.t. m > M |a2 L2 | < . n 94

Suppose that |an L| < . We obtain an upper bound on |a2 L2 |. n |a2 L2 | = |an L||an + L| < (|2L| + ) n Now we choose a value of such that |a2 L2 | < n (|2L| + ) = = L2 + |L| Consider any xed > 0. We see that since for = L2 + |L|, N s.t. n > N |an L| < implies that n > N |a2 L2 | < . n Therefore > 0, M s.t. m > M |a2 L2 | < . n We conclude that limn a2 = L2 . n 2. limn a2 = L2 does not imply that limn an = L. Consider an = 1. In this case limn a2 = 1 and n n limn an = 1. 3. If an > 0 for all n > 200, and limn an = L, then L is not necessarily positive. Consider an = 1/n, which satises the two constraints. 1 lim = 0 n n 4. The statement limx f (x) = L is equivalent to > 0, X s.t. x > X |f (x) L| < . This implies that for n > X , |f (n) L| < . > 0, N s.t. n > N |f (n) L| < lim f (n) = L
n

95

5. If f : R R is continuous and limn f (n) = L, then for x R, it is not necessarily true that limx f (x) = L. Consider f (x) = sin(x). lim sin(n) = lim 0 = 0
n n

limx sin(x) does not exist. Solution 3.8 a. d n (x + x)n xn (x ) = lim x0 dx x xn + nxn1 x + = lim
x0

n(n1) n2 x x2 2

+ + xn xn

= lim

x0

nxn1 +

n(n 1) n2 x x + + xn1 2

= nxn1 d n (x ) = nxn1 dx b. d f (x + x)g(x + x) f (x)g(x) (f (x)g(x)) = lim x0 dx x [f (x + x)g(x + x) f (x)g(x + x)] + [f (x)g(x + x) f (x)g(x)] = lim x0 x f (x + x) f (x) g(x + x) g(x) + f (x) lim = lim [g(x + x)] lim x0 x0 x0 x x = g(x)f (x) + f (x)g (x) 96

d (f (x)g(x)) = f (x)g (x) + f (x)g(x) dx c. Consider a right triangle with hypotenuse of length 1 in the rst quadrant of the plane. Label the vertices A, B, C, in clockwise order, starting with the vertex at the origin. The angle of A is . The length of a circular arc of radius cos that connects C to the hypotenuse is cos . The length of the side BC is sin . The length of a circular arc of radius 1 that connects B to the x axis is . (See Figure 3.20.)
B

cos sin

Figure 3.20: Considering the length of these three curves gives us the inequality: cos sin . Dividing by , cos 97 sin 1.

Taking the limit as 0 gives us sin = 1. 0 lim One more little tidbit well need to know is
0

lim

cos 1 cos 1 cos + 1 = lim 0 cos + 1 2 cos 1 = lim 0 (cos + 1) sin2 = lim 0 (cos + 1) sin sin = lim lim 0 (cos + 1) 0 0 = (1) 2 = 0.

Now were ready to nd the derivative of sin x. d sin(x + x) sin x (sin x) = lim x0 dx x cos x sin x + sin x cos x sin x = lim x0 x sin x cos x 1 = cos x lim + sin x lim x0 x0 x x = cos x d (sin x) = cos x dx 98

d. Let u = g(x). Consider a nonzero increment x, which induces the increments u and f . By denition, f = f (u + u) f (u), and f, u 0 as x 0. If u = 0 then we have = f df 0 as u 0. u du u = g(x + x) g(x),

If u = 0 for some values of x then f also vanishes and we dene = 0 for theses values. In either case, y = df u + u. du

We divide this equation by x and take the limit as x 0. df f = lim dx x0 x df u u = lim + x0 du x x df f lim + = x0 x du du df du + (0) = du dx dx df du = du dx Thus we see that d (f (g(x))) = f (g(x))g (x). dx

x0

lim

u x0 x lim

99

Solution 3.9 1. f (0) = lim 0 | |0

= lim 0| | =0 The function is dierentiable at x = 0. 2. f (0) = lim 0


1

1+| |1 (1 + | |)1/2 sign( ) 1

= lim 0 2

1 = lim 0 sign( ) 2 Since the limit does not exist, the function is not dierentiable at x = 0. Solution 3.10 a. d d d [x sin(cos x)] = [x] sin(cos x) + x [sin(cos x)] dx dx dx d = sin(cos x) + x cos(cos x) [cos x] dx = sin(cos x) x cos(cos x) sin x d [x sin(cos x)] = sin(cos x) x cos(cos x) sin x dx 100

b. d d [f (cos(g(x)))] = f (cos(g(x))) [cos(g(x))] dx dx d = f (cos(g(x))) sin(g(x)) [g(x)] dx = f (cos(g(x))) sin(g(x))g (x)

d [f (cos(g(x)))] = f (cos(g(x))) sin(g(x))g (x) dx c.


d [f (ln x)] 1 d = dx dx f (ln x) [f (ln x)]2 d f (ln x) dx [ln x] = [f (ln x)]2 f (ln x) = x[f (ln x)]2

d 1 f (ln x) = dx f (ln x) x[f (ln x)]2 d. First we write the expression in terms exponentials and logarithms, xx = xexp(x ln x) = exp(exp(x ln x) ln x). 101
x

Then we dierentiate using the chain rule and the product rule. d d exp(exp(x ln x) ln x) = exp(exp(x ln x) ln x) (exp(x ln x) ln x) dx dx 1 d x = xx exp(x ln x) (x ln x) ln x + exp(x ln x) dx x 1 x = xx xx (ln x + x ) ln x + x1 exp(x ln x) x xx x = x x (ln x + 1) ln x + x1 xx = xx
x +x

x1 + ln x + ln2 x

d xx x x = xx +x x1 + ln x + ln2 x dx

e. For x > 0, the expression is x sin x; for x < 0, the expression is (x) sin(x) = x sin x. Thus we see that |x| sin |x| = x sin x. The rst derivative of this is sin x + x cos x. d (|x| sin |x|) = sin x + x cos x dx

102

Solution 3.11 Let y(x) = sin x. Then y (x) = cos x. d 1 arcsin y = dy y (x) 1 = cos x 1 (1 sin2 x)1/2 1 = (1 y 2 )1/2 = d 1 arcsin x = dx (1 x2 )1/2 Let y(x) = tan x. Then y (x) = 1/ cos2 x. d 1 arctan y = dy y (x) = cos2 x = cos2 (arctan y) 1 = (1 + y 2 )1/2 1 = 1 + y2 d 1 arctan x = dx 1 + x2

103

Solution 3.12 Dierentiating the equation x2 + [y(x)]2 = 1 yields 2x + 2y(x)y (x) = 0. We can solve this equation for y (x). y (x) = To nd y (1/2) we need to nd y(x) in terms of x. y(x) = 1 x2 Thus y (x) is y (x) = y (1/2) can have the two values: y Solution 3.13 Dierentiating the equation x2 xy(x) + [y(x)]2 = 3 yields 2x y(x) xy (x) + 2y(x)y (x) = 0. Solving this equation for y (x) y (x) = y(x) 2x . 2y(x) x 104 1 2 1 = . 3 x . 1 x2 x y(x)

Now we dierentiate y (x) to get y (x). y (x) = (y (x) 2)(2y(x) x) (y(x) 2x)(2y (x) 1) , (2y(x) x)2 y (x) = 3 xy (x) y(x) , (2y(x) x)2 ,

y (x) = 3 y (x) = 3

x y(x)2x y(x) 2y(x)x (2y(x) x)2

x(y(x) 2x) y(x)(2y(x) x) , (2y(x) x)3 x2 xy(x) + [y(x)]2 , (2y(x) x)3 18 , (2y(x) x)3

y (x) = 6

y (x) = Solution 3.14 a.

f (x) = (12 2x)2 + 2x(12 2x)(2) = 4(x 6)2 + 8x(x 6) = 12(x 2)(x 6) There are critical points at x = 2 and x = 6. f (x) = 12(x 2) + 12(x 6) = 24(x 4) Since f (2) = 48 < 0, x = 2 is a local maximum. Since f (6) = 48 > 0, x = 6 is a local minimum. 105

b.

2 f (x) = (x 2)1/3 3 The rst derivative exists and is nonzero for x = 2. At x = 2, the derivative does not exist and thus x = 2 is a critical point. For x < 2, f (x) < 0 and for x > 2, f (x) > 0. x = 2 is a local minimum.

Solution 3.15 Let r be the radius and h the height of the cylinder. The volume of the cup is r2 h = 64. The radius and height are 64 related by h = r2 . The surface area of the cup is f (r) = r2 + 2rh = r2 + 128 . The rst derivative of the surface r area is f (r) = 2r 128 . Finding the zeros of f (r), 2 r 2r 128 = 0, r2

2r3 128 = 0, 4 r= . 3
4 The second derivative of the surface area is f (r) = 2 + 256 . Since f ( ) = 6, r = 3 r3 f (r). Since this is the only critical point for r > 0, it must be a global minimum. 4 4 The cup has a radius of cm and a height of . 3 3 4 3

is a local minimum of

Solution 3.16 We dene the function h(x) = f (x) f (a) f (b) f (a) (g(x) g(a)). g(b) g(a)

Note that h(x) is dierentiable and that h(a) = h(b) = 0. Thus h(x) satises the conditions of Rolles theorem and there exists a point (a, b) such that h () = f () f (b) f (a) g () = 0, g(b) g(a) 106

f () f (b) f (a) = . g () g(b) g(a) Solution 3.17 The rst few terms in the Taylor series of sin(x) about x = 0 are sin(x) = x x3 x5 x7 x9 + + + . 6 120 5040 362880 x5 cos x0 7 x3 + x, 6 120 5040

The seventh derivative of sin x is cos x. Thus we have that sin(x) = x

where 0 x0 x. Since we are considering x [1, 1] and 1 cos(x0 ) 1, the approximation sin x x has a maximum error of
1 5040

x3 x5 + 6 120

0.000198. Using this polynomial to approximate sin(1), 1 13 15 + 0.841667. 6 120 sin(1) 0.841471.

To see that this has the required accuracy, Solution 3.18 Expanding the terms in the approximation in Taylor series, x2 x3 x4 f (x + x) = f (x) + xf (x) + f (x) + f (x) + f (x1 ), 2 6 24 x2 x3 x4 f (x x) = f (x) xf (x) + f (x) f (x) + f (x2 ), 2 6 24 107

where x x1 x + x and x x x2 x. Substituting the expansions into the formula, f (x + x) 2f (x) + f (x x) x2 = f (x) + [f (x1 ) + f (x2 )]. x2 24 Thus the error in the approximation is x2 [f (x1 ) + f (x2 )]. 24 Solution 3.19

Solution 3.20 a. x sin x 1 cos x = lim x0 x3 3x2 sin x = lim x0 6x cos x = lim x0 6 1 = 6

x0

lim

x0

lim

x sin x 1 = x3 6

108

b.

x0

lim csc x

1 x

= lim = lim

x0

x0

= lim = lim 0 2 =0 =

x0

x0

1 1 sin x x x sin x x sin x 1 cos x x cos x + sin x sin x x sin x + cos x + cos x

x0

lim csc x

1 x

=0

109

c.

ln

x+

lim

1 1+ x

= lim = lim = lim = lim = lim =1

x+

x+

1 ln 1+ x 1 x ln 1 + x ln 1 + 1/x 1+
1 x

x+

1 1 x

1 x2

x+

1/x2 1+ 1 x
1

x+

Thus we have

x+

lim

1+

1 x

= e.

110

d. It takes four successive applications of LHospitals rule to evaluate the limit. lim csc2 x 1 x2 x2 sin2 x x0 x2 sin2 x 2x 2 cos x sin x = lim 2 x0 2x cos x sin x + 2x sin2 x 2 2 cos2 x + 2 sin2 x = lim 2 x0 2x cos2 x + 8x cos x sin x + 2 sin2 x 2x2 sin2 x 8 cos x sin x = lim x0 12x cos2 x + 12 cos x sin x 8x2 cos x sin x 12x sin2 x 8 cos2 x 8 sin2 x = lim x0 24 cos2 x 8x2 cos2 x 64x cos x sin x 24 sin2 x + 8x2 sin2 x 1 = 3 = lim

x0

It is easier to use a Taylor series expansion. lim csc2 x 1 x2 x2 sin2 x x0 x2 sin2 x x2 (x x3 /6 + O(x5 ))2 = lim x0 x2 (x + O(x3 ))2 x2 (x2 x4 /3 + O(x6 )) = lim x0 x4 + O(x6 ) 1 = lim + O(x2 ) x0 3 1 = 3 = lim

x0

111

Solution 3.21 To evaluate the rst limit, we use the identity ab = eb ln a and then apply LHospitals rule.
x

lim xa/x = lim e


x

a ln x x

a ln x x x a/x = exp lim x 1 = e0 = exp lim lim xa/x = 1

We use the same method to evaluate the second limit.


x

lim 1 +

a x

bx

a x x a = exp lim bx ln 1 + x x ln(1 + a/x) = exp lim b x 1/x 2 = lim exp bx ln 1 + = exp lim b
x a/x 1+a/x 1/x2

= exp a x

x bx

lim b

a 1 + a/x

lim 1 +

= eab

112

3.11

Quiz

Problem 3.1 Dene continuity. Solution Problem 3.2 Fill in the blank with necessary, sucient or necessary and sucient. Continuity is a condition for dierentiability. Dierentiability is a condition for continuity. f (x+x)f (x) Existence of limx0 is a condition for dierentiability. x Solution Problem 3.3 d Evaluate dx f (g(x)h(x)). Solution Problem 3.4 d Evaluate dx f (x)g(x) . Solution Problem 3.5 State the Theorem of the Mean. Interpret the theorem physically. Solution Problem 3.6 State Taylors Theorem of the Mean. Solution Problem 3.7 Evaluate limx0 (sin x)sin x . Solution

113

3.12

Quiz Solutions

Solution 3.1 A function y(x) is said to be continuous at x = if limx y(x) = y(). Solution 3.2 Continuity is a necessary condition for dierentiability. Dierentiability is a sucient condition for continuity. Existence of limx0 f (x+x)f (x) is a necessary and sucient condition for dierentiability. x Solution 3.3 d d f (g(x)h(x)) = f (g(x)h(x)) (g(x)h(x)) = f (g(x)h(x))(g (x)h(x) + g(x)h (x)) dx dx Solution 3.4 d d g(x) ln f (x) f (x)g(x) = e dx dx d = eg(x) ln f (x) (g(x) ln f (x)) dx = f (x)g(x) g (x) ln f (x) + g(x) f (x) f (x)

Solution 3.5 If f (x) is continuous in [a..b] and dierentiable in (a..b) then there exists a point x = such that f () = f (b) f (a) . ba

That is, there is a point where the instantaneous velocity is equal to the average velocity on the interval. 114

Solution 3.6 If f (x) is n + 1 times continuously dierentiable in (a..b) then there exists a point x = (a..b) such that f (b) = f (a) + (b a)f (a) + (b a)2 (b a)n (n) (b a)n+1 (n+1) f (a) + + f (a) + f (). 2! n! (n + 1)!

Solution 3.7 Consider limx0 (sin x)sin x . This is an indeterminate of the form 00 . The limit of the logarithm of the expression is limx0 sin x ln(sin x). This is an indeterminate of the form 0 . We can rearrange the expression to obtain an indeterminate of the form and then apply LHospitals rule. ln(sin x) cos x/ sin x = lim = lim ( sin x) = 0 x0 1/ sin x x0 cos x/ sin2 x x0 lim The original limit is
x0

lim (sin x)sin x = e0 = 1.

115

Chapter 4 Integral Calculus


4.1 The Indenite Integral

The opposite of a derivative is the anti-derivative or the indenite integral. The indenite integral of a function f (x) is denoted, f (x) dx. It is dened by the property that d dx f (x) dx = f (x).

While a function f (x) has a unique derivative if it is dierentiable, it has an innite number of indenite integrals, each of which dier by an additive constant. Zero Slope Implies a Constant Function. If the value of a functions derivative is identically zero, df = 0, dx then the function is a constant, f (x) = c. To prove this, we assume that there exists a non-constant dierentiable function whose derivative is zero and obtain a contradiction. Let f (x) be such a function. Since f (x) is non-constant, there exist points a and b such that f (a) = f (b). By the Mean Value Theorem of dierential calculus, there exists a 116

point (a, b) such that f (b) f (a) = 0, ba which contradicts that the derivative is everywhere zero. f () = Indenite Integrals Dier by an Additive Constant. Suppose that F (x) and G(x) are indenite integrals of f (x). Then we have d (F (x) G(x)) = F (x) G (x) = f (x) f (x) = 0. dx Thus we see that F (x) G(x) = c and the two indenite integrals must dier by a constant. For example, we have sin x dx = cos x + c. While every function that can be expressed in terms of elementary functions, (the exponent, logarithm, trigonometric functions, etc.), has a derivative that can be written explicitly in terms of elementary functions, the same is not true of integrals. For example, sin(sin x) dx cannot be written explicitly in terms of elementary functions. Properties. Since the derivative is linear, so is the indenite integral. That is, (af (x) + bg(x)) dx = a f (x) dx + b g(x) dx.
d (f (x))a dx

For each derivative identity there is a corresponding integral identity. Consider the power law identity, a(f (x))a1 f (x). The corresponding integral identity is (f (x))a f (x) dx = (f (x))a+1 + c, a+1 a = 1,
d dx

where we require that a = 1 to avoid division by zero. From the derivative of a logarithm, obtain, f (x) dx = ln |f (x)| + c. f (x) 117

ln(f (x)) =

f (x) , f (x)

we

Figure 4.1: Plot of ln |x| and 1/x. Note the absolute value signs. This is because this. Example 4.1.1 Consider I= (x2 x dx. + 1)2
d dx

ln |x| =

1 x

for x = 0. In Figure 4.1 is a plot of ln |x| and

1 x

to reinforce

We evaluate the integral by choosing u = x2 + 1, du = 2x dx. I= 2x 1 dx 2 + 1)2 2 (x 1 du = 2 u2 1 1 = 2 u 1 = . 2 + 1) 2(x

Example 4.1.2 Consider I= tan x dx = 118 sin x dx. cos x

By choosing f (x) = cos x, f (x) = sin x, we see that the integral is I= sin x dx = ln | cos x| + c. cos x

Change of Variable. The dierential of a function g(x) is dg = g (x) dx. Thus one might suspect that for = g(x), f () d = f (g(x))g (x) dx, (4.1)

since d = dg = g (x) dx. This turns out to be true. To prove it we will appeal to the the chain rule for dierentiation. Let be a function of x. The chain rule is d f () = f () (x), dx d df d f () = . dx d dx We can also write this as df dx df = , d d dx or in operator notation, d dx d = . d d dx Now were ready to start. The derivative of the left side of Equation 4.1 is d d f () d = f ().

119

Next we dierentiate the right side, d d f (g(x))g (x) dx = dx d f (g(x))g (x) dx d dx dx = f (g(x))g (x) d dx dg = f (g(x)) dg dx = f (g(x)) = f ()

to see that it is in fact an identity for = g(x). Example 4.1.3 Consider x sin(x2 ) dx. We choose = x2 , d = 2xdx to evaluate the integral. x sin(x2 ) dx = 1 sin(x2 )2x dx 2 1 = sin d 2 1 = ( cos ) + c 2 1 = cos(x2 ) + c 2

120

Integration by Parts. The product rule for dierentiation gives us an identity called integration by parts. We start with the product rule and then integrate both sides of the equation. d (u(x)v(x)) = u (x)v(x) + u(x)v (x) dx (u (x)v(x) + u(x)v (x)) dx = u(x)v(x) + c u (x)v(x) dx + u(x)v (x)) dx = u(x)v(x) v(x)u (x) dx

u(x)v (x)) dx = u(x)v(x) The theorem is most often written in the form u dv = uv

v du.

So what is the usefulness of this? Well, it may happen for some integrals and a good choice of u and v that the integral on the right is easier to evaluate than the integral on the left. Example 4.1.4 Consider x ex dx. If we choose u = x, dv = ex dx then integration by parts yields x ex dx = x ex ex dx = (x 1) ex .

Now notice what happens when we choose u = ex , dv = x dx. 1 x ex dx = x2 ex 2 The integral gets harder instead of easier. When applying integration by parts, one must choose u and dv wisely. As general rules of thumb: 121 1 2 x x e dx 2

Pick u so that u is simpler than u. Pick dv so that v is not more complicated, (hopefully simpler), than dv. Also note that you may have to apply integration by parts several times to evaluate some integrals.

4.2
4.2.1

The Denite Integral


Denition

The area bounded by the x axis, the vertical lines x = a and x = b and the function f (x) is denoted with a denite integral,
b

f (x) dx.
a

The area is signed, that is, if f (x) is negative, then the area is negative. We measure the area with a divide-and-conquer strategy. First partition the interval (a, b) with a = x0 < x1 < < xn1 < xn = b. Note that the area under the curve on the subinterval is approximately the area of a rectangle of base xi = xi+1 xi and height f (i ), where i [xi , xi+1 ]. If we add up the areas of the rectangles, we get an approximation of the area under the curve. See Figure 4.2
b n1

f (x) dx
a i=0

f (i )xi

As the xi s get smaller, we expect the approximation of the area to get better. Let x = max0in1 xi . We dene the denite integral as the sum of the areas of the rectangles in the limit that x 0.
b n1

f (x) dx = lim
a

x0

f (i )xi
i=0

The integral is dened when the limit exists. This is known as the Riemann integral of f (x). f (x) is called the integrand. 122

f(1 )

a x1 x2 x3

xi

x n-2 x n-1 b

Figure 4.2: Divide-and-Conquer Strategy for Approximating a Denite Integral.

4.2.2

Properties

Linearity and the Basics. Because summation is a linear operator, that is


n1 n1 n1

(cfi + dgi ) = c
i=0 i=0

fi + d
i=0

gi ,

denite integrals are linear,


b b b

(cf (x) + dg(x)) dx = c


a a

f (x) dx + d
a

g(x) dx.

One can also divide the range of integration.


b c b

f (x) dx =
a a

f (x) dx +
c

f (x) dx

123

We assume that each of the above integrals exist. If a b, and we integrate from b to a, then each of the xi will be negative. From this observation, it is clear that
b a

f (x) dx =
a b

f (x) dx.

If we integrate any function from a point a to that same point a, then all the xi are zero and
a

f (x) dx = 0.
a

Bounding the Integral. Recall that if fi gi , then


n1 n1

fi
i=0 i=0

gi .

Let m = minx[a,b] f (x) and M = maxx[a,b] f (x). Then


n1 n1 n1

(b a)m =
i=0

mxi
i=0

f (i )xi
i=0

M xi = (b a)M

implies that
b

(b a)m
a

f (x) dx (b a)M.
n1

Since

n1

fi
i=0 i=0 b

|fi |,

we have
a

f (x) dx
a

|f (x)| dx.

124

Mean Value Theorem of Integral Calculus. Let f (x) be continuous. We know from above that
b

(b a)m
a

f (x) dx (b a)M.

Therefore there exists a constant c [m, M ] satisfying


b

f (x) dx = (b a)c.
a

Since f (x) is continuous, there is a point [a, b] such that f () = c. Thus we see that
b

f (x) dx = (b a)f (),


a

for some [a, b].

4.3

The Fundamental Theorem of Integral Calculus

Denite Integrals with Variable Limits of Integration. Consider a to be a constant and x variable, then the function F (x) dened by
x

F (x) =
a

f (t) dt

(4.2)

125

is an anti-derivative of f (x), that is F (x) = f (x). To show this we apply the denition of dierentiation and the integral mean value theorem. F (x) = lim F (x + x) F (x) x0 x x+x x f (t) dt a f (t) dt a = lim x0 x x+x f (t) dt = lim x x0 x f ()x = lim , [x, x + x] x0 x = f (x)

The Fundamental Theorem of Integral Calculus. Let F (x) be any anti-derivative of f (x). Noting that all anti-derivatives of f (x) dier by a constant and replacing x by b in Equation 4.2, we see that there exists a constant c such that
b

f (x) dx = F (b) + c.
a

Now to nd the constant. By plugging in b = a,


a

f (x) dx = F (a) + c = 0,
a

we see that c = F (a). This gives us a result known as the Fundamental Theorem of Integral Calculus.
b

f (x) dx = F (b) F (a).


a

We introduce the notation [F (x)]b F (b) F (a). a

126

Example 4.3.1

sin x dx = [ cos x] = cos() + cos(0) = 2 0


0

4.4
4.4.1

Techniques of Integration
Partial Fractions
p(x) p(x) = q(x) (x a)n r(x)

A proper rational function

Can be written in the form p(x) = (x )n r(x) a0 a1 an1 + + + n n1 (x ) (x ) x + ( )

where the ak s are constants and the last ellipses represents the partial fractions expansion of the roots of r(x). The coecients are 1 dk p(x) ak = . k! dxk r(x) x= Example 4.4.1 Consider the partial fraction expansion of 1 + x + x2 . (x 1)3 The expansion has the form a0 a1 a2 + + . 3 2 (x 1) (x 1) x1 127

The coecients are 1 (1 + x + x2 )|x=1 = 3, 0! 1 d a1 = (1 + x + x2 )|x=1 = (1 + 2x)|x=1 = 3, 1! dx 1 d2 1 a2 = (1 + x + x2 )|x=1 = (2)|x=1 = 1. 2 2! dx 2 a0 = Thus we have 1 + x + x2 3 3 1 = + + . 3 3 2 (x 1) (x 1) (x 1) x1

Example 4.4.2 Suppose we want to evaluate 1 + x + x2 dx. (x 1)3 If we expand the integrand in a partial fraction expansion, then the integral becomes easy. 1 + x + x2 dx. = (x 1)3 3 3 1 + + dx 3 2 (x 1) (x 1) x1 3 3 = + ln(x 1) 2 2(x 1) (x 1)

Example 4.4.3 Consider the partial fraction expansion of 1 + x + x2 . x2 (x 1)2 The expansion has the form a0 a1 b0 b1 + + + . 2 2 x x (x 1) x1 128

The coecients are 1 1 + x + x2 = 1, 0! (x 1)2 x=0 1 d 1 + x + x2 a1 = = 1! dx (x 1)2 x=0 1 1 + x + x2 b0 = = 3, 0! x2 x=1 1 d 1 + x + x2 b1 = = 1! dx x2 x=1 a0 = Thus we have

1 + 2x 2(1 + x + x2 ) (x 1)2 (x 1)3

= 3,
x=0

1 + 2x 2(1 + x + x2 ) x2 x3

= 3,
x=1

1 + x + x2 1 3 3 3 = 2+ + . 2 (x 1)2 2 x x x (x 1) x1

If the rational function has real coecients and the denominator has complex roots, then you can reduce the work in nding the partial fraction expansion with the following trick: Let and be complex conjugate pairs of roots of the denominator. p(x) = (x )n (x )n r(x) a0 a1 an1 + + + (x )n (x )n1 x a0 a1 an1 + + + + n n1 (x ) (x ) x

+ ( )

Thus we dont have to calculate the coecients for the root at . We just take the complex conjugate of the coecients for . Example 4.4.4 Consider the partial fraction expansion of 1+x . x2 + 1 129

The expansion has the form a0 a0 + xi x+i The coecients are 1 1+x 1 = (1 i), 0! x + i x=i 2 1 1 a0 = (1 i) = (1 + i) 2 2 a0 = Thus we have 1+x 1i 1+i = + . 2+1 x 2(x i) 2(x + i)

4.5

Improper Integrals
b a

If the range of integration is innite or f (x) is discontinuous at some points then integral.

f (x) dx is called an improper

Discontinuous Functions. If f (x) is continuous on the interval a x b except at the point x = c where a < c < b then
b a c b

f (x) dx = lim +
0

f (x) dx + lim +
0

f (x) dx
c+

provided that both limits exist. Example 4.5.1 Consider the integral of ln x on the interval [0, 1]. Since the logarithm has a singularity at x = 0, this 130

is an improper integral. We write the integral in terms of a limit and evaluate the limit with LHospitals rule.
1 1

ln x dx = lim
0

ln x dx

= lim[x ln x x]1
0

= 1 ln(1) 1 lim( ln )
0

= 1 lim( ln )
0

= 1 lim

= 1 lim = 1

ln 1/ 1/ 1/ 2

Example 4.5.2 Consider the integral of xa on the range [0, 1]. If a < 0 then there is a singularity at x = 0. First assume that a = 1.
1 0

xa dx = lim +
0

xa+1 a+1

a+1 1 lim a + 1 0+ a + 1

This limit exists only for a > 1. Now consider the case that a = 1.
1 0

x1 dx = lim [ln x]1 +


0

= ln(0) lim ln +
0

131

This limit does not exist. We obtain the result,


1

xa dx =
0

1 , a+1

for a > 1.

Innite Limits of Integration. If the range of integration is innite, say [a, ) then we dene the integral as

f (x) dx = lim
a

f (x) dx,
a

provided that the limit exists. If the range of integration is (, ) then


a

f (x) dx = lim

f (x) dx + lim

f (x) dx.
a

Example 4.5.3
1

ln x dx = x2

ln x
1

d 1 dx x

dx

= ln x = lim

1 x

x+

= lim =1

x+

1 1 dx x x 1 1 ln x 1 x x 1 1/x 1 lim + 1 x x 1

132

Example 4.5.4 Consider the integral of xa on [1, ). First assume that a = 1.


1

xa+1 x dx = lim + a + 1 1 a+1 1 = lim + a + 1 a+1


a

The limit exists for < 1. Now consider the case a = 1.

x1 dx = lim [ln x] 1
1 +

= lim ln
+

1 a+1

This limit does not exist. Thus we have

xa dx =
1

1 , a+1

for a < 1.

133

4.6
4.6.1

Exercises
The Indenite Integral

Exercise 4.1 (mathematica/calculus/integral/fundamental.nb) Evaluate (2x + 3)10 dx. Hint, Solution Exercise 4.2 (mathematica/calculus/integral/fundamental.nb) x)2 Evaluate (lnx dx. Hint, Solution Exercise 4.3 (mathematica/calculus/integral/fundamental.nb) Evaluate x x2 + 3 dx. Hint, Solution Exercise 4.4 (mathematica/calculus/integral/fundamental.nb) x Evaluate cos x dx. sin Hint, Solution Exercise 4.5 (mathematica/calculus/integral/fundamental.nb) x2 Evaluate x3 5 dx. Hint, Solution

4.6.2

The Denite Integral

Exercise 4.6 (mathematica/calculus/integral/denite.nb) Use the result


b N 1

f (x) dx = lim
a

f (xn )x
n=0

134

where x =

ba N

and xn = a + nx, to show that


1 0

1 x dx = . 2

Hint, Solution Exercise 4.7 (mathematica/calculus/integral/denite.nb) Evaluate the following integral using integration by parts and the Pythagorean identity. Hint, Solution Exercise 4.8 (mathematica/calculus/integral/denite.nb) Prove that f (x) d h() d = h(f (x))f (x) h(g(x))g (x). dx g(x) (Dont use the limit denition of dierentiation, use the Fundamental Theorem of Integral Calculus.) Hint, Solution Exercise 4.9 (mathematica/calculus/integral/denite.nb) Let An be the area between the curves x and xn on the interval [0 . . . 1]. What is limn An ? Explain this result geometrically. Hint, Solution Exercise 4.10 (mathematica/calculus/integral/taylor.nb) a. Show that
x 0

sin2 x dx

f (x) = f (0) +
0

f (x ) d.

b. From the above identity show that


x

f (x) = f (0) + xf (0) +


0

f (x ) d.

135

c. Using induction, show that 1 1 f (x) = f (0) + xf (0) + x2 f (0) + + xn f (n) (0) + 2 n! Hint, Solution Exercise 4.11 Find a function f (x) whose arc length from 0 to x is 2x. Hint, Solution Exercise 4.12 Consider a curve C, bounded by 1 and 1, on the interval (1 . . . 1). Can the length of C be unbounded? What if we change to the closed interval [1 . . . 1]? Hint, Solution
x 0

1 n (n+1) f (x ) d. n!

4.6.3 4.6.4

The Fundamental Theorem of Integration Techniques of Integration

Exercise 4.13 (mathematica/calculus/integral/parts.nb) Evaluate x sin x dx. Hint, Solution Exercise 4.14 (mathematica/calculus/integral/parts.nb) Evaluate x3 e2x dx. Hint, Solution Exercise 4.15 (mathematica/calculus/integral/partial.nb) Evaluate x21 dx. 4 Hint, Solution 136

Exercise 4.16 (mathematica/calculus/integral/partial.nb) x+1 Evaluate x3 +x2 6x dx. Hint, Solution

4.6.5

Improper Integrals

Exercise 4.17 (mathematica/calculus/integral/improper.nb) 4 1 Evaluate 0 (x1)2 dx. Hint, Solution Exercise 4.18 (mathematica/calculus/integral/improper.nb) 1 1 Evaluate 0 x dx. Hint, Solution Exercise 4.19 (mathematica/calculus/integral/improper.nb) Evaluate 0 x21 dx. +4 Hint, Solution

137

4.7

Hints

Hint 4.1 Make the change of variables u = 2x + 3. Hint 4.2 Make the change of variables u = ln x. Hint 4.3 Make the change of variables u = x2 + 3. Hint 4.4 Make the change of variables u = sin x. Hint 4.5 Make the change of variables u = x3 5. Hint 4.6

N 1

x dx = lim
0

xn x
n=0 N 1

= lim

(nx)x
n=0

Hint 4.7 Let u = sin x and dv = sin x dx. Integration by parts will give you an equation for Hint 4.8 Let H (x) = h(x) and evaluate the integral in terms of H(x). 138

sin2 x dx.

Hint 4.9 CONTINUE Hint 4.10 a. Evaluate the integral. b. Use integration by parts to evaluate the integral. c. Use integration by parts with u = f (n+1) (x ) and dv = Hint 4.11 The arc length from 0 to x is
x 1 n . n!

1 + (f ())2 d
0

(4.3)

First show that the arc length of f (x) from a to b is 2(b a). Then conclude that the integrand in Equation 4.3 must everywhere be 2. Hint 4.12 CONTINUE Hint 4.13 Let u = x, and dv = sin x dx. Hint 4.14 Perform integration by parts three successive times. For the rst one let u = x3 and dv = e2x dx. Hint 4.15 Expanding the integrand in partial fractions, 1 1 a b = = + x2 4 (x 2)(x + 2) (x 2) (x + 2) 1 = a(x + 2) + b(x 2) 139

Set x = 2 and x = 2 to solve for a and b. Hint 4.16 Expanding the integral in partial fractions, x+1 x+1 a b c = = + + 2 6x +x x(x 2)(x + 3) x x2 x+3 x + 1 = a(x 2)(x + 3) + bx(x + 3) + cx(x 2) Set x = 0, x = 2 and x = 3 to solve for a, b and c. Hint 4.17
4 0

x3

1 dx = lim 0+ (x 1)2

1 0

1 dx + lim 0+ (x 1)2

4 1+

1 dx (x 1)2

Hint 4.18
1 0

1 dx = lim 0+ x

1 dx x

Hint 4.19 1 1 x dx = arctan 2 +a a a

x2

140

4.8

Solutions

Solution 4.1 (2x + 3)10 dx Let u = 2x + 3, g(u) = x =


u3 , 2

g (u) = 1 . 2 (2x + 3)10 dx = 1 u10 du 2 11 u 1 = 11 2 (2x + 3)11 = 22

Solution 4.2 (ln x)2 dx = x = Solution 4.3 x x2 + 3 dx = 1 d(x2 ) dx 2 dx d(ln x) dx dx

(ln x)2 (ln x)3 3

x2 + 3

1 (x2 + 3)3/2 = 2 3/2 2 (x + 3)3/2 = 3 141

Solution 4.4 1 d(sin x) cos x dx = dx sin x sin x dx = ln | sin x| Solution 4.5 x2 dx = x3 5 = Solution 4.6
1 N 1

1 1 d(x3 ) dx x3 5 3 dx 1 ln |x3 5| 3

x dx = lim
0

xn x
n=0 N 1

= lim

(nx)x
n=0 N 1

= lim x2
N n=0

N (N 1) = lim x2 N 2 N (N 1) = lim N 2N 2 1 = 2 142

Solution 4.7 Let u = sin x and dv = sin x dx. Then du = cos x dx and v = cos x.

sin2 x dx = sin x cos x


0

+
0

cos2 x dx

=
0

cos2 x dx (1 sin2 x) dx
0

=
0

sin2 x dx sin2 x dx =

2
0

sin2 x dx =
0

Solution 4.8 Let H (x) = h(x). d dx


f (x)

h() d =
g(x)

d (H(f (x)) H(g(x))) dx

= H (f (x))f (x) H (g(x))g (x) = h(f (x))f (x) h(g(x))g (x) Solution 4.9 First we compute the area for positive integer n.
1

An =
0

xn+1 x2 (x x ) dx = 2 n+1
n

=
0

1 1 2 n+1

143

Then we consider the area in the limit as n . lim An = lim 1 1 2 n+1 = 1 2

In Figure 4.3 we plot the functions x1 , x2 , x4 , x8 , . . . , x1024 . In the limit as n , xn on the interval [0 . . . 1] tends to the function 0 0x<1 1 x=1 Thus the area tends to the area of the right triangle with unit base and height.

0.8

0.6

0.4

0.2

0.2

0.4

0.6

0.8

Figure 4.3: Plots of x1 , x2 , x4 , x8 , . . . , x1024 .

144

Solution 4.10 1.
x

f (0) +
0

f (x ) d = f (0) + [f (x )]x 0 = f (0) f (0) + f (x) = f (x)

2.
x

f (0) + xf (0) +
0

f (x ) d = f (0) + xf (0) + [f (x

x )]0

0 )]x 0

f (x ) d

= f (0) + xf (0) xf (0) [f (x = f (0) f (0) + f (x) = f (x)

3. Above we showed that the hypothesis holds for n = 0 and n = 1. Assume that it holds for some n = m 0.
x 1 1 1 n (n+1) f (x ) d f (x) = f (0) + xf (0) + x2 f (0) + + xn f (n) (0) + 2 n! 0 n! 1 1 1 = f (0) + xf (0) + x2 f (0) + + xn f (n) (0) + n+1 f (n+1) (x ) 2 n! (n + 1)! x 1 n+1 f (n+2) (x ) d (n + 1)! 0 1 1 1 = f (0) + xf (0) + x2 f (0) + + xn f (n) (0) + xn+1 f (n+1) (0) 2 n! (n + 1)! x 1 n+1 f (n+2) (x ) d + (n + 1)! 0

x 0

This shows that the hypothesis holds for n = m + 1. By induction, the hypothesis hold for all n 0. 145

Solution 4.11 First note that the arc length from a to b is 2(b a).
b b a

1 + (f (x))2 dx =
a 0

1 + (f (x))2 dx
0

1 + (f (x))2 dx = 2b 2a

Since a and b are arbitrary, we conclude that the integrand must everywhere be 2. 1 + (f (x))2 = 2 f (x) = 3 f (x) is a continuous, piecewise dierentiable function which satises f (x) = 3 at the points where it is dierentiable. One example is f (x) = 3x Solution 4.12 CONTINUE Solution 4.13 Let u = x, and dv = sin x dx. Then du = dx and v = cos x. x sin x dx = x cos x + cos x dx

= x cos x + sin x + C Solution 4.14 Let u = x3 and dv = e2x dx. Then du = 3x2 dx and v = 1 e2x . 2 1 3 x3 e2x dx = x3 e2x 2 2 146 x2 e2x dx

Let u = x2 and dv = e2x dx. Then du = 2x dx and v = 1 e2x . 2 1 3 x3 e2x dx = x3 e2x 2 2 1 2 2x x e 2 x e2x dx x e2x dx

1 3 3 x3 e2x dx = x3 e2x x2 e2x + 2 4 2 Let u = x and dv = e2x dx. Then du = dx and v = 1 e2x . 2 3 3 1 x3 e2x dx = x3 e2x x2 e2x + 2 4 2

1 2x 1 xe 2 2

e2x dx

1 3 3 3 x3 e2x dx = x3 e2x x2 e2x + x e2x e2x +C 2 4 4 8 Solution 4.15 Expanding the integrand in partial fractions, 1 1 A B = = + 4 (x 2)(x + 2) (x 2) (x + 2) 1 = A(x + 2) + B(x 2) Setting x = 2 yields A = 1 . Setting x = 2 yields B = 1 . Now we can do the integral. 4 4 x2 1 dx = 4 1 1 dx 4(x 2) 4(x + 2) 1 1 = ln |x 2| ln |x + 2| + C 4 4 1 x2 = +C 4 x+2 147

x2

Solution 4.16 Expanding the integral in partial fractions, x+1 x+1 A B C = = + + 2 6x +x x(x 2)(x + 3) x x2 x+3

x3

x + 1 = A(x 2)(x + 3) + Bx(x + 3) + Cx(x 2) Setting x = 0 yields A = 1 . Setting x = 2 yields B = 6 x+1 dx = + x2 6x


3 . 10 2 Setting x = 3 yields C = 15 .

x3

1 3 2 + dx 6x 10(x 2) 15(x + 3) 1 3 2 = ln |x| + ln |x 2| ln |x + 3| + C 6 10 15 |x 2|3/10 = ln 1/6 +C |x| |x + 3|2/15

Solution 4.17

4 0

1 dx = lim 0+ (x 1)2

1 0

1 dx + lim 0+ (x 1)2
1

4 1+

1 dx (x 1)2
4 1+

1 0 0 x1 0 1 1 1 = lim 1 + lim + + + 0 0 3 =+ = lim + 1 x1 + lim + The integral diverges. 148

Solution 4.18
1 0 1

1 dx = lim 0+ x

1 dx x 1 = lim 2 x 0+ = lim 2(1 ) +


0

=2 Solution 4.19
0

1 dx = lim 2+4 x

1 dx +4 0 1 x = lim arctan 2 2 1 = 0 2 2 = 4 x2

149

4.9

Quiz
b a

Problem 4.1 Write the limit-sum denition of Solution Problem 4.2 2 Evaluate 1 |x| dx. Solution Problem 4.3 2 x d Evaluate dx x f () d. Solution Problem 4.4 2 Evaluate 1+x+x3 dx. (x+1) Solution

f (x) dx.

Problem 4.5 State the integral mean value theorem. Solution Problem 4.6 What is the partial fraction expansion of Solution
1 ? x(x1)(x2)(x3)

150

4.10

Quiz Solutions

Solution 4.1 Let a = x0 < x1 < < xn1 < xn = b be a partition of the interval (a..b). We dene xi = xi+1 xi and x = maxi xi and choose i [xi ..xi+1 ].
b n1

f (x) dx = lim
a

x0

f (i )xi
i=0

Solution 4.2
2 0

|x| dx =
1

x dx +
0 2

x dx

= =

1 1 0

x dx +
0 1

x dx
2 0

2 3/2 2 x + x3/2 3 3 0 2 2 = + 23/2 3 3 2 = (1 + 2 2) 3 Solution 4.3

d dx

x2 x

f () d = f (x2 )

d 2 d (x ) f (x) (x) dx dx 2 = 2xf (x ) f (x) 151

Solution 4.4 First we expand the integrand in partial fractions. 1 + x + x2 a b c = + + (x + 1)3 (x + 1)3 (x + 1)2 x + 1 a = (1 + x + x2 ) b= c= Then we can do the integration. 1 + x + x2 dx = (x + 1)3 1 1 1 + 3 2 (x + 1) (x + 1) x+1 1 1 = + + ln |x + 1| 2(x + 1)2 x + 1 x + 1/2 + ln |x + 1| = (x + 1)2 dx 1 1! 1 2!
x=1

=1 = (1 + 2x)
x=1 x=1

d (1 + x + x2 ) dx d2 (1 + x + x2 ) dx2

= 1

=
x=1

1 (2) 2

x=1

=1

Solution 4.5 Let f (x) be continuous. Then


b

f (x) dx = (b a)f (),


a

for some [a..b]. Solution 4.6 1 a b c d = + + + x(x 1)(x 2)(x 3) x x1 x2 x3 152

a=

1 1 = (0 1)(0 2)(0 3) 6 1 1 b= = (1)(1 2)(1 3) 2 1 1 c= = (2)(2 1)(2 3) 2 1 1 d= = (3)(3 1)(3 2) 6

1 1 1 1 1 = + + x(x 1)(x 2)(x 3) 6x 2(x 1) 2(x 2) 6(x 3)

153

Chapter 5 Vector Calculus


5.1 Vector Functions

Vector-valued Functions. A vector-valued function, r(t), is a mapping r : R Rn that assigns a vector to each value of t. r(t) = r1 (t)e1 + + rn (t)en . An example of a vector-valued function is the position of an object in space as a function of time. The function is continous at a point t = if lim r(t) = r( ).
t

This occurs if and only if the component functions are continuous. The function is dierentiable if dr r(t + t) r(t) lim dt t0 t exists. This occurs if and only if the component functions are dierentiable. If r(t) represents the position of a particle at time t, then the velocity and acceleration of the particle are dr dt and 154 d2 r , dt2

respectively. The speed of the particle is |r (t)|. Dierentiation Formulas. Let f (t) and g(t) be vector functions and a(t) be a scalar function. By writing out components you can verify the dierentiation formulas: d (f g) = f g + f g dt d (f g) = f g + f g dt d (af ) = a f + af dt

5.2

Gradient, Divergence and Curl

Scalar and Vector Fields. A scalar eld is a function of position u(x) that assigns a scalar to each point in space. A function that gives the temperature of a material is an example of a scalar eld. In two dimensions, you can graph a scalar eld as a surface plot, (Figure 5.1), with the vertical axis for the value of the function. A vector eld is a function of position u(x) that assigns a vector to each point in space. Examples of vectors elds are functions that give the acceleration due to gravity or the velocity of a uid. You can graph a vector eld in two or three dimension by drawing vectors at regularly spaced points. (See Figure 5.1 for a vector eld in two dimensions.) Partial Derivatives of Scalar Fields. Consider a scalar eld u(x). The partial derivative of u with respect to xk is the derivative of u in which xk is considered to be a variable and the remaining arguments are considered to be u parameters. The partial derivative is denoted xk u(x), xk or uxk and is dened u u(x1 , . . . , xk + x, . . . , xn ) u(x1 , . . . , xk , . . . , xn ) lim . xk x0 x Partial derivatives have the same dierentiation formulas as ordinary derivatives. 155

1 0.5 0 -0.5 -1 0 2 4 6 0 2 4 6

Figure 5.1: A Scalar Field and a Vector Field

Consider a scalar eld in R3 , u(x, y, z). Higher derivatives of u are denoted: 2u u , 2 x x x 2u u , xy x y 4u 2 u . x2 yz x2 y z 156

uxx uxy uxxyz

If uxy and uyx are continuous, then 2u 2u = . xy yx This is referred to as the equality of mixed partial derivatives. Partial Derivatives of Vector Fields. Consider a vector eld u(x). The partial derivative of u with respect to u xk is denoted xk u(x), xk or uxk and is dened u(x1 , . . . , xk + x, . . . , xn ) u(x1 , . . . , xk , . . . , xn ) u lim . xk x0 x Partial derivatives of vector elds have the same dierentiation formulas as ordinary derivatives. Gradient. We introduce the vector dierential operator, which is known as del or nabla. In R3 it is i+ j + k. x y z e1 + + en , x1 xn

Let u(x) be a dierential scalar eld. The gradient of u is, u u u e1 + + en , x1 xn

Directional Derivative. Suppose you are standing on some terrain. The slope of the ground in a particular direction is the directional derivative of the elevation in that direction. Consider a dierentiable scalar eld, u(x). The 157

derivative of the function in the direction of the unit vector a is the rate of change of the function in that direction. Thus the directional derivative, Da u, is dened: Da u(x) = lim
0

u(x + a) u(x) u(x1 + a1 , . . . , xn + an ) u(x1 , . . . , xn ) (u(x) + a1 ux1 (x) + + an uxn (x) + O( 2 )) u(x)

= lim
0

= lim
0

= a1 ux1 (x) + + an uxn (x) Da u(x) = u(x) a.

Tangent to a Surface. The gradient, f , is orthogonal to the surface f (x) = 0. Consider a point on the surface. Let the dierential dr = dx1 e1 + dxn en lie in the tangent plane at . Then df = since f (x) = 0 on the surface. Then f dr = f e1 + + x1 f = dx1 + + x1 =0 f en (dx1 e1 + + dxn en ) xn f dxn xn f f dx1 + + dxn = 0 x1 xn

Thus

f is orthogonal to the tangent plane and hence to the surface.

Example 5.2.1 Consider the paraboloid, x2 + y 2 z = 0. We want to nd the tangent plane to the surface at the point (1, 1, 2). The gradient is f = 2xi + 2yj k. 158

At the point (1, 1, 2) this is f (1, 1, 2) = 2i + 2j k. We know a point on the tangent plane, (1, 1, 2), and the normal, f (1, 1, 2) (x, y, z) = f (1, 1, 2). The equation of the plane is

f (1, 1, 2) (1, 1, 2)

2x + 2y z = 2 The gradient of the function f (x) = 0, f (x), is in the direction of the maximum directional derivative. The magnitude of the gradient, | f (x)|, is the value of the directional derivative in that direction. To derive this, note that Da f = f a = | f | cos ,

where is the angle between f and a. Da f is maximum when = 0, i.e. when a is the same direction as f . In this direction, Da f = | f |. To use the elevation example, f points in the uphill direction and | f | is the uphill slope. Example 5.2.2 Suppose that the two surfaces f (x) = 0 and g(x) = 0 intersect at the point x = . What is the angle between their tangent planes at that point? First we note that the angle between the tangent planes is by denition the angle between their normals. These normals are in the direction of f () and g(). (We assume these are nonzero.) The angle, , between the tangent planes to the surfaces is = arccos f () g() | f ()| | g()| .

Example 5.2.3 Let u be the distance from the origin: u(x) = x x = xi xi . In three dimensions, this is u(x, y, z) = x2 + y 2 + z 2 . 159

The gradient of u,

(x), is a unit vector in the direction of x. The gradient is: u(x) = xn x1 ,..., xx xx x i ei = . xj xj

In three dimensions, we have u(x, y, z) = x x2 + y 2 + z 2 , y x2 + y 2 + z 2 , z x2 + y 2 + z 2 .

This is a unit vector because the sum of the squared components sums to unity. u Figure 5.2 shows a plot of the vector eld of xi ei xk ek xi xi u= =1 xj xj xl xl xj xj u in two dimensions.

Example 5.2.4 Consider an ellipse. An implicit equation of an ellipse is x2 y 2 + 2 = 1. a2 b We can also express an ellipse as u(x, y) + v(x, y) = c where u and v are the distance from the two foci. That is, an ellipse is the set of points such that the sum of the distances from the two foci is a constant. Let n = (u + v). This is a vector which is orthogonal to the ellipse when evaluated on the surface. Let t be a unit tangent to the surface. Since n and t are orthogonal, nt=0 ( u + v) t = 0 u t = v (t). 160

Figure 5.2: The gradient of the distance from the origin.


n v -t u v u t

Figure 5.3: An ellipse and rays from the foci. Since these are unit vectors, the angle between u and t is equal to the angle between v and t. In other words: If we draw rays from the foci to a point on the ellipse, the rays make equal angles with the ellipse. If the ellipse were 161

a reective surface, a wave starting at one focus would be reected from the ellipse and travel to the other focus. See Figure 6.4. This result also holds for ellipsoids, u(x, y, z) + v(x, y, z) = c. We see that an ellipsoidal dish could be used to collect spherical waves, (waves emanating from a point). If the dish is shaped so that the source of the waves is located at one foci and a collector is placed at the second, then any wave starting at the source and reecting o the dish will travel to the collector. See Figure 5.4.

Figure 5.4: An elliptical dish.

162

5.3

Exercises

Vector Functions
Exercise 5.1 Consider the parametric curve r = cos Calculate dr and dt Hint, Solution
d2 r . dt2

t 2

i + sin

t 2

j.

Plot the position and some velocity and acceleration vectors.

Exercise 5.2 Let r(t) be the position of an object moving with constant speed. Show that the acceleration of the object is orthogonal to the velocity of the object. Hint, Solution

Vector Fields
Exercise 5.3 Consider the paraboloid x2 + y 2 z = 0. What is the angle between the two tangent planes that touch the surface at (1, 1, 2) and (1, 1, 2)? What are the equations of the tangent planes at these points? Hint, Solution Exercise 5.4 Consider the paraboloid x2 + y 2 z = 0. What is the point on the paraboloid that is closest to (1, 0, 0)? Hint, Solution Exercise 5.5 Consider the region R dened by x2 + xy + y 2 9. What is the volume of the solid obtained by rotating R about the y axis? Is this the same as the volume of the solid obtained by rotating R about the x axis? Give geometric and algebraic explanations of this. 163

Hint, Solution Exercise 5.6 Two cylinders of unit radius intersect at right angles as shown in Figure 5.5. What is the volume of the solid enclosed by the cylinders?

Figure 5.5: Two cylinders intersecting. Hint, Solution Exercise 5.7 Consider the curve f (x) = 1/x on the interval [1 . . . ). Let S be the solid obtained by rotating f (x) about the x axis. (See Figure 5.6.) Show that the length of f (x) and the lateral area of S are innite. Find the volume of S. 1 Hint, Solution Exercise 5.8 Suppose that a deposit of oil looks like a cone in the ground as illustrated in Figure 5.7. Suppose that the oil has a
1

You could ll S with a nite amount of paint, but it would take an innite amount of paint to cover its surface.

164

1 2 3 4 5 -1 0

-1 1

Figure 5.6: The rotation of 1/x about the x axis. density of 800kg/m3 and its vertical depth is 12m. How much work2 would it take to get the oil to the surface. Hint, Solution Exercise 5.9 Find the area and volume of a sphere of radius R by integrating in spherical coordinates. Hint, Solution
2

Recall that work = force distance and force = mass acceleration.

165

surface 32 m 12 m 12 m

ground

Figure 5.7: The oil deposit.

5.4

Hints

Vector Functions
Hint 5.1 Plot the velocity and acceleration vectors at regular intervals along the path of motion. Hint 5.2 If r(t) has constant speed, then |r (t)| = c. The condition that the acceleration is orthogonal to the velocity can be stated mathematically in terms of the dot product, r (t) r (t) = 0. Write the condition of constant speed in terms of a dot product and go from there.

Vector Fields
Hint 5.3 The angle between two planes is the angle between the vectors orthogonal to the planes. The angle between the two 166

vectors is = arccos

2, 2, 1 2, 2, 1 | 2, 2, 1 || 2, 2, 1 |

The equation of a line orthogonal to a and passing through the point b is a x = a b. Hint 5.4 Since the paraboloid is a dierentiable surface, the normal to the surface at the closest point will be parallel to the vector from the closest point to (1, 0, 0). We can express this using the gradient and the cross product. If (x, y, z) is the closest point on the paraboloid, then a vector orthogonal to the surface there is f = 2x, 2y, 1 . The vector from the surface to the point (1, 0, 0) is 1 x, y, z . These two vectors are parallel if their cross product is zero. Hint 5.5 CONTINUE Hint 5.6 CONTINUE Hint 5.7 CONTINUE Hint 5.8 Start with the formula for the work required to move the oil to the surface. Integrate over the mass of the oil. Work = (acceleration) (distance) d(mass)

Here (distance) is the distance of the dierential of mass from the surface. The acceleration is that of gravity, g. Hint 5.9 CONTINUE

167

5.5

Solutions

Vector Functions
Solution 5.1 The velocity is 1 r = sin 2 The acceleration is 1 r = cos 4 t 2 t 2 i+ 1 cos 2 1 sin 4 t 2 t 2 j.

j.

See Figure 5.8 for plots of position, velocity and acceleration.

Figure 5.8: A Graph of Position and Velocity and of Position and Acceleration Solution 5.2 If r(t) has constant speed, then |r (t)| = c. The condition that the acceleration is orthogonal to the velocity can be stated mathematically in terms of the dot product, r (t) r (t) = 0. Note that we can write the condition of constant 168

speed in terms of a dot product, r (t) r (t) = c, r (t) r (t) = c2 . Dierentiating this equation yields, r (t) r (t) + r (t) r (t) = 0 r (t) r (t) = 0. This shows that the acceleration is orthogonal to the velocity.

Vector Fields
Solution 5.3 The gradient, which is orthogonal to the surface when evaluated there is f = 2xi+2yjk. 2i+2jk and 2i2jk are orthogonal to the paraboloid, (and hence the tangent planes), at the points (1, 1, 2) and (1, 1, 2), respectively. The angle between the tangent planes is the angle between the vectors orthogonal to the planes. The angle between the two vectors is 2, 2, 1 2, 2, 1 = arccos | 2, 2, 1 || 2, 2, 1 | = arccos 1 9 1.45946.

Recall that the equation of a line orthogonal to a and passing through the point b is a x = a b. The equations of the tangent planes are 2, 2, 1 x, y, z = 2, 2, 1 1, 1, 2 , 2x 2y z = 2. The paraboloid and the tangent planes are shown in Figure 5.9. 169

-1

0 1 4

0 1 0 -1

Figure 5.9: Paraboloid and Two Tangent Planes Solution 5.4 Since the paraboloid is a dierentiable surface, the normal to the surface at the closest point will be parallel to the vector from the closest point to (1, 0, 0). We can express this using the gradient and the cross product. If (x, y, z) is the closest point on the paraboloid, then a vector orthogonal to the surface there is f = 2x, 2y, 1 . The vector from the surface to the point (1, 0, 0) is 1 x, y, z . These two vectors are parallel if their cross product is zero, 2x, 2y, 1 1 x, y, z = y 2yz, 1 + x + 2xz, 2y = 0. This gives us the three equations, y 2yz = 0, 1 + x + 2xz = 0, 2y = 0. The third equation requires that y = 0. The rst equation then becomes trivial and we are left with the second equation, 1 + x + 2xz = 0. Substituting z = x2 + y 2 into this equation yields, 2x3 + x 1 = 0. 170

The only real valued solution of this polynomial is x= 62/3 9 + 87 9 + 87


2/3

61/3

1/3

0.589755.

Thus the closest point to (1, 0, 0) on the paraboloid is 62/3 9 + 87 9 + 87


2/3

61/3

1/3

, 0,

62/3 9 +

87 9 + 87

2/3

61/3

(0.589755, 0, 0.34781).

1/3

The closest point is shown graphically in Figure 5.10.


1 1-1 0.5 -1 -0.5 0 -0.5 0 0.5 1

1.5

0.5

Figure 5.10: Paraboloid, Tangent Plane and Line Connecting (1, 0, 0) to Closest Point

171

Solution 5.5 We consider the region R dened by x2 + xy + y 2 9. The boundary of the region is an ellipse. (See Figure 5.11 for the ellipse and the solid obtained by rotating the region.) Note that in rotating the region about the y axis, only the
2 3 -2 2 1 0

2 1 2 3

-3

-2

-1 -1 -2

0 -2

-2 -3 0 2

Figure 5.11: The curve x2 + xy + y 2 = 9. portions in the second and fourth quadrants make a contribution. Since the solid is symmetric across the xz plane, we will nd the volume of the top half and then double this to get the volume of the whole solid. Now we consider rotating the region in the second quadrant about the y axis. In the equation for the ellipse, x2 + xy + y 2 = 9, we solve for x. x= 1 y 3 12 y 2 2

In the second quadrant, the curve (y 3 12 y 2 )/2 is dened on y [0 . . . 12] and the curve (y 3 12 y 2 )/2 is dened on y [3 . . . 12]. (See Figure 5.12.) We nd the volume obtained by rotating the 172

3.5 3 2.5 2 1.5 1 0.5

-3.5

-3

-2.5

-2

-1.5

-1

-0.5

Figure 5.12: (y

3 12 y 2 )/2 in red and (y +

3 12 y 2 )/2 in green.

rst curve and subtract the volume from rotating the second curve. 2 12 12 y 3 12 y 2 V = 2 dy 2 0 3

y +

3 12 2

y2

dy

V = 2

12

y+
0

12

3 12 y 2

dy
3 12

y +

3 12 y 2 12y

dy

V = 2 V = 2

12

2y 2 +
0

12y

12 y 2 + 36 dy
3 12 3/2

2y 2

12 y 2 + 36 dy
12 3/2

2 2 y 3 12 y 2 3 3

+ 36y
0

2 2 y 3 + 12 y 2 3 3

+ 36y
3

V = 72 173

Now consider the volume of the solid obtained by rotating R about the x axis? This as the same as the volume of the solid obtained by rotating R about the y axis. Geometrically we know this because R is symmetric about the line y = x. Now we justify it algebraically. Consider the phrase: Rotate the region x2 + xy + y 2 9 about the x axis. We formally swap x and y to obtain: Rotate the region y 2 + yx + x2 9 about the y axis. Which is the original problem. Solution 5.6 We nd of the volume of the intersecting cylinders by summing the volumes of the two cylinders and then subracting the volume of their intersection. The volume of each of the cylinders is 2. The intersection is shown in Figure 5.13. If we slice this solid along the plane z = const we have a square with side length 2 1 z 2 . The volume of the intersection of the cylinders is
1

4 1 z 2 dz.
1

We compute the volume of the intersecting cylinders.


1 0.5 0 -0.5 -1 1 0.5 0 -0.5 -1 -1 -0.5 0 0.5 1

Figure 5.13: The intersection of the two cylinders. 174

V = 2(2) 2
0

4 1 z 2 dz 16 3

V = 4 Solution 5.7 The length of f (x) is

L=
1

1 + 1/x2 dx.

Since 1 + 1/x2 > 1/x, the integral diverges. The length is innite. We nd the area of S by integrating the length of circles.

A=
1

2 dx x

This integral also diverges. The area is innite. Finally we nd the volume of S by integrating the area of disks.

V =
1

dx = 2 x x

=
1

Solution 5.8 First we write the formula for the work required to move the oil to the surface. We integrate over the mass of the oil. Work = (acceleration) (distance) d(mass)

Here (distance) is the distance of the dierential of mass from the surface. The acceleration is that of gravity, g. The dierential of mass can be represented an a dierential of volume time the density of the oil, 800 kg/m3 . Work = 800g(distance) d(volume) 175

We place the coordinate axis so that z = 0 coincides with the bottom of the cone. The oil lies between z = 0 and z = 12. The cross sectional area of the oil deposit at a xed depth is z 2 . Thus the dierential of volume is z 2 dz. This oil must me raised a distance of 24 z.
12

W =
0

800 g (24 z) z 2 dz W = 6912000g

W 2.13 108 Solution 5.9 The Jacobian in spherical coordinates is r2 sin .


2

kg m2 s2

area =
0 0

R2 sin d d

= 2R2
0

sin d

= 2R2 [ cos ] 0 area = 4R2


R 2 0 R 0 0 R 0

volume =
0

r2 sin d d dr r2 sin d dr r 3
3

= 2 = 2

[ cos ] 0
0

4 volume = R3 3

176

5.6

Quiz

Problem 5.1 What is the distance from the origin to the plane x + 2y + 3z = 4? Solution Problem 5.2 A bead of mass m slides frictionlessly on a wire determined parametrically by w(s). The bead moves under the force of gravity. What is the acceleration of the bead as a function of the parameter s? Solution

177

5.7

Quiz Solutions

Solution 5.1 Recall that the equation of a plane is x n = a n where a is a point in the plane and n is normal to the plane. We are considering the plane x + 2y + 3z = 4. A normal to the plane is 1, 2, 3 . The unit normal is 1 n = 1, 2, 3 . 15 By substituting in x = y = 0, we see that a point in the plane is a = 0, 0, 4/3 . The distance of the plane from the origin is a n = 4 . 15 Solution 5.2 The force of gravity is gk. The unit tangent to the wire is w (s)/|w (s)|. The component of the gravitational force in the tangential direction is gk w (s)/|w (s)|. Thus the acceleration of the bead is gk w (s) . m|w (s)|

178

Part III Functions of a Complex Variable

179

Chapter 6 Complex Numbers


Im sorry. You have reached an imaginary number. Please rotate your phone 90 degrees and dial again. -Message on answering machine of Cathy Vargas.

6.1

Complex Numbers

Shortcomings of Real Numbers. When you started algebra, you learned that the quadratic equation: x2 + 2ax + b = 0 has either two, one or no solutions. For example: x2 3x + 2 = 0 has the two solutions x = 1 and x = 2. For x2 2x + 1 = 0, x = 1 is a solution of multiplicity two. x2 + 1 = 0 has no solutions. 180

This is a little unsatisfactory. We can formally solve the general quadratic equation. x2 + 2ax + b = 0 (x + a)2 = a2 b x = a a2 b However, solutions are dened only when the discriminant, a2 b is positive. This is because the square root the function, x, is a bijection from R0+ to R0+ . (See Figure 6.1.)

Figure 6.1: y =

A New Mathematical Constant. We cannot solve x2 = 1 because 1 is dened. To overcome this not apparent shortcoming of the real number system, we create a new symbolic constant 1. Note that we can express the square root of any negative real number in terms of 1: r = 1 r. Now we can express the solutions of 2 2 x2 = 1 as x = 1 and x = 1. These satisfy the equation since 1 = 1 and 1 = 1. 181

Eulers Notation. Euler introduced the notation of using the letter i to denote 1. We will use the symbol , an i without a dot, to denote 1. This helps us distinguish it from i used as a variable or index.1 We call any number of the form b, b R, a pure imaginary number.2 We call numbers of the form a + b, where a, b R, complex numbers 3

The Quadratic. Now we return to the quadratic with real coecients, x2 + 2ax + b = 0. It has the solutions x = a a2 b. The solutions are real-valued only if a2 b 0. If not, then we can dene solutions as complex numbers. If the discriminant is negative, we write x = a b a2 . Thus every quadratic polynomial with real coecients has exactly two solutions, counting multiplicities. The fundamental theorem of algebra states that an nth degree polynomial with complex coecients has n, not necessarily distinct, complex roots. We will prove this result later using the theory of functions of a complex variable.

Component Operations. Consider the complex number z = x + y, (x, y R). The real part of z is (z) = x; the imaginary part of z is (z) = y. Two complex numbers, z1 = x1 + y1 and z2 = x2 + y2 , are equal if and only if x1 = x2 and y1 = y2 . The complex conjugate 4 of z = x + y is z x y. The notation z x y is also used.

Field Properties. The set of complex numbers, C, form a eld. That essentially means that we can do arithmetic with complex numbers. We treat as a symbolic constant with the property that 2 = 1. The eld of complex numbers satisfy the following properties: (Let z, z1 , z2 , z3 C.)
Electrical engineering types prefer to use or j to denote 1. 2 Imaginary is an unfortunate term. Real numbers are articial; constructs of the mind. Real numbers are no more real than imaginary numbers. 3 Here complex means composed of two or more parts, not hard to separate, analyze, or solve. Those who disagree have a complex number complex. 4 Conjugate: having features in common but opposite or inverse in some particular.
1

182

1. Closure under addition and multiplication. z1 + z2 = (x1 + y1 ) + (x2 + y2 ) = (x1 + x2 ) + (y1 + y2 ) C z1 z2 = (x1 + y1 ) (x2 + y2 ) = (x1 x2 y1 y2 ) + (x1 y2 + x2 y1 ) C 2. Commutativity of addition and multiplication. z1 + z2 = z2 + z1 . z1 z2 = z2 z1 . 3. Associativity of addition and multiplication. (z1 + z2 ) + z3 = z1 + (z2 + z3 ). (z1 z2 ) z3 = z1 (z2 z3 ). 4. Distributive law. z1 (z2 + z3 ) = z1 z2 + z1 z3 . 5. Identity with respect to addition and multiplication. z + 0 = z. z(1) = z. 6. Inverse with respect to addition. z + (z) = (x + y) + (x y) = 0. 7. Inverse with respect to multiplication for nonzero numbers. zz 1 = 1, where z 1 = 1 1 x y x y = = 2 = 2 2 2 2 z x + y x +y x +y x + y2

Properties of the Complex Conjugate. Using the eld properties of complex numbers, we can derive the following properties of the complex conjugate, z = x y. 1. (z) = z, 2. z + = z + , 3. z = z, 4. z z = . 183

6.2

The Complex Plane

Complex Plane. We can denote a complex number z = x + y as an ordered pair of real numbers (x, y). Thus we can represent a complex number as a point in R2 where the rst component is the real part and the second component is the imaginary part of z. This is called the complex plane or the Argand diagram. (See Figure 6.2.) A complex number written as z = x + y is said to be in Cartesian form, or a + b form.
Im(z) (x,y) r Re(z)

Figure 6.2: The Complex Plane

Recall that there are two ways of describing a point in the complex plane: an ordered pair of coordinates (x, y) that give the horizontal and vertical oset from the origin or the distance r from the origin and the angle from the positive horizontal axis. The angle is not unique. It is only determined up to an additive integer multiple of 2. Modulus. The magnitude or modulus of a complex number is the distance of the point from the origin. It is dened as |z| = |x + y| = x2 + y 2 . Note that zz = (x + y)(x y) = x2 + y 2 = |z|2 . The modulus has the following properties. 1. |z1 z2 | = |z1 | |z2 | 2. z1 |z1 | = for z2 = 0. z2 |z2 | 184

3. |z1 + z2 | |z1 | + |z2 | 4. |z1 + z2 | ||z1 | |z2 || We could prove the rst two properties by expanding in x + y form, but it would be fairly messy. The proofs will become simple after polar form has been introduced. The second two properties follow from the triangle inequalities in geometry. This will become apparent after the relationship between complex numbers and vectors is introduced. One can show that |z1 z2 zn | = |z1 | |z2 | |zn | and |z1 + z2 + + zn | |z1 | + |z2 | + + |zn | with proof by induction. Argument. The argument of a complex number is the angle that the vector with tail at the origin and head at z = x + y makes with the positive x-axis. The argument is denoted arg(z). Note that the argument is dened for all nonzero numbers and is only determined up to an additive integer multiple of 2. That is, the argument of a complex number is the set of values: { + 2n | n Z}. The principal argument of a complex number is that angle in the set arg(z) which lies in the range (, ]. The principal argument is denoted Arg(z). We prove the following identities in Exercise 6.10. arg(z) = arg(z) + arg() Arg(z) = Arg(z) + Arg() arg z 2 = arg(z) + arg(z) = 2 arg(z) Example 6.2.1 Consider the equation |z 1 | = 2. The set of points satisfying this equation is a circle of radius 2 and center at 1 + in the complex plane. You can see this by noting that |z 1 | is the distance from the point (1, 1). (See Figure 6.3.) 185

3 2 1 -1 -1
Figure 6.3: Solution of |z 1 | = 2 Another way to derive this is to substitute z = x + y into the equation. |x + y 1 | = 2 (x 1)2 + (y 1)2 = 2 (x 1)2 + (y 1)2 = 4 This is the analytic geometry equation for a circle of radius 2 centered about (1, 1). Example 6.2.2 Consider the curve described by |z| + |z 2| = 4. Note that |z| is the distance from the origin in the complex plane and |z 2| is the distance from z = 2. The equation is (distance from (0, 0)) + (distance from (2, 0)) = 4. 186

From geometry, we know that this is an ellipse with foci at (0, 0) and (2, 0), major axis 2, and minor axis Figure 6.4.)

3. (See

2 1 -1 -1 -2
Figure 6.4: Solution of |z| + |z 2| = 4 We can use the substitution z = x + y to get the equation in algebraic form. |z| + |z 2| = 4 |x + y| + |x + y 2| = 4 x2 + y 2 + (x 2)2 + y 2 = 4 x2 + y 2 = 16 8 (x 2)2 + y 2 + x2 4x + 4 + y 2 x 5 = 2 (x 2)2 + y 2 x2 10x + 25 = 4x2 16x + 16 + 4y 2 1 1 (x 1)2 + y 2 = 1 4 3 187

Thus we have the standard form for an equation describing an ellipse.

6.3

Polar Form

Polar Form. A complex number written in Cartesian form, z = x + y, can be converted polar form, z = r(cos + sin ), using trigonometry. Here r = |z| is the modulus and = arctan(x, y) is the argument of z. The argument is the angle between the x axis and the vector with its head at (x, y). (See Figure 6.5.) Note that is not unique. If z = r(cos + sin ) then z = r(cos( + 2n) + sin( + 2n)) for any n Z.
Im( z ) r r cos (x,y) r sin Re(z )

Figure 6.5: Polar Form

The Arctangent. Note that arctan(x, y) is not the same thing as the old arctangent that you learned about in y trigonometry arctan(x, y) is sensitive to the quadrant of the point (x, y), while arctan x is not. For example, arctan(1, 1) = whereas arctan 1 1 = arctan 1 1 = arctan(1). + 2n 4 and arctan(1, 1) = 3 + 2n, 4

188

Eulers Formula. Eulers formula, e = cos + sin ,5 allows us to write the polar form more compactly. Expressing the polar form in terms of the exponential function of imaginary argument makes arithmetic with complex numbers much more convenient. z = r(cos + sin ) = r e The exponential of an imaginary argument has all the nice properties that we know from studying functions of a real variable, like ea eb = e(a+b) . Later on we will introduce the exponential of a complex number. Using Eulers Formula, we can express the cosine and sine in terms of the exponential. e + e (cos() + sin()) + (cos() + sin()) = = cos() 2 2 e e (cos() + sin()) (cos() + sin()) = = sin() 2 2

Arithmetic With Complex Numbers. Note that it is convenient to add complex numbers in Cartesian form. (x1 + y1 ) + (x2 + y2 ) = (x1 + x2 ) + (y1 + y2 ) However, it is dicult to multiply or divide them in Cartesian form. (x1 + y1 ) (x2 + y2 ) = (x1 x2 y1 y2 ) + (x1 y2 + x2 y1 ) x1 + y1 (x1 + y1 ) (x2 y2 ) x1 x2 + y1 y2 x2 y1 x1 y2 = = + 2 2 2 x2 + y2 (x2 + y2 ) (x2 y2 ) x2 + y2 x2 + y2 2
5

See Exercise 6.17 for justication of Eulers formula.

189

On the other hand, it is dicult to add complex numbers in polar form.

r1 e1 +r2 e2 = r1 (cos 1 + sin 1 ) + r2 (cos 2 + sin 2 ) = r1 cos 1 + r2 cos 2 + (r1 sin 1 + r2 sin 2 ) = (r1 cos 1 + r2 cos 2 )2 + (r1 sin 1 + r2 sin 2 )2 e arctan(r1 cos 1 +r2 cos 2 ,r1 sin 1 +r2 sin 2 ) =
2 2 r1 + r2 + 2 cos (1 2 ) e arctan(r1 cos 1 +r2 cos 2 ,r1 sin 1 +r2 sin 2 )

However, it is convenient to multiply and divide them in polar form.

r1 e1 r2 e2 = r1 r2 e(1 +2 ) r1 r1 e1 = e(1 2 ) 2 r2 e r2

Keeping this in mind will make working with complex numbers a shade or two less grungy. 190

Result 6.3.1 Eulers formula is e = cos + sin . We can write the cosine and sine in terms of the exponential. e + e cos() = , 2 e e sin() = 2

To change between Cartesian and polar form, use the identities r e = r cos + r sin , x + y = x2 + y 2 e arctan(x,y) .

Cartesian form is convenient for addition. Polar form is convenient for multiplication and division.
Example 6.3.1 We write 5 + 7 in polar form. 5 + 7 = We write 2 e/6 in Cartesian form. 2 e/6 = 2 cos + 2 sin 6 6 = 3+ Example 6.3.2 We will prove the trigonometric identity cos4 = 1 1 3 cos(4) + cos(2) + . 8 2 8 191 74 e arctan(5,7)

We start by writing the cosine in terms of the exponential. cos4 = = e + e 2


4

1 4 e +4 e2 +6 + 4 e2 + e4 16 1 e4 + e4 1 e2 + e2 3 = + + 8 2 2 2 8 1 1 3 = cos(4) + cos(2) + 8 2 8

By the denition of exponentiation, we have en = e in deriving trigonometric identities.

We apply Eulers formula to obtain a result which is useful

cos(n) + sin(n) = (cos() + sin())n

Result 6.3.2 DeMoivres Theorem.a cos(n) + sin(n) = (cos() + sin())n


a

Its amazing what passes for a theorem these days. I would think that this would be a corollary at most.

Example 6.3.3 We will express cos(5) in terms of cos and sin(5) in terms of sin . We start with DeMoivres theorem. e5 = e 192
5

cos(5) + sin(5) = (cos + sin )5 5 5 5 5 cos5 + cos4 sin cos3 sin2 cos2 sin3 = 0 1 2 3 5 5 cos sin4 + sin5 + 4 5 5 3 2 = cos 10 cos sin + 5 cos sin4 + 5 cos4 sin 10 cos2 sin3 + sin5 Then we equate the real and imaginary parts. cos(5) = cos5 10 cos3 sin2 + 5 cos sin4 sin(5) = 5 cos4 sin 10 cos2 sin3 + sin5 Finally we use the Pythagorean identity, cos2 + sin2 = 1. cos(5) = cos5 10 cos3 1 cos2 + 5 cos 1 cos2 cos(5) = 16 cos5 20 cos3 + 5 cos sin(5) = 5 1 sin2
2 2

sin 10 1 sin2 sin3 + sin5

sin(5) = 16 sin5 20 sin3 + 5 sin

6.4

Arithmetic and Vectors

Addition. We can represent the complex number z = x + y = r e as a vector in Cartesian space with tail at the origin and head at (x, y), or equivalently, the vector of length r and angle . With the vector representation, we can add complex numbers by connecting the tail of one vector to the head of the other. The vector z + is the diagonal of the parallelogram dened by z and . (See Figure 6.6.) Negation. The negative of z = x + y is z = x y. In polar form we have z = r e and z = r e(+) , (more generally, z = r e(+(2n+1)) , n Z. In terms of vectors, z has the same magnitude but opposite direction as z. (See Figure 6.6.) 193

Multiplication. The product of z = r e and = e is z = r e(+) . The length of the vector z is the product of the lengths of z and . The angle of z is the sum of the angles of z and . (See Figure 6.6.) Note that arg(z) = arg(z) + arg(). Each of these arguments has an innite number of values. If we write out the multi-valuedness explicitly, we have { + + 2n : n Z} = { + 2n : n Z} + { + 2n : n Z} The same is not true of the principal argument. In general, Arg(z) = Arg(z) + Arg(). Consider the case z = = e3/4 . Then Arg(z) = Arg() = 3/4, however, Arg(z) = /2.

z+ =(x+ )+i(y+ ) =+i z=x+iy

z =(xy )+i(x+y ) =r e i(+) =+i=ei z=x+iy z=x+iy =rei i =re

z=xiy =re i(+ )


Figure 6.6: Addition, Negation and Multiplication

1 Multiplicative Inverse. Assume that z is nonzero. The multiplicative inverse of z = r e is z = 1 e . The r 1 1 length of z is the multiplicative inverse of the length of z. The angle of z is the negative of the angle of z. (See Figure 6.7.)

194

r Division. Assume that is nonzero. The quotient of z = r e and = e is z = e() . The length of the vector z is the quotient of the lengths of z and . The angle of z is the dierence of the angles of z and . (See Figure 6.7.)

Complex Conjugate. The complex conjugate of z = x + y = r e is z = x y = r e . z is the mirror image of z, reected across the x axis. In other words, z has the same magnitude as z and the angle of z is the negative of the angle of z. (See Figure 6.7.)

= e i z=re i z=re i z r _ = _ e i () _ z=xiy=rei z=x+iy=re i

1 1 _ = e i z r

Figure 6.7: Multiplicative Inverse, Division and Complex Conjugate

6.5

Integer Exponents

Consider the product (a + b)n , n Z. If we know arctan(a, b) then it will be most convenient to expand the product working in polar form. If not, we can write n in base 2 to eciently do the multiplications. 195

Example 6.5.1 Suppose that we want to write

in Cartesian form.6 We can do the multiplication directly. 2n Note that 20 is 10100 in base 2. That is, 20 = 24 + 22 . We rst calculate the powers of the form 3+ by successive squaring. 2 3 + = 2 + 2 3 4 3 + = 8 + 8 3 8 3 + = 128 128 3 16 3+ = 32768 + 32768 3 3+ Next we multiply 3+
4

20

and
20

3+

16

to obtain the answer.

= 32768 + 32768 3 8 + 8 3 = 524288 524288 3 Since we know that arctan 3, 1 = /6, it is easiest to do this problem by rst changing to modulus-argument form. 3+
20

3+

3
20

+ 12 e arctan(

20 3,1)

= 2 e/6

= 220 e4/3

1 3 = 1048576 2 2 = 524288 524288 3


No, I have no idea why we would want to do that. Just humor me. If you pretend that youre interested, Ill do the same. Believe me, expressing your real feelings here isnt going to do anyone any good.
6

196

Example 6.5.2 Consider (5 + 7)11 . We will do the exponentiation in polar form and write the result in Cartesian form. 11 (5 + 7)11 = 74 e arctan(5,7) = 745 74(cos(11 arctan(5, 7)) + sin(11 arctan(5, 7))) = 2219006624 74 cos(11 arctan(5, 7)) + 2219006624 74 sin(11 arctan(5, 7)) The result is correct, but not very satisfying. This expression could be simplied. You could evaluate the trigonometric functions with some fairly messy trigonometric identities. This would take much more work than directly multiplying (5 + 7)11 .

6.6

Rational Exponents

In this section we consider complex numbers with rational exponents, z p/q , where p/q is a rational number. First we consider unity raised to the 1/n power. We dene 11/n as the set of numbers {z} such that z n = 1. 11/n = {z | z n = 1} We can nd these values by writing z in modulus-argument form. zn = 1 rn en = 1 rn = 1 n = 0 mod 2 r=1 = 2k for k Z 11/n = e2k/n | k Z There are only n distinct values as a result of the 2 periodicity of e . e2 = e0 . 11/n = e2k/n | k = 0, . . . , n 1 197

These values are equally spaced points on the unit circle in the complex plane. Example 6.6.1 11/6 has the 6 values, e0 , e/3 , e2/3 , e , e4/3 , e5/3 . In Cartesian form this is 1 + 3 1 + 3 1 3 1 3 1, , , 1, , 2 2 2 2

The sixth roots of unity are plotted in Figure 6.8.

-1 -1

Figure 6.8: The Sixth Roots of Unity. The nth roots of the complex number c = e are the set of numbers z = r e such that z n = c = e r= n rn en = e n = mod 2

r=

= ( + 2k)/n for k = 0, . . . , n 1. 198

Thus c1/n =

e(+2k)/n | k = 0, . . . , n 1 =

|c| e(Arg(c)+2k)/n | k = 0, . . . , n 1

Principal Roots. The principal nth root is denoted n z n z e Arg(z)/n . Thus the principal root has the property /n < Arg z /n. This is consistent with the notation from functions of a real variable: n x denotes the positive nth root of a positive real number. We adopt the convention that z 1/n denotes the nth roots of z, which is a set of n numbers and n z is the principal nth root of z, which is a single number. The nth roots of z are the principal nth root of z times the nth roots of unity. z 1/n = n r e(Arg(z)+2k)/n | k = 0, . . . , n 1 z 1/n = n z e2k/n | k = 0, . . . , n 1 z 1/n = n z11/n Rational Exponents. We interpret z p/q to mean z (p/q) . That is, we rst simplify the exponent, i.e. reduce the fraction, before carrying out the exponentiation. Therefore z 2/4 = z 1/2 and z 10/5 = z 2 . If p/q is a reduced fraction, (p and q are relatively prime, in other words, they have no common factors), then z p/q (z p )1/q . Thus z p/q is a set of q values. Note that for an un-reduced fraction r/s, (z r )1/s = z 1/s
r

.
1/2

The former expression is a set of s values while the latter is a set of no more that s values. For instance, (12 ) 2 11/2 = 1 and 11/2 = (1)2 = 1. 199

Example 6.6.2 Consider 21/5 , (1 + )1/3 and (2 + )5/6 . 5 21/5 = 2 e2k/5 , 6

for k = 0, 1, 2, 3, 4
1/3

(1 + )1/3 = = = =
12

2 e/4

2 e/12 e2k/3 ,
5/6

for k = 0, 1, 2

(2 + )5/6 =

5 e Arctan(2,1)

55 e5 Arctan(2,1)
5

1/6

55 e 6 Arctan(2,1) ek/3 ,

for k = 0, 1, 2, 3, 4, 5

Example 6.6.3 We nd the roots of z 5 + 4. (4)1/5 = (4 e )1/5 5 = 4 e(1+2k)/5 ,

for k = 0, 1, 2, 3, 4

200

6.7

Exercises

Complex Numbers
Exercise 6.1 If z = x + y, write the following in the form a + b: 1. (1 + 2)7 2. 3. 1 zz z + z (3 + )9

Hint, Solution Exercise 6.2 Verify that: 1. 1 + 2 2 2 + = 3 4 5 5

2. (1 )4 = 4 Hint, Solution Exercise 6.3 Write the following complex numbers in the form a + b. 1. 1+ 3
10

2. (11 + 4)2 Hint, Solution 201

Exercise 6.4 Write the following complex numbers in the form a + b 1. 2+ 6 (1 2)


2

2. (1 )7 Hint, Solution Exercise 6.5 If z = x + y, write the following in the form u(x, y) + v(x, y). 1. 2. z z

z + 2 2 z Hint, Solution Exercise 6.6 Quaternions are sometimes used as a generalization of complex numbers. A quaternion u may be dened as u = u0 + u1 + u2 + ku3 where u0 , u1 , u2 and u3 are real numbers and , and k are objects which satisfy 2 = 2 = k 2 = 1, = k, = k

and the usual associative and distributive laws. Show that for any quaternions u, w there exists a quaternion v such that uv = w except for the case u0 = u1 = u2 = u3 . Hint, Solution 202

Exercise 6.7 Let = 0, = 0 be two complex numbers. Show that = t for some real number t (i.e. the vectors dened by and are parallel) if and only if = 0. Hint, Solution

The Complex Plane


Exercise 6.8 Find and depict all values of 1. (1 + )1/3 2. 1/4 Identify the principal root. Hint, Solution Exercise 6.9 Sketch the regions of the complex plane: 1. | (z)| + 2| (z)| 1 2. 1 |z | 2 3. |z | |z + | Hint, Solution Exercise 6.10 Prove the following identities. 1. arg(z) = arg(z) + arg() 2. Arg(z) = Arg(z) + Arg() 203

3. arg (z 2 ) = arg(z) + arg(z) = 2 arg(z) Hint, Solution Exercise 6.11 Show, both by geometric and algebraic arguments, that for complex numbers z1 and z2 the inequalities ||z1 | |z2 || |z1 + z2 | |z1 | + |z2 | hold. Hint, Solution Exercise 6.12 Find all the values of 1. (1)3/4 2. 81/6 and show them graphically. Hint, Solution Exercise 6.13 Find all values of 1. (1)1/4 2. 161/8 and show them graphically. Hint, Solution Exercise 6.14 Sketch the regions or curves described by 204

1. 1 < |z 2| < 2 2. | (z)| + 5| (z)| = 1 3. |z | = |z + | Hint, Solution Exercise 6.15 Sketch the regions or curves described by 1. |z 1 + | 1 2. (z) (z) = 5

3. |z | + |z + | = 1 Hint, Solution Exercise 6.16 Solve the equation | e 1| = 2 for (0 ) and verify the solution geometrically. Hint, Solution

Polar Form
Exercise 6.17 Show that Eulers formula, e = cos + sin , is formally consistent with the standard Taylor series expansions for the real functions ex , cos x and sin x. Consider the Taylor series of ex about x = 0 to be the denition of the exponential function for complex argument. Hint, Solution 205

Exercise 6.18 Use de Moivres formula to derive the trigonometric identity cos(3) = cos3 () 3 cos() sin2 (). Hint, Solution Exercise 6.19 Establish the formula 1 z n+1 , 1z for the sum of a nite geometric series; then derive the formulas 1 + z + z2 + + zn = 1. 1 + cos() + cos(2) + + cos(n) = 2. sin() + sin(2) + + sin(n) = where 0 < < 2. Hint, Solution 1 sin((n + 1/2)) + 2 2 sin(/2) (z = 1),

cos((n + 1/2)) 1 cot 2 2 2 sin(/2)

Arithmetic and Vectors


Exercise 6.20 Prove |z1 z2 | = |z1 ||z2 | and Hint, Solution Exercise 6.21 Prove that |z + |2 + |z |2 = 2 |z|2 + ||2 . Interpret this geometrically. Hint, Solution 206
z1 z2

|z1 | |z2 |

using polar form.

Integer Exponents
Exercise 6.22 Write (1 + )10 in Cartesian form with the following two methods: 1. Just do the multiplication. If it takes you more than four multiplications, you suck. 2. Do the multiplication in polar form. Hint, Solution

Rational Exponents
Exercise 6.23 1/2 Show that each of the numbers z = a + (a2 b) satises the equation z 2 + 2az + b = 0. Hint, Solution

207

6.8

Hints

Complex Numbers
Hint 6.1 Hint 6.2 Hint 6.3 Hint 6.4 Hint 6.5 Hint 6.6 Hint 6.7

The Complex Plane


Hint 6.8 Hint 6.9

208

Hint 6.10 Write the multivaluedness explicitly. Hint 6.11 Consider a triangle with vertices at 0, z1 and z1 + z2 . Hint 6.12 Hint 6.13 Hint 6.14 Hint 6.15 Hint 6.16

Polar Form
Hint 6.17 Find the Taylor series of e , cos and sin . Note that 2n = (1)n . Hint 6.18 Hint 6.19

Arithmetic and Vectors


209

Hint 6.20 | e | = 1. Hint 6.21 Consider the parallelogram dened by z and .

Integer Exponents
Hint 6.22 For the rst part, (1 + )10 = (1 + )2
2 2

(1 + )2 .

Rational Exponents
Hint 6.23 Substitite the numbers into the equation.

210

6.9

Solutions

Complex Numbers
Solution 6.1 1. We can do the exponentiation by directly multiplying. (1 + 2)7 = (1 + 2)(1 + 2)2 (1 + 2)4 = (1 + 2)(3 + 4)(3 + 4)2 = (11 2)(7 24) = 29 + 278 We can also do the problem using De Moivres Theorem. (1 + 2)7 = 5 e arctan(1,2) = 125 5 e7 arctan(1,2) = 125 5 cos(7 arctan(1, 2)) + 125 5 sin(7 arctan(1, 2))
7

2. 1 1 = zz (x y)2 (x + y)2 1 = (x y)2 (x + y)2 (x + y)2 = 2 (x + y 2 )2 2xy x2 y 2 = 2 + 2 2 )2 (x + y (x + y 2 )2 211

3. We can evaluate the expression using De Moivres Theorem.

z + z = (y + x + x y)(3 + )9 9 (3 + ) = (1 + )(x y) 10 e arctan(3,1) = (1 + )(x y)

1 e9 arctan(3,1) 10000 10 (1 + )(x y) = (cos(9 arctan(3, 1)) sin(9 arctan(3, 1))) 10000 10 (x y) (cos(9 arctan(3, 1)) + sin(9 arctan(3, 1))) = 10000 10 (x y) (cos(9 arctan(3, 1)) sin(9 arctan(3, 1))) + 10000 10 212

We can also do this problem by directly multiplying but its a little grungy. z + z (y + x + x y)(3 )9 = (3 + )9 109 (1 + )(x y)(3 ) ((3 )2 ) = = = = = = Solution 6.2 1. 1 + 2 2 1 + 2 3 + 4 2 + = + 3 4 5 3 4 3 + 4 5 5 + 10 1 2 = + 25 5 2 = 5 2. (1 )4 = (2)2 = 4 213
2 2

109 2 (1 + )(x y)(3 ) (8 6)2 109 (1 + )(x y)(3 )(28 96)2 109 (1 + )(x y)(3 )(8432 5376) 109 (x y)(22976 38368) 109 359(y x) 1199(y x) + 15625000 31250000

Solution 6.3 1. First we do the multiplication in Cartesian form.

1+ 3

10

= = = =

1+ 3

1+ 3

2 + 2 3 2 + 2 3 2 + 2 3

2 + 2 3 8 8 3

128 + 128 3
1

= 512 512 3 =

1 1 512 1 + 3 1 1 1 3 = 512 1 + 3 1 3 1 3 = + 2048 2048 214

Now we do the multiplication in modulus-argument, (polar), form. 1+ 3


10

= 2 e/3

10

= 210 e10/3 1 10 10 = cos + sin 1024 3 3 4 4 1 = cos sin 1024 3 3 1 3 1 + = 2 1024 2 1 3 = + 2048 2048 2. (11 + 4)2 = 105 + 88 Solution 6.4 1. 2+ 6 (1 2)
2

2+ = 1 + 8 3 + 4 = 63 16 3 + 4 63 + 16 = 63 16 63 + 16 253 204 = 4225 4225

215

2.

(1 )7 = (1 )2 (1 )2 (1 ) = (2)2 (2)(1 ) = (4)(2 2) = 8 + 8

Solution 6.5 1.

z z

= =

x + y x + y

x y x + y x + y = x y x + y x + y = x y x + y 2xy x2 y 2 = 2 + 2 x + y2 x + y2 216

2. x + y + 2 z + 2 = 2 z 2 (x y) x + (y + 2) = 2 y x x + (y + 2) 2 y + x = 2 y x 2 y + x x(2 y) (y + 2)x x2 + (y + 2)(2 y) = + (2 y)2 + x2 (2 y)2 + x2 2xy 4 + x2 y 2 = + (2 y)2 + x2 (2 y)2 + x2 Solution 6.6 Method 1. We expand the equation uv = w in its components. uv = w (u0 + u1 + u2 + ku3 ) (v0 + v1 + v2 + kv3 ) = w0 + w1 + w2 + kw3 (u0 v0 u1 v1 u2 v2 u3 v3 ) + (u1 v0 + u0 v1 u3 v2 + u2 v3 ) + (u2 v0 + u3 v1 + u0 v2 u1 v3 ) + k (u3 v0 u2 v1 + u1 v2 + u0 v3 ) = w0 + w1 + w2 + kw3 We can write this as a matrix equation.

u0 u1 u2 u3 v0 w0 u1 u0 u3 u2 v1 w1 = u 2 u 3 u0 u1 v2 w2 u3 u2 u1 u0 v3 w3

This linear system of equations has a unique solution for v if and only if the determinant of the matrix is nonzero. The 2 determinant of the matrix is (u2 + u2 + u2 + u2 ) . This is zero if and only if u0 = u1 = u2 = u3 = 0. Thus there 0 1 2 3 217

exists a unique v such that uv = w if u is nonzero. This v is v = (u0 w0 + u1 w1 + u2 w2 + u3 w3 ) + (u1 w0 + u0 w1 + u3 w2 u2 w3 ) + (u2 w0 u3 w1 + u0 w2 + u1 w3 ) + k (u3 w0 + u2 w1 u1 w2 + u0 w3 ) / u2 + u2 + u2 + u2 0 1 2 3 Method 2. Note that uu is a real number. uu = (u0 u1 u2 ku3 ) (u0 + u1 + u2 + ku3 ) = u2 + u2 + u2 + u2 + (u0 u1 u1 u0 u2 u3 + u3 u2 ) 0 1 2 3 + (u0 u2 + u1 u3 u2 u0 u3 u1 ) + k (u0 u3 u1 u2 + u2 u1 u3 u0 ) = u2 + u 2 + u 2 + u2 0 1 2 3 uu = 0 only if u = 0. We solve for v by multiplying by the conjugate of u and dividing by uu. uv = w uuv = uw uw v= uu (u0 u1 u2 ku3 ) (w0 + w1 + w2 + kw3 ) v= u2 + u2 + u 2 + u2 0 1 2 3 v = (u0 w0 + u1 w1 + u2 w2 + u3 w3 ) + (u1 w0 + u0 w1 + u3 w2 u2 w3 ) + (u2 w0 u3 w1 + u0 w2 + u1 w3 ) + k (u3 w0 + u2 w1 u1 w2 + u0 w3 ) / u2 + u2 + u2 + u2 0 1 2 3 Solution 6.7 If = t, then = t||2 , which is a real number. Hence = 0. Now assume that = 0. This implies that = r for some r R. We multiply by and simplify. ||2 = r r = ||2 By taking t =
r ||2

We see that = t for some real number t. 218

The Complex Plane


Solution 6.8 1. (1 + )1/3 = = = 6 6 2 e/4
1/3

2 e/12 11/3

2 e/12 e2k/3 , k = 0, 1, 2 /12 3/4 17/12 6 6 6 = 2e , 2e , 2e The principal root is The roots are depicted in Figure 6.9. 2. 1/4 = e/2
1/4

1+=

2 e/12 .

= e/8 11/4 = e/8 e2k/4 , The principal root is The roots are depicted in Figure 6.10. Solution 6.9 1. | (z)| + 2| (z)| 1 |x| + 2|y| 1 219 k = 0, 1, 2, 3 = e/8 , e5/8 , e9/8 , e13/8 4 = e/8 .

-1

-1

Figure 6.9: (1 + )1/3

In the rst quadrant, this is the triangle below the line y = (1x)/2. We reect this triangle across the coordinate axes to obtain triangles in the other quadrants. Explicitly, we have the set of points: {z = x + y | 1 x 1 |y| (1 |x|)/2}. See Figure 6.11.

2. |z | is the distance from the point in the complex plane. Thus 1 < |z | < 2 is an annulus centered at z = between the radii 1 and 2. See Figure 6.12.

3. The points which are closer to z = than z = are those points in the upper half plane. See Figure 6.13. Solution 6.10 Let z = r e and = e . 220

-1

-1

Figure 6.10: 1/4

1. arg(z) = arg(z) + arg() arg r e(+) = { + 2m} + { + 2n} { + + 2k} = { + + 2m}

2. Arg(z) = Arg(z) + Arg() Consider z = = 1. Arg(z) = Arg() = , however Arg(z) = Arg(1) = 0. The identity becomes 0 = 2. 221

Figure 6.11: | (z)| + 2| (z)| 1

4 3 2 1 -3 -2 -1 -1 -2
Figure 6.12: 1 < |z | < 2

1 2 3

222

Figure 6.13: The upper half plane. 3. arg z 2 = arg(z) + arg(z) = 2 arg(z) arg r2 e2 = { + 2k} + { + 2m} = 2{ + 2n} {2 + 2k} = {2 + 2m} = {2 + 4n} Solution 6.11 Consider a triangle in the complex plane with vertices at 0, z1 and z1 + z2 . (See Figure 6.14.) The lengths of the sides of the triangle are |z1 |, |z2 | and |z1 + z2 | The second inequality shows that one side of the triangle must be less than or equal to the sum of the other two sides. |z1 + z2 | |z1 | + |z2 | The rst inequality shows that the length of one side of the triangle must be greater than or equal to the dierence in 223

z1 |z | 1

|z2|

z +z2 1

|z +z2| 1

Figure 6.14: Triangle Inequality

the length of the other two sides. |z1 + z2 | ||z1 | |z2 ||

Now we prove the inequalities algebraically. We will reduce the inequality to an identity. Let z1 = r1 e1 , z2 = r2 e2 . ||z1 | |z2 || |z1 + z2 | |z1 | + |z2 | |r1 r2 | |r1 e1 +r2 e2 | r1 + r2 (r1 r2 )2 r1 e1 +r2 e2 r1 e1 +r2 e2 (r1 + r2 )2
2 2 2 2 2 2 r1 + r2 2r1 r2 r1 + r2 + r1 r2 e(1 2 ) +r1 r2 e(1 +2 ) r1 + r2 + 2r1 r2 2r1 r2 2r1 r2 cos (1 2 ) 2r1 r2 1 cos (1 2 ) 1

224

Solution 6.12 1. (1)3/4 = (1)3 = (1)1/4 = (e )1/4 = e/4 11/4 = e/4 ek/2 ,
/4 3/4 1/4

k = 0, 1, 2, 3

= e ,e , e5/4 , e7/4 1 + 1 + 1 1 = , , , 2 2 2 2 See Figure 6.15. 2. 81/6 = = 6 811/6

2 ek/3 , k = 0, 1, 2, 3, 4, 5 /3 2/3 4/3 5/3 = 2, 2 e , 2 e , 2e , 2e , 2e 1 + 3 1 + 3 1 3 1 3 = 2, , , 2, , 2 2 2 2 See Figure 6.16.

225

-1

-1

Figure 6.15: (1)3/4 Solution 6.13 1. (1)1/4 = ((1)1 )1/4 = (1)1/4 = (e )1/4 = e/4 11/4 = e/4 ek/2 , k = 0, 1, 2, 3 = e/4 , e3/4 , e5/4 , e7/4 1 + 1 + 1 1 = , , , 2 2 2 2 See Figure 6.17. 226

2 1 -2 -1 -1 -2
Figure 6.16: 81/6 2. 161/8 = = 8 1611/8

2 ek/4 , k = 0, 1, 2, 3, 4, 5, 6, 7 /4 /2 3/4 5/4 3/2 7/4 = 2, 2 e , 2 e , 2 e , 2e , 2e , 2e , 2e = 2, 1 + , 2, 1 + , 2, 1 , 2, 1 See Figure 6.18. Solution 6.14 1. |z 2| is the distance from the point 2 in the complex plane. Thus 1 < |z 2| < 2 is an annulus. See Figure 6.19. 227

-1

-1

Figure 6.17: (1)1/4 2. | (z)| + 5| (z)| = 1 |x| + 5|y| = 1 In the rst quadrant this is the line y = (1 x)/5. We reect this line segment across the coordinate axes to obtain line segments in the other quadrants. Explicitly, we have the set of points: {z = x + y | 1 < x < 1 y = (1 |x|)/5}. See Figure 6.20. 3. The set of points equidistant from and is the real axis. See Figure 6.21. Solution 6.15 1. |z 1 + | is the distance from the point (1 ). Thus |z 1 + | 1 is the disk of unit radius centered at (1 ). See Figure 6.22. 228

-1 -1

Figure 6.18: 161/8

5 4 3 2 1 -3 -2 -1 -1 1 2 3

Figure 6.19: 1 < |z 2| < 2

229

0.4 0.2 -1 -0.2 -0.4 1

Figure 6.20: | (z)| + 5| (z)| = 1

-1

-1

Figure 6.21: |z | = |z + |

230

1 -1 -1 -2 -3
Figure 6.22: |z 1 + | < 1

2. (z) (z) = 5 xy =5 y =x5 See Figure 6.23.

3. Since |z | + |z + | 2, there are no solutions of |z | + |z + | = 1.

231

-10

-5 -5

10

-10

-15

Figure 6.23: Solution 6.16

(z) (z) = 5

| e 1| = 2 e 1 e 1 = 4 1 e e +1 = 4 2 cos() = 2 = e | 0 is a unit semi-circle in the upper half of the complex plane from 1 to 1. The only point on this semi-circle that is a distance 2 from the point 1 is the point 1, which corresponds to = .

Polar Form

232

Solution 6.17 We recall the Taylor series expansion of ex about x = 0.

ex =
n=0

xn . n!

We take this as the denition of the exponential function for complex argument.

e =
n=0

()n n! n n n! (1)n 2n (1)n 2n+1 + (2n)! (2n + 1)! n=0


=
n=0

=
n=0

We compare this expression to the Taylor series for the sine and cosine.

cos =
n=0

(1)n 2n , (2n)!

sin =
n=0

(1)n 2n+1 , (2n + 1)!

Thus e and cos + sin have the same Taylor series expansions about = 0. e = cos + sin Solution 6.18 cos(3) + sin(3) = (cos() + sin())3 cos(3) + sin(3) = cos3 () + 3 cos2 () sin() 3 cos() sin2 () sin3 () We equate the real parts of the equation. cos(3) = cos3 () 3 cos() sin2 () 233

Solution 6.19 Dene the partial sum,


n

Sn (z) =
k=0

zk .

Now consider (1 z)Sn (z).


n

(1 z)Sn (z) = (1 z)
n

zk
k=0 n+1

(1 z)Sn (z) =
k=0

zk
k=1 n+1

zk

(1 z)Sn (z) = 1 z We divide by 1 z. Note that 1 z is nonzero.

1 z n+1 1z 1 z n+1 1 + z + z2 + + zn = , 1z Sn (z) = Now consider z = e where 0 < < 2 so that z is not unity.
n

(z = 1)

e
k=0 n

1 e = 1 e

n+1

ek =
k=0

1 e(n+1) 1 e

234

In order to get sin(/2) in the denominator, we multiply top and bottom by e/2 .
n

(cos(k) + sin(k)) =
k=0 n n

e/2 e(n+1/2) e/2 e/2

cos(k) +
k=0 n k=0 n

sin(k) = sin(k) =
k=1

cos(/2) sin(/2) cos((n + 1/2)) sin((n + 1/2)) 2 sin(/2) 1 sin((n + 1/2)) + + 2 sin(/2) 1 cos((n + 1/2)) cot(/2) 2 sin(/2)

cos(k) +
k=0

1. We take the real and imaginary part of this to obtain the identities.
n

cos(k) =
k=0

1 sin((n + 1/2)) + 2 2 sin(/2)

2.
n

sin(k) =
k=1

1 cos((n + 1/2)) cot(/2) 2 2 sin(/2)

Arithmetic and Vectors


Solution 6.20 |z1 z2 | = |r1 e1 r2 e2 | = |r1 r2 e(1 +2 ) | = |r1 r2 | = |r1 ||r2 | = |z1 ||z2 | 235

z1 r1 e1 = z2 r2 e2 r1 (1 2 ) e = r2 r1 = r2 |r1 | = |r2 | |z1 | = |z2 |

Solution 6.21

|z + |2 + |z |2 = (z + ) z + + (z ) z = zz + z + z + + zz z z + = 2 |z|2 + ||2

Consider the parallelogram dened by the vectors z and . The lengths of the sides are z and and the lengths of the diagonals are z + and z . We know from geometry that the sum of the squared lengths of the diagonals of a parallelogram is equal to the sum of the squared lengths of the four sides. (See Figure 6.24.)

Integer Exponents

236

z- z z+

Figure 6.24: The parallelogram dened by z and . Solution 6.22 1. (1 + )10 = (1 + )2


2 2 2

(1 + )2

= (2)2

(2)

= (4)2 (2) = 16(2) = 32 2. (1 + )10 = = 2 e/4


10 10

e10/4

= 32 e/2 = 32 237

Rational Exponents
Solution 6.23 We substitite the numbers into the equation to obtain an identity. z 2 + 2az + b = 0 a + a2 b a2 2a a2 b
1/2 2

+ 2a a + a2 b

1/2

+b=0
1/2

1/2

+ a2 b 2a2 + 2a a2 b 0=0

+b=0

238

Chapter 7 Functions of a Complex Variable


If brute force isnt working, youre not using enough of it. -Tim Mauch In this chapter we introduce the algebra of functions of a complex variable. We will cover the trigonometric and inverse trigonometric functions. The properties of trigonometric functions carry over directly from real-variable theory. However, because of multi-valuedness, the inverse trigonometric functions are signicantly trickier than their real-variable counterparts.

7.1

Curves and Regions

In this section we introduce curves and regions in the complex plane. This material is necessary for the study of branch points in this chapter and later for contour integration. Curves. Consider two continuous functions, x(t) and y(t), dened on the interval t [t0 . . . t1 ]. The set of points in the complex plane {z(t) = x(t) + y(t) | t [t0 . . . t1 ]} 239

denes a continuous curve or simply a curve. If the endpoints coincide, z (t0 ) = z (t1 ), it is a closed curve. (We assume that t0 = t1 .) If the curve does not intersect itself, then it is said to be a simple curve. If x(t) and y(t) have continuous derivatives and the derivatives do not both vanish at any point1 , then it is a smooth curve. This essentially means that the curve does not have any corners or other nastiness. A continuous curve which is composed of a nite number of smooth curves is called a piecewise smooth curve. We will use the word contour as a synonym for a piecewise smooth curve. See Figure 7.1 for a smooth curve, a piecewise smooth curve, a simple closed curve and a non-simple closed curve.

(a)

(b)

(c)

(d)

Figure 7.1: (a) Smooth Curve, (b) Piecewise Smooth Curve, (c) Simple Closed Curve, (d) Non-Simple Closed Curve Regions. A region R is connected if any two points in R can be connected by a curve which lies entirely in R. A region is simply-connected if every closed curve in R can be continuously shrunk to a point without leaving R. A region which is not simply-connected is said to be multiply-connected region. Another way of dening simply-connected is that a path connecting two points in R can be continuously deformed into any other path that connects those points. Figure 7.2 shows a simply-connected region with two paths which can be continuously deformed into one another and a multiply-connected region with paths which cannot be deformed into one another. Jordan Curve Theorem. A continuous, simple, closed curve is known as a Jordan curve. The Jordan Curve Theorem, which seems intuitively obvious but is dicult to prove, states that a Jordan curve divides the plane into
1

Why is it necessary that the derivatives do not both vanish?

240

Figure 7.2: Simply-connected and multiply-connected regions. a simply-connected, bounded region and an unbounded region. These two regions are called the interior and exterior regions, respectively. The two regions share the curve as a boundary. Points in the interior are said to be inside the curve; points in the exterior are said to be outside the curve.

Traversal of a Contour. Consider a Jordan curve. If you traverse the curve in the positive direction, then the inside is to your left. If you traverse the curve in the opposite direction, then the outside will be to your left and you will go around the curve in the negative direction. For circles, the positive direction is the counter-clockwise direction. The positive direction is consistent with the way angles are measured in a right-handed coordinate system, i.e. for a circle centered on the origin, the positive direction is the direction of increasing angle. For an oriented contour C, we denote the contour with opposite orientation as C.

Boundary of a Region. Consider a simply-connected region. The boundary of the region is traversed in the positive direction if the region is to the left as you walk along the contour. For multiply-connected regions, the boundary may be a set of contours. In this case the boundary is traversed in the positive direction if each of the contours is traversed in the positive direction. When we refer to the boundary of a region we will assume it is given the positive orientation. In Figure 7.3 the boundaries of three regions are traversed in the positive direction. 241

Figure 7.3: Traversing the boundary in the positive direction. Two Interpretations of a Curve. Consider a simple closed curve as depicted in Figure 7.4a. By giving it an orientation, we can make a contour that either encloses the bounded domain Figure 7.4b or the unbounded domain Figure 7.4c. Thus a curve has two interpretations. It can be thought of as enclosing either the points which are inside or the points which are outside.2

7.2

The Point at Innity and the Stereographic Projection

Complex Innity. In real variables, there are only two ways to get to innity. We can either go up or down the number line. Thus signed innity makes sense. By going up or down we respectively approach + and . In the complex plane there are an innite number of ways to approach innity. We stand at the origin, point ourselves in any direction and go straight. We could walk along the positive real axis and approach innity via positive real numbers. We could walk along the positive imaginary axis and approach innity via pure imaginary numbers. We could generalize the real variable notion of signed innity to a complex variable notion of directional innity, but this will not be useful
A farmer wanted to know the most ecient way to build a pen to enclose his sheep, so he consulted an engineer, a physicist and a mathematician. The engineer suggested that he build a circular pen to get the maximum area for any given perimeter. The physicist suggested that he build a fence at innity and then shrink it to t the sheep. The mathematician constructed a little fence around himself and then dened himself to be outside.
2

242

(a)

(b)

(c)

Figure 7.4: Two interpretations of a curve. for our purposes. Instead, we introduce complex innity or the point at innity as the limit of going innitely far along any direction in the complex plane. The complex plane together with the point at innity form the extended complex plane. Stereographic Projection. We can visualize the point at innity with the stereographic projection. We place a unit sphere on top of the complex plane so that the south pole of the sphere is at the origin. Consider a line passing through the north pole and a point z = x + y in the complex plane. In the stereographic projection, the point point z is mapped to the point where the line intersects the sphere. (See Figure 7.5.) Each point z = x + y in the complex plane is mapped to a unique point (X, Y, Z) on the sphere. 4x X= 2 , |z| + 4 4y Y = 2 , |z| + 4 2|z|2 Z= 2 |z| + 4

The origin is mapped to the south pole. The point at innity, |z| = , is mapped to the north pole. In the stereographic projection, circles in the complex plane are mapped to circles on the unit sphere. Figure ?? shows circles along the real and imaginary axes under the mapping. Lines in the complex plane are also mapped to circles on the unit sphere. The right diagram in Figure ?? shows lines emanating from the origin under the mapping. 243

Figure 7.5: The stereographic projection.

244

Figure 7.6: The stereographic projection of circles and lines.

245

7.3

Cartesian and Modulus-Argument Form

We can write a function of a complex variable z as a function of x and y or as a function of r and with the substitutions z = x + y and z = r e , respectively. Then we can separate the real and imaginary components or write the function in modulus-argument form, f (z) = u(x, y) + v(x, y), f (z) = (x, y) e(x,y) , or f (z) = u(r, ) + v(r, ), or f (z) = (r, ) e(r,) .
1 . 1z

Example 7.3.1 Consider the functions f (z) = z, f (z) = z 3 and f (z) = and y and separate them into their real and imaginary components. f (z) = z = x + y f (z) = z 3 = (x + y)3 = x3 + x2 y xy 2 y 3 = x3 xy 2 + x2 y y 3 f (z) =

We write the functions in terms of x

1 1z 1 = 1 x y 1 1 x + y = 1 x y 1 x + y y 1x = + 2 + y2 (1 x) (1 x)2 + y 2 246

Example 7.3.2 Consider the functions f (z) = z, f (z) = z 3 and f (z) = and and write them in modulus-argument form. f (z) = z = r e f (z) = z 3 = r e = r3 e3 f (z) = = = = = 1 1z 1 1 r e 1 1 1 r e 1re 1 r e 1 r e r e +r2 1 r cos + r sin 1 2r cos + r2
3

1 . 1z

We write the functions in terms of r

Note that the denominator is real and non-negative. 1 |1 r cos + r sin | e arctan(1r cos ,r sin ) 2 1 2r cos + r 1 = (1 r cos )2 + r2 sin2 e arctan(1r cos ,r sin ) 2 1 2r cos + r 1 = 1 2r cos + r2 cos2 + r2 sin2 e arctan(1r cos ,r sin ) 1 2r cos + r2 1 e arctan(1r cos ,r sin ) = 2 1 2r cos + r = 247

7.4

Graphing Functions of a Complex Variable

We cannot directly graph functions of a complex variable as they are mappings from R2 to R2 . To do so would require four dimensions. However, we can can use a surface plot to graph the real part, the imaginary part, the modulus or the argument of a function of a complex variable. Each of these are scalar elds, mappings from R2 to R. Example 7.4.1 Consider the identity function, f (z) = z. In Cartesian coordinates and Cartesian form, the function is f (z) = x + y. The real and imaginary components are u(x, y) = x and v(x, y) = y. (See Figure 7.7.) In modulus

2 1 0 -1 -2 -2

2 -1 1 0 y -1 2-2

2 1 0 -1 -2 -2

x0

-1

x0

2 1 0 y -1 2-2

Figure 7.7: The real and imaginary parts of f (z) = z = x + y argument form the function is f (z) = z = r e = x2 + y 2 e arctan(x,y) . The modulus of f (z) is a single-valued function which is the distance from the origin. The argument of f (z) is a multivalued function. Recall that arctan(x, y) has an innite number of values each of which dier by an integer multiple of 2. A few branches of arg(f (z)) are plotted in Figure 7.8. The modulus and principal argument of f (z) = z are plotted in Figure 7.9. Example 7.4.2 Consider the function f (z) = z 2 . In Cartesian coordinates and separated into its real and imaginary 248

-1 -2 5 0 -5 -2

y 12 0

-1 x

Figure 7.8: A Few Branches of arg(z)

2 1 0 -2

-1 x

2 1 0y -1 2-2

2 0 -2 -2

2 1 0y -1 -1 0 x 1 2 -2

Figure 7.9: Plots of |z| and Arg(z)

249

components the function is f (z) = z 2 = (x + y)2 = x2 y 2 + 2xy. Figure 7.10 shows surface plots of the real and imaginary parts of z 2 . The magnitude of z 2 is

4 2 0 -2 -4 -2 -1 x 0 1 2 -2 -1

2 1 0 y

5 0 -5 -2 -1 x 0 1 2 -2 -1

2 1 0 y

Figure 7.10: Plots of

(z 2 ) and

(z 2 )

|z 2 | = Note that

z 2 z 2 = zz = (x + y)(x y) = x2 + y 2 .

z 2 = r e In Figure 7.11 are plots of |z 2 | and a branch of arg (z 2 ).

= r2 e2 .

250

8 6 4 2 0 -2 -1 x 0 1 2 -2 -1

5 2 1 0 y 0 -5 -2 -1 x 0 1 2 -2 -1 1 0 y 2

Figure 7.11: Plots of |z 2 | and a branch of arg (z 2 )

7.5

Trigonometric Functions

The Exponential Function. Consider the exponential function ez . We can use Eulers formula to write ez = ex+y in terms of its real and imaginary parts. ez = ex+y = ex ey = ex cos y + ex sin y From this we see that the exponential function is 2 periodic: ez+2 = ez , and odd periodic: ez+ = ez . Figure 7.12 has surface plots of the real and imaginary parts of ez which show this periodicity. The modulus of ez is a function of x alone. |ez | = ex+y = ex The argument of ez is a function of y alone. arg (ez ) = arg ex+y = {y + 2n | n Z} In Figure 7.13 are plots of | ez | and a branch of arg (ez ). 251

20 10 0 -10 -20 -2 x 0 2 -5

5 0 y

20 10 0 -10 -20 -2 x 0 2 -5

5 0 y

Figure 7.12: Plots of

(ez ) and

(ez )

20 15 10 5 0 -2 x0 2 -5

5 0 y

5 0 -5 -2 x0 2 -5

5 0 y

Figure 7.13: Plots of | ez | and a branch of arg (ez )

252

Example 7.5.1 Show that the transformation w = ez maps the innite strip, < x < , 0 < y < , onto the upper half-plane. Method 1. Consider the line z = x + c, < x < . Under the transformation, this is mapped to w = ex+c = ec ex , < x < .

This is a ray from the origin to innity in the direction of ec . Thus we see that z = x is mapped to the positive, real w axis, z = x + is mapped to the negative, real axis, and z = x + c, 0 < c < is mapped to a ray with angle c in the upper half-plane. Thus the strip is mapped to the upper half-plane. See Figure 7.14.

3 2 1 -3 -2 -1 1 2 3 -3 -2 -1

3 2 1 1 2 3

Figure 7.14: ez maps horizontal lines to rays. Method 2. Consider the line z = c + y, 0 < y < . Under the transformation, this is mapped to w = ec+y + ec ey , 0 < y < .

This is a semi-circle in the upper half-plane of radius ec . As c , the radius goes to zero. As c , the radius goes to innity. Thus the strip is mapped to the upper half-plane. See Figure 7.15.

253

3 2 1 -1 1 -3 -2 -1

3 2 1 1 2 3

Figure 7.15: ez maps vertical lines to circular arcs. The Sine and Cosine. We can write the sine and cosine in terms of the exponential function. ez + ez cos(z) + sin(z) + cos(z) + sin(z) = 2 2 cos(z) + sin(z) + cos(z) sin(z) = 2 = cos z ez ez cos(z) + sin(z) cos(z) sin(z) = 2 2 cos(z) + sin(z) cos(z) + sin(z) = 2 = sin z We separate the sine and cosine into their real and imaginary parts. cos z = cos x cosh y sin x sinh y sin z = sin x cosh y + cos x sinh y

For xed y, the sine and cosine are oscillatory in x. The amplitude of the oscillations grows with increasing |y|. See Figure 7.16 and Figure 7.17 for plots of the real and imaginary parts of the cosine and sine, respectively. Figure 7.18 shows the modulus of the cosine and the sine. 254

5 2.5 0 -2.5 -5 -2 x 0 2

2 1 0 y -1 -2

5 2.5 0 -2.5 -5 -2 x 0 2

2 1 0 y -1 -2

Figure 7.16: Plots of

(cos(z)) and

(cos(z))

5 2.5 0 -2.5 -5 -2 x 0 2

2 1 0 y -1 -2

5 2.5 0 -2.5 -5 -2 x 0 2

2 1 0 y -1 -2

Figure 7.17: Plots of

(sin(z)) and

(sin(z))

255

4 2 -2 0 x 2 -2 1

2 0 y -1

4 2 0 -2 0 x 2 -2 -1 1

2 0 y

Figure 7.18: Plots of | cos(z)| and | sin(z)|

The Hyperbolic Sine and Cosine. The hyperbolic sine and cosine have the familiar denitions in terms of the exponential function. Thus not surprisingly, we can write the sine in terms of the hyperbolic sine and write the cosine in terms of the hyperbolic cosine. Below is a collection of trigonometric identities. 256

Result 7.5.1 ez = ex (cos y + sin y) ez + ez ez ez cos z = sin z = 2 2 cos z = cos x cosh y sin x sinh y sin z = sin x cosh y + cos x sinh y ez + ez ez ez sinh z = cosh z = 2 2 cosh z = cosh x cos y + sinh x sin y sinh z = sinh x cos y + cosh x sin y sin(z) = sinh z sinh(z) = sin z cos(z) = cosh z cosh(z) = cos z log z = ln |z| + arg(z) = ln |z| + Arg(z) + 2n, n Z

7.6

Inverse Trigonometric Functions

The Logarithm. The logarithm, log(z), is dened as the inverse of the exponential function ez . The exponential function is many-to-one and thus has a multi-valued inverse. From what we know of many-to-one functions, we conclude that elog z = z, but log (ez ) = z. This is because elog z is single-valued but log (ez ) is not. Because ez is 2 periodic, the logarithm of a number is a set of numbers which dier by integer multiples of 2. For instance, e2n = 1 so that log(1) = {2n : n Z}. The logarithmic function has an innite number of branches. The value of the function on the branches diers by integer multiples of 2. It has singularities at zero and innity. | log(z)| as either z 0 or z . We will derive the formula for the complex variable logarithm. For now, let ln(x) denote the real variable logarithm that is dened for positive real numbers. Consider w = log z. This means that ew = z. We write w = u + v in 257

Cartesian form and z = r e in polar form. eu+v = r e We equate the modulus and argument of this expression. eu = r u = ln r With log z = u + v, we have a formula for the logarithm. log z = ln |z| + arg(z) If we write out the multi-valuedness of the argument function we note that this has the form that we expected. log z = ln |z| + (Arg(z) + 2n), We check that our formula is correct by showing that elog z = z elog z = eln |z|+ arg(z) = eln r++2n = r e = z Note again that log (ez ) = z. log (ez ) = ln | ez | + arg (ez ) = ln (ex ) + arg ex+y = x + (y + 2n) = z + 2n = z The real part of the logarithm is the single-valued ln r; the imaginary part is the multi-valued arg(z). We dene the principal branch of the logarithm Log z to be the branch that satises < (Log z) . For positive, real numbers the principal branch, Log x is real-valued. We can write Log z in terms of the principal argument, Arg z. Log z = ln |z| + Arg(z) See Figure 7.19 for plots of the real and imaginary part of Log z. 258 nZ v = + 2n v = + 2n

1 0 -1 -2 -2 -1 x 0 1 2 -2 -1

2 1 0 y

2 0 -2 -2 -1 x 0 1 2 -2 -1

2 1 0 y

Figure 7.19: Plots of

(Log z) and

(Log z).

The Form: ab . Consider ab where a and b are complex and a is nonzero. We dene this expression in terms of the exponential and the logarithm as ab = eb log a . Note that the multi-valuedness of the logarithm may make ab multi-valued. First consider the case that the exponent is an integer. am = em log a = em(Log a+2n) = em Log a e2mn = em Log a Thus we see that am has a single value where m is an integer. Now consider the case that the exponent is a rational number. Let p/q be a rational number in reduced form. ap/q = e q log a = e q (Log a+2n) = e q Log a e2np/q . This expression has q distinct values as e2np/q = e2mp/q if and only if n = m 259 mod q.
p p p

Finally consider the case that the exponent b is an irrational number. ab = eb log a = eb(Log a+2n) = eb Log a e2bn Note that e2bn and e2bm are equal if and only if 2bn and 2bm dier by an integer multiple of 2, which means that bn and bm dier by an integer. This occurs only when n = m. Thus e2bn has a distinct value for each dierent integer n. We conclude that ab has an innite number of values. You may have noticed something a little shy. If b is not an integer and a is any non-zero complex number, then b a is multi-valued. Then why have we been treating eb as single-valued, when it is merely the case a = e? The answer is that in the realm of functions of a complex variable, ez is an abuse of notation. We write ez when we mean exp(z), the single-valued exponential function. Thus when we write ez we do not mean the number e raised to the z power, we mean the exponential function of z. We denote the former scenario as (e)z , which is multi-valued. Logarithmic Identities. Back in high school trigonometry when you thought that the logarithm was only dened for positive real numbers you learned the identity log xa = a log x. This identity doesnt hold when the logarithm is dened for nonzero complex numbers. Consider the logarithm of z a . log z a = Log z a + 2n a log z = a(Log z + 2n) = a Log z + 2an Note that log z a = a log z Furthermore, since Log z a = ln |z a | + Arg (z a ) , a Log z = a ln |z| + a Arg(z) and Arg (z a ) is not necessarily the same as a Arg(z) we see that Log z a = a Log z. 260

Consider the logarithm of a product. log(ab) = ln |ab| + arg(ab) = ln |a| + ln |b| + arg(a) + arg(b) = log a + log b There is not an analogous identity for the principal branch of the logarithm since Arg(ab) is not in general the same as Arg(a) + Arg(b). n Using log(ab) = log(a) + log(b) we can deduce that log (an ) = k=1 log a = n log a, where n is a positive integer. This result is simple, straightforward and wrong. I have led you down the merry path to damnation.3 In fact, log (a2 ) = 2 log a. Just write the multi-valuedness explicitly, log a2 = Log a2 + 2n, You can verify that log 1 a = log a. 2 log a = 2(Log a + 2n) = 2 Log a + 4n.

We can use this and the product identity to expand the logarithm of a quotient. log a = log a log b b

For general values of a, log z a = a log z. However, for some values of a, equality holds. We already know that a = 1 and a = 1 work. To determine if equality holds for other values of a, we explicitly write the multi-valuedness. log z a = log ea log z = a log z + 2k, k Z a log z = a ln |z| + a Arg z + a2m, m Z
3

Dont feel bad if you fell for it. The logarithm is a tricky bastard.

261

We see that log z a = a log z if and only if {am | m Z} = {am + k | k, m Z}. The sets are equal if and only if a = 1/n, n Z . Thus we have the identity: log z 1/n = 1 log z, n n Z

Result 7.6.1 Logarithmic Identities. ab = eb log a elog z = eLog z = z log(ab) = log a + log b log(1/a) = log a log(a/b) = log a log b 1 log z 1/n = log z, n Z n Logarithmic Inequalities. Log(uv) = Log(u) + Log(v) log z a = a log z Log z a = a Log z log ez = z

262

Example 7.6.1 Consider 1 . We apply the denition ab = eb log a . 1 = e log(1) = e(ln(1)+2n) = e2n
2

Thus we see that 1 has an innite number of values, all of which lie on the unit circle |z| = 1 in the complex plane. However, the set 1 is not equal to the set |z| = 1. There are points in the latter which are not in the former. This is analogous to the fact that the rational numbers are dense in the real numbers, but are a subset of the real numbers. Example 7.6.2 We nd the zeros of sin z. sin z = ez ez =0 2 ez = ez e2z = 1 2z mod 2 = 0 z = n, Equivalently, we could use the identity sin z = sin x cosh y + cos x sinh y = 0. This becomes the two equations (for the real and imaginary parts) sin x cosh y = 0 and cos x sinh y = 0. nZ

Since cosh is real-valued and positive for real argument, the rst equation dictates that x = n, n Z. Since cos(n) = (1)n for n Z, the second equation implies that sinh y = 0. For real argument, sinh y is only zero at y = 0. Thus the zeros are z = n, n Z 263

Example 7.6.3 Since we can express sin z in terms of the exponential function, one would expect that we could express the sin1 z in terms of the logarithm.

w = sin1 z z = sin w ew ew z= 2 e2w 2z ew 1 = 0 ew = z 1 z 2 w = log z 1 z 2

Thus we see how the multi-valued sin1 is related to the logarithm.

sin1 z = log z

1 z2

264

Example 7.6.4 Consider the equation sin3 z = 1. sin3 z = 1 sin z = 11/3 ez ez = 11/3 2 ez 2(1)1/3 ez = 0 e2z 2(1)1/3 ez 1 = 0 ez = 2(1)1/3 4(1)2/3 + 4 2 1 (1)2/3 1 12/3

ez = (1)1/3

z = log (1)1/3

Note that there are three sources of multi-valuedness in the expression for z. The two values of the square root are shown explicitly. There are three cube roots of unity. Finally, the logarithm has an innite number of branches. To show this multi-valuedness explicitly, we could write z = Log e2m/3 1 e4m/3 + 2n, m = 0, 1, 2, n = . . . , 1, 0, 1, . . .

Example 7.6.5 Consider the harmless looking equation, z = 1. Before we start with the algebra, note that the right side of the equation is a single number. z is single-valued only when z is an integer. Thus we know that if there are solutions for z, they are integers. We now proceed to solve the equation. z = 1 e/2
z

=1

265

Use the fact that z is an integer. ez/2 = 1 z/2 = 2n, for some n Z z = 4n, nZ Here is a dierent approach. We write down the multi-valued form of z . We solve the equation by requiring that all the values of z are 1. z = 1 ez log = 1 z log = 2n, for some n Z z + 2m = 2n, m Z, for some n Z 2 z + 2mz = 2n, m Z, for some n Z 2 The only solutions that satisfy the above equation are z = 4k, k Z. Now lets consider a slightly dierent problem: 1 z . For what values of z does z have 1 as one of its values. 1 z 1 ez log 1 {ez(/2+2n) | n Z} z(/2 + 2n) = 2m, m, n Z z= 4m , 1 + 4n m, n Z

There are an innite set of rational numbers for which z has 1 as one of its values. For example, 4/5 = 11/5 = 1, e2/5 , e4/5 , e6/5 , e8/5

266

7.7

Riemann Surfaces

Consider the mapping w = log(z). Each nonzero point in the z-plane is mapped to an innite number of points in the w plane. w = {ln |z| + arg(z)} = {ln |z| + (Arg(z) + 2n) | n Z} This multi-valuedness makes it hard to work with the logarithm. We would like to select one of the branches of the logarithm. One way of doing this is to decompose the z-plane into an innite number of sheets. The sheets lie above one another and are labeled with the integers, n Z. (See Figure 7.20.) We label the point z on the nth sheet as (z, n). Now each point (z, n) maps to a single point in the w-plane. For instance, we can make the zeroth sheet map to the principal branch of the logarithm. This would give us the following mapping. log(z, n) = Log z + 2n

2 1 0 -1 -2

Figure 7.20: The z-plane decomposed into at sheets. This is a nice idea, but it has some problems. The mappings are not continuous. Consider the mapping on the zeroth sheet. As we approach the negative real axis from above z is mapped to ln |z| + as we approach from below it is mapped to ln |z| . (Recall Figure 7.19.) The mapping is not continuous across the negative real axis. 267

Lets go back to the regular z-plane for a moment. We start at the point z = 1 and selecting the branch of the logarithm that maps to zero. (log(1) = 2n). We make the logarithm vary continuously as we walk around the origin once in the positive direction and return to the point z = 1. Since the argument of z has increased by 2, the value of the logarithm has changed to 2. If we walk around the origin again we will have log(1) = 4. Our at sheet decomposition of the z-plane does not reect this property. We need a decomposition with a geometry that makes the mapping continuous and connects the various branches of the logarithm. Drawing inspiration from the plot of arg(z), Figure 7.8, we decompose the z-plane into an innite corkscrew with axis at the origin. (See Figure 7.21.) We dene the mapping so that the logarithm varies continuously on this surface. Consider a point z on one of the sheets. The value of the logarithm at that same point on sheet directly above it is 2 more than the original value. We call this surface, the Riemann surface for the logarithm. The mapping from the Riemann surface to the w-plane is continuous and one-to-one.

Figure 7.21: The Riemann surface for the logarithm.

268

7.8

Branch Points

Example 7.8.1 Consider the function z 1/2 . For each value of z, there are two values of z 1/2 . We write z 1/2 in modulus-argument and Cartesian form. z 1/2 = z 1/2 = |z| e arg(z)/2

|z| cos(arg(z)/2) + |z| sin(arg(z)/2)

Figure 7.22 shows the real and imaginary parts of z 1/2 from three dierent viewpoints. The second and third views are looking down the x axis and y axis, respectively. Consider z 1/2 . This is a double layered sheet which intersects itself on the negative real axis. ( (z 1/2 ) has a similar structure, but intersects itself on the positive real axis.) Lets start at a point on the positive real axis on the lower sheet. If we walk around the origin once and return to the positive real axis, we will be on the upper sheet. If we do this again, we will return to the lower sheet. Suppose we are at a point in the complex plane. We pick one of the two values of z 1/2 . If the function varies continuously as we walk around the origin and back to our starting point, the value of z 1/2 will have changed. We will be on the other branch. Because walking around the point z = 0 takes us to a dierent branch of the function, we refer to z = 0 as a branch point. Now consider the modulus-argument form of z 1/2 : z 1/2 = |z| e arg(z)/2 .

Figure 7.23 shows the modulus and the principal argument of z 1/2 . We see that each time we walk around the origin, the argument of z 1/2 changes by . This means that the value of the function changes by the factor e = 1, i.e. the function changes sign. If we walk around the origin twice, the argument changes by 2, so that the value of the function does not change, e2 = 1. 1/2 z 1/2 is a continuous function except at z = 0. Suppose we start at z = 1 = e0 and the function value (e0 ) = 1. If we follow the rst path in Figure 7.24, the argument of z varies from up to about , down to about and back 4 4 1/2 to 0. The value of the function is still (e0 ) . Now suppose we follow a circular path around the origin in the positive, counter-clockwise, direction. (See the 269

1 0 -1 -2 -1 0 x 1 2 -2

2 1 0 y -1

1 0 -1 -2 -1 0 x 1 2 -2

2 1 0 y -1

2 -1 -201 1 x 0 -1 2

2 -1 -201 1 x 0 -1 2

-2

-1

0 y

-2

-1

0 y

1210-2 -1 y 0 -1 2

1210-2 -1 y 0 -1 2

0 x

-1

-2

0 x

-1

-2

Figure 7.22: Plots of

z 1/2 (left) and 270

z 1/2 (right) from three viewpoints.

1 0.5 0 -2-1

2 1 0y -1 0 x 1 2 -2

2 0 -2 -2

2 1 0y -1 -1 0 x 1 2 -2

Figure 7.23: Plots of |z 1/2 | and Arg z 1/2 .

Im(z)

Im(z)

Re(z)

Re(z)

Figure 7.24: A path that does not encircle the origin and a path around the origin

271

second path in Figure 7.24.) The argument of z increases by 2. The value of the function at half turns on the path is e0 e2
1/2

= 1, = e = 1

(e )1/2 = e/2 = ,
1/2

As we return to the point z = 1, the argument of the function has changed by and the value of the function has changed from 1 to 1. If we were to walk along the circular path again, the argument of z would increase by another 2. The argument of the function would increase by another and the value of the function would return to 1. e4
1/2

= e2 = 1

In general, any time we walk around the origin, the value of z 1/2 changes by the factor 1. We call z = 0 a branch point. If we want a single-valued square root, we need something to prevent us from walking around the origin. We achieve this by introducing a branch cut. Suppose we have the complex plane drawn on an innite sheet of paper. With a scissors we cut the paper from the origin to along the real axis. Then if we start at z = e0 , and draw a continuous line without leaving the paper, the argument of z will always be in the range < arg z < . This means that < arg z 1/2 < . No matter what path we follow in this cut plane, z = 1 has argument zero and (1)1/2 = 1. 2 2 By never crossing the negative real axis, we have constructed a single valued branch of the square root function. We call the cut along the negative real axis a branch cut. Example 7.8.2 Consider the logarithmic function log z. For each value of z, there are an innite number of values of log z. We write log z in Cartesian form. log z = ln |z| + arg z Figure 7.25 shows the real and imaginary parts of the logarithm. The real part is single-valued. The imaginary part is multi-valued and has an innite number of branches. The values of the logarithm form an innite-layered sheet. If we start on one of the sheets and walk around the origin once in the positive direction, then the value of the logarithm increases by 2 and we move to the next branch. z = 0 is a branch point of the logarithm. 272

1 0 -1 -2 -2

2 1 -1 x 0 1 0 y -1 2-2

5 0 -5 -2

2 1 -1 x 0 1 0 y -1 2 -2

Figure 7.25: Plots of

(log z) and a portion of

(log z).

The logarithm is a continuous function except at z = 0. Suppose we start at z = 1 = e0 and the function value log (e0 ) = ln(1) + 0 = 0. If we follow the rst path in Figure 7.24, the argument of z and thus the imaginary part of the logarithm varies from up to about , down to about and back to 0. The value of the logarithm is still 0. 4 4 Now suppose we follow a circular path around the origin in the positive direction. (See the second path in Figure 7.24.) The argument of z increases by 2. The value of the logarithm at half turns on the path is

log e0 = 0, log (e ) = , log e2 = 2

As we return to the point z = 1, the value of the logarithm has changed by 2. If we were to walk along the circular path again, the argument of z would increase by another 2 and the value of the logarithm would increase by another 2. 273

Result 7.8.1 A point z0 is a branch point of a function f (z) if the function changes value when you walk around the point on any path that encloses no singularities other than the one at z = z0 .
Branch Points at Innity : Mapping Innity to the Origin. Up to this point we have considered only branch points in the nite plane. Now we consider the possibility of a branch point at innity. As a rst method of approaching this problem we map the point at innity to the origin with the transformation = 1/z and examine the point = 0. Example 7.8.3 Again consider the function z 1/2 . Mapping the point at innity to the origin, we have f () = (1/)1/2 = 1/2 . For each value of , there are two values of 1/2 . We write 1/2 in modulus-argument form. 1/2 = 1 || e arg()/2

Like z 1/2 , 1/2 has a double-layered sheet of values. Figure 7.26 shows the modulus and the principal argument of 1/2 . We see that each time we walk around the origin, the argument of 1/2 changes by . This means that the value of the function changes by the factor e = 1, i.e. the function changes sign. If we walk around the origin twice, the argument changes by 2, so that the value of the function does not change, e2 = 1. Since 1/2 has a branch point at zero, we conclude that z 1/2 has a branch point at innity. Example 7.8.4 Again consider the logarithmic function log z. Mapping the point at innity to the origin, we have f () = log(1/) = log(). From Example 7.8.2 we known that log() has a branch point at = 0. Thus log z has a branch point at innity.

Branch Points at Innity : Paths Around Innity. We can also check for a branch point at innity by following a path that encloses the point at innity and no other singularities. Just draw a simple closed curve that 274

3 2.5 2 1.5 1 -2

2 -1 x 1 0 y -1 2 -2

2 0 -2 -2

2 1 -1 x 0 1 0 y -1 2 -2

Figure 7.26: Plots of | 1/2 | and Arg 1/2 .

separates the complex plane into a bounded component that contains all the singularities of the function in the nite plane. Then, depending on orientation, the curve is a contour enclosing all the nite singularities, or the point at innity and no other singularities. Example 7.8.5 Once again consider the function z 1/2 . We know that the function changes value on a curve that goes once around the origin. Such a curve can be considered to be either a path around the origin or a path around innity. In either case the path encloses one singularity. There are branch points at the origin and at innity. Now consider a curve that does not go around the origin. Such a curve can be considered to be either a path around neither of the branch points or both of them. Thus we see that z 1/2 does not change value when we follow a path that encloses neither or both of its branch points.
1/2

Example 7.8.6 Consider f (z) = (z 2 1)

. We factor the function. f (z) = (z 1)1/2 (z + 1)1/2 275

There are branch points at z = 1. Now consider the point at innity. f 1 = 2 1


1/2

= 1 1 2

1/2

Since f ( 1 ) does not have a branch point at = 0, f (z) does not have a branch point at innity. We could reach the same conclusion by considering a path around innity. Consider a path that circles the branch points at z = 1 once in the positive direction. Such a path circles the point at innity once in the negative direction. In traversing this 1/2 1/2 path, the value of f (z) is multiplied by the factor (e2 ) (e2 ) = e2 = 1. Thus the value of the function does not change. There is no branch point at innity.

Diagnosing Branch Points. We have the denition of a branch point, but we do not have a convenient criterion for determining if a particular function has a branch point. We have seen that log z and z for non-integer have branch points at zero and innity. The inverse trigonometric functions like the arcsine also have branch points, but they can be written in terms of the logarithm and the square root. In fact all the elementary functions with branch points can be written in terms of the functions log z and z . Furthermore, note that the multi-valuedness of z comes from the logarithm, z = e log z . This gives us a way of quickly determining if and where a function may have branch points.

Result 7.8.2 Let f (z) be a single-valued function. Then log(f (z)) and (f (z)) may have branch points only where f (z) is zero or singular.
Example 7.8.7 Consider the functions, 1. (z 2 )
1/2 2

2. z 1/2 3. z 1/2

Are they multi-valued? Do they have branch points? 276

1. z2
1/2 1/2

1/2

= z 2 = z

Because of the ()1/2 , the function is multi-valued. The only possible branch points are at zero and innity. If
2 2 1/2 (e0 ) = 1, then (e2 ) = (e4 ) = e2 = 1. Thus we see that the function does not change value when we walk around the origin. We can also consider this to be a path around innity. This function is multi-valued, but has no branch points.

2. z 1/2 This function is single-valued. 3. z 1/2


3

= z

=z

= z

3 3

This function is multi-valued. We consider the possible branch point at z = 0. If


3

(e0 )

1/2

= 1, then

1/2 (e2 ) = (e )3 = e3 = 1. Since the function changes value when we walk around the origin, it has a branch point at z = 0. Since this is also a path around innity, there is a branch point there.

1 1 Example 7.8.8 Consider the function f (z) = log z1 . Since z1 is only zero at innity and its only singularity is at z = 1, the only possibilities for branch points are at z = 1 and z = . Since

log

1 z1

= log(z 1)

and log w has branch points at zero and innity, we see that f (z) has branch points at z = 1 and z = . Example 7.8.9 Consider the functions, 277

1. elog z 2. log ez . Are they multi-valued? Do they have branch points? 1. elog z = exp(Log z + 2n) = eLog z e2n = z This function is single-valued. 2. log ez = Log ez +2n = z + 2m This function is multi-valued. It may have branch points only where ez is zero or innite. This only occurs at z = . Thus there are no branch points in the nite plane. The function does not change when traversing a simple closed path. Since this path can be considered to enclose innity, there is no branch point at innity. Consider (f (z)) where f (z) is single-valued and f (z) has either a zero or a singularity at z = z0 . (f (z)) may have a branch point at z = z0 . If f (z) is not a power of z, then it may be dicult to tell if (f (z)) changes value when we walk around z0 . Factor f (z) into f (z) = g(z)h(z) where h(z) is nonzero and nite at z0 . Then g(z) captures the important behavior of f (z) at the z0 . g(z) tells us how fast f (z) vanishes or blows up. Since (f (z)) = (g(z)) (h(z)) and (h(z)) does not have a branch point at z0 , (f (z)) has a branch point at z0 if and only if (g(z)) has a branch point there. Similarly, we can decompose log(f (z)) = log(g(z)h(z)) = log(g(z)) + log(h(z)) to see that log(f (z)) has a branch point at z0 if and only if log(g(z)) has a branch point there.

Result 7.8.3 Consider a single-valued function f (z) that has either a zero or a singularity at z = z0 . Let f (z) = g(z)h(z) where h(z) is nonzero and nite. (f (z)) has a branch point at z = z0 if and only if (g(z)) has a branch point there. log(f (z)) has a branch point at z = z0 if and only if log(g(z)) has a branch point there.

278

Example 7.8.10 Consider the functions, 1. sin z 1/2 2. (sin z)1/2 3. z 1/2 sin z 1/2 4. (sin z 2 )
1/2

Find the branch points and the number of branches. 1. sin z 1/2 = sin z = sin z sin z 1/2 is multi-valued. It has two branches. There may be branch points at zero and innity. Consider the unit 1/2 1/2 circle which is a path around the origin or innity. If sin (e0 ) = sin(1), then sin (e2 ) = sin (e ) = sin(1) = sin(1). There are branch points at the origin and innity. 2. (sin z)1/2 = sin z The function is multi-valued with two branches. The sine vanishes at z = n and is singular at innity. There could be branch points at these locations. Consider the point z = n. We can write sin z = (z n) Note that
sin z zn

sin z z n

is nonzero and has a removable singularity at z = n. sin z cos z = lim = (1)n zn z n zn 1 lim

Since (z n)1/2 has a branch point at z = n, (sin z)1/2 has branch points at z = n. 279

Since the branch points at z = n go all the way out to innity. It is not possible to make a path that encloses innity and no other singularities. The point at innity is a non-isolated singularity. A point can be a branch point only if it is an isolated singularity. 3. z 1/2 sin z 1/2 = z sin z = z sin z = z sin z The function is single-valued. Thus there could be no branch points. 4. sin z 2
1/2

= sin z 2

This function is multi-valued. Since sin z 2 = 0 at z = (n)1/2 , there may be branch points there. First consider the point z = 0. We can write sin z 2 sin z 2 = z 2 2 z 2 2 where sin (z ) /z is nonzero and has a removable singularity at z = 0. sin z 2 2z cos z 2 = lim = 1. z0 z 2 z0 2z lim Since (z 2 ) does not have a branch point at z = 0, (sin z 2 ) Now consider the point z = n. sin z 2 = z n sin (z 2 ) / (z
1/2 1/2

does not have a branch point there either.

sin z 2 z n

n) in nonzero and has a removable singularity at z = lim

n.

sin z 2 2z cos z 2 = lim = 2 n(1)n 1 n z n n z 280

Since (z

n)

1/2

has a branch point at z =


1/2

n, (sin z 2 )

1/2

also has a branch point there.

Thus we that (sin2 ) branch points at z = (n)1/2 for n Z \ {0}. This is the set of numbers: see z has { , 2, . . . , , 2, . . .}. The point at innity is a non-isolated singularity.

Example 7.8.11 Find the branch points of f (z) = z 3 z Introduce branch cuts. If f (2) = We expand f (z). 3 6 then what is f (2)? f (z) = z 1/3 (z 1)1/3 (z + 1)1/3 . There are branch points at z = 1, 0, 1. We consider the point at innity. f 1 = = 1
1/3 1/3

1 1

1/3

1 +1

1/3

1 (1 )1/3 (1 + )1/3

Since f (1/) does not have a branch point at = 0, f (z) does not have a branch point at innity. Consider the three possible branch cuts in Figure 7.27. The rst and the third branch cuts will make the function single valued, the second will not. It is clear that the rst set makes the function single valued since it is not possible to walk around any of the branch points. The second set of branch cuts would allow you to walk around the branch points at z = 1. If you walked around these two once in the positive direction, the value of the function would change by the factor e4/3 . The third set of branch cuts would allow you to walk around all three branch points together. You can verify that if you walk around the three branch points, the value of the function will not change (e6/3 = e2 = 1). Suppose we introduce the third set of branch cuts and are on the branch with f (2) = 3 6. f (2) = 2 e0
1/3

1 e0 281

1/3

3 e0

1/3

Figure 7.27: Three Possible Branch Cuts for f (z) = (z 3 z) The value of f (2) is f (2) = (2 e )1/3 (3 e )1/3 (1 e )1/3 3 3 3 = 2 e/3 3 e/3 1 e/3 3 = 6 e 3 = 6. Example 7.8.12 Find the branch points and number of branches for f (z) = z z . z z = exp z 2 log z
2 2

1/3

There may be branch points at the origin and innity due to the logarithm. Consider walking around a circle of radius r centered at the origin in the positive direction. Since the logarithm changes by 2, the value of f (z) changes by the 2 factor e2r . There are branch points at the origin and innity. The function has an innite number of branches.

282

Example 7.8.13 Construct a branch of f (z) = z 2 + 1 such that f (0) = First we factor f (z). f (z) = (z )1/3 (z + )1/3 There are branch points at z = . Figure 7.28 shows one way to introduce branch cuts.
1/3

1 1 + 3 . 2

Figure 7.28: Branch Cuts for f (z) = (z 2 + 1)

1/3

Since it is not possible to walk around any branch point, these cuts make the function single valued. We introduce the coordinates: z = e , z + = r e . f (z) = e r e = 3 r e(+)/3 283
1/3 1/3

The condition f (0) = can be stated 3

1 1 + 3 = e(2/3+2n) 2 1 e(+)/3 = e(2/3+2n) + = 2 + 6n

The angles must be dened to satisfy this relation. One choice is 5 << , 2 2 3 << . 2 2

Principal Branches. We construct the principal branch of the logarithm by putting a branch cut on the negative real axis choose z = r e , (, ). Thus the principal branch of the logarithm is Log z = ln r + , < < .

Note that the if x is a negative real number, (and thus lies on the branch cut), then Log x is undened. The principal branch of z is z = e Log z . Note that there is a branch cut on the negative real axis. < arg e Log z < is denoted z. The principal branch of z 1/n is denoted n z.

The principal branch of the z 1/2

1/2 Example 7.8.14 Construct 1 z 2 , the principal branch of (1 z 2 ) . 1/2 First note that since (1 z 2 ) = (1 z)1/2 (1 + z)1/2 there are branch points at z = 1 and z = 1. The principal branch of the square root has a branch cut on the negative real axis. 1 z 2 is a negative real number for z ( . . . 1) (1 . . . ). Thus we put branch cuts on ( . . . 1] and [1 . . . ).

284

7.9

Exercises

Cartesian and Modulus-Argument Form


Exercise 7.1 Find the image of the strip 2 < x < 3 under the mapping w = f (z) = z 2 . Does the image constitute a domain? Hint, Solution Exercise 7.2 For a given real number , 0 < 2, nd the image of the sector 0 arg(z) < under the transformation w = z 4 . How large should be so that the w plane is covered exactly once? Hint, Solution

Trigonometric Functions
Exercise 7.3 In Cartesian coordinates, z = x + y, write sin(z) in Cartesian and modulus-argument form. Hint, Solution Exercise 7.4 Show that ez is nonzero for all nite z. Hint, Solution Exercise 7.5 Show that ez When does equality hold? Hint, Solution Exercise 7.6 Solve coth(z) = 1. Hint, Solution 285
2

e|z| .

Exercise 7.7 Solve 2 2z . That is, for what values of z is 2 one of the values of 2z ? Derive this result then verify your answer by evaluating 2z for the solutions that your nd. Hint, Solution Exercise 7.8 Solve 1 1z . That is, for what values of z is 1 one of the values of 1z ? Derive this result then verify your answer by evaluating 1z for the solutions that your nd. Hint, Solution

Logarithmic Identities
Exercise 7.9 Show that if (z1 ) > 0 and (z2 ) > 0 then Log(z1 z2 ) = Log(z1 ) + Log(z2 ) and illustrate that this relationship does not hold in general. Hint, Solution Exercise 7.10 Find the fallacy in the following arguments: 1. log(1) = log
1 1

= log(1) log(1) = log(1), therefore, log(1) = 0.

2. 1 = 11/2 = ((1)(1))1/2 = (1)1/2 (1)1/2 = = 1, therefore, 1 = 1. Hint, Solution Exercise 7.11 Write the following expressions in modulus-argument or Cartesian form. Denote any multi-valuedness explicitly. 22/5 , Hint, Solution 286 31+ ,
1/4

1/4 .

Exercise 7.12 Solve cos z = 69. Hint, Solution Exercise 7.13 Solve cot z = 47. Hint, Solution Exercise 7.14 Determine all values of 1. log() 2. () 3. 3 4. log(log()) and plot them in the complex plane. Hint, Solution Exercise 7.15 Evaluate and plot the following in the complex plane: 1. (cosh())2 2. log 1 1+

3. arctan(3) Hint, Solution 287

Exercise 7.16 Determine all values of and log ((1 + ) ) and plot them in the complex plane. Hint, Solution Exercise 7.17 Find all z for which 1. ez = 2. cos z = sin z 3. tan2 z = 1 Hint, Solution Exercise 7.18 Prove the following identities and identify the branch points of the functions in the extended complex plane. 1. arctan(z) = log 2 1 log 2 +z z 1+z 1z
1/2

2. arctanh(z) =

3. arccosh(z) = log z + z 2 1 Hint, Solution

Branch Points and Branch Cuts


Exercise 7.19 Identify the branch points of the function f (z) = log z(z + 1) z1

288

and introduce appropriate branch cuts to ensure that the function is single-valued. Hint, Solution Exercise 7.20 Identify all the branch points of the function w = f (z) = z 3 + z 2 6z
1/2

in the extended complex plane. Give a polar description of f (z) and specify branch cuts so that your choice of angles gives a single-valued function that is continuous at z = 1 with f (1) = 6. Sketch the branch cuts in the stereographic projection. Hint, Solution Exercise 7.21 Consider the mapping w = f (z) = z 1/3 and the inverse mapping z = g(w) = w3 . 1. Describe the multiple-valuedness of f (z). 2. Describe a region of the w-plane that g(w) maps one-to-one to the whole z-plane. 3. Describe and attempt to draw a Riemann surface on which f (z) is single-valued and to which g(w) maps oneto-one. Comment on the misleading nature of your picture. 4. Identify the branch points of f (z) and introduce a branch cut to make f (z) single-valued. Hint, Solution Exercise 7.22 Determine the branch points of the function f (z) = z 3 1
1/2

Construct cuts and dene a branch so that z = 0 and z = 1 do not lie on a cut, and such that f (0) = . What is f (1) for this branch? Hint, Solution 289

Exercise 7.23 Determine the branch points of the function w(z) = ((z 1)(z 6)(z + 2))1/2 Construct cuts and dene a branch so that z = 4 does not lie on a cut, and such that w = 6 when z = 4. Hint, Solution Exercise 7.24 Give the number of branches and locations of the branch points for the functions 1. cos z 1/2 2. (z + )z Hint, Solution Exercise 7.25 Find the branch points of the following functions in the extended complex plane, (the complex plane including the point at innity). 1. z 2 + 1 2. z 3 z
1/2

1/2

3. log z 2 1 4. log z+1 z1

Introduce branch cuts to make the functions single valued. Hint, Solution 290

Exercise 7.26 Find all branch points and introduce cuts to make the following functions single-valued: For the rst function, choose cuts so that there is no cut within the disk |z| < 2. 1. f (z) = z 3 + 8
1/2

2. f (z) = log 5 + 3. f (z) = (z + 3)1/2 Hint, Solution

z+1 z1

1/2

Exercise 7.27 Let f (z) have branch points at z = 0 and z = , but nowhere else in the extended complex plane. How does the value and argument of f (z) change while traversing the contour in Figure 7.29? Does the branch cut in Figure 7.29 make the function single-valued?

Figure 7.29: Contour Around the Branch Points and Branch Cut. 291

Hint, Solution Exercise 7.28 Let f (z) be analytic except for no more than a countably innite number of singularities. Suppose that f (z) has only one branch point in the nite complex plane. Does f (z) have a branch point at innity? Now suppose that f (z) has two or more branch points in the nite complex plane. Does f (z) have a branch point at innity? Hint, Solution Exercise 7.29 1/4 Find all branch points of (z 4 + 1) in the extended complex plane. Which of the branch cuts in Figure 7.30 make the function single-valued.

Figure 7.30: Four Candidate Sets of Branch Cuts for (z 4 + 1) Hint, Solution Exercise 7.30 Find the branch points of f (z) = z 2+1 z
1/3

1/4

in the extended complex plane. Introduce branch cuts that make the function single-valued and such that the function 292

is dened on the positive real axis. Dene a branch such that f (1) = 1/ 3 2. Write down an explicit formula for the value of the branch. What is f (1 + )? What is the value of f (z) on either side of the branch cuts? Hint, Solution Exercise 7.31 Find all branch points of f (z) = ((z 1)(z 2)(z 3))1/2 in the extended complex plane. Which of the branch cuts in Figure 7.31 will make the function single-valued. Using the rst set of branch cuts in this gure dene a branch on which f (0) = 6. Write out an explicit formula for the value of the function on this branch.

Figure 7.31: Four Candidate Sets of Branch Cuts for ((z 1)(z 2)(z 3))1/2 Hint, Solution

293

Exercise 7.32 Determine the branch points of the function w= z 2 2 (z + 2)


1/3

Construct and dene a branch so that the resulting cut is one line of nite extent and w(2) = 2. What is w(3) for this branch? What are the limiting values of w on either side of the branch cut? Hint, Solution Exercise 7.33 Construct the principal branch of arccos(z). (Arccos(z) has the property that if x [1, 1] then Arccos(x) [0, ]. In particular, Arccos(0) = ). 2 Hint, Solution Exercise 7.34 Find the branch points of z 1/2 1 single-valued. Hint, Solution
1/2

in the nite complex plane. Introduce branch cuts to make the function

Exercise 7.35 For the linkage illustrated in Figure 7.32, use complex variables to outline a scheme for expressing the angular position, velocity and acceleration of arm c in terms of those of arm a. (You neednt work out the equations.) Hint, Solution Exercise 7.36 Find the image of the strip | (z)| < 1 and of the strip 1 < (z) < 2 under the transformations: 1. w = 2z 2 2. w =
z+1 z1

Hint, Solution 294

b a l
Figure 7.32: A linkage Exercise 7.37 Locate and classify all the singularities of the following functions: 1. (z + 1)1/2 z+2 1 1+z

2. cos 3.

1 (1 ez )2 In each case discuss the possibility of a singularity at the point . Hint, Solution Exercise 7.38 Describe how the mapping w = sinh(z) transforms the innite strip < x < , 0 < y < into the w-plane. Find cuts in the w-plane which make the mapping continuous both ways. What are the images of the lines (a) y = /4; (b) x = 1? Hint, Solution

295

7.10
Hint 7.1

Hints

Cartesian and Modulus-Argument Form

Hint 7.2

Trigonometric Functions
Hint 7.3 Recall that sin(z) =
1 2

(ez ez ). Use Result 6.3.1 to convert between Cartesian and modulus-argument form.

Hint 7.4 Write ez in polar form. Hint 7.5 The exponential is an increasing function for real variables. Hint 7.6 Write the hyperbolic cotangent in terms of exponentials. Hint 7.7 Write out the multi-valuedness of 2z . There is a doubly-innite set of solutions to this problem. Hint 7.8 Write out the multi-valuedness of 1z .

Logarithmic Identities

296

Hint 7.9

Hint 7.10 Write out the multi-valuedness of the expressions. Hint 7.11 Do the exponentiations in polar form. Hint 7.12 Write the cosine in terms of exponentials. Multiply by ez to get a quadratic equation for ez . Hint 7.13 Write the cotangent in terms of exponentials. Get a quadratic equation for ez . Hint 7.14

Hint 7.15

Hint 7.16 has an innite number of real, positive values. = e log . log ((1 + ) ) has a doubly innite set of values. log ((1 + ) ) = log(exp( log(1 + ))). Hint 7.17

Hint 7.18

Branch Points and Branch Cuts


297

Hint 7.19 Hint 7.20 Hint 7.21 Hint 7.22 Hint 7.23 Hint 7.24 Hint 7.25 1/2 1. (z 2 + 1) = (z )1/2 (z + )1/2 2. (z 3 z)
1/2

= z 1/2 (z 1)1/2 (z + 1)1/2

3. log (z 2 1) = log(z 1) + log(z + 1) 4. log Hint 7.26 Hint 7.27 Reverse the orientation of the contour so that it encircles innity and does not contain any branch points. 298
z+1 z1

= log(z + 1) log(z 1)

Hint 7.28 Consider a contour that encircles all the branch points in the nite complex plane. Reverse the orientation of the contour so that it contains the point at innity and does not contain any branch points in the nite complex plane. Hint 7.29 Factor the polynomial. The argument of z 1/4 changes by /2 on a contour that goes around the origin once in the positive direction. Hint 7.30

Hint 7.31 To dene the branch, dene angles from each of the branch points in the nite complex plane. Hint 7.32

Hint 7.33

Hint 7.34

Hint 7.35

Hint 7.36

Hint 7.37

299

Hint 7.38

300

7.11

Solutions

Cartesian and Modulus-Argument Form


Solution 7.1 Let w = u + v. We consider the strip 2 < x < 3 as composed of vertical lines. Consider the vertical line: z = c + y, y R for constant c. We nd the image of this line under the mapping. w = (c + y)2 w = c2 y 2 + 2cy u = c2 y 2 , v = 2cy This is a parabola that opens to the left. We can parameterize the curve in terms of v. u = c2 1 2 v , 4c2 vR

The boundaries of the region, x = 2 and x = 3, are respectively mapped to the parabolas: u=4 1 2 v , 16 v R and u = 9 1 2 v , 36 vR

We write the image of the mapping in set notation. w = u + v : v R and 4 1 2 1 v < u < 9 v2 . 16 36

See Figure 7.33 for depictions of the strip and its image under the mapping. The mapping is one-to-one. Since the image of the strip is open and connected, it is a domain. Solution 7.2 We write the mapping w = z 4 in polar coordinates. w = z 4 = r e 301
4

= r4 e4

3 2 1 -1 -1 -2 -3 1 2 3 4 5 -5

10 5 5 -5 -10 10 15

Figure 7.33: The domain 2 < x < 3 and its image under the mapping w = z 2 .

Thus we see that w : {r e | r 0, 0 < } {r4 e4 | r 0, 0 < } = {r e | r 0, 0 < 4}.

We can state this in terms of the argument.

w : {z | 0 arg(z) < } {z | 0 arg(z) < 4}

If = /2, the sector will be mapped exactly to the whole complex plane.

Trigonometric Functions

302

Solution 7.3 1 z e ez 2 1 y+x e = eyx 2 1 y e (cos x + sin x) ey (cos x sin x) = 2 1 y e (sin x cos x) + ey (sin x + cos x) = 2 = sin x cosh y + cos x sinh y

sin z =

sin z = = =

sin2 x cosh2 y + cos2 x sinh2 y exp( arctan(sin x cosh y, cos x sinh y)) cosh2 y cos2 x exp( arctan(sin x cosh y, cos x sinh y)) 1 (cosh(2y) cos(2x)) exp( arctan(sin x cosh y, cos x sinh y)) 2

Solution 7.4 In order that ez be zero, the modulus, ex must be zero. Since ex has no nite solutions, ez = 0 has no nite solutions. Solution 7.5 We write the expressions in terms of Cartesian coordinates. ez
2

= e(x+y) = ex = ex

2 y 2 +2xy

2 y 2

303

e|z| = e|x+y| = ex ez Equality holds only when y = 0. Solution 7.6


2

2 +y 2 2 y 2

The exponential function is an increasing function for real variables. Since x2 y 2 x2 + y 2 , ex e|z|
2

ex

2 +y 2

coth(z) = 1 (e + ez ) /2 =1 (ez ez ) /2 ez + ez = ez ez ez = 0
z

There are no solutions. Solution 7.7 We write out the multi-valuedness of 2z . 2 2z eln 2 ez log(2) eln 2 {ez(ln(2)+2n) | n Z} ln 2 z{ln 2 + 2n + 2m | m, n Z} z= ln(2) + 2m | m, n Z ln(2) + 2n

We verify this solution. Consider m and n to be xed integers. We express the multi-valuedness in terms of k. 2(ln(2)+2m)/(ln(2)+2n) = e(ln(2)+2m)/(ln(2)+2n) log(2) = e(ln(2)+2m)/(ln(2)+2n)(ln(2)+2k) 304

For k = n, this has the value, eln(2)+2m = eln(2) = 2. Solution 7.8 We write out the multi-valuedness of 1z . 1 1z 1 ez log(1) 1 {ez2n | n Z} The element corresponding to n = 0 is e0 = 1. Thus 1 1z has the solutions, z C. That is, z may be any complex number. We verify this solution. 1z = ez log(1) = ez2n For n = 0, this has the value 1.

Logarithmic Identities
Solution 7.9 We write the relationship in terms of the natural logarithm and the principal argument. Log(z1 z2 ) = Log(z1 ) + Log(z2 ) ln |z1 z2 | + Arg(z1 z2 ) = ln |z1 | + Arg(z1 ) + ln |z2 | + Arg(z2 ) Arg(z1 z2 ) = Arg(z1 ) + Arg(z2 ) (zk ) > 0 implies that Arg(zk ) (/2 . . . /2). Thus Arg(z1 ) + Arg(z2 ) ( . . . ). In this case the relationship holds. The relationship does not hold in general because Arg(z1 ) + Arg(z2 ) is not necessarily in the interval ( . . . ]. Consider z1 = z2 = 1. Arg((1)(1)) = Arg(1) = 0, Log((1)(1)) = Log(1) = 0, 305 Arg(1) + Arg(1) = 2 Log(1) + Log(1) = 2

Solution 7.10 1. The algebraic manipulations are ne. We write out the multi-valuedness of the logarithms. log(1) = log 1 1 = log(1) log(1) = log(1)

{ + 2n : n Z} = { + 2n : n Z} = {2n : n Z} { + 2n : n Z} = { 2n : n Z} Thus log(1) = log(1). However this does not imply that log(1) = 0. This is because the logarithm is a set-valued function log(1) = log(1) is really saying: { + 2n : n Z} = { 2n : n Z} 2. We consider 1 = 11/2 = ((1)(1))1/2 = (1)1/2 (1)1/2 = = 1. There are three multi-valued expressions above. 11/2 = 1 ((1)(1))1/2 = 1 (1)1/2 (1)1/2 = ()() = 1 Thus we see that the rst and fourth equalities are incorrect. 1 = 11/2 , Solution 7.11 22/5 = 41/5 5 = 411/5 5 = 4 e2n/5 , 306 (1)1/2 (1)1/2 =

n = 0, 1, 2, 3, 4

31+ = e(1+) log 3 = e(1+)(ln 3+2n) = eln 32n e(ln 3+2n) , nZ

1/4

= 2 e/6 4 = 2 e/24 11/4 4 = 2 e(n/2/24) ,

1/4

n = 0, 1, 2, 3

1/4 = e(/4) log 1 = e(/4)(2n) = en/2 , nZ

307

Solution 7.12

cos z = 69 ez + ez = 69 2 e2z 138 ez +1 = 0 1 ez = 138 1382 4 2 z = log 69 2 1190 z = ln 69 2 1190 + 2n z = 2n ln 69 2 1190 , nZ

308

Solution 7.13 cot z = 47 (e + ez ) /2 = 47 (ez ez ) /(2) ez + ez = 47 ez ez 46 e2z 48 = 0 24 2z = log 23 24 z = log 2 23 24 z= ln + 2n , n Z 2 23


z

z = n Solution 7.14 1.

24 ln , 2 23

nZ

log() = ln | | + arg() = ln(1) + + 2n , n Z 2 log() = + 2n, n Z 2 These are equally spaced points in the imaginary axis. See Figure 7.34. 2. () = e log() = e(/2+2n) , 309 nZ

10 -1 -10 1

Figure 7.34: log() () = e/2+2n , nZ

These are points on the positive real axis with an accumulation point at the origin. See Figure 7.35.

-1
Figure 7.35: () 3. 3 = e log(3) = e(ln(3)+ arg(3)) 310

3 = e(ln(3)+2n) ,

nZ

These points all lie on the circle of radius |e | centered about the origin in the complex plane. See Figure 7.36.
10 5 -10 -5 -5 -10 5 10

Figure 7.36: 3

4. log(log()) = log + 2m , m Z 2 = ln + 2m + Arg + 2m + 2n, m, n Z 2 2 = ln + 2m + sign(1 + 4m) + 2n, m, n Z 2 2 These points all lie in the right half-plane. See Figure 7.37.

311

20 10 1 -10 -20 2 3 4 5

Figure 7.37: log(log())

Solution 7.15 1.

e + e (cosh()) = 2 2 = (1)
2

= e2 log(1) = e2(ln(1)++2n) , =e
2(1+2n)

nZ

nZ

These are points on the positive real axis with an accumulation point at the origin. See Figure 7.38. 312

1000

-1
Figure 7.38: The values of (cosh())2 .

2.

log

1 1+

= log(1 + ) = log 2 e/4

1 = ln(2) log e/4 2 1 = ln(2) /4 + 2n, 2

nZ

These are points on a vertical line in the complex plane. See Figure 7.39. 313

10 -1 -10 1

Figure 7.39: The values of log

1 1+

3.

arctan(3) =

1 3 log 2 + 3 1 1 = log 2 2 1 1 ln + + 2n , = 2 2 = + n + ln(2) 2 2

nZ

These are points on a horizontal line in the complex plane. See Figure 7.40.

314

-5

-1
Figure 7.40: The values of arctan(3). Solution 7.16 = e log() = e(ln ||+ Arg()+2n) , =e
(/2+2n)

nZ

nZ nZ

= e(1/2+2n) ,

These are points on the positive real axis. There is an accumulation point at z = 0. See Figure 7.41. log ((1 + ) ) = log e log(1+) = log(1 + ) + 2n, n Z = (ln |1 + | + Arg(1 + ) + 2m) + 2n, m, n Z 1 = ln 2 + + 2m + 2n, m, n Z 2 4 1 1 = 2 + 2m + ln 2 + 2n , m, n Z 4 2 315

25

50

75 100

-1
Figure 7.41:

See Figure 7.42 for a plot.

10 5 -40 -20 -5 -10


Figure 7.42: log ((1 + ) )

20

316

Solution 7.17 1. ez = z = log z = ln || + arg() z = ln(1) + + 2n , n Z 2 z = + 2n, n Z 2

2. We can solve the equation by writing the cosine and sine in terms of the exponential.

cos z = sin z ez + ez ez ez = 2 2 (1 + ) ez = (1 + ) ez 1 + e2z = 1+ 2z e = 2z = log() 2z = + 2n, n Z 2 z = + n, n Z 4 317

3.

tan2 z = 1 sin2 z = cos2 z cos z = sin z ez + ez ez ez = 2 2 ez = ez or ez = ez ez = 0 or ez = 0 eyx = 0 or ey+x = 0 ey = 0 or ey = 0 z=

There are no solutions for nite z.

318

Solution 7.18 1. w = arctan(z) z = tan(w) sin(w) z= cos(w) w (e ew ) /(2) z= (ew + ew ) /2 z ew +z ew = ew + ew ( + z) e2w = ( z) e


w

z +z

1/2

w = log arctan(z) = We identify the branch points of the arctangent.

z +z

1/2

log 2

+z z

(log( + z) log( z)) 2 There are branch points at z = due to the logarithm terms. We examine the point at innity with the change of variables = 1/z. arctan(z) = arctan(1/) = + 1/ log 2 1/ + 1 arctan(1/) = log 2 1 319

As 0, the argument of the logarithm term tends to 1 The logarithm does not have a branch point at that point. Since arctan(1/) does not have a branch point at = 0, arctan(z) does not have a branch point at innity. 2. w = arctanh(z) z = tanh(w) sinh(w) z= cosh(w) w (e ew ) /2 z= w (e + ew ) /2 z ew +z ew = ew ew (z 1) e2w = z 1 ew = w = log arctanh(z) = z 1 z1 z+1 1z 1 log 2
1/2

1/2

1+z 1z

We identify the branch points of the hyperbolic arctangent. arctanh(z) = 1 (log(1 + z) log(1 z)) 2

There are branch points at z = 1 due to the logarithm terms. We examine the point at innity with the change 320

of variables = 1/z. arctanh(1/) = 1 + 1/ 1 log 2 1 1/ 1 +1 arctanh(1/) = log 2 1

As 0, the argument of the logarithm term tends to 1 The logarithm does not have a branch point at that point. Since arctanh(1/) does not have a branch point at = 0, arctanh(z) does not have a branch point at innity. 3. w = arccosh(z) z = cosh(w) ew + ew z= 2 e2w 2z ew +1 = 0 ew = z + z 2 1
1/2 1/2

w = log z + z 2 1

arccosh(z) = log z + z 2 1 We identify the branch points of the hyperbolic arc-cosine.

1/2

arccosh(z) = log z + (z 1)1/2 (z + 1)1/2 First we consider branch points due to the square root. There are branch points at z = 1 due to the square 1/2 root terms. If we walk around the singularity at z = 1 and no other singularities, the (z 2 1) term changes 321

sign. This will change the value of arccosh(z). The same is true for the point z = 1. The point at innity is 1/2 not a branch point for (z 2 1) . We factor the expression to verify this. z2 1
1/2 1/2

= z2

1/2

1 z 2

1/2

(z 2 ) does not have a branch point at innity. It is multi-valued, but it has no branch points. (1 z 2 ) does not have a branch point at innity, The argument of the square root function tends to unity there. In summary, there are branch points at z = 1 due to the square root. If we walk around either one of the these branch points. the square root term will change value. If we walk around both of these points, the square root term will not change value. Now we consider branch points due to logarithm. There may be branch points where the argument of the logarithm vanishes or tends to innity. We see if the argument of the logarithm vanishes. z + z2 1 =0 2 2 z =z 1 z + (z 2 1) is non-zero and nite everywhere in the complex plane. The only possibility for a branch point 1/2 in the logarithm term is the point at innity. We see if the argument of z + (z 2 1) changes when we walk around innity but no other singularity. We consider a circular path with center at the origin and radius greater than unity. We can either say that this path encloses the two branch points at z = 1 and no other singularities or we can say that this path encloses the point at innity and no other singularities. We examine the value of the argument of the logarithm on this path. z + z2 1 Neither (z 2 ) nor (1 z 2 ) square root in the expression.
1/2 1/2 1/2 1/2 1/2

1/2

= z + z2

1/2

1 z 2

1/2

changes value as we walk the path. Thus we can use the principal branch of the
1/2

z + z2 1

= z z 1 z 2 = z 1 1 z 2 322

First consider the + branch. z 1+

1 z 2

As we walk the path around innity, the argument of z changes by 2 while the argument of 1 + 1 z 2 1/2 does not change. Thus the argument of z + (z 2 1) changes by 2 when we go around innity. This makes the value of the logarithm change by 2. There is a branch point at innity. First consider the branch. z 1 1 1 z 2 = z 1 1 z 2 + O z 4 2 1 2 =z z + O z 4 2 1 = z 1 1 + O z 2 2

As we walk the path around innity, the argument of z 1 changes by 2 while the argument of (1 + O (z 2 )) 1/2 does not change. Thus the argument of z + (z 2 1) changes by 2 when we go around innity. This makes the value of the logarithm change by 2. Again we conclude that there is a branch point at innity. For the sole purpose of overkill, lets repeat the above analysis from a geometric viewpoint. Again we consider the possibility of a branch point at innity due to the logarithm. We walk along the circle shown in the rst plot of Figure 7.43. Traversing this path, we go around innity, but no other singularities. We consider the mapping 1/2 w = z + (z 2 1) . Depending on the branch of the square root, the circle is mapped to one one of the contours shown in the second plot. For each branch, the argument of w changes by 2 as we traverse the circle in the 1/2 z-plane. Therefore the value of arccosh(z) = log z + (z 2 1) changes by 2 as we traverse the circle. We again conclude that there is a branch point at innity due to the logarithm. To summarize: There are branch points at z = 1 due to the square root and a branch point at innity due to the logarithm.

Branch Points and Branch Cuts


323

1 1 -1 1 -1 -1 -1 1

Figure 7.43: The mapping of a circle under w = z + (z 2 1) Solution 7.19 We expand the function to diagnose the branch points in the nite complex plane. f (z) = log z(z + 1) z1 = log(z) + log(z + 1) log(z 1)

1/2

The are branch points at z = 1, 0, 1. Now we examine the point at innity. We make the change of variables z = 1/. f 1 (1/)(1/ + 1) (1/ 1) 1 (1 + = log 1 = log(1 + ) log(1 ) log() = log

log() has a branch point at = 0. The other terms do not have branch points there. Since f (1/) has a branch point at = 0 f (z) has a branch point at innity. Note that in walking around either z = 1 or z = 0 once in the positive direction, the argument of z(z + 1)/(z 1) changes by 2. In walking around z = 1, the argument of z(z + 1)/(z 1) changes by 2. This argument does not 324

change if we walk around both z = 0 and z = 1. Thus we put a branch cut between z = 0 and z = 1. Next be put a branch cut between z = 1 and the point at innity. This prevents us from walking around either of these branch points. These two branch cuts separate the branches of the function. See Figure 7.44

-3

-2

-1

Figure 7.44: Branch cuts for log

z(z+1) z1

Solution 7.20 First we factor the function. f (z) = (z(z + 3)(z 2))1/2 = z 1/2 (z + 3)1/2 (z 2)1/2 There are branch points at z = 3, 0, 2. Now we examine the point at innity. f 1 = 1 1 +3 1 2
1/2

= 3/2 ((1 + 3)(1 2))1/2

Since 3/2 has a branch point at = 0 and the rest of the terms are analytic there, f (z) has a branch point at innity. Consider the set of branch cuts in Figure 7.45. These cuts do not permit us to walk around any single branch point. We can only walk around none or all of the branch points, (which is the same thing). The cuts can be used to dene a single-valued branch of the function. 325

3 2 1 -4 -2 -1 -2 -3
Figure 7.45: Branch Cuts for (z 3 + z 2 6z)
1/2

326

Now to dene the branch. We make a choice of angles. z + 3 = r1 e1 , z = r2 e2 , z 2 = r3 e3 , The function is f (z) = r1 e1 r2 e2 r3 e3 We evaluate the function at z = 1. f (1) = (2)(1)(3) e(0++)/2 = 6
1/2

< 1 < 3 < 2 < 2 2 0 < 3 < 2 r1 r2 r3 e(1 +2 +3 )/2 .

We see that our choice of angles gives us the desired branch. The stereographic projection is the projection from the complex plane onto a unit sphere with south pole at the origin. The point z = x + y is mapped to the point (X, Y, Z) on the sphere with X= 4x , |z|2 + 4 Y = 4y , |z|2 + 4 Z= 2|z|2 . |z|2 + 4

Figure 7.46 rst shows the branch cuts and their stereographic projections and then shows the stereographic projections alone. Solution 7.21 1. For each value of z, f (z) = z 1/3 has three values. f (z) = z 1/3 = 2. g(w) = w3 = |w|3 e3 arg(w) 327 3 z ek2/3 , k = 0, 1, 2

0 2 0 -4 0 -1 2 4 0 4 -4
1/2

1 0 -1

Figure 7.46: Branch cuts for (z 3 + z 2 6z)

and their stereographic projections.

Any sector of the w plane of angle 2/3 maps one-to-one to the whole z-plane. g : r e | r 0, 0 < 0 + 2/3 r3 e3 | r 0, 0 < 0 + 2/3 g : r e | r 0, 0 < 0 + 2/3 r e | r 0, 30 < 30 + 2 g : r e | r 0, 0 < 0 + 2/3 C See Figure 7.47 to see how g(w) maps the sector 0 < 2/3. 3. See Figure 7.48 for a depiction of the Riemann surface for f (z) = z 1/3 . We show two views of the surface and a curve that traces the edge of the shown portion of the surface. The depiction is misleading because the surface is not self-intersecting. We would need four dimensions to properly visualize the this Riemann surface. 4. f (z) = z 1/3 has branch points at z = 0 and z = . Any branch cut which connects these two points would prevent us from walking around the points singly and would thus separate the branches of the function. For example, we could put a branch cut on the negative real axis. Dening the angle < < for the mapping f r e = denes a single-valued branch of the function. 328 3 r e/3

Figure 7.47: The mapping g(w) = w3 maps the sector 0 < 2/3 one-to-one to the whole z-plane.

329

Figure 7.48: Riemann surface for f (z) = z 1/3 . Solution 7.22 The cube roots of 1 are 1, e2/3 , e4/3 = We factor the polynomial. z3 1
1/2

1 + 3 1 3 1, , 2 2

= (z 1)1/2

1 3 z+ 2

1/2

1+ 3 z+ 2

1/2

There are branch points at each of the cube roots of unity. 1 + 3 1 3 z = 1, , 2 2 Now we examine the point at innity. We make the change of variables z = 1/. f (1/) = 1/ 3 1
1/2

= 3/2 1 3

1/2

330

3/2 has a branch point at = 0, while (1 3 ) f (z) has a branch point at innity.

1/2

is not singular there. Since f (1/) has a branch point at = 0,

There are several ways of introducing branch cuts to separate the branches of the function. The easiest approach is to put a branch cut from each of the three branch points in the nite complex plane out to the branch point at innity. See Figure 7.49a. Clearly this makes the function single valued as it is impossible to walk around any of the branch points. Another approach is to have a branch cut from one of the branch points in the nite plane to the branch point at innity and a branch cut connecting the remaining two branch points. See Figure 7.49bcd. Note that in walking around any one of the nite branch points, (in the positive direction), the argument of the function changes by . This means that the value of the function changes by e , which is to say the value of the function changes sign. In walking around any two of the nite branch points, (again in the positive direction), the argument of the function changes by 2. This means that the value of the function changes by e2 , which is to say that the value of the function does not change. This demonstrates that the latter branch cut approach makes the function single-valued.

c
Figure 7.49: (z 3 1)
1/2

Now we construct a branch. We will use the branch cuts in Figure 7.49a. We introduce variables to measure radii 331

and angles from the three nite branch points. z 1 = r1 e1 , 0 < 1 < 2 1 3 2 z+ = r2 e2 , < 2 < 2 3 3 1+ 3 2 z+ = r3 e3 , < 3 < 2 3 3 We compute f (0) to see if it has the desired value. f (z) = r1 r2 r3 e(1 +2 +3 )/2

f (0) = e(/3+/3)/2 = Since it does not have the desired value, we change the range of 1 . z 1 = r1 e1 , f (0) now has the desired value. f (0) = e(3/3+/3)/2 = We compute f (1). f (1) = Solution 7.23 First we factor the function. w(z) = ((z + 2)(z 1)(z 6))1/2 = (z + 2)1/2 (z 1)1/2 (z 6)1/2 There are branch points at z = 2, 1, 6. Now we examine the point at innity. w 1 = 1 +2 1 1 1 6
1/2

2 < 1 < 4

2 e(32/3+2/3)/2 = 2

3/2

2 1+

1 1

6 1

1/2

332

Since 3/2 has a branch point at = 0 and the rest of the terms are analytic there, w(z) has a branch point at innity. Consider the set of branch cuts in Figure 7.50. These cuts let us walk around the branch points at z = 2 and z = 1 together or if we change our perspective, we would be walking around the branch points at z = 6 and z = together. Consider a contour in this cut plane that encircles the branch points at z = 2 and z = 1. Since the argument of (z z0 )1/2 changes by when we walk around z0 , the argument of w(z) changes by 2 when we traverse the contour. Thus the value of the function does not change and it is a valid set of branch cuts.

Figure 7.50: Branch Cuts for ((z + 2)(z 1)(z 6))1/2 Now to dene the branch. We make a choice of angles. z + 2 = r1 e1 , z 1 = r2 e2 , z 6 = r3 e3 , The function is w(z) = r1 e1 r2 e2 r3 e3 We evaluate the function at z = 4. w(4) = (6)(3)(2) e(2n+2n+)/2 = 6 1 = 2 for z (1 . . . 6), 2 = 1 for z (1 . . . 6), 0 < 3 < 2
1/2

r1 r2 r3 e(1 +2 +3 )/2 .

We see that our choice of angles gives us the desired branch. 333

Solution 7.24 1.

cos z 1/2 = cos z = cos z

This is a single-valued function. There are no branch points. 2. (z + )z = ez log(z+) = ez(ln |z+|+ Arg(z+)+2n) , nZ

There is a branch point at z = . There are an innite number of branches. Solution 7.25 1. f (z) = z 2 + 1
1/2

= (z + )1/2 (z )1/2

We see that there are branch points at z = . To examine the point at innity, we substitute z = 1/ and examine the point = 0. 1
2 1/2

+1

1 ( 2 )1/2

1 + 2

1/2

Since there is no branch point at = 0, f (z) has no branch point at innity. A branch cut connecting z = would make the function single-valued. We could also accomplish this with two branch cuts starting z = and going to innity. 2. f (z) = z 3 z
1/2

= z 1/2 (z 1)1/2 (z + 1)1/2

There are branch points at z = 1, 0, 1. Now we consider the point at innity. f 1 = 1


3

1 334

1/2

= 3/2 1 2

1/2

There is a branch point at innity. One can make the function single-valued with three branch cuts that start at z = 1, 0, 1 and each go to innity. We can also make the function single-valued with a branch cut that connects two of the points z = 1, 0, 1 and another branch cut that starts at the remaining point and goes to innity. 3. f (z) = log z 2 1 = log(z 1) + log(z + 1) There are branch points at z = 1. f 1 = log 1 1 2 = log 2 + log 1 2

log ( 2 ) has a branch point at = 0. log 2 = ln 2 + arg 2 = ln 2 2 arg() Every time we walk around the point = 0 in the positive direction, the value of the function changes by 4. f (z) has a branch point at innity. We can make the function single-valued by introducing two branch cuts that start at z = 1 and each go to innity. 4. f (z) = log There are branch points at z = 1. f 1 = log 1/ + 1 1/ 1 = log 1+ 1 z+1 z1 = log(z + 1) log(z 1)

There is no branch point at = 0. f (z) has no branch point at innity. 335

We can make the function single-valued by introducing two branch cuts that start at z = 1 and each go to innity. We can also make the function single-valued with a branch cut that connects the points z = 1. This is because log(z + 1) and log(z 1) change by 2 and 2, respectively, when you walk around their branch points once in the positive direction. Solution 7.26 1. The cube roots of 8 are 2, 2 e2/3 , 2 e4/3 = 2, 1 + 3, 1 3 . Thus we can write z3 + 8
1/2

= (z + 2)1/2 z 1 3

1/2

z1+ 3

1/2

There are three branch points on the circle of radius 2. z = 2, 1 + 3, 1 3 . We examine the point at innity. f (1/) = 1/ 3 + 8
1/2

= 3/2 1 + 8 3

1/2

Since f (1/) has a branch point at = 0, f (z) has a branch point at innity. There are several ways of introducing branch cuts outside of the disk |z| < 2 to separate the branches of the function. The easiest approach is to put a branch cut from each of the three branch points in the nite complex plane out to the branch point at innity. See Figure 7.51a. Clearly this makes the function single valued as it is impossible to walk around any of the branch points. Another approach is to have a branch cut from one of the branch points in the nite plane to the branch point at innity and a branch cut connecting the remaining two branch points. See Figure 7.51bcd. Note that in walking around any one of the nite branch points, (in the positive direction), the argument of the function changes by . This means that the value of the function changes by e , which is to say the value of the function changes sign. In walking around any two of the nite 336

c
Figure 7.51: (z 3 + 8)
1/2

branch points, (again in the positive direction), the argument of the function changes by 2. This means that the value of the function changes by e2 , which is to say that the value of the function does not change. This demonstrates that the latter branch cut approach makes the function single-valued. 2. f (z) = log 5 + First we deal with the function g(z) = z+1 z1
1/2

z+1 z1
1/2

1/2

Note that it has branch points at z = 1. Consider the point at innity. g(1/) = 1/ + 1 1/ 1 = 1+ 1
1/2

Since g(1/) has no branch point at = 0, g(z) has no branch point at innity. This means that if we walk around both of the branch points at z = 1, the function does not change value. We can verify this with another method: When we walk around the point z = 1 once in the positive direction, the argument of z + 1 changes by 2, the argument of (z + 1)1/2 changes by and thus the value of (z + 1)1/2 changes by e = 1. When we 337

walk around the point z = 1 once in the positive direction, the argument of z 1 changes by 2, the argument of (z 1)1/2 changes by and thus the value of (z 1)1/2 changes by e = 1. f (z) has branch points 1/2 does at z = 1. When we walk around both points z = 1 once in the positive direction, the value of z+1 z1 not change. Thus we can make the function single-valued with a branch cut which enables us to walk around either none or both of these branch points. We put a branch cut from 1 to 1 on the real axis. f (z) has branch points where 5+ z+1 z1
1/2

is either zero or innite. The only place in the extended complex plane where the expression becomes innite is at z = 1. Now we look for the zeros. 5+ z+1 z1
1/2 1/2

=0

z+1 = 5 z1 z+1 = 25 z1 z + 1 = 25z 25 13 z= 12 Note that 13/12 + 1 13/12 1


1/2

= 251/2 = 5.

On one branch, (which we call the positive branch), of the function g(z) the quantity 5+ z+1 z1
1/2

338

is always nonzero. On the other (negative) branch of the function, this quantity has a zero at z = 13/12. The logarithm introduces branch points at z = 1 on both the positive and negative branch of g(z). It introduces a branch point at z = 13/12 on the negative branch of g(z). To determine if additional branch cuts are needed to separate the branches, we consider 1/2 z+1 w =5+ z1 and see where the branch cut between 1 gets mapped to in the w plane. We rewrite the mapping. 2 w =5+ 1+ z1 The mapping is the following sequence of simple transformations: (a) z z 1 1 (b) z z (c) z 2z (d) z z + 1 (e) z z 1/2 (f) z z + 5 We show these transformations graphically below.
-1 1 -2 0 -1/2 -1
1/2

z z1

1 z 339

z 2z

z z+1

z z 1/2

z z+5

For the positive branch of g(z), the branch cut is mapped to the line x = 5 and the z plane is mapped to the half-plane x > 5. log(w) has branch points at w = 0 and w = . It is possible to walk around only one of these points in the half-plane x > 5. Thus no additional branch cuts are needed in the positive sheet of g(z). For the negative branch of g(z), the branch cut is mapped to the line x = 5 and the z plane is mapped to the half-plane x < 5. It is possible to walk around either w = 0 or w = alone in this half-plane. Thus we need an additional branch cut. On the negative sheet of g(z), we put a branch cut beteen z = 1 and z = 13/12. This puts a branch cut between w = and w = 0 and thus separates the branches of the logarithm. Figure 7.52 shows the branch cuts in the positive and negative sheets of g(z).
Im(z) g(13/12)=5 Re(z) Im(z) g(13/12)=-5 Re(z)

Figure 7.52: The branch cuts for f (z) = log 5 +

z+1 1/2 z1

3. The function f (z) = (z + 3)1/2 has a branch point at z = 3. The function is made single-valued by connecting this point and the point at innity with a branch cut. Solution 7.27 Note that the curve with opposite orientation goes around innity in the positive direction and does not enclose any branch points. Thus the value of the function does not change when traversing the curve, (with either orientation, of 340

course). This means that the argument of the function must change my an integer multiple of 2. Since the branch cut only allows us to encircle all three or none of the branch points, it makes the function single valued. Solution 7.28 We suppose that f (z) has only one branch point in the nite complex plane. Consider any contour that encircles this branch point in the positive direction. f (z) changes value if we traverse the contour. If we reverse the orientation of the contour, then it encircles innity in the positive direction, but contains no branch points in the nite complex plane. Since the function changes value when we traverse the contour, we conclude that the point at innity must be a branch point. If f (z) has only a single branch point in the nite complex plane then it must have a branch point at innity. If f (z) has two or more branch points in the nite complex plane then it may or may not have a branch point at innity. This is because the value of the function may or may not change on a contour that encircles all the branch points in the nite complex plane. Solution 7.29 First we factor the function, f (z) = z 4 + 1
1/4

1+ z 2

1/4

1 + 2

1/4

1 2

1/4

1 z 2

1/4

There are branch points at z =

1 . 2

We make the substitution z = 1/ to examine the point at innity. f 1 = = 1 +1 4 1 ( 4 )1/4


1/4

1 + 4

1/4

1/4 has a removable singularity at the point = 0, but no branch point there. Thus (z 4 + 1) has no branch point at innity. 1/4 Note that the argument of (z 4 z0 ) changes by /2 on a contour that goes around the point z0 once in the 1/4 positive direction. The argument of (z 4 + 1) changes by n/2 on a contour that goes around n of its branch points. 341

1/4

Thus any set of branch cuts that permit you to walk around only one, two or three of the branch points will not make the function single valued. A set of branch cuts that permit us to walk around only zero or all four of the branch points will make the function single-valued. Thus we see that the rst two sets of branch cuts in Figure 7.30 will make the function single-valued, while the remaining two will not. Consider the contour in Figure ??. There are two ways to see that the function does not change value while traversing the contour. The rst is to note that each of the branch points makes the argument of the function increase 1/4 by /2. Thus the argument of (z 4 + 1) changes by 4(/2) = 2 on the contour. This means that the value of the function changes by the factor e2 = 1. If we change the orientation of the contour, then it is a contour that encircles innity once in the positive direction. There are no branch points inside the this contour with opposite orientation. (Recall that the inside of a contour lies to your left as you walk around it.) Since there are no branch points inside this contour, the function cannot change value as we traverse it. Solution 7.30 f (z) = There are branch points at z = 0, . f 1 = 1/ (1/)2 + 1
1/3

z 2+1 z

1/3

= z 1/3 (z )1/3 (z + )1/3

1/3 (1 + 2 )1/3

There is a branch point at = 0. f (z) has a branch point at innity. We introduce branch cuts from z = 0 to innity on the negative real axis, from z = to innity on the positive imaginary axis and from z = to innity on the negative imaginary axis. As we cannot walk around any of the branch points, this makes the function single-valued. We dene a branch by dening angles from the branch points. Let z = r e (z ) = s e

< < , 3/2 < < /2, /2 < < 3/2. 342

(z + ) = t e

With f (z) = z 1/3 (z )1/3 (z + )1/3 1 1 = 3 r e/3 e/3 e/3 3 3 s t r ()/3 e = 3 st we have an explicit formula for computing the value of the function for this branch. Now we compute f (1) to see if we chose the correct ranges for the angles. (If not, well just change one of them.) f (1) =
3

1 1 e(0/4(/4))/3 = 3 2 2 2

We made the right choice for the angles. Now to compute f (1 + ). f (1 + ) =


3

2 e(/40Arctan(2))/3 = 1 5

2 (/4Arctan(2))/3 e 5

Consider the value of the function above and below the branch cut on the negative real axis. Above the branch cut the function is x e()/3 f (x + 0) = 3 2 + 1 x2 + 1 x Note that = so that f (x + 0) = Below the branch cut = and f (x 0) =
3 3

x e/3 = 2+1 x

x 1+ 3 . x2 + 1 2 x 1 3 . x2 + 1 2

x e()/3 = 2+1 x 343

For the branch cut along the positive imaginary axis, y e(/2/2/2)/3 (y 1)(y + 1) y e/6 (y 1)(y + 1) 3 y , (y 1)(y + 1) 2

f (y + 0) = = =

f (y 0) =

y e(/2(3/2)/2)/3 (y 1)(y + 1) y e/2 = 3 (y 1)(y + 1) y =3 . (y 1)(y + 1)


3

For the branch cut along the negative imaginary axis, y e(/2(/2)(/2))/3 (y + 1)(y 1) y e/6 (y + 1)(y 1) y 3+ , (y + 1)(y 1) 2 344

f (y + 0) = = =

f (y 0) =

y e(/2(/2)(3/2))/3 (y + 1)(y 1) y e/2 = 3 (y + 1)(y 1) y . = 3 (y + 1)(y 1)


3

Solution 7.31 First we factor the function. f (z) = ((z 1)(z 2)(z 3))1/2 = (z 1)1/2 (z 2)1/2 (z 3)1/2 There are branch points at z = 1, 2, 3. Now we examine the point at innity. f 1 = 1 1 1 2 1 3
1/2 1/2

= 3/2

Since 3/2 has a branch point at = 0 and the rest of the terms are analytic there, f (z) has a branch point at innity. The rst two sets of branch cuts in Figure 7.31 do not permit us to walk around any of the branch points, including the point at innity, and thus make the function single-valued. The third set of branch cuts lets us walk around the branch points at z = 1 and z = 2 together or if we change our perspective, we would be walking around the branch points at z = 3 and z = together. Consider a contour in this cut plane that encircles the branch points at z = 1 and z = 2. Since the argument of (z z0 )1/2 changes by when we walk around z0 , the argument of f (z) changes by 2 when we traverse the contour. Thus the value of the function does not change and it is a valid set of branch cuts. Clearly the fourth set of branch cuts does not make the function single-valued as there are contours that encircle the branch point at innity and no other branch points. The other way to see this is to note that the argument of f (z) changes by 3 as we traverse a contour that goes around the branch points at z = 1, 2, 3 once in the positive direction. Now to dene the branch. We make the preliminary choice of angles, z 1 = r1 e1 , z 2 = r2 e , z 3 = r3 e3 , 345
2

0 < 1 < 2, 0 < 2 < 2, 0 < 3 < 2.

The function is f (z) = r1 e1 r2 e2 r3 e3 The value of the function at the origin is f (0) =
1/2

r1 r2 r3 e(1 +2 +3 )/2 .

6 e(3)/2 = 6,

which is not what we wanted. We will change range of one of the angles to get the desired result. z 1 = r1 e1 , z 2 = r2 e2 , z 3 = r3 e , f (0) = Solution 7.32 w= z 2 2 (z + 2)
1/3 3

0 < 1 < 2, 0 < 2 < 2, 2 < 3 < 4.

6 e(5)/2 = 6,

z+

1/3

1/3

(z + 2)1/3

There are branch points at z = 2 and z = 2. If we walk around any one of the branch points once in the positive direction, the argument of w changes by 2/3 and thus the value of the function changes by e2/3 . If we walk around all three branch points then the argument of w changes by 2/3 = 2. The value of the function is unchanged as 3 e2 = 1. Thus the branch cut on the real axis from 2 to 2 makes the function single-valued. Now we dene a branch. Let z 2 = a e , z+ 2 = b e , z + 2 = c e .

We constrain the angles as follows: On the positive real axis, = = . See Figure 7.53. 346

Im(z) c b a Re(z)

Figure 7.53: A branch of ((z 2 2) (z + 2))

1/3

Now we determine w(2).

w(2) = 2 =
3

1/3

2
3

2+

1/3

2 2 e0

(2 + 2)1/3 3 4 e0

2 2 e0 3 3 = 2 4 = 2.

2+

Note that we didnt have to choose the angle from each of the branch points as zero. Choosing any integer multiple of 2 would give us the same result. 347

w(3) = 3 = =
3

1/3

2
3

3 + 3

1/3

(3 + 2)1/3 3 1 e/3

7 e 3 = 7 3

3+

2 e/3

2 e/3

The value of the function is w=

abc e(++)/3 .

Consider the interval 2 . . . 2 . As we approach the branch cut from above, the function has the value, w= 3 abc e/3 =
3

2x

x+

2 (x + 2) e/3 .

As we approach the branch cut from below, the function has the value, w= 3 abc e/3 =
3

2x

x+

2 (x + 2) e/3 .

Consider the interval 2 . . . w=

2 . As we approach the branch cut from above, the function has the value,
3

abc e2/3 =

2x

2 (x + 2) e2/3 .

As we approach the branch cut from below, the function has the value, w= 3 abc e2/3 =
3

2x

2 (x + 2) e2/3 .

348

3 2.5 2 1.5 1 0.5 -1 -0.5 0.5 1

Figure 7.54: The Principal Branch of the arc cosine, Arccos(x). Solution 7.33 Arccos(x) is shown in Figure 7.54 for real variables in the range [1 . . . 1]. First we write arccos(z) in terms of log(z). If cos(w) = z, then w = arccos(z). cos(w) = z ew + ew =z 2 (ew )2 2z ew +1 = 0 ew = z + z 2 1
1/2 1/2

w = log z + z 2 1 Thus we have

arccos(z) = log z + z 2 1 Since Arccos(0) = , we must nd the branch such that 2 log 0 + 02 1
1/2

1/2

=0

log (1)1/2 = 0. 349

Since log() = and

+ 2n = + 2n 2 2

+ 2n = + 2n 2 2 1/2 we must choose the branch of the square root such that (1) = and the branch of the logarithm such that log() = 2 . First we construct the branch of the square root. log() = z2 1
1/2

= (z + 1)1/2 (z 1)1/2

We see that there are branch points at z = 1 and z = 1. In particular we want the Arccos to be dened for z = x, x [1 . . . 1]. Hence we introduce branch cuts on the lines < x 1 and 1 x < . Dene the local coordinates z + 1 = r e , z 1 = e . With the given branch cuts, the angles have the possible ranges {} = {. . . , ( . . . ), ( . . . 3), . . .}, {} = {. . . , (0 . . . 2), (2 . . . 4), . . .}.

Now we choose ranges for and and see if we get the desired branch. If not, we choose a dierent range for one of the angles. First we choose the ranges ( . . . ), If we substitute in z = 0 we get 02 1
1/2

(0 . . . 2).

= 1 e0

1/2

(1 e )1/2 = e0 e/2 =

Thus we see that this choice of angles gives us the desired branch. Now we go back to the expression arccos(z) = log z + z 2 1 350
1/2

= =

=0 =2

Figure 7.55: Branch Cuts and Angles for (z 2 1)

1/2

We have already seen that there are branch points at z = 1 and z = 1 because of (z 2 1) . Now we must determine if the logarithm introduces additional branch points. The only possibilities for branch points are where the argument of the logarithm is zero. z + z2 1 =0 2 2 z =z 1 0 = 1 We see that the argument of the logarithm is nonzero and thus there are no additional branch points. Introduce the 1/2 variable, w = z + (z 2 1) . What is the image of the branch cuts in the w plane? We parameterize the branch cut connecting z = 1 and z = + with z = r + 1, r [0 . . . ). w = r + 1 + (r + 1)2 1 =r+1 =r 1r r(r + 2) 1 + 2/r + 1
1/2 1/2

1/2

r 1 + 1 + 2/r + 1 is the interval [1 . . . ); r 1 1 + 2/r + 1 is the interval (0 . . . 1]. Thus we see that this branch cut is mapped to the interval (0 . . . ) in the w plane. Similarly, we could show that the branch cut ( . . .1] 351

in the z plane is mapped to ( . . . 0) in the w plane. In the w plane there is a branch cut along the real w axis from to . Thus cut makes the logarithm single-valued. For the branch of the square root that we chose, all the points in the z plane get mapped to the upper half of the w plane. With the branch cuts we have introduced so far and the chosen branch of the square root we have arccos(0) = log 0 + 02 1 = log = + 2n 2 = + 2n 2 Choosing the n = 0 branch of the logarithm will give us Arccos(z). We see that we can write Arccos(z) = Log z + z 2 1
1/2 1/2

Solution 7.34 1/2 We consider the function f (z) = z 1/2 1 . First note that z 1/2 has a branch point at z = 0. We place a branch cut on the negative real axis to make it single valued. f (z) will have a branch point where z 1/2 1 = 0. This occurs at z = 1 on the branch of z 1/2 on which 11/2 = 1. (11/2 has the value 1 on one branch of z 1/2 and 1 on the other branch.) For this branch we introduce a branch cut connecting z = 1 with the point at innity. (See Figure 7.56.)
1/2 1/2

1 =1

1 =-1

Figure 7.56: Branch Cuts for z 1/2 1

1/2

352

Solution 7.35 The distance between the end of rod a and the end of rod c is b. In the complex plane, these points are a e and l + c e , respectively. We write this out mathematically. l + c e a e = b l + c e a e l + c e a e = b2 1 2 b a2 c 2 l 2 2 l2 + cl e al e +cl e +c2 ac e() al e ac e() +a2 = b2 cl cos ac cos( ) al cos =

This equation relates the two angular positions. One could dierentiate the equation to relate the velocities and accelerations. Solution 7.36 1. Let w = u + v. First we do the strip: | (z)| < 1. Consider the vertical line: z = c + y, y R. This line is mapped to w = 2(c + y)2 w = 2c2 2y 2 + 4cy u = 2c2 2y 2 , v = 4cy This is a parabola that opens to the left. For the case c = 0 it is the negative u axis. We can parametrize the curve in terms of v. 1 u = 2c2 2 v 2 , v R 8c The boundaries of the region are both mapped to the parabolas: 1 u = 2 v2, 8 The image of the mapping is 1 w = u + v : v R and u < 2 v 2 . 8 353 v R.

Note that the mapping is two-to-one. Now we do the strip 1 < (z) < 2. Consider the horizontal line: z = x + c, x R. This line is mapped to w = 2(x + c)2 w = 2x2 2c2 + 4cx u = 2x2 2c2 , v = 4cx This is a parabola that opens upward. We can parametrize the curve in terms of v. u= The boundary (z) = 1 is mapped to 1 u = v 2 2, 8 The boundary (z) = 2 is mapped to u= The image of the mapping is w = u + v : v R and 2. We write the transformation as 1 2 1 v 8 < u < v2 2 . 32 8 1 2 v 8, 32 vR v R. 1 2 v 2c2 , 2 8c vR

z+1 2 =1+ . z1 z1 Thus we see that the transformation is the sequence: (a) translation by 1 (b) inversion 354

(c) magnication by 2 (d) translation by 1 Consider the strip | (z)| < 1. The translation by 1 maps this to 2 < (z) < 0. Now we do the inversion. The left edge, (z) = 0, is mapped to itself. The right edge, (z) = 2, is mapped to the circle |z +1/4| = 1/4. Thus the current image is the left half plane minus a circle: (z) < 0 and The magnication by 2 yields (z) < 0 and The nal step is a translation by 1. (z) < 1 and z 1 1 > . 2 2 z+ 1 1 > . 2 2 z+ 1 1 > . 4 4

Now consider the strip 1 < (z) < 2. The translation by 1 does not change the domain. Now we do the inversion. The bottom edge, (z) = 1, is mapped to the circle |z + /2| = 1/2. The top edge, (z) = 2, is mapped to the circle |z + /4| = 1/4. Thus the current image is the region between two circles: z+ The magnication by 2 yields |z + | < 1 and The nal step is a translation by 1. |z 1 + | < 1 and z1+ 1 > . 2 2 z+ 1 > . 2 2 1 < 2 2 and z+ 1 > . 4 4

355

Solution 7.37 1. There is a simple pole at z = 2. The function has a branch point at z = 1. Since this is the only branch point in the nite complex plane there is also a branch point at innity. We can verify this with the substitution z = 1/. f 1 (1/ + 1)1/2 1/ + 2 1/2 (1 + )1/2 = 1 + 2 =

Since f (1/) has a branch point at = 0, f (z) has a branch point at innity. 2. cos z is an entire function with an essential singularity at innity. Thus f (z) has singularities only where 1/(1 + z) has singularities. 1/(1 + z) has a rst order pole at z = 1. It is analytic everywhere else, including the point at innity. Thus we conclude that f (z) has an essential singularity at z = 1 and is analytic elsewhere. To explicitly show that z = 1 is an essential singularity, we can nd the Laurent series expansion of f (z) about z = 1. cos 1 1+z

=
n=0

(1)n (z + 1)2n (2n)!

3. 1 ez has simple zeros at z = 2n, n Z. Thus f (z) has second order poles at those points. The point at innity is a non-isolated singularity. To justify this: Note that f (z) = 1 (1 ez )2

1 has second order poles at z = 2n, n Z. This means that f (1/) has second order poles at = 2n , n Z. These second order poles get arbitrarily close to = 0. There is no deleted neighborhood around = 0 in which f (1/) is analytic. Thus the point = 0, (z = ), is a non-isolated singularity. There is no Laurent series expansion about the point = 0, (z = ).

356

The point at innity is neither a branch point nor a removable singularity. It is not a pole either. If it were, there would be an n such that limz z n f (z) = const = 0. Since z n f (z) has second order poles in every deleted neighborhood of innity, the above limit does not exist. Thus we conclude that the point at innity is an essential singularity. Solution 7.38 We write sinh z in Cartesian form. w = sinh z = sinh x cos y + cosh x sin y = u + v Consider the line segment x = c, y (0 . . . ). Its image is {sinh c cos y + cosh c sin y | y (0 . . . )}. This is the parametric equation for the upper half of an ellipse. Also note that u and v satisfy the equation for an ellipse. u2 v2 + =1 sinh2 c cosh2 c The ellipse starts at the point (sinh(c), 0), passes through the point (0, cosh(c)) and ends at (sinh(c), 0). As c varies from zero to or from zero to , the semi-ellipses cover the upper half w plane. Thus the mapping is 2-to-1. Consider the innite line y = c, x ( . . . ).Its image is {sinh x cos c + cosh x sin c | x ( . . . )}. This is the parametric equation for the upper half of a hyperbola. Also note that u and v satisfy the equation for a hyperbola. v2 u2 2 + =1 cos c sin2 c As c varies from 0 to /2 or from /2 to , the semi-hyperbola cover the upper half w plane. Thus the mapping is 2-to-1. 357

We look for branch points of sinh1 w. w = sinh z ez ez w= 2 e2z 2w ez 1 = 0 ez = w + w 2 + 1


1/2

z = log w + (w )1/2 (w + )1/2 There are branch points at w = . Since w + (w2 + 1) is nonzero and nite in the nite complex plane, the logarithm does not introduce any branch points in the nite plane. Thus the only branch point in the upper half w plane is at w = . Any branch cut that connects w = with the boundary of (w) > 0 will separate the branches under the inverse mapping. Consider the line y = /4. The image under the mapping is the upper half of the hyperbola 2u2 + 2v 2 = 1. Consider the segment x = 1.The image under the mapping is the upper half of the ellipse u2 v2 + = 1. sinh2 1 cosh2 1
1/2

358

Chapter 8 Analytic Functions


Students need encouragement. So if a student gets an answer right, tell them it was a lucky guess. That way, they develop a good, lucky feeling.1 -Jack Handey

8.1

Complex Derivatives

Functions of a Real Variable. The derivative of a function of a real variable is f (x + x) f (x) d f (x) = lim . x0 dx x If the limit exists then the function is dierentiable at the point x. Note that x can approach zero from above or below. The limit cannot depend on the direction in which x vanishes. Consider f (x) = |x|. The function is not dierentiable at x = 0 since
x0
1

lim +

|0 + x| |0| =1 x

Quote slightly modied.

359

and
x0

lim

|0 + x| |0| = 1. x

Analyticity. The complex derivative, (or simply derivative if the context is clear), is dened, f (z + z) f (z) d f (z) = lim . z0 dz z The complex derivative exists if this limit exists. This means that the value of the limit is independent of the manner in which z 0. If the complex derivative exists at a point, then we say that the function is complex dierentiable there. A function of a complex variable is analytic at a point z0 if the complex derivative exists in a neighborhood about that point. The function is analytic in an open set if it has a complex derivative at each point in that set. Note that complex dierentiable has a dierent meaning than analytic. Analyticity refers to the behavior of a function on an open set. A function can be complex dierentiable at isolated points, but the function would not be analytic at those points. Analytic functions are also called regular or holomorphic. If a function is analytic everywhere in the nite complex plane, it is called entire. Example 8.1.1 Consider z n , n Z+ , Is the function dierentiable? Is it analytic? What is the value of the derivative? We determine dierentiability by trying to dierentiate the function. We use the limit denition of dierentiation. We will use Newtons binomial formula to expand (z + z)n . d n (z + z)n z n z = lim z0 dz z z n + nz n1 z + = lim = lim
z0 n(n1) n2 z z 2 2

+ + z n z n

z nz n1 + n(n 1) n2 z z + + z n1 2

z0

= nz n1 360

The derivative exists everywhere. The function is analytic in the whole complex plane so it is entire. The value of the d derivative is dz = nz n1 . Example 8.1.2 We will show that f (z) = z is not dierentiable. Consider its derivative. d f (z + z) f (z) f (z) = lim . z0 dz z

z + z z d z = lim z0 z dz z = lim z0 z First we take z = x and evaluate the limit. x =1 x0 x lim Then we take z = y.
y0

lim

y = 1 y

Since the limit depends on the way that z 0, the function is nowhere dierentiable. Thus the function is not analytic.

Complex Derivatives in Terms of Plane Coordinates. Let z = (, ) be a system of coordinates in the complex plane. (For example, we could have Cartesian coordinates z = (x, y) = x + y or polar coordinates z = (r, ) = r e ). Let f (z) = (, ) be a complex-valued function. (For example we might have a function in the form (x, y) = u(x, y) + v(x, y) or (r, ) = R(r, ) e(r,) .) If f (z) = (, ) is analytic, its complex derivative is 361

equal to the derivative in any direction. In particular, it is equal to the derivatives in the coordinate directions. df f (z + z) f (z) ( + , ) (, ) = = lim = lim 0,=0 0 dz z df f (z + z) f (z) (, + ) (, ) = lim = lim = =0,0 0 dz z
1

Example 8.1.3 Consider the Cartesian coordinates z = x + y. We write the complex derivative as derivatives in the coordinate directions for f (z) = (x, y). df = dz df = dz We write this in operator notation. d = = . dz x y Example 8.1.4 In Example 8.1.1 we showed that z n , n Z+ , is an entire function and that corroborate this by calculating the complex derivative in the Cartesian coordinate directions. d n z = (x + y)n dz x = n(x + y)n1 = nz n1 362
d n z dz

(x + y) x (x + y) y

= x x

= y y

= nz n1 . Now we

d n z = (x + y)n dz y = n(x + y)n1 = nz n1

Complex Derivatives are Not the Same as Partial Derivatives Recall from calculus that f (x, y) = g(s, t) f g s g t = + x s x t x

Do not make the mistake of using a similar formula for functions of a complex variable. If f (z) = (x, y) then df x y = + . dz x z y z
d This is because the dz operator means The derivative in any direction in the complex plane. Since f (z) is analytic, f (z) is the same no matter in which direction we take the derivative.

Rules of Dierentiation. For an analytic function dened in terms of z we can calculate the complex derivative using all the usual rules of dierentiation that we know from calculus like the product rule, d f (z)g(z) = f (z)g(z) + f (z)g (z), dz or the chain rule, d f (g(z)) = f (g(z))g (z). dz This is because the complex derivative derives its properties from properties of limits, just like its real variable counterpart. 363

Result 8.1.1 The complex derivative is, f (z + z) f (z) d f (z) = lim . z0 dz z The complex derivative is dened if the limit exists and is independent of the manner in which z 0. A function is analytic at a point if the complex derivative exists in a neighborhood of that point. Let z = (, ) dene coordinates in the complex plane. The complex derivative in the coordinate directions is 1 1 d = = . dz In Cartesian coordinates, this is d = = . dz x y In polar coordinates, this is d = e = e dz r r Since the complex derivative is dened with the same limit formula as real derivatives, all the rules from the calculus of functions of a real variable may be used to dierentiate functions of a complex variable.

Example 8.1.5 We have shown that z n , n Z+ , is an entire function. Now we corroborate that 364

d n z dz

= nz n1 by

calculating the complex derivative in the polar coordinate directions. d n z = e rn en dz r = e nrn1 en = nrn1 e(n1) = nz n1

d n z = e rn en dz r n = e r n en r = nrn1 e(n1) = nz n1

Analytic Functions can be Written in Terms of z. Consider an analytic function expressed in terms of x and y, (x, y). We can write as a function of z = x + y and z = x y. f (z, z) = z+z zz , 2 2

We treat z and z as independent variables. We nd the partial derivatives with respect to these variables. x y 1 = + = z z x z y 2 x y 1 = + = z z x z y 2 365 x y + x y

Since is analytic, the complex derivatives in the x and y directions are equal. = x y The partial derivative of f (z, z) with respect to z is zero. f 1 = z 2 + x y =0

Thus f (z, z) has no functional dependence on z, it can be written as a function of z alone. If we were considering an analytic function expressed in polar coordinates (r, ), then we could write it in Cartesian coordinates with the substitutions: r = x2 + y 2 , = arctan(x, y). Thus we could write (r, ) as a function of z alone.

Result 8.1.2 Any analytic function (x, y) or (r, ) can be written as a function of z alone.

8.2

Cauchy-Riemann Equations

If we know that a function is analytic, then we have a convenient way of determining its complex derivative. We just express the complex derivative in terms of the derivative in a coordinate direction. However, we dont have a nice way of determining if a function is analytic. The denition of complex derivative in terms of a limit is cumbersome to work with. In this section we remedy this problem. A necessary condition for analyticity. Consider a function f (z) = (x, y). If f (z) is analytic, the complex derivative is equal to the derivatives in the coordinate directions. We equate the derivatives in the x and y directions to obtain the Cauchy-Riemann equations in Cartesian coordinates. x = y 366 (8.1)

This equation is a necessary condition for the analyticity of f (z). Let (x, y) = u(x, y) + v(x, y) where u and v are real-valued functions. We equate the real and imaginary parts of Equation 8.1 to obtain another form for the Cauchy-Riemann equations in Cartesian coordinates. ux = v y , uy = vx .

Note that this is a necessary and not a sucient condition for analyticity of f (z). That is, u and v may satisfy the Cauchy-Riemann equations but f (z) may not be analytic. At this point, Cauchy-Riemann equations give us an easy test for determining if a function is not analytic. Example 8.2.1 In Example 8.1.2 we showed that z is not analytic using the denition of complex dierentiation. Now we obtain the same result using the Cauchy-Riemann equations. z = x y ux = 1, vy = 1 We see that the rst Cauchy-Riemann equation is not satised; the function is not analytic at any point.

A sucient condition for analyticity. A sucient condition for f (z) = (x, y) to be analytic at a point z0 = (x0 , y0 ) is that the partial derivatives of (x, y) exist and are continuous in some neighborhood of z0 and satisfy the Cauchy-Riemann equations there. If the partial derivatives of exist and are continuous then (x + x, y + y) = (x, y) + xx (x, y) + yy (x, y) + o(x) + o(y). 367

Here the notation o(x) means terms smaller than x. We calculate the derivative of f (z). f (z) = lim f (z + z) f (z) z0 z (x + x, y + y) (x, y) = lim x,y0 x + y (x, y) + xx (x, y) + yy (x, y) + o(x) + o(y) (x, y) = lim x,y0 x + y xx (x, y) + yy (x, y) + o(x) + o(y) = lim x,y0 x + y

Here we use the Cauchy-Riemann equations. (x + y)x (x, y) o(x) + o(y) + lim x,y0 x,y0 x + y x + y = x (x, y) = lim Thus we see that the derivative is well dened. Cauchy-Riemann Equations in General Coordinates Let z = (, ) be a system of coordinates in the complex plane. Let (, ) be a function which we write in terms of these coordinates, A necessary condition for analyticity of (, ) is that the complex derivatives in the coordinate directions exist and are equal. Equating the derivatives in the and directions gives us the Cauchy-Riemann equations.
1

We could separate this into two equations by equating the real and imaginary parts or the modulus and argument. 368

Result 8.2.1 A necessary condition for analyticity of (, ), where z = (, ), at z = z0 is that the Cauchy-Riemann equations are satised in a neighborhood of z = z0 .
1

(We could equate the real and imaginary parts or the modulus and argument of this to obtain two equations.) A sucient condition for analyticity of f (z) is that the Cauchy-Riemann equations hold and the rst partial derivatives of exist and are continuous in a neighborhood of z = z0 . Below are the Cauchy-Riemann equations for various forms of f (z). f (z) = (x, y), f (z) = u(x, y) + v(x, y), f (z) = (r, ), f (z) = u(r, ) + v(r, ), f (z) = R(r, ) e(r,) , f (z) = R(x, y) e(x,y) , x = y ux = vy , uy = vx r = r 1 ur = v , u = rvr r R 1 Rr = , R = Rr r r Rx = Ry , Ry = Rx

Example 8.2.2 Consider the Cauchy-Riemann equations for f (z) = u(r, ) + v(r, ). From Exercise 8.3 we know that the complex derivative in the polar coordinate directions is d = e = e . dz r r 369

From Result 8.2.1 we have the equation, e [u + v] = e [u + v]. r r

We multiply by e and equate the real and imaginary components to obtain the Cauchy-Riemann equations. 1 ur = v , r Example 8.2.3 Consider the exponential function. ez = (x, y) = ex (cos y + sin(y)) We use the Cauchy-Riemann equations to show that the function is entire. x = y e (cos y + sin(y)) = ex ( sin y + cos(y)) ex (cos y + sin(y)) = ex (cos y + sin(y))
x

u = rvr

Since the function satises the Cauchy-Riemann equations and the rst partial derivatives are continuous everywhere in the nite complex plane, the exponential function is entire. Now we nd the value of the complex derivative. d z e = = ex (cos y + sin(y)) = ez dz x The dierentiability of the exponential function implies the dierentiability of the trigonometric functions, as they can be written in terms of the exponential. In Exercise 8.13 you can show that the logarithm log z is dierentiable for z = 0. This implies the dierentiability of z and the inverse trigonometric functions as they can be written in terms of the logarithm. 370

Example 8.2.4 We compute the derivative of z z . d z d z log z e (z ) = dz dz = (1 + log z) ez log z = (1 + log z)z z = z z + z z log z

8.3

Harmonic Functions

A function u is harmonic if its second partial derivatives exist, are continuous and satisfy Laplaces equation u = 0.2 (In Cartesian coordinates the Laplacian is u uxx + uyy .) If f (z) = u + v is an analytic function then u and v are harmonic functions. To see why this is so, we start with the Cauchy-Riemann equations. ux = v y , uy = vx

We dierentiate the rst equation with respect to x and the second with respect to y. (We assume that u and v are twice continuously dierentiable. We will see later that they are innitely dierentiable.) uxx = vxy , Thus we see that u is harmonic. u uxx + uyy = vxy vyx = 0 One can use the same method to show that v = 0.
2

uyy = vyx

The capital Greek letter is used to denote the Laplacian, like u(x, y), and dierentials, like x.

371

If u is harmonic on some simply-connected domain, then there exists a harmonic function v such that f (z) = u + v is analytic in the domain. v is called the harmonic conjugate of u. The harmonic conjugate is unique up to an additive constant. To demonstrate this, let w be another harmonic conjugate of u. Both the pair u and v and the pair u and w satisfy the Cauchy-Riemann equations. ux = v y , We take the dierence of these equations. vx wx = 0, vy wy = 0 uy = vx , ux = wy , uy = wx

On a simply connected domain, the dierence between v and w is thus a constant. To prove the existence of the harmonic conjugate, we rst write v as an integral.
(x,y)

v(x, y) = v (x0 , y0 ) +
(x0 ,y0 )

vx dx + vy dy

On a simply connected domain, the integral is path independent and denes a unique v in terms of vx and vy . We use the Cauchy-Riemann equations to write v in terms of ux and uy .
(x,y)

v(x, y) = v (x0 , y0 ) +
(x0 ,y0 )

uy dx + ux dy

Changing the starting point (x0 , y0 ) changes v by an additive constant. The harmonic conjugate of u to within an additive constant is v(x, y) = uy dx + ux dy. This proves the existence3 of the harmonic conjugate. This is not the formula one would use to construct the harmonic conjugate of a u. One accomplishes this by solving the Cauchy-Riemann equations.
A mathematician returns to his oce to nd that a cigarette tossed in the trash has started a small re. Being calm and a quick thinker he notes that there is a re extinguisher by the window. He then closes the door and walks away because the solution exists.
3

372

Result 8.3.1 If f (z) = u + v is an analytic function then u and v are harmonic functions. That is, the Laplacians of u and v vanish u = v = 0. The Laplacian in Cartesian and polar coordinates is 2 2 = 2 + 2, x y 1 = r r r r 1 2 + 2 2. r

Given a harmonic function u in a simply connected domain, there exists a harmonic function v, (unique up to an additive constant), such that f (z) = u + v is analytic in the domain. One can construct v by solving the Cauchy-Riemann equations.
Example 8.3.1 Is x2 the real part of an analytic function? The Laplacian of x2 is [x2 ] = 2 + 0 x2 is not harmonic and thus is not the real part of an analytic function. Example 8.3.2 Show that u = ex (x sin y y cos y) is harmonic. u = ex sin y ex (x sin y y cos y) x = ex sin y x ex sin y + y ex cos y 2u = ex sin y ex sin y + x ex sin y y ex cos y 2 x = 2 ex sin y + x ex sin y y ex cos y u = ex (x cos y cos y + y sin y) y 373

2u = ex (x sin y + sin y + y cos y + sin y) y 2 = x ex sin y + 2 ex sin y + y ex cos y Thus we see that
2u x2

2u y 2

= 0 and u is harmonic.

Example 8.3.3 Consider u = cos x cosh y. This function is harmonic. uxx + uyy = cos x cosh y + cos x cosh y = 0 Thus it is the real part of an analytic function, f (z). We nd the harmonic conjugate, v, with the Cauchy-Riemann equations. We integrate the rst Cauchy-Riemann equation. vy = ux = sin x cosh y v = sin x sinh y + a(x) Here a(x) is a constant of integration. We substitute this into the second Cauchy-Riemann equation to determine a(x). vx = uy cos x sinh y + a (x) = cos x sinh y a (x) = 0 a(x) = c Here c is a real constant. Thus the harmonic conjugate is v = sin x sinh y + c. The analytic function is f (z) = cos x cosh y sin x sinh y + c We recognize this as f (z) = cos z + c. 374

Example 8.3.4 Here we consider an example that demonstrates the need for a simply connected domain. Consider u = Log r in the multiply connected domain, r > 0. u is harmonic. 1 2 1 r Log r + 2 2 Log r = 0 r r r r We solve the Cauchy-Riemann equations to try to nd the harmonic conjugate. 1 ur = v , u = rvr r vr = 0, v = 1 v =+c Log r = We are able to solve for v, but it is multi-valued. Any single-valued branch of that we choose will not be continuous on the domain. Thus there is no harmonic conjugate of u = Log r for the domain r > 0. If we had instead considered the simply-connected domain r > 0, | arg(z)| < then the harmonic conjugate would be v = Arg(z) + c. The corresponding analytic function is f (z) = Log z + c. Example 8.3.5 Consider u = x3 3xy 2 + x. This function is harmonic. uxx + uyy = 6x 6x = 0 Thus it is the real part of an analytic function, f (z). We nd the harmonic conjugate, v, with the Cauchy-Riemann equations. We integrate the rst Cauchy-Riemann equation. vy = ux = 3x2 3y 2 + 1 v = 3x2 y y 3 + y + a(x) Here a(x) is a constant of integration. We substitute this into the second Cauchy-Riemann equation to determine a(x). vx = uy 6xy + a (x) = 6xy a (x) = 0 a(x) = c 375

Here c is a real constant. The harmonic conjugate is v = 3x2 y y 3 + y + c. The analytic function is f (z) = x3 3xy 2 + x + 3x2 y y 3 + y + c f (z) = x3 + 3x2 y 3xy 2 y 2 + x + y + c f (z) = z 3 + z + c

8.4

Singularities

Any point at which a function is not analytic is called a singularity. In this section we will classify the dierent avors of singularities.

Result 8.4.1 Singularities. If a function is not analytic at a point, then that point is a singular point or a singularity of the function. 8.4.1 Categorization of Singularities

Branch Points. If f (z) has a branch point at z0 , then we cannot dene a branch of f (z) that is continuous in a neighborhood of z0 . Continuity is necessary for analyticity. Thus all branch points are singularities. Since function are discontinuous across branch cuts, all points on a branch cut are singularities. Example 8.4.1 Consider f (z) = z 3/2 . The origin and innity are branch points and are thus singularities of f (z). We choose the branch g(z) = z 3 . All the points on the negative real axis, including the origin, are singularities of g(z).

376

Removable Singularities. Example 8.4.2 Consider sin z . z This function is undened at z = 0 because f (0) is the indeterminate form 0/0. f (z) is analytic everywhere in the nite complex plane except z = 0. Note that the limit as z 0 of f (z) exists. f (z) = cos z sin z = lim =1 z0 z z0 1 lim If we were to ll in the hole in the denition of f (z), we could make it dierentiable at z = 0. Consider the function g(z) =
sin z z

z = 0, z = 0.

We calculate the derivative at z = 0 to verify that g(z) is analytic there. f (0) = lim f (0) f (z) z0 z 1 sin(z)/z = lim z0 z z sin(z) = lim z0 z2 1 cos(z) = lim z0 2z sin(z) = lim z0 2 =0

We call the point at z = 0 a removable singularity of sin(z)/z because we can remove the singularity by dening the value of the function to be its limiting value there. 377

Consider a function f (z) that is analytic in a deleted neighborhood of z = z0 . If f (z) is not analytic at z0 , but limzz0 f (z) exists, then the function has a removable singularity at z0 . The function g(z) = f (z) z = z0 limzz0 f (z) z = z0 g (z0 ) g(z) zz0 z0 z g (z) = lim zz0 1 = lim f (z)
zz0

is analytic in a neighborhood of z = z0 . We show this by calculating g (z0 ). g (z0 ) = lim

This limit exists because f (z) is analytic in a deleted neighborhood of z = z0 . Poles. If a function f (z) behaves like c/ (z z0 )n near z = z0 then the function has an nth order pole at that point. More mathematically we say lim (z z0 )n f (z) = c = 0.
zz0

We require the constant c to be nonzero so we know that it is not a pole of lower order. We can denote a removable singularity as a pole of order zero. Another way to say that a function has an nth order pole is that f (z) is not analytic at z = z0 , but (z z0 )n f (z) is either analytic or has a removable singularity at that point. Example 8.4.3 1/ sin (z 2 ) has a second order pole at z = 0 and rst order poles at z = (n)1/2 , n Z . z2 2z = lim 2) z0 sin (z z0 2z cos (z 2 ) lim = lim =1 378
z0

2 cos (z 2 )

2 4z 2 sin (z 2 )

lim
z(n)1/2

z (n)1/2 1 = lim 2) 1/2 2z cos (z 2 ) sin (z z(n) 1 = 1/2 (1)n 2(n)

Example 8.4.4 e1/z is singular at z = 0. The function is not analytic as limz0 e1/z does not exist. We check if the function has a pole of order n at z = 0. e n e = lim n!

z0

lim z n e1/z = lim

Since the limit does not exist for any value of n, the singularity is not a pole. We could say that e1/z is more singular than any power of 1/z.

Essential Singularities. If a function f (z) is singular at z = z0 , but the singularity is not a branch point, or a pole, the the point is an essential singularity of the function.

The point at innity. We can consider the point at innity z by making the change of variables z = 1/ and considering 0. If f (1/) is analytic at = 0 then f (z) is analytic at innity. We have encountered branch points at innity before (Section 7.8). Assume that f (z) is not analytic at innity. If limz f (z) exists then f (z) has a removable singularity at innity. If limz f (z)/z n = c = 0 then f (z) has an nth order pole at innity. 379

Result 8.4.2 Categorization of Singularities. Consider a function f (z) that has a singularity at the point z = z0 . Singularities come in four avors: Branch Points. Branch points of multi-valued functions are singularities. Removable Singularities. If limzz0 f (z) exists, then z0 is a removable singularity. It is thus named because the singularity could be removed and thus the function made analytic at z0 by redening the value of f (z0 ). Poles. If limzz0 (z z0 )n f (z) = const = 0 then f (z) has an nth order pole at z0 . Essential Singularities. Instead of dening what an essential singularity is, we say what it is not. If z0 neither a branch point, a removable singularity nor a pole, it is an essential singularity.

A pole may be called a non-essential singularity. This is because multiplying the function by an integral power of z z0 will make the function analytic. Then an essential singularity is a point z0 such that there does not exist an n such that (z z0 )n f (z) is analytic there.

8.4.2

Isolated and Non-Isolated Singularities

Result 8.4.3 Isolated and Non-Isolated Singularities. Suppose f (z) has a singularity at z0 . If there exists a deleted neighborhood of z0 containing no singularities then the point is an isolated singularity. Otherwise it is a non-isolated singularity.

380

If you dont like the abstract notion of a deleted neighborhood, you can work with a deleted circular neighborhood. However, this will require the introduction of more math symbols and a Greek letter. z = z0 is an isolated singularity if there exists a > 0 such that there are no singularities in 0 < |z z0 | < . Example 8.4.5 We classify the singularities of f (z) = z/ sin z. z has a simple zero at z = 0. sin z has simple zeros at z = n. Thus f (z) has a removable singularity at z = 0 and has rst order poles at z = n for n Z . We can corroborate this by taking limits. lim f (z) = lim z 1 = lim =1 z0 sin z z0 cos z (z n)z zn sin z 2z n = lim zn cos z n = (1)n =0

z0

zn

lim (z n)f (z) = lim

Now to examine the behavior at innity. There is no neighborhood of innity that does not contain rst order poles of f (z). (Another way of saying this is that there does not exist an R such that there are no singularities in R < |z| < .) Thus z = is a non-isolated singularity. We could also determine this by setting = 1/z and examining the point = 0. f (1/) has rst order poles at = 1/(n) for n Z \ {0}. These rst order poles come arbitrarily close to the point = 0 There is no deleted neighborhood of = 0 which does not contain singularities. Thus = 0, and hence z = is a non-isolated singularity. The point at innity is an essential singularity. It is certainly not a branch point or a removable singularity. It is not a pole, because there is no n such that limz z n f (z) = const = 0. z n f (z) has rst order poles in any neighborhood of innity, so this limit does not exist.

381

8.5

Application: Potential Flow

Example 8.5.1 We consider 2 dimensional uniform ow in a given direction. The ow corresponds to the complex potential (z) = v0 e0 z, where v0 is the uid speed and 0 is the direction. We nd the velocity potential and stream function . (z) = + = v0 (cos(0 )x + sin(0 )y), = v0 ( sin(0 )x + cos(0 )y) These are plotted in Figure 8.1 for 0 = /6.

1 0 -1 -1 -0.5 0 0.5

1 0.5 0 -0.5 1-1

1 0 -1 -1 -0.5 0 0.5

1 0.5 0 -0.5 1-1

Figure 8.1: The velocity potential and stream function for (z) = v0 e0 z. Next we nd the stream lines, = c. v0 ( sin(0 )x + cos(0 )y) = c c y= + tan(0 )x v0 cos(0 ) 382

1 0.5 0 -0.5 -1 -1 -0.5 0 0.5 1

Figure 8.2: Streamlines for = v0 ( sin(0 )x + cos(0 )y). Figure 8.2 shows how the streamlines go straight along the 0 direction. Next we nd the velocity eld. v= v = x x + y y v = v0 cos(0 ) + v0 sin(0 ) x y The velocity eld is shown in Figure 8.3. Example 8.5.2 Steady, incompressible, inviscid, irrotational ow is governed by the Laplace equation. We consider ow around an innite cylinder of radius a. Because the ow does not vary along the axis of the cylinder, this is a two-dimensional problem. The ow corresponds to the complex potential (z) = v0 z + 383 a2 z .

Figure 8.3: Velocity eld and velocity direction eld for = v0 (cos(0 )x + sin(0 )y).

We nd the velocity potential and stream function .

(z) = + = v0 r + a r
2

cos ,

= v0 r

a2 r

sin

These are plotted in Figure 8.4. 384

Figure 8.4: The velocity potential and stream function for (z) = v0 z +

a2 z

Next we nd the stream lines, = c.

v0 r r= c

a2 r

sin = c

c2 + 4v0 sin2 2v0 sin

Figure 8.5 shows how the streamlines go around the cylinder. Next we nd the velocity eld. 385

Figure 8.5: Streamlines for = v0 r v = r + r r v = v0 1 The velocity eld is shown in Figure 8.6. a2 r2 cos v0 1 + r a2 r2 v=

a2 r

sin .

sin

386

Figure 8.6: Velocity eld and velocity direction eld for = v0 r +

a2 r

cos .

8.6

Exercises

Complex Derivatives
Exercise 8.1 Consider two functions f (z) and g(z) analytic at z0 with f (z0 ) = g(z0 ) = 0 and g (z0 ) = 0. 1. Use the denition of the complex derivative to justify LHospitals rule:
zz0

lim

f (z) f (z0 ) = g(z) g (z0 ) sinh(z) z ez +1 lim

2. Evaluate the limits

lim

1 + z2 , z 2 + 2z 6 387

Hint, Solution Exercise 8.2 Show that if f (z) is analytic and (x, y) = f (z) is twice continuously dierentiable then f (z) is analytic. Hint, Solution Exercise 8.3 Find the complex derivative in the coordinate directions for f (z) = (r, ). Hint, Solution Exercise 8.4 Show that the following functions are nowhere analytic by checking where the derivative with respect to z exists. 1. sin x cosh y cos x sinh y 2. x2 y 2 + x + (2xy y) Hint, Solution Exercise 8.5 f (z) is analytic for all z, (|z| < ). f (z1 + z2 ) = f (z1 ) f (z2 ) for all z1 and z2 . (This is known as a functional equation). Prove that f (z) = exp (f (0)z). Hint, Solution

Cauchy-Riemann Equations
Exercise 8.6 If f (z) is analytic in a domain and has a constant real part, a constant imaginary part, or a constant modulus, show that f (z) is constant. Hint, Solution

388

Exercise 8.7 Show that the function f (z) = ez 0


4

for z = 0, for z = 0.

satises the Cauchy-Riemann equations everywhere, including at z = 0, but f (z) is not analytic at the origin. Hint, Solution Exercise 8.8 Find the Cauchy-Riemann equations for the following forms. 1. f (z) = R(r, ) e(r,) 2. f (z) = R(x, y) e(x,y) Hint, Solution Exercise 8.9 1. Show that ez is not analytic. 2. f (z) is an analytic function of z. Show that f (z) = f (z) is also an analytic function of z. Hint, Solution Exercise 8.10 1. Determine all points z = x + y where the following functions are dierentiable with respect to z: (a) x3 + y 3 x1 y (b) 2 + y2 (x 1) (x 1)2 + y 2 2. Determine all points z where these functions are analytic. 3. Determine which of the following functions v(x, y) are the imaginary part of an analytic function u(x, y)+v(x, y). For those that are, compute the real part u(x, y) and re-express the answer as an explicit function of z = x + y: 389

(a) x2 y 2 (b) 3x2 y Hint, Solution Exercise 8.11 Let f (z) =


x4/3 y 5/3 +x5/3 y 4/3 x2 +y 2

for z = 0, for z = 0.

Show that the Cauchy-Riemann equations hold at z = 0, but that f is not dierentiable at this point. Hint, Solution Exercise 8.12 Consider the complex function f (z) = u + v =
x3 (1+)y 3 (1) x2 +y 2

for z = 0, for z = 0.

Show that the partial derivatives of u and v with respect to x and y exist at z = 0 and that ux = vy and uy = vx there: the Cauchy-Riemann equations are satised at z = 0. On the other hand, show that lim f (z) z

z0

does not exist, that is, f is not complex-dierentiable at z = 0. Hint, Solution Exercise 8.13 Show that the logarithm log z is dierentiable for z = 0. Find the derivative of the logarithm. Hint, Solution

390

Exercise 8.14 Show that the Cauchy-Riemann equations for the analytic function f (z) = u(r, ) + v(r, ) are ur = v /r, Hint, Solution Exercise 8.15 w = u + v is an analytic function of z. (x, y) is an arbitrary smooth function of x and y. When expressed in terms of u and v, (x, y) = (u, v). Show that (w = 0) = u v Deduce dw 2 2 + = 2 2 u v dz Hint, Solution Exercise 8.16 Show that the functions dened by f (z) = log |z|+ arg(z) and f (z) = | arg(z)| < . What are the corresponding derivatives df /dz? Hint, Solution |z| e arg(z)/2 are analytic in the sector |z| > 0, dw dz
1

u = rvr .

x y 2 2 + x2 y 2

Exercise 8.17 Show that the following functions are harmonic. For each one of them nd its harmonic conjugate and form the corresponding holomorphic function. 1. u(x, y) = x Log(r) y arctan(x, y) (r = 0) 2. u(x, y) = arg(z) (| arg(z)| < , r = 0) 3. u(x, y) = rn cos(n) 391

4. u(x, y) = y/r2 (r = 0) Hint, Solution Exercise 8.18 1. Use the Cauchy-Riemann equations to determine where the function f (z) = (x y)2 + 2(x + y) is dierentiable and where it is analytic. 2. Evaluate the derivative of f (z) = ex and describe the domain of analyticity. Hint, Solution Exercise 8.19 Consider the function f (z) = u + v with real and imaginary parts expressed in terms of either x and y or r and . 1. Show that the Cauchy-Riemann equations ux = vy , uy = vx
2 y 2

(cos(2xy) + sin(2xy))

are satised and these partial derivatives are continuous at a point z if and only if the polar form of the CauchyRiemann equations 1 1 ur = v , u = vr r r is satised and these partial derivatives are continuous there. 2. Show that it is easy to verify that Log z is analytic for r > 0 and < < using the polar form of the Cauchy-Riemann equations and that the value of the derivative is easily obtained from a polar dierentiation formula. 392

3. Show that in polar coordinates, Laplaces equation becomes 1 1 rr + r + 2 = 0. r r Hint, Solution Exercise 8.20 Determine which of the following functions are the real parts of an analytic function. 1. u(x, y) = x3 y 3 2. u(x, y) = sinh x cos y + x 3. u(r, ) = rn cos(n) and nd f (z) for those that are. Hint, Solution Exercise 8.21 Consider steady, incompressible, inviscid, irrotational ow governed by the Laplace equation. Determine the form of the velocity potential and stream function contours for the complex potentials 1. (z) = (x, y) + (x, y) = log z + log z 2. (z) = log(z 1) + log(z + 1) Plot and describe the features of the ows you are considering. Hint, Solution Exercise 8.22 1. Classify all the singularities (removable, poles, isolated essential, branch points, non-isolated essential) of the following functions in the extended complex plane z (a) 2 z +1 393

(b)

1 sin z (c) log 1 + z 2 tan1 (z) (e) z sinh2 (z)

(d) z sin(1/z)

2. Construct functions that have the following zeros or singularities: (a) a simple zero at z = and an isolated essential singularity at z = 1. (b) a removable singularity at z = 3, a pole of order 6 at z = and an essential singularity at z . Hint, Solution

394

8.7

Hints

Complex Derivatives
Hint 8.1 Hint 8.2 Start with the Cauchy-Riemann equation and then dierentiate with respect to x. Hint 8.3 Read Example 8.1.3 and use Result 8.1.1. Hint 8.4 Use Result 8.1.1. Hint 8.5 Take the logarithm of the equation to get a linear equation.

Cauchy-Riemann Equations
Hint 8.6 Hint 8.7 Hint 8.8 For the rst part use the result of Exercise 8.3. Hint 8.9 Use the Cauchy-Riemann equations. 395

Hint 8.10

Hint 8.11 To evaluate ux (0, 0), etc. use the denition of dierentiation. Try to nd f (z) with the denition of complex dierentiation. Consider z = r e . Hint 8.12 To evaluate ux (0, 0), etc. use the denition of dierentiation. Try to nd f (z) with the denition of complex dierentiation. Consider z = r e . Hint 8.13

Hint 8.14

Hint 8.15

Hint 8.16

Hint 8.17

Hint 8.18

Hint 8.19

396

Hint 8.20 Hint 8.21 Hint 8.22 CONTINUE

397

8.8

Solutions

Complex Derivatives
Solution 8.1 1. We consider LHospitals rule.
zz0

lim

f (z0 ) f (z) = g(z) g (z0 )

We start with the right side and show that it is equal to the left side. First we apply the denition of complex dierentiation. lim 0 f (z0 + )f (z0 ) lim 0 f (z0 + ) f (z0 ) = = g (z0 ) lim0 g(z0 +)g(z0 ) lim0 g(z0 +)

Since both of the limits exist, we may take the limits with

= .

f (z0 ) f (z0 + ) = lim 0 g(z0 + ) g (z0 ) f (z0 ) f (z) = lim zz0 g(z) g (z0 ) This proves LHospitals rule. 2. lim 1 + z2 2z = 6 z 2 + 2z 12z 5 =
z=

1 6

lim

sinh(z) cosh(z) = ez +1 ez

=1
z=

398

Solution 8.2 We start with the Cauchy-Riemann equation and then dierentiate with respect to x. x = y xx = yx We interchange the order of dierentiation. (x )x = (x )y (f )x = (f )y Since f (z) satises the Cauchy-Riemann equation and its partial derivatives exist and are continuous, it is analytic. Solution 8.3 We calculate the complex derivative in the coordinate directions. df = dz df = dz We can write this in operator notation. d = e = e dz r r Solution 8.4 1. Consider f (x, y) = sin x cosh y cos x sinh y. The derivatives in the x and y directions are f = cos x cosh y + sin x sinh y x f = cos x cosh y sin x sinh y y 399 r e r r e
1

= e , r r

= e . r

These derivatives exist and are everywhere continuous. We equate the expressions to get a set of two equations. cos x cosh y = cos x cosh y, sin x sinh y = sin x sinh y cos x cosh y = 0, sin x sinh y = 0 x = + n and (x = m or y = 0) 2 The function may be dierentiable only at the points x= Thus the function is nowhere analytic. 2. Consider f (x, y) = x2 y 2 + x + (2xy y). The derivatives in the x and y directions are f = 2x + 1 + 2y x f = 2y + 2x 1 y These derivatives exist and are everywhere continuous. We equate the expressions to get a set of two equations. 2x + 1 = 2x 1, 2y = 2y. + n, 2 y = 0.

Since this set of equations has no solutions, there are no points at which the function is dierentiable. The function is nowhere analytic. Solution 8.5 f (z1 + z2 ) = f (z1 ) f (z2 ) log (f (z1 + z2 )) = log (f (z1 )) + log (f (z2 )) 400

We dene g(z) = log(f (z)). g (z1 + z2 ) = g (z1 ) + g (z2 ) This is a linear equation which has exactly the solutions: g(z) = cz. Thus f (z) has the solutions: f (z) = ecz , where c is any complex constant. We can write this constant in terms of f (0). We dierentiate the original equation with respect to z1 and then substitute z1 = 0. f (z1 + z2 ) = f (z1 ) f (z2 ) f (z2 ) = f (0)f (z2 ) f (z) = f (0)f (z) We substitute in the form of the solution. c ecz = f (0) ecz c = f (0) Thus we see that f (z) = ef (0)z .

Cauchy-Riemann Equations
Solution 8.6 Constant Real Part. First assume that f (z) has constant real part. We solve the Cauchy-Riemann equations to determine the imaginary part. ux = vy , uy = vx vx = 0, vy = 0 401

We integrate the rst equation to obtain v = a + g(y) where a is a constant and g(y) is an arbitrary function. Then we substitute this into the second equation to determine g(y). g (y) = 0 g(y) = b We see that the imaginary part of f (z) is a constant and conclude that f (z) is constant. Constant Imaginary Part. Next assume that f (z) has constant imaginary part. We solve the Cauchy-Riemann equations to determine the real part. ux = vy , uy = vx ux = 0, uy = 0 We integrate the rst equation to obtain u = a + g(y) where a is a constant and g(y) is an arbitrary function. Then we substitute this into the second equation to determine g(y). g (y) = 0 g(y) = b We see that the real part of f (z) is a constant and conclude that f (z) is constant. Constant Modulus. Finally assume that f (z) has constant modulus. |f (z)| = constant u2 + v 2 = constant u2 + v 2 = constant We dierentiate this equation with respect to x and y. 2uux + 2vvx = 0, ux v x uy v y 402 2uuy + 2vvy = 0 u v =0

This system has non-trivial solutions for u and v only if the matrix is non-singular. (The trivial solution u = v = 0 is the constant function f (z) = 0.) We set the determinant of the matrix to zero. ux v y u y v x = 0 We use the Cauchy-Riemann equations to write this in terms of ux and uy . u2 + u2 = 0 y x ux = uy = 0 Since its partial derivatives vanish, u is a constant. From the Cauchy-Riemann equations we see that the partial derivatives of v vanish as well, so it is constant. We conclude that f (z) is a constant. Constant Modulus. Here is another method for the constant modulus case. We solve the Cauchy-Riemann equations in polar form to determine the argument of f (z) = R(x, y) e(x,y) . Since the function has constant modulus R, its partial derivatives vanish. Rx = Ry , Ry = Rx Ry = 0, Rx = 0 The equations are satised for R = 0. For this case, f (z) = 0. We consider nonzero R. y = 0, x = 0

We see that the argument of f (z) is a constant and conclude that f (z) is constant. Solution 8.7 First we verify that the Cauchy-Riemann equations are satised for z = 0. Note that the form fx = fy will be far more convenient than the form ux = v y , uy = vx 403

for this problem. fx = 4(x + y)5 e(x+y) fy = 4(x + y)5 e(x+y) The Cauchy-Riemann equations are satised for z = 0. Now we consider the point z = 0. fx (0, 0) = lim f (x, 0) f (0, 0) x0 x 4 ex = lim x0 x =0
4 4 4

= 4(x + y)5 e(x+y)

fy (0, 0) = lim

f (0, y) f (0, 0) y0 y y 4 e = lim y0 y =0

The Cauchy-Riemann equations are satised for z = 0. f (z) is not analytic at the point z = 0. We show this by calculating the derivative. f (0) = lim
z0

f (z) f (z) f (0) = lim z0 z z f r e r0 r e 4 4 er e = lim r0 r e 404

Let z = r e , that is, we approach the origin at an angle of . f (0) = lim

For most values of the limit does not exist. Consider = /4. er f (0) = lim = r0 r e/4 Because the limit does not exist, the function is not dierentiable at z = 0. Recall that satisfying the Cauchy-Riemann equations is a necessary, but not a sucient condition for dierentiability. Solution 8.8 1. We nd the Cauchy-Riemann equations for f (z) = R(r, ) e(r,) . From Exercise 8.3 we know that the complex derivative in the polar coordinate directions is d = e = e . dz r r We equate the derivatives in the two directions. R e = e R e r r (Rr + Rr ) e = (R + R ) e r e We divide by e and equate the real and imaginary components to obtain the Cauchy-Riemann equations. Rr = 2. We nd the Cauchy-Riemann equations for f (z) = R(x, y) e(x,y) . 405 R , r 1 R = Rr r
4

We equate the derivatives in the x and y directions. R e = R e x y (Rx + Ry ) e = (Rx + Ry ) e We divide by e and equate the real and imaginary components to obtain the Cauchy-Riemann equations. Rx = Ry , Ry = Rx

Solution 8.9 1. A necessary condition for analyticity in an open set is that the Cauchy-Riemann equations are satised in that set. We write ez in Cartesian form. ez = exy = ex cos y ex sin y. Now we determine where u = ex cos y and v = ex sin y satisfy the Cauchy-Riemann equations. ux = vy , uy = vx x e cos y = e cos y, ex sin y = ex sin y cos y = 0, sin y = 0 y = n y = + m, 2
x

Thus we see that the Cauchy-Riemann equations are not satised anywhere. ez is nowhere analytic. 2. Since f (z) = u + v is analytic, u and v satisfy the Cauchy-Riemann equations and their rst partial derivatives are continuous. f (z) = f (z) = u(x, y) + v(x, y) = u(x, y) v(x, y) 406

We dene f (z) (x, y) + (x, y) = u(x, y) v(x, y). Now we see if and satisfy the Cauchy-Riemann equations. x = y , (u(x, y))x = (v(x, y))y , ux (x, y) = vy (x, y), ux = vy , y = x (u(x, y))y = (v(x, y))x uy (x, y) = vx (x, y) uy = vx

Thus we see that the Cauchy-Riemann equations for and are satised if and only if the Cauchy-Riemann equations for u and v are satised. The continuity of the rst partial derivatives of u and v implies the same of and . Thus f (z) is analytic. Solution 8.10 1. The necessary condition for a function f (z) = u + v to be dierentiable at a point is that the Cauchy-Riemann equations hold and the rst partial derivatives of u and v are continuous at that point. (a) f (z) = x3 + y 3 + 0 The Cauchy-Riemann equations are ux = vy and uy = vx 3x2 = 0 and 3y 2 = 0 x = 0 and y = 0 The rst partial derivatives are continuous. Thus we see that the function is dierentiable only at the point z = 0. (b) f (z) = x1 y 2 + y2 (x 1) (x 1)2 + y 2 407

The Cauchy-Riemann equations are ux = vy and uy = vx (x 1) + y (x 1)2 + y 2 2(x 1)y 2(x 1)y = and = ((x 1)2 + y 2 )2 ((x 1)2 + y 2 )2 ((x 1)2 + y 2 )2 ((x 1)2 + y 2 )2
2 2

The Cauchy-Riemann equations are each identities. The rst partial derivatives are continuous everywhere except the point x = 1, y = 0. Thus the function is dierentiable everywhere except z = 1. 2. (a) The function is not dierentiable in any open set. Thus the function is nowhere analytic. (b) The function is dierentiable everywhere except z = 1. Thus the function is analytic everywhere except z = 1. 3. (a) First we determine if the function is harmonic. v = x2 y 2 vxx + vyy = 0 22=0 The function is harmonic in the complex plane and this is the imaginary part of some analytic function. By inspection, we see that this function is z 2 + c = 2xy + c + x2 y 2 , where c is a real constant. We can also nd the function by solving the Cauchy-Riemann equations. ux = vy and uy = vx ux = 2y and uy = 2x We integrate the rst equation. u = 2xy + g(y) 408

Here g(y) is a function of integration. We substitute this into the second Cauchy-Riemann equation to determine g(y). uy = 2x 2x + g (y) = 2x g (y) = 0 g(y) = c u = 2xy + c f (z) = 2xy + c + x2 y 2 f (z) = z 2 + c (b) First we determine if the function is harmonic. v = 3x2 y vxx + vyy = 6y The function is not harmonic. It is not the imaginary part of some analytic function. Solution 8.11 We write the real and imaginary parts of f (z) = u + v.
x4/3 y 5/3 x2 +y 2

u=

for z = 0, , for z = 0.

v=

x5/3 y 4/3 x2 +y 2

for z = 0, for z = 0.

The Cauchy-Riemann equations are ux = v y , uy = vx . 409

We calculate the partial derivatives of u and v at the point x = y = 0 using the denition of dierentiation. ux (0, 0) = lim u(x, 0) u(0, 0) 00 = lim =0 x0 x0 x x v(x, 0) v(0, 0) 00 = lim =0 vx (0, 0) = lim x0 x0 x x u(0, y) u(0, 0) 00 uy (0, 0) = lim = lim =0 y0 y0 y y v(0, y) v(0, 0) 00 vy (0, 0) = lim = lim =0 y0 y0 y y

Since ux (0, 0) = uy (0, 0) = vx (0, 0) = vy (0, 0) = 0 the Cauchy-Riemann equations are satised. f (z) is not analytic at the point z = 0. We show this by calculating the derivative there. f (0) = lim f (z) f (z) f (0) = lim z0 z z0 z

We let z = r e , that is, we approach the origin at an angle of . Then x = r cos and y = r sin . f (0) = lim = f r e r0 r e

+ cos sin r0 e The value of the limit depends on and is not a constant. Thus this limit does not exist. The function is not dierentiable at z = 0. = lim Solution 8.12
x3 y 3 x2 +y 2

r 4/3 cos4/3 r5/3 sin5/3 +r 5/3 cos5/3 r4/3 sin4/3 r2 lim r0 r e 5/3 4/3 4/3 5/3

cos

sin

u=

for z = 0, , for z = 0. 410

v=

x3 +y 3 x2 +y 2

for z = 0, for z = 0.

The Cauchy-Riemann equations are ux = v y , uy = vx . The partial derivatives of u and v at the point x = y = 0 are, ux (0, 0) = lim u(x, 0) u(0, 0) x0 x x 0 = lim x0 x = 1, v(x, 0) v(0, 0) x0 x x 0 = lim x0 x = 1, u(0, y) u(0, 0) y0 y y 0 = lim y0 y = 1, v(0, y) v(0, 0) y0 y y 0 = lim y0 y = 1. 411

vx (0, 0) = lim

uy (0, 0) = lim

vy (0, 0) = lim

We see that the Cauchy-Riemann equations are satised at x = y = 0 f (z) is not analytic at the point z = 0. We show this by calculating the derivative. f (0) = lim f (z) f (0) f (z) = lim z0 z z

z0

Let z = r e , that is, we approach the origin at an angle of . Then x = r cos and y = r sin . f (0) = lim = f r e r0 r e
(1+)r3 cos3 (1)r3 sin3 r2 lim r0 r e 3 3

= lim

(1 + ) cos (1 ) sin r0 e

The value of the limit depends on and is not a constant. Thus this limit does not exist. The function is not dierentiable at z = 0. Recall that satisfying the Cauchy-Riemann equations is a necessary, but not a sucient condition for dierentiability. Solution 8.13 We show that the logarithm log z = (r, ) = Log r + satises the Cauchy-Riemann equations. r = r 1 = r r 1 1 = r r Since the logarithm satises the Cauchy-Riemann equations and the rst partial derivatives are continuous for z = 0, the logarithm is analytic for z = 0. 412

Now we compute the derivative. d log z = e (Log r + ) dz r 1 = e r 1 = z Solution 8.14 The complex derivative in the coordinate directions is d = e = e . dz r r We substitute f = u + v into this identity to obtain the Cauchy-Riemann equation in polar coordinates. f f = e r r f f = r r ur + vr = (u + v ) r e We equate the real and imaginary parts. 1 ur = v , r 1 ur = v , r 1 vr = u r u = rvr

Solution 8.15 Since w is analytic, u and v satisfy the Cauchy-Riemann equations, ux = vy and uy = vx . 413

Using the chain rule we can write the derivatives with respect to x and y in terms of u and v. = ux + vx x u v = uy + vy y u v Now we examine x y . x y = ux u + vx v (uy u + vy v ) x y = (ux uy ) u + (vx vy ) v x y = (ux uy ) u (vy + vx ) v We use the Cauchy-Riemann equations to write uy and vy in terms of ux and vx . x y = (ux + vx ) u (ux + vx ) v Recall that w = ux + vx = vy uy . x y = Thus we see that, = u v We write this in operator notation. = u v dw dz 414
1

dw (u v ) dz

dw dz

x y

x y

The complex conjugate of this relation is + = u v Now we apply both these operators to = . + u v u v = dw dz
1

dw dz

+ x y

+ x y

dw dz

x y

2 2 2 2 + + 2 u2 uv vu v = dw dz
1

dw dz
1

+ x y

x y

dw dz

+ x y

x y

(w )1 is an analytic function. Recall that for analytic functions f , f = fx = fy . So that fx + fy = 0. 2 2 + = u2 v 2 dw dz


1

dw dz
2

2 2 + 2 x2 y

2 2 dw + = 2 2 u v dz Solution 8.16 1. We consider

2 2 + x2 y 2

f (z) = log |z| + arg(z) = log r + . The Cauchy-Riemann equations in polar coordinates are 1 ur = v , r 415 u = rvr .

We calculate the derivatives. 1 1 1 ur = , v = r r r u = 0, rvr = 0 Since the Cauchy-Riemann equations are satised and the partial derivatives are continuous, f (z) is analytic in |z| > 0, | arg(z)| < . The complex derivative in terms of polar coordinates is d = e = e . dz r r We use this to dierentiate f (z). df 1 1 = e [log r + ] = e = dz r r z 2. Next we consider f (z) = |z| e arg(z)/2 = R , r r e/2 .

The Cauchy-Riemann equations for polar coordinates and the polar form f (z) = R(r, ) e(r,) are Rr = We calculate the derivatives for R = 1 R = Rr . r R 1 = r 2 r Rr = 0

r, = /2. 1 Rr = , 2 r 1 R = 0, r

Since the Cauchy-Riemann equations are satised and the partial derivatives are continuous, f (z) is analytic in |z| > 0, | arg(z)| < . The complex derivative in terms of polar coordinates is d = e = e . dz r r 416

We use this to dierentiate f (z). df 1 1 = e [ r e/2 ] = /2 = dz r 2e r 2 z Solution 8.17 1. We consider the function u = x Log r y arctan(x, y) = r cos Log r r sin We compute the Laplacian. u = u 1 2u 1 r + 2 2 r r r r 1 1 = (cos (r + r Log r) sin ) + 2 (r( sin 2 cos ) r cos Log r) r r r 1 1 = (2 cos + cos Log r sin ) + ( sin 2 cos cos Log r) r r =0

The function u is harmonic. We nd the harmonic conjugate v by solving the Cauchy-Riemann equations. 1 vr = u , v = rur r vr = sin (1 + Log r) + cos , v = r (cos (1 + Log r) sin ) We integrate the rst equation with respect to r to determine v to within the constant of integration g(). v = r(sin Log r + cos ) + g() We dierentiate this expression with respect to . v = r (cos (1 + Log r) sin ) + g () 417

We compare this to the second Cauchy-Riemann equation to see that g () = 0. Thus g() = c. We have determined the harmonic conjugate. v = r(sin Log r + cos ) + c The corresponding analytic function is f (z) = r cos Log r r sin + (r sin Log r + r cos + c). On the positive real axis, ( = 0), the function has the value f (z = r) = r Log r + c. We use analytic continuation to determine the function in the complex plane. f (z) = z log z + c 2. We consider the function u = Arg(z) = . We compute the Laplacian. u = 1 r r r u r + 1 2u =0 r2 2

The function u is harmonic. We nd the harmonic conjugate v by solving the Cauchy-Riemann equations. 1 v r = u , r 1 vr = , r v = rur v = 0

We integrate the rst equation with respect to r to determine v to within the constant of integration g(). v = Log r + g() 418

We dierentiate this expression with respect to . v = g () We compare this to the second Cauchy-Riemann equation to see that g () = 0. Thus g() = c. We have determined the harmonic conjugate. v = Log r + c The corresponding analytic function is f (z) = Log r + c On the positive real axis, ( = 0), the function has the value f (z = r) = Log r + c We use analytic continuation to determine the function in the complex plane. f (z) = log z + c 3. We consider the function u = rn cos(n) We compute the Laplacian. u = 1 u 1 2u r + 2 2 r r r r 1 = (nrn cos(n)) n2 rn2 cos(n) r r = n2 rn2 cos(n) n2 rn2 cos(n) =0 419

The function u is harmonic. We nd the harmonic conjugate v by solving the Cauchy-Riemann equations. 1 v r = u , r vr = nrn1 sin(n), v = rur v = nrn cos(n)

We integrate the rst equation with respect to r to determine v to within the constant of integration g(). v = rn sin(n) + g() We dierentiate this expression with respect to . v = nrn cos(n) + g () We compare this to the second Cauchy-Riemann equation to see that g () = 0. Thus g() = c. We have determined the harmonic conjugate. v = rn sin(n) + c The corresponding analytic function is f (z) = rn cos(n) + rn sin(n) + c On the positive real axis, ( = 0), the function has the value f (z = r) = rn + c We use analytic continuation to determine the function in the complex plane. f (z) = z n 4. We consider the function u=

y sin = r2 r

420

We compute the Laplacian. u = 1 u 1 2u r + 2 2 r r r r 1 sin sin = 3 r r r r sin sin = 3 3 r r =0

The function u is harmonic. We nd the harmonic conjugate v by solving the Cauchy-Riemann equations. 1 vr = u , v = rur r cos sin v r = 2 , v = r r We integrate the rst equation with respect to r to determine v to within the constant of integration g(). v= We dierentiate this expression with respect to . sin + g () r We compare this to the second Cauchy-Riemann equation to see that g () = 0. Thus g() = c. We have determined the harmonic conjugate. cos v= +c r The corresponding analytic function is v = f (z) = sin cos + + c r r 421 cos + g() r

On the positive real axis, ( = 0), the function has the value f (z = r) = + c. r

We use analytic continuation to determine the function in the complex plane. f (z) = + c z

Solution 8.18 1. We calculate the rst partial derivatives of u = (x y)2 and v = 2(x + y). ux uy vx vy = 2(x y) = 2(y x) =2 =2

We substitute these expressions into the Cauchy-Riemann equations. ux = vy , uy = vx 2(x y) = 2, 2(y x) = 2 x y = 1, y x = 1 y =x1 Since the Cauchy-Riemann equation are satised along the line y = x1 and the partial derivatives are continuous, the function f (z) is dierentiable there. Since the function is not dierentiable in a neighborhood of any point, it is nowhere analytic. 422

2. We calculate the rst partial derivatives of u and v. u x = 2 ex vx = 2 e vy = 2 e


2 y 2

(x cos(2xy) y sin(2xy)) (y cos(2xy) + x sin(2xy)) (y cos(2xy) + x sin(2xy)) (x cos(2xy) y sin(2xy))

uy = 2 e

x2 y 2

x2 y 2 x2 y 2

Since the Cauchy-Riemann equations, ux = vy and uy = vx , are satised everywhere and the partial derivatives are continuous, f (z) is everywhere dierentiable. Since f (z) is dierentiable in a neighborhood of every point, it is analytic in the complex plane. (f (z) is entire.) Now to evaluate the derivative. The complex derivative is the derivative in any direction. We choose the x direction. f (z) = ux + vx f (z) = 2 e
x2 y 2

(x cos(2xy) y sin(2xy)) + 2 ex
x2 y 2

2 y 2

(y cos(2xy) + x sin(2xy))

f (z) = 2 e

((x + y) cos(2xy) + (y + x) sin(2xy))

Finding the derivative is easier if we rst write f (z) in terms of the complex variable z and use complex dierentiation. f (z) = ex
2 y 2

(cos(2x, y) + sin(2xy))
2 y 2

f (z) = ex

e2xy
2 2 2

f (z) = e(x+y) f (z) = ez f (z) = 2z ez

423

Solution 8.19 1. Assume that the Cauchy-Riemann equations in Cartesian coordinates ux = vy , uy = vx

are satised and these partial derivatives are continuous at a point z. We write the derivatives in polar coordinates in terms of derivatives in Cartesian coordinates to verify the Cauchy-Riemann equations in polar coordinates. First we calculate the derivatives. x = r cos , y = r sin y x wr = wx + wy = cos wx + sin wy r r x y w = wx + wy = r sin wx + r cos wy Then we verify the Cauchy-Riemann equations in polar coordinates. ur = cos ux + sin uy = cos vy sin vx 1 = v r 1 u = sin ux + cos uy r = sin vy cos vx = vr This proves that the Cauchy-Riemann equations in Cartesian coordinates hold only if the Cauchy-Riemann equations in polar coordinates hold. (Given that the partial derivatives are continuous.) Next we prove the converse. Assume that the Cauchy-Riemann equations in polar coordinates 1 ur = v , r 424 1 u = vr r

are satised and these partial derivatives are continuous at a point z. We write the derivatives in Cartesian coordinates in terms of derivatives in polar coordinates to verify the Cauchy-Riemann equations in Cartesian coordinates. First we calculate the derivatives. x2 + y 2 , = arctan(x, y) r x y wx = wr + w = wr 2 w x x r r r y x wy = wr + w = wr + 2 w y y r r r= Then we verify the Cauchy-Riemann equations in Cartesian coordinates. x y ux = ur 2 u r r x y = 2 v + vr r r = uy y x uy = ur + 2 u r r y x = 2 v vr r r = ux This proves that the Cauchy-Riemann equations in polar coordinates hold only if the Cauchy-Riemann equations in Cartesian coordinates hold. We have demonstrated the equivalence of the two forms. 2. We verify that log z is analytic for r > 0 and < < using the polar form of the Cauchy-Riemann equations. Log z = ln r + 1 1 u = vr ur = v , r r 1 1 1 = 1, 0 = 0 r r r 425

Since the Cauchy-Riemann equations are satised and the partial derivatives are continuous for r > 0, log z is analytic there. We calculate the value of the derivative using the polar dierentiation formulas. d 1 1 Log z = e (ln r + ) = e = dz r r z d 1 Log z = (ln r + ) = = dz z z z 3. Let {xi } denote rectangular coordinates in two dimensions and let {i } be an orthogonal coordinate system . The distance metric coecients hi are dened hi = The Laplacian is
2

x1 i 1

x2 i 2

u=

1 h1 h2

h2 u h1 1

h1 u h2 2

First we calculate the distance metric coecients in polar coordinates. hr = h = Then we nd the Laplacian.
2

x r x
2

+ +

y r y
2

= =

cos2 + sin2 = 1 r2 sin2 + r2 cos2 = r

1 r

(rr ) + r

1 r

In polar coordinates, Laplaces equation is 1 1 rr + r + 2 = 0. r r 426

Solution 8.20 1. We compute the Laplacian of u(x, y) = x3 y 3 .


2

u = 6x 6y

Since u is not harmonic, it is not the real part of on analytic function. 2. We compute the Laplacian of u(x, y) = sinh x cos y + x.
2

u = sinh x cos y sinh x cos y = 0

Since u is harmonic, it is the real part of on analytic function. We determine v by solving the Cauchy-Riemann equations. vx = uy , vy = ux vx = sinh x sin y, vy = cosh x cos y + 1 We integrate the rst equation to determine v up to an arbitrary additive function of y. v = cosh x sin y + g(y) We substitute this into the second Cauchy-Riemann equation. This will determine v up to an additive constant. vy = cosh x cos y + 1 cosh x cos y + g (y) = cosh x cos y + 1 g (y) = 1 g(y) = y + a v = cosh x sin y + y + a f (z) = sinh x cos y + x + (cosh x sin y + y + a) Here a is a real constant. We write the function in terms of z. f (z) = sinh z + z + a 427

3. We compute the Laplacian of u(r, ) = rn cos(n).


2

u = n(n 1)rn2 cos(n) + nrn2 cos(n) n2 rn2 cos(n) = 0

Since u is harmonic, it is the real part of on analytic function. We determine v by solving the Cauchy-Riemann equations. 1 v r = u , r n1 vr = nr sin(n), v = rur v = nrn cos(n)

We integrate the rst equation to determine v up to an arbitrary additive function of . v = rn sin(n) + g() We substitute this into the second Cauchy-Riemann equation. This will determine v up to an additive constant. v = nrn cos(n) nrn cos(n) + g () = nrn cos(n) g () = 0 g() = a v = rn sin(n) + a f (z) = rn cos(n) + (rn sin(n) + a) Here a is a real constant. We write the function in terms of z. f (z) = z n + a Solution 8.21 1. We nd the velocity potential and stream function . (z) = log z + log z (z) = ln r + + (ln r + ) = ln r , = ln r + 428

Figure 8.7: The velocity potential and stream function for (z) = log z + log z. A branch of these are plotted in Figure 8.7. Next we nd the stream lines, = c. ln r + = c r = ec These are spirals which go counter-clockwise as we follow them to the origin. See Figure 8.8. Next we nd the velocity eld. v = r + r r r v= r r 429 v=

Figure 8.8: Streamlines for = ln r + . The velocity eld is shown in the rst plot of Figure 8.9. We see that the uid ows out from the origin along the spiral paths of the streamlines. The second plot shows the direction of the velocity eld. 2. We nd the velocity potential and stream function . (z) = log(z 1) + log(z + 1) (z) = ln |z 1| + arg(z 1) + ln |z + 1| + arg(z + 1) = ln |z 2 1|, = arg(z 1) + arg(z + 1) The velocity potential and a branch of the stream function are plotted in Figure 8.10. The stream lines, arg(z 1) + arg(z + 1) = c, are plotted in Figure 8.11. Next we nd the velocity eld. v=
2 2

v=

2x(x + y 1) 2y(x2 + y 2 + 1) x+ 4 y x4 + 2x2 (y 2 1) + (y 2 + 1)2 x + 2x2 (y 2 1) + (y 2 + 1)2 430

Figure 8.9: Velocity eld and velocity direction eld for = ln r . The velocity eld is shown in the rst plot of Figure 8.12. The uid is owing out of sources at z = 1. The second plot shows the direction of the velocity eld. Solution 8.22 1. (a) We factor the denominator to see that there are rst order poles at z = . z2 z z = +1 (z )(z + ) 431

2 1 0 -1 -2

2 1 -1 0 1 0 -1 2-2

6 4 2 0 -2

2 1 -1 0 1 0 -1 2-2

Figure 8.10: The velocity potential and stream function for (z) = log(z 1) + log(z + 1). Since the function behaves like 1/z at innity, it is analytic there. (b) The denominator of 1/ sin z has rst order zeros at z = n, n Z. Thus the function has rst order poles at these locations. Now we examine the point at innity with the change of variables z = 1/. 1 1 2 = = / e e/ sin z sin(1/) We see that the point at innity is a singularity of the function. Since the denominator grows exponentially, there is no multiplicative factor of n that will make the function analytic at = 0. We conclude that the point at innity is an essential singularity. Since there is no deleted neighborhood of the point at innity that does contain rst order poles at the locations z = n, the point at innity is a non-isolated singularity. (c) log 1 + z 2 = log(z + ) + log(z ) There are branch points at z = . Since the argument of the logarithm is unbounded as z there is a branch point at innity as well. Branch points are non-isolated singularities. 432

-1

-2 -2

-1

Figure 8.11: Streamlines for = arg(z 1) + arg(z + 1). (d) 1 z sin(1/z) = z e/z + e/z 2 The point z = 0 is a singularity. Since the function grows exponentially at z = 0. There is no multiplicative factor of z n that will make the function analytic. Thus z = 0 is an essential singularity. There are no other singularities in the nite complex plane. We examine the point at innity. z sin 1 z = 1 sin

The point at innity is a singularity. We take the limit 0 to demonstrate that it is a removable 433

Figure 8.12: Velocity eld and velocity direction eld for = ln |z 2 1|. singularity. sin cos = lim =1 0 0 1 lim (e) log +z tan1 (z) z = 2 2 z sinh (z) 2z sinh (z) 434

There are branch points at z = due to the logarithm. These are non-isolated singularities. Note that sinh(z) has rst order zeros at z = n, n Z. The arctangent has a rst order zero at z = 0. Thus there is a second order pole at z = 0. There are second order poles at z = n, n Z \ {0} due to the hyperbolic sine. Since the hyperbolic sine has an essential singularity at innity, the function has an essential singularity at innity as well. The point at innity is a non-isolated singularity because there is no neighborhood of innity that does not contain second order poles. 2. (a) (z ) e1/(z1) has a simple zero at z = and an isolated essential singularity at z = 1. (b) sin(z 3) (z 3)(z + )6 has a removable singularity at z = 3, a pole of order 6 at z = and an essential singularity at z .

435

Chapter 9 Analytic Continuation


For every complex problem, there is a solution that is simple, neat, and wrong. - H. L. Mencken

9.1

Analytic Continuation

Suppose there is a function, f1 (z) that is analytic in the domain D1 and another analytic function, f2 (z) that is analytic in the domain D2 . (See Figure 9.1.) If the two domains overlap and f1 (z) = f2 (z) in the overlap region D1 D2 , then f2 (z) is called an analytic continuation of f1 (z). This is an appropriate name since f2 (z) continues the denition of f1 (z) outside of its original domain of denition D1 . We can dene a function f (z) that is analytic in the union of the domains D1 D2 . On the domain D1 we have f (z) = f1 (z) and f (z) = f2 (z) on D2 . f1 (z) and f2 (z) are called function elements. There is an analytic continuation even if the two domains only share an arc and not a two dimensional region. With more overlapping domains D3 , D4 , . . . we could perhaps extend f1 (z) to more of the complex plane. Sometimes it is impossible to extend a function beyond the boundary of a domain. This is known as a natural boundary. If a 436

Im(z) D1

D2 Re(z)

Figure 9.1: Overlapping Domains function f1 (z) is analytically continued to a domain Dn along two dierent paths, (See Figure 9.2.), then the two analytic continuations are identical as long as the paths do not enclose a branch point of the function. This is the uniqueness theorem of analytic continuation.

Dn D1

Figure 9.2: Two Paths of Analytic Continuation

Consider an analytic function f (z) dened in the domain D. Suppose that f (z) = 0 on the arc AB, (see Figure 9.3.) Then f (z) = 0 in all of D. Consider a point on AB. The Taylor series expansion of f (z) about the point z = converges in a circle C at 437

D C A B

Figure 9.3: Domain Containing Arc Along Which f (z) Vanishes

least up to the boundary of D. The derivative of f (z) at the point z = is

f () = lim

f ( + z) f () z0 z

If z is in the direction of the arc, then f () vanishes as well as all higher derivatives, f () = f () = f () = = 0. Thus we see that f (z) = 0 inside C. By taking Taylor series expansions about points on AB or inside of C we see that f (z) = 0 in D.

Result 9.1.1 Let f1 (z) and f2 (z) be analytic functions dened in D. If f1 (z) = f2 (z) for the points in a region or on an arc in D, then f1 (z) = f2 (z) for all points in D.
To prove Result 9.1.1, we dene the analytic function g(z) = f1 (z) f2 (z). Since g(z) vanishes in the region or on the arc, then g(z) = 0 and hence f1 (z) = f2 (z) for all points in D. 438

Result 9.1.2 Consider analytic functions f1 (z) and f2 (z) dened on the domains D1 and D2 , respectively. Suppose that D1 D2 is a region or an arc and that f1 (z) = f2 (z) for all z D1 D2 . (See Figure 9.4.) Then the function f (z) = is analytic in D1 D2 . f1 (z) for z D1 , f2 (z) for z D2 ,

D1

D2

D1

D2

Figure 9.4: Domains that Intersect in a Region or an Arc Result 9.1.2 follows directly from Result 9.1.1.

9.2

Analytic Continuation of Sums

Example 9.2.1 Consider the function f1 (z) =


n=0

zn.

The sum converges uniformly for D1 = |z| r < 1. Since the derivative also converges in this domain, the function is analytic there. 439

Re(z)

Re(z) D2

D1 Im(z) Im(z)

Figure 9.5: Domain of Convergence for Now consider the function f2 (z) =

n=0

zn.

1 . 1z

This function is analytic everywhere except the point z = 1. On the domain D1 , 1 = f2 (z) = 1z

z n = f1 (z)
n=0

Analytic continuation tells us that there is a function that is analytic on the union of the two domains. Here, the domain is the entire z plane except the point z = 1 and the function is f (z) =
1 1z

1 . 1z

is said to be an analytic continuation of

n=0

zn.

440

9.3

Analytic Functions Dened in Terms of Real Variables

Result 9.3.1 An analytic function, u(x, y) + v(x, y) can be written in terms of a function of a complex variable, f (z) = u(x, y) + v(x, y).
Result 9.3.1 is proved in Exercise 9.1. Example 9.3.1 f (z) = cosh y sin x (x ex cos y y ex sin y) cos x sinh y (y ex cos y + x ex sin y) + cosh y sin x (y ex cos y + x ex sin y) + cos x sinh y (x ex cos y y ex sin y) is an analytic function. Express f (z) in terms of z. On the real line, y = 0, f (z) is f (z = x) = x ex sin x (Recall that cos(0) = cosh(0) = 1 and sin(0) = sinh(0) = 0.) The analytic continuation of f (z) into the complex plane is f (z) = z ez sin z. Alternatively, for x = 0 we have f (z = y) = y sinh y(cos y sin y). The analytic continuation from the imaginary axis to the complex plane is f (z) = z sinh(z)(cos(z) sin(z)) = z sinh(z)(cos(z) + sin(z)) = z sin z ez .

441

Example 9.3.2 Consider u = ex (x sin y y cos y). Find v such that f (z) = u + v is analytic. From the Cauchy-Riemann equations, v u = = ex sin y x ex sin y + y ex cos y y x v u = = ex cos y x ex cos y y ex sin y x y Integrate the rst equation with respect to y. v = ex cos y + x ex cos y + ex (y sin y + cos y) + F (x) = y ex sin y + x ex cos y + F (x) F (x) is an arbitrary function of x. Substitute this expression for v into the equation for v/x. y ex sin y x ex cos y + ex cos y + F (x) = y ex sin y x ex cos y + ex cos y Thus F (x) = 0 and F (x) = c. v = ex (y sin y + x cos y) + c Example 9.3.3 Find f (z) in the previous example. (Up to the additive constant.) Method 1 f (z) = u + v = ex (x sin y y cos y) + ex (y sin y + x cos y) ey ey ey + ey = ex x y + ex y 2 2 = (x + y) e(x+y) = z ez 442

ey ey 2

+x

ey + ey 2

Method 2 f (z) = f (x + y) = u(x, y) + v(x, y) is an analytic function. On the real axis, y = 0, f (z) is f (z = x) = u(x, 0) + v(x, 0) = ex (x sin 0 0 cos 0) + ex (0 sin 0 + x cos 0) = x ex Suppose there is an analytic continuation of f (z) into the complex plane. If such a continuation, f (z), exists, then it must be equal to f (z = x) on the real axis An obvious choice for the analytic continuation is f (z) = u(z, 0) + v(z, 0) since this is clearly equal to u(x, 0) + v(x, 0) when z is real. Thus we obtain f (z) = z ez Example 9.3.4 Consider f (z) = u(x, y) + v(x, y). Show that f (z) = ux (z, 0) uy (z, 0). f (z) = ux + vx = ux uy f (z) is an analytic function. On the real axis, z = x, f (z) is f (z = x) = ux (x, 0) uy (x, 0) Now f (z = x) is dened on the real line. An analytic continuation of f (z = x) into the complex plane is f (z) = ux (z, 0) uy (z, 0).

443

Example 9.3.5 Again consider the problem of nding f (z) given that u(x, y) = ex (x sin y y cos y). Now we can use the result of the previous example to do this problem. ux (x, y) = u = ex sin y x ex sin y + y ex cos y x u = x ex cos y + y ex sin y ex cos y uy (x, y) = y f (z) = ux (z, 0) uy (z, 0) = 0 z ez ez = z ez + ez Integration yields the result f (z) = z ez +c Example 9.3.6 Find f (z) given that u(x, y) = cos x cosh2 y sin x + cos x sin x sinh2 y v(x, y) = cos2 x cosh y sinh y cosh y sin2 x sinh y f (z) = u(x, y) + v(x, y) is an analytic function. On the real line, f (z) is f (z = x) = u(x, 0) + v(x, 0) = cos x cosh2 0 sin x + cos x sin x sinh2 0 + cos2 x cosh 0 sinh 0 cosh 0 sin2 x sinh 0 = cos x sin x Now we know the denition of f (z) on the real line. We would like to nd an analytic continuation of f (z) into the complex plane. An obvious choice for f (z) is f (z) = cos z sin z 444

Using trig identities we can write this as f (z) = sin(2z) . 2

Example 9.3.7 Find f (z) given only that u(x, y) = cos x cosh2 y sin x + cos x sin x sinh2 y. Recall that f (z) = ux + vx = ux uy Dierentiating u(x, y), ux = cos2 x cosh2 y cosh2 y sin2 x + cos2 x sinh2 y sin2 x sinh2 y uy = 4 cos x cosh y sin x sinh y f (z) is an analytic function. On the real axis, f (z) is f (z = x) = cos2 x sin2 x Using trig identities we can write this as f (z = x) = cos(2x) Now we nd an analytic continuation of f (z = x) into the complex plane. f (z) = cos(2z) Integration yields the result f (z) = sin(2z) +c 2

445

9.3.1

Polar Coordinates
u(r, ) = r(log r cos sin )

Example 9.3.8 Is the real part of an analytic function? The Laplacian in polar coordinates is = We calculate the partial derivatives of u. u r u r r u r r r u 1 r r r r u 2u 2 1 2u r2 2 From the above we see that u = = cos + log r cos sin = r cos + r log r cos r sin = 2 cos + log r cos sin = 1 (2 cos + log r cos sin ) r 1 r r r r + 1 2 . r2 2

= r ( cos + sin + log r sin ) = r (2 cos log r cos + sin ) = 1 (2 cos log r cos + sin ) r r u r + 1 2u = 0. r2 2

1 r r

Therefore u is harmonic and is the real part of some analytic function. 446

Example 9.3.9 Find an analytic function f (z) whose real part is u(r, ) = r (log r cos sin ) . Let f (z) = u(r, ) + v(r, ). The Cauchy-Riemann equations are v ur = , u = rvr . r Using the partial derivatives in the above example, we obtain two partial dierential equations for v(r, ). u vr = = cos + sin + log r sin r v = rur = r (cos + log r cos sin ) Integrating the equation for v yields v = r ( cos + log r sin ) + F (r) where F (r) is a constant of integration. Substituting our expression for v into the equation for vr yields cos + log r sin + sin + F (r) = cos + sin + log r sin F (r) = 0 F (r) = const Thus we see that f (z) = u + v = r (log r cos sin ) + r ( cos + log r sin ) + const f (z) is an analytic function. On the line = 0, f (z) is f (z = r) = r(log r) + r(0) + const = r log r + const 447

The analytic continuation into the complex plane is f (z) = z log z + const

Example 9.3.10 Find the formula in polar coordinates that is analogous to f (z) = ux (z, 0) uy (z, 0). We know that df f = e . dz r If f (z) = u(r, ) + v(r, ) then df = e (ur + vr ) dz From the Cauchy-Riemann equations, we have vr = u /r. u df = e ur dz r f (z) is an analytic function. On the line = 0, f (z) is f (z = r) = ur (r, 0) The analytic continuation of f (z) into the complex plane is f (z) = ur (z, 0) u (z, 0). r u (r, 0) r

448

Example 9.3.11 Find an analytic function f (z) whose real part is u(r, ) = r (log r cos sin ) .

ur (r, ) = (log r cos sin ) + cos u (r, ) = r ( log r sin sin cos ) f (z) = ur (z, 0) u (z, 0) r = log z + 1 Integrating f (z) yields f (z) = z log z + c.

9.3.2

Analytic Functions Dened in Terms of Their Real or Imaginary Parts

Consider an analytic function: f (z) = u(x, y) + v(x, y). We dierentiate this expression. f (z) = ux (x, y) + vx (x, y) We apply the Cauchy-Riemann equation vx = uy . f (z) = ux (x, y) uy (x, y). Now consider the function of a complex variable, g(): g() = ux (x, ) uy (x, ) = ux (x, + ) uy (x, + ). 449 (9.1)

This function is analytic where f () is analytic. To show this we rst verify that the derivatives in the and directions are equal. g() = uxy (x, + ) uyy (x, + ) g() = (uxy (x, + ) + uyy (x, + )) = uxy (x, + ) uyy (x, + )

Since these partial derivatives are equal and continuous, g() is analytic. We evaluate the function g() at = x. (Substitute y = x into Equation 9.1.) f (2x) = ux (x, x) uy (x, x) We make a change of variables to solve for f (x). f (x) = ux x x x x , uy , . 2 2 2 2

If the expression is non-singular, then this denes the analytic function, f (z), on the real axis. The analytic continuation to the complex plane is z z z z f (z) = ux , uy , . 2 2 2 2 d Note that dz 2u(z/2, z/2) = ux (z/2, z/2) uy (z/2, z/2). We integrate the equation to obtain: f (z) = 2u z z , + c. 2 2

We know that the real part of an analytic function determines that function to within an additive constant. Assuming that the above expression is non-singular, we have found a formula for writing an analytic function in terms of its real part. With the same method, we can nd how to write an analytic function in terms of its imaginary part, v. We can also derive formulas if u and v are expressed in polar coordinates: f (z) = u(r, ) + v(r, ). 450

Result 9.3.2 If f (z) = u(x, y) + v(x, y) is analytic and the expressions are non-singular, then z z f (z) = 2u , + const (9.2) 2 2 z z f (z) = 2v , + const. (9.3) 2 2 If f (z) = u(r, ) + v(r, ) is analytic and the expressions are non-singular, then f (z) = 2u z 1/2 , log z + const 2 f (z) = 2v z 1/2 , log z + const. 2
Example 9.3.12 Consider the problem of nding f (z) given that u(x, y) = ex (x sin y y cos y). z z , 2 2 z z z z/2 z = 2e sin + cos +c 2 2 2 2 z z = z ez/2 sin + cos +c 2 2 = z ez/2 ez/2 + c = z ez +c

(9.4) (9.5)

f (z) = 2u

Example 9.3.13 Consider Log z = 1 Log x2 + y 2 + Arctan(x, y). 2 451

We try to construct the analytic function from its real part using Equation 9.2. f (z) = 2u z z , +c 2 2 1 z 2 z = 2 Log + 2 2 2 = Log(0) + c

+c

We obtain a singular expression, so the method fails. Example 9.3.14 Again consider the logarithm, this time written in terms of polar coordinates. Log z = Log r + We try to construct the analytic function from its real part using Equation 9.4. f (z) = 2u z 1/2 , log z + c 2 = 2 Log z 1/2 + c = Log z + c With this method we recover the analytic function.

452

9.4

Exercises

Exercise 9.1 Consider two functions, f (x, y) and g(x, y). They are said to be functionally dependent if there is a an h(g) such that f (x, y) = h(g(x, y)). f and g will be functionally dependent if and only if their Jacobian vanishes. If f and g are functionally dependent, then the derivatives of f are fx = h (g)gx fy = h (g)gy . Thus we have (f, g) f f = x y = fx gy fy gx = h (g)gx gy h (g)gy gx = 0. gx gy (x, y) fx gy fy gx = 0. This is a rst order partial dierential equation for f that has the general solution f (x, y) = h(g(x, y)). Prove that an analytic function u(x, y) + v(x, y) can be written in terms of a function of a complex variable, f (z) = u(x, y) + v(x, y). Exercise 9.2 Which of the following functions are the real part of an analytic function? For those that are, nd the harmonic conjugate, v(x, y), and nd the analytic function f (z) = u(x, y) + v(x, y) as a function of z. 1. x3 3xy 2 2xy + y 2. ex sinh y 453

If the Jacobian of f and g vanishes, then

3. ex (sin x cos y cosh y cos x sin y sinh y) Exercise 9.3 For an analytic function, f (z) = u(r, ) + v(r, ) prove that under suitable restrictions: f (z) = 2u z 1/2 , log z + const. 2

454

9.5

Hints

Hint 9.1 Show that u(x, y) + v(x, y) is functionally dependent on x + y so that you can write f (z) = f (x + y) = u(x, y) + v(x, y). Hint 9.2 Hint 9.3 Check out the derivation of Equation 9.2.

455

9.6

Solutions

Solution 9.1 u(x, y) + v(x, y) is functionally dependent on z = x + y if and only if (u + v, x + y) = 0. (x, y) (u + v, x + y) u + vx uy + vy = x 1 (x, y) = vx uy + (ux vy ) Since u and v satisfy the Cauchy-Riemann equations, this vanishes. =0 Thus we see that u(x, y) + v(x, y) is functionally dependent on x + y so we can write f (z) = f (x + y) = u(x, y) + v(x, y). Solution 9.2 1. Consider u(x, y) = x3 3xy 2 2xy + y. The Laplacian of this function is u uxx + uyy = 6x 6x =0 Since the function is harmonic, it is the real part of an analytic function. Clearly the analytic function is of the form, az 3 + bz 2 + cz + d, 456

with a, b and c complex-valued constants and d a real constant. Substituting z = x + y and expanding products yields, a x3 + 3x2 y 3xy 2 y 3 + b x2 + 2xy y 2 + c(x + y) + d. By inspection, we see that the analytic function is f (z) = z 3 + z 2 z + d. The harmonic conjugate of u is the imaginary part of f (z), v(x, y) = 3x2 y y 3 + x2 y 2 x + d. We can also do this problem with analytic continuation. The derivatives of u are ux = 3x2 3y 2 2y, uy = 6xy 2x + 1. The derivative of f (z) is f (z) = ux uy = 3x2 2y 2 2y + (6xy 2x + 1). On the real axis we have f (z = x) = 3x2 2x + . Using analytic continuation, we see that f (z) = 3z 2 2z + . Integration yields f (z) = z 3 z 2 + z + const 457

2. Consider u(x, y) = ex sinh y. The Laplacian of this function is u = ex sinh y + ex sinh y = 2 ex sinh y. Since the function is not harmonic, it is not the real part of an analytic function. 3. Consider u(x, y) = ex (sin x cos y cosh y cos x sin y sinh y). The Laplacian of the function is u = x (e (sin x cos y cosh y cos x sin y sinh y + cos x cos y cosh y + sin x sin y sinh y)) x x + (e ( sin x sin y cosh y cos x cos y sinh y + sin x cos y sinh y cos x sin y cosh y)) y = 2 ex (cos x cos y cosh y + sin x sin y sinh y) 2 ex (cos x cos y cosh y + sin x sin y sinh y) = 0.

Thus u is the real part of an analytic function. The derivative of the analytic function is f (z) = ux + vx = ux uy From the derivatives of u we computed before, we have f (z) = (ex (sin x cos y cosh y cos x sin y sinh y + cos x cos y cosh y + sin x sin y sinh y)) (ex ( sin x sin y cosh y cos x cos y sinh y + sin x cos y sinh y cos x sin y cosh y)) Along the real axis, f (z) has the value, f (z = x) = ex (sin x + cos x). By analytic continuation, f (z) is f (z) = ez (sin z + cos z) 458

We obtain f (z) by integrating. f (z) = ez sin z + const. u is the real part of the analytic function f (z) = ez sin z + c, where c is a real constant. We nd the harmonic conjugate of u by taking the imaginary part of f . f (z) = ex (cosy + sin y)(sin x cosh y + cos x sinh y) + c v(x, y) = ex sin x sin y cosh y + cos x cos y sinh y + c Solution 9.3 We consider the analytic function: f (z) = u(r, ) + v(r, ). Recall that the complex derivative in terms of polar coordinates is d = e = e . dz r r The Cauchy-Riemann equations are 1 1 ur = v , v r = u . r r We dierentiate f (z) and use the partial derivative in r for the right side. f (z) = e (ur + vr ) We use the Cauchy-Riemann equations to right f (z) in terms of the derivatives of u. 1 f (z) = e ur u r Now consider the function of a complex variable, g(): 1 g() = e ur (r, ) u (r, ) r 1 = e ur (r, + ) u (r, + ) r 459 (9.6)

This function is analytic where f () is analytic. It is a simple calculus exercise to show that the complex derivative in the direction, , and the complex derivative in the direction, , are equal. Since these partial derivatives are equal and continuous, g() is analytic. We evaluate the function g() at = log r. (Substitute = log r into Equation 9.6.) 1 f r e( log r) = e( log r) ur (r, log r) u (r, log r) r 1 rf r2 = ur (r, log r) u (r, log r) r If the expression is non-singular, then it denes the analytic function, f (z), on a curve. The analytic continuation to the complex plane is 1 zf z 2 = ur (z, log z) u (z, log z). z 2 We integrate to obtain an expression for f (z ). 1 f z 2 = u(z, log z) + const 2 We make a change of variables and solve for f (z). f (z) = 2u z 1/2 , log z + const. 2 Assuming that the above expression is non-singular, we have found a formula for writing the analytic function in terms of its real part, u(r, ). With the same method, we can nd how to write an analytic function in terms of its imaginary part, v(r, ).

460

Chapter 10 Contour Integration and the Cauchy-Goursat Theorem


Between two evils, I always pick the one I never tried before. - Mae West

10.1

Line Integrals

In this section we will recall the denition of a line integral in the Cartesian plane. In the next section we will use this to dene the contour integral in the complex plane. Limit Sum Denition. First we develop a limit sum denition of a line integral. Consider a curve C in the Cartesian plane joining the points (a0 , b0 ) and (a1 , b1 ). We partition the curve into n segments with the points (x0 , y0 ), . . . , (xn , yn ) where the rst and last points are at the endpoints of the curve. We dene the dierences, xk = xk+1 xk and yk = yk+1 yk , and let (k , k ) be points on the curve between (xk , yk ) and (xk+1 , yk+1 ). This is shown pictorially in Figure 10.1. 461

y (1 ,1 ) (x1 ,y1 ) (x2 ,y2 ) (xn ,yn )

( n1 ,n1 ) (2 ,2 ) (0 ,0 ) (xn1 ,yn1 ) (x0 ,y0 ) x

Figure 10.1: A curve in the Cartesian plane. Consider the sum

n1

(P (k , k )xk + Q(k , k )yk ) ,


k=0

where P and Q are continuous functions on the curve. (P and Q may be complex-valued.) In the limit as each of the xk and yk approach zero the value of the sum, (if the limit exists), is denoted by P (x, y) dx + Q(x, y) dy.
C

This is a line integral along the curve C. The value of the line integral depends on the functions P (x, y) and Q(x, y), the endpoints of the curve and the curve C. We can also write a line integral in vector notation. f (x) dx
C

Here x = (x, y) and f (x) = (P (x, y), Q(x, y)). 462

Evaluating Line Integrals with Parameterization. Let the curve C be parametrized by x = x(t), y = y(t) for t0 t t1 . Then the dierentials on the curve are dx = x (t) dt and dy = y (t) dt. Using the parameterization we can evaluate a line integral in terms of a denite integral.
t1

P (x, y) dx + Q(x, y) dy =
C t0

P (x(t), y(t))x (t) + Q(x(t), y(t))y (t) dt

Example 10.1.1 Consider the line integral x2 dx + (x + y) dy,


C

where C is the semi-circle from (1, 0) to (1, 0) in the upper half plane. We parameterize the curve with x = cos t, y = sin t for 0 t .

x2 dx + (x + y) dy =
C 0

cos2 t( sin t) + (cos t + sin t) cos t dt

2 = 2 3

10.2

Contour Integrals

Limit Sum Denition. We develop a limit sum denition for contour integrals. It will be analogous to the denition for line integrals except that the notation is cleaner in complex variables. Consider a contour C in the complex plane joining the points c0 and c1 . We partition the contour into n segments with the points z0 , . . . , zn where the rst and last points are at the endpoints of the contour. We dene the dierences zk = zk+1 zk and let k be points on the contour between zk and zk+1 . Consider the sum
n1

f (k )zk ,
k=0

463

where f is a continuous function on the contour. In the limit as each of the zk approach zero the value of the sum, (if the limit exists), is denoted by f (z) dz.
C

This is a contour integral along C. We can write a contour integral in terms of a line integral. Let f (z) = (x, y). ( : R2 C.) f (z) dz =
C C

(x, y)(dx + dy) ((x, y) dx + (x, y) dy) (10.1)

f (z) dz =
C C

Further, we can write a contour integral in terms of two real-valued line integrals. Let f (z) = u(x, y) + v(x, y). f (z) dz =
C C

(u(x, y) + v(x, y))(dx + dy) (v(x, y) dx + u(x, y) dy)


C

f (z) dz =
C C

(u(x, y) dx v(x, y) dy) +

(10.2)

Evaluation. Let the contour C be parametrized by z = z(t) for t0 t t1 . Then the dierential on the contour is dz = z (t) dt. Using the parameterization we can evaluate a contour integral in terms of a denite integral.
t1

f (z) dz =
C t0

f (z(t))z (t) dt

Example 10.2.1 Let C be the positively oriented unit circle about the origin in the complex plane. Evaluate: 1. 2.
C

z dz dz 464

1 C z

3.

1 C z

|dz|

In each case we parameterize the contour and then do the integral. 1. z = e , dz = e d


2

z dz =
C 0

e e d
2

1 2 e = 2 0 1 4 1 0 e e = 2 2 =0 2.
C

1 dz = z

2 0

1 e d = e

d = 2
0

3. |dz| = e d = e |d| = |d| Since d is positive in this case, |d| = d. 1 |dz| = z


2 0

1 d = e e

2 0

=0

465

10.2.1

Maximum Modulus Integral Bound

The absolute value of a real integral obeys the inequality


b b

f (x) dx
a a

|f (x)| |dx| (b a) max |f (x)|.


axb

Now we prove the analogous result for the modulus of a contour integral.
n1

f (z) dz = lim
C

z0

f (k )zk |f (k )| |zk |

k=0 n1

lim =
C

z0

k=0

|f (z)| |dz| max |f (z)|


C zC

= =

|dz| |dz|
C

max |f (z)|
zC

max |f (z)| (length of C)


zC

Result 10.2.1 Maximum Modulus Integral Bound. f (z) dz


C C

|f (z)| |dz|

max |f (z)| (length of C)


zC

466

10.3

The Cauchy-Goursat Theorem

Let f (z) be analytic in a compact, closed, connected domain D. We consider the integral of f (z) on the boundary of the domain. f (z) dz = (x, y)(dx + dy) = dx + dy
D D D

Recall Greens Theorem. P dx + Q dy =


D D

(Qx Py ) dx dy

If we assume that f (z) is continuous, we can apply Greens Theorem to the integral of f (z) on D. f (z) dz =
D D

dx + dy =
D

(x y ) dx dy

Since f (z) is analytic, it satises the Cauchy-Riemann equation x = y . The integrand in the area integral, x y , is zero. Thus the contour integral vanishes. f (z) dz = 0
D

This is known as Cauchys Theorem. The assumption that f (z) is continuous is not necessary, but it makes the proof much simpler because we can use Greens Theorem. If we remove this restriction the result is known as the Cauchy-Goursat Theorem. The proof of this result is omitted.

Result 10.3.1 The Cauchy-Goursat Theorem. If f (z) is analytic in a compact, closed, connected domain D then the integral of f (z) on the boundary of the domain vanishes. f (z) dz =
D k Ck

f (z) dz = 0

Here the set of contours {Ck } make up the positively oriented boundary D of the domain D.
467

As a special case of the Cauchy-Goursat theorem we can consider a simply-connected region. For this the boundary is a Jordan curve. We can state the theorem in terms of this curve instead of referring to the boundary.

Result 10.3.2 The Cauchy-Goursat Theorem for Jordan Curves. If f (z) is analytic inside and on a simple, closed contour C, then f (z) dz = 0
C

Example 10.3.1 Let C be the unit circle about the origin with positive orientation. In Example 10.2.1 we calculated that z dz = 0
C

Now we can evaluate the integral without parameterizing the curve. We simply note that the integrand is analytic inside and on the circle, which is simple and closed. By the Cauchy-Goursat Theorem, the integral vanishes. We cannot apply the Cauchy-Goursat theorem to evaluate 1 dz = 2 z

as the integrand is not analytic at z = 0.

Example 10.3.2 Consider the domain D = {z | |z| > 1}. The boundary of the domain is the unit circle with negative orientation. f (z) = 1/z is analytic on D and its boundary. However D f (z) dz does not vanish and we cannot apply the Cauchy-Goursat Theorem. This is because the domain is not compact.

468

10.4

Contour Deformation

Path Independence. Consider a function f (z) that is analytic on a simply connected domain a contour C in that domain with end points a and b. The contour integral C f (z) dz is independent of the path connecting the end points b and can be denoted a f (z) dz. This result is a direct consequence of the Cauchy-Goursat Theorem. Let C1 and C2 be two dierent paths connecting the points. Let C2 denote the second contour with the opposite orientation. Let C be the contour which is the union of C1 and C2 . By the Cauchy-Goursat theorem, the integral along this contour vanishes. f (z) dz = f (z) dz + f (z) dz = 0
C C1 C2

This implies that the integrals along C1 and C2 are equal. f (z) dz =
C1 C2

f (z) dz

Thus contour integrals on simply connected domains are independent of path. This result does not hold for multiply connected domains.

Result 10.4.1 Path Independence. Let f (z) be analytic on a simply connected domain. For points a and b in the domain, the contour integral,
b

f (z) dz
a

is independent of the path connecting the points.


Deforming Contours. Consider two simple, closed, positively oriented contours, C1 and C2 . Let C2 lie completely within C1 . If f (z) is analytic on and between C1 and C2 then the integrals of f (z) along C1 and C2 are equal. f (z) dz =
C1 C2

f (z) dz

469

Again, this is a direct consequence of the Cauchy-Goursat Theorem. Let D be the domain on and between C1 and C2 . By the Cauchy-Goursat Theorem the integral along the boundary of D vanishes.

f (z) dz +
C1 C2

f (z) dz = 0 f (z) dz
C2

f (z) dz =
C1

By following this line of reasoning, we see that we can deform a contour C without changing the value of as long as we stay on the domain where f (z) is analytic.

f (z) dz

Result 10.4.2 Contour Deformation. Let f (z) be analytic on a domain D. If a set of closed contours {Cm } can be continuously deformed on the domain D to a set of contours {n } then the integrals along {Cm } and {n } are equal. f (z) dz =
{Cm } {n }

f (z) dz

10.5

Moreras Theorem.

The converse of the Cauchy-Goursat theorem is Moreras Theorem. If the integrals of a continuous function f (z) vanish along all possible simple, closed contours in a domain, then f (z) is analytic on that domain. To prove Moreras Theorem we will assume that rst partial derivatives of f (z) = u(x, y) + v(x, y) are continuous, although the result can be derived without this restriction. Let the simple, closed contour C be the boundary of D which is contained in 470

the domain . f (z) dz =


C C

(u + v)(dx + dy) u dx v dy +
C C

= =
D

v dx + u dy (ux vy ) dx dy
D

(vx uy ) dx dy +

=0 Since the two integrands are continuous and vanish for all C in , we conclude that the integrands are identically zero. This implies that the Cauchy-Riemann equations, ux = v y , uy = vx ,

are satised. f (z) is analytic in . The converse of the Cauchy-Goursat theorem is Moreras Theorem. If the integrals of a continuous function f (z) vanish along all possible simple, closed contours in a domain, then f (z) is analytic on that domain. To prove Moreras Theorem we will assume that rst partial derivatives of f (z) = (x, y) are continuous, although the result can be derived without this restriction. Let the simple, closed contour C be the boundary of D which is contained in the domain . f (z) dz =
C C

( dx + dy) (x y ) dx dy
D

= =0

Since the integrand, x y is continuous and vanishes for all C in , we conclude that the integrand is identically zero. This implies that the Cauchy-Riemann equation, x = y , 471

is satised. We conclude that f (z) is analytic in .

Result 10.5.1 Moreras Theorem. If f (z) is continuous in a simply connected domain and f (z) dz = 0
C

for all possible simple, closed contours C in the domain, then f (z) is analytic in .

10.6

Indenite Integrals

Consider a function f (z) which is analytic in a domain D. An anti-derivative or indenite integral (or simply integral) is a function F (z) which satises F (z) = f (z). This integral exists and is unique up to an additive constant. Note that if the domain is not connected, then the additive constants in each connected component are independent. The indenite integrals are denoted: f (z) dz = F (z) + c.

We will prove existence later by writing an indenite integral as a contour integral. We briey consider uniqueness of the indenite integral here. Let F (z) and G(z) be integrals of f (z). Then F (z) G (z) = f (z) f (z) = 0. Although we do not prove it, it certainly makes sense that F (z) G(z) is a constant on each connected component of the domain. Indenite integrals are unique up to an additive constant. Integrals of analytic functions have all the nice properties of integrals of functions of a real variable. All the formulas from integral tables, including things like integration by parts, carry over directly. 472

10.7
10.7.1

Fundamental Theorem of Calculus via Primitives


Line Integrals and Primitives

Here we review some concepts from vector calculus. Analagous to an integral in functions of a single variable is a primitive in functions of several variables. Consider a function f (x). F (x) is an integral of f (x) if and only if dF = f dx. Now we move to functions of x and y. Let P (x, y) and Q(x, y) be dened on a simply connected domain. A primitive satises d = P dx + Q dy. A necessary and sucient condition for the existence of a primitive is that Py = Qx . The denite integral can be evaluated in terms of the primitive.
(c,d)

P dx + Q dy = (c, d) (a, b)
(a,b)

10.7.2

Contour Integrals

Now consider integral along the contour C of the function f (z) = (x, y). f (z) dz =
C C

( dx + dy)

A primitive of dx + dy exists if and only if y = x . We recognize this as the Cauch-Riemann equation, x = y . Thus a primitive exists if and only if f (z) is analytic. If so, then d = dx + dy. How do we nd the primitive that satises x = and y = ? Note that choosing (x, y) = F (z) where F (z) is an anti-derivative of f (z), F (z) = f (z), does the trick. We express the complex derivative as partial derivatives in the coordinate directions to show this. F (z) = f (z) = (x, y), 473 F (z) = x = y

From this we see that x = and y = so (x, y) = F (z) is a primitive. Since we can evaluate the line integral of ( dx + dy),
(c,d)

( dx + dy) = (c, d) (a, b),


(a,b)

We can evaluate a denite integral of f in terms of its indenite integral, F .


b

f (z) dz = F (b) F (a)


a

This is the Fundamental Theorem of Calculus for functions of a complex variable.

10.8

Fundamental Theorem of Calculus via Complex Calculus

Result 10.8.1 Constructing an Indenite Integral. If f (z) is analytic in a simply connected domain D and a is a point in the domain, then
z

F (z) =
a

f () d

is analytic in D and is an indenite integral of f (z), (F (z) = f (z)).


Now we consider anti-derivatives and denite integrals without using vector calculus. From real variables we know that we can construct an integral of f (x) with a denite integral.
x

F (x) =
a

f () d

Now we will prove the analogous property for functions of a complex variable.
z

F (z) =
a

f () d

474

Let f (z) be analytic in a simply connected domain D and let a be a point in the domain. To show that F (z) = is an integral of f (z), we apply the limit denition of dierentiation. F (z) = lim F (z + z) F (z) z z+z 1 = lim f () d z0 z a z+z 1 f () d = lim z0 z z
z0

z a

f () d

f () d
a

The integral is independent of path. We choose a straight line connecting z and z + z. We add and subtract z+z zf (z) = z f (z) d from the expression for F (z). 1 F (z) = lim z0 z
z+z

zf (z) +
z z+z

(f () f (z)) d (f () f (z)) d

= f (z) + lim

1 z0 z

Since f (z) is analytic, it is certainly continuous. This means that


z

lim f () = 0.

The limit term vanishes as a result of this continuity. lim 1 z


z+z z

z0

(f () f (z)) d lim

1 |z| max |f () f (z)| z0 |z| [z...z+z] = lim max |f () f (z)|


z0 [z...z+z]

=0 Thus F (z) = f (z). 475

This results demonstrates the existence of the indenite integral. We will use this to prove the Fundamental Theorem of Calculus for functions of a complex variable.

Result 10.8.2 Fundamental Theorem of Calculus. If f (z) is analytic in a simply connected domain D then
b

f (z) dz = F (b) F (a)


a

where F (z) is any indenite integral of f (z).


From Result 10.8.1 we know that
a b

f (z) dz = F (b) + c. (Here we are considering b to be a variable.) The case b = a determines the constant.
a

f (z) dz = F (a) + c = 0
a

c = F (a) This proves the Fundamental Theorem of Calculus for functions of a complex variable. Example 10.8.1 Consider the integral 1 dz C za where C is any closed contour that goes around the point z = a once in the positive direction. We use the Fundamental Theorem of Calculus to evaluate the integral. We start at a point on the contour z a = r e . When we traverse the contour once in the positive direction we end at the point z a = r e(+2) . 1 e(+2) dz = [log(z a)]za=r e za=r C za = Log r + ( + 2) (Log r + ) = 2 476

10.9

Exercises
(z) 0, directed from z = 1 to z = 1. Evaluate

Exercise 10.1 C is the arc corresponding to the unit semi-circle, |z| = 1, 1.


C

z 2 dz z 2 dz
C

2. 3.
C

z 2 |dz| z 2 |dz|
C

4.

Hint, Solution Exercise 10.2 Evaluate

e(ax

2 +bx)

dx,

where a, b C and

(a) > 0. Use the fact that


ex dx = Hint, Solution Exercise 10.3 Evaluate

2
0

eax cos(x) dx,

and 2
0

x eax sin(x)dx,

477

where (a) > 0 and R. Hint, Solution Exercise 10.4 Use an admissible parameterization to evaluate (z z0 )n dz,
C

nZ

for the following cases: 1. C is the circle |z z0 | = 1 traversed in the counterclockwise direction. 2. C is the circle |z z0 2| = 1 traversed in the counterclockwise direction. 3. z0 = 0, n = 1 and C is the closed contour dened by the polar equation r = 2 sin2 Is this result compatible with the results of part (a)? Hint, Solution Exercise 10.5 1. Use bounding arguments to show that
R

lim

CR

z + Log z dz = 0 z3 + 1

where CR is the positive closed contour |z| = R. 2. Place a bound on Log z dz


C

where C is the arc of the circle |z| = 2 from 2 to 2. 478

3. Deduce that
C

z2 1 R2 + 1 dz r 2 z2 + 1 R 1

where C is a semicircle of radius R > 1 centered at the origin. Hint, Solution Exercise 10.6 Let C denote the entire positively oriented boundary of the half disk 0 r 1, 0 in the upper half plane. Consider the branch 3 f (z) = r e/2 , < < 2 2 of the multi-valued function z 1/2 . Show by separate parametric evaluation of the semi-circle and the two radii constituting the boundary that f (z) dz = 0.
C

Does the Cauchy-Goursat theorem apply here? Hint, Solution Exercise 10.7 Evaluate the following contour integrals using anti-derivatives and justify your approach for each. 1. z 3 + z 3 dz,
C

where C is the line segment from z1 = 1 + to z2 = . 2. sin2 z cos z dz


C

where C is a right-handed spiral from z1 = to z2 = . 479

3.

1 + e z dz = (1 ) 2 C

with z = e Log z , < Arg z < . C joins z1 = 1 and z2 = 1, lying above the real axis except at the end points. (Hint: redene z so that it remains unchanged above the real axis and is dened continuously on the real axis.) Hint, Solution

480

10.10
Hint 10.1

Hints

Hint 10.2 2 Let C be the parallelogram in the complex plane with corners at R and R + b/(2a). Consider the integral of eaz on this contour. Take the limit as R . Hint 10.3 Extend the range of integration to ( . . . ). Use ex = cos(x) + sin(x) and the result of Exercise 10.2. Hint 10.4 Hint 10.5 Hint 10.6 Hint 10.7

481

10.11

Solutions

Solution 10.1 We parameterize the path with z = e , with ranging from to 0.

dz = e d |dz| = | e d| = |d| = d

1.

z 2 dz =
C 0

e2 e d e3 d

= =

1 3 e 3

1 0 e e3 = 3 1 = (1 (1)) 3 2 = 3 482

2.
0

|z | dz =
C 0

| e2 | e d e d
0 e

= =

= 1 (1) =2 3.
0

z 2 |dz| =
C 0

e2 | e d| e2 d

= =

2 0 e 2 = (1 1) 2 =0 4.
0

|z 2 | |dz| =
C 0

| e2 || e d| d

= []0 = 483

Solution 10.2

I=

e(ax

2 +bx)

dx

First we complete the square in the argument of the exponential. I=e


b2 /(4a)

ea(x+b/(2a)) dx
2

Consider the parallelogram in the complex plane with corners at R and R + b/(2a). The integral of eaz on this contour vanishes as it is an entire function. We relate the integral along one side of the parallelogram to the integrals along the other three sides.
R+b/(2a) R+b/(2a)

eaz dz =
R+b/(2a)

R+b/(2a)

+
R

+
R

eaz dz.

The rst and third integrals on the right side vanish as R because the integrand vanishes and the lengths of the paths of integration are nite. Taking the limit as R we have,
+b/(2a) +b/(2a)

eaz dz

ea(x+b/(2a)) dx =

eax dx.

Now we have I = eb We make the change of variables = ax. I = eb

2 /(4a) 2 /(4a)

eax dx.

1 a

e d b2 /(4a) e a

e(ax

2 +bx)

dx =

484

Solution 10.3 Consider

I=2
0

eax cos(x) dx.

Since the integrand is an even function, I=

eax cos(x) dx.


Since

2 eax

sin(x) is an odd function, I=

eax ex dx.

We evaluate this integral with the result of Exercise 10.2.

2
0

eax cos(x) dx =

2 /(4a) e a

Consider I=2
0

x eax sin(x) dx.

Since the integrand is an even function, I=

x eax sin(x) dx.

Since x

2 eax

cos(x) is an odd function,

I =

x eax ex dx.

We add a dash of integration by parts to get rid of the x factor.


1 1 2 2 + eax ex I = eax ex 2a 2a 2 eax ex dx I= 2a

dx

485

2
0

x eax sin(x) dx =

2a

2 /(4a) e a

Solution 10.4 1. We parameterize the contour and do the integration. z z0 = e ,


2

[0 . . . 2)

(z z0 )n dz =
C

en e d
0 2 e(n+1) n+1 0 []2 0

for n = 1 for n = 1

0 2

for n = 1 for n = 1

2. We parameterize the contour and do the integration. z z0 = 2 + e ,


2

[0 . . . 2)

(z z0 ) dz =
C

2 + e
0 (2+e )n+1 n+1

e d for n = 1 for n = 1

log 2 +

0 2 e 0

=0

3. We parameterize the contour and do the integration. z = r e , r = 2 sin2 4 , [0 . . . 4)

486

-1 -1

Figure 10.2: The contour: r = 2 sin2 The contour encircles the origin twice. See Figure 10.2.
4

z 1 dz =
C 0 4

=
0

1 (r () + r()) e d r() e r () + d r()

= [log(r()) + ]4 0 487

Since r() does not vanish, the argument of r() does not change in traversing the contour and thus the logarithmic term has the same value at the beginning and end of the path. z 1 dz = 4
C

This answer is twice what we found in part (a) because the contour goes around the origin twice. Solution 10.5 1. We parameterize the contour with z = R e and bound the modulus of the integral. z + Log z dz z3 + 1 z + Log z |dz| z3 + 1

CR

R + ln R + R d R3 1 0 R + ln R + = 2r R3 1 The upper bound on the modulus on the integral vanishes as R . lim 2r R + ln R + =0 R3 1

CR 2

We conclude that the integral vanishes as R . lim z + Log z dz = 0 z3 + 1

CR

2. We parameterize the contour and bound the modulus of the integral. z = 2 e , [/2 . . . /2] 488

Log z dz
C

|Log z| |dz|
C /2

=
/2

| ln 2 + |2 d
/2

2
/2 /2

(ln 2 + ||) d (ln 2 + ) d


0

=4 =

( + 4 ln 2) 2

3. We parameterize the contour and bound the modulus of the integral. z = R e , [0 . . . 0 + ]

z2 1 dz z2 + 1

z2 1 |dz| 2 C z +1 0 + R2 e2 1 |R d| R2 e2 +1 0
0 +

R2 + 1 d R2 1 0 R2 + 1 = r 2 R 1 R

489

Solution 10.6

f (z) dz =
C 0

r dr +
0

e/2 e d +
1

r (dr)

2 2 2 = + 3 3 3 =0

2 + 3

The Cauchy-Goursat theorem does not apply because the function is not analytic at z = 0, a point on the boundary. Solution 10.7 1. z + z
C 3 3

z 4 1 dz = 2 4 2z 1 = + 2

1+

In this example, the anti-derivative is single-valued. 2. sin3 z sin z cos z dz = 3 C 1 sin3 () sin3 () = 3 sinh3 () = 3
2

Again the anti-derivative is single-valued. 490

3. We choose the branch of z with /2 < arg(z) < 3/2. This matches the principal value of z above the real axis and is dened continuously on the path of integration. z 1+ z dz = 1+ C
e0 e e0

1 (1+) log z e = 2 e 1 0 e e(1+) = 2 1 + e = (1 ) 2

491

Chapter 11 Cauchys Integral Formula


If I were founding a university I would begin with a smoking room; next a dormitory; and then a decent reading room and a library. After that, if I still had more money that I couldnt use, I would hire a professor and get some text books.

- Stephen Leacock 492

11.1

Cauchys Integral Formula

Result 11.1.1 Cauchys Integral Formula. If f () is analytic in a compact, closed, connected domain D and z is a point in the interior of D then f (z) = 1 2 f () 1 d = z 2 f () d. z (11.1)

Ck

Here the set of contours {Ck } make up the positively oriented boundary D of the domain D. More generally, we have f (n) (z) = n! 2 f () n! d = ( z)n+1 2 f () d. ( z)n+1 (11.2)

Ck

Cauchys Formula shows that the value of f (z) and all its derivatives in a domain are determined by the value of f (z) on the boundary of the domain. Consider the rst formula of the result, Equation 11.1. We deform the contour to a circle of radius about the point = z. f () d = z =
C

f () d z f (z) d + z

f () f (z) d z

We use the result of Example 10.8.1 to evaluate the rst integral. f () d = 2f (z) + z 493 f () f (z) d z

The remaining integral along C vanishes as 0 because f () is continuous. We demonstrate this with the maximum modulus integral bound. The length of the path of integration is 2. lim f () f (z) 1 d lim (2) max |f () f (z)| 0 z |z|= lim 2 max |f () f (z)|
0 |z|=

=0 This gives us the desired result. f (z) = 1 2


C

f () d z

We derive the second formula, Equation 11.2, from the rst by dierentiating with respect to z. Note that the integral converges uniformly for z in any closed subset of the interior of C. Thus we can dierentiate with respect to z and interchange the order of dierentiation and integration. f (n) (z) = 1 dn f () d n 2 dz C z 1 dn f () = d 2 C dz n z n! f () = d 2 C ( z)n+1

Example 11.1.1 Consider the following integrals where C is the positive contour on the unit circle. For the third integral, the point z = 1 is removed from the contour. 1.
C

sin cos z 5

dz

494

2.
C

1 dz (z 3)(3z 1) z dz

3.
C

1. Since sin (cos (z 5 )) is an analytic function inside the unit circle, sin cos z 5
C 1 (z3)(3z1)

dz = 0

2.

has singularities at z = 3 and z = 1/3. Since z = 3 is outside the contour, only the singularity at z = 1/3 will contribute to the value of the integral. We will evaluate this integral using the Cauchy integral formula. 1 1 dz = 2 = (1/3 3)3 4 C (z 3)(3z 1)

3. Since the curve is not closed, we cannot apply the Cauchy integral formula. Note that z is single-valued and analytic in the complex plane with a branch cut on the negative real axis. Thus we use the Fundamental Theorem of Calculus.
C

2 3 z z dz = 3 e 2 3/2 e = e3/2 3 2 = ( ) 3 4 = 3

495

Cauchys Inequality. Suppose the f () is analytic in the closed disk | z| r. By Cauchys integral formula, n! f () d, 2 C ( z)n+1 where C is the circle of radius r centered about the point z. We use this to obtain an upper bound on the modulus of f (n) (z). f (n) (z) = f (n) (z) = f () n! d 2 C ( z)n+1 n! f () 2r max |z|=r ( z)n+1 2 n! = n max |f ()| r |z|=r

Result 11.1.2 Cauchys Inequality. If f () is analytic in | z| r then f (n) (z) where |f ()| M for all | z| = r.
Liouvilles Theorem. Consider a function f (z) that is analytic and bounded, (|f (z)| M ), in the complex plane. From Cauchys inequality, M |f (z)| r for any positive r. By taking r , we see that f (z) is identically zero for all z. Thus f (z) is a constant.

n!M rn

Result 11.1.3 Liouvilles Theorem. If f (z) is analytic and |f (z)| is bounded in the complex plane then f (z) is a constant.

496

The Fundamental Theorem of Algebra. We will prove that every polynomial of degree n 1 has exactly n roots, counting multiplicities. First we demonstrate that each such polynomial has at least one root. Suppose that an nth degree polynomial p(z) has no roots. Let the lower bound on the modulus of p(z) be 0 < m |p(z)|. The function f (z) = 1/p(z) is analytic, (f (z) = p (z)/p2 (z)), and bounded, (|f (z)| 1/m), in the extended complex plane. Using Liouvilles theorem we conclude that f (z) and hence p(z) are constants, which yields a contradiction. Therefore every such polynomial p(z) must have at least one root. Now we show that we can factor the root out of the polynomial. Let
n

p(z) =
k=0

pk z k .

We note that
n1

(z c ) = (z c)
k=0

cn1k z k .

Suppose that the nth degree polynomial p(z) has a root at z = c. p(z) = p(z) p(c)
n n

=
k=0 n

pk z
k=0

pk ck

=
k=0 n

pk z k ck
k1

=
k=0

pk (z c)
j=0

ck1j z j

= (z c)q(z) Here q(z) is a polynomial of degree n 1. By induction, we see that p(z) has exactly n roots. 497

Result 11.1.4 Fundamental Theorem of Algebra. Every polynomial of degree n 1 has exactly n roots, counting multiplicities.
Gauss Mean Value Theorem. Let f () be analytic in | z| r. By Cauchys integral formula, f (z) = 1 2 f () d, z

where C is the circle | z| = r. We parameterize the contour with = z + r e . f (z) = Writing this in the form, f (z) = 1 2r
2

1 2

2 0

f (z + r e ) r e d r e

f (z + r e )r d,
0

we see that f (z) is the average value of f () on the circle of radius r about the point z.

Result 11.1.5 Gauss Average Value Theorem. If f () is analytic in | z| r then 1 f (z) = 2


2

f (z + r e ) d.
0

That is, f (z) is equal to its average value on a circle of radius r about the point z.
Extremum Modulus Theorem. Let f (z) be analytic in closed, connected domain, D. The extreme values of the modulus of the function must occur on the boundary. If |f (z)| has an interior extrema, then the function is a constant. We will show this with proof by contradiction. Assume that |f (z)| has an interior maxima at the point z = c. This 498

means that there exists an neighborhood of the point z = c for which |f (z)| |f (c)|. Choose an |z c| lies inside this neighborhood. First we use Gauss mean value theorem. 1 2
2

so that the set

f (c) =

f c + e d
0

We get an upper bound on |f (c)| with the maximum modulus integral bound. |f (c)| 1 2
2

f c + e
0

Since z = c is a maxima of |f (z)| we can get a lower bound on |f (c)|. |f (c)| 1 2


2

f c + e
0

If |f (z)| < |f (c)| for any point on |z c| = , then the continuity of f (z) implies that |f (z)| < |f (c)| in a neighborhood of that point which would make the value of the integral of |f (z)| strictly less than |f (c)|. Thus we conclude that |f (z)| = |f (c)| for all |z c| = . Since we can repeat the above procedure for any circle of radius smaller than , |f (z)| = |f (c)| for all |z c| , i.e. all the points in the disk of radius about z = c are also maxima. By recursively repeating this procedure points in this disk, we see that |f (z)| = |f (c)| for all z D. This implies that f (z) is a constant in the domain. By reversing the inequalities in the above method we see that the minimum modulus of f (z) must also occur on the boundary.

Result 11.1.6 Extremum Modulus Theorem. Let f (z) be analytic in a closed, connected domain, D. The extreme values of the modulus of the function must occur on the boundary. If |f (z)| has an interior extrema, then the function is a constant.

499

11.2

The Argument Theorem

Result 11.2.1 The Argument Theorem. Let f (z) be analytic inside and on C except for isolated poles inside the contour. Let f (z) be nonzero on C. 1 2 f (z) dz = N P f (z)

Here N is the number of zeros and P the number of poles, counting multiplicities, of f (z) inside C.

First we will simplify the problem and consider a function f (z) that has one zero or one pole. Let f (z) be analytic and nonzero inside and on A except for a zero of order n at z = a. Then we can write f (z) = (z a)n g(z) where g(z) (z) is analytic and nonzero inside and on A. The integral of f (z) along A is f 1 2 f (z) 1 dz = f (z) 2 1 = 2 1 = 2 1 = 2 =n d (log(f (z))) dz dz d (log((z a)n ) + log(g(z))) dz dz d (log((z a)n )) dz dz n dz za

500

Now let f (z) be analytic and nonzero inside and on B except for a pole of order p at z = b. Then we can write g(z) (z) f (z) = (zb)p where g(z) is analytic and nonzero inside and on B. The integral of f (z) along B is f 1 2
B

f (z) 1 dz = f (z) 2 1 = 2 1 = 2 1 = 2 = p

d (log(f (z))) dz dz d log((z b)p ) + log(g(z)) dz dz d log((z b)p )+ dz dz p dz zb

Now consider a function f (z) that is analytic inside an on the contour C except for isolated poles at the points b1 , . . . , bp . Let f (z) be nonzero except at the isolated points a1 , . . . , an . Let the contours Ak , k = 1, . . . , n, be simple, positive contours which contain the zero at ak but no other poles or zeros of f (z). Likewise, let the contours Bk , k = 1, . . . , p be simple, positive contours which contain the pole at bk but no other poles of zeros of f (z). (See Figure 11.1.) By deforming the contour we obtain f (z) dz = f (z)
n Aj

j=1

f (z) dz + f (z) k=1

Bj

f (z) dz. f (z)

From this we obtain Result 11.2.1.

11.3

Rouches Theorem

Result 11.3.1 Rouches Theorem. Let f (z) and g(z) be analytic inside and on a simple, closed contour C. If |f (z)| > |g(z)| on C then f (z) and f (z) + g(z) have the same number of zeros inside C and no zeros on C.
501

A1 B1 B2 B3

A2

Figure 11.1: Deforming the contour C. First note that since |f (z)| > |g(z)| on C, f (z) is nonzero on C. The inequality implies that |f (z) + g(z)| > 0 on C so f (z) + g(z) has no zeros on C. We well count the number of zeros of f (z) and g(z) using the Argument Theorem, (Result 11.2.1). The number of zeros N of f (z) inside the contour is N= 1 2 f (z) dz. f (z)

Now consider the number of zeros M of f (z) + g(z). We introduce the function h(z) = g(z)/f (z). M= 1 2 1 = 2 1 = 2 =N+ =N 502 f (z) + g (z) dz f (z) + g(z) f (z) + f (z)h(z) + f (z)h (z) dz f (z) + f (z)h(z) f (z) 1 h (z) dz + dz f (z) 2 C 1 + h(z)

1 [log(1 + h(z))]C 2

(Note that since |h(z)| < 1 on C, (1 + h(z)) > 0 on C and the value of log(1 + h(z)) does not not change in traversing the contour.) This demonstrates that f (z) and f (z) + g(z) have the same number of zeros inside C and proves the result.

503

11.4

Exercises
(arg(sin z))

Exercise 11.1 What is


C

where C is the unit circle? Exercise 11.2 Let C be the circle of radius 2 centered about the origin and oriented in the positive direction. Evaluate the following integrals: 1. 2. 3.
sin z C z 2 +5 z C z 2 +1 C z 2 +1 z

dz dz dz

Exercise 11.3 Let f (z) be analytic and bounded (i.e. |f (z)| < M ) for |z| > R, but not necessarily analytic for |z| R. Let the points and lie inside the circle |z| = R. Evaluate f (z) dz (z )(z )

where C is any closed contour outside |z| = R, containing the circle |z| = R. [Hint: consider the circle at innity] Now suppose that in addition f (z) is analytic everywhere. Deduce that f () = f (). Exercise 11.4 Using Rouches theorem show that all the roots of the equation p(z) = z 6 5z 2 + 10 = 0 lie in the annulus 1 < |z| < 2. Exercise 11.5 Evaluate as a function of t = 1 2
C

ezt dz, z 2 (z 2 + a2 ) 504

where C is any positively oriented contour surrounding the circle |z| = a. Exercise 11.6 Consider C1 , (the positively oriented circle |z| = 4), and C2 , (the positively oriented boundary of the square whose sides lie along the lines x = 1, y = 1). Explain why f (z) dz =
C1 C2

f (z) dz

for the functions 1 +1 z 2. f (z) = 1 ez 1. f (z) = 3z 2 Exercise 11.7 Show that if f (z) is of the form f (z) = k k1 1 + k1 + + + g(z), zk z z k1

where g is analytic inside and on C, (the positive circle |z| = 1), then f (z) dz = 21 .
C

Exercise 11.8 Show that if f (z) is analytic within and on a simple closed contour C and z0 is not on C then f (z) dz = z z0 f (z) dz. (z z0 )2

Note that z0 may be either inside or outside of C. 505

Exercise 11.9 If C is the positive circle z = e show that for any real constant a, eaz dz = 2 z

and hence
0

ea cos cos(a sin ) d = . Exercise 11.10 Use Cauchy-Goursat, the generalized Cauchy integral formula, and suitable extensions to multiply-connected domains to evaluate the following integrals. Be sure to justify your approach in each case. 1. z dz 3 C z 9 where C is the positively oriented rectangle whose sides lie along x = 5, y = 3. sin z dz, 4)

2.
C

z 2 (z

where C is the positively oriented circle |z| = 2. 3.


C

(z 3 + z + ) sin z dz, z 4 + z 3

where C is the positively oriented circle |z| = . 4.


C

ezt dz z 2 (z + 1)

where C is any positive simple closed contour surrounding |z| = 1. 506

Exercise 11.11 Use Liouvilles theorem to prove the following: 1. If f (z) is entire with (f (z)) M for all z then f (z) is constant.

2. If f (z) is entire with |f (5) (z)| M for all z then f (z) is a polynomial of degree at most ve. Exercise 11.12 Find all functions f (z) analytic in the domain D : |z| < R that satisfy f (0) = e and |f (z)| 1 for all z in D. Exercise 11.13 Let f (z) = k 4 k=0 1.
C z k 4

and evaluate the following contour integrals, providing justication in each case: C is the positive circle |z 1| = 1.

cos(z)f (z) dz f (z) dz z3

2.
C

C is the positive circle |z| = .

507

11.5

Hints

Hint 11.1 Use the argument theorem. Hint 11.2

Hint 11.3 To evaluate the integral, consider the circle at innity. Hint 11.4

Hint 11.5

Hint 11.6

Hint 11.7

Hint 11.8

Hint 11.9

Hint 11.10

508

Hint 11.11 Hint 11.12 Hint 11.13

509

11.6

Solutions

Solution 11.1 Let f (z) be analytic inside and on the contour C. Let f (z) be nonzero on the contour. The argument theorem states that 1 f (z) dz = N P, 2 C f (z) where N is the number of zeros and P is the number of poles, (counting multiplicities), of f (z) inside C. The theorem is aptly named, as 1 2 f (z) 1 dz = [log(f (z))]C f (z) 2 1 = [log |f (z)| + arg(f (z))]C 2 1 [arg(f (z))]C . = 2 f (z) 1 dz = [arg(f (z))]C = N P. f (z) 2

Thus we could write the argument theorem as 1 2

Since sin z has a single zero and no poles inside the unit circle, we have 1 arg(sin(z)) 2 arg(sin(z)) Solution 11.2 1. Since the integrand zsin z is analytic inside and on the contour, (the only singularities are at z = 5 and at 2 +5 innity), the integral is zero by Cauchys Theorem. 510
C

=10 = 2

2. First we expand the integrand in partial fractions. z2 a= z z+ z a b = + +1 z z+ 1 = , 2 b= z z =


z=

z=

1 2

Now we can do the integral with Cauchys formula. z2 z dz = +1 1/2 dz + C z 1 1 = 2 + 2 2 2 = 2 1/2 dz z+

3. z2 + 1 dz = z =
C

z+
C

1 z
C

dz 1 dz z

z dz +

= 0 + 2 = 2 Solution 11.3 Let C be the circle of radius r, (r > R), centered at the origin. We get an upper bound on the integral with the Maximum Modulus Integral Bound, (Result 10.2.1). f (z) f (z) M dz 2r max 2r |z|=r (z )(z ) (z )(z ) (r ||)(r ||) 511

By taking the limit as r we see that the modulus of the integral is bounded above by zero. Thus the integral vanishes. Now we assume that f (z) is analytic and evaluate the integral with Cauchys Integral Formula. (We assume that = .) f (z) dz = 0 C (z )(z ) f (z) f (z) dz + dz = 0 (z )( ) C ( )(z ) f () f () + 2 =0 2 f () = f ()

Solution 11.4 Consider the circle |z| = 2. On this circle: |z 6 | = 64 | 5z 2 + 10| | 5z 2 | + |10| = 30 Since |z 6 | < | 5z 2 + 10| on |z| = 2, p(z) has the same number of roots as z 6 in |z| < 2. p(z) has 6 roots in |z| < 2. Consider the circle |z| = 1. On this circle: |10| = 10 |z 6 5z 2 | |z 6 | + | 5z 2 | = 6 Since |z 6 5z 2 | < |10| on |z| = 1, p(z) has the same number of roots as 10 in |z| < 1. p(z) has no roots in |z| < 1. On the unit circle, |p(z)| |10| |z 6 | |5z 2 | = 4. Thus p(z) has no roots on the unit circle. We conclude that p(z) has exactly 6 roots in 1 < |z| < 2. 512

Solution 11.5 We evaluate the integral with Cauchys Integral Formula. ezt dz 2 2 2 C z (z + a ) ezt ezt ezt 1 = + 3 3 2 C a2 z 2 2a (z a) 2a (z + a) eat eat d ezt + = dz a2 z=0 2a3 2a3 t sin(at) = 2 a a3 at sin(at) = a3 = 1 2 Solution 11.6 1. We factor the denominator of the integrand. 3z 2 1 1 = +1 3(z 3/3)(z + 3/3)

dz

There are two rst order poles which could contribute to the value of an integral on a closed path. Both poles lie inside both contours. See Figure 11.2. We see that C1 can be continuously deformed to C2 on the domain where the integrand is analytic. Thus the integrals have the same value. 2. We consider the integrand z . 1 ez Since ez = 1 has the solutions z = 2n for n Z, the integrand has singularities at these points. There is a removable singularity at z = 0 and rst order poles at z = 2n for n Z \ {0}. Each contour contains only the singularity at z = 0. See Figure 11.3. We see that C1 can be continuously deformed to C2 on the domain where the integrand is analytic. Thus the integrals have the same value. 513

4 2 -4 -2 -2 -4 2 4

Figure 11.2: The contours and the singularities of

1 . 3z 2 +1

6 4 2 -6 -4 -2 -2 -4 -6 2 4 6

Figure 11.3: The contours and the singularities of

z . 1ez

514

Solution 11.7 First we write the integral of f (z) as a sum of integrals. f (z) dz =
C C

=
C

k k1 1 + k1 + + + g(z) dz k z z z k k1 1 dz + dz + + dz + k k1 z C z C z

g(z) dz
C

The integral of g(z) vanishes by the Cauchy-Goursat theorem. We evaluate the integral of 1 /z with Cauchys integral formula. 1 dz = 21 C z We evaluate the remaining n /z n terms with anti-derivatives. Each of these integrals vanish. f (z) dz =
C

k1 k dz + dz + + k k1 C z C z k 2 = + + k1 (k 1)z z C = 21

1 dz + z + 21

g(z) dz
C

Solution 11.8 We evaluate the integrals with the Cauchy integral formula. (z0 is required to not be on C so the integrals exist.) f (z) dz = z z0 2f (z0 ) if z0 is inside C 0 if z0 is outside C
2 f 1!

f (z) dz = (z z0 )2

(z0 ) if z0 is inside C if z0 is outside C

Thus we see that the integrals are equal. 515

Solution 11.9 First we evaluate the integral using the Cauchy Integral Formula. eaz dz = [eaz ]z=0 = 2 z eaz dz = 2 z e d = 2

Next we parameterize the path of integration. We use the periodicity of the cosine and sine to simplify the integral.
C ea e

2 0 2 0 2

ea(cos + sin ) d = 2 ea cos (cos(sin ) + sin(sin )) d = 2


0 2

ea cos cos(sin ) d = 2
0

ea cos cos(sin ) d =
0

Solution 11.10 1. We factor the integrand to see that there are singularities at the cube roots of 9. z z = 3 3 z3 9 z 9 z 9 e2/3 z 3 9 e2/3 Let C1 , C2 and C3 be contours around z = 3 9, z = 3 9 e2/3 and z = 3 9 e2/3 . See Figure 11.4. Let D be the domain between C, C1 and C2 , i.e. the boundary of D is the union of C, C1 and C2 . Since the integrand is analytic in D, the integral along the boundary of D vanishes.
D

z dz = z3 9

z dz + z3 9

C1

z dz + z3 9 516

C2

z dz + z3 9

C3

z dz = 0 z3 9

From this we see that the integral along C is equal to the sum of the integrals along C1 , C2 and C3 . (We could also see this by deforming C onto C1 , C2 and C3 .) z dz = 9 z dz + 9 z dz + 9 z dz 9

z3

C1

z3

C2

z3

C3

z3

We use the Cauchy Integral Formula to evaluate the integrals along C1 , C2 and C2 . z dz = z3 9 3 dz 9 e2/3 z 3 9 e2/3 z dz 3 2/3 z 9e z 3 9 e2/3 z dz 3 z 9 e2/3 z 3 9 e2/3 z 9 e2/3 3 3 z 3 9 e2/3
z= 3 9

C1

z
C2

9 3 3 9 9

+ +
C3

z z z

= 2

+ 2

z z 3 9 e2/3 z z 3 9 e2/3

z= 3 9 e2/3

+ 2

z= 3 9 e2/3

= 235/3 1 e/3 + e2/3 =0

2. The integrand has singularities at z = 0 and z = 4. Only the singularity at z = 0 lies inside the contour. We use 517

C C1
2 4 6

C2
-6 -4 -2

C3

-2 -4

Figure 11.4: The contours for the Cauchy Integral Formula to evaluate the integral. z 2 (z

z . z 3 9

d sin z sin z dz = 2 4) dz z 4 z=0 cos z sin z = 2 z 4 (z 4)2 = 2

z=0

3. We factor the integrand to see that there are singularities at z = 0 and z = . (z 3 + z + ) sin z dz = z 4 + z 3 (z 3 + z + ) sin z dz z 3 (z + )

Let C1 and C2 be contours around z = 0 and z = . See Figure 11.5. Let D be the domain between C, C1 and C2 , i.e. the boundary of D is the union of C, C1 and C2 . Since the integrand is analytic in D, the integral along the boundary of D vanishes. =
D C

+
C1

+
C2

=0

518

From this we see that the integral along C is equal to the sum of the integrals along C1 and C2 . (We could also see this by deforming C onto C1 and C2 .) =
C C1

+
C2

We use the Cauchy Integral Formula to evaluate the integrals along C1 and C2 . (z 3 + z + ) sin z dz = z 4 + z 3 (z 3 + z + ) sin z (z 3 + z + ) sin z dz + dz z 3 (z + ) z 3 (z + ) C1 C2 2 d2 (z 3 + z + ) sin z (z 3 + z + ) sin z + = 2 z3 2! dz 2 z+ z=
2 3

z=0

3z + 1 z + z + cos z z+ (z + )2 6z 2(3z 2 + 1) 2(z 3 + z + ) z 3 + z + + + z+ (z + )2 (z + )3 z+ = 2 sinh(1) = 2( sinh(1)) + 2

sin z
z=0

4. We consider the integral


C

ezt dz. z 2 (z + 1)

There are singularities at z = 0 and z = 1. Let C1 and C2 be contours around z = 0 and z = 1. See Figure 11.6. We deform C onto C1 and C2 . =
C C1

+
C2

519

4 2 -4 -2 -2 -4
Figure 11.5: The contours for
(z 3 +z+) sin z . z 4 +z 3

C1 C2 2

C
4

We use the Cauchy Integral Formula to evaluate the integrals along C1 and C2 . ezt dz = z 2 (z + 1) ezt ezt dz + dz 2 2 C1 z (z + 1) C1 z (z + 1) ezt d ezt = 2 2 + 2 z z=1 dz (z + 1) z=0 zt ezt te = 2 et +2 (z + 1) (z + 1)2 z=0 = 2(et +t 1)

Solution 11.11 Liouvilles Theorem states that if f (z) is analytic and bounded in the complex plane then f (z) is a constant. 520

2 1

C2
-2 -1 -1 -2

C1
1

C
2

Figure 11.6: The contours for

ezt . z 2 (z+1)

1. Since f (z) is analytic, ef (z) is analytic. The modulus of ef (z) is bounded. ef (z) = e
(f (z))

eM

By Liouvilles Theorem we conclude that ef (z) is constant and hence f (z) is constant. 2. We know that f (z) is entire and |f (5) (z)| is bounded in the complex plane. Since f (z) is analytic, so is f (5) (z). We apply Liouvilles Theorem to f (5) (z) to conclude that it is a constant. Then we integrate to determine the form of f (z). f (z) = c5 z 5 + c4 z 4 + c3 z 3 + c2 z 2 + c1 z + c0 Here c5 is the value of f (5) (z) and c4 through c0 are constants of integration. We see that f (z) is a polynomial of degree at most ve. Solution 11.12 For this problem we will use the Extremum Modulus Theorem: Let f (z) be analytic in a closed, connected domain, D. The extreme values of the modulus of the function must occur on the boundary. If |f (z)| has an interior extrema, then the function is a constant. 521

Since |f (z)| has an interior extrema, |f (0)| = | e | = 1, we conclude that f (z) is a constant on D. Since we know the value at z = 0, we know that f (z) = e . Solution 11.13 First we determine the radius of convergence of the series with the ratio test. R = lim k 4 /4k k (k + 1)4 /4k+1 k4 = 4 lim k (k + 1)4 24 = 4 lim k 24 =4

The series converges absolutely for |z| < 4. 1. Since the integrand is analytic inside and on the contour of integration, the integral vanishes by Cauchys Theorem. 2. f (z) dz = z3 =
C k=1

k4
C k=0

z 4

1 dz z3

k 4 k3 z dz 4k

= =

(k + 3)4 k z dz 4k+3 C k=2


C

1 dz + 4z 2

1 dz + z

C k=0

(k + 3)4 k z dz 4k+3

522

We can parameterize the rst integral to show that it vanishes. The second integral has the value 2 by the Cauchy-Goursat Theorem. The third integral vanishes by Cauchys Theorem as the integrand is analytic inside and on the contour. f (z) dz = 2 3 C z

523

Chapter 12 Series and Convergence


You are not thinking. You are merely being logical. - Neils Bohr

12.1
12.1.1

Series of Constants
Denitions
lim an = a

Convergence of Sequences. The innite sequence {an } a0 , a1 , a2 , . . . is said to converge if n=0


n

for some constant a. If the limit does not exist, then the sequence diverges. Recall the denition of the limit in the above formula: For any > 0 there exists an N Z such that |a an | < for all n > N . Example 12.1.1 The sequence {sin(n)} is divergent. The sequence is bounded above and below, but boundedness does not imply convergence.

524

Cauchy Convergence Criterion. Note that there is something a little shy about the above denition. We should be able to say if a sequence converges without rst nding the constant to which it converges. We x this problem with the Cauchy convergence criterion. A sequence {an } converges if and only if for any > 0 there exists an N such that |an am | < for all n, m > N . The Cauchy convergence criterion is equivalent to the denition we had before. For some problems it is handier to use. Now we dont need to know the limit of a sequence to show that it converges. Convergence of Series. The series That is,
n=1

an converges if the sequence of partial sums, SN =


N 1

N 1 n=0

an , converges.

lim SN = lim

an = constant.
n=0

If the limit does not exist, then the series diverges. A necessary condition for the convergence of a series is that
n

lim an = 0.

(See Exercise 12.1.) Otherwise the sequence of partial sums would not converge. Example 12.1.2 The series (1)n = 1 1 + 1 1 + is divergent because the sequence of partial sums, n=0 {SN } = 1, 0, 1, 0, 1, 0, . . . is divergent.

Tail of a Series. An innite series, an , converges or diverges with its tail. That is, for xed N , an n=0 n=0 converges if and only if an converges. This is because the sum of the rst N terms of a series is just a number. n=N Adding or subtracting a number to a series does not change its convergence. Absolute Convergence. The series an converges absolutely if |an | converges. Absolute convergence n=0 n=0 implies convergence. If a series is convergent, but not absolutely convergent, then it is said to be conditionally convergent. 525

The terms of an absolutely convergent series can be rearranged in any order and the series will still converge to the same sum. This is not true of conditionally convergent series. Rearranging the terms of a conditionally convergent series may change the sum. In fact, the terms of a conditionally convergent series may be rearranged to obtain any desired sum. Example 12.1.3 The alternating harmonic series, 1 converges, (Exercise 12.4). Since 1+ 1 1 1 + + + 2 3 4 1 1 1 + + , 2 3 4

diverges, (Exercise 12.5), the alternating harmonic series is not absolutely convergent. Thus the terms can be rearranged to obtain any sum, (Exercise 12.6).

Finite Series and Residuals. Consider the series f (z) = terms in the series as
N 1

n=0

an (z). We will denote the sum of the rst N

SN (z) =
n=0

an (z).

We will denote the residual after N terms as

RN (z) f (z) SN (z) =


n=N

an (z).

12.1.2

Special Series

526

Geometric Series. One of the most important series in mathematics is the geometric series,

zn = 1 + z + z2 + z3 + .
n=0

The series clearly diverges for |z| 1 since the terms do not vanish as n . Consider the partial sum, SN (z) N 1 n n=0 z , for |z| < 1.
N 1

(1 z)SN (z) = (1 z)
N 1

zn
n=0 N

=
n=0

zn
n=1

zn

= 1 + z + + z N 1 z + z 2 + + z N = 1 zN
N 1

zn =
n=0

1 zN 1 1z 1z

as N .

The limit of the partial sums is

1 . 1z

zn =
n=0

1 1z

for |z| < 1

Harmonic Series. Another important series is the harmonic series,

n=1
1

1 1 1 = 1 + + + . n 2 3

The series is so named because the terms grow or decay geometrically. Each term in the series is a constant times the previous term.

527

The series is absolutely convergent for () > 1 and absolutely divergent for Riemann zeta function () is dened as the sum of the harmonic series.

() 1, (see the Exercise 12.8). The

() =
n=1

1 n

The alternating harmonic series is

n=1

(1)n+1 1 1 1 = 1 + + . n 2 3 4 () > 1 and absolutely divergent for () 1.

Again, the series is absolutely convergent for

12.1.3

Convergence Tests

The Comparison Test.

Result 12.1.1 The series of positive terms an converges if there exists a convergent series bn such that an bn for all n. Similarly, an diverges if there exists a divergent series bn such that an bn for all n.
Example 12.1.4 Consider the series
n=1

1 . 2n2 1 . 2n

We can rewrite this as

n=1 n a perfect square

528

Then by comparing this series to the geometric series,

n=1

1 = 1, 2n

we see that it is convergent.

Integral Test.

Result 12.1.2 If the coecients an of a series an are monotonically decreasing and n=0 can be extended to a monotonically decreasing function of the continuous variable x, a(x) = an for x Z0+ ,

then the series converges or diverges with the integral

a(x) dx.
0

Example 12.1.5 Consider the series

1 n=1 n2 .

Dene the functions sl (x) and sr (x), (left and right), 1 , ( x )2 sr (x) = 1 . ( x )2

sl (x) =

Recall that x is the greatest integer function, the greatest integer which is less than or equal to x. x is the least integer function, the least integer greater than or equal to x. We can express the series as integrals of these functions.

n=1

1 = n2

sl (x) dx =
0 1

sr (x) dx

529

In Figure 12.1 these functions are plotted against y = 1/x2 . From the graph, it is clear that we can obtain a lower and upper bound for the series.
1 1

1 dx x2

n=1

1 1+ n2 1 2 n2

1 dx x2

1
n=1

1
n=1

Figure 12.1: Upper and Lower bounds to

1/n2 .

In general, we have

a(x) dx
m n=m

an am +
m

a(x) dx.

Thus we see that the sum converges or diverges with the integral. 530

The Ratio Test.

Result 12.1.3 The series

an converges absolutely if
n

lim

an+1 < 1. an

If the limit is greater than unity, then the series diverges. If the limit is unity, the test fails.
If the limit is greater than unity, then the terms are eventually increasing with n. Since the terms do not vanish, the sum is divergent. If the limit is less than unity, then there exists some N such that an+1 r < 1 for all n N. an From this we can show that
n=0

an is absolutely convergent by comparing it to the geometric series.


|an | |aN |
n=N n=0

rn 1 1r

= |aN |

Example 12.1.6 Consider the series,

n=1

en . n!

531

We apply the ratio test to test for absolute convergence. en+1 n! an+1 = lim n n e (n + 1)! an e = lim n n + 1 =0

lim

The series is absolutely convergent.

Example 12.1.7 Consider the series,

n=1

1 , n2

which we know to be absolutely convergent. We apply the ratio test. an+1 1/(n + 1)2 = lim n an 1/n2 n2 = lim 2 n n + 2n + 1 1 = lim n 1 + 2/n + 1/n2 =1

lim

The test fails to predict the absolute convergence of the series.

532

The Root Test.

Result 12.1.4 The series

an converges absolutely if
n

lim |an |1/n < 1.

If the limit is greater than unity, then the series diverges. If the limit is unity, the test fails. More generally, we can test that lim sup |an |1/n < 1.
If the limit is greater than unity, then the terms in the series do not vanish as n . This implies that the sum does not converge. If the limit is less than unity, then there exists some N such that |an |1/n r < 1 for all n N. We bound the tail of the series of |an |.

|an | =
n=N n=N

|an |1/n rn
n=N N

=
n=0

r 1r

an is absolutely convergent.

Example 12.1.8 Consider the series

n a bn ,
n=0

533

where a and b are real constants. We use the root test to check for absolute convergence.
n

lim |na bn |1/n < 1 |b| lim na/n < 1


n

|b| exp

1 ln n n n 0 |b| e < 1 |b| < 1 lim

<1

Thus we see that the series converges absolutely for |b| < 1. Note that the value of a does not aect the absolute convergence. Example 12.1.9 Consider the absolutely convergent series,

n=1

1 . n2

We aply the root test. lim |an |


1/n

1 = lim 2 n n
n
2

1/n

= lim n2/n = lim e n ln n =e =1 It fails to predict the convergence of the series.


n 0

534

Raabes Test

Result 12.1.5 The series

an converges absolutely if
n

lim n 1

an+1 an

> 1.

If the limit is less than unity, then the series diverges or converges conditionally. If the limit is unity, the test fails.
Gauss Test

Result 12.1.6 Consider the series

an . If an+1 L bn =1 + 2 an n n

where bn is bounded then the series converges absolutely if L > 1. Otherwise the series diverges or converges conditionally.

12.2

Uniform Convergence

Continuous Functions. A function f (z) is continuous in a closed domain if, given any > 0, there exists a > 0 such that |f (z) f ()| < for all |z | < in the domain. An equivalent denition is that f (z) is continuous in a closed domain if
z

lim f () = f (z) 535

for all z in the domain. Convergence. Consider a series in which the terms are functions of z, an (z). The series is convergent in a n=0 domain if the series converges for each point z in the domain. We can then dene the function f (z) = an (z). n=0 We can state the convergence criterion as: For any given > 0 there exists a function N (z) such that
N (z)1

|f (z) SN (z) (z)| = f (z)


n=0

an (z) <

for all z in the domain. Note that the rate of convergence, i.e. the number of terms, N (z) required for for the absolute error to be less than , is a function of z. Uniform Convergence. Consider a series an (z) that is convergent in some domain. If the rate of convergence n=0 is independent of z then the series is said to be uniformly convergent. Stating this a little more mathematically, the series is uniformly convergent in the domain if for any given > 0 there exists an N , independent of z, such that
N

|f (z) SN (z)| = f (z)


n=1

an (z) <

for all z in the domain.

12.2.1

Tests for Uniform Convergence

Weierstrass M-test. The Weierstrass M-test is useful in determining if a series is uniformly convergent. The series n=0 an (z) is uniformly and absolutely convergent in a domain if there exists a convergent series of positive terms n=0 Mn such that |an (z)| Mn for all z in the domain. This condition rst implies that the series is absolutely convergent for all z in the domain. The condition |an (z)| Mn also ensures that the rate of convergence is independent of z, which is the criterion for uniform convergence. Note that absolute convergence and uniform convergence are independent. A series of functions may be absolutely convergent without being uniformly convergent or vice versa. The Weierstrass M-test is a sucient but not a necessary 536

condition for uniform convergence. The Weierstrass M-test can succeed only if the series is uniformly and absolutely convergent. Example 12.2.1 The series

f (x) =
n=1

sin x n(n + 1)
1 n2

sin x is uniformly and absolutely convergent for all real x because | n(n+1) | <

and

1 n=1 n2

converges.

Dirichlet Test. Consider a sequence of monotone decreasing, positive constants cn with limit zero. If all the partial sums of an (z) are bounded in some closed domain, that is
N

an (z) < constant


n=1

for all N , then cn an (z) is uniformly convergent in that closed domain. Note that the Dirichlet test does not n=1 imply that the series is absolutely convergent. Example 12.2.2 Consider the series,

n=1

sin(nx) . n

We cannot use the Weierstrass M-test to determine if the series is uniformly convergent on an interval. While it is easy to bound the terms with | sin(nx)/n| 1/n, the sum

n=1

1 n

537

does not converge. Thus we will try the Dirichlet test. Consider the sum in closed form. (See Exercise 12.9.)
N 1

N 1 n=1

sin(nx). This sum can be evaluated

sin(nx) =
n=1

0
cos(x/2)cos((N 1/2)x) 2 sin(x/2)

for x = 2k for x = 2k

The partial sums have innite discontinuities at x = 2k, k Z. The partial sums are bounded on any closed interval that does not contain an integer multiple of 2. By the Dirichlet test, the sum sin(nx) is uniformly convergent n=1 n on any such closed interval. The series may not be uniformly convergent in neighborhoods of x = 2k.

12.2.2

Uniform Convergence and Continuous Functions.

Consider a series f (z) = an (z) that is uniformly convergent in some domain and whose terms an (z) are continuous n=1 functions. Since the series is uniformly convergent, for any given > 0 there exists an N such that |RN | < for all z in the domain. Since the nite sum SN is continuous, for that there exists a > 0 such that |SN (z) SN ()| < for all in the domain satisfying |z | < . We combine these two results to show that f (z) is continuous. |f (z) f ()| = |SN (z) + RN (z) SN () RN ()| |SN (z) SN ()| + |RN (z)| + |RN ()| < 3 for |z | <

Result 12.2.1 A uniformly convergent series of continuous terms represents a continuous function.
Example 12.2.3 Again consider sin(nx) . In Example 12.2.2 we showed that the convergence is uniform in any n=1 n closed interval that does not contain an integer multiple of 2. In Figure 12.2 is a plot of the rst 10 and then 50 terms 538

in the series and nally the function to which the series converges. We see that the function has jump discontinuities at x = 2k and is continuous on any closed interval not containing one of those points.

Figure 12.2: Ten, Fifty and all the Terms of

sin(nx) . n=1 n

12.3

Uniformly Convergent Power Series

Power Series. Power series are series of the form an (z z0 )n .


n=0 n=0

Domain of Convergence of a Power Series Consider the series n point z0 . Then |an z0 | is bounded by some constant A for all n, so
n |an z n | = |an z0 |

an z n . Let the series converge at some

z z0

<A

z z0

This comparison test shows that the series converges absolutely for all z satisfying |z| < |z0 |. 539

Suppose that the series diverges at some point z1 . Then the series could not converge for any |z| > |z1 | since this would imply convergence at z1 . Thus there exists some circle in the z plane such that the power series converges absolutely inside the circle and diverges outside the circle.

Result 12.3.1 The domain of convergence of a power series is a circle in the complex plane.
Radius of Convergence of Power Series. Consider a power series

f (z) =
n=0

an z n

Applying the ratio test, we see that the series converges if |an+1 z n+1 | <l n |an z n | |an+1 | |z| < 1 lim n |an | |an | |z| < lim n |an+1 | lim

Result 12.3.2 Ratio formula. The radius of convergence of the power series

an z n
n=0

is

R = lim when the limit exists.

|an | n |an+1 |

540

Result 12.3.3 Cauchy-Hadamard formula. The radius of convergence of the power series:

an z n
n=0

is R=

1 lim sup
n

|an |

Absolute Convergence of Power Series. Consider a power series

f (z) =
n=0

an z n

n that converges for z = z0 . Let M be the value of the greatest term, an z0 . Consider any point z such that |z| < |z0 |. n We can bound the residual of n=0 |an z |,

RN (z) =
n=N

|an z n | an z n n |an z0 | n an z0

=
n=N

M
n=N

z z0

541

Since |z/z0 | < 1, this is a convergent geometric series. =M z 1 z0 1 |z/z0 | 0 as N


N

Thus the power series is absolutely convergent for |z| < |z0 |.

Result 12.3.4 If the power series absolutely for |z| < |z0 |.

n n=0 an z

converges for z = z0 , then the series converges

Example 12.3.1 Find the radii of convergence of the following series.

1.
n=1

nz n

2.
n=1

n!z n

3.
n=1

n!z n!

1. We apply the ratio test to determine the radius of convergence. R = lim The series converges absolutely for |z| < 1. 542 an n = lim =1 n n + 1 an+1

2. We apply the ratio test to the series. R = lim n! (n + 1)! 1 = lim n n + 1 =0


n

The series has a vanishing radius of convergence. It converges only for z = 0. 3. Again we apply the ration test to determine the radius of convergence. lim (n + 1)!z (n+1)! <1 n!z n!

n n

lim (n + 1)|z|(n+1)!n! < 1


n

lim (n + 1)|z|(n)n! < 1

lim (ln(n + 1) + (n)n! ln |z|) < 0 ln |z| < lim ln(n + 1) n (n)n! ln |z| < 0 |z| < 1

The series converges absolutely for |z| < 1. Alternatively we could determine the radius of convergence of the series with the comparison test.

n!z
n=1

n!

n=1

|nz n |

543

nz n has a radius of convergence of 1. Thus the series must have a radius of convergence of at least 1. Note that if |z| > 1 then the terms in the series do not vanish as n . Thus the series must diverge for all |z| 1. Again we see that the radius of convergence is 1. Uniform Convergence of Power Series. Consider a power series an z n that converges in the disk |z| < r0 . n=0 The sum converges absolutely for z in the closed disk, |z| r < r0 . Since |an z n | |an rn | and |an rn | converges, n=0 the power series is uniformly convergent in |z| r < r0 .

n=1

Result 12.3.5 If the power series uniformly for |z| r < r0 .

n n=0 an z

converges for |z| < r0 then the series converges

Example 12.3.2 Convergence and Uniform Convergence. Consider the series

log(1 z) =
n=1

zn . n

This series converges for |z| 1, z = 1. Is the series uniformly convergent in this domain? The residual after N terms RN is zn RN (z) = . n n=N +1 We can get a lower bound on the absolute value of the residual for real, positive z. |RN (x)| = xn n n=N +1

x d N +1 = Ei((N + 1) ln x) 544

The exponential integral function, Ei(z), is dened

Ei(z) =
z

et dt. t

The exponential integral function is plotted in Figure 12.3. Since Ei(z) diverges as z 0, by choosing x suciently close to 1 the residual can be made arbitrarily large. Thus this series is not uniformly convergent in the domain |z| 1, z = 1. The series is uniformly convergent for |z| r < 1.

-4

-3

-2

-1 -1 -2 -3

Figure 12.3: The Exponential Integral Function.

Analyticity. Recall that a sucient condition for the analyticity of a function f (z) in a domain is that C f (z) dz = 0 for all simple, closed contours in the domain. Consider a power series f (z) = an z n that is uniformly convergent in |z| r. If C is any simple, closed n=0 contour in the domain then C f (z) dz exists. Expanding f (z) into a nite series and a residual, f (z) dz =
C C

(SN (z) + RN (z)) dz. 545

Since the series is uniformly convergent, for any given > 0 there exists an N such that |RN | < for all z in |z| r. Let L be the length of the contour C. RN (z) dz L 0 as N
C N 1

f (z) dz = lim
C

an z n + RN (z)
C n=0

dz

=
C n=0

an z n an
n=0 C

= =0 Thus f (z) is analytic for |z| < r.

z n dz

Result 12.3.6 A power series is analytic in its domain of uniform convergence.

12.4

Integration and Dierentiation of Power Series

Consider a power series f (z) = an z n that is convergent in the disk |z| < r0 . Let C be any contour of nite n=0 length L lying entirely within the closed domain |z| r < r0 . The integral of f (z) along C is f (z) dz =
C C

(SN (z) + RN (z)) dz.

Since the series is uniformly convergent in the closed disk, for any given > 0, there exists an N such that |RN (z)| < for all |z| r. 546

We bound the absolute value of the integral of RN (z). RN (z) dz


C C

|RN (z)| dz

< L 0 as N Thus
N

f (z) dz = lim
C

an z n dz
C n=0 N

= lim

an
n=0 C

z n dz

=
n=0

an
C

z n dz

Result 12.4.1 If C is a contour lying in the domain of uniform convergence of the power series an z n then n=0

an z dz =
C n=0 n=0

an
C

z n dz.

In the domain of uniform convergence of a series we can interchange the order of summation and a limit process. That is,
zz0

lim

an (z) =
n=0 n=0

zz0

lim an (z).

We can do this because the rate of convergence does not depend on z. Since dierentiation is a limit process, d f (z + h) f (z) f (z) = lim , h0 dz h 547

we would expect that we could dierentiate a uniformly convergent series. Since we showed that a uniformly convergent power series is equal to an analytic function, we can dierentiate a power series in its domain of uniform convergence.

Result 12.4.2 Power series can be dierentiated in their domain of uniform convergence. d dz

an z =
n=0 n=0

(n + 1)an+1 z n .

Example 12.4.1 Dierentiating a Series. Consider the series from Example 12.3.2.

log(1 z) =
n=1

zn n

We dierentiate this to obtain the geometric series. 1 = z n1 1z n=1 1 = 1z


zn
n=0

The geometric series is convergent for |z| < 1 and uniformly convergent for |z| r < 1. Note that the domain of convergence is dierent than the series for log(1 z). The geometric series does not converge for |z| = 1, z = 1. However, the domain of uniform convergence has remained the same.

548

12.5

Taylor Series

Result 12.5.1 Taylors Theorem. Let f (z) be a function that is single-valued and analytic in |z z0 | < R. For all z in this open disk, f (z) has the convergent Taylor series

f (z) =
n=0

f (n) (z0 ) (z z0 )n . n!

(12.1)

We can also write this as

f (z) =
n=0

an (z z0 ) ,

f (n) (z0 ) 1 an = = n! 2

f (z) dz, (z z0 )n+1

(12.2)

where C is a simple, positive, closed contour in 0 < |z z0 | < R that goes once around the point z0 .

Proof of Taylors Theorem. Lets see why Result 12.5.1 is true. Consider a function f (z) that is analytic in |z| < R. (Considering z0 = 0 is only trivially more general as we can introduce the change of variables = z z0 .) According to Cauchys Integral Formula, (Result ??), 1 2 f () d, z

f (z) =

(12.3)

where C is a positive, simple, closed contour in 0 < | z| < R that goes once around z. We take this contour to be the circle about the origin of radius r where |z| < r < R. (See Figure 12.4.) 549

Im(z)

r C

R Re(z) z

Figure 12.4: Graph of Domain of Convergence and Contour of Integration.

We expand

1 z

in a geometric series, 1 1/ = z 1 z/ 1 = =
n=0

n=0 n

z ,

for |z| < ||

n+1

for |z| < ||

We substitute this series into Equation 12.3. 1 2


C

f (z) =

n=0

f ()z n n+1

550

The series converges uniformly so we can interchange integration and summation.

=
n=0

zn 2

f () d n+1

Now we have derived Equation 12.2. To obtain Equation 12.1, we apply Cauchys Integral Formula.

=
n=0

f (n) (0) n z n!

There is a table of some commonly encountered Taylor series in Appendix H. Example 12.5.1 Consider the Taylor series expansion of 1/(1 z) about z = 0. Previously, we showed that this function is the sum of the geometric series z n and we used the ratio test to show that the series converged n=0 absolutely for |z| < 1. Now we nd the series using Taylors theorem. Since the nearest singularity of the function is at z = 1, the radius of convergence of the series is 1. The coecients in the series are 1 dn 1 an = n! dz n 1 z z=0 1 n! = n! (1 z)n z=0 =1 Thus we have 1 = 1z

zn,
n=0

for |z| < 1.

551

12.5.1

Newtons Binomial Formula.

Result 12.5.2 For all |z| < 1, a complex: (1 + z)a = 1 + where a a 2 a 3 z+ z + z + 1 2 3

a r

a(a 1)(a 2) (a r + 1) . r!

If a is complex, then the expansion is of the principle branch of (1 + z)a . We dene r 0 = 1, 0 r = 0, for r = 0, 0 0 = 1.

Example 12.5.2 Evaluate limn (1 + 1/n)n . First we expand (1 + 1/n)n using Newtons binomial formula. lim 1 1+ n
n

n 1 n 1 n 1 + + + 2 n 1 n 2 n 3 n3 n(n 1) n(n 1)(n 2) = lim 1 + 1 + + + n 2!n2 3!n3 1 1 = 1 + 1 + + + 2! 3! = lim 1+

We recognize this as the Taylor series expansion of e1 . =e 552

We can also evaluate the limit using LHospitals rule. ln


x

lim

1 1+ x

1 = lim ln 1+ x x 1 = lim x ln 1 + x x ln(1 + 1/x) = lim x 1/x =


1/x2 1+1/x lim x 1/x2

=1 lim 1 1+ x
x

=e

Example 12.5.3 Find the Taylor series expansion of 1/(1 + z) about z = 0. For |z| < 1, 1 1 1 2 1 3 z + =1+ z+ z + 1+z 1 2 3 = 1 + (1)1 z + (1)2 z 2 + (1)3 z 3 + = 1 z + z2 z3 + Example 12.5.4 Find the rst few terms in the Taylor series expansion of about the origin. 553 1 z 2 + 5z + 6

We factor the denominator and then apply Newtons binomial formula. 1 1 1 = z+3 z+2 z 2 + 5z + 6 1 1 = 3 1 + z/3 2 1 + z/2 1/2 z 2 1 1/2 z + = 1+ + 1 3 2 3 6 z z2 1 z 3z 2 = 1 + + 1 + + 6 24 4 32 6 5 17 1 = 1 z + z2 + 12 96 6

1+

1/2 z 1/2 + 1 2 2

z 2

12.6

Laurent Series

Result 12.6.1 Let f (z) be single-valued and analytic in the annulus R1 < |z z0 | < R2 . For points in the annulus, the function has the convergent Laurent series

f (z) =
n=

an z n ,

where

1 f (z) dz 2 C (z z0 )n+1 and C is a positively oriented, closed contour around z0 lying in the annulus. an =
To derive this result, consider a function f () that is analytic in the annulus R1 < || < R2 . Consider any point z 554

in the annulus. Let C1 be a circle of radius r1 with R1 < r1 < |z|. Let C2 be a circle of radius r2 with |z| < r2 < R2 . Let Cz be a circle around z, lying entirely between C1 and C2 . (See Figure 12.5 for an illustration.) Consider the integral of points outside the annulus,
f () z

around the C2 contour. Since the the only singularities of f () d = z f () d + z f () d. z

f () z

occur at = z and at

C2

Cz

C1

By Cauchys Integral Formula, the integral around Cz is f () d = 2f (z). z

Cz

This gives us an expression for f (z). 1 2 f () 1 d z 2 f () d z

f (z) =

(12.4)

C2

C1

On the C2 contour, |z| < ||. Thus 1 1/ = z 1 z/ 1 = =


n=0

n=0 n

z ,

for |z| < ||

n+1

for |z| < ||

555

On the C1 contour, || < |z|. Thus 1 1/z = z 1 /z = =


n=0 1

1 z

n=0 n

for || < |z|

, z n+1 zn n+1 ,

for || < |z| for || < |z|

=
n=

We substitute these geometric series into Equation 12.4. 1 f (z) = 2


C2

n=0

f ()z n n+1

1 d + 2

C1

f ()z n n+1 n=

Since the sums converge uniformly, we can interchange the order of integration and summation. 1 f (z) = 2
C2

n=0

f ()z n 1 d + n+1 2

1 C1

n=

f ()z n d n+1

Since the only singularities of the integrands lie outside of the annulus, the C1 and C2 contours can be deformed to any positive, closed contour C that lies in the annulus and encloses the origin. (See Figure 12.5.) Finally, we combine the two integrals to obtain the desired result. f (z) = 1 2 n=

f () d z n n+1

For the case of arbitrary z0 , simply make the transformation z z z0 . 556

Im(z)

Im(z)

r2 r1 C1 C2

R2 R1 z Cz

R2 R1

Re(z)

Re(z) C

Figure 12.5: Contours for a Laurent Expansion in an Annulus.

Example 12.6.1 Find the Laurent series expansions of 1/(1 + z). For |z| < 1,

1 1 1 2 1 3 =1+ z+ z + z + 1+z 1 2 3 = 1 + (1)1 z + (1)2 z 2 + (1)3 z 3 + = 1 z + z2 z3 + 557

For |z| > 1, 1 1/z = 1+z 1 + 1/z 1 1 1 2 1 = 1+ z + z + z 1 2 = z 1 z 2 + z 3

558

12.7
12.7.1

Exercises
Series of Constants

Exercise 12.1 Show that if an converges then limn an = 0. That is, limn an = 0 is a necessary condition for the convergence of the series. Hint, Solution Exercise 12.2 Answer the following questions true or false. Justify your answers. 1. There exists a sequence which converges to both 1 and 1. 2. There exists a sequence {an } such that an > 1 for all n and limn an = 1. 3. There exists a divergent geometric series whose terms converge. 4. There exists a sequence whose even terms are greater than 1, whose odd terms are less than 1 and that converges to 1. 5. There exists a divergent series of non-negative terms,
n=0

an , such that an < (1/2)n .

6. There exists a convergent sequence, {an }, such that limn (an+1 an ) = 0. 7. There exists a divergent sequence, {an }, such that limn |an | = 2. 8. There exists divergent series, an and bn , such that (an + bn ) is convergent.

9. There exists 2 dierent series of nonzero terms that have the same sum. 10. There exists a series of nonzero terms that converges to zero. 11. There exists a series with an innite number of non-real terms which converges to a real number. 559

12. There exists a convergent series 13. There exists a divergent series 14. There exists a convergent series 15. There exists a divergent series

an with limn |an+1 /an | = 1. an with limn |an+1 /an | = 1. an with limn an with limn
n n

|an | = 1.

|an | = 1. an , for which an , for which a2 diverges. n an diverges.

16. There exists a convergent series of non-negative terms, 17. There exists a convergent series of non-negative terms, 18. There exists a convergent series, 19. There exists a power series 20. There exists a power series Hint, Solution Exercise 12.3 Determine if the following series converge.

an , for which

|an | diverges.

an (z z0 )n which converges for z = 0 and z = 3 but diverges for z = 2. an (z z0 )n which converges for z = 0 and z = 2 but diverges for z = 2.

1.
n=2

1 n ln(n) 1 ln (nn ) ln n ln n

2.
n=2

3.
n=2

560

4.

1 n(ln n)(ln(ln n)) n=10

5.
n=1

ln (2n ) ln (3n ) + 1 1 ln(n + 20) 4n + 1 3n 2 (Log 2)n

6.
n=0

7.
n=0

8.
n=0

9.
n=2

n2 1 n4 1 n2 (ln n)n (1)n ln 1 n

10.
n=2

11.
n=2

12.
n=2

(n!)2 (2n)! 3n + 4n + 5 5n 4n 3 561

13.
n=2

14.
n=2

n! (ln n)n en ln(n!) (n!)2 (n2 )! n8 + 4n4 + 8 3n9 n5 + 9n 1 1 n n+1 cos(n) n ln n n11/10

15.
n=2

16.
n=1

17.
n=1

18.
n=1

19.
n=1

20.
n=2

Hint, Solution Exercise 12.4 (mathematica/fcv/series/constants.nb) Show that the alternating harmonic series,

n=1

1 1 1 (1)n+1 = 1 + + , n 2 3 4

is convergent. Hint, Solution 562

Exercise 12.5 (mathematica/fcv/series/constants.nb) Show that the series

n=1

1 n

is divergent with the Cauchy convergence criterion. Hint, Solution Exercise 12.6 The alternating harmonic series has the sum:

n=1

(1)n = ln(2). n

Show that the terms in this series can be rearranged to sum to . Hint, Solution Exercise 12.7 (mathematica/fcv/series/constants.nb) Is the series, n! , nn n=1 convergent? Hint, Solution Exercise 12.8 Show that the harmonic series,

n=1

1 1 1 = 1 + + + , n 2 3

converges for > 1 and diverges for 1. Hint, Solution 563

Exercise 12.9 Evaluate N 1 sin(nx). n=1 Hint, Solution Exercise 12.10 Evaluate
n n

kz
k=1

and
k=1

k2z k

for z = 1. Hint, Solution Exercise 12.11 Which of the following series converge? Find the sum of those that do. 1. 1 1 1 1 + + + + 2 6 12 20

2. 1 + (1) + 1 + (1) +

3.
n=1

1 2n1

1 1 3n 5n+1

Hint, Solution Exercise 12.12 Evaluate the following sum.


k1 =0 k2 =k1 kn =kn1

1 2kn

Hint, Solution 564

12.7.2 12.7.3

Uniform Convergence Uniformly Convergent Power Series

Exercise 12.13 Determine the domain of convergence of the following series.

1.
n=0

zn (z + 3)n Log z ln n z n (z + 2)2 n2 (z e)n nn z 2n 2nz z n! (n!)2 z ln(n!) n! 565

2.
n=2

3.
n=1

4.
n=1

5.
n=1

6.
n=1

7.
n=0

8.
n=0

9.
n=0

(z )2n+1 n n! ln n zn

10.
n=0

Hint, Solution Exercise 12.14 Find the circle of convergence of the following series. 1. z + ( )

z3 z4 z2 + ( )( 2) + ( )( 2)( 3) + 2! 3! 4!

2.
n=1

n (z )n 2n nn z n

3.
n=1

4.
n=1

n! n z nn (3 + (1)n )n z n

5.
n=1

6.
n=1

(n + n ) z n

(|| > 1)

Hint, Solution 566

Exercise 12.15 Find the circle of convergence of the following series:

1.
k=0

kz k

2.
k=1

kk z k k! k z kk (z + 5)2k (k + 1)2
k=0

3.
k=1

4.

5.
k=0

(k + 2k )z k

Hint, Solution

12.7.4

Integration and Dierentiation of Power Series

Exercise 12.16 Using the geometric series, show that 1 = (1 z)2 and log(1 z) =
n=1

(n + 1)z n ,
n=0

for |z| < 1,

zn , n

for |z| < 1.

567

Hint, Solution

12.7.5

Taylor Series

Exercise 12.17 1 Find the Taylor series of 1+z2 about the z = 0. Determine the radius of convergence of the Taylor series from the singularities of the function. Determine the radius of convergence with the ratio test. Hint, Solution Exercise 12.18 Use two methods to nd the Taylor series expansion of log(1 + z) about z = 0 and determine the circle of convergence. First directly apply Taylors theorem, then dierentiate a geometric series. Hint, Solution Exercise 12.19 Let f (z) = (1 + z) be the branch for which f (0) = 1. Find its Taylor series expansion about z = 0. What is the radius of convergence of the series? ( is an arbitrary complex number.) Hint, Solution Exercise 12.20 Find the Taylor series expansions about the point z = 1 for the following functions. What are the radii of convergence? 1. 1 z

2. Log z 3. 1 z2

4. z Log z z Hint, Solution 568

Exercise 12.21 Find the Taylor series expansion about the point z = 0 for ez . What is the radius of convergence? Use this to nd the Taylor series expansions of cos z and sin z about z = 0. Hint, Solution Exercise 12.22 Find the Taylor series expansion about the point z = for the cosine and sine. Hint, Solution Exercise 12.23 Sum the following series.

1.
n=0

(ln 2)n n! (n + 1)(n + 2) 2n (1)n n! (1)n 2n+1 (2n + 1)! (1)n 2n (2n)! ()n (2n)!

2.
n=0

3.
n=0

4.
n=0

5.
n=0

6.
n=0

Hint, Solution 569

Exercise 12.24 1. Find the rst three terms in the following Taylor series and state the convergence properties for the following. (a) ez around z0 = 0 (b) 1+z around z0 = 1z ez (c) around z0 = 0 z1

It may be convenient to use the Cauchy product of two Taylor series. 2. Consider a function f (z) analytic for |z z0 | < R. Show that the series obtained by dierentiating the Taylor series for f (z) termwise is actually the Taylor series for f (z) and hence argue that this series converges uniformly to f (z) for |z z0 | < R. 3. Find the Taylor series for 1 (1 z)3 by appropriate dierentiation of the geometric series and state the radius of convergence. 4. Consider the branch of f (z) = (z + 1) corresponding to f (0) = 1. Find the Taylor series expansion about z0 = 0 and state the radius of convergence. Hint, Solution

12.7.6

Laurent Series

Exercise 12.25 Find the Laurent series about z = 0 of 1/(z ) for |z| < 1 and |z| > 1. Hint, Solution 570

Exercise 12.26 Obtain the Laurent expansion of f (z) = centered on z = 0 for the three regions: 1. |z| < 1 2. 1 < |z| < 2 3. 2 < |z| Hint, Solution Exercise 12.27 By comparing the Laurent expansion of (z + 1/z)m , m Z+ , with the binomial expansion of this quantity, show that
2

1 (z + 1)(z + 2)

(cos )m cos(n) d =
0

m 2m1 (mn)/2

m n m and m n even otherwise

Hint, Solution Exercise 12.28 The function f (z) is analytic in the entire z-plane, including , except at the point z = /2, where it has a simple pole, and at z = 2, where it has a pole of order 2. In addition f (z) dz = 2,
|z|=1 |z|=3

f (z) dz = 0,
|z|=3

(z 1)f (z) dz = 0.

Find f (z) and its complete Laurent expansion about z = 0. Hint, Solution 571

Exercise 12.29 k Let f (z) = k 3 z . Compute each of the following, giving justication in each case. The contours are circles of k=1 3 radius one about the origin. 1.
|z|=1

ez f (z) dz f (z) dz z4 f (z) ez dz z2

2.
|z|=1

3.
|z|=1

Hint, Solution Exercise 12.30 1. Expand f (z) =


1 z(1z)

in Laurent series that converge in the following domains:

(a) 0 < |z| < 1 (b) |z| > 1 (c) |z + 1| > 2 2. Without determining the series, specify the region of convergence for a Laurent series representing f (z) = 1/(z 4 + 4) in powers of z 1 that converges at z = . Hint, Solution

572

12.8

Hints

Hint 12.1 Use the Cauchy convergence criterion for series. In particular, consider |SN +1 SN |. Hint 12.2 CONTINUE Hint 12.3 1.

n=2

1 n ln(n)

Use the integral test. 2.

n=2

1 ln (nn )

Simplify the summand. 3.

ln
n=2

ln n

Simplify the summand. Use the comparison test. 4.

1 n(ln n)(ln(ln n)) n=10 Use the integral test. 573

5.

n=1

ln (2n ) ln (3n ) + 1

Show that the terms in the sum do not vanish as n 6.

n=0

1 ln(n + 20)

Shift the indices. 7.

n=0

4n + 1 3n 2

Show that the terms in the sum do not vanish as n 8.

(Log 2)n
n=0

This is a geometric series. 9.

n=2

n2 1 n4 1

Simplify the integrand. Use the comparison test. 10.

n=2

n2 (ln n)n

Compare to a geometric series. 574

11.

(1)n ln
n=2

1 n

Group pairs of consecutive terms to obtain a series of positive terms. 12.

n=2

(n!)2 (2n)!

Use the comparison test. 13.

n=2

3n + 4n + 5 5n 4n 3

Use the root test. 14.

n=2

n! (ln n)n

Show that the terms do not vanish as n . 15.

n=2

en ln(n!)

Show that the terms do not vanish as n . 16.

n=1

(n!)2 (n2 )!

Apply the ratio test. 575

17.

n=1

n8 + 4n4 + 8 3n9 n5 + 9n

Use the comparison test. 18.

n=1

1 1 n n+1

Use the comparison test. 19.

n=1

cos(n) n

Simplify the integrand. 20.

n=2

ln n n11/10

Use the integral test. Hint 12.4 Group the terms. 1 1 1 = 2 2 1 1 1 = 3 4 12 1 1 1 = 5 6 30 576

Hint 12.5 Show that 1 |S2n Sn | > . 2 Hint 12.6 The alternating harmonic series is conditionally convergent. Let {an } and {bn } be the positive and negative terms in the sum, respectively, ordered in decreasing magnitude. Note that both an and bn are divergent. Devise a n=1 n=1 method for alternately taking terms from {an } and {bn }. Hint 12.7 Use the ratio test. Hint 12.8 Use the integral test. Hint 12.9 Note that sin(nx) = (enx ). This substitute will yield a nite geometric series. Hint 12.10 Let Sn be the sum. Consider Sn zSn . Use the nite geometric sum. Hint 12.11 1. The summand is a rational function. Find the rst few partial sums. 2. 3. This a geometric series. Hint 12.12 CONTINUE

577

Hint 12.13 CONTINUE

1.
n=0

zn (z + 3)n Log z ln n z n (z + 2)2 n2 (z e)n nn z 2n 2nz z n! (n!)2 z ln(n!) n! (z )2n+1 n n! 578

2.
n=2

3.
n=1

4.
n=1

5.
n=1

6.
n=1

7.
n=0

8.
n=0

9.
n=0

10.
n=0

ln n zn

Hint 12.14 Hint 12.15 CONTINUE Hint 12.16 Dierentiate the geometric series. Integrate the geometric series. Hint 12.17 The Taylor series is a geometric series. Hint 12.18 Hint 12.19 Hint 12.20 1. 1 1 = z 1 + (z 1) The right side is the sum of a geometric series. 2. Integrate the series for 1/z. 3. Dierentiate the series for 1/z. 4. Integrate the series for Log z. 579

Hint 12.21 Evaluate the derivatives of ez at z = 0. Use Taylors Theorem. Write the cosine and sine in terms of the exponential function. Hint 12.22 cos z = cos(z ) sin z = sin(z ) Hint 12.23 CONTINUE Hint 12.24 CONTINUE Hint 12.25 Hint 12.26 Hint 12.27 Hint 12.28 Hint 12.29 Hint 12.30 CONTINUE 580

12.9

Solutions

Solution 12.1 n=0 an converges only if the partial sums, Sn , are a Cauchy sequence. > 0 N s.t. m, n > N |Sm Sn | < , In particular, we can consider m = n + 1. > 0 N s.t. n > N |Sn+1 Sn | < Now we note that Sn+1 sn = an . > 0 N s.t. n > N |an | < This is exactly the Cauchy convergence criterion for the sequence {an }. Thus we see that limn an = 0 is a necessary condition for the convergence of the series an . n=0 Solution 12.2 CONTINUE Solution 12.3 1.

n=2

1 n ln(n)
ln 2

Since this is a series of positive, monotone decreasing terms, the sum converges or diverges with the integral,
2

1 dx = x ln x

1 d

Since the integral diverges, the series also diverges. 2.

n=2

1 = ln (nn ) 581

n=2

1 n ln(n)

The sum converges. 3.

ln
n=2

ln n =
n=2

1 ln(ln n) n

n=2

1 n

The sum is divergent by the comparison test. 4. 1 n(ln n)(ln(ln n)) n=10 Since this is a series of positive, monotone decreasing terms, the sum converges or diverges with the integral,
10

1 dx = x ln x ln(ln x)

ln(10)

1 dy = y ln y

ln(ln(10))

1 dz z

Since the integral diverges, the series also diverges. 5.

n=1

ln (2n ) = ln (3n ) + 1

n=1

n ln 2 = n ln 3 + 1

n=1

ln 2 ln 3 + 1/n

Since the terms in the sum do not vanish as n , the series is divergent. 6.

n=0

1 1 = ln(n + 20) n=20 ln n

The series diverges. 7.

n=0

4n + 1 3n 2

Since the terms in the sum do not vanish as n , the series is divergent. 582

8.

(Log 2)n
n=0

This is a geometric series. Since | Log 2| < 1, the series converges. 9.

n=2

n2 1 = n4 1

n=2

1 < 2+1 n

n=2

1 n2

The series converges by comparison to the harmonic series. 10.

n=2

n2 = (ln n)n

n=2

n2/n ln n

Since n2/n 1 as n , n2/n / ln n 0 as n . The series converges by comparison to a geometric series. 11. We group pairs of consecutive terms to obtain a series of positive terms.

(1) ln
n=2

1 n

=
n=1

ln

1 2n

ln

1 2n + 1

=
n=1

ln

2n + 1 2n

The series on the right side diverges because the terms do not vanish as n . 12.

n=2

(n!)2 = (2n)!

n=2

(1)(2) n < (n + 1)(n + 2) (2n)

n=2

1 2n

The series converges by comparison with a geometric series. 583

13.

n=2

3n + 4n + 5 5n 4n 3

We use the root test to check for convergence. lim |an |1/n = lim = lim 3n + 4n + 5 5n 4n 3
1/n

4 (3/4)n + 1 + 5/4n n 5 1 (4/5)n 3/5n 4 = 5 <1 We see that the series is absolutely convergent. 14. We will use the comparison test.

1/n

n=2

n! > (ln n)n

n=2

(n/2)n/2 = (ln n)n

n=2

n/2 ln n

Since the terms in the series on the right side do not vanish as n , the series is divergent. 15. We will use the comparison test.

n=2

en > ln(n!)

n=2

en = ln(nn )

n=2

en n ln(n)

Since the terms in the series on the right side do not vanish as n , the series is divergent. 584

16.

n=1

(n!)2 (n2 )!

We apply the ratio test. lim ((n + 1)!)2 (n2 )! an+1 = lim n ((n + 1)2 )!(n!)2 an (n + 1)2 = lim n ((n + 1)2 n2 )! (n + 1)2 = lim n (2n + 1)! =0

The series is convergent. 17.

n=1

n8 + 4n4 + 8 = 3n9 n5 + 9n >

n=1

1 1 + 4n4 + 8n8 n 3 n4 + 9n8 1 n

1 4

n=1

We see that the series is divergent by comparison to the harmonic series. 18.

n=1

1 1 n n+1

=
n=1

1 < 2+n n

n=1

1 n2

The series converges by the comparison test. 585

19.

n=1

cos(n) = n

n=1

(1)n n

We recognize this as the alternating harmonic series, which is conditionally convergent. 20.

n=2

ln n n11/10

Since this is a series of positive, monotone decreasing terms, the sum converges or diverges with the integral,
2

ln x dx = x11/10

y ey/10 dy
ln 2

Since the integral is convergent, so is the series. Solution 12.4

n=1

(1)n+1 = n =

n=1

1 1 2n 1 2n 1 (2n 1)(2n) 1 (2n 1)2 1 n2

n=1

< < = Thus the series is convergent. 1 2

n=1

12

n=1 2

586

Solution 12.5 Since


2n1

|S2n Sn | =
j=n 2n1

1 j

j=n

1 2n 1

n 2n 1 1 > 2 = the series does not satisfy the Cauchy convergence criterion. Solution 12.6 The alternating harmonic series is conditionally convergent. That is, the sum is convergent but not absolutely convergent. Let {an } and {bn } be the positive and negative terms in the sum, respectively, ordered in decreasing magnitude. Note that both an and bn are divergent. Otherwise the alternating harmonic series would be absolutely n=1 n=1 convergent. To sum the terms in the series to we repeat the following two steps indenitely: 1. Take terms from {an } until the sum is greater than . 2. Take terms from {bn } until the sum is less than . Each of these steps can always be accomplished because the sums, an and bn are both divergent. Hence the n=1 n=1 tails of the series are divergent. No matter how many terms we take, the remaining terms in each series are divergent. In each step a nite, nonzero number of terms from the respective series is taken. Thus all the terms will be used. Since the terms in each series vanish as n , the running sum converges to .

587

Solution 12.7 Applying the ratio test, an+1 (n + 1)!nn lim = lim n n n!(n + 1)(n+1) an nn = lim n (n + 1)n n n = lim n (n + 1) 1 = e < 1, we see that the series is absolutely convergent. Solution 12.8 The harmonic series,

n=1

1 1 1 = 1 + + + , n 2 3

converges or diverges absolutely with the integral,


1

1 dx = |x |

1 x
()

dx =

[ln x] 1
x1 () 1 ()

for

() = 1, () = 1. () > 1 and diverges

for
1

The integral converges only for absolutely for () 1.

() > 1. Thus the harmonic series converges absolutely for

588

Solution 12.9

N 1

N 1

sin(nx) =
n=1 n=0 N 1

sin(nx) (enx )
n=0 N 1

=
n=0

(ex )n (N )
1enx 1ex

= 0

for x = 2k for x = 2k for x = 2k

ex/2 e(N 1/2)x ex/2 ex/2

for x = 2k for x = 2k

0
ex/2 e(N 1/2)x 2 sin(x/2)

for x = 2k for x = 2k

0
ex/2 e(N 1/2)x 2 sin(x/2)

for x = 2k

N 1

sin(nx) =
n=1

0
cos(x/2)cos((N 1/2)x) 2 sin(x/2)

for x = 2k for x = 2k

589

Solution 12.10 Let


n

Sn =
k=1 n

kz k .
n

Sn zSn =
k=1 n

kz
k=1 n+1

kz k+1 (k 1)z k
k=2

=
k=1 n

kz k

=
k=1

z k nz n+1 z z n+1 nz n+1 1z

=
n

kz k =
k=1

z(1 (n + 1)z n + nz n+1 ) (1 z)2


n

Let Sn =

k2z k .
k=1

Sn zSn = =2

(k 2 (k 1)2 )z k n2 z n+1
k=1 n n

kz
k=1 k=1

z k n2 z n+1

z(1 (n + 1)z n + nz n+1 ) z z n+1 =2 n2 z n+1 (1 z)2 1z 590

k2z k =
k=1

z(1 + z z n (1 + z + n(n(z 1) 2)(z 1))) (1 z)3

Solution 12.11 1.

an =
n=1

1 1 1 1 + + + + 2 6 12 20

We conjecture that the terms in the sum are rational functions of summation index. That is, an = 1/p(n) where p(n) is a polynomial. We use divided dierences to determine the order of the polynomial. 2 4 2 6 6 2 12 8 20

We see that the polynomial is second order. p(n) = an2 + bn + c. We solve for the coecients. a+b+c=2 4a + 2b + c = 6 9a + 3b + c = 12 p(n) = n2 + n We examine the rst few partial sums. S1 = 1 2 2 S2 = 3 3 S3 = 4 4 S4 = 5

591

We conjecture that Sn = n/(n+1). We prove this with induction. The base case is n = 1. S1 = 1/(1+1) = 1/2. Now we assume the induction hypothesis and calculate Sn+1 . Sn+1 = Sn + an+1 n 1 = + 2 + (n + 1) n + 1 (n + 1) n+1 = n+2 This proves the induction hypothesis. We calculate the limit of the partial sums to evaluate the series.

n=1

n2

1 n = lim n n + 1 +n 1 =1 +n

n=1

n2

2.

(1)n = 1 + (1) + 1 + (1) +


n=0

Since the terms in the series do not vanish as n , the series is divergent. 3. We can directly sum this geometric series.

n=1

1 1 1 1 2 = = 2n1 3n 5n+1 75 1 1/30 145 1

CONTINUE 592

Solution 12.12 The innermost sum is a geometric series.

kn =kn1

1 1 1 = kn1 = 21kn1 2kn 2 1 1/2

This gives us a relationship between n nested sums and n 1 nested sums.


k1 =0 k2 =k1 kn =kn1

1 =2 2kn k

1 =0

1 2kn1

k2 =k1

kn1 =kn2

We evaluate the n nested sums by induction.


k1 =0 k2 =k1 kn =kn1

1 = 2n 2kn

Solution 12.13 CONTINUE.

1.
n=0

zn (z + 3)n Log z ln n z n (z + 2)2 n2 593

2.
n=2

3.
n=1

4.
n=1

5.
n=1

(z e)n nn z 2n 2nz z n! (n!)2 z ln(n!) n! (z )2n+1 n n! ln n zn

6.
n=1

7.
n=0

8.
n=0

9.
n=0

10.
n=0

Solution 12.14 1. We assume that = 0. We determine the radius of convergence with the ratio test. R = lim an n an+1 ( ) ( (n 1))/n! = lim n ( ) ( n)/(n + 1)! n+1 = lim n n 1 = || 594

The series converges absolutely for |z| < 1/||. 2. By the ratio test formula, the radius of absolute convergence is R = lim n/2n n (n + 1)/2n+1 n = 2 lim n n + 1 =2

By the root test formula, the radius of absolute convergence is R= 1 limn n |n/2n | 2 = limn n n =2

The series converges absolutely for |z | < 2. 3. We determine the radius of convergence with the Cauchy-Hadamard formula. R= = 1 lim sup n |an | 1

lim sup n |nn | 1 = lim sup n =0 The series converges only for z = 0. 595

4. By the ratio test formula, the radius of absolute convergence is R = lim = lim
n

= lim = exp = exp = exp = exp = exp = e1

n!/nn (n + 1)!/(n + 1)n+1 (n + 1)n nn n n+1 n n n+1 lim ln n n n+1 lim n ln n n ln(n + 1) ln(n) lim n 1/n 1/(n + 1) 1/n lim n 1/n2 n lim n n + 1

The series converges absolutely in the circle, |z| < e. 5. By the Cauchy-Hadamard formula, the radius of absolute convergence is 1 R= n lim sup | (3 + (1)n )n | 1 = lim sup (3 + (1)n ) 1 = 4 596

Thus the series converges absolutely for |z| < 1/4. 6. By the Cauchy-Hadamard formula, the radius of absolute convergence is R= = 1 lim sup
n

|n + n | 1

lim sup || n |1 + n/n | 1 = || Thus the sum converges absolutely for |z| < 1/||. Solution 12.15 1.

kz k
k=0

We determine the radius of convergence with the ratio formula. R = lim k k k + 1 1 = lim k 1 =1

The series converges absolutely for |z| < 1. 2.

kk z k
k=1

597

We determine the radius of convergence with the Cauchy-Hadamard formula. R= 1 lim sup k |k k | 1 = lim sup k =0

The series converges only for z = 0. 3.

k=1

k! k z kk

We determine the radius of convergence with the ratio formula. R = lim k!/k k k (k + 1)!/(k + 1)(k+1) (k + 1)k = lim k kk k+1 = exp lim k ln k k ln(k + 1) ln(k) = exp lim k 1/k 1/(k + 1) 1/k = exp lim k 1/k 2 k = exp lim k k + 1 = exp(1) =e 598

The series converges absolutely for |z| < e. 4.

(z + 5)2k (k + 1)2
k=0

We use the ratio formula to determine the domain of convergence. (z + 5)2(k+1) (k + 2)2 <1 k (z + 5)2k (k + 1)2 (k + 2)2 |z + 5|2 lim <1 k (k + 1)2 2(k + 2) |z + 5|2 lim <1 k 2(k + 1) 2 |z + 5|2 lim < 1 k 2 |z + 5|2 < 1 lim 5.

(k + 2k )z k
k=0

We determine the radius of convergence with the Cauchy-Hadamard formula. R= = 1 lim sup
k

|k + 2k | 1

lim sup 2 k |1 + k/2k | 1 = 2

The series converges for |z| < 1/2. 599

Solution 12.16 The geometric series is 1 = 1z

zn.
n=0

This series is uniformly convergent in the domain, |z| r < 1. Dierentiating this equation yields, 1 = (1 z)2 =
n=0

nz n1
n=1

(n + 1)z n

for |z| < 1.

Integrating the geometric series yields

log(1 z) =
n=0

z n+1 n+1 for |z| < 1.

log(1 z) =
n=1

z , n

Solution 12.17 1 = 1 + z2 The function


1 1+z 2

z
n=0

2 n

=
n=0

(1)n z 2n

1 (1z)(1+z)

has singularities at z = . Thus the radius of convergence is 1. Now we use the ratio 600

test to corroborate that the radius of convergence is 1. an+1 (z) <1 n an (z) (1)n+1 z 2(n+1) lim <1 n (1)n z 2n lim z 2 < 1 lim
n

|z| < 1 Solution 12.18 Method 1. log(1 + z) = [log(1 + z)]z=0 + d z d2 z2 log(1 + z) + log(1 + z) + dz dz 2 z=0 1! z=0 2! z 1 z2 2 z3 1 =0+ + + + 1 + z z=0 1! (1 + z)2 z=0 2! (1 + z)3 z=0 3! z2 z3 z4 =z + + 2 3 4 n n+1 z = (1) n n=1

Since the nearest singularity of log(1 + z) is at z = 1, the radius of convergence is 1. Method 2. We know the geometric series converges for |z| < 1. 1 = 1+z

(1)n z n
n=0

We integrate this equation to get the series for log(1 + z) in the domain |z| < 1. z n+1 = log(1 + z) = (1) n+1 n=0
n

(1)n+1
n=1

zn n

601

We calculate the radius of convergence with the ratio test. R = lim an (n + 1) = lim =1 n an+1 n

Thus the series converges absolutely for |z| < 1. Solution 12.19 The Taylor series expansion of f (z) about z = 0 is

f (z) =
n=0

f (n) (0) n z . n!

The derivatives of f (z) are


n1

f Thus f (n) (0) is

(n)

(z) =
k=0

( k) (1 + z)n .

n1

(n)

(0) =
k=0

( k).

If = m is a non-negative integer, then only the rst m + 1 terms are nonzero. The Taylor series is a polynomial and the series has an innite radius of convergence.
m

(1 + z) =
n=0

n1 k=0 (

k)

n!

zn

If is not a non-negative integer, then all of the terms in the series are non-zero.

(1 + z) =
n=0

n1 k=0 (

k)

n!

zn

602

The radius of convergence of the series is the distance to the nearest singularity of (1 + z) . This occurs at z = 1. Thus the series converges for |z| < 1. We can corroborate this with the ratio test. The radius of convergence is R = lim
n1 k=0 ( n k=0 (

k) /n! n+1 = lim = 1. n n k)) /(n + 1)!

If we use the binomial coecient, we can write the series in a compact form. n
n1 k=0 (

k) n z n

n!

(1 + z) =
n=0

Solution 12.20 1. We nd the series for 1/z by writing it in terms of z 1 and using the geometric series. 1 1 = z 1 + (z 1) 1 = z

(1)n (z 1)n
n=0

for |z 1| < 1

Since the nearest singularity is at z = 0, the radius of convergence is 1. The series converges absolutely for |z 1| < 1. We could also determine the radius of convergence with the Cauchy-Hadamard formula. R= = 1 lim sup
n

|an | |(1)n |

1
n

lim sup =1 603

2. We integrate 1/ from 1 to z for in the circle |z 1| < 1.


z 1

1 d = [Log ]z = Log z 1

The series we derived for 1/z is uniformly convergent for |z 1| r < 1. We can integrate the series in this domain.
z

Log z =
1 n=0

(1)n ( 1)n d
z

=
n=0

(1)

n 1

( 1)n d (z 1)n+1 n+1

=
n=0

(1)n

Log z =
n=1

(1)n1 (z 1)n n

for |z 1| < 1

3. The series we derived for 1/z is uniformly convergent for |z 1| r < 1. We can dierentiate the series in this domain. 1 d 1 = 2 z dz z d = (1)n (z 1)n dz n=0

=
n=1

(1)n+1 n(z 1)n1

604

1 = z2

(1)n (n + 1)(z 1)n


n=0

for |z 1| < 1

4. We integrate Log from 1 to z for in the circle |z 1| < 1.


z

Log d = [ Log ]z = z Log z z + 1 1


1

The series we derived for Log z is uniformly convergent for |z 1| r < 1. We can integrate the series in this domain.
z

z Log z z = = 1 +
1

Log d
z

= 1 +
1 n=1

(1)n1 ( 1)n d n

= 1 +
n=1

(1)n1 (z 1)n+1 n(n + 1) for |z 1| < 1

z Log z z = 1 +
n=2

(1)n (z 1)n n(n 1)

Solution 12.21 We evaluate the derivatives of ez at z = 0. Then we use Taylors Theorem. dn z e = ez dz n dn z e = ez n dz

=1
z=0

ez =
n=0

zn n!

605

Since the exponential function has no singularities in the nite complex plane, the radius of convergence is innite. We nd the Taylor series for the cosine and sine by writing them in terms of the exponential function. cos z = ez + ez 2 1 (z)n (z)n = + 2 n=0 n! n! n=0

=
n=0 even n

(z)n n!

cos z =
n=0

(1)n z 2n (2n)!

sin z =

ez ez 2 (z)n (z)n 1 = 2 n=0 n! n! n=0

=
n=0 odd n

(z)n n!

sin z =
n=0

(1)n z 2n+1 (2n + 1)!

606

Solution 12.22 cos z = cos(z )

=
n=0

(1)n (z )2n (2n)!

=
n=0

(1)n+1 (z )2n (2n)!

sin z = sin(z )

=
n=0

(1)n (z )2n+1 (2n + 1)!

=
n=0

(1)n+1 (z )2n+1 (2n + 1)!

Solution 12.23 CONTINUE Solution 12.24 1. (a) f (z) = ez f (0) = 1 f (0) = 1 f (0) = 1 z2 + O z3 2 is entire, the Taylor series converges in the complex plane. ez = 1 z + 607

Since ez

(b) 1+z , f () = 1z 2 f (z) = , f () = (1 z)2 4 f (z) = , f () = 1 + (1 z)3 f (z) = 1+z 1 + = + (z ) + (z )2 + O (z )3 1z 2 Since the nearest singularity, (at z = 1), is a distance of 2 from z0 = , the radius of convergence is 2. The series converges absolutely for |z | < 2. (c) ez z2 = 1+z+ + O z3 z1 2 5 2 = 1 2z z + O z 3 2 1 + z + z2 + O z3

Since the nearest singularity, (at z = 1), is a distance of 1 from z0 = 0, the radius of convergence is 1. The series converges absolutely for |z| < 1. 2. Since f (z) is analytic in |z z0 | < R, its Taylor series converges absolutely on this domain.

f (z) =
n=0

f (n) (z0 )z n n!

The Taylor series converges uniformly on any closed sub-domain of |z z0 | < R. We consider the sub-domain 608

|z z0 | < R. On the domain of uniform convergence we can interchange dierentiation and summation. d f (z) = dz

n=0

f (n) (z0 )z n n!

f (z) =
n=1

nf (n) (z0 )z n1 n! f (n+1) (z0 )z n n!

f (z) =
n=0

Note that this is the Taylor series that we could obtain directly for f (z). Since f (z) is analytic on |z z0 | < R so is f (z).

f (z) =
n=0

f (n+1) (z0 )z n n!

3. 1 d2 1 1 = 2 (1 z)3 dz 2 1 z 1 d2 = 2 dz 2 = = 1 2 1 2

zn
n=0

n(n 1)z n2
n=2

(n + 2)(n + 1)z n
n=0

The radius of convergence is 1, which is the distance to the nearest singularity at z = 1. 609

4. The Taylor series expansion of f (z) about z = 0 is

f (z) =
n=0

f (n) (0) n z . n!

We compute the derivatives of f (z).


n1

(n)

(z) =
k=0

( k) (1 + z)n .

Now we determine the coecients in the series.


n1

(n)

(0) =

( k)
k=0 n1 k=0 (

(1 + z) =
n=0

k)

n!

zn

The radius of convergence of the series is the distance to the nearest singularity of (1 + z) . This occurs at z = 1. Thus the series converges for |z| < 1. We can corroborate this with the ratio test. We compute the radius of convergence. n1 n+1 k=0 ( k) /n! R = lim = lim =1 n n ( n n k=0 ( k)) /(n + 1)! If we use the binomial coecient, n then we can write the series in a compact form.
n1 k=0 (

k)

n!

(1 + z) =
n=0

n z n

610

Solution 12.25 For |z| < 1: 1 = z 1 + z

=
n=0

(z)n

(Note that |z| < 1 | z| < 1.) For |z| > 1: 1 1 1 = z z (1 /z) (Note that |z| > 1 | /z| < 1.) 1 = z =

1 n z n z n=
0

n=0 0

=
1

()n z n1
n=

= Solution 12.26 We expand the function in partial fractions. f (z) =

()n+1 z n
n=

1 1 1 = (z + 1)(z + 2) z+1 z+2 611

The Taylor series about z = 0 for 1/(z + 1) is 1 1 = 1+z 1 (z)

=
n=0

(z)n , (1)n z n ,
n=0

for |z| < 1 for |z| < 1

= The series about z = for 1/(z + 1) is

1/z 1 = 1+z 1 + 1/z = = = The Taylor series about z = 0 for 1/(z + 2) is 1 1/2 = 2+z 1 + z/2 1 = 2 =
n=0

1 z

(1/z)n , (1)n z n1 ,

for |1/z| < 1 for |z| > 1 for |z| > 1

n=0

n=0 1

(1)n+1 z n ,
n=

(z/2)n , (1)n n z , 2n+1

for |z/2| < 1 for |z| < 2

n=0

612

The series about z = for 1/(z + 2) is 1 1/z = 2+z 1 + 2/z = = = 1 z

(2/z)n ,

for |2/z| < 1 for |z| > 2 for |z| > 2

n=0

(1)n 2n z n1 , (1)n+1 n z , 2n+1 n=


n=0 1

To nd the expansions in the three regions, we just choose the appropriate series. 1. f (z) = =
n=0

1 1 1+z 2+z

(1)n z n
n=0

(1)n n z , 2n+1 zn,

for |z| < 1 for |z| < 1

=
n=0

(1)n 1

1 2n+1

f (z) =
n=0

(1)n

2n+1 1 n z , 2n+1

for |z| < 1

613

2. f (z) =
1

1 1 1+z 2+z

f (z) =

(1)
n=

n+1 n

z
n=0

(1)n n z , 2n+1

for 1 < |z| < 2

3. f (z) = = 1 1 1+z 2+z


1

(1)n+1 z n
n= 1

(1)n+1 n z , 2n+1 n= 2n+1 1 n z , 2n+1

for 2 < |z|

f (z) =

(1)n+1
n=

for 2 < |z|

Solution 12.27 Laurent Series. We assume that m is a non-negative integer and that n is an integer. The Laurent series about the point z = 0 of m 1 f (z) = z + z is f (z) =
n=

an z n f (z) dz z n+1

where an =

1 2

614

and C is a contour going around the origin once in the positive direction. We manipulate the coecient integral into the desired form. an = 1 (z + 1/z)m dz 2 C z n+1 2 1 (e + e )m e d = e(n+1) 2 0 2 1 2m cosm en d = 2 0 2m1 2 cosm (cos(n) sin(n)) d = 0

Note that cosm is even and sin(n) is odd about = . 2m1 =


2

cosm cos(n) d
0

Binomial Series. Now we nd the binomial series expansion of f (z). 1 z+ z


m m

=
n=0 m

m mn z n m m2n z n

1 z

=
n=0 m

=
n=m mn even

m zn (m n)/2

615

The coecients in the series f (z) = an =

n=

an z n are m n m and m n even otherwise

m (mn)/2

By equating the coecients found by the two methods, we evaluate the desired integral.
2

(cos )m cos(n) d =
0

m 2m1 (mn)/2

m n m and m n even otherwise

Solution 12.28 First we write f (z) in the form f (z) = g(z) . (z /2)(z 2)2

g(z) is an entire function which grows no faster that z 3 at innity. By expanding g(z) in a Taylor series about the origin, we see that it is a polynomial of degree no greater than 3. f (z) = z 3 + z 2 + z + (z /2)(z 2)2

Since f (z) is a rational function we expand it in partial fractions to obtain a form that is convenient to integrate. f (z) = a b c + + +d z /2 z 2 (z 2)2

We use the value of the integrals of f (z) to determine the constants, a, b, c and d. a b c + + +d z /2 z 2 (z 2)2 2a = 2 a=1 616 dz = 2

|z|=1

|z|=3

1 b c + + +d z /2 z 2 (z 2)2 2(1 + b) = 0 b = 1

dz = 0

Note that by applying the second constraint, we can change the third constraint to zf (z) dz = 0.
|z|=3

|z|=3

1 c 1 + + d dz = 0 z /2 z 2 (z 2)2 |z|=3 (z /2) + /2 (z 2) + 2 c(z 2) + 2c + dz = 0 z /2 z2 (z 2)2 2 2+c =0 2 c=2 2 z 1 2 /2 1 + + d, z /2 z 2 (z 2)2

Thus we see that the function is f (z) =

where d is an arbitrary constant. We can also write the function in the form: dz 3 + 15 8 f (z) = . 4(z /2)(z 2)2 Complete Laurent Series. We nd the complete Laurent series about z = 0 for each of the terms in the partial 617

fraction expansion of f (z).

2 1 = z /2 1 + 2z

= 2
n=0

(2z)n ,

for | 2z| < 1 for |z| < 1/2

=
n=0

(2)n+1 z n ,

1/z 1 = z /2 1 /(2z) 1 = z =
n=0 1

n=0

2z 2
n

, z n1 ,
n1

for |/(2z)| < 1 for |z| < 2 for |z| < 2 for |z| < 2

=
n= 1

zn,

(2)n+1 z n ,
n=

618

1 1/2 = z2 1 z/2 1 = 2 =
n=0

n=0 n

z 2

for |z/2| < 1

z , 2n+1

for |z| < 2

1 1/z = z2 1 2/z = =
n=0 1

1 z

n=0

2 z

for |2/z| < 1 for |z| > 2 for |z| > 2

2n z n1 , 2n1 z n ,
n=

619

2 /2 1 = (2 /2) (1 z/2)2 2 (z 2) 4 4 = 8 = = 4 8 4 8

n=0

2 n

z 2

for |z/2| < 1 for |z| < 2

(1)n (n + 1)(1)n 2n z n ,
n=0

n=0

n+1 n z , 2n

for |z| < 2

2 /2 2 /2 = 2 (z 2) z2 2 /2 = z2

2 1 z

n=0

2 n

2 z

for |2/z| < 1 for |z| > 2

= (2 /2) = (2 /2) = (2 /2)

(1)n (n + 1)(1)n 2n z n2 ,
n=0 2

(n 1)2n2 z n ,
n=

for |z| > 2

n+1 n z , 2n+2 n=

for |z| > 2

We take the appropriate combination of these series to nd the Laurent series expansions in the regions: |z| < 1/2, 620

1/2 < |z| < 2 and 2 < |z|. For |z| < 1/2, we have

f (z) = f (z) =

(2)
n=0

n+1 n

z +
n=0

zn 4 + n+1 2 8 1 +

n=0

n+1 n z +d 2n zn + d

(2)n+1 +
n=0

2n+1

4n+1 8 2n

f (z) =
n=0

(2)n+1 +

1 2n+1

1+

4 (n + 1) 4

z n + d,

for |z| < 1/2

For 1/2 < |z| < 2, we have


1

f (z) =
1

(2)n+1 z n +
n= n=0

4 zn + 2n+1 8 4 (n + 1) 4

n=0

n+1 n z +d 2n for 1/2 < |z| < 2

f (z) = For 2 < |z|, we have

(2)
n=

n+1 n

z +
n=0

1 2n+1

1+

z n + d,

f (z) =
2

(2)
n=

n+1 n

z
n=

n1 n

z (2 /2)

n+1 n z +d 2n+2 n= for 2 < |z|

f (z) =
n=

(2)n+1

1 2n+1

(1 + (1 /4)(n + 1)) z n + d,

Solution 12.29 The radius of convergence of the series for f (z) is R = lim
n

k 3 /3k k3 = 3 lim = 3. n (k + 1)3 (k + 1)3 /3k+1 621

Thus f (z) is a function which is analytic inside the circle of radius 3. 1. The integrand is analytic. Thus by Cauchys theorem the value of the integral is zero. ez f (z) dz = 0
|z|=1

2. We use Cauchys integral formula to evaluate the integral. 2 (3) 2 3!33 f (z) dz = f (0) = = 2 z4 3! 3! 33 f (z) dz = 2 z4

|z|=1

|z|=1

3. We use Cauchys integral formula to evaluate the integral. f (z) ez 2 d dz = (f (z) ez ) 2 z 1! dz


z=0

= 2

|z|=1

1!13 31

|z|=1

f (z) ez 2 dz = 2 z 3

Solution 12.30 1. (a) 1 1 1 = + z(1 z) z 1z = 1 zn, + z n=0


for 0 < |z| < 1 for 0 < |z| < 1

1 = + zn, z n=1 622

(b)

1 1 1 = + z(1 z) z 1z 1 1 1 = z z 1 1/z = 1 1 z z 1 z

n=0

1 z

for |z| > 1

= =

z n , zn,

for |z| > 1 for |z| > 1

n=1

n=2

623

(c) 1 1 1 = + z(1 z) z 1z 1 1 = + (z + 1) 1 2 (z + 1) 1 1 1 1 = , (z + 1) 1 1/(z + 1) (z + 1) 1 2/(z + 1) 1 = (z + 1) = =


n=1

for |z + 1| > 1 and |z + 1| > 2 for |z + 1| > 1 and |z + 1| > 2

1 (z + 1)

n=0

1 1 n (z + 1) (z + 1) 12 , (z + 1)n
n

n=0

2n , (z + 1)n

for |z + 1| > 2

n=0

1 2n , (z + 1)n+1

for |z + 1| > 2 for |z + 1| > 2

=
n=2

1 2n1 (z + 1)n ,

2. First we factor the denominator of f (z) = 1/(z 4 + 4). z 4 + 4 = (z 1 )(z 1 + )(z + 1 )(z + 1 + ) We look for an annulus about z = 1 containing the point z = where f (z) is analytic. The singularities at z = 1 are a distance of 1 from z = 1; the singularities at z = 1 are at a distance of 5. Since f (z) is analytic in the domain 1 < |z 1| < 5 there is a convergent Laurent series in that domain.

624

Chapter 13 The Residue Theorem


Man will occasionally stumble over the truth, but most of the time he will pick himself up and continue on.

- Winston Churchill

13.1

The Residue Theorem

We will nd that many integrals on closed contours may be evaluated in terms of the residues of a function. We rst dene residues and then prove the Residue Theorem. 625

Result 13.1.1 Residues. Let f (z) be single-valued an analytic in a deleted neighborhood of z0 . Then f (z) has the Laurent series expansion

f (z) =
n=

an (z z0 )n ,
1 zz0

The residue of f (z) at z = z0 is the coecient of the

term:

Res(f (z), z0 ) = a1 . The residue at a branch point or non-isolated singularity is undened as the Laurent series does not exist. If f (z) has a pole of order n at z = z0 then we can use the Residue Formula: Res(f (z), z0 ) = lim
zz0

1 dn1 (z z0 )n f (z) n1 (n 1)! dz

See Exercise 13.4 for a proof of the Residue Formula.

Example 13.1.1 In Example 8.4.5 we showed that f (z) = z/ sin z has rst order poles at z = n, n Z \ {0}. Now 626

Figure 13.1: Deform the contour to lie in the deleted disk.

we nd the residues at these isolated singularities. z z , z = n = lim (z n) zn sin z sin z z n = n lim zn sin z 1 = n lim zn cos z 1 = n (1)n = (1)n n

Res

Residue Theorem. We can evaluate many integrals in terms of the residues of a function. Suppose f (z) has only one singularity, (at z = z0 ), inside the simple, closed, positively oriented contour C. f (z) has a convergent Laurent series in some deleted disk about z0 . We deform C to lie in the disk. See Figure 13.1. We now evaluate C f (z) dz by deforming the contour and using the Laurent series expansion of the function. 627

f (z) dz =
C B

f (z) dz

=
B n=

an (z z0 )n dz an
n= n=1

(z z0 )n+1 n+1

r e(+2)

+ a1 [log(z z0 )]r e r
r e

e(+2)

= a1 2

f (z) dz = 2 Res(f (z), z0 )


C

Now assume that f (z) has n singularities at {z1 , . . . , zn }. We deform C to n contours C1 , . . . , Cn which enclose the singularities and lie in deleted disks about the singularities in which f (z) has convergent Laurent series. See Figure 13.2. We evaluate C f (z) dz by deforming the contour.
n n

f (z) dz =
C k=1 Ck

f (z) dz = 2
k=1

Res(f (z), zk )

Now instead let f (z) be analytic outside and on C except for isolated singularities at {n } in the domain outside C and perhaps an isolated singularity at innity. Let a be any point in the interior of C. To evaluate C f (z) dz we make the change of variables = 1/(z a). This maps the contour C to C . (Note that C is negatively oriented.) All the points outside C are mapped to points inside C and vice versa. We can then evaluate the integral in terms of the singularities inside C . 628

C2 C C1 C3

Figure 13.2: Deform the contour n contours which enclose the n singularities.

f (z) dz =
C

1 1 +a d 2 C 1 1 = f + a dz 2 z C z 1 1 1 = 2 Res f +a , 2 z z n a n f 629

+ 2 Res

1 f z2

1 + a ,0 . z

C a C

Figure 13.3: The change of variables = 1/(z a).

Result 13.1.2 Residue Theorem. If f (z) is analytic in a compact, closed, connected domain D except for isolated singularities at {zn } in the interior of D then f (z) dz =
D k Ck

f (z) dz = 2
n

Res(f (z), zn ).

Here the set of contours {Ck } make up the positively oriented boundary D of the domain D. If the boundary of the domain is a single contour C then the formula simplies. f (z) dz = 2
C n

Res(f (z), zn )

If instead f (z) is analytic outside and on C except for isolated singularities at {n } in the domain outside C and perhaps an isolated singularity at innity then f (z) dz = 2
C n

Res

1 f z2

1 1 +a , z n a
630

+ 2 Res

1 f z2

1 + a ,0 . z

Here a is a any point in the interior of C.

Example 13.1.2 Consider 1 2


C

sin z dz z(z 1)

where C is the positively oriented circle of radius 2 centered at the origin. Since the integrand is single-valued with only isolated singularities, the Residue Theorem applies. The value of the integral is the sum of the residues from singularities inside the contour. The only places that the integrand could have singularities are z = 0 and z = 1. Since sin z cos z = lim = 1, z0 z z0 1 lim there is a removable singularity at the point z = 0. There is no residue at this point. Now we consider the point z = 1. Since sin(z)/z is analytic and nonzero at z = 1, that point is a rst order pole of the integrand. The residue there is Res sin z ,z = 1 z(z 1) = lim (z 1)
z1

sin z = sin(1). z(z 1)

There is only one singular point with a residue inside the path of integration. The residue at this point is sin(1). Thus the value of the integral is sin z 1 dz = sin(1) 2 C z(z 1) Example 13.1.3 Evaluate the integral
C

cot z coth z dz z3

where C is the unit circle about the origin in the positive direction. The integrand is cos z cosh z cot z coth z = 3 z3 z sin z sinh z 631

sin z has zeros at n. sinh z has zeros at n. Thus the only pole inside the contour of integration is at z = 0. Since sin z and sinh z both have simple zeros at z = 0, sin z = z + O(z 3 ), sinh z = z + O(z 3 )

the integrand has a pole of order 5 at the origin. The residue at z = 0 is 1 d4 z0 4! dz 4 lim 1 d4 z 2 cot z coth z z0 4! dz 4 1 = lim 24 cot(z) coth(z)csc(z)2 32z coth(z)csc(z)4 4! z0 16z cos(2z) coth(z)csc(z)4 + 22z 2 cot(z) coth(z)csc(z)4 + 2z 2 cos(3z) coth(z)csc(z)5 + 24 cot(z) coth(z)csch(z)2 + 24csc(z)2 csch(z)2 48z cot(z)csc(z)2 csch(z)2 48z coth(z)csc(z)2 csch(z)2 + 24z 2 cot(z) coth(z)csc(z)2 csch(z)2 + 16z 2 csc(z)4 csch(z)2 + 8z 2 cos(2z)csc(z)4 csch(z)2 32z cot(z)csch(z)4 16z cosh(2z) cot(z)csch(z)4 + 22z 2 cot(z) coth(z)csch(z)4 + 16z 2 csc(z)2 csch(z)4 + 8z 2 cosh(2z)csc(z)2 csch(z)4 + 2z 2 cosh(3z) cot(z)csch(z)5 = 1 4! 7 45 632 56 15

z5

cot z coth z z3

= lim

Since taking the fourth derivative of z 2 cot z coth z really sucks, we would like a more elegant way of nding the residue. We expand the functions in the integrand in Taylor series about the origin.
z z 1 z2 + 24 1 + z2 + 24 + cos z cosh z = 3 3 z5 z5 z 3 sin z sinh z z 3 z z6 + 120 z + z6 + 120 +
2 4 2 4

= = =

z3

1 z2 + z6
z4 6 z4 90

z4 + 6 1 1 + 60 36

1 1 z5 1

1 z5 1 = 5 z 1 = 5 z

+ z4 z4 1 + 1+ + 6 90 7 1 z4 + 45 7 1 + 45 z cot z coth z 14 dz = z3 45

7 Thus we see that the residue is 45 . Now we can evaluate the integral.

13.2
13.2.1

Cauchy Principal Value for Real Integrals


The Cauchy Principal Value
b a x0 b

First we recap improper integrals. If f (x) has a singularity at x0 (a . . . b) then f (x) dx lim +
0

f (x) dx + lim +
0

f (x) dx.
x0 +

633

For integrals on ( . . . ),
b

f (x) dx
1 1 1 x

a, b

lim

f (x) dx.
a

Example 13.2.1

dx is divergent. We show this with the denition of improper integrals.


1 1

1 dx = lim 0+ x = lim +
0 0

1 dx + lim 0+ x
0

1 dx x

[ln |x|] 1

+ lim [ln |x|]1 +

= lim ln lim ln + +
0

The integral diverges because and approach zero independently. Since 1/x is an odd function, it appears that the area under the curve is zero. Consider what would happen if and were not independent. If they approached zero symmetrically, = , then the value of the integral would be zero.
1

lim +
0

+
1

1 dx = lim (ln ln ) = 0 0+ x
1

We could make the integral have any value we pleased by choosing = c .


1

lim +
0

+
1 c

1 dx = lim (ln ln(c )) = ln c 0+ x


1 1

We have seen it is reasonable that

1 dx x

has some meaning, and if we could evaluate the integral, the most reasonable value would be zero. The Cauchy principal value provides us with a way of evaluating such integrals. If f (x) is continuous on (a, b) except at the point x0 (a, b)
1

This may remind you of conditionally convergent series. You can rearrange the terms to make the series sum to any number.

634

then the Cauchy principal value of the integral is dened


b x0 0 b

f (x) dx = lim +
a

f (x) dx +
a x0 +

f (x) dx .

The Cauchy principal value is obtained by approaching the singularity symmetrically. The principal value of the integral may exist when the integral diverges. If the integral exists, it is equal to the principal value of the integral. 1 1 The Cauchy principal value of 1 x dx is dened
1

1 dx lim 0+ x
0 0

1 dx + x

1 dx x

= lim [log |x|] [log |x|]1 1 + = lim (log | | log | |) + = 0. (Another notation for the principal value of an integral is PV f (x) dx.) Since the limits of integration approach zero symmetrically, the two halves of the integral cancel. If the limits of integration approached zero independently, (the denition of the integral), then the two halves would both diverge. Example 13.2.2
x x2 +1

dx is divergent. We show this with the denition of improper integrals.


x dx = lim 2+1 a, b x

b a

x2

x dx +1
b a

1 = lim ln(x2 + 1) a, b 2 b2 + 1 1 lim ln = 2 a, b a2 + 1 635

The integral diverges because a and b approach innity independently. Now consider what would happen if a and b were not independent. If they approached zero symmetrically, a = b, then the value of the integral would be zero.

1 lim ln 2 b

b2 + 1 b2 + 1

=0

We could make the integral have any value we pleased by choosing a = cb. We can assign a meaning to divergent integrals of the form Cauchy principal value of the integral is dened

f (x) dx with the Cauchy principal value. The

f (x) dx = lim

f (x) dx.
a

The Cauchy principal value is obtained by approaching innity symmetrically. The Cauchy principal value of
x x2 +1

dx is dened

x dx = lim 2+1 a x = lim = 0. 636


a

a a

x2

x dx +1
a a

1 ln x2 + 1 2

Result 13.2.1 Cauchy Principal Value. If f (x) is continuous on (a, b) except at the point x0 (a, b) then the integral of f (x) is dened
b a x0 b

f (x) dx = lim +
0

f (x) dx + lim +
0

f (x) dx.
x0 +

The Cauchy principal value of the integral is dened


b x0 0 b

f (x) dx = lim +
a

f (x) dx +
a x0 +

f (x) dx .

If f (x) is continuous on (, ) then the integral of f (x) is dened


b

f (x) dx =

a, b

lim

f (x) dx.
a

The Cauchy principal value of the integral is dened


a

f (x) dx = lim

f (x) dx.
a

The principal value of the integral may exist when the integral diverges. If the integral exists, it is equal to the principal value of the integral.
Example 13.2.3 Clearly

x dx diverges, however the Cauchy principal value exists.

x dx = lim

x2 2

a=0
a

637

In general, if f (x) is an odd function with no singularities on the nite real axis then

f (x) dx = 0.

13.3

Cauchy Principal Value for Contour Integrals

Example 13.3.1 Consider the integral 1 dz, z1

Cr

where Cr is the positively oriented circle of radius r and center at the origin. From the residue theorem, we know that the integral is 1 dz = z1 0 2 for r < 1, for r > 1.

Cr

When r = 1, the integral diverges, as there is a rst order pole on the path of integration. However, the principal value of the integral exists. 1 dz = lim 0+ Cr z 1
0 2

1 e d 1
2

= lim log(e 1) +

638

We choose the branch of the logarithm with a branch cut on the positive real axis and arg log z (0, 2). = lim log e(2 ) 1 log (e 1) +
0

= lim log +
0 0

1 i + O( 2 ) 1 log

1 + i + O( 2 ) 1

= lim log i + O( 2 ) log i + O( 2 ) + = lim Log +


0

+ O( 2 ) + arg + O( 2 ) Log

+ O( 2 ) arg + O( 2 )

3 2 2 = = Thus we obtain 0 1 dz = Cr z 1 2 for r < 1, for r = 1, for r > 1.

In the above example we evaluated the contour integral by parameterizing the contour. This approach is only feasible when the integrand is simple. We would like to use the residue theorem to more easily evaluate the principal value of the integral. But before we do that, we will need a preliminary result.

Result 13.3.1 Let f (z) have a rst order pole at z = z0 and let (z z0 )f (z) be analytic in some neighborhood of z0 . Let the contour C be a circular arc from z0 + e to z0 + e . (We assume that > and < 2.) lim +
0

f (z) dz = ( ) Res(f (z), z0 )


C

The contour is shown in Figure 13.4. (See Exercise 13.9 for a proof of this result.)
639

C z0

Figure 13.4: The C Contour

Cp C

Figure 13.5: The indented contour. Example 13.3.2 Consider


C

1 dz z1

where C is the unit circle. Let Cp be the circular arc of radius 1 that starts and ends a distance of from z = 1. Let C be the positive, circular arc of radius with center at z = 1 that joins the endpoints of Cp . Let Ci , be the union of Cp and C . (Cp stands for Principal value Contour; Ci stands for Indented Contour.) Ci is an indented contour that avoids the rst order pole at z = 1. Figure 13.5 shows the three contours. 640

Note that the principal value of the integral is 1 dz = lim 0+ z1 1 dz. z1

Cp

We can calculate the integral along Ci with the residue theorem. 1 dz = 2 z1 0+ , the contour becomes a semi-circle,

Ci

We can calculate the integral along C using Result 13.3.1. Note that as a circular arc of radians. lim +
0

1 dz = Res z1

1 ,1 z1

Now we can write the principal value of the integral along C in terms of the two known integrals. 1 1 dz = dz z1 Ci z 1 = 2 = 1 dz z1

In the previous example, we formed an indented contour that included the rst order pole. You can show that if we had indented the contour to exclude the pole, we would obtain the same result. (See Exercise 13.11.) We can extend the residue theorem to principal values of integrals. (See Exercise 13.10.) 641

Result 13.3.2 Residue Theorem for Principal Values. Let f (z) be analytic inside and on a simple, closed, positive contour C, except for isolated singularities at z1 , . . . , zm inside the contour and rst order poles at 1 , . . . , n on the contour. Further, let the contour be C 1 at the locations of these rst order poles. (i.e., the contour does not have a corner at any of the rst order poles.) Then the principal value of the integral of f (z) along C is
m n

f (z) dz = 2
C j=1

Res(f (z), zj ) +
j=1

Res(f (z), j ).

13.4

Integrals on the Real Axis

Example 13.4.1 We wish to evaluate the integral


x2

1 dx. +1

We can evaluate this integral directly using calculus.


x2

1 dx = [arctan x] +1 =

Now we will evaluate the integral using contour integration. Let CR be the semicircular arc from R to R in the upper half plane. Let C be the union of CR and the interval [R, R]. We can evaluate the integral along C with the residue theorem. The integrand has rst order poles at z = . For 642

R > 1, we have z2 1 dz = 2 Res +1 1 = 2 2 = . z2 1 , +1

Now we examine the integral along CR . We use the maximum modulus integral bound to show that the value of the integral vanishes as R . z2 1 1 dz R max 2 zCR z + 1 +1 1 = R 2 R 1 0 as R .

CR

Now we are prepared to evaluate the original real integral. 1 dz = +1 C 1 1 dx + dz = 2+1 2+1 x CR z z2

R R

We take the limit as R .


x2

1 dx = +1

We would get the same result by closing the path of integration in the lower half plane. Note that in this case the closed contour would be in the negative direction. 643

If you are really observant, you may have noticed that we did something a little funny in evaluating

1 dx. x2 + 1

The denition of this improper integral is


1 dx = lim 2+1 a+ x

0 a

1 dx+ = lim 2+1 b+ x


R

b 0

x2

1 dx. +1

In the above example we instead computed


R+

lim

1 dx. x2 + 1
x . x2 +1

Note that for some integrands, the former and latter are not the same. Consider the integral of

x dx = lim 2+1 a+ x = lim


a+

0 a

x dx + lim 2+1 b+ x

b 0

x2

1 log |a2 + 1| + lim b+ 2

x dx +1 1 log |b2 + 1| 2

Note that the limits do not exist and hence the integral diverges. We get a dierent result if the limits of integration approach innity symmetrically.
R R+

lim

x dx = lim R+ x2 + 1 =0

1 (log |R2 + 1| log |R2 + 1|) 2

(Note that the integrand is an odd function, so the integral from R to R is zero.) We call this the principal value of the integral and denote it by writing PV in front of the integral sign or putting a dash through the integral.
R

PV

f (x) dx

f (x) dx lim

R+

f (x) dx
R

644

The principal value of an integral may exist when the integral diverges. If the integral does converge, then it is equal to its principal value. We can use the method of Example 13.4.1 to evaluate the principal value of integrals of functions that vanish fast enough at innity.

Result 13.4.1 Let f (z) be analytic except for isolated singularities, with only rst order poles on the real axis. Let CR be the semi-circle from R to R in the upper half plane. If
R

lim

R max |f (z)|
zCR

=0
n

then

f (x) dx = 2
k=1

Res (f (z), zk ) +
k=1

Res(f (z), xk )

where z1 , . . . zm are the singularities of f (z) in the upper half plane and x1 , . . . , xn are the rst order poles on the real axis. Now let CR be the semi-circle from R to R in the lower half plane. If
R

lim

R max |f (z)|
zCR

=0
n

then

f (x) dx = 2
k=1

Res (f (z), zk )
k=1

Res(f (z), xk )

where z1 , . . . zm are the singularities of f (z) in the lower half plane and x1 , . . . , xn are the rst order poles on the real axis.
645

This result is proved in Exercise 13.13. Of course we can use this result to evaluate the integrals of the form

f (z) dz,
0

where f (x) is an even function.

13.5

Fourier Integrals

In order to do Fourier transforms, which are useful in solving dierential equations, it is necessary to be able to calculate Fourier integrals. Fourier integrals have the form

ex f (x) dx.

We evaluate these integrals by closing the path of integration in the lower or upper half plane and using techniques of contour integration. Consider the integral
/2

eR sin d.
0

Since 2/ sin for 0 /2, eR sin eR2/


/2

for 0 /2
/2

e
0

R sin

d
0

eR2/ d

R2/ /2 e = 2R 0 R = (e 1) 2R 2R 0 as R 646

We can use this to prove the following Result 13.5.1. (See Exercise 13.17.)

Result 13.5.1 Jordans Lemma.

eR sin d <
0

. R

Suppose that f (z) vanishes as |z| . If is a (positive/negative) real number and CR is a semi-circle of radius R in the (upper/lower) half plane then the integral f (z) ez dz
CR

vanishes as R .
We can use Jordans Lemma and the Residue Theorem to evaluate many Fourier integrals. Consider f (x) ex dx, where is a positive real number. Let f (z) be analytic except for isolated singularities, with only rst order poles on the real axis. Let C be the contour from R to R on the real axis and then back to R along a semi-circle in the upper half plane. If R is large enough so that C encloses all the singularities of f (z) in the upper half plane then
m n

f (z) ez dz = 2
C k=1

Res(f (z) ez , zk ) +
k=1

Res(f (z) ez , xk )

where z1 , . . . zm are the singularities of f (z) in the upper half plane and x1 , . . . , xn are the rst order poles on the real axis. If f (z) vanishes as |z| then the integral on CR vanishes as R by Jordans Lemma.
m n

f (x) ex dx = 2
k=1

Res(f (z) ez , zk ) +
k=1

Res(f (z) ez , xk )

For negative we close the path of integration in the lower half plane. Note that the contour is then in the negative direction. 647

Result 13.5.2 Fourier Integrals. Let f (z) be analytic except for isolated singularities, with only rst order poles on the real axis. Suppose that f (z) vanishes as |z| . If is a positive real number then
m n

f (x) e

dx = 2
k=1

Res(f (z) e

, zk ) +
k=1

Res(f (z) ez , xk )

where z1 , . . . zm are the singularities of f (z) in the upper half plane and x1 , . . . , xn are the rst order poles on the real axis. If is a negative real number then
m n

f (x) e

dx = 2
k=1

Res(f (z) e

, zk )
k=1

Res(f (z) ez , xk )

where z1 , . . . zm are the singularities of f (z) in the lower half plane and x1 , . . . , xn are the rst order poles on the real axis.

13.6

Fourier Cosine and Sine Integrals

Fourier cosine and sine integrals have the form,


f (x) cos(x) dx and


0 0

f (x) sin(x) dx.

If f (x) is even/odd then we can evaluate the cosine/sine integral with the method we developed for Fourier integrals. 648

Let f (z) be analytic except for isolated singularities, with only rst order poles on the real axis. Suppose that f (x) is an even function and that f (z) vanishes as |z| . We consider real > 0.

1 f (x) cos(x) dx = f (x) cos(x) dx 2

Since f (x) sin(x) is an odd function, 1 f (x) sin(x) dx = 0. 2 Thus

f (x) cos(x) dx =

1 f (x) ex dx 2

Now we apply Result 13.5.2.


m n

f (x) cos(x) dx =
k=1

Res(f (z) e

, zk ) + 2

Res(f (z) ez , xk )
k=1

where z1 , . . . zm are the singularities of f (z) in the upper half plane and x1 , . . . , xn are the rst order poles on the real axis. If f (x) is an odd function, we note that f (x) cos(x) is an odd function to obtain the analogous result for Fourier sine integrals. 649

Result 13.6.1 Fourier Cosine and Sine Integrals. Let f (z) be analytic except for isolated singularities, with only rst order poles on the real axis. Suppose that f (x) is an even function and that f (z) vanishes as |z| . We consider real > 0.
m

f (x) cos(x) dx =
k=1

Res(f (z) e

, zk ) + 2

Res(f (z) ez , xk )
k=1

where z1 , . . . zm are the singularities of f (z) in the upper half plane and x1 , . . . , xn are the rst order poles on the real axis. If f (x) is an odd function then,

f (x) sin(x) dx =
k=1

Res(f (z) e

, k ) + 2

Res(f (z) ez , xk )
k=1

where 1 , . . . are the singularities of f (z) in the lower half plane and x1 , . . . , xn are the rst order poles on the real axis.
Now suppose that f (x) is neither even nor odd. We can evaluate integrals of the form:

f (x) cos(x) dx and


f (x) sin(x) dx

by writing them in terms of Fourier integrals 1 2 f (x) sin(x) dx = 2 f (x) cos(x) dx =


f (x) ex dx + f (x) ex

1 2 dx + 2

f (x) ex dx

f (x) ex dx

650

CR C

Figure 13.6:

13.7

Contour Integration and Branch Cuts


0

Example 13.7.1 Consider xa dx, x+1 0 < a < 1,

where xa denotes exp(a ln(x)). We choose the branch of the function f (z) = z a z+1 |z| > 0, 0 < arg z < 2

with a branch cut on the positive real axis. Let C and CR denote the circular arcs of radius and R where < 1 < R. C is negatively oriented; CR is positively oriented. Consider the closed contour C that is traced by a point moving from C to CR above the branch cut, next around CR , then below the cut to C , and nally around C . (See Figure 13.11.) We write f (z) in polar coordinates. f (z) = exp(a(log r + i)) exp(a log z) = z+1 r e +1 651

We evaluate the function above, (z = r e0 ), and below, (z = r e2 ), the branch cut. exp[a(log r + i0)] ra = r+1 r+1 exp[a(log r + 2)] ra e2a f (r e2 ) = = . r+1 r+1 f (r e0 ) = We use the residue theorem to evaluate the integral along C. f (z) dz = 2 Res(f (z), 1)
C R

ra dr + r+1

f (z) dz
CR

ra e2a dr + r+1

f (z) dz = 2 Res(f (z), 1)


C

The residue is Res(f (z), 1) = exp(a log(1)) = exp(a(log 1 + )) = ea . We bound the integrals along C and CR with the maximum modulus integral bound.
a 1a

= 2 1 1 a R R1a f (z) dz 2R = 2 R1 R1 CR
C

f (z) dz 2

Since 0 < a < 1, the values of the integrals tend to zero as 0 and R . Thus we have
0

ea ra dr = 2 r+1 1 e2a
0

xa dx = x+1 sin a 652

Result 13.7.1 Integrals from Zero to Innity. Let f (z) be a single-valued analytic function with only isolated singularities and no singularities on the positive, real axis, [0, ). Let a Z. If the integrals exist then,
n

f (x) dx =
0 0 0 k=1

Res (f (z) log z, zk ) ,


n

2 x f (x) dx = 1 e2a
a n

Res (z a f (z), zk ) ,
k=1 n 2

1 f (x) log x dx = 2

Res f (z) log z, zk +


k=1 n k=1

Res (f (z) log z, zk ) ,

2 x f (x) log x dx = 1 e2a


a

Res (z a f (z) log z, zk )


k=1

2a + sin2 (a)
0

Res (z a f (z), zk ) ,
k=1

m x f (x) log x dx = m a
a m

2 1 e2a

Res (z a f (z), zk ) ,
k=1

where z1 , . . . , zn are the singularities of f (z) and there is a branch cut on the positive real axis with 0 < arg(z) < 2.

653

13.8

Exploiting Symmetry

We have already used symmetry of the integrand to evaluate certain integrals. For f (x) an even function we were able to evaluate 0 f (x) dx by extending the range of integration from to . For

x f (x) dx
0

we put a branch cut on the positive real axis and noted that the value of the integrand below the branch cut is a constant multiple of the value of the function above the branch cut. This enabled us to evaluate the real integral with contour integration. In this section we will use other kinds of symmetry to evaluate integrals. We will discover that periodicity of the integrand will produce this symmetry.

13.8.1

Wedge Contours

We note that z n = rn en is periodic in with period 2/n. The real and imaginary parts of z n are odd periodic in with period /n. This observation suggests that certain integrals on the positive real axis may be evaluated by closing the path of integration with a wedge contour. Example 13.8.1 Consider
0

1 dx 1 + xn 654

where n N, n 2. We can evaluate this integral using Result 13.7.1.


0

1 dx = Res 1 + xn k=0
n1

n1

log z (1+2k)/n ,e 1 + zn lim (z e(1+2k)/n ) log z 1 + zn log z + (z e(1+2k)/n )/z nz n1

=
k=0 n1

ze(1+2k)/n

=
k=0 n1

lim
ze(1+2k)/n

=
k=0

(1 + 2k)/n n e(1+2k)(n1)/n
n1

n2 e(n1)/n
n1 e/n k=1

(1 + 2k) e2k/n
k=0

= =

2 n2

k e2k/n

2 n 2 2/n 1 e n = n sin(/n) This is a bit grungy. To nd a spier way to evaluate the integral we note that if we write the integrand as a function of r and , it is periodic in with period 2/n. 1 1 = n 1+z 1 + rn en The integrand along the rays = 2/n, 4/n, 6/n, . . . has the same value as the integrand on the real axis. Consider the contour C that is the boundary of the wedge 0 < r < R, 0 < < 2/n. There is one singularity inside the 655

e/n

contour. We evaluate the residue there. Res 1 , e/n 1 + zn z e/n n ze/n 1 + z 1 = lim n1 ze/n nz e/n = n = lim

We evaluate the integral along C with the residue theorem. 1 2 e/n dz = 1 + zn n

Let CR be the circular arc. The integral along CR vanishes as R . 2R 1 1 dz max n zCR 1 + z n 1+z n 2R 1 n Rn 1 0 as R

CR

We parametrize the contour to evaluate the desired integral.


0

1 dx + 1 + xn
0

1 2 e/n e2/n dx = n n 1+x /n 1 2 e dx = n 1+x n(1 e2/n ) 1 dx = n 1+x n sin(/n)

656

13.8.2

Box Contours

Recall that ez = ex+y is periodic in y with period 2. This implies that the hyperbolic trigonometric functions cosh z, sinh z and tanh z are periodic in y with period 2 and odd periodic in y with period . We can exploit this property to evaluate certain integrals on the real axis by closing the path of integration with a box contour. Example 13.8.2 Consider the integral

x 1 dx = log tanh + cosh x 4 2 = log(1) log(1) = .

We will evaluate this integral using contour integration. Note that cosh(x + ) = ex+ + ex = cosh(x). 2

Consider the box contour C that is the boundary of the region R < x < R, 0 < y < . The only singularity of the integrand inside the contour is a rst order pole at z = /2. We evaluate the integral along C with the residue theorem. 1 dz = 2 Res cosh z 1 , cosh z 2 z /2 = 2 lim z/2 cosh z 1 = 2 lim z/2 sinh z = 2 657

The integrals along the sides of the box vanish as R .


R+ R

1 1 dz max z[R...R+] cosh z cosh z 2 max R+y y[0...] e + e Ry 2 = R e eR sinh R 0 as R

The value of the integrand on the top of the box is the negative of its value on the bottom. We take the limit as R .

1 dx + cosh x

1 dx = 2 cosh x

1 dx = cosh x

13.9

Denite Integrals Involving Sine and Cosine


2

Example 13.9.1 For real-valued a, evaluate the integral: f (a) =


0

d . 1 + a sin

What is the value of the integral for complex-valued a. 658

Real-Valued a. For 1 < a < 1, the integrand is bounded, hence the integral exists. For |a| = 1, the integrand has a second order pole on the path of integration. For |a| > 1 the integrand has two rst order poles on the path of integration. The integral is divergent for these two cases. Thus we see that the integral exists for 1 < a < 1. For a = 0, the value of the integral is 2. Now consider a = 0. We make the change of variables z = e . The real integral from = 0 to = 2 becomes a contour integral along the unit circle, |z| = 1. We write the sine, cosine and the dierential in terms of z. sin = z z 1 , 2 cos = z + z 1 , 2 dz = e d, d = dz z

We write f (a) as an integral along C, the positively oriented unit circle |z| = 1. f (a) =
C

1/(z) dz = 1 + a(z z 1 )/(2)

z2

2/a dz + (2/a)z 1

We factor the denominator of the integrand. 2/a dz f (a) = C (z z1 )(z z2 ) 1 + 1 a2 1 1 a2 , z2 = a a

z1 =

Because |a| < 1, the second root is outside the unit circle. |z2 | = 1+ 1 a2 > 1. |a|

Since |z1 z2 | = 1, |z1 | < 1. Thus the pole at z1 is inside the contour and the pole at z2 is outside. We evaluate the 659

contour integral with the residue theorem. f (a) = 2/a dz + (2/a)z 1 C 2/a = 2 z1 z2 1 = 2 1 a2 z2

f (a) =

2 1 a2

Complex-Valued a. We note that the integral converges except for real-valued a satisfying |a| 1. On any closed subset of C \ {a R | |a| 1} the integral is uniformly convergent. Thus except for the values {a R | |a| 1}, we can dierentiate the integral with respect to a. f (a) is analytic in the complex plane except for the set of points on the real axis: a ( . . . 1] and a [1 . . . ). The value of the analytic function f (a) on the real axis for the interval (1 . . . 1) is f (a) = 2 . 1 a2

By analytic continuation we see that the value of f (a) in the complex plane is the branch of the function f (a) = 2 (1 a2 )1/2

where f (a) is positive, real-valued for a (1 . . . 1) and there are branch cuts on the real axis on the intervals: ( . . . 1] and [1 . . . ). 660

Result 13.9.1 For evaluating integrals of the form


a+2

F (sin , cos ) d
a

it may be useful to make the change of variables z = e . This gives us a contour integral along the unit circle about the origin. We can write the sine, cosine and dierential in terms of z. z + z 1 dz z z 1 , cos = , d = sin = 2 2 z

13.10

Innite Sums

The function g(z) = cot(z) has simple poles at z = n Z. The residues at these points are all unity. (z n) cos(z) zn sin(z) cos(z) (z n) sin(z) = lim zn cos(z) =1

Res( cot(z), n) = lim

Let Cn be the square contour with corners at z = (n + 1/2)(1 ). Recall that cos z = cos x cosh y sin x sinh y and 661 sin z = sin x cosh y + cos x sinh y.

First we bound the modulus of cot(z). | cot(z)| = = cos x cosh y sin x sinh y sin x cosh y + cos x sinh y cos2 x cosh2 y + sin2 x sinh2 y sin2 x cosh2 y + cos2 x sinh2 y cosh2 y sinh2 y

= | coth(y)| The hyperbolic cotangent, coth(y), has a simple pole at y = 0 and tends to 1 as y . Along the top and bottom of Cn , (z = x (n + 1/2)), we bound the modulus of g(z) = cot(z). | cot(z)| coth((n + 1/2)) Along the left and right sides of Cn , (z = (n + 1/2) + y), the modulus of the function is bounded by a constant. |g((n + 1/2) + y)| = cos((n + 1/2)) cosh(y) sin((n + 1/2)) sinh(y) sin((n + 1/2)) cosh(y) + cos((n + 1/2)) sinh(y) = | tanh(y)|

Thus the modulus of cot(z) can be bounded by a constant M on Cn . Let f (z) be analytic except for isolated singularities. Consider the integral, cot(z)f (z) dz.
Cn

662

We use the maximum modulus integral bound. cot(z)f (z) dz (8n + 4)M max |f (z)|
Cn zCn

Note that if
|z|

lim |zf (z)| = 0,

then
n

lim

cot(z)f (z) dz = 0.
Cn

This implies that the sum of all residues of cot(z)f (z) is zero. Suppose further that f (z) is analytic at z = n Z. The residues of cot(z)f (z) at z = n are f (n). This means

f (n) = ( sum of the residues of cot(z)f (z) at the poles of f (z) ).


n=

Result 13.10.1 If
|z|

lim |zf (z)| = 0,

then the sum of all the residues of cot(z)f (z) is zero. If in addition f (z) is analytic at z = n Z then

f (n) = ( sum of the residues of cot(z)f (z) at the poles of f (z) ).


n=

Example 13.10.1 Consider the sum

1 , (n + a)2 n= 663

a Z.

By Result 13.10.1 with f (z) = 1/(z + a)2 we have 1 1 = Res cot(z) , a 2 (n + a) (z + a)2 n= d cot(z) za dz sin2 (z) cos2 (z) = . sin2 (z) = lim 2 1 = (n + a)2 sin2 (a) n= Example 13.10.2 Derive /4 = 1 1/3 + 1/5 1/7 + 1/9 . Consider the integral 1 dw In = 2 Cn w(w z) sin w where Cn is the square with corners at w = (n + 1/2)(1 ), n Z+ . With the substitution w = x + y, | sin w|2 = sin2 x + sinh2 y, we see that |1/ sin w| 1 on Cn . Thus In 0 as n . We use the residue theorem and take the limit n .

0=
n=1

(1)n (1)n 1 1 + + 2 n(n z) n(n + z) z sin z z 1 (1)n 1 = 2z sin z z n2 2 z 2 n=1 = 1 (1)n (1)n z n=1 n z n + z 664

We substitute z = /2 into the above expression to obtain /4 = 1 1/3 + 1/5 1/7 + 1/9

665

13.11

Exercises

The Residue Theorem


Exercise 13.1 Evaluate the following closed contour integrals using Cauchys residue theorem. 1.
C

z2

dz , 1

where C is the contour parameterized by r = 2 cos(2), 0 2. where C is the positive circle |z| = 3.

2.
C

ez dz, z 2 (z 2)(z + 5) e1/z sin(1/z) dz,

3.
C

where C is the positive circle |z| = 1.

Hint, Solution Exercise 13.2 Derive Cauchys integral formula from Cauchys residue theorem. Hint, Solution Exercise 13.3 Calculate the residues of the following functions at each of the poles in the nite part of the plane. 1. 2. 3. 4. z4 1 a4

sin z z2 1 + z2 z(z 1)2 ez z 2 + a2 666

5.

(1 cos z)2 z7

Hint, Solution Exercise 13.4 Let f (z) have a pole of order n at z = z0 . Prove the Residue Formula: Res(f (z), z0 ) = lim Hint, Solution Exercise 13.5 Consider the function f (z) = z4 . z2 + 1 1 dn1 [(z z0 )n f (z)] . n1 (n 1)! dz

zz0

Classify the singularities of f (z) in the extended complex plane. Calculate the residue at each pole and at innity. Find the Laurent series expansions and their domains of convergence about the points z = 0, z = and z = . Hint, Solution Exercise 13.6 Let P (z) be a polynomial none of whose roots lie on the closed contour . Show that 1 2 P (z) dz = number of roots of P (z) which lie inside . P (z)

where the roots are counted according to their multiplicity. Hint: From the fundamental theorem of algebra, it is always possible to factor P (z) in the form P (z) = (z z1 )(z z2 ) (z zn ). Using this form of P (z) the integrand P (z)/P (z) reduces to a very simple expression. Hint, Solution 667

Exercise 13.7 Find the value of


C

ez dz (z ) tan z

where C is the positively-oriented circle 1. |z| = 2 2. |z| = 4 Hint, Solution

Cauchy Principal Value for Real Integrals


Solution 13.1 Show that the integral
1 1

1 dx. x

is divergent. Evaluate the integral


1 1

1 dx, x
1 0

R, = 0. 1 dx x 1 dx. x

Evaluate lim+

1 1

and
0

lim

The integral exists for arbitrarily close to zero, but diverges when = 0. Plot the real and imaginary part of the integrand. If one were to assign meaning to the integral for = 0, what would the value of the integral be? 668

Exercise 13.8 Do the principal values of the following integrals exist? 1. 2. 3.


1 1 1 x2 1 1 1 x3

dx, dx, dx.

1 f (x) 1 x3

Assume that f (x) is real analytic on the interval (1, 1). Hint, Solution

Cauchy Principal Value for Contour Integrals


Exercise 13.9 Let f (z) have a rst order pole at z = z0 and let (z z0 )f (z) be analytic in some neighborhood of z0 . Let the contour C be a circular arc from z0 + e to z0 + e . (Assume that > and < 2.) Show that lim +
0

f (z) dz = ( ) Res(f (z), z0 )


C

Hint, Solution Exercise 13.10 Let f (z) be analytic inside and on a simple, closed, positive contour C, except for isolated singularities at z1 , . . . , zm inside the contour and rst order poles at 1 , . . . , n on the contour. Further, let the contour be C 1 at the locations of these rst order poles. (i.e., the contour does not have a corner at any of the rst order poles.) Show that the principal value of the integral of f (z) along C is
m n

f (z) dz = 2
C j=1

Res(f (z), zj ) +
j=1

Res(f (z), j ).

Hint, Solution 669

Exercise 13.11 Let C be the unit circle. Evaluate


C

1 dz z1

by indenting the contour to exclude the rst order pole at z = 1. Hint, Solution

Integrals on the Real Axis


Exercise 13.12 Evaluate the following improper integrals.

1.
0

x2 dx = 2 + 1)(x2 + 4) (x 6 dx , (x + b)2 + a2 a>0

2.

Hint, Solution Exercise 13.13 Prove Result 13.4.1. Hint, Solution Exercise 13.14 Evaluate

x2

2x . +x+1

Hint, Solution Exercise 13.15 Use contour integration to evaluate the integrals 670

1.

dx , 1 + x4 x2 dx , (1 + x2 )2 cos(x) dx. 1 + x2

2.

3.

Hint, Solution Exercise 13.16 Evaluate by contour integration


0

x6 dx. (x4 + 1)2

Hint, Solution

Fourier Integrals
Exercise 13.17 Suppose that f (z) vanishes as |z| . If is a (positive / negative) real number and CR is a semi-circle of radius R in the (upper / lower) half plane then show that the integral f (z) ez dz
CR

vanishes as R . Hint, Solution Exercise 13.18 Evaluate by contour integration


cos 2x dx. x 671

Hint, Solution

Fourier Cosine and Sine Integrals


Exercise 13.19 Evaluate

sin x dx. x

Hint, Solution Exercise 13.20 Evaluate


1 cos x dx. x2

Hint, Solution Exercise 13.21 Evaluate


0

sin(x) dx. x(1 x2 )

Hint, Solution

Contour Integration and Branch Cuts


Exercise 13.22 Evaluate the following integrals.

1.
0

ln2 x 3 dx = 1 + x2 8 ln x dx = 0 1 + x2 672

2.
0

Hint, Solution Exercise 13.23 By methods of contour integration nd


0

x2

dx + 5x + 6

[ Recall the trick of considering Hint, Solution Exercise 13.24 Show that

f (z) log z dz with a suitably chosen contour and branch for log z. ]

xa a dx = 2 (x + 1) sin(a) log x dx = 0, (x + 1)2

for 1 < (a) < 1.


0

From this derive that


0

log2 x 2 dx = . (x + 1)2 3

Hint, Solution Exercise 13.25 Consider the integral

I(a) =
0

xa dx. 1 + x2

1. For what values of a does the integral exist? 2. Evaluate the integral. Show that I(a) = 3. Deduce from your answer in part (b) the results
0

2 cos(a/2)

log x dx = 0, 1 + x2 673

log2 x 3 dx = . 1 + x2 8

You may assume that it is valid to dierentiate under the integral sign. Hint, Solution Exercise 13.26 Let f (z) be a single-valued analytic function with only isolated singularities and no singularities on the positive real axis, [0, ). Give sucient conditions on f (x) for absolute convergence of the integral

xa f (x) dx.
0

Assume that a is not an integer. Evaluate the integral by considering the integral of z a f (z) on a suitable contour. (Consider the branch of z a on which 1a = 1.) Hint, Solution Exercise 13.27 Using the solution to Exercise 13.26, evaluate

xa f (x) log x dx,


0

and
0

xa f (x) logm x dx, where m is a positive integer. Hint, Solution Exercise 13.28 Using the solution to Exercise 13.26, evaluate

f (x) dx,
0

i.e. examine a = 0. The solution will suggest a way to evaluate the integral with contour integration. Do the contour integration to corroborate the value of 0 f (x) dx. Hint, Solution 674

Exercise 13.29 Let f (z) be an analytic function with only isolated singularities and no singularities on the positive real axis, [0, ). Give sucient conditions on f (x) for absolute convergence of the integral

f (x) log x dx
0

Evaluate the integral with contour integration. Hint, Solution Exercise 13.30 For what values of a does the following integral exist?
0

xa dx. 1 + x4

Evaluate the integral. (Consider the branch of xa on which 1a = 1.) Hint, Solution Exercise 13.31 By considering the integral of f (z) = z 1/2 log z/(z + 1)2 on a suitable contour, show that
0

x1/2 log x dx = , (x + 1)2

x1/2 dx = . 2 (x + 1) 2

Hint, Solution

Exploiting Symmetry
Exercise 13.32 Evaluate by contour integration, the principal value integral

I(a) =

eax dx ex ex

675

for a real and |a| < 1. [Hint: Consider the contour that is the boundary of the box, R < x < R, 0 < y < , but indented around z = 0 and z = . Hint, Solution Exercise 13.33 Evaluate the following integrals.

1.
0

dx , (1 + x2 )2 dx . 1 + x3

2.
0

Hint, Solution Exercise 13.34 Find the value of the integral I

I=
0

dx 1 + x6

by considering the contour integral

dz 1 + z6

with an appropriately chosen contour . Hint, Solution Exercise 13.35 2 Let C be the boundary of the sector 0 < r < R, 0 < < /4. By integrating ez on C and letting R show that 1 2 2 2 ex dx. cos(x ) dx = sin(x ) dx = 2 0 0 0 Hint, Solution 676

Exercise 13.36 Evaluate


x dx sinh x

using contour integration. Hint, Solution Exercise 13.37 Show that


eax dx = x +1 e sin(a)

for 0 < a < 1.

Use this to derive that

cosh(bx) dx = cosh x cos(b/2)

for 1 < b < 1.

Hint, Solution Exercise 13.38 Using techniques of contour integration nd for real a and b:

F (a, b) =
0

d (a + b cos )2

What are the restrictions on a and b if any? Can the result be applied for complex a, b? How? Hint, Solution Exercise 13.39 Show that

cos x dx = /2 ex + ex e + e/2

[ Hint: Begin by considering the integral of ez /(ez + ez ) around a rectangle with vertices: R, R + .] Hint, Solution 677

Denite Integrals Involving Sine and Cosine


Exercise 13.40 Evaluate the following real integrals.

1.

d = 2 1 + sin2 sin4 d

/2

2.
0

Hint, Solution Exercise 13.41 Use contour integration to evaluate the integrals
2

1.
0

d , 2 + sin() cos(n) d 1 2a cos() + a2 for |a| < 1, n Z0+ .

2.

Hint, Solution Exercise 13.42 By integration around the unit circle, suitably indented, show that

cos(n) sin(n) d = . cos cos sin

Hint, Solution 678

Exercise 13.43 Evaluate


1 0

x2 dx. (1 + x2 ) 1 x2

Hint, Solution

Innite Sums
Exercise 13.44 Evaluate

n=1

1 . n4

Hint, Solution Exercise 13.45 Sum the following series using contour integration:

n=

n2

1 2

Hint, Solution

679

13.12
Hint 13.1

Hints

The Residue Theorem

Hint 13.2

Hint 13.3

Hint 13.4 Substitute the Laurent series into the formula and simplify. Hint 13.5 Use that the sum of all residues of the function in the extended complex plane is zero in calculating the residue at innity. To obtain the Laurent series expansion about z = , write the function as a proper rational function, (numerator has a lower degree than the denominator) and expand in partial fractions. Hint 13.6

Hint 13.7

Cauchy Principal Value for Real Integrals


Hint 13.8

680

Hint 13.9 For the third part, does the integrand have a term that behaves like 1/x2 ?

Cauchy Principal Value for Contour Integrals


Hint 13.10 Expand f (z) in a Laurent series. Only the rst term will make a contribution to the integral in the limit as 0+ . Hint 13.11 Use the result of Exercise 13.9. Hint 13.12 Look at Example 13.3.2.

Integrals on the Real Axis


Hint 13.13

Hint 13.14 Close the path of integration in the upper or lower half plane with a semi-circle. Use the maximum modulus integral bound, (Result 10.2.1), to show that the integral along the semi-circle vanishes. Hint 13.15 Make the change of variables x = 1/. Hint 13.16 Use Result 13.4.1. Hint 13.17

681

Fourier Integrals
Hint 13.18 Use

eR sin d <
0

. R

Hint 13.19

Fourier Cosine and Sine Integrals


Hint 13.20 Consider the integral of Hint 13.21 Show that
1 cos x 1 ex dx = dx. x2 x2 ex . x

Hint 13.22 Show that


0

ex sin(x) dx = dx. x(1 x2 ) 2 x(1 x2 )

Contour Integration and Branch Cuts


Hint 13.23 Integrate a branch of log2 z/(1 + z 2 ) along the boundary of the domain < r < R, 0 < < . Hint 13.24

682

Hint 13.25 Note that


1

xa dx
0

converges for

(a) > 1; and

xa dx
1

converges for (a) < 1. Consider f (z) = z a /(z + 1)2 with a branch cut along the positive real axis and the contour in Figure 13.11 in the limit as 0 and R . To derive the last two integrals, dierentiate with respect to a. Hint 13.26

Hint 13.27 Consider the integral of z a f (z) on the contour in Figure 13.11. Hint 13.28 Dierentiate with respect to a. Hint 13.29 Take the limit as a 0. Use LHospitals rule. To corroborate the result, consider the integral of f (z) log z on an appropriate contour. Hint 13.30 Consider the integral of f (z) log2 z on the contour in Figure 13.11.

683

Hint 13.31 Consider the integral of za 1 + z4 on the boundary of the region < r < R, 0 < < /2. Take the limits as f (z) =

0 and R .

Hint 13.32 Consider the branch of f (z) = z 1/2 log z/(z + 1)2 with a branch cut on the positive real axis and 0 < arg z < 2. Integrate this function on the contour in Figure 13.11.

Exploiting Symmetry
Hint 13.33 Hint 13.34 For the second part, consider the integral along the boundary of the region, 0 < r < R, 0 < < 2/3. Hint 13.35 Hint 13.36 To show that the integral on the quarter-circle vanishes as R establish the inequality, cos 2 1 4 , 0 . 4

Hint 13.37 Consider the box contour C this is the boundary of the rectangle, R x R, 0 y . The value of the integral is 2 /2. Hint 13.38 Consider the rectangular contour with corners at R and R + 2. Let R . 684

Hint 13.39 Hint 13.40

Denite Integrals Involving Sine and Cosine


Hint 13.41 Hint 13.42 Hint 13.43 Hint 13.44 Make the changes of variables x = sin and then z = e .

Innite Sums
Hint 13.45 Use Result 13.10.1. Hint 13.46

685

1 -1 -1 1

Figure 13.7: The contour r = 2 cos(2).

13.13

Solutions

The Residue Theorem


Solution 13.2 1. We consider dz z2 1

where C is the contour parameterized by r = 2 cos(2), 0 2. (See Figure 13.7.) There are rst order 686

poles at z = 1. We evaluate the integral with Cauchys residue theorem. z2 dz 1 = 2 Res , z = 1 + Res 21 1 z 1 1 = 2 + z + 1 z=1 z 1 z=1 =0 2. We consider the integral ez dz, 2 C z (z 2)(z + 5) where C is the positive circle |z| = 3. There is a second order pole at z = 0, and rst order poles at z = 2 and z = 5. The poles at z = 0 and z = 2 lie inside the contour. We evaluate the integral with Cauchys residue theorem. ez ez dz = 2 Res ,z = 0 2 z 2 (z 2)(z + 5) C z (z 2)(z + 5) ez + Res ,z = 2 z 2 (z 2)(z + 5) ez ez d + 2 = 2 dz (z 2)(z + 5) z=0 z (z + 5) z=2 ez ez d = 2 + 2 dz (z 2)(z + 5) z=0 z (z + 5) z=2 1 5 (z 2 + (7 2)z 5 12) ez e2 + = 2 2 (z + 5)2 (z 2) 58 116 z=0 3 1 5 e2 = 2 + + 25 20 58 116 5 1 6 1 5 = + cos 2 sin 2 + + cos 2 + sin 2 10 58 29 25 29 58 687 z2 1 , z = 1 1

3. We consider the integral e1/z sin(1/z) dz


C

where C is the positive circle |z| = 1. There is an essential singularity at z = 0. We determine the residue there by expanding the integrand in a Laurent series. e1/z sin(1/z) = 1 +O z 1 1 = +O z z2 1+ 1 z2 1 +O z 1 z3

The residue at z = 0 is 1. We evaluate the integral with the residue theorem. e1/z sin(1/z) dz = 2
C

Solution 13.3 If f () is analytic in a compact, closed, connected domain D and z is a point in the interior of D then Cauchys integral formula states n! f () f (n) (z) = d. 2 D ( z)n+1 To corroborate this, we evaluate the integral with Cauchys residue theorem. There is a pole of order n + 1 at the point = z. n! 2 f () n! 2 dn d. = f () ( z)n+1 2 n! d n =f Solution 13.4 1. z4 1 1 = 4 a (z a)(z + a)(z a)(z + a) 688
(n)

=z

(z)

There are rst order poles at z = a and z = a. We calculate the residues there. 1 1 = 3 (z + a)(z a)(z + a) z=a 4a 1 1 1 Res , z = a = = 3 4 a4 z (z a)(z a)(z + a) z=a 4a 1 1 Res , z = a = = 3 4 a4 z (z a)(z + a)(z + a) z=a 4a 1 1 Res , z = a = = 3 4 a4 z (z a)(z + a)(z a) z=a 4a Res z4 = 2. sin z z2 Since denominator has a second order zero at z = 0 and the numerator has a rst order zero there, the function has a rst order pole at z = 0. We calculate the residue there. Res sin z ,z = 0 z2 = lim sin z z0 z cos z = lim z0 1 =1 1 ,z = a a4

3.

1 + z2 z(z 1)2 There is a rst order pole at z = 0 and a second order pole at z = 1. Res 1 + z2 ,z = 0 z(z 1)2 689 1 + z2 = (z 1)2 =1
z=0

Res

1 + z2 ,z = 1 z(z 1)2

d 1 + z2 dz z 1 = 1 2 z =0

z=1

z=1

4. ez / (z 2 + a2 ) has rst order poles at z = a. We calculate the residues there. ez ez ea Res , z = a = = z 2 + a2 z + a z=a 2a z z e e ea Res , z = a = = z 2 + a2 z a z=a 2a 5. Since 1 cos z has a second order zero at z = 0, (1cos z) has a third order pole at that point. We nd the z7 residue by expanding the function in a Laurent series. z2 z4 (1 cos z)2 = z 7 1 1 + + O z6 z7 2 24 z2 z4 =z + O z6 2 24 z4 z6 = z 7 + O z8 4 24 1 1 = 3 + O(z) 4z 24z
7 2 2
2

The residue at z = 0 is 1/24.

690

Solution 13.5 Since f (z) has an isolated pole of order n at z = z0 , it has a Laurent series that is convergent in a deleted neighborhood about that point. We substitute this Laurent series into the Residue Formula to verify it. Res(f (z), z0 ) = lim 1 dn1 [(z z0 )n f (z)] (n 1)! dz n1 1 dn1 (z z0 )n ak (z z0 )k (n 1)! dz n1 k=n 1 dn1 (n 1)! dz n1

zz0

= lim

zz0

= lim = lim

zz0

akn (z z0 )k
k=0

zz0

k! 1 akn (z z0 )kn+1 (n 1)! k=n1 (k n + 1)! 1 (n 1)!

= lim =

zz0

ak1
k=0

(k + n 1)! (z z0 )k k!

1 (n 1)! a1 (n 1)! 0! = a1 This proves the Residue Formula. Solution 13.6 Classify Singularities. f (z) = z4 z4 = . z2 + 1 (z )(z + )

There are rst order poles at z = . Since the function behaves like z 2 at innity, there is a second order pole there. 691

To see this more slowly, we can make the substitution z = 1/ and examine the point = 0. f 1 1 1 4 = 2 = 2 = 2 4 +1 + (1 + 2 )

f (1/) has a second order pole at = 0, which implies that f (z) has a second order pole at innity. Residues. The residues at z = are, Res Res The residue at innity is Res(f (z), ) = Res 1 1 f , = 0 2 1 4 = Res , = 0 2 2 + 1 4 = Res , = 0 1 + 2 z4 , z2 + 1 z4 , z2 + 1 = lim z4 = , z z + 2 z4 = . z z 2

= lim

Here we could use the residue formula, but its easier to nd the Laurent expansion.

= Res =0

4 n=0

(1)n 2n , = 0

We could also calculate the residue at innity by recalling that the sum of all residues of this function in the extended complex plane is zero. + + Res(f (z), ) = 0 2 2 692

Res(f (z), ) = 0 Laurent Series about z = 0. Since the nearest singularities are at z = , the Taylor series will converge in the disk |z| < 1. z4 1 = z4 2+1 z 1 (z)2

=z

4 n=0

(z 2 )n (1)n z 2n
n=0

= z4

=
n=2

(1)n z 2n

This geometric series converges for | z 2 | < 1, or |z| < 1. The series expansion of the function is z4 = z2 + 1

(1)n z 2n
n=2

for |z| < 1

Laurent Series about z = . We expand f (z) in partial fractions. First we write the function as a proper rational function, (i.e. the numerator has lower degree than the denominator). By polynomial division, we see that f (z) = z 2 1 + Now we expand the last term in partial fractions. f (z) = z 2 1 + 693 /2 /2 + z z+ z2 1 . +1

Since the nearest singularity is at z = , the Laurent series will converge in the annulus 0 < |z | < 2. z 2 1 = ((z ) + )2 1 = (z )2 + 2(z ) 2

/2 /2 = z+ 2 + (z ) 1/4 = 1 (z )/2 1 = 4 = 1 4

n=0

(z ) 2 n (z )n 2n

n=0

This geometric series converges for |(z )/2| < 1, or |z | < 2. The series expansion of f (z) is /2 1 f (z) = 2 + 2(z ) + (z )2 + z 4

n=0

n (z )n . 2n

z4 /2 1 = 2 + 2(z ) + (z )2 + z2 + 1 z 4

n=0

n (z )n n 2

for |z | < 2

Laurent Series about z = . Since the nearest singularities are at z = , the Laurent series will converge in 694

the annulus 1 < |z| < . z4 z2 = z2 + 1 1 + 1/z 2

=z =

2 n=0 0

1 z2

(1)n z 2(n+1)
n= 1

(1)n+1 z 2n
n=

This geometric series converges for | 1/z 2 | < 1, or |z| > 1. The series expansion of f (z) is z4 = (1)n+1 z 2n z 2 + 1 n=
1

for 1 < |z| <

Solution 13.7 Method 1: Residue Theorem. We factor P (z). Let m be the number of roots, counting multiplicities, that lie inside the contour . We nd a simple expression for P (z)/P (z).
n

P (z) = c P (z) = c

(z zk )
k=1 n n

(z zj )
k=1 j=1 j=k

695

P (z) = P (z) =

n k=1

n j=1 (z j=k

zj )

c
n

k=1 n

n k=1 (z zk ) n j=1 (z zj ) j=k n j=1 (z zj )

=
k=1

1 z zk

Now we do the integration using the residue theorem.

1 2

P (z) 1 dz = P (z) 2
n

n k=1

1 dz z zk 1 dz z zk

=
k=1

1 2

=
zk inside

1 2 1

1 dz z zk

=
zk inside

=m

Method 2: Fundamental Theorem of Calculus. We factor the polynomial, P (z) = c 696

n k=1 (z

zk ). Let m be

the number of roots, counting multiplicities, that lie inside the contour . 1 2 P (z) 1 dz = [log P (z)]C P (z) 2 = 1 log (z zk ) 2 k=1 1 2
n n

log(z zk )
k=1 C

The value of the logarithm changes by 2 for the terms in which zk is inside the contour. Its value does not change for the terms in which zk is outside the contour. 1 2

log(z zk )
zk inside C

1 = 2 =m Solution 13.8 1.

2
zk inside

ez dz = (z ) tan z

ez cos z dz (z ) sin z

The integrand has rst order poles at z = n, n Z, n = 1 and a double pole at z = . The only pole inside 697

the contour occurs at z = 0. We evaluate the integral with the residue theorem. ez cos z dz = 2 Res (z ) sin z ez cos z ,z = 0 (z ) sin z ez cos z = 2 lim z z=0 (z ) sin z z = 2 lim z=0 sin z 1 = 2 lim z=0 cos z = 2

ez dz = 2 (z ) tan z

2. The integrand has a rst order poles at z = 0, and a second order pole at z = inside the contour. The value of the integral is 2 times the sum of the residues at these points. From the previous part we know that residue at z = 0. ez cos z 1 Res ,z = 0 = (z ) sin z We nd the residue at z = with the residue formula. Res ez cos z , z = (z ) sin z ez cos z = lim (z + ) z (z ) sin z e (1) z+ = lim 2 z sin z e 1 = lim 2 z cos z e = 2

698

We nd the residue at z = by nding the rst few terms in the Laurent series of the integrand. ez cos z (e + e (z ) + O ((z )2 )) (1 + O ((z )2 )) = (z ) sin z (z ) ((z ) + O ((z )3 )) e e (z ) + O ((z )2 ) = (z )2 + O ((z )4 ) e e + z + O(1) (z)2 = 1 + O ((z )2 ) e e = + + O(1) 1 + O (z )2 (z )2 z e e = + + O(1) (z )2 z With this we see that Res The integral is ez cos z dz = 2 Res (z ) sin z ez cos z , z = + Res (z ) sin z ez cos z + Res ,z = (z ) sin z 1 e = 2 + e 2 ez dz = 2 e 2 e (z ) tan z ez cos z ,z = 0 (z ) sin z ez cos z ,z = (z ) sin z = e .

Cauchy Principal Value for Real Integrals


699

Solution 13.9 Consider the integral


1 1

1 dx. x 1 dx + lim 0+ x
0 1

By the denition of improper integrals we have


1 1

1 dx = lim 0+ x = lim +
0 0

1 dx x

[log |x|] 1

+ lim [log |x|]1 +

= lim log lim log + +


0

This limit diverges. Thus the integral diverges. Now consider the integral 1 dx 1 x where R, = 0. Since the integrand is bounded, the integral exists.
1 1 1

1 dx = x

x + dx 2 2 1 x + 1 = dx 2 + 2 1 x 1 = 2 dx 2 + 2 0 x 1/ 1 = 2 d 2+1 0 = 2 [arctan ]1/ 0 1 = 2 arctan 700

Figure 13.8: The real and imaginary part of the integrand for several values of . Note that the integral exists for all nonzero real and that
1 0

lim+

1 1

1 dx = x

and lim

1 dx = . 0 1 x The integral exists for arbitrarily close to zero, but diverges when = 0. The real part of the integrand is an odd function with two humps that get thinner and taller with decreasing . The imaginary part of the integrand is an even function with a hump that gets thinner and taller with decreasing . (See Figure 13.8.) 1 x Note that
0

=
1

x , x2 + 2

1 x

x2 + 2

1 dx + as 0+ x 1 dx as 0 . x 701

and

0 1

However,
1 0

lim

1 dx = 0 x

because the two integrals above cancel each other. Now note that when = 0, the integrand is real. Of course the integral doesnt converge for this case, but if we could assign some value to
1 1

1 dx x

it would be a real number. Since


1 0

lim

1 dx = 0, x

This number should be zero. Solution 13.10 1. 1 2 dx = lim 0+ 1 x = lim +


0 1 1

1 dx + x2

1 dx x2
1

1 x 1

1 + x 1 1

= lim +
0

11+

The principal value of the integral does not exist. 702

2.
1

1 dx = lim 0+ x3 = lim +
0

1 dx + x3

1 dx x3 1 2x2
1

1 2x2

+
1

= lim +
0

1 1 1 1 + + 2 2( )2 2(1)2 2(1)2 2

=0 3. Since f (x) is real analytic,

f (x) =
n=1

fn xn

for x (1, 1).

We can rewrite the integrand as f0 f1 f2 f (x) f0 f1 x f2 x2 f (x) = 3+ 2+ + . x3 x x x x3 Note that the nal term is real analytic on (1, 1). Thus the principal value of the integral exists if and only if f2 = 0.

Cauchy Principal Value for Contour Integrals


Solution 13.11 We can write f (z) as f (z) = f0 (z z0 )f (z) f0 + . z z0 z z0

Note that the second term is analytic in a neighborhood of z0 . Thus it is bounded on the contour. Let M be the 703

maximum modulus of

(zz0 )f (z)f0 zz0

on C . By using the maximum modulus integral bound, we have (z z0 )f (z) f0 dz ( ) M z z0 0 as 0+ . lim +


0

Thus we see that


C

f (z) dz lim +
0

f0 dz. z z0

We parameterize the path of integration with z = z0 + e , Now we evaluate the integral. lim +
0

(, ).

f0 dz = lim 0+ z z0 = lim +
0

f0 e d e f0 d

= ( )f0 ( ) Res(f (z), z0 ) This proves the result. Solution 13.12 Let Ci be the contour that is indented with circular arcs or radius at each of the rst order poles on C so as to enclose these poles. Let A1 , . . . , An be these circular arcs of radius centered at the points 1 , . . . , n . Let Cp be the contour, (not necessarily connected), obtained by subtracting each of the Aj s from Ci . Since the curve is C 1 , (or continuously dierentiable), at each of the rst order poles on C, the Aj s becomes semi-circles as 0+ . Thus f (z) dz = Res(f (z), j ) for j = 1, . . . , n.
Aj

704

CONTINUE Figure 13.9: The Indented Contour.

The principal value of the integral along C is f (z) dz = lim +


C 0

f (z) dz
Cp n

= lim +
0

f (z) dz
Ci m j=1 Aj n

f (z) dz
n

= 2
j=1

Res(f (z), zj ) +
j=1 m

Res(f (z), j )
n


j=1

Res(f (z), j )

f (z) dz = 2
C j=1

Res(f (z), zj ) +
j=1

Res(f (z), j ).

Solution 13.13 Consider 1 dz C z1 where C is the unit circle. Let Cp be the circular arc of radius 1 that starts and ends a distance of from z = 1. Let C be the negative, circular arc of radius with center at z = 1 that joins the endpoints of Cp . Let Ci , be the union of Cp and C . (Cp stands for Principal value Contour; Ci stands for Indented Contour.) Ci is an indented contour that avoids the rst order pole at z = 1. Figure 13.9 shows the three contours. Note that the principal value of the integral is
C

1 dz = lim 0+ z1 705

Cp

1 dz. z1

We can calculate the integral along Ci with Cauchys theorem. The integrand is analytic inside the contour. 1 dz = 0 z1 0+ , the contour becomes a semi-circle,

Ci

We can calculate the integral along C using Result 13.3.1. Note that as a circular arc of radians in the negative direction. lim +
0

1 dz = Res z1

1 ,1 z1

Now we can write the principal value of the integral along C in terms of the two known integrals.
C

1 1 dz = dz z1 Ci z 1 = 0 () =

1 dz z1

Integrals on the Real Axis


Solution 13.14 1. First we note that the integrand is an even function and extend the domain of integration.
0

x2 1 dx = 2 + 1)(x2 + 4) (x 2

x2 dx (x2 + 1)(x2 + 4)

Next we close the path of integration in the upper half plane. Consider the integral along the boundary of the 706

domain 0 < r < R, 0 < < . 1 2 z2 1 dz = 2 + 1)(z 2 + 4) (z 2 z2 dz C (z )(z + )(z 2)(z + 2) z2 1 = 2 Res ,z = 2 (z 2 + 1)(z 2 + 4) z2 + Res , z = 2 (z 2 + 1)(z 2 + 4) z2 z2 + 2 = (z + )(z 2 + 4) z= (z + 1)(z + 2) = 6 3 = 6
C

z=2

Let CR be the circular arc portion of the contour. as R with the maximum modulus bound.

R R

CR

. We show that the integral along CR vanishes

CR

z2 z2 dz R max 2 zCR (z + 1)(z 2 + 4) (z 2 + 1)(z 2 + 4) R2 = R 2 (R 1)(R2 4) 0 as R

We take the limit as R to evaluate the integral along the real axis. 1 R 2 lim
0

x2 dx = 2 + 1)(x2 + 4) 6 R (x 2 x dx = 2 + 1)(x2 + 4) (x 6 707

2. We close the path of integration in the upper half plane. Consider the integral along the boundary of the domain 0 < r < R, 0 < < . dz = (z + b)2 + a2 dz C (z + b a)(z + b + a) 1 = 2 Res , z = b + a (z + b a)(z + b + a) 1 = 2 z + b + a z=b+a = a
C

Let CR be the circular arc portion of the contour. as R with the maximum modulus bound.

R R

CR

. We show that the integral along CR vanishes

CR

dz 1 R max 2 + a2 zCR (z + b)2 + a2 (z + b) 1 = R (R b)2 + a2 0 as R

We take the limit as R to evaluate the integral along the real axis. dx = 2 + a2 R R (x + b) a dx = 2 + a2 a (x + b) lim Solution 13.15 Let CR be the semicircular arc from R to R in the upper half plane. Let C be the union of CR and the interval 708
R

[R, R]. We can evaluate the principal value of the integral along C with Result 13.3.2.
m n

f (x) dx = 2
C k=1

Res (f (z), zk ) +
k=1

Res(f (z), xk )

We examine the integral along CR as R . f (z) dz R max |f (z)|


CR zCR

0 as R . Now we are prepared to evaluate the real integral.


R

f (x) dx = lim f (x) dx


R R

= lim f (z) dz
R C m n

= 2
k=1

Res (f (z), zk ) +
k=1

Res(f (z), xk )

If we close the path of integration in the lower half plane, the contour will be in the negative direction.
m n

f (x) dx = 2
k=1

Res (f (z), zk )
k=1

Res(f (z), xk )

Solution 13.16 We consider

x2

2x dx. +x+1 709

With the change of variables x = 1/, this becomes

2 1 2 + 1 + 1

1 2

d,

2 1 d 2 + + 1 There are rst order poles at = 0 and = 1/2 3/2. We close the path of integration in the upper half plane with a semi-circle. Since the integrand decays like 3 the integrand along the semi-circle vanishes as the radius tends to innity. The value of the integral is thus 2z 1 2z 1 1 3 Res , z = 0 + 2 Res ,z = + z2 + z + 1 z2 + z + 1 2 2 lim 2 2+z+1 z

z0

+ 2

z(1+ 3)/2

lim

2z 1 z + (1 + 3)/2

2x 2 dx = x2 + x + 1 3

Solution 13.17 1. Consider


x4

1 dx. +1

The integrand

1 z 4 +1

is analytic on the real axis and has isolated singularities at the points z = {e/4 , e3/4 , e5/4 , e7/4 }.

Let CR be the semi-circle of radius R in the upper half plane. Since


R

lim

R max
zCR

z4

1 +1 710

= lim

R4

1 1

= 0,

we can apply Result 13.4.1.


x4

1 dx = 2 Res +1

z4

1 , e/4 +1

+ Res

z4

1 , e3/4 +1

The appropriate residues are, Res 1 , e/4 4+1 z z e/4 4 ze/4 z + 1 1 = lim 3 ze/4 4z 1 = e3/4 4 1 = , 4 2 = lim 1 4(e3/4 )3

Res

z4

1 , e3/4 +1

= =

1 /4 e 4 1 = , 4 2 We evaluate the integral with the residue theorem.


x4

1 dx = 2 +1

1 1 + 4 2 4 2

x4

1 dx = +1 2

711

2. Now consider

x2 dx. (x2 + 1)2

The integrand is analytic on the real axis and has second order poles at z = . Since the integrand decays suciently fast at innity, lim R max
zCR

z2 (z 2 + 1)2

= lim

R2 (R2 1)2

=0

we can apply Result 13.4.1.


x2 dx = 2 Res (x2 + 1)2

z2 ,z = (z 2 + 1)2

Res

z2 ,z = (z 2 + 1)2

d z2 (z )2 2 z dz (z + 1)2 d z2 = lim z dz (z + )2 (z + )2 2z z 2 2(z + ) = lim z (z + )4 = 4 = lim x2 dx = 2 + 1)2 (x 2

3. Since sin(x) 1 + x2 712

is an odd function,

cos(x) dx = 1 + x2

ex dx 1 + x2

Since ez /(1 + z 2 ) is analytic except for simple poles at z = and the integrand decays suciently fast in the upper half plane, ez 1 lim R max = lim R 2 =0 2 R zCR 1 + z R R 1 we can apply Result 13.4.1.

ex dx = 2 Res 1 + x2 e1 = 2 2

ez ,z = (z )(z + )

cos(x) dx = 2 1+x e

Solution 13.18 Consider the function f (z) = The value of the function on the imaginary axis: y 6 (y 4 + 1)2 is a constant multiple of the value of the function on the real axis: x6 . (x4 + 1)2 713 z6 . (z 4 + 1)2

Thus to evaluate the real integral we consider the path of integration, C, which starts at the origin, follows the real axis to R, follows a circular path to R and then follows the imaginary axis down to the origin. f (z) has second back order poles at the fourth roots of 1: (1 )/ 2. Of these only (1 + )/ 2 lies inside the path of integration. We evaluate the contour integral with the Residue Theorem. For R > 1:

z6 dz = 2 Res (z 4 + 1)2

z6 , z = e/4 (z 4 + 1)2 z6 d (z e/4 )2 4 = 2 lim (z + 1)2 ze/4 dz d z6 = 2 lim (z e3/4 )2 (z e5/4 )2 (z e7/4 )2 ze/4 dz = 2 lim
ze/4

z6 (z e3/4 )2 (z e5/4 )2 (z e7/4 )2

6 2 2 2 3/4 5/4 z ze ze z e7/4 6 2 2 2 2 2 = 2 (2)(4)(2) 1 + 2 2 + 2 2 3 = 2 (1 ) 2 32 3 = (1 + ) 8 2

The integral along the circular part of the contour, CR , vanishes as R . We demonstrate this with the maximum 714

modulus integral bound. R z6 z6 dz max (z 4 + 1)2 4 zCR (z 4 + 1)2 R R6 = 4 (R4 1)2 0 as R

CR

Taking the limit R , we have:


0 x6 (y)6 3 dx + dy = (1 + ) 4 2 (x4 + 1)2 8 2 ((y) + 1) 0 6 6 y 3 x dx + dy = (1 + ) 4 + 1)2 4 + 1)2 (x (y 8 2 0 0 6 x 3 (1 + ) dx = (1 + ) 4 + 1)2 (x 8 2 0 0

x6 3 dx = 4 + 1)2 (x 8 2

Fourier Integrals
Solution 13.19 We know that

eR sin d <
0

. R

715

First take the case that is positive and the semi-circle is in the upper half plane. f (z) ez dz
CR CR

ez dz max |f (z)|
zCR

eR e R e d max |f (z)|
zCR

=R
0

eR sin d max |f (z)|


zCR

max |f (z)| <R R zCR = max |f (z)| zCR 0 as R The procedure is almost the same for negative . Solution 13.20 First we write the integral in terms of Fourier integrals.

cos 2x dx = x

e2x dx + 2(x )

e2x dx 2(x )

1 Note that 2(z) vanishes as |z| . We close the former Fourier integral in the upper half plane and the latter in the lower half plane. There is a rst order pole at z = in the upper half plane.

e2x e2z dx = 2 Res , z = 2(z ) 2(x ) e2 = 2 2 There are no singularities in the lower half plane.

e2x dx = 0 2(x ) 716

Thus the value of the original real integral is


cos 2x dx = e2 x

Fourier Cosine and Sine Integrals


Solution 13.21 We are considering the integral sin x dx. x The integrand is an entire function. So it doesnt appear that the residue theorem would directly apply. Also the integrand is unbounded as x + and x , so closing the integral in the upper or lower half plane is not directly applicable. In order to proceed, we must write the integrand in a dierent form. Note that

cos x dx = 0 x

since the integrand is odd and has only a rst order pole at x = 0. Thus
x sin x e dx = dx. x x

Let CR be the semicircular arc in the upper half plane from R to R. Let C be the closed contour that is the union of CR and the real interval [R, R]. If we close the path of integration with a semicircular arc in the upper half plane, we have sin x ez ez dx = lim dz dz , R x C z CR z provided that all the integrals exist. The integral along CR vanishes as R by Jordans lemma. By the residue theorem for principal values we have ez dz = Res z 717 ez ,0 z = .

Combining these results,


sin x dx = . x

Solution 13.22 1 Note that (1 cos x)/x2 has a removable singularity at x = 0. The integral decays like x2 at innity, so the integral 2 exists. Since (sin x)/x is a odd function with a simple pole at x = 0, the principal value of its integral vanishes. sin x dx = 0 2 x 1 cos x 1 cos x sin x 1 ex dx = dx = dx x2 x2 x2

Let CR be the semi-circle of radius R in the upper half plane. Since lim R max
zCR

1 ez z2

= lim R
R

2 =0 R2

the integral along CR vanishes as R . 1 ez dz 0 as R z2

CR

We can apply Result 13.4.1.

1 ex dx = Res x2

1 ez ,z = 0 z2

1 ez ez = lim = lim z0 z0 z 1

1 cos x dx = x2

718

Solution 13.23 Consider


0

sin(x) dx. x(1 x2 )

Note that the integrand has removable singularities at the points x = 0, 1 and is an even function.
0

sin(x) 1 dx = 2) x(1 x 2

sin(x) dx. x(1 x2 )

Note that

cos(x) is an odd function with rst order poles at x = 0, 1. x(1 x2 ) cos(x) dx = 0 2 x(1 x ) ex sin(x) dx = dx. x(1 x2 ) 2 x(1 x2 )

Let CR be the semi-circle of radius R in the upper half plane. Since lim R max
zCR

ez z(1 z 2 )

= lim R
R

1 R(R2 1)

=0

the integral along CR vanishes as R . ez dz 0 as R z(1 z 2 ) 719

CR

We can apply Result 13.4.1. ex dx = 2 x(1 x2 ) 2 ez ez , z = 0 + Res ,z = 1 z(1 z 2 ) z(1 z 2 ) ez + Res , z = 1 z(1 z 2 ) ez ez ez lim lim + lim z0 1 z 2 z0 z(1 + z) z0 z(1 z) 1 1 1 + 2 2 Res
0

2 = 2

sin(x) dx = x(1 x2 )

Contour Integration and Branch Cuts


Solution 13.24 Let C be the boundary of the region < r < R, 0 < < . Choose the branch of the logarithm with a branch cut on the negative imaginary axis and the angle range /2 < < 3/2. We consider the integral of log2 z/(1 + z 2 ) on this contour. log2 z dz = 2 Res 1 + z2 log2 z ,z = 1 + z2 log2 z = 2 lim z z + (/2)2 = 2 2 3 = 4 720

Let CR be the semi-circle from R to R in the upper half plane. We show that the integral along CR vanishes as R with the maximum modulus integral bound. log2 z log2 z dz R max zCR 1 + z 2 1 + z2 ln2 R + 2 ln R + 2 R R2 1 0 as R Let C be the semi-circle from to in the upper half plane. We show that the integral along C vanishes as 0 with the maximum modulus integral bound. log2 z log2 z dz max zC 1 + z 2 1 + z2 ln2 2 ln + 2 1 2 0 as 0

CR

Now we take the limit as 0 and R for the integral along C. log2 z 3 dz = 2 4 C 1+z 2 0 ln r (ln r + )2 3 dr + dr = 1 + r2 1 + r2 4 0 2 ln x ln x 1 3 dx + 2 dx = 2 dx 1 + x2 1 + x2 1 + x2 4 0 0

2
0

(13.1)

We evaluate the integral of 1/(1 + x2 ) by extending the path of integration to ( . . . ) and closing the path of integration in the upper half plane. Since
R

lim

R max
zCR

1 1 + z2

lim

R2

1 1

= 0,

721

the integral of 1/(1 + z 2 ) along CR vanishes as R . We evaluate the integral with the Residue Theorem.

2 0

1 2 1 dx = dx 1 + x2 2 1 + x2 1 2 = 2 Res ,z = 2 1 + z2 1 = 3 lim z z + 3 = 2 ln2 x dx + 2 1 + x2


0

Now we return to Equation 13.1.

2
0

ln x 3 dx = 1 + x2 4

We equate the real and imaginary parts to solve for the desired integrals.
0

ln2 x 3 dx = 1 + x2 8 ln x dx = 0 1 + x2

Solution 13.25 We consider the branch of the function f (z) = z2 log z + 5z + 6

with a branch cut on the real axis and 0 < arg(z) < 2. Let C and CR denote the circles of radius and R where < 1 < R. C is negatively oriented; CR is positively oriented. Consider the closed contour, C, that is traced by a point moving from to R above the branch cut, next around CR back to R, then below the cut to , and nally around C back to . (See Figure 13.11.) 722

CR C

Figure 13.10: The path of integration.

We can evaluate the integral of f (z) along C with the residue theorem. For R > 3, there are rst order poles inside the path of integration at z = 2 and z = 3.

z2

log z dz = 2 Res + 5z + 6

log z , z = 2 + Res + 5z + 6 log z log z + lim = 2 lim z3 z + 2 z2 z + 3 log(2) log(3) + = 2 1 1 = 2 (log(2) + log(3) ) 2 = 2 log 3 z2 723

z2

log z , z = 3 + 5z + 6

In the limit as

0, the integral along C vanishes. We demonstrate this with the maximum modulus theorem. z2 log z log z dz 2 max 2 zC z + 5z + 6 + 5z + 6 2 log 2 65 2 0 as 0

In the limit as R , the integral along CR vanishes. We again demonstrate this with the maximum modulus theorem. z2 log z log z dz 2R max 2 zCR z + 5z + 6 + 5z + 6 log R + 2 2R 2 R 5R 6 0 as R

CR

Taking the limit as

0 and R , the integral along C is: log z dz = z 2 + 5z + 6


0 log x log x + 2 dx + dx 2 x2 + 5x + 6 0 x + 5x + 6 log x = 2 dx 2 + 5x + 6 x 0

Now we can evaluate the real integral.

2
0 0

x2

log x dx = 2 log + 5x + 6 3 2

2 3

log x dx = log x2 + 5x + 6

724

Solution 13.26 We consider the integral

I(a) =
0

xa dx. (x + 1)2

To examine convergence, we split the domain of integration.


0

xa dx = (x + 1)2

1 0

xa dx + (x + 1)2

xa dx (x + 1)2

First we work with the integral on (0 . . . 1).


1 0

xa dx (x + 1)2 =

1 0 1 0 1

xa |dx| (x + 1)2 x (a) dx (x + 1)2 x


(a)

dx

This integral converges for (a) > 1. Next we work with the integral on (1 . . . ).
1

xa dx (x + 1)2 =

1 1

xa |dx| (x + 1)2 x (a) dx (x + 1)2 x


(a)2

dx

This integral converges for

(a) < 1. 725

Thus we see that the integral dening I(a) converges in the strip, 1 < (a) < 1. The integral converges uniformly in any closed subset of this domain. Uniform convergence means that we can dierentiate the integral with respect to a and interchange the order of integration and dierentiation.

I (a) =
0

xa log x dx (x + 1)2

Thus we see that I(a) is analytic for 1 < (a) < 1. For 1 < (a) < 1 and a = 0, z a is multi-valued. Consider the branch of the function f (z) = z a /(z + 1)2 with a branch cut on the positive real axis and 0 < arg(z) < 2. We integrate along the contour in Figure 13.11. The integral on C vanishes as 0. We show this with the maximum modulus integral bound. First we write z a in modulus-argument form, z = e , where a = + . z a = ea log z = e(+)(ln = e ln = e e
+) +( ln +)

( log +)

Now we bound the integral. za za dz 2 max zC (z + 1)2 (z + 1)2 e2|| 2 (1 )2 0 as 0 726

The integral on CR vanishes as R . za za dz 2R max zCR (z + 1)2 (z + 1)2 R e2|| 2R (R 1)2 0 as R

CR

Above the branch cut, (z = r e0 ), the integrand is f (r e0 ) = Below the branch cut, (z = r e2 ), we have, f (r e2 ) = Now we use the residue theorem.
0

ra . (r + 1)2

e2a ra . (r + 1)2

ra dr + (r + 1)2

e2a ra za dr = 2 Res , 1 2 (z + 1)2 (r + 1) ra d a 1 e2a dr = 2 lim (z ) 2 z1 dz (r + 1) 0 ra a e(a1) dr = 2 (r + 1)2 1 e2a 0 a r 2a dr = a 2 e (r + 1) ea 0 xa a dx = for 1 < (a) < 1, a = 0 2 (x + 1) sin(a) 727

The right side has a removable singularity at a = 0. We use analytic continuation to extend the answer to a = 0.

I(a) =
0

xa dx = (x + 1)2

a sin(a)

for 1 < (a) < 1, a = 0 for a = 0 xa log2 x dx (x + 1)2 0 log2 x I (0) = dx (x + 1)2 0

We can derive the last two integrals by dierentiating this formula with respect to a and taking the limit a 0. I (a) = xa log x dx, (x + 1)2 0 log x I (0) = dx, (x + 1)2 0 I (a) =

We can nd I (0) and I (0) either by dierentiating the expression for I(a) or by nding the rst few terms in the Taylor series expansion of I(a) about a = 0. The latter approach is a little easier.

I(a) =
n=0

I (n) (0) n a n!

I(a) = =

a sin(a)

a a (a)3 /6 + O(a5 ) 1 = 2 /6 + O(a4 ) 1 (a) 2 a2 =1+ + O(a4 ) 6

I (0) =
0

log x dx = 0 (x + 1)2 log2 x 2 dx = (x + 1)2 3

I (0) =
0

728

Solution 13.27 1. We consider the integral

I(a) =
0

xa dx. 1 + x2

To examine convergence, we split the domain of integration.


0

xa dx = 1 + x2

1 0

xa dx + 1 + x2

xa dx 1 + x2

First we work with the integral on (0 . . . 1).


1 0

xa dx 1 + x2 =

1 0 1 0 1

xa |dx| 1 + x2 x (a) dx 1 + x2 x
(a)

dx

This integral converges for

(a) > 1.

Next we work with the integral on (1 . . . ).


1

xa dx 1 + x2 =

1 1

xa |dx| 1 + x2 x (a) dx 1 + x2 x
(a)2

dx

This integral converges for

(a) < 1. 729

CR C

Figure 13.11:

Thus we see that the integral dening I(a) converges in the strip, 1 < (a) < 1. The integral converges uniformly in any closed subset of this domain. Uniform convergence means that we can dierentiate the integral with respect to a and interchange the order of integration and dierentiation.

I (a) =
0

xa log x dx 1 + x2

Thus we see that I(a) is analytic for 1 < (a) < 1.

2. For 1 < (a) < 1 and a = 0, z a is multi-valued. Consider the branch of the function f (z) = z a /(1 + z 2 ) with a branch cut on the positive real axis and 0 < arg(z) < 2. We integrate along the contour in Figure 13.11. The integral on C vanishes are 0. We show this with the maximum modulus integral bound. First we write 730

z a in modulus-argument form, where z = e and a = + . z a = ea log z = e(+)(log +) = e log +( log +) = a e e( log +)

Now we bound the integral. za za dz 2 max zC 1 + z 2 1 + z2 e2|| 1 2 0 as 0 2 The integral on CR vanishes as R . za za dz 2R max zCR 1 + z 2 1 + z2 R e2|| 2R 2 R 1 0 as R

CR

Above the branch cut, (z = r e0 ), the integrand is f (r e0 ) = 731 ra . 1 + r2

Below the branch cut, (z = r e2 ), we have, e2a ra . 1 + r2

f (r e2 ) = Now we use the residue theorem.


0

ra dr + 1 + r2

e2a ra za za dr = 2 Res , + Res , 2 1 + z2 1 + z2 1+r xa za za 1 e2a dx = 2 lim + lim z z + z z 1 + x2 0 a/2 a e ea3/2 x 1 e2a dx = 2 + 1 + x2 2 2 0 ea/2 ea3/2 xa dx = 1 + x2 1 e2a 0 ea/2 (1 ea ) xa dx = 1 + x2 (1 + ea )(1 ea ) 0 xa dx = a/2 2 e 1+x + ea/2 0 xa dx = for 1 < (a) < 1, a = 0 2 1+x 2 cos(a/2) 0

We use analytic continuation to extend the answer to a = 0.

I(a) =
0

xa dx = 1 + x2 2 cos(a/2)

for 1 < (a) < 1

732

3. We can derive the last two integrals by dierentiating this formula with respect to a and taking the limit a 0. xa log x I (a) = dx, 1 + x2 0 log x I (0) = dx, 1 + x2 0

xa log2 x I (a) = dx 1 + x2 0 log2 x I (0) = dx 1 + x2 0

We can nd I (0) and I (0) either by dierentiating the expression for I(a) or by nding the rst few terms in the Taylor series expansion of I(a) about a = 0. The latter approach is a little easier.

I(a) =
n=0

I (n) (0) n a n!

I(a) =

2 cos(a/2) 1 = 2 /2 + O(a4 ) 2 1 (a/2) = 1 + (a/2)2 /2 + O(a4 ) 2 3 /8 2 = + a + O(a4 ) 2 2

I (0) =
0

log x dx = 0 1 + x2 log2 x 3 dx = 1 + x2 8

I (0) =
0

733

Solution 13.28 Convergence. If xa f (x)

x as x 0 for some > 1 then the integral


1

xa f (x) dx
0

will converge absolutely. If xa f (x)

x as x for some < 1 then the integral

xa f (x)
1

will converge absolutely. These are sucient conditions for the absolute convergence of

xa f (x) dx.
0

Contour Integration. We put a branch cut on the positive real axis and choose 0 < arg(z) < 2. We consider the integral of z a f (z) on the contour in Figure 13.11. Let the singularities of f (z) occur at z1 , . . . , zn . By the residue theorem,
n

z a f (z) dz = 2
C 1 k=1

Res (z a f (z), zk ) .

On the circle of radius , the integrand is o( ). Since the length of C is 2 , the integral on C vanishes as 0. On the circle of radius R, the integrand is o(R1 ). Since the length of CR is 2R, the integral on CR vanishes as R . The value of the integrand below the branch cut, z = x e2 , is f (x e2 ) = xa e2a f (x) In the limit as 0 and R we have
0 n

x f (x) dx +
0

x e

a 2a

f (x) dx = 2
k=1

Res (z a f (z), zk ) .

734

2 x f (x) dx = 1 e2a
a

Res (z a f (z), zk ) .
k=1

Solution 13.29 In the interval of uniform convergence of th integral, we can dierentiate the formula

xa f (x) dx =
0

2 1 e2a

Res (z a f (z), zk ) ,
k=1

with respect to a to obtain,


0

2 x f (x) log x dx = 1 e2a


a a

4 2 a e2a Res (z f (z) log z, zk ) , (1 e2a )2 k=1


a

Res (z a f (z), zk ) .
k=1 n

2 x f (x) log x dx = 1 e2a

2a Res (z f (z) log z, zk ) , + 2 sin (a) k=1


a

Res (z a f (z), zk ) ,
k=1

Dierentiating the solution of Exercise 13.26 m times with respect to a yields


0

m x f (x) log x dx = m a
a m

2 1 e2a

Res (z a f (z), zk ) ,
k=1

Solution 13.30 Taking the limit as a 0 Z in the solution of Exercise 13.26 yields

f (x) dx = 2 lim
0

n k=1

a0

Res (z a f (z), zk ) 1 e2a

The numerator vanishes because the sum of all residues of z n f (z) is zero. Thus we can use LHospitals rule.

f (x) dx = 2 lim
0

n k=1

a0

Res (z a f (z) log z, zk ) 2 e2a

735

f (x) dx =
0 k=1

Res (f (z) log z, zk )

This suggests that we could have derived the result directly by considering the integral of f (z) log z on the contour in Figure 13.11. We put a branch cut on the positive real axis and choose the branch arg z = 0. Recall that we have assumed that f (z) has only isolated singularities and no singularities on the positive real axis, [0, ). By the residue theorem,
n

f (z) log z dz = 2
C k=1

Res (f (z) log z, z = zk ) .

By assuming that f (z) z as z 0 where > 1 the integral on C will vanish as 0. By assuming that f (z) z as z where < 1 the integral on CR will vanish as R . The value of the integrand below the branch cut, z = x e2 is f (x)(log x + 2). Taking the limit as 0 and R , we have
0 n

f (x) log x dx +
0

f (x)(log x + 2) dx = 2
k=1

Res (f (z) log z, zk ) .

Thus we corroborate the result.


0

f (x) dx =
k=1

Res (f (z) log z, zk )

Solution 13.31 Consider the integral of f (z) log2 z on the contour in Figure 13.11. We put a branch cut on the positive real axis and choose the branch 0 < arg z < 2. Let z1 , . . . zn be the singularities of f (z). By the residue theorem,
n

f (z) log z dz = 2
C k=1

Res f (z) log2 z, zk .

If f (z) z as z 0 for some > 1 then the integral on C will vanish as 0. f (z) z as z for some < 1 then the integral on CR will vanish as R . Below the branch cut the integrand is f (x)(log x + 2)2 . 736

Thus we have
0 n

f (x) log x dx +
0

f (x)(log x + 4 log x 4 ) dx = 2
k=1 n 2 0 n

Res f (z) log2 z, zk .

4
0 0

f (x) log x dx + 4

f (x) dx = 2
k=1

Res f (z) log2 z, zk .


n

1 f (x) log x dx = 2

Res f (z) log z, zk +


k=1 k=1

Res (f (z) log z, zk )

Solution 13.32 Convergence. We consider


0 a

xa dx. 1 + x4

Since the integrand behaves like x near x = 0 we must have (a) > 1. Since the integrand behaves like xa4 at innity we must have (a 4) < 1. The integral converges for 1 < (a) < 3. Contour Integration. The function za f (z) = 1 + z4 has rst order poles at z = (1 )/ 2 and a branch point at z = 0. We could evaluate the real integral by putting a branch cut on the positive real axis with 0 < arg(z) < 2 and integrating f (z) on the contour in Figure 13.12. Integrating on this contour would work because the value of the integrand below the branch cut is a constant times the value of the integrand above the branch cut. After demonstrating that the integrals along C and CR vanish in the limits as 0 and R we would see that the value of the integral is a constant times the sum of the residues at the four poles. However, this is not the only, (and not the best), contour that can be used to evaluate the real integral. Consider the value of the integral on the line arg(z) = . f (r e ) = ra ea 1 + r4 e4

737

CR C

Figure 13.12: Possible path of integration for f (z) = If is a integer multiple of /2 then the integrand is a constant multiple of f (x) = ra . 1 + r4

za 1+z 4

Thus any of the contours in Figure 13.13 can be used to evaluate the real integral. The only dierence is how many residues we have to calculate. Thus we choose the rst contour in Figure 13.13. We put a branch cut on the negative real axis and choose the branch < arg(z) < to satisfy f (1) = 1. We evaluate the integral along C with the Residue Theorem. za dz = 2 Res 1 + z4 za 1+ ,z = 1 + z4 2

Let a = + and z = r e . Note that |z a | = |(r e )+ | = r e . 738

CR C C

CR C

CR

Figure 13.13: Possible Paths of Integration for f (z) =

za 1+z 4

The integral on C vanishes as 0. We demonstrate this with the maximum modulus integral bound.
C

za za dz max 1 + z4 2 zC 1 + z 4 e||/2 2 1 4 0 as 0 za R za dz max 1 + z4 2 zCR 1 + z 4 R R e||/2 2 R4 1 0 as R 739

The integral on CR vanishes as R .


CR

The value of the integrand on the positive imaginary axis, z = x e/2 , is (x e/2 )a xa ea/2 = . 1 + (x e/2 )4 1 + x4 We take the limit as 0 and R . xa dx + 1 + x4 0 xa ea/2 /2 za e dx = 2 Res , e/4 1 + x4 1 + z4 z a (z e/2 ) xa dx = 2 lim 1 e(a+1)/2 1 + x4 1 + z4 ze/4 0 2 az a (z e/2 ) + z a xa dx = lim 1 + x4 1 e(a+1)/2 ze/4 4z 3 0 ea/4 2 xa dx = 1 + x4 1 e(a+1)/2 4 e3/4 0 xa dx = 4 (a+1)/4 e(a+1)/4 ) 1+x 2(e 0
0 0

xa dx = csc 4 1+x 4

(a + 1) 4

Solution 13.33 Consider the branch of f (z) = z 1/2 log z/(z + 1)2 with a branch cut on the positive real axis and 0 < arg z < 2. We integrate this function on the contour in Figure 13.11. We use the maximum modulus integral bound to show that the integral on C vanishes as 0. z 1/2 log z z 1/2 log z dz 2 max C (z + 1)2 (z + 1)2 1/2 (2 log ) = 2 (1 )2 0 as 0 740

The integral on CR vanishes as R . z 1/2 log z z 1/2 log z dz 2R max CR (z + 1)2 (z + 1)2 R1/2 (log R + 2) (R 1)2 0 as R = 2R Above the branch cut, (z = x e0 ), the integrand is, f (x e0 ) = Below the branch cut, (z = x e2 ), we have, f (x e2 ) = x1/2 (log x + ) . (x + 1)2 x1/2 log x . (x + 1)2

CR

Taking the limit as 0 and R , the residue theorem gives us


0

x1/2 log x dx + (x + 1)2

x1/2 (log x + 2) dx = 2 Res (x + 1)2


0

z 1/2 log z , 1 . (z + 1)2

2
0

x1/2 log x dx + 2 (x + 1)2


0

x1/2 d 1/2 dx = 2 lim (z log z) z1 dz (x + 1)2 1 1/2 1 z log z + z 1/2 2 z 1 ()() 2

2
0

x1/2 log x dx + 2 (x + 1)2

x1/2 dx = 2 lim z1 (x + 1)2


0

2
0

x1/2 log x dx + 2 (x + 1)2

x1/2 dx = 2 (x + 1)2 741

2
0

x1/2 log x dx + 2 (x + 1)2

x1/2 dx = 2 + 2 2 (x + 1)

Equating real and imaginary parts,


0

x1/2 log x dx = , (x + 1)2

x1/2 dx = . (x + 1)2 2

Exploiting Symmetry
Solution 13.34 Convergence. The integrand, eaz eaz = , ez ez 2 sinh(z) has rst order poles at z = n, n Z. To study convergence, we split the domain of integration.
1 1

+
1

+
1

The principal value integral


1

eax dx ex ex

exists for any a because the integrand has only a rst order pole on the path of integration. Now consider the integral on (1 . . . ).
1

eax dx = ex ex

e(a1)x dx 1 e2x

1 1 e2

e(a1)x dx
1

This integral converges for a 1 < 0; a < 1. 742

Finally consider the integral on ( . . . 1).


1

eax dx = ex ex

e(a+1)x dx 1 e2x
1

1 1 e2

e(a+1)x dx

This integral converges for a + 1 > 0; a > 1. Thus we see that the integral for I(a) converges for real a, |a| < 1. Choice of Contour. Consider the contour C that is the boundary of the region: R < x < R, 0 < y < . The integrand has no singularities inside the contour. There are rst order poles on the contour at z = 0 and z = . The value of the integral along the contour is times the sum of these two residues. The integrals along the vertical sides of the contour vanish as R .
R+ R

eaz eaz dz max ez ez z(R...R+) ez ez eaR R e eR 0 as R

R+ R

eaz eaz dz max ez ez z(R...R+) ez ez eaR R e eR 0 as R 743

Evaluating the Integral. We take the limit as R and apply the residue theorem.

eax dx + ex ex

+ +

eaz dz ez ez = Res eaz , z = 0 + Res ez ez eaz , z = ez ez

eax dx + ex ex

(1 + ea )

ea(x+ z eaz (z ) eaz dz = lim + lim z0 2 sinh(z) z 2 sinh(z) ex+ ex az ax az az e +az e e +a(z ) eaz e dx = lim + lim z0 2 cosh(z) z ex ex 2 cosh(z) ea eax 1 (1 + ea ) dx = + x ex 2 2 e ax a e (1 e ) dx = x ex 2(1 + ea ) e eax (ea/2 ea/2 ) dx = x x 2 ea/2 + ea/2 e e

eax a dx = tan x ex e 2 2

Solution 13.35 1. dx 1 dx = 2 2 (1 + x2 )2 (1 + x2 ) 0 We apply Result 13.4.1 to the integral on the real axis. First we verify that the integrand vanishes fast enough in the upper half plane.
R

lim

R max
zCR

1 (1 + z 2 )2 744

= lim

1 (R2 1)2

=0

Then we evaluate the integral with the residue theorem.


dx = 2 Res (1 + x2 )2

1 ,z = (1 + z 2 )2 1 = 2 Res ,z = 2 (z + )2 (z ) 1 d = 2 lim z dz (z + )2 2 = 2 lim z (z + )3 = 2

dx = 2 )2 4 (1 + x

2. We wish to evaluate
0

dx . x3 + 1

Let the contour C be the boundary of the region 0 < r < R, 0 < < 2/3. We factor the denominator of the integrand to see that the contour encloses the simple pole at e/3 for R > 1. z 3 + 1 = (z e/3 )(z + 1)(z e/3 ) 745

We calculate the residue at that point. Res z3 1 , z = e/3 +1 = lim


ze/3

(z e/3 )

z3

1 +1

1 (z + 1)(z e/3 ) 1 = /3 (e +1)(e/3 e/3 ) e/3 = 3 We use the residue theorem to evaluate the integral. = lim
ze/3

dz 2 e/3 = z3 + 1 3
R 0

Let CR be the circular arc portion of the contour.


C

dz = 3+1 z

R 0

dx + 3+1 x
0

CR R

dz 3+1 z

e2/3 dx x3 + 1

= (1 + e/3 )

dx + x3 + 1

CR

dz z3 + 1

We show that the integral along CR vanishes as R with the maximum modulus integral bound.
CR

z3

2R 1 dz 0 as R +1 3 R3 1 2 e/3 dx = x3 + 1 3 0 dx 2 = 3+1 x 3 3 0 746

We take R and solve for the desired integral. 1 + e/3

Figure 13.14: The semi-circle contour. Solution 13.36 Method 1: Semi-Circle Contour. We wish to evaluate the integral

I=
0

dx . 1 + x6

We note that the integrand is an even function and express I as an integral over the whole real axis. I= 1 2

dx 1 + x6

Now we will evaluate the integral using contour integration. We close the path of integration in the upper half plane. Let R be the semicircular arc from R to R in the upper half plane. Let be the union of R and the interval [R, R]. (See Figure 13.14.) We can evaluate the integral along with the residue theorem. The integrand has rst order poles at z = e(1+2k)/6 , 747

k = 0, 1, 2, 3, 4, 5. Three of these poles are in the upper half plane. For R > 1, we have

1 dz = 2 Res z6 + 1 k=0
2

1 , e(1+2k)/6 z6 + 1 lim z e(1+2k)/6 z6 + 1

= 2
k=0

ze(1+2k)/6

Since the numerator and denominator vanish, we apply LHospitals rule.

= 2
k=0 2

lim
ze(1+2k)/6

1 6z 5

e5(1+2k)/6 3 k=0 5/6 e = + e15/6 + e25/6 3 5/6 e = + e/2 + e/6 3 3 3 + = 3 2 2 = = 2 3

Now we examine the integral along R . We use the maximum modulus integral bound to show that the value of the 748

integral vanishes as R . 1 1 dz R max 6 zR z + 1 z6 + 1 1 = R 6 R 1 0 as R .

Now we are prepared to evaluate the original real integral. 1 2 dz = 6 3 z +1 1 1 2 dx + dz = 6+1 6+1 x 3 R z

R R

We take the limit as R .


0

x6

1 2 dx = +1 3 1 dx = 6+1 x 3

We would get the same result by closing the path of integration in the lower half plane. Note that in this case the closed contour would be in the negative direction. Method 2: Wedge Contour. Consider the contour , which starts at the origin, goes to the point R along the real axis, then to the point R e/3 along a circle of radius R and then back to the origin along the ray = /3. (See Figure 13.15.) We can evaluate the integral along with the residue theorem. The integrand has one rst order pole inside the 749

Figure 13.15: The wedge contour. contour at z = e/6 . For R > 1, we have z6 1 dz = 2 Res +1 1 , e/6 +1 z e/6 = 2 lim 6 ze/6 z + 1 z6

Since the numerator and denominator vanish, we apply LHospitals rule. = 2 lim
ze/6

1 6z 5

5/6 e = 3 = e/3 3 Now we examine the integral along the circular arc, R . We use the maximum modulus integral bound to show that 750

the value of the integral vanishes as R . z6 R 1 1 dz max 6 +1 3 zR z + 1 R 1 = 3 R6 1 0 as R .

Now we are prepared to evaluate the original real integral. 1 dz = e/3 6 3 z +1 0 1 1 1 dx + dz + dz = e/3 6+1 6+1 6+1 x 3 R z R e/3 z
R

R 0 R 0

1 dx + x6 + 1

1 dz + z6 + 1

0 R

1 e/3 dx = e/3 x6 + 1 3

We take the limit as R . 1 e/3 1 dx = e/3 +1 3 0 /3 1 e dx = 6+1 x 3 1 e/3 0 1 (1 3)/2 dx = x6 + 1 3 1 (1 + 3)/2 1 dx = 6+1 x 3 0 x6

Solution 13.37 First note that cos(2) 1 4 , 751 0 . 4

4 Figure 13.16: cos(2) and 1

These two functions are plotted in Figure 13.16. To prove this inequality analytically, note that the two functions are equal at the endpoints of the interval and that cos(2) is concave downward on the interval, d2 cos(2) = 4 cos(2) 0 for 0 , d2 4 while 1 4/ is linear. Let CR be the quarter circle of radius R from = 0 to = /4. The integral along this contour vanishes as R . e
CR z 2 /4

dz
0 /4

e(R e R eR
0 /4

)2

R e d d d
/4 0

cos(2)

R eR

2 (14/)

= R

R2 (14/) e 4R2 2 = 1 eR 4R 0 as R 752

Let C be the boundary of the domain 0 < r < R, 0 < < /4. Since the integrand is analytic inside C the integral along C is zero. Taking the limit as R , the integral from r = 0 to along = 0 is equal to the integral from r = 0 to along = /4.
0 0 0 0

ex dx =
0

1+ x 2

1+ dx 2
2

1+ 2 ex dx = 2

ex dx

1+ 2 ex dx = 2

cos(x2 ) sin(x2 ) dx
0

1 2 ex dx = 2

cos(x2 ) dx +
0 0

sin(x2 ) dx +

cos(x2 ) dx
0 0

sin(x2 ) dx

We equate the imaginary part of this equation to see that the integrals of cos(x2 ) and sin(x2 ) are equal.

cos(x2 ) dx =
0 0

sin(x2 ) dx

The real part of the equation then gives us the desired identity.

cos(x2 ) dx =
0 0

1 sin(x2 ) dx = 2

ex dx

Solution 13.38 Consider the box contour C that is the boundary of the rectangle R x R, 0 y . There is a removable 753

singularity at z = 0 and a rst order pole at z = . By the residue theorem,


C

z z dz = Res , sinh z sinh z z(z ) = lim z sinh z 2z = lim z cosh z 2 =

The integrals along the side of the box vanish as R .


R+ R

z z dz max z[R,R+] sinh z sinh z R+ sinh R 0 as R

The value of the integrand on the top of the box is x + x + = . sinh(x + ) sinh x Taking the limit as R ,
x x + dx + dx = 2 . sinh x sinh x

Note that

1 dx = 0 sinh x as there is a rst order pole at x = 0 and the integrand is odd.


x 2 dx = sinh x 2

754

Solution 13.39 First we evaluate


eax dx. ex +1

Consider the rectangular contour in the positive direction with corners at R and R + 2. With the maximum modulus integral bound we see that the integrals on the vertical sides of the contour vanish as R .
R+2 R R R+2

eaR eaz dz 2 R 0 as R ez +1 e 1 eaz eaR dz 2 0 as R ez +1 1 eR

In the limit as R tends to innity, the integral on the rectangular contour is the sum of the integrals along the top and bottom sides.
eaz eax dz = dx + x ez +1 e +1 eaz dz = (1 e2a ) ez +1 C

ea(x+2) dx ex+2 +1 eax dx ex +1

The only singularity of the integrand inside the contour is a rst order pole at z = . We use the residue theorem to evaluate the integral. eaz dz = 2 Res ez +1 eaz , ez +1 (z ) eaz = 2 lim z ez +1 a(z ) eaz + eaz = 2 lim z ez = 2 ea 755

We equate the two results for the value of the contour integral. (1 e2a )

eax dx = 2 ea x e +1 eax 2 dx = a x +1 e e ea eax dx = x +1 e sin(a)

Now we derive the value of,

cosh(bx) dx. cosh x First make the change of variables x 2x in the previous result. e2ax 2 dx = 2x +1 sin(a) e (2a1)x e dx = x x sin(a) e + e Now we set b = 2a 1.

ebx dx = = cosh x sin((b + 1)/2) cos(b/2)


for 1 < b < 1

Since the cosine is an even function, we also have, ebx dx = cosh x cos(b/2) for 1 < b < 1

Adding these two equations and dividing by 2 yields the desired result.

cosh(bx) dx = cosh x cos(b/2)

for 1 < b < 1

756

Solution 13.40 Real-Valued Parameters. For b = 0, the integral has the value: /a2 . If b is nonzero, then we can write the integral as d 1 F (a, b) = 2 . b 0 (a/b + cos )2 We dene the new parameter c = a/b and the function,

G(c) = b2 F (a, b) =
0

d . (c + cos )2

If 1 c 1 then the integrand has a double pole on the path of integration. The integral diverges. Otherwise the integral exists. To evaluate the integral, we extend the range of integration to (0..2) and make the change of variables, z = e to integrate along the unit circle in the complex plane. G(c) = For this change of variables, we have, cos = z + z 1 , 2 d = dz . z 1 2
2 0

d (c + cos )2

G(c) =

dz/(z) 1 2 C (c + (z + z )/2) z = 2 dz 2 2 C (2cz + z + 1) z = 2 dz 2 1)2 (z + c c c2 1)2 C (z + c + 757

1 2

If c > 1, then c c2 1 is outside the unit circle and c + c2 1 is inside the unit circle. The integrand has a second order pole inside the path of integration. We evaluate the integral with the residue theorem.

G(c) = 22 Res

z , z = c + c2 1 (z + c + c2 1)2 (z + c c2 1)2 d z = 4 lim zc+ c2 1 dz (z + c + c2 1)2 1 2z = 4 lim zc+ c2 1 (z + c + c2 1)2 (z + c + c2 1)3 c + c2 1 z = 4 lim zc+ c2 1 (z + c + c2 1)3 2c = 4 (2 c2 1)3 c = 2 1)3 (c 758

If c < 1, then c

c2 1 is outside the unit circle. z G(c) = 22 Res , z = c c2 1 (z + c + c2 1)2 (z + c c2 1)2 d z = 4 lim zc c2 1 dz (z + c c2 1)2 1 2z = 4 lim zc c2 1 (z + c c2 1)2 (z + c c2 1)3 c c2 1 z = 4 lim zc c2 1 (z + c c2 1)3 2c = 4 (2 c2 1)3 c = 2 1)3 (c

c2 1 is inside the unit circle and c +

Thus we see that = (cc 3 2 1) G(c) = c 3 (c2 1) is divergent In terms of F (a, b), this is = (aa 2 )3 2 b F (a, b) = a 2 3 (a2 b ) is divergent Complex-Valued Parameters. Consider

for c > 1, for c < 1, for 1 c 1. for a/b > 1, for a/b < 1, for 1 a/b 1.

G(c) =
0

d , (c + cos )2

759

for complex c. Except for real-valued c between 1 and 1, the integral converges uniformly. We can interchange dierentiation and integration. The derivative of G(c) is G (c) = =
0

d dc

d 2 0 (c + cos ) 2 d (c + cos )3

Thus we see that G(c) is analytic in the complex plane with a cut on the real axis from 1 to 1. The value of the function on the positive real axis for c > 1 is G(c) = c (c2 1)3 .

We use analytic continuation to determine G(c) for complex c. By inspection we see that G(c) is the branch of (c2 c , 1)3/2

with a branch cut on the real axis from 1 to 1 and which is real-valued and positive for real c > 1. Using F (a, b) = G(c)/b2 we can determine F for complex-valued a and b. Solution 13.41 First note that

cos x dx = ex + ex

ex dx ex + ex

since sin x/(e +

ex

) is an odd function. For the function f (z) = ez ez + ez

we have f (x + ) =

ex ex = e x = e f (x). ex+ + ex e + ex 760

Thus we consider the integral ez dz ez + ez

where C is the box contour with corners at R and R + . We can evaluate this integral with the residue theorem. We can write the integrand as ez . 2 cosh z

We see that the integrand has rst order poles at z = (n + 1/2). The only pole inside the path of integration is at z = /2.

ez dz = 2 Res ez + ez

ez ,z = ez + ez 2 z (z /2) e = 2 lim ez + ez z/2 z e +(z /2) ez = 2 lim ez ez z/2 e/2 = 2 /2 e e/2 = e/2 761

The integrals along the vertical sides of the box vanish as R .


R+ R

ez ez dz max ez + ez z[R...R+] ez + ez 1 max R+y y[0...] e + eRy 1 max R y[0...] e + eR2y 1 = 2 sinh R 0 as R

Taking the limit as R , we have


ex dx + ex + ex

+ +

ez dz = e/2 ez + ez

(1 + e )

ex dx = e/2 x + ex e ex dx = /2 x + ex e e + e/2

Finally we have,

ex

cos x dx = /2 . x e + e/2 +e

Denite Integrals Involving Sine and Cosine


Solution 13.42 1. To evaluate the integral we make the change of variables z = e . The path of integration in the complex plane 762

is the positively oriented unit circle.


d = 1 + sin2 =

dz 1 1 )2 /4 z 1 (z z 4z dz 4 6z 2 + 1 z

4z dz 2 z1+ 2 z+1 2 z+1+ 2 C z 1 There are rst order poles at z = 1 2. The poles at z = 1 + 2 and z = 1 2 are inside the path of integration. We evaluate the integral with Cauchys Residue Formula. = z4 4z dz = 2 Res 6z 2 + 1 + Res = 8 4z , z = 1 + 2 2+1 6z 4z ,z = 1 2 z 4 6z 2 + 1 z4 2 z z1+ 2 z z+1 2 z+1+ 2
z=1+ 2

z1

z1

z+1+

z=1 2

1 1 = 8 8 2 8 2 = 2 2. First we use symmetry to expand the domain of integration.


/2

sin4 d =
0

1 4

sin4 d
0

763

Next we make the change of variables z = e . The path of integration in the complex plane is the positively oriented unit circle. We evaluate the integral with the residue theorem.

1 4

2 0

sin4 d =

dz 1 1 1 z 4 C 16 z z (z 2 1)4 1 = dz 64 C z5 6 4 1 = z 3 4z + 3 + 5 64 C z z z = 2 6 64 3 = 16

dz

Solution 13.43 1. Let C be the positively oriented unit circle about the origin. We parametrize this contour.

z = e ,

dz = e d, 764

(0 . . . 2)

We write sin and the dierential d in terms of z. Then we evaluate the integral with the Residue theorem.
2 0

1 d = 2 + sin

1 dz C 2 + (z 1/z)/(2) z 2 = dz 2 + 4z 1 C z 2 = dz 3 z+ 2 3 C z+ 2+ z+ 2 3 = 2 Res z + 2 + 3

, z = 2 +

2 = 2 2 3 2 = 3 2. First consider the case a = 0.

cos(n) d =

0 2

for n Z+ for n = 0

Now we consider |a| < 1, a = 0. Since sin(n) 1 2a cos + a2 is an even function,


cos(n) d = 1 2a cos + a2

en d 1 2a cos + a2

Let C be the positively oriented unit circle about the origin. We parametrize this contour. z = e , dz = e d, 765 ( . . . )

We write the integrand and the dierential d in terms of z. Then we evaluate the integral with the Residue theorem. en zn dz d = 2 2 z 1 2a cos + a C 1 a(z + 1/z) + a zn = dz 2 2 C az + (1 + a )z a zn dz = a C z 2 (a + 1/a)z + 1 zn = dz a C (z a)(z 1/a) zn = 2 Res ,z = a a (z a)(z 1/a) 2 an = a a 1/a 2an = 1 a2 We write the value of the integral for |a| < 1 and n Z0+ .

cos(n) d = 1 2a cos + a2

2
2an 1a2

for a = 0, n = 0 otherwise

Solution 13.44 Convergence. We consider the integral cos(n) sin(n) d = . sin 0 cos cos We assume that is real-valued. If is an integer, then the integrand has a second order pole on the path of integration, the principal value of the integral does not exist. If is real, but not an integer, then the integrand has a rst order pole on the path of integration. The integral diverges, but its principal value exists. I() = 766

Contour Integration. We will evaluate the integral for real, non-integer . I() = cos(n) d 0 cos cos 1 2 cos(n) d = 2 0 cos cos 2 en 1 d = 2 cos cos 0

We make the change of variables: z = e . I() = = Now we use the residue theorem. = zn , z = e (z e )(z e ) zn + Res , z = e e )(z e ) (z zn zn lim + lim ze z e ze z e en en + e e e e en en e e sin(n) sin() () Res 767 1 2 zn dz C (z + 1/z)/2 cos z z n dz ) C (z e )(z e

= = = =

I() =
0

cos(n) sin(n) d = . cos cos sin

Solution 13.45 Consider the integral x2 dx. 2 2 0 (1 + x ) 1 x We make the change of variables x = sin to obtain,
/2 0 1

sin2 (1 + sin2 ) 1 sin2


/2 0 /2 0

cos d

sin2 d 1 + sin2

1 cos(2) d 3 cos(2)
2

1 4 Now we make the change of variables z = e 1 4 4

1 cos d 3 cos

to obtain a contour integral on the unit circle. 1 (z + 1/z)/2 3 (z + 1/z)/2 z dz

(z 1)2 dz C z(z 3 + 2 2)(z 3 2 2) There are two rst order poles inside the contour. The value of the integral is 2 4 Res (z 1)2 , 0 + Res z(z 3 + 2 2)(z 3 2 2) 768 (z 1)2 ,z = 3 2 2 z(z 3 + 2 2)(z 3 2 2)

z0

lim

(z 1)2 (z 3 + 2 2)(z 3 2 2)

z32 2

lim

(z 1)2 z(z 3 2 2)

1 0

(2 2) x2 dx = 4 (1 + x2 ) 1 x2

Innite Sums
Solution 13.46 From Result 13.10.1 we see that the sum of the residues of cot(z)/z 4 is zero. This function has simples poles at nonzero integers z = n with residue 1/n4 . There is a fth order pole at z = 0. Finding the residue with the formula 1 d4 lim 4 (z cot(z)) 4! z0 dz would be a real pain. After doing the dierentiation, we would have to apply LHospitals rule multiple times. A better way of nding the residue is with the Laurent series expansion of the function. Note that 1 1 = sin(z) z (z)3 /6 + (z)5 /120 1 1 = 2 /6 + (z)4 /120 z 1 (z) = 1 z 1+ 2 2 4 4 z z + 6 120 + 2 2 4 4 z z + 6 120
2

769

Now we nd the z 1 term in the Laurent series expansion of cot(z)/z 4 . cos(z) = 4 z 4 sin(z) z = 1 z5 1 2 2 4 4 z + z 2 24 1 z 1+ 2 2 4 4 z z + 6 120 + 2 2 4 4 z z + 6 120
2

4 4 4 4 + + 120 36 12 24

z4 +

4 1 + 45 z
1

Thus the residue at z = 0 is 4 /45. Summing the residues, 4 1 1 + = 0. 4 n 45 n=1 n4 n=

n=1

4 1 = n4 90

Solution 13.47 For this problem we will use the following result: If
|z|

lim |zf (z)| = 0,

then the sum of all the residues of cot(z)f (z) is zero. If in addition, f (z) is analytic at z = n Z then

f (n) = ( sum of the residues of cot(z)f (z) at the poles of f (z) ).


n=

We assume that is not an integer, otherwise the sum is not dened. Consider f (z) = 1/(z 2 2 ). Since
|z|

lim z

z2

1 = 0, 2

770

and f (z) is analytic at z = n, n Z, we have

n=

n2

1 = ( sum of the residues of cot(z)f (z) at the poles of f (z) ). 2

f (z) has rst order poles at z = .

n=

n2

1 = Res 2 = lim

cot(z) , z = Res z 2 2

cot(z) , z = z 2 2

cot(z) cot(z) lim z z z+ z cot() cot() = 2 2

n=

n2

1 cot() = 2

771

Part IV Ordinary Dierential Equations

772

Chapter 14 First Order Dierential Equations


Dont show me your technique. Show me your heart. -Tetsuyasu Uekuma

14.1

Notation

A dierential equation is an equation involving a function, its derivatives, and independent variables. If there is only one independent variable, then it is an ordinary dierential equation. Identities such as d f 2 (x) = 2f (x)f (x), dx and dy dx =1 dx dy The following equations for y(x) are

are not dierential equations. The order of a dierential equation is the order of the highest derivative. rst, second and third order, respectively. y = xy 2 773

y + 3xy + 2y = x2 y =y y The degree of a dierential equation is the highest power of the highest derivative in the equation. The following equations are rst, second and third degree, respectively. y 3y 2 = sin x (y )2 + 2x cos y = ex (y )3 + y 5 = 0 An equation is said to be linear if it is linear in the dependent variable. y cos x + x2 y = 0 is a linear dierential equation. y + xy 2 = 0 is a nonlinear dierential equation. A dierential equation is homogeneous if it has no terms that are functions of the independent variable alone. Thus an inhomogeneous equation is one in which there are terms that are functions of the independent variables alone. y + xy + y = 0 is a homogeneous equation. y + y + x2 = 0 is an inhomogeneous equation. A rst order dierential equation may be written in terms of dierentials. Recall that for the function y(x) the dierential dy is dened dy = y (x) dx. Thus the dierential equations y = x2 y can be denoted: dy = x2 y dx and dy + xy 2 dx = sin(x) dx. 774 and y + xy 2 = sin(x)

A solution of a dierential equation is a function which when substituted into the equation yields an identity. For example, y = x ln |x| is a solution of y y = 1. x We verify this by substituting it into the dierential equation. ln |x| + 1 ln |x| = 1 We can also verify that y = c ex is a solution of y y = 0 for any value of the parameter c. c ex c ex = 0

14.2

Example Problems

In this section we will discuss physical and geometrical problems that lead to rst order dierential equations.

14.2.1

Growth and Decay

Example 14.2.1 Consider a culture of bacteria in which each bacterium divides once per hour. Let n(t) N denote the population, let t denote the time in hours and let n0 be the population at time t = 0. The population doubles every hour. Thus for integer t, the population is n0 2t . Figure 14.1 shows two possible populations when there is initially a single bacterium. In the rst plot, each of the bacteria divide at times t = m for m N . In the second plot, they divide at times t = m 1/2. For both plots the population is 2t for integer t. We model this problem by considering a continuous population y(t) R which approximates the discrete population. In Figure 14.2 we rst show the population when there is initially 8 bacteria. The divisions of bacteria is spread out over each one second interval. For integer t, the populations is 8 2t . Next we show the population with a plot of the continuous function y(t) = 8 2t . We see that y(t) is a reasonable approximation of the discrete population. In the discrete problem, the growth of the population is proportional to its number; the population doubles every hour. For the continuous problem, we assume that this is true for y(t). We write this as an equation: y (t) = y(t). 775

16 12 8 4 1 2 3 4

16 12 8 4 1 2 3 4

Figure 14.1: The population of bacteria.


128 96 64 32 1 2 3 4 128 96 64 32 1 2 3 4

Figure 14.2: The discrete population of bacteria and a continuous population approximation. That is, the rate of change y (t) in the population is proportional to the population y(t), (with constant of proportionality ). We specify the population at time t = 0 with the initial condition: y(0) = n0 . Note that y(t) = n0 et satises the problem: y (t) = y(t), For our bacteria example, = ln 2. y(0) = n0 .

Result 14.2.1 A quantity y(t) whose growth or decay is proportional to y(t) is modelled by the problem: y (t) = y(t), y(t0 ) = y0 . Here we assume that the quantity is known at time t = t0 . e is the factor by which the quantity grows/decays in unit time. The solution of this problem is y(t) = y0 e(tt0 ) .

776

14.3

One Parameter Families of Functions


F (x, y(x), c) = 0, (14.1)

Consider the equation: which implicitly denes a one-parameter family of functions y(x; c). Here y is a function of the variable x and the parameter c. For simplicity, we will write y(x) and not explicitly show the parameter dependence. Example 14.3.1 The equation y = cx denes family of lines with slope c, passing through the origin. The equation x2 + y 2 = c2 denes circles of radius c, centered at the origin. Consider a chicken dropped from a height h. The elevation y of the chicken at time t after its release is y(t) = hgt2 , where g is the acceleration due to gravity. This is family of functions for the parameter h. It turns out that the general solution of any rst order dierential equation is a one-parameter family of functions. This is not easy to prove. However, it is easy to verify the converse. We dierentiate Equation 14.1 with respect to x. Fx + Fy y = 0 (We assume that F has a non-trivial dependence on y, that is Fy = 0.) This gives us two equations involving the independent variable x, the dependent variable y(x) and its derivative and the parameter c. If we algebraically eliminate c between the two equations, the eliminant will be a rst order dierential equation for y(x). Thus we see that every one-parameter family of functions y(x) satises a rst order dierential equation. This y(x) is the primitive of the dierential equation. Later we will discuss why y(x) is the general solution of the dierential equation. Example 14.3.2 Consider the family of circles of radius c centered about the origin. x2 + y 2 = c2 Dierentiating this yields: 2x + 2yy = 0. It is trivial to eliminate the parameter and obtain a dierential equation for the family of circles. x + yy = 0 777

y = x/y y x

Figure 14.3: A circle and its tangent. We can see the geometric meaning in this equation by writing it in the form: x y = . y For a point on the circle, the slope of the tangent y is the negative of the cotangent of the angle x/y. (See Figure 14.3.)

Example 14.3.3 Consider the one-parameter family of functions: y(x) = f (x) + cg(x), where f (x) and g(x) are known functions. The derivative is y = f + cg . 778

We eliminate the parameter. gy g y = gf g f g gf y y=f g g Thus we see that y(x) = f (x)+cg(x) satises a rst order linear dierential equation. Later we will prove the converse: the general solution of a rst order linear dierential equation has the form: y(x) = f (x) + cg(x).

We have shown that every one-parameter family of functions satises a rst order dierential equation. We do not prove it here, but the converse is true as well.

Result 14.3.1 Every rst order dierential equation has a one-parameter family of solutions y(x) dened by an equation of the form: F (x, y(x); c) = 0. This y(x) is called the general solution. If the equation is linear then the general solution expresses the totality of solutions of the dierential equation. If the equation is nonlinear, there may be other special singular solutions, which do not depend on a parameter.

This is strictly an existence result. It does not say that the general solution of a rst order dierential equation can be determined by some method, it just says that it exists. There is no method for solving the general rst order dierential equation. However, there are some special forms that are soluble. We will devote the rest of this chapter to studying these forms. 779

14.4

Integrable Forms

In this section we will introduce a few forms of dierential equations that we may solve through integration.

14.4.1

Separable Equations

Any dierential equation that can written in the form P (x) + Q(y)y = 0 is a separable equation, (because the dependent and independent variables are separated). We can obtain an implicit solution by integrating with respect to x. P (x) dx + P (x) dx + Q(y) dy dx = c dx

Q(y) dy = c

Result 14.4.1 The separable equation P (x) + Q(y)y = 0 may be solved by integrating with respect to x. The general solution is P (x) dx + Q(y) dy = c.

Example 14.4.1 Consider the dierential equation y = xy 2 . We separate the dependent and independent variables 780

and integrate to nd the solution.

dy = xy 2 dx y 2 dy = x dx y 2 dy = y 1 = x dx + c

x2 +c 2 1 y= 2 x /2 + c

Example 14.4.2 The equation y = y y 2 is separable.

y =1 y y2

We expand in partial fractions and integrate.

1 1 y =1 y y1 ln |y| ln |y 1| = x + c 781

We have an implicit equation for y(x). Now we solve for y(x). ln y =x+c y1 y = ex+c y1 y = ex+c y1 y = c ex 1 y1 c ex y= x c e 1 1 y= 1 + c ex

14.4.2

Exact Equations
P (x, y) dx + Q(x, y) dy = 0.

Any rst order ordinary dierential equation of the rst degree can be written as the total dierential equation,

If this equation can be integrated directly, that is if there is a primitive, u(x, y), such that du = P dx + Q dy, then this equation is called exact. The (implicit) solution of the dierential equation is u(x, y) = c, where c is an arbitrary constant. Since the dierential of a function, u(x, y), is du u u dx + dy, x y 782

P and Q are the partial derivatives of u: P (x, y) = In an alternate notation, the dierential equation P (x, y) + Q(x, y) is exact if there is a primitive u(x, y) such that du u u dy dy + = P (x, y) + Q(x, y) . dx x y dx dx The solution of the dierential equation is u(x, y) = c. Example 14.4.3 x+y is an exact dierential equation since d dx The solution of the dierential equation is 1 2 (x + y 2 ) = c. 2 Example 14.4.4 , Let f (x) and g(x) be known functions. g(x)y + g (x)y = f (x) 783 1 2 (x + y 2 ) 2 =x+y dy dx dy =0 dx dy = 0, dx (14.2) u , x Q(x, y) = u . y

is an exact dierential equation since d (g(x)y(x)) = gy + g y. dx The solution of the dierential equation is g(x)y(x) = y(x) = 1 g(x) f (x) dx + c f (x) dx + c . g(x)

A necessary condition for exactness. The solution of the exact equation P + Qy = 0 is u = c where u is the primitive of the equation, du = P + Qy . At present the only method we have for determining the primitive is dx guessing. This is ne for simple equations, but for more dicult cases we would like a method more concrete than divine inspiration. As a rst step toward this goal we determine a criterion for determining if an equation is exact. Consider the exact equation, P + Qy = 0, with primitive u, where we assume that the functions P and Q are continuously dierentiable. Since the mixed partial derivatives of u are equal, 2u 2u = , xy yx a necessary condition for exactness is P Q = . y x A sucient condition for exactness. This necessary condition for exactness is also a sucient condition. We demonstrate this by deriving the general solution of (14.2). Assume that P + Qy = 0 is not necessarily exact, but satises the condition Py = Qx . If the equation has a primitive, du u u dy dy + = P (x, y) + Q(x, y) , dx x y dx dx 784

then it satises

u = P, x
x

u = Q. y

(14.3)

Integrating the rst equation of (14.3), we see that the primitive has the form u(x, y) =
x0

P (, y) d + f (y),

for some f (y). Now we substitute this form into the second equation of (14.3). u = Q(x, y) y
x

Py (, y) d + f (y) = Q(x, y)
x0

Now we use the condition Py = Qx .


x

Qx (, y) d + f (y) = Q(x, y)
x0

Q(x, y) Q(x0 , y) + f (y) = Q(x, y) f (y) = Q(x0 , y)


y

f (y) =
y0

Q(x0 , ) d
y

Thus we see that u=

P (, y) d +
x0 y0

Q(x0 , ) d

is a primitive of the derivative; the equation is exact. The solution of the dierential equation is
x y

P (, y) d +
x0 y0

Q(x0 , ) d = c.

785

Even though there are three arbitrary constants: x0 , y0 and c, the solution is a one-parameter family. This is because changing x0 or y0 only changes the left side by an additive constant.

Result 14.4.2 Any rst order dierential equation of the rst degree can be written in the form dy P (x, y) + Q(x, y) = 0. dx This equation is exact if and only if Py = Qx . In this case the solution of the dierential equation is given by
x y

P (, y) d +
x0 y0

Q(x0 , ) d = c.

Exercise 14.1 Solve the following dierential equations by inspection. That is, group terms into exact derivatives and then integrate. f (x) and g(x) are known functions. 1.
y (x) y(x)

= f (x)

2. y (x)y (x) = f (x) 3.


y cos x x + y tan x = cos x cos

Hint, Solution

14.4.3

Homogeneous Coecient Equations

Homogeneous coecient, rst order dierential equations form another class of soluble equations. We will nd that a change of dependent variable will make such equations separable or we can determine an integrating factor that will 786

make such equations exact. First we dene homogeneous functions. Eulers Theorem on Homogeneous Functions. The function F (x, y) is homogeneous of degree n if F (x, y) = n F (x, y). From this denition we see that F (x, y) = xn F 1, (Just formally substitute 1/x for .) For example, x2 y + 2y 3 , x+y are homogeneous functions of orders 3, 2 and 1, respectively. Eulers theorem for a homogeneous function of order n is: xy 2 , x cos(y/x) y . x

xFx + yFy = nF. To prove this, we dene = x, = y. From the denition of homogeneous functions, we have F (, ) = n F (x, y). We dierentiate this equation with respect to . F (, ) F (, ) + = nn1 F (x, y) xF + yF = nn1 F (x, y) Setting = 1, (and hence = x, = y), proves Eulers theorem.

Result 14.4.3 Eulers Theorem on Homogeneous Functions. If F (x, y) is a homogeneous function of degree n, then xFx + yFy = nF.

787

Homogeneous Coecient Dierential Equations. If the coecient functions P (x, y) and Q(x, y) are homogeneous of degree n then the dierential equation, P (x, y) + Q(x, y) dy = 0, dx (14.4)

is called a homogeneous coecient equation. They are often referred to simply as homogeneous equations. Transformation to a Separable Equation. We can write the homogeneous equation in the form, xn P 1, y dy y + xn Q 1, = 0, x x dx y dy y P 1, + Q 1, = 0. x x dx
y(x) . x

This suggests the change of dependent variable u(x) =

P (1, u) + Q(1, u) u + x This equation is separable.

du dx

=0

P (1, u) + uQ(1, u) + xQ(1, u)

du =0 dx

1 Q(1, u) du + =0 x P (1, u) + uQ(1, u) dx 1 ln |x| + du = c u + P (1, u)/Q(1, u) By substituting ln |c| for c, we can write this in a simpler form. 1 c du = ln . u + P (1, u)/Q(1, u) x 788

Integrating Factor. One can show that 1 xP (x, y) + yQ(x, y)

(x, y) =

is an integrating factor for the Equation 14.4. The proof of this is left as an exercise for the reader. (See Exercise 14.2.)

Result 14.4.4 Homogeneous Coecient Dierential Equations. If P (x, y) and Q(x, y) are homogeneous functions of degree n, then the equation P (x, y) + Q(x, y) dy =0 dx

is made separable by the change of independent variable u(x) = y(x) . The solution is deterx mined by 1 c du = ln . u + P (1, u)/Q(1, u) x Alternatively, the homogeneous equation can be made exact with the integrating factor (x, y) = 1 . xP (x, y) + yQ(x, y)

Example 14.4.5 Consider the homogeneous coecient equation dy = 0. dx

x2 y 2 + xy 789

The solution for u(x) = y(x)/x is determined by 1 u+


1u2 u

du = ln c x

c x

u du = ln

1 2 c u = ln 2 x u = 2 ln |c/x| Thus the solution of the dierential equation is y = x Exercise 14.2 Show that (x, y) = 1 xP (x, y) + yQ(x, y) 2 ln |c/x|

is an integrating factor for the homogeneous equation, P (x, y) + Q(x, y) Hint, Solution Exercise 14.3 (mathematica/ode/rst order/exact.nb) Find the general solution of the equation dy y y =2 + dt t t Hint, Solution 790 dy = 0. dx

14.5
14.5.1

The First Order, Linear Dierential Equation


Homogeneous Equations
dy + p(x)y = 0. dx

The rst order, linear, homogeneous equation has the form

Note that if we can nd one solution, then any constant times that solution also satises the equation. If fact, all the solutions of this equation dier only by multiplicative constants. We can solve any equation of this type because it is separable. y = p(x) y ln |y| = y = e y = c e p(x) dx + c
p(x) dx+c p(x) dx

Result 14.5.1 First Order, Linear Homogeneous Dierential Equations. The rst order, linear, homogeneous dierential equation, dy + p(x)y = 0, dx has the solution y = c e The solutions dier by multiplicative constants.
p(x) dx

(14.5)

791

Example 14.5.1 Consider the equation dy 1 + y = 0. dx x We use Equation 14.5 to determine the solution. y(x) = c e
1/x dx

for x = 0

y(x) = c e y(x) =

ln |x|

c |x| c y(x) = x

14.5.2

Inhomogeneous Equations
dy + p(x)y = f (x). dx

The rst order, linear, inhomogeneous dierential equation has the form (14.6)

This equation is not separable. Note that it is similar to the exact equation we solved in Example 14.4.4, g(x)y (x) + g (x)y(x) = f (x). To solve Equation 14.6, we multiply by an integrating factor. Multiplying a dierential equation by its integrating factor changes it to an exact equation. Multiplying Equation 14.6 by the function, I(x), yields, I(x) dy + p(x)I(x)y = f (x)I(x). dx

In order that I(x) be an integrating factor, it must satisfy d I(x) = p(x)I(x). dx 792

This is a rst order, linear, homogeneous equation with the solution I(x) = c e
p(x) dx

This is an integrating factor for any constant c. For simplicity we will choose c = 1.

To solve Equation 14.6 we multiply by the integrating factor and integrate. Let P (x) = eP (x) dy + p(x) eP (x) y = eP (x) f (x) dx d P (x) e y = eP (x) f (x) dx eP (x) f (x) dx + c eP (x) y yp + c yh

p(x) dx.

y = eP (x)

Note that the general solution is the sum of a particular solution, yp , that satises y + p(x)y = f (x), and an arbitrary constant times a homogeneous solution, yh , that satises y + p(x)y = 0. Example 14.5.2 Consider the dierential equation 1 y + y = x2 , x First we nd the integrating factor. I(x) = exp 1 dx x 793 = eln x = x x > 0.

10 5 -1 -5 -10
Figure 14.4: Solutions to y + y/x = x2 . We multiply by the integrating factor and integrate. d (xy) = x3 dx 1 xy = x4 + c 4 1 c y = x3 + . 4 x The particular and homogeneous solutions are 1 yp = x3 4 and yh = 1 . x

Note that the general solution to the dierential equation is a one-parameter family of functions. The general solution is plotted in Figure 14.4 for various values of c.

794

Exercise 14.4 (mathematica/ode/rst order/linear.nb) Solve the dierential equation 1 y y = x , x Hint, Solution

x > 0.

14.5.3

Variation of Parameters.

We could also have found the particular solution with the method of variation of parameters. Although we can solve rst order equations without this method, it will become important in the study of higher order inhomogeneous equations. We begin by assuming that the particular solution has the form yp = u(x)yh (x) where u(x) is an unknown function. We substitute this into the dierential equation. d yp + p(x)yp = f (x) dx d (uyh ) + p(x)uyh = f (x) dx u yh + u(yh + p(x)yh ) = f (x) Since yh is a homogeneous solution, yh + p(x)yh = 0. u = u= Recall that the homogeneous solution is yh = eP (x) . u= eP (x) f (x) dx f (x) yh f (x) dx yh (x)

795

Thus the particular solution is yp = eP (x) eP (x) f (x) dx.

14.6

Initial Conditions

In physical problems involving rst order dierential equations, the solution satises both the dierential equation and a constraint which we call the initial condition. Consider a rst order linear dierential equation subject to the initial condition y(x0 ) = y0 . The general solution is y = yp + cyh = eP (x) eP (x) f (x) dx + c eP (x) .

For the moment, we will assume that this problem is well-posed. A problem is well-posed if there is a unique solution to the dierential equation that satises the constraint(s). Recall that eP (x) f (x) dx denotes any integral of eP (x) f (x). x For convenience, we choose x0 eP () f () d. The initial condition requires that y(x0 ) = y0 = eP (x0 )
x0 x0

eP () f () d + c eP (x0 ) = c eP (x0 ) .

Thus c = y0 eP (x0 ) . The solution subject to the initial condition is


x

y = eP (x)
x0

eP () f () d + y0 eP (x0 )P (x) .

Example 14.6.1 Consider the problem y + (cos x)y = x, From Result 14.6.1, the solution subject to the initial condition is
x

y(0) = 2.

y=e

sin x 0

esin d + 2 e sin x .

796

14.6.1

Piecewise Continuous Coecients and Inhomogeneities


dy + p(x)y = f (x) dx

If the coecient function p(x) and the inhomogeneous term f (x) in the rst order linear dierential equation

are continuous, then the solution is continuous and has a continuous rst derivative. To see this, we note that the solution y = eP (x) eP (x) f (x) dx + c eP (x) is continuous since the integral of a piecewise continuous function is continuous. The rst derivative of the solution can be found directly from the dierential equation. y = p(x)y + f (x) Since p(x), y, and f (x) are continuous, y is continuous. If p(x) or f (x) is only piecewise continuous, then the solution will be continuous since the integral of a piecewise continuous function is continuous. The rst derivative of the solution will be piecewise continuous. Example 14.6.2 Consider the problem y y = H(x 1), where H(x) is the Heaviside function. H(x) = 1 0 for x > 0, for x < 0. y(0) = 1,

To solve this problem, we divide it into two equations on separate domains. y1 y1 = 0, y2 y2 = 1, y1 (0) = 1, y2 (1) = y1 (1), 797 for x < 1 for x > 1

8 6 4 2 -1 1 2

Figure 14.5: Solution to y y = H(x 1). With the condition y2 (1) = y1 (1) on the second equation, we demand that the solution be continuous. The solution to the rst equation is y = ex . The solution for the second equation is
x

y = ex
1

e d + e1 ex1 = 1 + ex1 + ex .

Thus the solution over the whole domain is y= The solution is graphed in Figure 14.5. Example 14.6.3 Consider the problem, y + sign(x)y = 0, 798 y(1) = 1. ex (1 + e1 ) ex 1 for x < 1, for x > 1.

Recall that 1 sign x = 0 1 Since sign x is piecewise dened, we solve the two problems, y+ + y+ = 0, y y = 0, and dene the solution, y, to be y(x) = y+ (x), y (x), for x 0, for x 0. y+ (1) = 1, y (0) = y+ (0), for x > 0 for x < 0, for x < 0 for x = 0 for x > 0.

The initial condition for y demands that the solution be continuous. Solving the two problems for positive and negative x, we obtain e1x , e1+x , for x > 0, for x < 0.

y(x) =

This can be simplied to y(x) = e1|x| . This solution is graphed in Figure 14.6. 799

2 1

-3

-2

-1

Figure 14.6: Solution to y + sign(x)y = 0.

Result 14.6.1 Existence, Uniqueness Theorem. Let p(x) and f (x) be piecewise continuous on the interval [a, b] and let x0 [a, b]. Consider the problem, dy + p(x)y = f (x), dx The general solution of the dierential equation is y = eP (x) eP (x) f (x) dx + c eP (x) . y(x0 ) = y0 .

The unique, continuous solution of the dierential equation subject to the initial condition is
x

y=e where P (x) = p(x) dx.

P (x) x0

eP () f () d + y0 eP (x0 )P (x) ,

800

Exercise 14.5 (mathematica/ode/rst order/exact.nb) Find the solutions of the following dierential equations which satisfy the given initial conditions: 1. 2. dy + xy = x2n+1 , dx dy 2xy = 1, dx y(1) = 1, nZ

y(0) = 1

Hint, Solution Exercise 14.6 (mathematica/ode/rst order/exact.nb) Show that if > 0 and > 0, then for any real , every solution of dy + y(x) = ex dx satises limx+ y(x) = 0. (The case = requires special treatment.) Find the solution for = = 1 which satises y(0) = 1. Sketch this solution for 0 x < for several values of . In particular, show what happens when 0 and . Hint, Solution

14.7

Well-Posed Problems
1 y y = 0, x y(0) = 1.

Example 14.7.1 Consider the problem,

The general solution is y = cx. Applying the initial condition demands that 1 = c 0, which cannot be satised. The general solution for various values of c is plotted in Figure 14.7.

801

-1

-1

Figure 14.7: Solutions to y y/x = 0. Example 14.7.2 Consider the problem 1 1 y y= , x x The general solution is y = 1 + cx. The initial condition is satised for any value of c so there are an innite number of solutions. Example 14.7.3 Consider the problem 1 y + y = 0, y(0) = 1. x c The general solution is y = x . Depending on whether c is nonzero, the solution is either singular or zero at the origin and cannot satisfy the initial condition. y(0) = 1.

802

The above problems in which there were either no solutions or an innite number of solutions are said to be ill-posed. If there is a unique solution that satises the initial condition, the problem is said to be well-posed. We should have suspected that we would run into trouble in the above examples as the initial condition was given at a singularity of the coecient function, p(x) = 1/x. Consider the problem, y + p(x)y = f (x), y(x0 ) = y0 . We assume that f (x) bounded in a neighborhood of x = x0 . The dierential equation has the general solution, y = eP (x) eP (x) f (x) dx + c eP (x) .

If the homogeneous solution, eP (x) , is nonzero and nite at x = x0 , then there is a unique value of c for which the initial condition is satised. If the homogeneous solution vanishes at x = x0 then either the initial condition cannot be satised or the initial condition is satised for all values of c. The homogeneous solution can vanish or be innite only if P (x) as x x0 . This can occur only if the coecient function, p(x), is unbounded at that point.

Result 14.7.1 If the initial condition is given where the homogeneous solution to a rst order, linear dierential equation is zero or innite then the problem may be ill-posed. This may occur only if the coecient function, p(x), is unbounded at that point.

14.8
14.8.1

Equations in the Complex Plane


Ordinary Points

Consider the rst order homogeneous equation dw + p(z)w = 0, dz 803

where p(z), a function of a complex variable, is analytic in some domain D. The integrating factor, I(z) = exp p(z) dz ,

is an analytic function in that domain. As with the case of real variables, multiplying by the integrating factor and integrating yields the solution, w(z) = c exp We see that the solution is analytic in D. Example 14.8.1 It does not make sense to pose the equation dw + |z|w = 0. dz For the solution to exist, w and hence w (z) must be analytic. Since p(z) = |z| is not analytic anywhere in the complex plane, the equation has no solution. Any point at which p(z) is analytic is called an ordinary point of the dierential equation. Since the solution is analytic we can expand it in a Taylor series about an ordinary point. The radius of convergence of the series will be at least the distance to the nearest singularity of p(z) in the complex plane. Example 14.8.2 Consider the equation 1 dw w = 0. dz 1z The general solution is w =
c . 1z

p(z) dz .

Expanding this solution about the origin, c w= =c zn. 1z n=0 804

The radius of convergence of the series is, R = lim


n

an = 1, an+1
1 . 1z

which is the distance from the origin to the nearest singularity of p(z) =

We do not need to solve the dierential equation to nd the Taylor series expansion of the homogeneous solution. We could substitute a general Taylor series expansion into the dierential equation and solve for the coecients. Since we can always solve rst order equations, this method is of limited usefulness. However, when we consider higher order equations in which we cannot solve the equations exactly, this will become an important method. Example 14.8.3 Again consider the equation 1 dw w = 0. dz 1z Since we know that the solution has a Taylor series expansion about z = 0, we substitute w = dierential equation. d (1 z) dz
n=0

an z n into the

an z
n=0 n n=0

an z n = 0

nan z
n=1

n1

n=1 n

nan z
n=0

an z n = 0

(n + 1)an+1 z
n=0 n=0

nan z
n=0

an z n = 0

((n + 1)an+1 (n + 1)an ) z n = 0.


n=0

Now we equate powers of z to zero. For z n , the equation is (n + 1)an+1 (n + 1)an = 0, or an+1 = an . Thus we have 805

that an = a0 for all n 1. The solution is then

w = a0
n=0

zn,

which is the result we obtained by expanding the solution in Example 14.8.2.

Result 14.8.1 Consider the equation dw + p(z)w = 0. dz If p(z) is analytic at z = z0 then z0 is called an ordinary point of the dierential equation. The Taylor series expansion of the solution can be found by substituting w = an (z z0 )n n=0 into the equation and equating powers of (z z0 ). The radius of convergence of the series is at least the distance to the nearest singularity of p(z) in the complex plane.
Exercise 14.7 Find the Taylor series expansion about the origin of the solution to dw 1 + w=0 dz 1z with the substitution w = 1 nearest singularity of 1z ? Hint, Solution
n=0

an z n . What is the radius of convergence of the series? What is the distance to the

806

14.8.2

Regular Singular Points

If the coecient function p(z) has a simple pole at z = z0 then z0 is a regular singular point of the rst order dierential equation. Example 14.8.4 Consider the equation dw + w = 0, dz z = 0.

This equation has a regular singular point at z = 0. The solution is w = cz . Depending on the value of , the solution can have three dierent kinds of behavior.

is a negative integer. The solution is analytic in the nite complex plane.

is a positive integer The solution has a pole at the origin. w is analytic in the annulus, 0 < |z|.

is not an integer. w has a branch point at z = 0. The solution is analytic in the cut annulus 0 < |z| < , 0 < arg z < 0 + 2.

Consider the dierential equation dw + p(z)w = 0, dz 807

where p(z) has a simple pole at the origin and is analytic in the annulus, 0 < |z| < r, for some positive r. Recall that the solution is w = c exp = c exp p(z) dz

b0 b0 + p(z) dz z z zp(z) b0 = c exp b0 log z dz z zp(z) b0 = cz b0 exp dz z The exponential factor has a removable singularity at z = 0 and is analytic in |z| < r. We consider the following cases for the z b0 factor: b0 is a negative integer. Since z b0 is analytic at the origin, the solution to the dierential equation is analytic in the circle |z| < r. b0 is a positive integer. The solution has a pole of order b0 at the origin and is analytic in the annulus 0 < |z| < r. b0 is not an integer. The solution has a branch point at the origin and thus is not single-valued. The solution is analytic in the cut annulus 0 < |z| < r, 0 < arg z < 0 + 2. Since the exponential factor has a convergent Taylor series in |z| < r, the solution can be expanded in a series of the form w = z b0
n=0

an z n ,

where a0 = 0 and b0 = lim z p(z).


z0

In the case of a regular singular point at z = z0 , the series is

w = (z z0 )b0
n=0

an (z z0 )n ,

where a0 = 0 and b0 = lim (z z0 ) p(z).


zz0

808

Series of this form are known as Frobenius series. Since we can write the solution as w = c(z z0 )b0 exp p(z) b0 z z0 dz ,

we see that the Frobenius expansion of the solution will have a radius of convergence at least the distance to the nearest singularity of p(z).

Result 14.8.2 Consider the equation, dw + p(z)w = 0, dz where p(z) has a simple pole at z = z0 , p(z) is analytic in some annulus, 0 < |z z0 | < r, and limzz0 (z z0 )p(z) = . The solution to the dierential equation has a Frobenius series expansion of the form

w = (z z0 )

n=0

an (z z0 )n ,

a0 = 0.

The radius of convergence of the expansion will be at least the distance to the nearest singularity of p(z).
Example 14.8.5 We will nd the rst two nonzero terms in the series solution about z = 0 of the dierential equation, dw 1 + w = 0. dz sin z First we note that the coecient function has a simple pole at z = 0 and z 1 = lim = 1. z0 sin z z0 cos z lim 809

Thus we look for a series solution of the form

w=z

1 n=0

an z n ,

a0 = 0.

The nearest singularities of 1/ sin z in the complex plane are at z = . Thus the radius of convergence of the series will be at least . Substituting the rst three terms of the expansion into the dierential equation, 1 d (a0 z 1 + a1 + a2 z) + (a0 z 1 + a1 + a2 z) = O(z). dz sin z Recall that the Taylor expansion of sin z is sin z = z 1 z 3 + O(z 5 ). 6 z3 + O(z 5 ) (a0 z 2 + a2 ) + (a0 z 1 + a1 + a2 z) = O(z 2 ) z 6 a0 a0 z 1 + a2 + z + a0 z 1 + a1 + a2 z = O(z 2 ) 6 a0 z = O(z 2 ) a1 + 2a2 + 6 a0 is arbitrary. Equating powers of z, z0 : z1 : Thus the solution has the expansion, w = a0 z 1 z + O(z 2 ). 12 a1 = 0. a0 = 0. 2a2 + 6

In Figure 14.8 the exact solution is plotted in a solid line and the two term approximation is plotted in a dashed line. The two term approximation is very good near the point x = 0. 810

4 2 2 -2 -4
Figure 14.8: Plot of the exact solution and the two term approximation. Example 14.8.6 Find the rst two nonzero terms in the series expansion about z = 0 of the solution to cos z w i w = 0. z Since cos z has a simple pole at z = 0 and limz0 i cos z = i we see that the Frobenius series will have the form z

w=z Recall that cos z has the Taylor expansion equation yields

i n=0

an z n ,

a0 = 0.

(1)n z 2n . n=0 (2n)!

Substituting the Frobenius expansion into the dierential

iz i1
n=0

an z n + z i
n=0

nan z n1

i
n=0

(1)n z 2n (2n)!

zi
n=0

an z n

=0

(n + i)an z n i
n=0 n=0

(1) z (2n)!

n 2n

an z n
n=0

= 0.

811

Equating powers of z, z 0 : ia0 ia0 = 0 a0 is arbitrary 1 z : (1 + i)a1 ia1 = 0 a1 = 0 i i z 2 : (2 + i)a2 ia2 + a0 = 0 a2 = a0 . 2 4 Thus the solution is i w = a0 z i 1 z 2 + O(z 3 ) . 4

14.8.3

Irregular Singular Points

If a point is not an ordinary point or a regular singular point then it is called an irregular singular point. The following equations have irregular singular points at the origin. w + zw = 0 w z 2 w = 0 w + exp(1/z)w = 0 Example 14.8.7 Consider the dierential equation dw + z w = 0, = 0, = 1, 0, 1, 2, . . . dz This equation has an irregular singular point at the origin. Solving this equation, d exp z dz w = 0 dz w = c exp +1 z +1

=c
n=0

(1)n n!

+1

z (+1)n .

812

If is not an integer, then the solution has a branch point at the origin. If is an integer, < 1, then the solution has an essential singularity at the origin. The solution cannot be expanded in a Frobenius series, w = z an z n . n=0

Although we will not show it, this result holds for any irregular singular point of the dierential equation. We cannot approximate the solution near an irregular singular point using a Frobenius expansion.

Now would be a good time to summarize what we have discovered about solutions of rst order dierential equations in the complex plane. 813

Result 14.8.3 Consider the rst order dierential equation dw + p(z)w = 0. dz Ordinary Points If p(z) is analytic at z = z0 then z0 is an ordinary point of the dierential equation. The solution can be expanded in the Taylor series w = an (z z0 )n . n=0 The radius of convergence of the series is at least the distance to the nearest singularity of p(z) in the complex plane. Regular Singular Points If p(z) has a simple pole at z = z0 and is analytic in some annulus 0 < |z z0 | < r then z0 is a regular singular point of the dierential equation. The solution at z0 will either be analytic, have a pole, or have a branch point. The solution can be expanded in the Frobenius series w = (z z0 ) an (z z0 )n where a0 = 0 n=0 and = limzz0 (z z0 )p(z). The radius of convergence of the Frobenius series will be at least the distance to the nearest singularity of p(z). Irregular Singular Points If the point z = z0 is not an ordinary point or a regular singular point, then it is an irregular singular point of the dierential equation. The solution cannot be expanded in a Frobenius series about that point.

14.8.4

The Point at Innity

Now we consider the behavior of rst order linear dierential equations at the point at innity. Recall from complex variables that the complex plane together with the point at innity is called the extended complex plane. To study the 1 behavior of a function f (z) at innity, we make the transformation z = and study the behavior of f (1/) at = 0. 814

Example 14.8.8 Lets examine the behavior of sin z at innity. We make the substitution z = 1/ and nd the Laurent expansion about = 0. (1)n sin(1/) = (2n + 1)! (2n+1) n=0 Since sin(1/) has an essential singularity at = 0, sin z has an essential singularity at innity.

We use the same approach if we want to examine the behavior at innity of a dierential equation. Starting with the rst order dierential equation, dw + p(z)w = 0, dz we make the substitution d 1 d z= , = 2 , w(z) = u() dz d to obtain du + p(1/)u = 0 2 d du p(1/) u = 0. d 2

Result 14.8.4 The behavior at innity of dw + p(z)w = 0 dz is the same as the behavior at = 0 of du p(1/) u = 0. d 2
815

Example 14.8.9 We classify the singular points of the equation dw 1 + 2 w = 0. dz z +9 We factor the denominator of the fraction to see that z = 3 and z = 3 are regular singular points. 1 dw + w=0 dz (z 3)(z + 3) We make the transformation z = 1/ to examine the point at innity. 1 du 1 2 u=0 d (1/)2 + 9 1 du 2 u=0 d 9 + 1 Since the equation for u has a ordinary point at = 0, z = is a ordinary point of the equation for w.

816

14.9

Additional Exercises

Exact Equations
Exercise 14.8 (mathematica/ode/rst order/exact.nb) Find the general solution y = y(x) of the equations 1. x2 + xy + y 2 dy = , dx x2

2. (4y 3x) dx + (y 2x) dy = 0. Hint, Solution Exercise 14.9 (mathematica/ode/rst order/exact.nb) Determine whether or not the following equations can be made exact. If so nd the corresponding general solution. 1. (3x2 2xy + 2) dx + (6y 2 x2 + 3) dy = 0 2. Hint, Solution Exercise 14.10 (mathematica/ode/rst order/exact.nb) Find the solutions of the following dierential equations which satisfy the given initial condition. In each case determine the interval in which the solution is dened. 1. dy = (1 2x)y 2 , dx y(0) = 1/6. y(0) = 1. dy ax + by = dx bx + cy

2. x dx + y ex dy = 0, Hint, Solution 817

Exercise 14.11 Are the following equations exact? If so, solve them. 1. (4y x)y (9x2 + y 1) = 0 2. (2x 2y)y + (2x + 4y) = 0. Hint, Solution Exercise 14.12 (mathematica/ode/rst order/exact.nb) Find all functions f (t) such that the dierential equation y 2 sin t + yf (t) is exact. Solve the dierential equation for these f (t). Hint, Solution dy =0 dt (14.7)

The First Order, Linear Dierential Equation


Exercise 14.13 (mathematica/ode/rst order/linear.nb) Solve the dierential equation y y + = 0. sin x Hint, Solution

Initial Conditions Well-Posed Problems


Exercise 14.14 Find the solutions of dy + Ay = 1 + t2 , dt which are bounded at t = 0. Consider all (real) values of A. Hint, Solution t 818 t>0

Equations in the Complex Plane


Exercise 14.15 Classify the singular points of the following rst order dierential equations, (include the point at innity). 1. w + 2. w +
sin z w z 1 w z3

=0 =0

3. w + z 1/2 w = 0 Hint, Solution Exercise 14.16 Consider the equation w + z 2 w = 0. The point z = 0 is an irregular singular point of the dierential equation. Thus we know that we cannot expand the solution about z = 0 in a Frobenius series. Try substituting the series solution

w=z

n=0

an z n ,

a0 = 0

into the dierential equation anyway. What happens? Hint, Solution

819

14.10
Hint 14.1

Hints

1. 2.

d dx

ln |u| =

1 u

d c u dx

= uc1 u

Hint 14.2 Hint 14.3 The equation is homogeneous. Make the change of variables u = y/t. Hint 14.4 Make sure you consider the case = 0. Hint 14.5 Hint 14.6 Hint 14.7 The radius of convergence of the series and the distance to the nearest singularity of

1 1z

are not the same.

Exact Equations
Hint 14.8 1. 2. 820

Hint 14.9 1. The equation is exact. Determine the primitive u by solving the equations ux = P , uy = Q. 2. The equation can be made exact. Hint 14.10 1. This equation is separable. Integrate to get the general solution. Apply the initial condition to determine the constant of integration. 2. Ditto. You will have to numerically solve an equation to determine where the solution is dened. Hint 14.11 Hint 14.12

The First Order, Linear Dierential Equation


Hint 14.13 Look in the appendix for the integral of csc x.

Initial Conditions Well-Posed Problems


Hint 14.14

Equations in the Complex Plane


Hint 14.15

821

Hint 14.16 Try to nd the value of by substituting the series into the dierential equation and equating powers of z.

822

14.11

Solutions

Solution 14.1

1. y (x) = f (x) y(x) d ln |y(x)| = f (x) dx ln |y(x)| = y(x) = e y(x) = c e f (x) dx + c


f (x) dx+c f (x) dx

2. y (x)y (x) = f (x) y +1 (x) = f (x) dx + c +1


1/(+1)

y(x) =

( + 1)

f (x) dx + a

823

3. y tan x +y = cos x cos x cos x d y = cos x dx cos x y = sin x + c cos x y(x) = sin x cos x + c cos x Solution 14.2 We consider the homogeneous equation, dy = 0. dx That is, both P and Q are homogeneous of degree n. We hypothesize that multiplying by P (x, y) + Q(x, y) (x, y) = 1 xP (x, y) + yQ(x, y)

will make the equation exact. To prove this we use the result that M (x, y) + N (x, y) is exact if and only if My = Nx . My = P y xP + yQ Py (xP + yQ) P (xPy + Q + yQy ) = (xP + yQ)2 824 dy =0 dx

Nx =

Q x xP + yQ Qx (xP + yQ) Q(P + xPx + yQx ) = (xP + yQ)2

M y = Nx Py (xP + yQ) P (xPy + Q + yQy ) = Qx (xP + yQ) Q(P + xPx + yQx ) yPy Q yP Qy = xP Qx xPx Q xPx Q + yPy Q = xP Qx + yP Qy (xPx + yPy )Q = P (xQx + yQy ) With Eulers theorem, this reduces to an identity. nP Q = P nQ Thus the equation is exact. (x, y) is an integrating factor for the homogeneous equation. Solution 14.3 We note that this is a homogeneous dierential equation. The coecient of dy/dt and the inhomogeneity are homogeneous of degree zero. dy y y 2 =2 + . dt t t We make the change of variables u = y/t to obtain a separable equation. tu + u = 2u + u2 u 1 = 2+u u t 825

Now we integrate to solve for u. u 1 = u(u + 1) t u u 1 = u u+1 t ln |u| ln |u + 1| = ln |t| + c u ln = ln |ct| u+1 u = ct u+1 u = ct u+1 ct u= 1 ct t u= ct t2 y= ct Solution 14.4 We consider 1 y y = x , x First we nd the integrating factor. I(x) = exp 1 dx x 826 = exp ( ln x) = 1 . x x > 0.

We multiply by the integrating factor and integrate. 1 1 y 2 y = x1 x x d 1 y = x1 dx x 1 y = x1 dx + c x y=x y= Solution 14.5 1. y + xy = x2n+1 , We nd the integrating factor. I(x) = e
x dx x+1

x1 dx + cx for = 0, for = 0.

+ cx x ln x + cx

y(1) = 1, = ex
2 /2

nZ

We multiply by the integrating factor and integrate. Since the initial condition is given at x = 1, we will take the lower bound of integration to be that point. d 2 2 ex /2 y = x2n+1 ex /2 dx y = ex
2 /2

2n+1 e
1

2 /2

d + c ex

2 /2

We choose the constant of integration to satisfy the initial condition. y = ex


2 /2

2n+1 e
1

2 /2

d + e(1x

2 )/2

827

If n 0 then we can use integration by parts to write the integral as a sum of terms. If n < 0 we can write the integral in terms of the exponential integral function. However, the integral form above is as nice as any other and we leave the answer in that form. 2. dy 2xy(x) = 1, dx y(0) = 1.

We determine the integrating factor and then integrate the equation. I(x) = e 2x dx = ex d 2 2 ex y = ex dx y = ex
2 2

e d + c ex
0

We choose the constant of integration to satisfy the initial condition. y = ex


2

1+
0

e d

We can write the answer in terms of the Error function, 2 erf(x) y = ex


2

x 0

e d. erf(x) 2

1+

828

Solution 14.6 We determine the integrating factor and then integrate the equation. I(x) = e dx = ex d x (e y) = e()x dx y = ex First consider the case = . y = ex y= Clearly the solution vanishes as x . Next consider = . y = ex x + c ex y = (c + x) ex We use LHospitals rule to show that the solution vanishes as x . lim c + x = lim =0 x x ex e for = 1, for = 1. e()x + c ex e()x dx + c ex

ex +c ex

For = = 1, the solution is y= ex +c ex (c + x) ex


1 1

829

8
Figure 14.9: The Solution for a Range of

12

16

The solution which satises the initial condition is

y=

(ex +( 2) ex ) for = 1, (1 + x) ex for = 1.


1 1

In Figure 14.9 the solution is plotted for = 1/16, 1/8, . . . , 16. Consider the solution in the limit as 0. 1 ex +( 2) ex 0 1 = 2 ex 830

lim y(x) = lim

1 1

Figure 14.10: The Solution as 0 and

In the limit as we have,

lim y(x) = lim

1 ex +( 2) ex 1 2 x e = lim 1 1 for x = 0, = 0 for x > 0.

This behavior is shown in Figure 14.10. The rst graph plots the solutions for = 1/128, 1/64, . . . , 1. The second graph plots the solutions for = 1, 2, . . . , 128.

831

Solution 14.7 We substitute w =

n=0

an z n into the equation d dz

dw dz

+
n

1 w 1z

= 0.

1 an z + 1z n=0

an z n = 0
n=0

(1 z)
n=1

nan z n1 +
n=0 n n

an z n = 0

(n + 1)an+1 z
n=0 n=0

nan z +
n=0

an z n = 0

((n + 1)an+1 (n 1)an ) z n = 0


n=0

Equating powers of z to zero, we obtain the relation, an+1 = n1 an . n+1

a0 is arbitrary. We can compute the rest of the coecients from the recurrence relation. 1 a0 = a0 1 0 a2 = a1 = 0 2 a1 = We see that the coecients are zero for n 2. Thus the Taylor series expansion, (and the exact solution), is w = a0 (1 z).
1 The radius of convergence of the series in innite. The nearest singularity of 1z is at z = 1. Thus we see the radius of convergence can be greater than the distance to the nearest singularity of the coecient function, p(z).

832

Exact Equations
Solution 14.8 1. x2 + xy + y 2 dy = dx x2 Since the right side is a homogeneous function of order zero, this is a homogeneous dierential equation. We make the change of variables u = y/x and then solve the dierential equation for u.

xu + u = 1 + u + u2 du dx = 2 1+u x arctan(u) = ln |x| + c u = tan(ln(|cx|)) y = x tan(ln(|cx|))

2. (4y 3x) dx + (y 2x) dy = 0

Since the coecients are homogeneous functions of order one, this is a homogeneous dierential equation. We 833

make the change of variables u = y/x and then solve the dierential equation for u. y y 2 dy = 0 4 3 dx + x x (4u 3) dx + (u 2)(u dx + x du) = 0 (u2 + 2u 3) dx + x(u 2) du = 0 dx u2 + du = 0 x (u + 3)(u 1) 5/4 dx 1/4 + du = 0 x u+3 u1 1 5 ln(x) + ln(u + 3) ln(u 1) = c 4 4 x4 (u + 3)5 =c u1 x4 (y/x + 3)5 =c y/x 1 (y + 3x)5 =c yx Solution 14.9 1. (3x2 2xy + 2) dx + (6y 2 x2 + 3) dy = 0 We check if this form of the equation, P dx + Q dy = 0, is exact. Py = 2x, Qx = 2x

Since Py = Qx , the equation is exact. Now we nd the primitive u(x, y) which satises du = (3x2 2xy + 2) dx + (6y 2 x2 + 3) dy. 834

The primitive satises the partial dierential equations ux = P, uy = Q. (14.8)

We integrate the rst equation of 14.8 to determine u up to a function of integration. ux = 3x2 2xy + 2 u = x3 x2 y + 2x + f (y) We substitute this into the second equation of 14.8 to determine the function of integration up to an additive constant. x2 + f (y) = 6y 2 x2 + 3 f (y) = 6y 2 + 3 f (y) = 2y 3 + 3y The solution of the dierential equation is determined by the implicit equation u = c. x3 x2 y + 2x + 2y 3 + 3y = c 2. dy ax + by = dx bx + cy (ax + by) dx + (bx + cy) dy = 0 We check if this form of the equation, P dx + Q dy = 0, is exact. Py = b, Qx = b

Since Py = Qx , the equation is exact. Now we nd the primitive u(x, y) which satises du = (ax + by) dx + (bx + cy) dy 835

The primitive satises the partial dierential equations ux = P, uy = Q. (14.9) We integrate the rst equation of 14.9 to determine u up to a function of integration. ux = ax + by 1 u = ax2 + bxy + f (y) 2 We substitute this into the second equation of 14.9 to determine the function of integration up to an additive constant. bx + f (y) = bx + cy f (y) = cy 1 f (y) = cy 2 2 The solution of the dierential equation is determined by the implicit equation u = d. ax2 + 2bxy + cy 2 = d Solution 14.10 Note that since these equations are nonlinear, we cannot predict where the solutions will be dened from the equation alone. 1. This equation is separable. We integrate to get the general solution. dy = (1 2x)y 2 dx dy = (1 2x) dx y2 1 = x x2 + c y 1 y= 2 x xc 836

Now we apply the initial condition. 1 1 = c 6 1 y= 2 x x6 1 y= (x + 2)(x 3) y(0) = The solution is dened on the interval (2 . . . 3). 2. This equation is separable. We integrate to get the general solution. x dx + y ex dy = 0 x ex dx + y dy = 0 1 (x 1) ex + y 2 = c 2 y = 2(c + (1 x) ex ) We apply the initial condition to determine the constant of integration. y(0) = 2(c + 1) = 1 1 c= 2 2(1 x) ex 1

y=

The function 2(1 x) ex 1 is plotted in Figure 14.11. We see that the argument of the square root in the solution is non-negative only on an interval about the origin. Because 2(1 x) ex 1 == 0 is a mixed algebraic / transcendental equation, we cannot solve it analytically. The solution of the dierential equation is dened on the interval (1.67835 . . . 0.768039). 837

1 -5 -4 -3 -2 -1 -1 -2 -3 1

Figure 14.11: The function 2(1 x) ex 1. Solution 14.11 1. We consider the dierential equation, (4y x)y (9x2 + y 1) = 0. 1 y 9x2 = 1 y Qx = (4y x) = 1 x This equation is exact. It is simplest to solve the equation by rearranging terms to form exact derivatives. Py = 4yy xy y + 1 9x2 = 0 d 2y 2 xy + 1 9x2 = 0 dx 2y 2 xy + x 3x3 + c = 0 y= 2. We consider the dierential equation, (2x 2y)y + (2x + 4y) = 0. 838 1 x 4 x2 8(c + x 3x3 )

(2x + 4y) = 4 y Qx = (2x 2y) = 2 x Since Py = Qx , this is not an exact equation. Py = Solution 14.12 Recall that the dierential equation P (x, y) + Q(x, y)y = 0 is exact if and only if Py = Qx . For Equation 14.7, this criterion is 2y sin t = yf (t) f (t) = 2 sin t f (t) = 2(a cos t). In this case, the dierential equation is y 2 sin t + 2yy (a cos t) = 0. We can integrate this exact equation by inspection. d 2 y (a cos t) = 0 dt y 2 (a cos t) = c c y = a cos t

The First Order, Linear Dierential Equation


Solution 14.13 Consider the dierential equation y +

y = 0. sin x 839

We use Equation 14.5 to determine the solution.

y = ce

1/ sin x dx

y = c e ln | tan(x/2)| x y = c cot 2 x y = c cot 2

Initial Conditions Well-Posed Problems


Solution 14.14 First we write the dierential equation in the standard form.

dy A 1 + y = + t, dt t t

t>0

We determine the integrating factor.


A/t dt

I(t) = e

= eA ln t = tA

840

We multiply the dierential equation by the integrating factor and integrate. dy A 1 + y = +t dt t t d A t y = tA1 + tA+1 dt A tA+2 t A = 0, 2 A + A+2 + c, A 1 2 t y = ln t + 2 t + c, A=0 1 2 t + ln t + c, A = 2 2 2 1 t A + A+2 + ctA , A = 2 y = ln t + 1 t2 + c, A=0 1 22 2 2 + t ln t + ct , A = 2 For positive A, the solution is bounded at the origin only for c = 0. For A = 0, there are no bounded solutions. For negative A, the solution is bounded there for any value of c and thus we have a one-parameter family of solutions. In summary, the solutions which are bounded at the origin are: 1 t2 A + A+2 , A>0 1 t2 A y = A + A+2 + ct , A < 0, A = 2 1 2 + t2 ln t + ct2 , A = 2

Equations in the Complex Plane


Solution 14.15 1. Consider the equation w + sin z w = 0. The point z = 0 is the only point we need to examine in the nite plane. z Since sin z has a removable singularity at z = 0, there are no singular points in the nite plane. The substitution z 1 z = yields the equation sin(1/) u u = 0. 841

Since sin(1/) has an essential singularity at = 0, the point at innity is an irregular singular point of the original dierential equation.
1 1 2. Consider the equation w + z3 w = 0. Since z3 has a simple pole at z = 3, the dierential equation has a regular singular point there. Making the substitution z = 1/, w(z) = u()

1 u=0 2 (1/ 3) 1 u u = 0. (1 3)

Since this equation has a simple pole at = 0, the original equation has a regular singular point at innity. 3. Consider the equation w + z 1/2 w = 0. There is an irregular singular point at z = 0. With the substitution z = 1/, w(z) = u(), 1/2 u=0 2 u 5/2 u = 0. u We see that the point at innity is also an irregular singular point of the original dierential equation. Solution 14.16 We start with the equation w + z 2 w = 0. Substituting w = z
n=0

an z n , a0 = 0 yields d dz

n=0

an z

+z z

2 n=0

an z n = 0

z 1
n=0

an z n + z
n=1

nan z n1 + z
n=0

an z n2 = 0

842

The lowest power of z in the expansion is z 2 . The coecient of this term is a0 . Equating powers of z demands that a0 = 0 which contradicts our initial assumption that it was nonzero. Thus we cannot nd a such that the solution can be expanded in the form,

w=z

n=0

an z n ,

a0 = 0.

843

14.12

Quiz

Problem 14.1 What is the general solution of a rst order dierential equation? Solution Problem 14.2 Write a statement about the functions P and Q to make the following statement correct. The rst order dierential equation dy P (x, y) + Q(x, y) =0 dx is exact if and only if . It is separable if . Solution Problem 14.3 Derive the general solution of dy + p(x)y = f (x). dx Solution Problem 14.4 Solve y = y y 2 . Solution

844

14.13

Quiz Solutions

Solution 14.1 The general solution of a rst order dierential equation is a one-parameter family of functions which satises the equation. Solution 14.2 The rst order dierential equation P (x, y) + Q(x, y) dy =0 dx

is exact if and only if Py = Qx . It is separable if P = P (x) and Q = Q(y). Solution 14.3 dy + p(x)y = f (x) dx We multiply by the integrating factor (x) = exp(P (x)) = exp p(x) dx , and integrate.

dy P (x) e +p(x)y eP (x) = eP (x) f (x) dx d y eP (x) = eP (x) f (x) dx y eP (x) = y = eP (x) eP (x) f (x) dx + c eP (x) f (x) dx + c eP (x)

845

Solution 14.4 y = y y 2 is separable. y = y y2 y =1 y y2 y y =1 y y1 ln y ln(y 1) = x + c We do algebraic simplications and rename the constant of integration to write the solution in a nice form. y = c ex y1 y = (y 1)c ex c ex y= 1 c ex ex y= x e c 1 y= 1 c ex

846

Chapter 15 First Order Linear Systems of Dierential Equations


We all agree that your theory is crazy, but is it crazy enough? - Niels Bohr

15.1

Introduction

In this chapter we consider rst order linear systems of dierential equations. That is, we consider equations of the form, x (t) = Ax(t) + f (t), a11 a12 x1 (t) a21 a22 . x(t) = . , A= . . . . . . . xn (t) an1 an2 847

. . . a1n . . . a2n . . .. . . . . . . ann

Initially we will consider the homogeneous problem, x (t) = Ax(t). (Later we will nd particular solutions with variation of parameters.) The best way to solve these equations is through the use of the matrix exponential. Unfortunately, using the matrix exponential requires knowledge of the Jordan canonical form and matrix functions. Fortunately, we can solve a certain class of problems using only the concepts of eigenvalues and eigenvectors of a matrix. We present this simple method in the next section. In the following section we will take a detour into matrix theory to cover Jordan canonical form and its applications. Then we will be able to solve the general case.

15.2

Using Eigenvalues and Eigenvectors to nd Homogeneous Solutions

If you have forgotten what eigenvalues and eigenvectors are and how to compute them, go nd a book on linear algebra and spend a few minutes re-aquainting yourself with the rudimentary material. Recall that the single dierential equation x (t) = Ax has the general solution x = c eAt . Maybe the system of dierential equations x (t) = Ax(t) (15.1) has similiar solutions. Perhaps it has a solution of the form x(t) = et for some constant vector and some value . Lets substitute this into the dierential equation and see what happens. x (t) = Ax(t) et = A et A = We see that if is an eigenvalue of A with eigenvector then x(t) = et satises the dierential equation. Since the dierential equation is linear, c et is a solution. Suppose that the n n matrix A has the eigenvalues {k } with a complete set of linearly independent eigenvectors { k }. Then each of k ek t is a homogeneous solution of Equation 15.1. We note that each of these solutions is linearly 848

independent. Without any kind of justication I will tell you that the general solution of the dierential equation is a linear combination of these n linearly independent solutions.

Result 15.2.1 Suppose that the n n matrix A has the eigenvalues {k } with a complete set of linearly independent eigenvectors { k }. The system of dierential equations, x (t) = Ax(t), has the general solution, x(t) =
k=1 n

c k k e k t

Example 15.2.1 (mathematica/ode/systems/systems.nb) Find the solution of the following initial value problem. Describe the behavior of the solution as t . x = Ax 2 1 x, 5 4 x(0) = x0 1 3

The matrix has the distinct eigenvalues 1 = 1, 2 = 3. The corresponding eigenvectors are x1 = 1 , 1 x2 = 1 . 5

The general solution of the system of dierential equations is x = c1 1 t 1 3t e +c2 e . 1 5 849

We apply the initial condition to determine the constants. 1 1 1 5 1 c1 = , 2 The solution subject to the initial condition is x= For large t, the solution looks like x 1 2 1 3t e . 5 1 2 1 t 1 e + 1 2 1 3t e 5 c1 c2 1 3 1 c2 = 2 =

Both coordinates tend to innity. Figure 15.1 shows some homogeneous solutions in the phase plane. Example 15.2.2 (mathematica/ode/systems/systems.nb) Find the solution of the following initial value problem. Describe the behavior of the solution as t . 1 1 2 2 x = Ax 0 2 2 x, x(0) = x0 0 1 1 3 1 The matrix has the distinct eigenvalues 1 = 1, 2 = 2, 3 = 3. The corresponding eigenvectors are 0 1 2 2 , x2 = 1 , x3 = 2 . x1 = 1 0 1 850

10 7.5 5 2.5 -10 -7.5 -5 -2.5 -2.5 -5 -7.5 -10 2.5 5 7.5 10

Figure 15.1: Homogeneous solutions in the phase plane.

The general solution of the system of dierential equations is

0 1 2 2 et +c2 1 e2t +c3 2 e3t . x = c1 1 0 1 851

We apply the initial condition to determine the constants. 0 1 2 c1 2 2 1 2 c2 = 0 1 0 1 c3 1 c1 = 1, The solution subject to the initial condition is 0 1 2 et +2 1 e2t . x= 1 0 As t , all coordinates tend to innity. Exercise 15.1 (mathematica/ode/systems/systems.nb) Find the solution of the following initial value problem. Describe the behavior of the solution as t . x = Ax Hint, Solution Exercise 15.2 (mathematica/ode/systems/systems.nb) Find the solution of the following initial value problem. Describe the behavior of the solution as t . 3 0 2 1 1 1 0 x, x(0) = x0 0 x = Ax 2 1 0 0 Hint, Solution 852 1 5 x, 1 3 x(0) = x0 1 1 c2 = 2, c3 = 0

Exercise 15.3 Use the matrix form of the method of variation of parameters to nd the general solution of dx = dt Hint, Solution 4 2 t3 x+ , 8 4 t2 t > 0.

15.3

Matrices and Jordan Canonical Form

Functions of Square Matrices. Consider a function f (x) with a Taylor series. f (x) =
n=0

f (n) (0) n x n!

We can dene the function to take square matrices as arguments. The function of the square matrix A is dened in terms of the Taylor series. f (n) (0) n f (A) = A n! n=0 (Note that this denition is usually not the most convenient method for computing a function of a matrix. Use the Jordan canonical form for that.) Eigenvalues and Eigenvectors. Consider a square matrix A. A nonzero vector x is an eigenvector of the matrix with eigenvalue if Ax = x. Note that we can write this equation as (A I)x = 0. 853

This equation has solutions for nonzero x if and only if A I is singular, (det(A I) = 0). We dene the characteristic polynomial of the matrix () as this determinant. () = det(A I) The roots of the characteristic polynomial are the eigenvalues of the matrix. The eigenvectors of distinct eigenvalues are linearly independent. Thus if a matrix has distinct eigenvalues, the eigenvectors form a basis. If is a root of () of multiplicity m then there are up to m linearly independent eigenvectors corresponding to that eigenvalue. That is, it has from 1 to m eigenvectors. Diagonalizing Matrices. Consider an nn matrix A that has a complete set of n linearly independent eigenvectors. A may or may not have distinct eigenvalues. Consider the matrix S with eigenvectors as columns. S = x1 x2 xn A is diagonalized by the similarity transformation: = S1 AS. is a diagonal matrix with the eigenvalues of A as the diagonal elements. Furthermore, the k th diagonal element is k , the eigenvalue corresponding to the the eigenvector, xk . Generalized Eigenvectors. A vector xk is a generalized eigenvector of rank k if (A I)k xk = 0 but (A I)k1 xk = 0. Eigenvectors are generalized eigenvectors of rank 1. An nn matrix has n linearly independent generalized eigenvectors. A chain of generalized eigenvectors generated by the rank m generalized eigenvector xm is the set: {x1 , x2 , . . . , xm }, where xk = (A I)xk+1 , for k = m 1, . . . , 1. 854

Computing Generalized Eigenvectors. Let be an eigenvalue of multiplicity m. Let n be the smallest integer such that rank (nullspace ((A I)n )) = m. Let Nk denote the number of eigenvalues of rank k. These have the value: Nk = rank nullspace (A I)k rank nullspace (A I)k1 .

One can compute the generalized eigenvectors of a matrix by looping through the following three steps until all the the Nk are zero: 1. Select the largest k for which Nk is positive. Find a generalized eigenvector xk of rank k which is linearly independent of all the generalized eigenvectors found thus far. 2. From xk generate the chain of eigenvectors {x1 , x2 , . . . , xk }. Add this chain to the known generalized eigenvectors. 3. Decrement each positive Nk by one. Example 15.3.1 Consider the matrix 1 1 1 A = 2 1 1 . 3 2 4 The characteristic polynomial of the matrix is () = 1 1 1 2 1 1 3 2 4

= (1 )2 (4 ) + 3 + 4 + 3(1 ) 2(4 ) + 2(1 ) = ( 2)3 . 855

Thus we see that = 2 is an eigenvalue of multiplicity 3. A 2I is 1 1 1 A 2I = 2 1 1 3 2 2 The rank of the nullspace space of A 2I is less than 3. 0 0 0 1 (A 2I)2 = 1 1 1 1 1 The rank of nullspace((A 2I)2 ) is less than 3 as well, so we have to take one more step. 0 0 0 (A 2I)3 = 0 0 0 0 0 0 The rank of nullspace((A 2I)3 ) is 3. Thus there are generalized eigenvectors of ranks 1, 2 and 3. The generalized eigenvector of rank 3 satises: (A 2I)3 x3 = 0 0 0 0 0 0 0 x3 = 0 0 0 0 We choose the solution 1 0 . x3 = 0 856

Now to compute the chain generated by x3 . 1 x2 = (A 2I)x3 = 2 3 0 x1 = (A 2I)x2 = 1 1 Thus a set of generalized eigenvectors corresponding to the eigenvalue = 2 are 0 x1 = 1 , 1 1 x2 = 2 , 3 1 x3 = 0 . 0

Jordan Block. A Jordan block is a square matrix super-diagonal: 1 0 0 0 . . . . . . 0 0 0 0

which has the constant, , on the diagonal and ones on the rst .. . .. .. . . .. . 0 0 0 1 857 0 0 0 0 . .. . . . 1 0 0 0

Jordan Canonical Form. A matrix J is in Jordan canonical form if all the elements are zero except for Jordan blocks Jk along the diagonal. J1 0 0 0 . 0 J2 . . 0 0 . . J = . ... ... ... . . . .. 0 0 . Jn1 0 0 0 0 Jn The Jordan canonical form of a matrix is obtained with the similarity transformation: J = S1 AS, where S is the matrix of the generalized eigenvectors of A and the generalized eigenvectors are grouped in chains. Example 15.3.2 Again consider the matrix 1 1 1 A = 2 1 1 . 3 2 4 Since = 2 is an eigenvalue of multiplicity 3, the Jordan canonical form of the matrix is 2 1 0 J = 0 2 1 . 0 0 2 In Example 15.3.1 we found the generalized eigenvectors of columns: 0 1 S= 1 A. We dene the matrix with generalized eigenvectors as 1 1 2 0 . 3 0

858

We can verify that J = S1 AS. J = S1 AS 0 3 2 1 1 1 0 1 1 = 0 1 1 2 1 1 1 2 0 1 1 1 3 2 4 1 3 0 2 1 0 = 0 2 1 0 0 2

Functions of Matrices in Jordan Canonical Form. The function of an n n Jordan block is the uppertriangular matrix: (n2) () () f () f (n1) () f () f 1! f (n2)! 2! (n1)! (n3) () () f (n2) () 0 f () f 1! f (n3)! (n2)! (n4) () . f (n3) () 0 f () . . f (n4)! (n3)! f (Jk ) = 0 . . . .. .. .. . . . . . . . . . .. f () 0 . 0 0 f () 1! 0 0 0 0 f () The function of a matrix in Jordan canonical form is f (J1 ) 0 0 f (J2 ) . .. f (J) = . . . 0 0 0 0

0 0 .. . 0 0 . .. .. . . . . .. . f (Jn1 ) 0 0 f (Jn )

859

The Jordan canonical form of a matrix satises: f (J) = S1 f (A)S, where S is the matrix of the generalized eigenvectors of A. This gives us a convenient method for computing functions of matrices. Example 15.3.3 Consider the matrix exponential function 1 2 A= 3 eA for our old friend: 1 1 1 1 . 2 4

In Example 15.3.2 we showed that the Jordan canonical form of the matrix is 2 1 0 J = 0 2 1 . 0 0 2 Since all the derivatives of e are just e , it is especially easy to compute eJ . 2 2 2 e e e /2 J e = 0 e2 e2 e2 0 0 We nd eA with a similarity transformation of eJ . We use the matrix of generalized eigenvectors found in Example 15.3.2. eA = S eJ S1 2 2 2 e e e /2 0 1 1 0 3 2 eA = 1 2 0 0 e2 e2 0 1 1 e2 1 1 1 1 3 0 0 0 0 2 2 e2 A 3 1 1 e = 2 5 3 5

860

15.4

Using the Matrix Exponential


x (t) = Ax(t)

The homogeneous dierential equation has the solution x(t) = eAt c where c is a vector of constants. The solution subject to the initial condition, x(t0 ) = x0 is x(t) = eA(tt0 ) x0 . The homogeneous dierential equation 1 x (t) = Ax(t) t has the solution x(t) = tA c eA Log t c, where c is a vector of constants. The solution subject to the initial condition, x(t0 ) = x0 is x(t) = The inhomogeneous problem x (t) = Ax(t) + f (t), has the solution x(t) = eA(tt0 ) x0 + eAt
t0

t t0

x0 eA Log(t/t0 ) x0 .

x(t0 ) = x0
t

eA f ( ) d.

Example 15.4.1 Consider the system 1 1 1 dx 2 1 1 x. = dt 3 2 4 861

The general solution of the system of dierential equations is x(t) = eAt c. In Example 15.3.3 we found eA . At is just a constant times A. The eigenvalues of At are {k t} where {k } are the eigenvalues of A. The generalized eigenvectors of At are the same as those of A. Consider eJt . The derivatives of f () = et are f () = t et and f () = t2 et . Thus we have 2t e t e2t t2 e2t /2 eJt = 0 e2t t e2t e2t 0 0 1 t t2 /2 eJt = 0 1 t e2t 0 0 1 We nd eAt with a similarity transformation. eAt = S eJt S1 0 1 1 1 t t2 /2 0 3 2 eAt = 1 2 0 0 1 t e2t 0 1 1 1 3 0 0 0 1 1 1 1 1t t t eAt = 2t t2 /2 1 t + t2 /2 t + t2 /2 e2t 2 2 3t + t /2 2t t /2 1 + 2t t2 /2 The solution of the system of dierential equations is 1t t t x(t) = c1 2t t2 /2 + c2 1 t + t2 /2 + c3 t + t2 /2 e2t 3t + t2 /2 2t t2 /2 1 + 2t t2 /2

862

Example 15.4.2 Consider the Euler equation system dx 1 1 = Ax dt t t 1 0 x. 1 1

The solution is x(t) = tA c. Note that A is almost in Jordan canonical form. It has a one on the sub-diagonal instead of the super-diagonal. It is clear that a function of A is dened f (A) = f (1) 0 . f (1) f (1)

The function f () = t has the derivative f () = t log t. Thus the solution of the system is x(t) = t 0 t log t t c1 c2 = c1 t 0 + c2 t log t t

Example 15.4.3 Consider an inhomogeneous system of dierential equations. dx = Ax + f (t) dt The general solution is x(t) = eAt c + eAt eAt f (t) dt. 4 2 t3 x+ , 8 4 t2 t > 0.

First we nd homogeneous solutions. The characteristic equation for the matrix is () = 4 2 = 2 = 0 8 4

= 0 is an eigenvalue of multiplicity 2. Thus the Jordan canonical form of the matrix is J= 0 1 . 0 0 863

Since rank(nullspace(A 0I)) = 1 there is only one eigenvector. A generalized eigenvector of rank 2 satises (A 0I)2 x2 = 0 0 0 x =0 0 0 2 We choose x2 = Now we generate the chain from x2 . x1 = (A 0I)x2 = We dene the matrix of generalized eigenvectors S. S= The derivative of f () = et is f () = t et . Thus eJt = 1 t 0 1 4 1 8 0 4 8 1 0

The homogeneous solution of the dierential equation system is xh = eAt c where eAt = S eJt S1 eAt = 4 1 1 t . 8 0 0 1 eAt = 0 1/8 1 1/2

1 + 4t 2t 8t 1 4t 864

The general solution of the inhomogeneous system of equations is x(t) = eAt c + eAt x(t) = eAt f (t) dt 1 4t 2t 8t 1 + 4t t3 t2 dt

1 + 4t 2t 1 + 4t 2t c+ 8t 1 4t 8t 1 4t x(t) = c1

1 1 + 4t 2t 2 2 Log t + 6 2t2 t + c2 + 4 4 Log t + 13 8t 1 4t t

We can tidy up the answer a little bit. First we take linear combinations of the homogeneous solutions to obtain a simpler form. x(t) = c1
1 1 2t 2 2 Log t + 6 2t2 t + c2 + 2 4t 1 4 4 Log t + 13 t

Then we subtract 2 times the rst homogeneous solution from the particular solution. x(t) = c1
1 1 2t 2 Log t + 6 2t2 t + c2 + 2 4t 1 4 Log t + 13 t

865

15.5

Exercises

Exercise 15.4 (mathematica/ode/systems/systems.nb) Find the solution of the following initial value problem. x = Ax Hint, Solution Exercise 15.5 (mathematica/ode/systems/systems.nb) Find the solution of the following initial value problem. 1 1 2 x = Ax 0 2 2 x, 1 1 3 Hint, Solution Exercise 15.6 (mathematica/ode/systems/systems.nb) Find the solution of the following initial value problem. Describe the behavior of the solution as t . x = Ax Hint, Solution Exercise 15.7 (mathematica/ode/systems/systems.nb) Find the solution of the following initial value problem. Describe the behavior of the solution as t . 3 0 2 1 1 1 0 x, x(0) = x0 0 x = Ax 2 1 0 0 Hint, Solution 866 1 5 x, 1 3 x(0) = x0 1 1 2 1 x, 5 4 x(0) = x0 1 3

2 x(0) = x0 0 1

Exercise 15.8 (mathematica/ode/systems/systems.nb) Find the solution of the following initial value problem. Describe the behavior of the solution as t . x = Ax Hint, Solution Exercise 15.9 (mathematica/ode/systems/systems.nb) Find the solution of the following initial value problem. Describe the behavior of the solution as t . 1 0 0 1 x = Ax 4 1 0 x, x(0) = x0 2 3 6 2 30 Hint, Solution Exercise 15.10 1. Consider the system 1 1 1 x = Ax = 2 1 1 x. 3 2 4 (15.2) 1 4 x, 4 7 x(0) = x0 3 2

(a) Show that = 2 is an eigenvalue of multiplicity 3 of the coecient matrix A, and that there is only one corresponding eigenvector, namely 0 (1) 1 . = 1 (b) Using the information in part (i), write down one solution x(1) (t) of the system (15.2). There is no other solution of a purely exponential form x = et . (c) To nd a second solution use the form x = t e2t + e2t , and nd appropriate vectors and . This gives a solution of the system (15.2) which is independent of the one obtained in part (ii). 867

(d) To nd a third linearly independent solution use the form x = (t2 /2) e2t +t e2t + e2t . Show that , and satisfy the equations (A 2I) = 0, (A 2I) = , (A 2I) = .

The rst two equations can be taken to coincide with those obtained in part (iii). Solve the third equation, and write down a third independent solution of the system (15.2). 2. Consider the system 5 3 2 x = Ax = 8 5 4 x. 4 3 3 (15.3)

(a) Show that = 1 is an eigenvalue of multiplicity 3 of the coecient matrix A, and that there are only two linearly independent eigenvectors, which we may take as (1) 1 = 0 , 2 (2) 0 = 2 3

Find two independent solutions of equation (15.3). (b) To nd a third solution use the form x = t et +et ; then show that and must satisfy (A I) = 0, (A I) = .

Show that the most general solution of the rst of these equations is = c1 1 + c2 2 , where c1 and c2 are arbitrary constants. Show that, in order to solve the second of these equations it is necessary to take c1 = c2 . Obtain such a vector , and use it to obtain a third independent solution of the system (15.3). Hint, Solution 868

Exercise 15.11 (mathematica/ode/systems/systems.nb) Consider the system of ODEs dx = Ax, x(0) = x0 dt where A is the constant 3 3 matrix 1 1 1 1 1 A= 2 8 5 3 1. Find the eigenvalues and associated eigenvectors of A. [HINT: notice that = 1 is a root of the characteristic polynomial of A.] 2. Use the results from part (a) to construct eAt and therefore the solution to the initial value problem above. 3. Use the results of part (a) to nd the general solution to dx 1 = Ax. dt t Hint, Solution Exercise 15.12 (mathematica/ode/systems/systems.nb) 1. Find the general solution to dx = Ax dt where 2 0 1 A = 0 2 0 0 1 3 2. Solve dx = Ax + g(t), dt 869

x(0) = 0

using A from part (a). Hint, Solution Exercise 15.13 Let A be an n n matrix of constants. The system dx 1 = Ax, dt t is analogous to the Euler equation. 1. Verify that when A is a 2 2 constant matrix, elimination of (15.4) yields a second order Euler dierential equation. 2. Now assume that A is an n n matrix of constants. Show that this system, in analogy with the Euler equation has solutions of the form x = at where a is a constant vector provided a and satisfy certain conditions. 3. Based on your experience with the treatment of multiple roots in the solution of constant coecient systems, what form will the general solution of (15.4) take if is a multiple eigenvalue in the eigenvalue problem derived in part (b)? 4. Verify your prediction by deriving the general solution for the system dx 1 = dt t Hint, Solution 1 0 x. 1 1 (15.4)

870

15.6
Hint 15.1

Hints

Hint 15.2

Hint 15.3

Hint 15.4

Hint 15.5

Hint 15.6

Hint 15.7

Hint 15.8

Hint 15.9

Hint 15.10

871

Hint 15.11 Hint 15.12 Hint 15.13

872

15.7

Solutions

Solution 15.1 We consider an initial value problem. x = Ax 1 5 x, 1 3 2 , 1 x(0) = x0 1 1

The matrix has the distinct eigenvalues 1 = 1 , 2 = 1 + . The corresponding eigenvectors are x1 = x2 = 2+ . 1

The general solution of the system of dierential equations is x = c1 2 (1)t 2 + (1+)t e e +c2 . 1 1 2 cos(t) sin(t) t cos(t) + 2 sin(t) t e + e cos(t) sin(t)

We can take the real and imaginary parts of either of these solution to obtain real-valued solutions. 2 + (1+)t e = 1 x = c1

2 cos(t) sin(t) t cos(t) + 2 sin(t) t e +c2 e cos(t) sin(t) 2 1 1 0 c1 = 1, c1 c2 1 1

We apply the initial condition to determine the constants. =

c2 = 1

The solution subject to the initial condition is x= cos(t) 3 sin(t) t e . cos(t) sin(t)

Plotted in the phase plane, the solution spirals in to the origin as t increases. Both coordinates tend to zero as t . 873

Solution 15.2 We consider an initial value problem. 3 0 2 x = Ax 1 1 0 x, 2 1 0 1 0 x(0) = x0 0

The matrix has the distinct eigenvalues 1 = 2, 2 = 1 2, 3 = 1 + 2. The corresponding eigenvectors are 2 x1 = 2 , 1 2+ 2 x2 = 1 + 2 , 3 2 2 x3 = 1 2 . 3

The general solution of the system of dierential equations is 2 2 2 2 2+ x = c1 2 e2t +c2 1 + 2 e(1 2)t +c3 1 2 e(1+ 2)t . 1 3 3 We can take the real and imaginary parts of the second or third solution to obtain two real-valued solutions. 2 cos( 2t) + 2 sin( 2t) 2 cos( 2 sin( 2t) 2t) 2+ 2 1 + 2 e(1 2)t = cos( 2t) + 2 sin( 2t) et + 2 cos( 2t) + sin( 2t) et 3 3 cos( 2t) 3 sin( 2t) 2 cos( 2 sin( 2t) 2t) 2 cos( 2t) + 2 sin( 2t) 2 x = c1 2 e2t +c2 cos( 2t) + 2 sin( 2t) et +c3 2 cos( 2t) + sin( 2t) et 1 3 cos( 2t) 3 sin( 2t) 874

We apply the initial condition to determine the constants. 2 2 2 1 c1 2 1 2 c2 = 0 c3 0 1 3 0 1 1 5 c 1 = , c 2 = , c3 = 3 9 9 2 The solution subject to the initial condition is 2 cos( 4 2 sin( 2t) 2t) 2 1 1 x = 2 e2t + 4 cos( 2t) + sin( et . 2 2t) 3 6 1 2 cos( 2t) 5 2 sin( 2t) As t , all coordinates tend to innity. Plotted in the phase plane, the solution would spiral in to the origin. Solution 15.3 Homogeneous Solution, Method 1. We designate the inhomogeneous system of dierential equations x = Ax + g(t). First we nd homogeneous solutions. The characteristic equation for the matrix is () = 4 2 = 2 = 0 8 4

= 0 is an eigenvalue of multiplicity 2. The eigenvectors satisfy 4 2 8 4 1 2 = 0 . 0

Thus we see that there is only one linearly independent eigenvector. We choose = 1 . 2

875

One homogeneous solution is then x1 = 1 0t e = 2 1 . 2

We look for a second homogeneous solution of the form x2 = t + . We substitute this into the homogeneous equation. x2 = Ax2 = A(t + ) We see that and satisfy A = 0, A = . We choose to be the eigenvector that we found previously. The equation for is then 4 2 8 4 1 2 = 1 . 2

is determined up to an additive multiple of . We choose = Thus a second homogeneous solution is x2 = The general homogeneous solution of the system is xh = c1 1 t + c2 2 2t 1/2 876 1 0 t+ . 2 1/2 0 . 1/2

We can write this in matrix notation using the fundamental matrix (t). xh = (t)c = 1 t 2 2t 1/2 c1 c2

Homogeneous Solution, Method 2. The similarity transform C1 AC with C= will convert the matrix A= 1 0 2 1/2 4 2 8 4

to Jordan canonical form. We make the change of variables, y= The homogeneous system becomes dy = dt 1 0 4 2 y1 y2 The equation for y2 is y2 = 0. y2 = c2 The equation for y1 becomes y1 = c2 . y1 = c1 + c2 t 877 = 4 2 8 4 0 1 0 0 1 0 y 2 1/2 y1 y2 1 0 x. 2 1/2

The solution for y is then y = c1 1 t + c2 . 0 1

We multiply this by C to obtain the homogeneous solution for x. xh = c1 1 t + c2 2 2t 1/2

Inhomogeneous Solution. By the method of variation of parameters, a particular solution is xp = (t) xp = xp = xp = 1 t 2 2t 1/2 1 t 2 2t 1/2 1 t 2 2t 1/2 xp = 1 (t)g(t) dt. 1 4t 2t 4 2 t3 t2 dt dt

2t1 4t2 + t3 2t2 + 4t3 2 log t + 4t1 1 t2 2 2t1 2t2

1 2 2 log t + 2t1 2 t2 4 4 log t + 5t1

By adding 2 times our rst homogeneous solution, we obtain xp = 2 log t + 2t1 1 t2 2 4 log t + 5t1

The general solution of the system of dierential equations is x = c1 1 t 2 log t + 2t1 1 t2 2 + c2 + 2 2t 1/2 4 log t + 5t1

878

Solution 15.4 We consider an initial value problem. x = Ax The Jordan canonical form of the matrix is J= The solution of the initial value problem is x = eAt x0 . x = eAt x0 = S eJt S1 x0 = = 1 2 1 1 1 5 et 0 0 e3t 1 4 5 1 1 1 1 3 1 0 . 0 3 2 1 x, 5 4 x(0) = x0 1 3

et + e3t et +5 e3t x= 1 2 1 t 1 e + 1 2 1 3t e 5

Solution 15.5 We consider an initial value problem. 1 1 2 x = Ax 0 2 2 x, 1 1 3 The Jordan canonical form of the matrix is 1 0 0 J = 0 2 0 . 0 0 3 879 2 x(0) = x0 0 1

The solution of the initial value problem is x = eAt x0 . x = eAt x0 = S eJt S1 x0 t e 0 0 0 1 2 1 1 0 2 1 = 2 1 2 0 e2t 0 4 2 4 0 2 0 0 e3t 1 0 1 1 1 2 1 2t 2e 2 et +2 e2t = et

0 2 2 et + 2 e2t . x= 1 0 Solution 15.6 We consider an initial value problem. 1 5 x, 1 3 1 1

x = Ax

x(0) = x0

The Jordan canonical form of the matrix is 1 0 . 0 1 + 880

J=

The solution of the initial value problem is x = eAt x0 . x = eAt x0 = S eJt S1 x0 = = 2 2+ 1 1 e(1)t 0 (1+)t e 0 1 2 1 2 1 + 2 1 1

(cos(t) 3 sin(t)) et (cos(t) sin(t)) et

x= Solution 15.7 We consider an initial value problem.

1 t 3 t e sin(t) e cos(t) 1 1

3 0 2 x = Ax 1 1 0 x, 2 1 0

1 0 x(0) = x0 0

The Jordan canonical form of the matrix is 2 0 0 0 . J = 0 1 2 0 0 1 + 2 881

The solution of the initial value problem is x = eAt x0 .

x = eAt x0 = S eJt S1 x0 2t e 0 0 6 2+ 2 2 2 1 0 = 6 1 + 2 1 2 0 e(1 2)t 3 (1+ 2)t 3 3 3 e 0 0 2 2 2 1 1 0 1 52/2 1 22 4 + 2 6 0 1 + 5 2/2 1 + 2 2 4 2

2 cos( 4 2 sin( 2t) 2t) 2 1 1 x = 2 e2t + 4 cos( 2t) + sin( et . 2 2t) 3 6 1 2 cos( 2t) 5 2 sin( 2t) Solution 15.8 We consider an initial value problem.

x = Ax

1 4 x, 4 7

x(0) = x0

3 2

Method 1. Find Homogeneous Solutions. The matrix has the double eigenvalue 1 = 2 = 3. There is only 882

one corresponding eigenvector. We compute a chain of generalized eigenvectors. (A + 3I)2 x2 = 0 0x2 = 0 x2 = 1 0 4 4

(A + 3I)x2 = x1 x1 = The general solution of the system of dierential equations is x = c1 1 3t e +c2 1 4 1 t+ 4 0 e3t .

We apply the initial condition to determine the constants. 1 1 1 0 c1 = 2, The solution subject to the initial condition is x= 3 + 4t 3t e . 2 + 4t c1 c2 = 3 2

c2 = 1

Both coordinates tend to zero as t . Method 2. Use the Exponential Matrix. The Jordan canonical form of the matrix is J= 3 1 . 0 3 883

The solution of the initial value problem is x = eAt x0 . x = eAt x0 = S eJt S1 x0 = 1 1/4 1 0 e3t t e3t e3t 0 0 1 4 4 3 2

x= Solution 15.9 We consider an initial value problem.

3 + 4t 3t e . 2 + 4t

1 0 0 x = Ax 4 1 0 x, 3 6 2

1 x(0) = x0 2 30

Method 1. Find Homogeneous Solutions. The matrix has the distinct eigenvalues 1 = 1, 2 = 1, 3 = 2. The corresponding eigenvectors are 1 x1 = 2 , 5 0 x2 = 1 , 6 0 x3 = 0 . 1

The general solution of the system of dierential equations is 1 0 0 2 et +c2 1 et +c3 0 e2t . x = c1 5 6 1 884

We apply the initial condition to determine the constants. 1 0 0 c1 1 2 1 0 c2 = 2 5 6 1 c3 30 c1 = 1, c2 = 4, c3 = 11

The solution subject to the initial condition is 1 0 0 2 et 4 1 et 11 0 e2t . x= 5 6 1 As t , the rst coordinate vanishes, the second coordinate tends to and the third coordinate tends to Method 2. Use the Exponential Matrix. The Jordan canonical form of the matrix is 1 0 0 J = 0 1 0 . 0 0 2 The solution of the initial value problem is x = eAt x0 . x = eAt x0 = S eJt S1 x0 1 0 = 2 1 5 6 t e 0 0 0 1 0 0 1 1 0 0 et 0 2 1 0 2 2 1 0 0 e2t 7 6 1 30

1 0 0 x = 2 et 4 1 et 11 0 e2t . 5 6 1

885

Solution 15.10 1. (a) We compute the eigenvalues of the matrix. 1 1 1 2 1 1 = 3 + 62 12 + 8 = ( 2)3 () = 3 2 4 = 2 is an eigenvalue of multiplicity 3. The rank of the null space of A 2I is 1. (The rst two rows are linearly independent, but the third is a linear combination of the rst two.) 1 1 1 A 2I = 2 1 1 3 2 2 Thus there is only one eigenvector. 1 1 1 1 2 1 1 2 = 0 3 2 2 3 0 (1) = 1 1 (b) One solution of the system of dierential equations is x(1)

0 = 1 e2t . 1

(c) We substitute the form x = t e2t + e2t into the dierential equation. x = Ax e +2t e +2 e2t = At e2t +A e2t (A 2I) = 0, (A 2I) =
2t 2t

886

We already have a solution of the rst equation, we need the generalized eigenvector . Note that is only determined up to a constant times . Thus we look for the solution whose second component vanishes to simplify the algebra. (A 2I) = 1 1 1 1 0 2 1 1 0 = 1 3 2 2 3 1 1 + 3 = 0, 21 3 = 1, 1 0 = 1 31 + 23 = 1

A second linearly independent solution is 0 1 = 1 t e2t + 0 e2t . 1 1

x(2)

(d) To nd a third solution we substutite the form x = (t2 /2) e2t +t e2t + e2t into the dierential equation. x = Ax 2(t /2) e +( + 2)t e +( + 2) e2t = A(t2 /2) e2t +At e2t +A e2t (A 2I) = 0, (A 2I) = , (A 2I) =
2 2t 2t

We have already solved the rst two equations, we need the generalized eigenvector . Note that is only determined up to a constant times . Thus we look for the solution whose second component vanishes to 887

simplify the algebra. (A 2I) = 1 1 1 1 1 2 1 1 0 = 0 3 2 2 3 1 1 + 3 = 1, 21 3 = 0, 1 0 = 2 31 + 23 = 1

A third linearly independent solution is 0 1 1 1 (t2 /2) e2t + 0 t e2t + 0 e2t = 1 1 2

x(3)

2. (a) We compute the eigenvalues of the matrix. 5 3 2 8 5 4 = 3 + 32 3 + 1 = ( 1)3 4 3 3

() =

= 1 is an eigenvalue of multiplicity 3. The rank of the null space of A I is 2. (The second and third rows are multiples of the rst.) 4 3 2 A I = 8 6 4 4 3 2 888

Thus there are two eigenvectors. 4 3 2 1 8 6 4 2 = 0 4 3 2 3 1 0 (1) 0 , (2) = 2 = 2 3 Two linearly independent solutions of the dierential equation are 1 0 x(1) = 0 et , x(2) = 2 et . 2 3 (b) We substitute the form x = t et + et into the dierential equation. x = Ax e +t e + et = At et +A et (A I) = 0, (A I) =
t t

The general solution of the rst equation is a linear combination of the two solutions we found in the previous part. = c1 1 + c2 2 Now we nd the generalized eigenvector, . Note that is only determined up to a linear combination of 1 and 2 . Thus we can take the rst two components of to be zero. 4 3 2 0 1 0 8 6 4 0 = c1 0 + c2 2 4 3 2 3 2 3 23 = c1 , 43 = 2c2 , c1 = c2 , 23 = 2c1 3c2 c1 3 = 2

889

We see that we must take c1 = c2 in order to obtain a solution. We choose c1 = c2 = 2 A third linearly independent solution of the dierential equation is 2 0 (3) 4 t et + 0 et . x = 2 1 Solution 15.11 1. The characteristic polynomial of the matrix is 1 1 1 2 1 1 () = 8 5 3 = (1 )2 (3 ) + 8 10 5(1 ) 2(3 ) 8(1 ) = 3 2 + 4 + 4 = ( + 2)( + 1)( 2) Thus we see that the eigenvalues are = 2, 1, 2. The eigenvectors satisfy (A I) = 0. For = 2, we have (A + 2I) = 0. 3 1 1 1 0 2 2 = 0 3 1 8 5 1 3 0 If we take 3 = 1 then the rst two rows give us the system, 3 1 2 3 1 2 890 = 1 1

which has the solution 1 = 4/7, 2 = 5/7. For the rst eigenvector we choose: 4 = 5 7 For = 1, we have (A + I) = 0. 2 1 1 1 0 2 2 = 0 2 1 8 5 2 3 0 If we take 3 = 1 then the rst two rows give us the system, 2 1 2 2 1 2 = 1 1

which has the solution 1 = 3/2, 2 = 2. For the second eigenvector we choose: 3 4 = 2 For = 2, we have (A + I) = 0. 1 1 1 1 0 2 1 1 2 = 0 8 5 5 3 0 891

If we take 3 = 1 then the rst two rows give us the system, 1 1 2 1 1 2 = 1 1

which has the solution 1 = 0, 2 = 1. For the third eigenvector we choose: 0 1 = 1 In summary, the eigenvalues and eigenvectors are 3 0 4 5 , 4 , 1 = 7 2 1

= {2, 1, 2},

2. The matrix is diagonalized with the similarity transformation J = S1 AS, where S is the matrix with eigenvectors as columns: 4 3 0 4 1 S= 5 7 2 1 The matrix exponential, eAt is given by eA = S eJ S1 . 2t e 6 3 3 4 3 0 0 0 1 eA = 5 12 4 4 . 4 1 0 et 0 12 2t 18 13 1 7 2 1 0 0 e 892

2 e2t +3 et
8 et +3 et 2 7 e2t 4 et 3 et 2
2t

e2t + et
15 e2t 16 et +13 et 12 21 e2t 8 et 13 et 12

e2t + et

eAt = 5 e

15 e2t 16 et + et 12 21 e2t 8 et et 12

The solution of the initial value problem is eAt x0 . 3. The general solution of the Euler equation is 4 3 0 5 t2 + c2 4 t1 + c3 1 t2 . c1 7 2 1 We could also write the solution as x = tA c eA log t c, Solution 15.12 1. The characteristic polynomial of the matrix is 2 0 1 0 2 0 () = 0 1 3 = (2 )2 (3 ) Thus we see that the eigenvalues are = 2, 2, 3. Consider 0 0 1 A 2I = 0 0 0 . 0 1 3 Since rank(nullspace(A 2I)) = 1 there is one eigenvector and one generalized eigenvector of rank two for 893

= 2. The generalized eigenvector of rank two satises (A 2I)2 2 = 0 0 1 1 0 0 0 2 = 0 0 1 1 We choose the solution 0 2 = 1 . 1 1 1 = (A 2I) 2 = 0 . 0 The eigenvector for = 3 satises (A 3I)2 = 0 1 0 1 0 1 0 = 0 0 1 0 We choose the solution 1 0 . = 1 0 1 1 0 , 1 , 0 . = 0 1 1 894

The eigenvector for = 2 is

The eigenvalues and generalized eigenvectors are = {2, 2, 3},

The matrix of eigenvectors and its inverse is 1 0 1 S = 0 1 0 , 0 1 1 The Jordan canonical form of the matrix, which satises 2 J = 0 0 Recall that the function of a Jordan block is: 0 f 0 0 and that the function of a matrix J1 0 f 0 0 1 0 0 0 1 0

S1

1 1 1 = 0 1 0 . 0 1 1

J = S1 AS is 1 0 2 0 0 3

() f () f () f () f 1! 0 2! 3! f () f () 0 0 f () = 1! 2! , f () 0 1 0 f () 1! 0 0 0 f ()

in Jordan canonical form is 0 0 0 f (J1 ) 0 0 0 J2 0 0 0 f (J2 ) 0 0 = . 0 J 3 0 0 0 f (J3 ) 0 0 0 J4 0 0 0 f (J4 ) et , which has the derivative f () = t et . Thus 0 0 e3t

We want to compute eJt so we consider the function f () = we see that 2t e t e2t eJt = 0 e2t 0 0 895

The exponential matrix is eAt = S eJt S1 , eAt e2t (1 + t) e2t + e3t e2t + e3t . e2t 0 =0 2t 3t 3t e 0 e +e

The general solution of the homogeneous dierential equation is x = eAt C. 2. The solution of the inhomogeneous dierential equation subject to the initial condition is
t

x = eAt 0 + eAt
0 t

eA g( ) d eA g( ) d

x = eAt
0

Solution 15.13 1. dx 1 = Ax dt t x1 a b t = x2 c d The rst component of this equation is tx1 = ax1 + bx2 . 896

x1 x2

We dierentiate and multiply by t to obtain a second order coupled equation for x1 . We use (15.4) to eliminate the dependence on x2 . t2 x1 + tx1 = atx1 + btx2 t2 x1 + (1 a)tx1 = b(cx1 + dx2 ) t2 x1 + (1 a)tx1 bcx1 = d(tx1 ax1 ) t2 x1 + (1 a d)tx1 + (ad bc)x1 = 0 Thus we see that x1 satises a second order, Euler equation. By symmetry we see that x2 satises, t2 x2 + (1 b c)tx2 + (bc ad)x2 = 0. 2. We substitute x = at into (15.4). 1 at1 = Aat t Aa = a Thus we see that x = at is a solution if is an eigenvalue of A with eigenvector a. 3. Suppose that = is an eigenvalue of multiplicity 2. If = has two linearly independent eigenvectors, a and b then at and bt are linearly independent solutions. If = has only one linearly independent eigenvector, a, then at is a solution. We look for a second solution of the form x = t log t + t . Substituting this into the dierential equation yields t1 log t + t1 + t1 = At1 log t + At1 We equate coecients of t1 log t and t1 to determine and . (A I) = 0, 897 (A I) =

These equations have solutions because = has generalized eigenvectors of rst and second order. Note that the change of independent variable = log t, y( ) = x(t), will transform (15.4) into a constant coecient system. dy = Ay d Thus all the methods for solving constant coecient systems carry over directly to solving (15.4). In the case of eigenvalues with multiplicity greater than one, we will have solutions of the form, t , t log t + t , t (log t)2 + t log t + t , ...,

analogous to the form of the solutions for a constant coecient system, e , 4. Method 1. Now we consider e + e , 2 e + e + e , ....

dx 1 = dt t

1 0 x. 1 1

The characteristic polynomial of the matrix is () = 1 0 = (1 )2 . 1 1

= 1 is an eigenvalue of multiplicity 2. The equation for the associated eigenvectors is 0 0 1 0 1 2 = 0 . 0

There is only one linearly independent eigenvector, which we choose to be a= 0 . 1

898

One solution of the dierential equation is x1 = We look for a second solution of the form x2 = at log t + t. satises the equation (A I) = 0 0 = 1 0 1 . 0 0 . 1 0 t. 1

The solution is determined only up to an additive multiple of a. We choose = Thus a second linearly independent solution is x2 = The general solution of the dierential equation is x = c1 0 t + c2 1 0 1 t log t + t . 1 0 1 0 t log t + t. 1 0

Method 2. Note that the matrix is lower triangular. x1 x2 We have an uncoupled equation for x1 . 1 x1 = x1 t x1 = c1 t 899 = 1 t 1 0 1 1 x1 x2 (15.5)

By substituting the solution for x1 into (15.5), we obtain an uncoupled equation for x2 . x2 = 1 (c1 t + x2 ) t 1 x2 x2 = c1 t 1 c1 x2 = t t

1 x2 = c1 log t + c2 t x2 = c1 t log t + c2 t Thus the solution of the system is x= x = c1 c1 t , c1 t log t + c2 t t 0 + c2 , t log t t

which is equivalent to the solution we obtained previously.

900

Chapter 16 Theory of Linear Ordinary Dierential Equations


A little partyin is good for the soul. -Matt Metz

16.1

Exact Equations

Exercise 16.1 Consider a second order, linear, homogeneous dierential equation: P (x)y + Q(x)y + R(x)y = 0. Show that P Q + R = 0 is a necessary and sucient condition for this equation to be exact. Hint, Solution Exercise 16.2 Determine an equation for the integrating factor (x) for Equation 16.1. 901 (16.1)

Hint, Solution Exercise 16.3 Show that y + xy + y = 0 is exact. Find the solution. Hint, Solution

16.2

Nature of Solutions

Result 16.2.1 Consider the nth order ordinary dierential equation of the form dn y dn1 y dy L[y] = n + pn1 (x) n1 + + p1 (x) + p0 (x)y = f (x). dx dx dx (16.2)

If the coecient functions pn1 (x), . . . , p0 (x) and the inhomogeneity f (x) are continuous on some interval a < x < b then the dierential equation subject to the conditions, y(x0 ) = v0 , y (x0 ) = v1 , ... y (n1) (x0 ) = vn1 , a < x0 < b,

has a unique solution on the interval.


Exercise 16.4 On what intervals do the following problems have unique solutions? 1. xy + 3y = x 2. x(x 1)y + 3xy + 4y = 2 902

3. ex y + x2 y + y = tan x Hint, Solution Linearity of the Operator. The dierential operator L is linear. To verify this, dn1 d dn L[cy] = n (cy) + pn1 (x) n1 (cy) + + p1 (x) (cy) + p0 (x)(cy) dx dx dx dn1 d dn = c n y + cpn1 (x) n1 y + + cp1 (x) y + cp0 (x)y dx dx dx = cL[y] dn dn1 d L[y1 + y2 ] = n (y1 + y2 ) + pn1 (x) n1 (y1 + y2 ) + + p1 (x) (y1 + y2 ) + p0 (x)(y1 + y2 ) dx dx dx dn dn1 d = n (y1 ) + pn1 (x) n1 (y1 ) + + p1 (x) (y1 ) + p0 (x)(y1 ) dx dx dx dn1 d dn + n (y2 ) + pn1 (x) n1 (y2 ) + + p1 (x) (y2 ) + p0 (x)(y2 ) dx dx dx = L[y1 ] + L[y2 ]. Homogeneous Solutions. The general homogeneous equation has the form L[y] = dn1 y dy dn y + pn1 (x) n1 + + p1 (x) + p0 (x)y = 0. n dx dx dx

From the linearity of L, we see that if y1 and y2 are solutions to the homogeneous equation then c1 y1 + c2 y2 is also a solution, (L[c1 y1 + c2 y2 ] = 0). On any interval where the coecient functions are continuous, the nth order linear homogeneous equation has n linearly independent solutions, y1 , y2 , . . . , yn . (We will study linear independence in Section 16.4.) The general solution to the homogeneous problem is then yh = c1 y1 + c2 y2 + + cn yn . 903

Particular Solutions. Any function, yp , that satises the inhomogeneous equation, L[yp ] = f (x), is called a particular solution or particular integral of the equation. Note that for linear dierential equations the particular solution is not unique. If yp is a particular solution then yp +yh is also a particular solution where yh is any homogeneous solution. The general solution to the problem L[y] = f (x) is the sum of a particular solution and a linear combination of the homogeneous solutions y = yp + c1 y1 + + cn yn . Example 16.2.1 Consider the dierential equation y y = 1. You can verify that two homogeneous solutions are ex and 1. A particular solution is x. Thus the general solution is y = x + c1 ex +c2 . Exercise 16.5 Suppose you are able to nd three linearly independent particular solutions u1 (x), u2 (x) and u3 (x) of the second order linear dierential equation L[y] = f (x). What is the general solution? Hint, Solution Real-Valued Solutions. If the coecient function and the inhomogeneity in Equation 16.2 are real-valued, then the general solution can be written in terms of real-valued functions. Let y be any, homogeneous solution, (perhaps complex-valued). By taking the complex conjugate of the equation L[y] = 0 we show that y is a homogeneous solution as well. L[y] = 0 L[y] = 0 y (n) + pn1 y (n1) + + p0 y = 0 y (n) + pn1 y (n1) + + p0 y = 0 L [] = 0 y 904

For the same reason, if yp is a particular solution, then yp is a particular solution as well. Since the real and imaginary parts of a function y are linear combinations of y and y , (y) = y+y , 2 (y) = yy , 2

if y is a homogeneous solution then both y and (y) are homogeneous solutions. Likewise, if yp is a particular solution then (yp ) is a particular solution. yp + yp f f = + =f 2 2 2

L [ (yp )] = L

Thus we see that the homogeneous solution, the particular solution and the general solution of a linear dierential equation with real-valued coecients and inhomogeneity can be written in terms of real-valued functions.

Result 16.2.2 The dierential equation dn y dn1 y dy L[y] = n + pn1 (x) n1 + + p1 (x) + p0 (x)y = f (x) dx dx dx with continuous coecients and inhomogeneity has a general solution of the form y = yp + c1 y1 + + cn yn where yp is a particular solution, L[yp ] = f , and the yk are linearly independent homogeneous solutions, L[yk ] = 0. If the coecient functions and inhomogeneity are real-valued, then the general solution can be written in terms of real-valued functions.

905

16.3

Transformation to a First Order System

Any linear dierential equation can be put in the form of a system of rst order dierential equations. Consider y (n) + pn1 y (n1) + + p0 y = f (x). We introduce the functions, y1 = y, y2 = y , ,..., yn = y (n1) .

The dierential equation is equivalent to the system y1 = y2 y2 = y3 . . .=. . . yn = f (x) pn1 yn p0 y1 . The rst order system is more useful when numerically solving the dierential equation. Example 16.3.1 Consider the dierential equation y + x2 y + cos x y = sin x. The corresponding system of rst order equations is y1 = y2 y2 = sin x x2 y2 cos x y1 .

906

16.4
16.4.1

The Wronskian
Derivative of a Determinant.

Before investigating the Wronskian, we will need a preliminary result from matrix theory. Consider an n n matrix A whose elements aij (x) are functions of x. We will denote the determinant by [A(x)]. We then have the following theorem.

Result 16.4.1 Let aij (x), the elements of the matrix A, be dierentiable functions of x. Then n d [A(x)] = k [A(x)] dx
k=1

where k [A(x)] is the determinant of the matrix A with the k th row replaced by the derivative of the k th row.
Example 16.4.1 Consider the the matrix A(x) = x x2 x2 x4

The determinant is x5 x4 thus the derivative of the determinant is 5x4 4x3 . To check the theorem, d x x2 d [A(x)] = dx dx x2 x4 1 2x x x2 = 2 4 + x x 2x 4x3 = x4 2x3 + 4x4 2x3 = 5x4 4x3 .

907

16.4.2

The Wronskian of a Set of Functions.

A set of functions {y1 , y2 , . . . , yn } is linearly dependent on an interval if there are constants c1 , . . . , cn not all zero such that c1 y1 + c2 y2 + + cn yn = 0 (16.3) identically on the interval. The set is linearly independent if all of the constants must be zero to satisfy c1 y1 + cn yn = 0 on the interval. Consider a set of functions {y1 , y2 , . . . , yn } that are linearly dependent on a given interval and n 1 times dierentiable. There are a set of constants, not all zero, that satisfy equation 16.3 Dierentiating equation 16.3 n 1 times gives the equations, c1 y1 + c2 y2 + + cn yn = 0 c1 y1 + c2 y2 + + cn yn = 0 c1 y1
(n1)

+ c2 y2

(n1)

(n1) + + cn yn = 0.

We could write the problem to nd the constants as y1 y2 y1 y2 y y2 1 . . . . . . y1


(n1)

y2

(n1)

c1 c2 c3 = 0 . ... . . (n1) cn . . . yn ... ... ... .. . yn yn yn

From linear algebra, we know that this equation has a solution for a nonzero constant vector only if the determinant of the matrix is zero. Here we dene the Wronskian ,W (x), of a set of functions. y1 y1 . . . y1
(n1)

W (x) =

y2 y2 . . . y2
(n1)

... yn ... yn .. . ... (n1) . . . yn

908

Thus if a set of functions is linearly dependent on an interval, then the Wronskian is identically zero on that interval. Alternatively, if the Wronskian is identically zero, then the above matrix equation has a solution for a nonzero constant vector. This implies that the the set of functions is linearly dependent.

Result 16.4.2 The Wronskian of a set of functions vanishes identically over an interval if and only if the set of functions is linearly dependent on that interval. The Wronskian of a set of linearly independent functions does not vanish except possibly at isolated points.
Example 16.4.2 Consider the set, {x, x2 }. The Wronskian is x x2 1 2x

W (x) =

= 2x2 x2 = x2 . Thus the functions are independent. Example 16.4.3 Consider the set {sin x, cos x, ex }. The Wronskian is ex sin x cos x W (x) = cos x sin x ex . sin x cos x ex Since the last row is a constant multiple of the rst row, the determinant is zero. The functions are dependent. We could also see this with the identity ex = cos x + sin x.

909

16.4.3

The Wronskian of the Solutions to a Dierential Equation

Consider the nth order linear homogeneous dierential equation y (n) + pn1 (x)y (n1) + + p0 (x)y = 0. Let {y1 , y2 , . . . , yn } be any set of n linearly independent solutions. Let Y (x) be the matrix such that W (x) = [Y (x)]. Now lets dierentiate W (x). W (x) = =
k=1

d [Y (x)] dx
n

k [Y (x)]

We note that the all but the last term in this sum is zero. To see this, lets take a look at the rst term. y1 y1 . . . y1
(n1)

1 [Y (x)] =

y2 y2 . . . y2
(n1)

yn yn . .. . . . (n1) yn

The rst two rows in the matrix are identical. Since the rows are dependent, the determinant is zero. The last term in the sum is y1 y2 yn . . . ... . . . . . . n [Y (x)] = (n2) (n2) (n2) . y1 y2 yn (n) (n) (n) y2 yn y1 p0 (x)yi . Recalling that we In the last row of this matrix we make the substitution yi = pn1 (x)yi can add a multiple of a row to another without changing the determinant, we add p0 (x) times the rst row, and p1 (x) 910
(n) (n1)

times the second row, etc., to the last row. Thus we have the determinant, y1 . . .
(n2)

W (x) =

y2 . . .
(n2)

y1 y2 (n1) (n1) pn1 (x)y1 pn1 (x)y2 y1 . . .


(n2)

yn . .. . . . (n2) yn (n1) pn1 (x)yn yn . . .


(n2)

= pn1 (x)

y2 . . .
(n2)

...

y1 y2 yn (n1) (n1) (n1) y1 y2 yn

= pn1 (x)W (x) Thus the Wronskian satises the rst order dierential equation, W (x) = pn1 (x)W (x). Solving this equation we get a result known as Abels formula. W (x) = c exp pn1 (x) dx

Thus regardless of the particular set of solutions that we choose, we can compute their Wronskian up to a constant factor.

Result 16.4.3 The Wronskian of any linearly independent set of solutions to the equation y (n) + pn1 (x)y (n1) + + p0 (x)y = 0 is, (up to a multiplicative constant), given by W (x) = exp pn1 (x) dx .
911

Example 16.4.4 Consider the dierential equation y 3y + 2y = 0. The Wronskian of the two independent solutions is W (x) = c exp = c e3x . For the choice of solutions {ex , e2x }, the Wronskian is W (x) = ex e2x = 2 e3x e3x = e3x . ex 2 e2x 3 dx

16.5

Well-Posed Problems
dn1 y dy dn y + pn1 (x) n1 + + p1 (x) + p0 (x)y = f (x) n dx dx dx y(x0 ) = v1 , y (x0 ) = v2 , . . . , y (n1) (x0 ) = vn

Consider the initial value problem for an nth order linear dierential equation.

Since the general solution to the dierential equation is a linear combination of the n homogeneous solutions plus the particular solution y = yp + c1 y1 + c2 y2 + + cn yn , the problem to nd the constants ci can be written yp (x0 ) y1 (x0 ) y2 (x0 ) ... yn (x0 ) c1 v1 y (x0 ) y2 (x0 ) ... yn (x0 ) c2 yp (x0 ) v2 1 . . . + = . . . .. . . . . . . . . ... . . . (n1) (n1) (n1) (n1) cn vn (x0 ) y2 (x0 ) . . . yn y1 (x0 ) yp (x0 ) 912

From linear algebra we know that this system of equations has a unique solution only if the determinant of the matrix is nonzero. Note that the determinant of the matrix is just the Wronskian evaluated at x0 . Thus if the Wronskian vanishes at x0 , the initial value problem for the dierential equation either has no solutions or innitely many solutions. Such problems are said to be ill-posed. From Abels formula for the Wronskian W (x) = exp pn1 (x) dx ,

we see that the only way the Wronskian can vanish is if the value of the integral goes to . Example 16.5.1 Consider the initial value problem 2 2 y y + 2 y = 0, x x 2 dx x vanishes at x = 0. Thus this problem is not well-posed. The general solution of the dierential equation is W (x) = exp The Wronskian y(0) = y (0) = 1.

= exp (2 log x) = x2

y = c1 x + c2 x2 . We see that the general solution cannot satisfy the initial conditions. If instead we had the initial conditions y(0) = 0, y (0) = 1, then there would be an innite number of solutions. Example 16.5.2 Consider the initial value problem y The Wronskian W (x) = exp 0 dx =1 2 y = 0, x2 y(0) = y (0) = 1.

913

does not vanish anywhere. However, this problem is not well-posed. The general solution, y = c1 x1 + c2 x2 , cannot satisfy the initial conditions. Thus we see that a non-vanishing Wronskian does not imply that the problem is well-posed.

Result 16.5.1 Consider the initial value problem dn y dn1 y dy + pn1 (x) n1 + + p1 (x) + p0 (x)y = 0 dxn dx dx y(x0 ) = v1 , y (x0 ) = v2 , . . . , y (n1) (x0 ) = vn . If the Wronskian W (x) = exp pn1 (x) dx

vanishes at x = x0 then the problem is ill-posed. The problem may be ill-posed even if the Wronskian does not vanish.

16.6

The Fundamental Set of Solutions

Consider a set of linearly independent solutions {u1 , u2 , . . . , un } to an nth order linear homogeneous dierential equation. This is called the fundamental set of solutions at x0 if they satisfy the relations u1 (x0 ) = 1 u1 (x0 ) = 0 . . . u1
(n1)

u2 (x0 ) = 0 u2 (x0 ) = 1 . . .
(n1)

... ... .. .

un (x0 ) = 0 un (x0 ) = 0 . . .
(n1)

(x0 ) = 0 u2

(x0 ) = 0 . . . un 914

(x0 ) = 1

Knowing the fundamental set of solutions is handy because it makes the task of solving an initial value problem trivial. Say we are given the initial conditions, y(x0 ) = v1 , y (x0 ) = v2 , ..., y (n1) (x0 ) = vn .

If the ui s are a fundamental set then the solution that satises these constraints is just y = v1 u1 (x) + v2 u2 (x) + + vn un (x). Of course in general, a set of solutions is not the fundamental set. If the Wronskian of the solutions is nonzero and nite we can generate a fundamental set of solutions that are linear combinations of our original set. Consider the case of a second order equation Let {y1 , y2 } be two linearly independent solutions. We will generate the fundamental set of solutions, {u1 , u2 }. u1 u2 = c11 c12 c21 c22 y1 y2

For {u1 , u2 } to satisfy the relations that dene a fundamental set, it must satisfy the matrix equation u1 (x0 ) u1 (x0 ) u2 (x0 ) u2 (x0 ) = c11 c12 c21 c22 = y1 (x0 ) y1 (x0 ) y2 (x0 ) y2 (x0 )
1

1 0 0 1

c11 c12 c21 c22

y1 (x0 ) y1 (x0 ) y2 (x0 ) y2 (x0 )

If the Wronskian is non-zero and nite, we can solve for the constants, cij , and thus nd the fundamental set of solutions. To generalize this result to an equation of order n, simply replace all the 2 2 matrices and vectors of length 2 with n n matrices and vectors of length n. I presented the case of n = 2 simply to save having to write out all the ellipses involved in the general case. (It also makes for easier reading.) Example 16.6.1 Two linearly independent solutions to the dierential equation y + y = 0 are y1 = ex and y2 = ex . y1 (0) y1 (0) y2 (0) y2 (0) 915 = 1 1 i

To nd the fundamental set of solutions, {u1 , u2 }, at x = 0 we solve the equation c11 c12 c21 c22 c11 c12 c21 c22 The fundamental set is ex + ex , 2 Using trigonometric identities we can rewrite these as u1 = u1 = cos x, = = 1 2 1 1
1

1 1 ex ex . 2

u2 =

u2 = sin x.

Result 16.6.1 The fundamental set of solutions at x = x0 , {u1 , u2 , . . . , un }, to an nth order linear dierential equation, satisfy the relations u1 (x0 ) = 1 u2 (x0 ) = 0 u1 (x0 ) = 0 u2 (x0 ) = 1 . . . . . . (n1) (n1) u1 (x0 ) = 0 u2 (x0 ) = 0 ... un (x0 ) = 0 ... un (x0 ) = 0 . ... . . (n1) . . . un (x0 ) = 1.

If the Wronskian of the solutions is nonzero and nite at the point x0 then you can generate the fundamental set of solutions from any linearly independent set of solutions.
Exercise 16.6 Two solutions of y y = 0 are ex and ex . Show that the solutions are independent. Find the fundamental set of solutions at x = 0. Hint, Solution 916

16.7

Adjoint Equations

For the nth order linear dierential operator dn y dn1 y + pn1 n1 + + p0 y dxn dx (where the pj are complex-valued functions) we dene the adjoint of L L[y] = pn L [y] = (1)n dn1 dn (pn y) + (1)n1 n1 (pn1 y) + + p0 y. dx dxn

Here f denotes the complex conjugate of f . Example 16.7.1 1 L[y] = xy + y + y x has the adjoint L [y] = d2 d 1 [xy] y +y 2 dx dx x 1 1 = xy + 2y y + 2 y + y x x 1 1 = xy + 2 y + 1+ 2 x x 1 x 1 x2

y.

Taking the adjoint of L yields L [y] = d2 d [xy] 2 dx dx 2 1 x y + 1+ 1 x2 y 1 x2 y

= xy + 2y 2 1 = xy + y + y. x

y+ 1+

917

Thus by taking the adjoint of L , we obtain the original operator. In general, L = L. Consider L[y] = pn y (n) + + p0 y. If each of the pk is k times continuously dierentiable and u and v are n times continuously dierentiable on some interval, then on that interval vL[u] uL [v] = where B[u, v], the bilinear concomitant, is the bilinear form
n

d B[u, v] dx

B[u, v] =
m=1 j+k=m1 j0,k0

(1)j u(k) (pm v)(j) .

This equation is known as Lagranges identity. If L is a second order operator then vL[u] uL [v] = d up1 v + u p2 v u(p2 v) dx = u p2 v + u p1 v + u p2 v + (2p2 + p1 )v + (p2 + p1 )v .

Example 16.7.2 Verify Lagranges identity for the second order operator, L[y] = p2 y + p1 y + p0 y. vL[u] uL [v] = v(p2 u + p1 u + p0 u) u d2 d (p2 v) (p1 v) + p0 v 2 dx dx

= v(p2 u + p1 u + p0 u) u(p2 v + (2p2 p1 )v + (p2 p1 + p0 )v) = u p2 v + u p1 v + u p2 v + (2p2 + p1 )v + (p2 + p1 )v . We will not verify Lagranges identity for the general case. 918

Integrating Lagranges identity on its interval of validity gives us Greens formula.


b

vL[u] uL [v] dx = B[u, v]


a

x=b

B[u, v]

x=a

Result 16.7.1 The adjoint of the operator dn y dn1 y L[y] = pn n + pn1 n1 + + p0 y dx dx


n1 dn n1 d (pn1 y) + + p0 y. (pn y) + (1) L [y] = (1) dxn1 dxn If each of the pk is k times continuously dierentiable and u and v are n times continuously dierentiable, then Lagranges identity states n

is dened

d d vL[y] uL [v] = B[u, v] = dx dx m=1

(1)j u(k) (pm v)(j) .


j+k=m1 j0,k0

Integrating Lagranges identity on its domain of validity yields Greens formula,


b

vL[u] uL [v] dx = B[u, v]


a

x=b

B[u, v]

x=a

919

16.8

Additional Exercises

Exact Equations Nature of Solutions Transformation to a First Order System The Wronskian Well-Posed Problems The Fundamental Set of Solutions Adjoint Equations
Exercise 16.7 Find the adjoint of the Bessel equation of order , x2 y + xy + (x2 2 )y = 0, and the Legendre equation of order , (1 x2 )y 2xy + ( + 1)y = 0. Hint, Solution Exercise 16.8 Find the adjoint of x2 y xy + 3y = 0. Hint, Solution

920

16.9
Hint 16.1

Hints

Hint 16.2

Hint 16.3

Hint 16.4

Hint 16.5 The dierence of any two of the ui s is a homogeneous solution. Hint 16.6

Exact Equations Nature of Solutions Transformation to a First Order System The Wronskian Well-Posed Problems The Fundamental Set of Solutions Adjoint Equations
Hint 16.7

921

Hint 16.8

922

16.10

Solutions

Solution 16.1 The second order, linear, homogeneous dierential equation is P (x)y + Q(x)y + R(x)y = 0. An exact equation can be written in the form: d [a(x)y + b(x)y] = 0. dx If Equation 16.4 is exact, then we can write it in the form: d [P (x)y + f (x)y] = 0 dx for some function f (x). We carry out the dierentiation to write the equation in standard form: P (x)y + (P (x) + f (x)) y + f (x)y = 0 We equate the coecients of Equations 16.4 and 16.5 to obtain a set of equations. P (x) + f (x) = Q(x), f (x) = R(x). (16.5) (16.4)

In order to eliminate f (x), we dierentiate the rst equation and substitute in the expression for f (x) from the second equation. This gives us a necessary condition for Equation 16.4 to be exact: P (x) Q (x) + R(x) = 0 (16.6)

Now we demonstrate that Equation 16.6 is a sucient condition for exactness. Suppose that Equation 16.6 holds. Then we can replace R by Q P in the dierential equation. P y + Qy + (Q P )y = 0 923

We recognize the right side as an exact dierential. (P y + (Q P )y) = 0 Thus Equation 16.6 is a sucient condition for exactness. We can integrate to reduce the problem to a rst order dierential equation. P y + (Q P )y = c Solution 16.2 Suppose that there is an integrating factor (x) that will make P (x)y + Q(x)y + R(x)y = 0 exact. We multiply by this integrating factor. (x)P (x)y + (x)Q(x)y + (x)R(x)y = 0. We apply the exactness condition from Exercise 16.1 to obtain a dierential equation for the integrating factor. (P ) (Q) + R = 0 P + 2 P + P Q Q + R = 0 P + (2P Q) + (P Q + R) = 0 Solution 16.3 We consider the dierential equation, y + xy + y = 0. Since (1) (x) + 1 = 0 924 (16.7)

we see that this is an exact equation. We rearrange terms to form exact derivatives and then integrate. (y ) + (xy) = 0 y + xy = c d x2 /2 2 e y = c ex /2 dx y = c ex Solution 16.4 Consider the initial value problem, y + p(x)y + q(x)y = f (x), y(x0 ) = y0 , y (x0 ) = y1 . If p(x), q(x) and f (x) are continuous on an interval (a . . . b) with x0 (a . . . b), then the problem has a unique solution on that interval. 1. xy + 3y = x 3 y + y=1 x Unique solutions exist on the intervals ( . . . 0) and (0 . . . ). 2. x(x 1)y + 3xy + 4y = 2 4 2 3 y + y + y= x1 x(x 1) x(x 1) Unique solutions exist on the intervals ( . . . 0), (0 . . . 1) and (1 . . . ). 925
2 /2

ex

2 /2

dx + d ex

2 /2

3. ex y + x2 y + y = tan x y + x2 ex y + ex y = ex tan x Unique solutions exist on the intervals Solution 16.5 We know that the general solution is y = yp + c1 y1 + c2 y2 , where yp is a particular solution and y1 and y2 are linearly independent homogeneous solutions. Since yp can be any particular solution, we choose yp = u1 . Now we need to nd two homogeneous solutions. Since L[ui ] = f (x), L[u1 u2 ] = L[u2 u3 ] = 0. Finally, we note that since the ui s are linearly independent, y1 = u1 u2 and y2 = u2 u3 are linearly independent. Thus the general solution is y = u1 + c1 (u1 u2 ) + c2 (u2 u3 ). Solution 16.6 The Wronskian of the solutions is W (x) = ex ex = 2. ex ex
(2n1) 2

. . . (2n+1) 2

for n Z.

Since the Wronskian is nonzero, the solutions are independent. The fundamental set of solutions, {u1 , u2 }, is a linear combination of ex and ex . u1 u2 = c11 c12 c21 c22 926 ex ex

The coecients are c11 c12 c21 c22 = e0 e0 e0 e0


1 1

1 1 = 1 1 1 1 1 = 2 1 1 1 1 1 = 2 1 1 1 u1 = (ex + ex ), 2 The fundamental set of solutions at x = 0 is {cosh x, sinh x}. 1 u2 = (ex ex ). 2

Exact Equations Nature of Solutions Transformation to a First Order System The Wronskian Well-Posed Problems The Fundamental Set of Solutions Adjoint Equations
Solution 16.7 1. The Bessel equation of order is x2 y + xy + (x2 2 )y = 0. 927

The adjoint equation is x2 + (4x x) + (2 1 + x2 2 ) = 0 x2 + 3x + (1 + x2 2 ) = 0. 2. The Legendre equation of order is (1 x2 )y 2xy + ( + 1)y = 0 The adjoint equation is (1 x2 ) + (4x + 2x) + (2 + 2 + ( + 1)) = 0 (1 x2 ) 2x + ( + 1) = 0 Solution 16.8 The adjoint of x2 y xy + 3y = 0 is d d2 2 (x y) + (xy) + 3y = 0 dx2 dx (x2 y + 4xy + 2y) + (xy + y) + 3y = 0 x2 y + 5xy + 6y = 0.

928

16.11

Quiz

Problem 16.1 What is the dierential equation whose solution is the two parameter family of curves y = c1 sin(2x + c2 )? Solution

929

16.12

Quiz Solutions

Solution 16.1 We take the rst and second derivative of y = c1 sin(2x + c2 ). y = 2c1 cos(2x + c2 ) y = 4c1 sin(2x + c2 ) This gives us three equations involving x, y, y , y and the parameters c1 and c2 . We eliminate the the parameters to obtain the dierential equation. Clearly we have, y + 4y = 0.

930

Chapter 17 Techniques for Linear Dierential Equations


My new goal in life is to take the meaningless drivel out of human interaction. -Dave Ozenne The nth order linear homogeneous dierential equation can be written in the form: y (n) + an1 (x)y (n1) + + a1 (x)y + a0 (x)y = 0. In general it is not possible to solve second order and higher linear dierential equations. In this chapter we will examine equations that have special forms which allow us to either reduce the order of the equation or solve it.

17.1

Constant Coecient Equations


y (n) + an1 y (n1) + + a1 y + a0 y = 0. 931

The nth order constant coecient dierential equation has the form:

We will nd that solving a constant coecient dierential equation is no more dicult than nding the roots of a polynomial. For notational simplicity, we will rst consider second order equations. Then we will apply the same techniques to higher order equations.

17.1.1

Second Order Equations

Factoring the Dierential Equation. Consider the second order constant coecient dierential equation: y + 2ay + by = 0. Just as we can factor a second degree polynomial: 2 + 2a + b = ( )( ), = a + we can factor Equation 17.1. d d2 + 2a +b y = 2 dx dx d dx d y dx a2 b and = a a2 b, (17.1)

Once we have factored the dierential equation, we can solve it by solving a series of two rst order dierential equations. d We set u = dx y to obtain a rst order equation: d u = 0, dx which has the solution: u = c1 ex . To nd the solution of Equation 17.1, we solve d y = u = c1 ex . dx 932

We multiply by the integrating factor and integrate. d x e y = c1 e()x dx y = c1 ex We rst consider the case that and are distinct. y = c1 ex 1 e()x +c2 ex e()x dx + c2 ex

We choose new constants to write the solution in a simpler form. y = c1 ex +c2 ex Now we consider the case = . y = c1 ex 1 dx + c2 ex

y = c1 x ex +c2 ex The solution of Equation 17.1 is y= c1 ex +c2 ex , = , c1 ex +c2 x ex , = . (17.2)

Example 17.1.1 Consider the dierential equation: y + y = 0. To obtain the general solution, we factor the equation and apply the result in Equation 17.2. d d + y =0 dx dx y = c1 ex +c2 ex . 933

Example 17.1.2 Next we solve y = 0. d d 0 0 y =0 dx dx y = c1 e0x +c2 x e0x y = c1 + c2 x Substituting the Form of the Solution into the Dierential Equation. Note that if we substitute y = ex into the dierential equation (17.1), we will obtain the quadratic polynomial (17.1.1) for . y + 2ay + by = 0 e
2 x

+2a ex +b ex = 0 2 + 2a + b = 0

This gives us a supercially dierent method for solving constant coecient equations. We substitute y = ex into the dierential equation. Let and be the roots of the quadratic in . If the roots are distinct, then the linearly independent solutions are y1 = ex and y2 = ex . If the quadratic has a double root at = , then the linearly independent solutions are y1 = ex and y2 = x ex . Example 17.1.3 Consider the equation: y 3y + 2y = 0. The substitution y = ex yields 2 3 + 2 = ( 1)( 2) = 0. Thus the solutions are ex and e2x . Example 17.1.4 Next consider the equation: y 2y + 4y = 0. 934

The substitution y = ex yields 2 2 + 4 = ( 2)2 = 0. Because the polynomial has a double root, the solutions are e2x and x e2x .

Result 17.1.1 Consider the second order constant coecient dierential equation: y + 2ay + by = 0. We can factor the dierential equation into the form: d dx which has the solution: y= c1 ex +c2 ex , = , c1 ex +c2 x ex , = . d y = 0, dx

We can also determine and by substituting y = ex into the dierential equation and factoring the polynomial in .
Shift Invariance. Note that if u(x) is a solution of a constant coecient equation, then u(x + c) is also a solution. This is useful in applying initial or boundary conditions. Example 17.1.5 Consider the problem y 3y + 2y = 0, We know that the general solution is y = c1 ex +c2 e2x . 935 y(0) = a, y (0) = b.

Applying the initial conditions, we obtain the equations, c1 + c2 = a, The solution is y = (2a b) ex +(b a) e2x . Now suppose we wish to solve the same dierential equation with the boundary conditions y(1) = a and y (1) = b. All we have to do is shift the solution to the right. y = (2a b) ex1 +(b a) e2(x1) . c1 + 2c2 = b.

17.1.2

Real-Valued Solutions

If the coecients of the dierential equation are real, then the solution can be written in terms of real-valued functions (Result 16.2.2). For a real root = of the polynomial in , the corresponding solution, y = ex , is real-valued. Now recall that the complex roots of a polynomial with real coecients occur in complex conjugate pairs. Assume that are roots of n + an1 n1 + + a1 + a0 = 0. The corresponding solutions of the dierential equation are e(+)x and e()x . Note that the linear combinations e(+)x + e()x = ex cos(x), 2 e(+)x e()x = ex sin(x), 2

are real-valued solutions of the dierential equation. We could also obtain real-valued solution by taking the real and imaginary parts of either e(+)x or e()x . e(+)x = ex cos(x), Example 17.1.6 Consider the equation y 2y + 2y = 0. 936 e(+)x = ex sin(x)

The substitution y = ex yields 2 2 + 2 = ( 1 )( 1 + ) = 0. The linearly independent solutions are e(1+)x , and e(1)x .

We can write the general solution in terms of real functions. y = c1 ex cos x + c2 ex sin x

Exercise 17.1 Find the general solution of y + 2ay + by = 0 for a, b R. There are three distinct forms of the solution depending on the sign of a2 b. Hint, Solution Exercise 17.2 Find the fundamental set of solutions of y + 2ay + by = 0 at the point x = 0, for a, b R. Use the general solutions obtained in Exercise 17.1. Hint, Solution 937

Result 17.1.2 . Consider the second order constant coecient equation y + 2ay + by = 0. The general solution of this dierential equation is eax c1 e a2 b x +c2 e a2 b x y = eax c1 cos( b a2 x) + c2 sin( b a2 x) ax e (c + c x) 1 2 The fundamental set of solutions at x = 0 is
eax cosh( a2 b x) + a sinh( a2 b x) , eax 1 sinh( a2 b x) 2 b 2 b a a eax cos( b a2 x) + a 2 sin( b a2 x) , eax 1 2 sin( b a2 x) ba ba {(1 + ax) eax , x eax } if a2 > b, if a2 < b, if a2 = b.

if a2 > b, if a2 < b, if a2 = b.

To obtain the fundamental set of solutions at the point x = , substitute (x ) for x in the above solutions.

17.1.3

Higher Order Equations

The constant coecient equation of order n has the form L[y] = y (n) + an1 y (n1) + + a1 y + a0 y = 0. 938

(17.3)

The substitution y = ex will transform this dierential equation into an algebraic equation. L[ex ] = n ex +an1 n1 ex + + a1 ex +a0 ex = 0 n + an1 n1 + + a1 + a0 ex = 0 n + an1 n1 + + a1 + a0 = 0 Assume that the roots of this equation, 1 , . . . , n , are distinct. Then the n linearly independent solutions of Equation 17.3 are e1 x , . . . , en x . If the roots of the algebraic equation are not distinct then we will not obtain all the solutions of the dierential equation. Suppose that 1 = is a double root. We substitute y = ex into the dierential equation. L[ex ] = [( )2 ( 3 ) ( n )] ex = 0 Setting = will make the left side of the equation zero. Thus y = ex is a solution. Now we dierentiate both sides of the equation with respect to and interchange the order of dierentiation. d x d e L[ex ] = L = L x ex d d Let p() = ( 3 ) ( n ). We calculate L x ex by applying L and then dierentiating with respect to . L x ex = d L[ex ] d d = [( )2 ( 3 ) ( n )] ex d d = [( )2 p()] ex d = 2( )p() + ( )2 p () + ( )2 p()x ex = ( ) [2p() + ( )p () + ( )p()x] ex 939

Since setting = will make this expression zero, L[x ex ] = 0, x ex is a solution of Equation 17.3. You can verify that ex and x ex are linearly independent. Now we have generated all of the solutions for the dierential equation. If = is a root of multiplicity m then by repeatedly dierentiating with respect to you can show that the corresponding solutions are ex , x ex , x2 ex , . . . , xm1 ex . Example 17.1.7 Consider the equation y 3y + 2y = 0. The substitution y = ex yields 3 3 + 2 = ( 1)2 ( + 2) = 0. Thus the general solution is y = c1 ex +c2 x ex +c3 e2x .

Result 17.1.3 Consider the nth order constant coecient equation dn y dn1 y dy + an1 n1 + + a1 + a0 y = 0. dxn dx dx Let the factorization of the algebraic equation obtained with the substitution y = ex be ( 1 )m1 ( 2 )m2 ( p )mp = 0. A set of linearly independent solutions is given by {e1 x , x e1 x , . . . , xm1 1 e1 x , . . . , ep x , x ep x , . . . , xmp 1 ep x }. If the coecients of the dierential equation are real, then we can nd a real-valued set of solutions.
940

Example 17.1.8 Consider the equation d2 y d4 y + 2 2 + y = 0. dx4 dx The substitution y = ex yields 4 + 22 + 1 = ( i)2 ( + i)2 = 0. Thus the linearly independent solutions are ex , x ex , ex and x ex . Noting that ex = cos(x) + sin(x), we can write the general solution in terms of sines and cosines. y = c1 cos x + c2 sin x + c3 x cos x + c4 x sin x

17.2

Euler Equations
L[y] = x2

Consider the equation d2 y dy + ax + by = 0, x > 0. 2 dx dx Lets say, for example, that y has units of distance and x has units of time. Note that each term in the dierential equation has the same dimension. (time)2 (distance) (distance) = (time) = (distance) 2 (time) (time)

Thus this is a second order Euler, or equidimensional equation. We know that the rst order Euler equation, xy +ay = 0, has the solution y = cxa . Thus for the second order equation we will try a solution of the form y = x . The substitution 941

y = x will transform the dierential equation into an algebraic equation. L[x ] = x2 d2 d [x ] + ax [x ] + bx = 0 2 dx dx ( 1)x + ax + bx = 0 ( 1) + a + b = 0

Factoring yields ( 1 )( 2 ) = 0. If the two roots, 1 and 2 , are distinct then the general solution is y = c1 x1 + c2 x2 . If the roots are not distinct, 1 = 2 = , then we only have the one solution, y = x . To generate the other solution we use the same approach as for the constant coecient equation. We substitute y = x into the dierential equation and dierentiate with respect to . d d L[x ] = L[ x ] d d = L[ln x x ] Note that d ln x d e x = = ln x e ln x = ln x x . d d Now we apply L and then dierentiate with respect to . d d L[x ] = ( )2 x d d = 2( )x + ( )2 ln x x 942

Equating these two results, L[ln x x ] = 2( )x + ( )2 ln x x . Setting = will make the right hand side zero. Thus y = ln x x is a solution. If you are in the mood for a little algebra you can show by repeatedly dierentiating with respect to that if = is a root of multiplicity m in an nth order Euler equation then the associated solutions are x , ln x x , (ln x)2 x , . . . , (ln x)m1 x . Example 17.2.1 Consider the Euler equation xy y + The substitution y = x yields the algebraic equation ( 1) + 1 = ( 1)2 = 0. Thus the general solution is y = c1 x + c2 x ln x. y = 0. x

17.2.1

Real-Valued Solutions

If the coecients of the Euler equation are real, then the solution can be written in terms of functions that are real-valued when x is real and positive, (Result 16.2.2). If are the roots of ( 1) + a + b = 0 then the corresponding solutions of the Euler equation are x+ We can rewrite these as x e ln x and x e ln x . 943 and x .

Note that the linear combinations x e ln x +x e ln x = x cos( ln x), 2 and x e ln x x e ln x = x sin( ln x), 2

are real-valued solutions when x is real and positive. Equivalently, we could take the real and imaginary parts of either x+ or x . x e ln x = x cos( ln x), x e ln x = x sin( ln x)

Result 17.2.1 Consider the second order Euler equation x2 y + (2a + 1)xy + by = 0. The general solution of this dierential equation is xa c1 x a2 b + c2 x a2 b y = xa c1 cos b a2 ln x + c2 sin b a2 ln x a x (c + c ln x) 1 2 The fundamental set of solutions at x = is

x a

if a2 > b, if a2 < b, if a2 = b.

y=

a2 b ln x + aa b sinh a2 b ln x , 2 a x sinh a2 b ln x if a2 > b, a2 b a a cos b a2 ln x + ba2 sin b a2 ln x , a x x sin b a2 ln if a2 < b, ba2 cosh 1 + a ln x ,
x a

ln x

if a2 = b.

944

Example 17.2.2 Consider the Euler equation x2 y 3xy + 13y = 0. The substitution y = x yields ( 1) 3 + 13 = ( 2 3)( 2 + 3) = 0. The linearly independent solutions are x2+3 , x23 . We can put this in a more understandable form. x2+3 = x2 e3 ln x = x2 cos(3 ln x) + x2 sin(3 ln x) We can write the general solution in terms of real-valued functions. y = c1 x2 cos(3 ln x) + c2 x2 sin(3 ln x)

Result 17.2.2 Consider the nth order Euler equation


n nd y x dxn

n1 y n1 d an1 x dxn1

+ + a1 x

dy + a0 y = 0. dx

Let the factorization of the algebraic equation obtained with the substitution y = x be ( 1 )m1 ( 2 )m2 ( p )mp = 0. A set of linearly independent solutions is given by {x1 , ln x x1 , . . . , (ln x)m1 1 x1 , . . . , xp , ln x xp , . . . , (ln x)mp 1 xp }. If the coecients of the dierential equation are real, then we can nd a set of solutions that are real valued when x is real and positive.
945

17.3

Exact Equations
d F (x, y, y , y , . . .) = f (x). dx

Exact equations have the form

If you can write an equation in the form of an exact equation, you can integrate to reduce the order by one, (or solve the equation for rst order). We will consider a few examples to illustrate the method. Example 17.3.1 Consider the equation y + x2 y + 2xy = 0. We can rewrite this as d y + x2 y = 0. dx Integrating yields a rst order inhomogeneous equation. y + x2 y = c1 We multiply by the integrating factor I(x) = exp( x2 dx) to make this an exact equation. d 3 3 ex /3 y = c1 ex /3 dx ex
3 /3

y = c1
3 /3

ex ex

3 /3

dx + c2
3 /3

y = c1 ex

3 /3

dx + c2 ex

946

Result 17.3.1 If you can write a dierential equation in the form d F (x, y, y , y , . . .) = f (x), dx then you can integrate to reduce the order of the equation. F (x, y, y , y , . . .) = f (x) dx + c

17.4

Equations Without Explicit Dependence on y


y + d dx xy = 0.

Example 17.4.1 Consider the equation

This is a second order equation for y, but note that it is a rst order equation for y . We can solve directly for y . 2 3/2 x y =0 3 2 y = c1 exp x3/2 3 exp

Now we just integrate to get the solution for y. y = c1 2 exp x3/2 3 dx + c2

Result 17.4.1 If an nth order equation does not explicitly depend on y then you can consider it as an equation of order n 1 for y .

947

17.5

Reduction of Order

Consider the second order linear equation L[y] y + p(x)y + q(x)y = f (x). Suppose that we know one homogeneous solution y1 . We make the substitution y = uy1 and use that L[y1 ] = 0. L[uy1 ] = 0u y1 + 2u y1 + uy1 + p(u y1 + uy1 ) + quy1 = 0 u y1 + u (2y1 + py1 ) + u(y1 + py1 + qy1 ) = 0 u y1 + u (2y1 + py1 ) = 0 Thus we have reduced the problem to a rst order equation for u . An analogous result holds for higher order equations.

Result 17.5.1 Consider the nth order linear dierential equation y (n) + pn1 (x)y (n1) + + p1 (x)y + p0 (x)y = f (x). Let y1 be a solution of the homogeneous equation. The substitution y = uy1 will transform the problem into an (n 1)th order equation for u . For the second order problem y + p(x)y + q(x)y = f (x) this reduced equation is u y1 + u (2y1 + py1 ) = f (x).
Example 17.5.1 Consider the equation y + xy y = 0. 948

By inspection we see that y1 = x is a solution. We would like to nd another linearly independent solution. The substitution y = xu yields xu + (2 + x2 )u = 0 2 +x u =0 u + x The integrating factor is I(x) = exp(2 ln x + x2 /2) = x2 exp(x2 /2). d 2 x2 ex /2 u = 0 dx 2 u = c1 x2 ex /2 u = c1 y = c1 x Thus we see that a second solution is y2 = x x2 ex
2 /2

x2 ex x2 ex

2 /2

dx + c2 dx + c2 x

2 /2

dx.

17.6

*Reduction of Order and the Adjoint Equation

Let L be the linear dierential operator dn1 y dn y + pn1 n1 + + p0 y, dxn dx where each pj is a j times continuously dierentiable complex valued function. Recall that the adjoint of L is L[y] = pn L [y] = (1)n dn dn1 (pn y) + (1)n1 n1 (pn1 y) + + p0 y. dxn dx 949

If u and v are n times continuously dierentiable, then Lagranges identity states vL[u] uL [v] = where B[u, v] =
m=1 j+k=m1 j0,k0 n

d B[u, v], dx

(1)j u(k) (pm v)(j) .

For second order equations, B[u, v] = up1 v + u p2 v u(p2 v) . (See Section 16.7.) If we can nd a solution to the homogeneous adjoint equation, L [y] = 0, then we can reduce the order of the equation L[y] = f (x). Let satisfy L [] = 0. Substituting u = y, v = into Lagranges identity yields L[y] yL [] = L[y] = The equation L[y] = f (x) is equivalent to the equation d B[y, ] = f dx B[y, ] = which is a linear equation in y of order n 1. Example 17.6.1 Consider the equation L[y] = y x2 y 2xy = 0. 950 (x)f (x) dx, d B[y, ] dx

d B[y, ]. dx

Method 1. Note that this is an exact equation. d (y x2 y) = 0 dx y x2 y = c1 d 3 3 ex /3 y = c1 ex /3 dx y = c 1 ex Method 2. The adjoint equation is L [y] = y + x2 y = 0. By inspection we see that = (constant) is a solution of the adjoint equation. To simplify the algebra we will choose = 1. Thus the equation L[y] = 0 is equivalent to B[y, 1] = c1 d d y(x2 ) + [y](1) y [1] = c1 dx dx y x2 y = c1 . By using the adjoint equation to reduce the order we obtain the same solution as with Method 1.
3 /3

ex

3 /3

dx + c2 ex

3 /3

951

17.7

Additional Exercises

Constant Coecient Equations


Exercise 17.3 (mathematica/ode/techniques linear/constant.nb) Find the solution of each one of the following initial value problems. Sketch the graph of the solution and describe its behavior as t increases. 1. 6y 5y + y = 0, y(0) = 4, y (0) = 0 2. y 2y + 5y = 0, y(/2) = 0, y (/2) = 2 3. y + 4y + 4y = 0, y(1) = 2, y (1) = 1 Hint, Solution Exercise 17.4 (mathematica/ode/techniques linear/constant.nb) Substitute y = ex to nd two linearly independent solutions to y 4y + 13y = 0. that are real-valued when x is real-valued. Hint, Solution Exercise 17.5 (mathematica/ode/techniques linear/constant.nb) Find the general solution to y y + y y = 0. Write the solution in terms of functions that are real-valued when x is real-valued. Hint, Solution Exercise 17.6 Substitute y = ex to nd the fundamental set of solutions at x = 0 for each of the equations: 1. y + y = 0, 952

2. y y = 0, 3. y = 0. What are the fundamental set of solutions at x = 1 for each of these equations. Hint, Solution Exercise 17.7 Consider a ball of mass m hanging by an ideal spring of spring constant k. The ball is suspended in a uid which damps the motion. This resistance has a coecient of friction, . Find the dierential equation for the displacement of the mass from its equilibrium position by balancing forces. Denote this displacement by y(t). If the damping force is weak, the mass will have a decaying, oscillatory motion. If the damping force is strong, the mass will not oscillate. The displacement will decay to zero. The value of the damping which separates these two behaviors is called critical damping. Find the solution which satises the initial conditions y(0) = 0, y (0) = 1. Use the solutions obtained in Exercise 17.2 or refer to Result 17.1.2. Consider the case m = k = 1. Find the coecient of friction for which the displacement of the mass decays most rapidly. Plot the displacement for strong, weak and critical damping. Hint, Solution Exercise 17.8 Show that y = c cos(x ) is the general solution of y + y = 0 where c and are constants of integration. (It is not sucient to show that y = c cos(x ) satises the dierential equation. y = 0 satises the dierential equation, but is is certainly not the general solution.) Find constants c and such that y = sin(x). Is y = c cosh(x ) the general solution of y y = 0? Are there constants c and such that y = sinh(x)? Hint, Solution Exercise 17.9 (mathematica/ode/techniques linear/constant.nb) Let y(t) be the solution of the initial-value problem y + 5y + 6y = 0; y(0) = 1, y (0) = V.

For what values of V does y(t) remain nonnegative for all t > 0? 953

Hint, Solution Exercise 17.10 (mathematica/ode/techniques linear/constant.nb) Find two linearly independent solutions of y + sign(x)y = 0, < x < .

where sign(x) = 1 according as x is positive or negative. (The solution should be continuous and have a continuous rst derivative.) Hint, Solution

Euler Equations
Exercise 17.11 Find the general solution of x2 y + xy + y = 0, Hint, Solution Exercise 17.12 Substitute y = x to nd the general solution of x2 y 2xy + 2y = 0. Hint, Solution Exercise 17.13 (mathematica/ode/techniques linear/constant.nb) Substitute y = x to nd the general solution of 1 xy + y + y = 0. x Write the solution in terms of functions that are real-valued when x is real-valued and positive. Hint, Solution 954 x > 0.

Exercise 17.14 Find the general solution of x2 y + (2a + 1)xy + by = 0. Hint, Solution Exercise 17.15 Show that y1 = eax , are linearly indepedent solutions of y a2 y = 0 for all values of a. It is common to abuse notation and write the second solution as eax eax y2 = a where the limit is taken if a = 0. Likewise show that y1 = xa , are linearly indepedent solutions of x2 y + xy a2 y = 0 for all values of a. Hint, Solution Exercise 17.16 (mathematica/ode/techniques linear/constant.nb) Find two linearly independent solutions (i.e., the general solution) of (a) x2 y 2xy + 2y = 0, Hint, Solution 955 (b) x2 y 2y = 0, (c) x2 y xy + y = 0. y2 = xa xa a y2 = lim ex ex a

Exact Equations
Exercise 17.17 Solve the dierential equation y + y sin x + y cos x = 0. Hint, Solution

Equations Without Explicit Dependence on y Reduction of Order


Exercise 17.18 Consider (1 x2 )y 2xy + 2y = 0, Verify that y = x is a solution. Find the general solution. Hint, Solution Exercise 17.19 Consider the dierential equation y x+1 1 y + y = 0. x x 1 < x < 1.

1 Since the coecients sum to zero, (1 x+1 + x = 0), y = ex is a solution. Find another linearly independent solution. x Hint, Solution

Exercise 17.20 One solution of (1 2x)y + 4xy 4y = 0 is y = x. Find the general solution. Hint, Solution

956

Exercise 17.21 Find the general solution of (x 1)y xy + y = 0, given that one solution is y = ex . (you may assume x > 1) Hint, Solution

*Reduction of Order and the Adjoint Equation

957

17.8

Hints

Hint 17.1 Substitute y = ex into the dierential equation. Hint 17.2 The fundamental set of solutions is a linear combination of the homogeneous solutions.

Constant Coecient Equations


Hint 17.3

Hint 17.4

Hint 17.5 It is a constant coecient equation. Hint 17.6 Use the fact that if u(x) is a solution of a constant coecient equation, then u(x + c) is also a solution. Hint 17.7 The force on the mass due to the spring is ky(t). The frictional force is y (t). Note that the initial conditions describe the second fundamental solution at t = 0. Note that for large t, t et is much small than et if < . (Prove this.) Hint 17.8 By denition, the general solution of a second order dierential equation is a two parameter family of functions that satises the dierential equation. The trigonometric identities in Appendix Q may be useful.

958

Hint 17.9

Hint 17.10

Euler Equations
Hint 17.11

Hint 17.12

Hint 17.13

Hint 17.14 Substitute y = x into the dierential equation. Consider the three cases: a2 > b, a2 < b and a2 = b. Hint 17.15

Hint 17.16

Exact Equations
Hint 17.17 It is an exact equation.

Equations Without Explicit Dependence on y


959

Reduction of Order
Hint 17.18 Hint 17.19 Use reduction of order to nd the other solution. Hint 17.20 Use reduction of order to nd the other solution. Hint 17.21

*Reduction of Order and the Adjoint Equation

960

17.9

Solutions

Solution 17.1 We substitute y = ex into the dierential equation. y + 2ay + by = 0 2 + 2a + b = 0 = a a2 b If a2 > b then the two roots are distinct and real. The general solution is y = c1 e(a+
a2 b)x

+c2 e(a

a2 b)x

If a2 < b then the two roots are distinct and complex-valued. We can write them as = a b a2 . The general solution is y = c1 e(a+
ba2 )x

+c2 e(a

ba2 )x

By taking the sum and dierence of the two linearly independent solutions above, we can write the general solution as y = c1 eax cos b a2 x + c2 eax sin b a2 x . If a2 = b then the only root is = a. The general solution in this case is then y = c1 eax +c2 x eax . In summary, the general solution is eax c1 e a2 b x +c2 e a2 b x y = eax c1 cos b a2 x + c2 sin b a2 x ax e (c + c x) 1 2

if a2 > b, if a2 < b, if a2 = b.

961

Solution 17.2 First we note that the general solution can be written, eax c1 cosh a2 b x + c2 sinh a2 b x y = eax c1 cos b a2 x + c2 sin b a2 x ax e (c1 + c2 x) if a2 > b, if a2 < b, if a2 = b.

We rst consider the case a2 > b. The derivative is y = eax ac1 + a2 b c2 cosh a2 b x + ac2 + a2 b c1 sinh a2 b x The conditions, y1 (0) = 1 and y1 (0) = 0, for the rst solution become, c1 = 1, ac1 + a2 b c2 = 0, a c1 = 1, c2 = . a2 b The conditions, y2 (0) = 0 and y2 (0) = 1, for the second solution become, c1 = 0, ac1 + a2 b c2 = 1, 1 c1 = 0, c2 = . a2 b The fundamental set of solutions is eax cosh a2 b x + a a2 b sinh a2 b x , eax 1 a2 b sinh a2 b x .

Now consider the case a2 < b. The derivative is y = eax ac1 + b a2 c2 cos b a2 x + ac2 b a2 c1 sin b a2 x 962

Clearly, the fundamental set of solutions is eax cos b a2 x + a sin b a2 x b a2 , eax 1 sin b a2 x b a2 .

Finally we consider the case a2 = b. The derivative is y = eax (ac1 + c2 + ac2 x). The conditions, y1 (0) = 1 and y1 (0) = 0, for the rst solution become, c1 = 1, ac1 + c2 = 0, c1 = 1, c2 = a. The conditions, y2 (0) = 0 and y2 (0) = 1, for the second solution become, c1 = 0, ac1 + c2 = 1, c1 = 0, c2 = 1. The fundamental set of solutions is (1 + ax) eax , x eax . In summary, the fundamental set of solutions at x = 0 is eax cosh a2 b x + a sinh a2 b x , eax 1 sinh a2 b x 2 b 2 b a a ax ax 1 2x + a 2x e cos b a ba ,e b a2 x 2 sin 2 sin ba ba {(1 + ax) eax , x eax } if a2 > b, if a2 < b, if a2 = b.

Constant Coecient Equations


963

Solution 17.3 1. We consider the problem 6y 5y + y = 0, y(0) = 4, y (0) = 0. We make the substitution y = ex in the dierential equation. 62 5 + 1 = 0 (2 1)(3 1) = 0 1 1 = , 3 2 The general solution of the dierential equation is y = c1 et/3 +c2 et/2 . We apply the initial conditions to determine the constants. c1 c2 c1 + c2 = 4, + =0 3 2 c1 = 12, c2 = 8 The solution subject to the initial conditions is y = 12 et/3 8 et/2 . The solution is plotted in Figure 17.1. The solution tends to as t . 2. We consider the problem y 2y + 5y = 0, y(/2) = 0, y (/2) = 2. We make the substitution y = ex in the dierential equation. 2 2 + 5 = 0 =1 15 = {1 + 2, 1 2} 964

-5 -10 -15 -20 -25 -30

Figure 17.1: The solution of 6y 5y + y = 0, y(0) = 4, y (0) = 0. The general solution of the dierential equation is y = c1 et cos(2t) + c2 et sin(2t). We apply the initial conditions to determine the constants. y(/2) = 0 y (/2) = 2 c1 e/2 = 0
/2

c1 = 0 c2 = e/2

2c2 e

=2

The solution subject to the initial conditions is y = et/2 sin(2t). The solution is plotted in Figure 17.2. The solution oscillates with an amplitude that tends to as t . 3. We consider the problem y + 4y + 4y = 0, y(1) = 2, 965 y (1) = 1.

50 40 30 20 10 3 -10 4 5 6

Figure 17.2: The solution of y 2y + 5y = 0, y(/2) = 0, y (/2) = 2. We make the substitution y = ex in the dierential equation. 2 + 4 + 4 = 0 ( + 2)2 = 0 = 2 The general solution of the dierential equation is y = c1 e2t +c2 t e2t . We apply the initial conditions to determine the constants. c1 e2 c2 e2 = 2, 2c1 e2 +3c2 e2 = 1 c1 = 7 e2 , c2 = 5 e2 The solution subject to the initial conditions is y = (7 + 5t) e2(t+1) 966

2 1.5 1 0.5 -1 1 2 3 4 5

Figure 17.3: The solution of y + 4y + 4y = 0, y(1) = 2, y (1) = 1. The solution is plotted in Figure 17.3. The solution vanishes as t . lim (7 + 5t) e2(t+1) = lim 7 + 5t 5 = lim =0 2(t+1) 2(t+1) t e t 2 e

Solution 17.4 y 4y + 13y = 0. With the substitution y = ex we obtain 2 ex 4 ex +13 ex = 0 2 4 + 13 = 0 = 2 3i. Thus two linearly independent solutions are e(2+3i)x , and 967 e(23i)x .

Noting that e(2+3i)x = e2x [cos(3x) + sin(3x)] e(23i)x = e2x [cos(3x) sin(3x)], we can write the two linearly independent solutions y1 = e2x cos(3x), Solution 17.5 We note that y y +y y =0 is a constant coecient equation. The substitution, y = ex , yields 3 2 + 1 = 0 ( 1)( i)( + i) = 0. The corresponding solutions are ex , ex , and ex . We can write the general solution as y = c1 ex +c2 cos x + c3 sin x. Solution 17.6 We start with the equation y + y = 0. We substitute y = ex into the dierential equation to obtain 2 + 1 = 0, A linearly independent set of solutions is {ex , ex }. The fundamental set of solutions has the form y1 = c1 ex +c2 ex , y2 = c3 ex +c4 ex . 968 = i. y2 = e2x sin(3x).

By applying the constraints y1 (0) = 1, y1 (0) = 0, y2 (0) = 0, y2 (0) = 1, we obtain ex + ex = cos x, 2 ex + ex y2 = = sin x. 2 y1 = Now consider the equation y y = 0. By substituting y = ex we nd that a set of solutions is {ex , ex }. By taking linear combinations of these we see that another set of solutions is {cosh x, sinh x}. Note that this is the fundamental set of solutions. Next consider y = 0. We can nd the solutions by substituting y = ex or by integrating the equation twice. The fundamental set of solutions as x = 0 is {1, x}. Note that if u(x) is a solution of a constant coecient dierential equation, then u(x + c) is also a solution. Also note that if u(x) satises y(0) = a, y (0) = b, then u(x x0 ) satises y(x0 ) = a, y (x0 ) = b. Thus the fundamental sets of solutions at x = 1 are 1. {cos(x 1), sin(x 1)}, 2. {cosh(x 1), sinh(x 1)}, 3. {1, x 1}. 969

Solution 17.7 Let y(t) denote the displacement of the mass from equilibrium. The forces on the mass are ky(t) due to the spring and y (t) due to friction. We equate the external forces to my (t) to nd the dierential equation of the motion. my = ky y y + k y + y=0 m m

The solution which satises the initial conditions y(0) = 0, y (0) = 1 is et/(2m) 2m sinh 2 4km y(t) = et/(2m) 2m sin 2 t/(2m) 4km t e 2 4km t/(2m) 4km 2 t/(2m) if 2 > km, if 2 < km, if 2 = km.

We respectively call these cases: strongly damped, weakly damped and critically damped. In the case that m = k = 1 the solution is et/2 2 sinh 2 4 t/2 if > 2, 2 4 y(t) = et/2 2 sin 4 2 t/2 if < 2, 42 t t e if = 2. Note that when t is large, t et is much smaller than et/2 for < 2. To prove this we examine the ratio of these functions as t . t et t lim = lim (1/2)t t et/2 t e 1 = lim t (1 /2) e(1)t =0 970

0.5 0.4 0.3 0.2 0.1 2 -0.1 4 6

Strong Weak Critical

10

Figure 17.4: Strongly, weakly and critically damped solutions. Using this result, we see that the critically damped solution decays faster than the weakly damped solution. We can write the strongly damped solution as et/2 For large t, the dominant factor is e 2 2 4 e
2 4 t/2

2 4 t/2

2 4 t/2

. Note that for > 2, ( + 2)( 2) > 2.

2 4 = Therefore we have the bounds 2 <

2 4 < 0.

This shows that the critically damped solution decays faster than the strongly damped solution. = 2 gives the fastest decaying solution. Figure 17.4 shows the solution for = 4, = 1 and = 2.

971

Solution 17.8 Clearly y = c cos(x ) satises the dierential equation y + y = 0. Since it is a two-parameter family of functions, it must be the general solution. Using a trigonometric identity we can rewrite the solution as y = c cos cos x + c sin sin x. Setting this equal to sin x gives us the two equations c cos = 0, c sin = 1, which has the solutions c = 1, = (2n + 1/2), and c = 1, = (2n 1/2), for n Z. Clearly y = c cosh(x ) satises the dierential equation y y = 0. Since it is a two-parameter family of functions, it must be the general solution. Using a trigonometric identity we can rewrite the solution as y = c cosh cosh x + c sinh sinh x. Setting this equal to sinh x gives us the two equations c cosh = 0, c sinh = 1, which has the solutions c = i, = (2n + 1/2), and c = i, = (2n 1/2), for n Z. Solution 17.9 We substitute y = et into the dierential equation. 2 et +5 et +6 et = 0 2 + 5 + 6 = 0 ( + 2)( + 3) = 0 972

The general solution of the dierential equation is y = c1 e2t +c2 e3t . The initial conditions give us the constraints: c1 + c2 = 1, 2c1 3c2 = V. The solution subject to the initial conditions is y = (3 + V ) e2t (2 + V ) e3t . This solution will be non-negative for t > 0 if V 3. Solution 17.10 For negative x, the dierential equation is y y = 0. We substitute y = ex into the dierential equation to nd the solutions. 2 1 = 0 = 1 y = ex , ex We can take linear combinations to write the solutions in terms of the hyperbolic sine and cosine. y = {cosh(x), sinh(x)} For positive x, the dierential equation is y + y = 0. 973

We substitute y = ex into the dierential equation to nd the solutions. 2 + 1 = 0 = y = ex , ex We can take linear combinations to write the solutions in terms of the sine and cosine. y = {cos(x), sin(x)} We will nd the fundamental set of solutions at x = 0. That is, we will nd a set of solutions, {y1 , y2 } that satisfy the conditions: y1 (0) = 1 y1 (0) = 0 y2 (0) = 0 y2 (0) = 1

Clearly, these solutions are y1 = cosh(x) x < 0 cos(x) x0 y2 = sinh(x) x < 0 sin(x) x0

Euler Equations
Solution 17.11 We consider an Euler equation, x2 y + xy + y = 0, x > 0. We make the change of independent variable = ln x, u() = y(x) to obtain u + u = 0. 974

We make the substitution u() = e . 2 + 1 = 0 = i A set of linearly independent solutions for u() is {e , e }. e + e e e and sin = , 2 2 another linearly independent set of solutions is {cos , sin }. cos = The general solution for y(x) is y(x) = c1 cos(ln x) + c2 sin(ln x). Solution 17.12 Consider the dierential equation x2 y 2xy + 2y = 0. With the substitution y = x this equation becomes ( 1) 2 + 2 = 0 2 3 + 2 = 0 = 1, 2. The general solution is then y = c1 x + c2 x2 . Since

975

Solution 17.13 We note that 1 xy + y + y = 0 x is an Euler equation. The substitution y = x yields 3 32 + 2 + 2 + = 0 3 22 + 2 = 0. The three roots of this algebraic equation are = 0, = 1 + i, =1

The corresponding solutions to the dierential equation are y = x0 y=1 We can write the general solution as y = c1 + c2 x cos(ln x) + c3 sin(ln x). Solution 17.14 We substitute y = x into the dierential equation. x2 y + (2a + 1)xy + by = 0 ( 1) + (2a + 1) + b = 0 2 + 2a + b = 0 = a a2 b 976 y = x1+ y = x e ln x y = x1 y = x e ln x .

For a2 > b then the general solution is y = c1 xa+ For a2 < b, then the general solution is y = c1 xa+
ba2 a2 b

+ c2 xa

a2 b

+ c2 xa

ba2

By taking the sum and dierence of these solutions, we can write the general solution as y = c1 xa cos b a2 ln x + c2 xa sin b a2 ln x . For a2 = b, the quadratic in lambda has a double root at = a. The general solution of the dierential equation is y = c1 xa + c2 xa ln x. In summary, the general solution is: xa c1 x a2 b + c2 x a2 b y = xa c1 cos b a2 ln x + c2 sin b a2 ln x a x (c + c ln x)
1 2

if a2 > b, if a2 < b, if a2 = b.

Solution 17.15 For a = 0, two linearly independent solutions of y a2 y = 0 are y1 = eax , For a = 0, we have y1 = e0x = 1, y2 = x e0x = x. 977 y2 = eax .

In this case the solution are dened by y1 = [eax ]a=0 , By the denition of dierentiation, f (0) is f (0) = lim Thus the second solution in the case a = 0 is eax eax y2 = lim a0 a ex ex . a Clearly y1 is a solution for all a. For a = 0, y2 is a linear combination of eax and eax and is thus a solution. Since the coecient of eax in this linear combination is non-zero, it is linearly independent to y1 . For a = 0, y2 is one half the derivative of eax evaluated at a = 0. Thus it is a solution. For a = 0, two linearly independent solutions of y1 = eax , y2 = lim x2 y + xy a2 y = 0 are y1 = xa , For a = 0, we have y1 = [xa ]a=0 = 1, Consider the solutions y1 = xa , y2 = d a x da = ln x.
a=0

y2 =

d ax e da

.
a=0

f (a) f (a) . a0 2a

Consider the solutions

y2 = xa .

xa xa a Clearly y1 is a solution for all a. For a = 0, y2 is a linear combination of xa and xa and is thus a solution. For a = 0, y2 is one half the derivative of xa evaluated at a = 0. Thus it is a solution. y2 = 978

Solution 17.16 1. x2 y 2xy + 2y = 0 We substitute y = x into the dierential equation. ( 1) 2 + 2 = 0 2 3 + 2 = 0 ( 1)( 2) = 0 y = c1 x + c2 x2 2. x2 y 2y = 0 We substitute y = x into the dierential equation. ( 1) 2 = 0 2 2 = 0 ( + 1)( 2) = 0 c1 y= + c2 x2 x 3. x2 y xy + y = 0 We substitute y = x into the dierential equation. ( 1) + 1 = 0 2 2 + 1 = 0 ( 1)2 = 0 979

Since there is a double root, the solution is: y = c1 x + c2 x ln x.

Exact Equations
Solution 17.17 We note that y + y sin x + y cos x = 0 is an exact equation. d [y + y sin x] = 0 dx y + y sin x = c1 d y e cos x = c1 e cos x dx y = c1 ecos x e cos x dx + c2 ecos x

Equations Without Explicit Dependence on y Reduction of Order


Solution 17.18 (1 x2 )y 2xy + 2y = 0, 1 < x < 1

We substitute y = x into the dierential equation to check that it is a solution. (1 x2 )(0) 2x(1) + 2x = 0 980

We look for a second solution of the form y = xu. We substitute this into the dierential equation and use the fact that x is a solution. (1 x2 )(xu + 2u ) 2x(xu + u) + 2xu = 0 (1 x2 )(xu + 2u ) 2x(xu ) = 0 (1 x2 )xu + (2 4x2 )u = 0 u 2 4x2 = u x(x2 1) u 2 1 1 = + u x 1x 1+x ln(u ) = 2 ln(x) ln(1 x) ln(1 + x) + const c ln(u ) = ln 2 (1 x)(1 + x) x c u = 2 x (1 x)(1 + x) 1 1 1 u =c + + 2 x 2(1 x) 2(1 + x) 1 1 1 u = c ln(1 x) + ln(1 + x) + const x 2 2 1 1 1+x u = c + ln + const x 2 1x A second linearly independent solution is y = 1 + x ln 2 1+x 1x .

981

Solution 17.19 We are given that y = ex is a solution of y 1 x+1 y + y = 0. x x

To nd another linearly independent solution, we will use reduction of order. Substituting y = u ex y = (u + u) ex y = (u + 2u + u) ex into the dierential equation yields u + 2u + u 1 x+1 (u + u) + u = 0. x x x1 u + u =0 x d 1 u exp 1 dx =0 dx x u exln x = c1 u = c1 x ex u = c1 x ex dx + c2

u = c1 (x ex + ex ) + c2 y = c1 (x + 1) + c2 ex Thus a second linearly independent solution is y = x + 1.

982

Solution 17.20 We are given that y = x is a solution of (1 2x)y + 4xy 4y = 0. To nd another linearly independent solution, we will use reduction of order. Substituting y = xu y = xu + u y = xu + 2u into the dierential equation yields (1 2x)(xu + 2u ) + 4x(xu + u) 4xu = 0, (1 2x)xu + (4x2 4x + 2)u = 0, u 4x2 4x + 2 = , u x(2x 1) u 2 2 =2 + , u x 2x 1 ln(u ) = 2x 2 ln x + ln(2x 1) + const, 2 1 u = c1 2 e2x , x x 1 u = c1 e2x +c2 , x y = c1 e2x +c2 x. Solution 17.21 One solution of (x 1)y xy + y = 0, 983

is y1 = ex . We nd a second solution with reduction of order. We make the substitution y2 = u ex in the dierential equation. We determine u up to an additive constant. (x 1)(u + 2u + u) ex x(u + u) ex +u ex = 0 (x 1)u + (x 2)u = 0 u x2 1 = = 1 + u x1 x1 ln |u | = x + ln |x 1| + c u = c(x 1) ex u = cx ex The second solution of the dierential equation is y2 = x.

*Reduction of Order and the Adjoint Equation

984

Chapter 18 Techniques for Nonlinear Dierential Equations


In mathematics you dont understand things. You just get used to them. - Johann von Neumann

18.1

Bernoulli Equations

Sometimes it is possible to solve a nonlinear equation by making a change of the dependent variable that converts it into a linear equation. One of the most important such equations is the Bernoulli equation dy + p(t)y = q(t)y , dt = 1.

The change of dependent variable u = y 1 will yield a rst order linear equation for u which when solved will give us an implicit solution for y. (See Exercise ??.) 985

Result 18.1.1 The Bernoulli equation y + p(t)y = q(t)y , = 1 can be transformed to the rst order linear equation du + (1 )p(t)u = (1 )q(t) dt with the change of variables u = y 1 .
Example 18.1.1 Consider the Bernoulli equation 2 y + y2. x

y =

First we divide by y 2 .

y 2 y =

2 1 y +1 x

We make the change of variable u = y 1 .

u =

2 u+1 x

2 u + u = 1 x 986

The integrating factor is I(x) = exp(

2 x

dx) = x2 . d 2 (x u) = x2 dx 1 x2 u = x3 + c 3 1 c u= x+ 2 3 x 1 1 c y = x+ 2 3 x

Thus the solution for y is y= 3x2 . c x2

18.2

Riccati Equations
L[y] = d2 d + p(x) + q(x) y = y + p(x)y + q(x)y = f (x). 2 dx dx d + a(x) dx d + b(x) , dx

Factoring Second Order Operators. Consider the second order linear equation

If we were able to factor the linear operator L into the form L= (18.1)

then we would be able to solve the dierential equation. Factoring reduces the problem to a system of rst order equations. We start with the factored equation d + a(x) dx d + b(x) y = f (x). dx 987

We set u =

d dx

+ b(x) y and solve the problem d + a(x) u = f (x). dx

Then to obtain the solution we solve d + b(x) y = u. dx Example 18.2.1 Consider the equation y + x 1 x y + 1 1 y = 0. x2

Lets say by some insight or just random luck we are able to see that this equation can be factored into d +x dx We rst solve the equation d + x u = 0. dx u + xu = 0 d 2 ex /2 u = 0 dx 2 u = c1 ex /2 988 d 1 y = 0. dx x

Then we solve for y with the equation 1 d 2 y = u = c1 ex /2 . dx x 1 2 y y = c1 ex /2 x d 2 1 x y = c1 x1 ex /2 dx y = c1 x x1 ex


2 /2

dx + c2 x

If we were able to solve for a and b in Equation 18.1 in terms of p and q then we would be able to solve any second order dierential equation. Equating the two operators, d2 d d d +p +q = +a +b dx2 dx dx dx d2 d = 2 + (a + b) + (b + ab). dx dx Thus we have the two equations a + b = p, Eliminating a, b + (p b)b = q b = b2 pb + q Now we have a nonlinear equation for b that is no easier to solve than the original second order linear equation. Riccati Equations. Equations of the form y = a(x)y 2 + b(x)y + c(x) 989 and b + ab = q.

are called Riccati equations. From the above derivation we see that for every second order dierential equation there is a corresponding Riccati equation. Now we will show that the converse is true. We make the substitution u u (u )2 a u y= , y = + + 2 , au au au2 au in the Riccati equation. y = ay 2 + by + c u u (u )2 a u (u )2 + + 2 =a 2 2 b +c au au2 au au au u au u + 2 +b c=0 au a u au a u + b u + acu = 0 a Now we have a second order linear equation for u.
u Result 18.2.1 The substitution y = au transforms the Riccati equation

y = a(x)y 2 + b(x)y + c(x) into the second order linear equation u


Example 18.2.2 Consider the Riccati equation 1 1 y = y2 + y + 2 . x x 990

a + b u + acu = 0. a

With the substitution y = u we obtain u 1 1 u u + 2 u = 0. x x This is an Euler equation. The substitution u = x yields ( 1) + 1 = ( 1)2 = 0. Thus the general solution for u is u = c1 x + c2 x log x. Since y = u , u y= c1 + c2 (1 + log x) c1 x + c2 x log x 1 + c(1 + log x) x + cx log x

y=

18.3

Exchanging the Dependent and Independent Variables

Some dierential equations can be put in a more elementary form by exchanging the dependent and independent variables. If the new equation can be solved, you will have an implicit solution for the initial equation. We will consider a few examples to illustrate the method. Example 18.3.1 Consider the equation y = y3 1 . xy 2

991

Instead of considering y to be a function of x, consider x to be a function of y. That is, x = x(y), x = dy 1 = 3 dx y xy 2 dx = y 3 xy 2 dy x + y2x = y3 Now we have a rst order equation for x. d 3 3 ey /3 x = y 3 ey /3 dy x = ey Example 18.3.2 Consider the equation
3 /3

dx . dy

y 3 ey

3 /3

dy + c ey

3 /3

y . + 2x Interchanging the dependent and independent variables yields y = y2 1 y = 2 x y + 2x x x =y+2 y x x 2 =y y d 2 (y x) = y 1 dy y 2 x = log y + c x = y 2 log y + cy 2

992

Result 18.3.1 Some dierential equations can be put in a simpler form by exchanging the dependent and independent variables. Thus a dierential equation for y(x) can be written as an equation for x(y). Solving the equation for x(y) will give an implicit solution for y(x).

18.4

Autonomous Equations

Autonomous equations have no explicit dependence on x. The following are examples. y + 3y 2y = 0 y = y + (y )2 y +y y =0 The change of variables u(y) = y reduces an nth order autonomous equation in y to a non-autonomous equation of order n 1 in u(y). Writing the derivatives of y in terms of u, y = u(y) d u(y) y = dx dy d u(y) = dx dy =yu =uu y = (u u + (u )2 )u. Thus we see that the equation for u(y) will have an order of one less than the original equation.

Result 18.4.1 Consider an autonomous dierential equation for y(x), (autonomous equations have no explicit dependence on x.) The change of variables u(y) = y reduces an nth order autonomous equation in y to a non-autonomous equation of order n 1 in u(y).
993

Example 18.4.1 Consider the equation y = y + (y )2 . With the substitution u(y) = y , the equation becomes u u = y + u2 u = u + yu1 . We recognize this as a Bernoulli equation. The substitution v = u2 yields 1 v =v+y 2 v 2v = 2y d 2y e v = 2y e2y dy v(y) = c1 e2y + e2y 2y e2y dy e2y dy

v(y) = c1 e2y + e2y y e2y +

1 v(y) = c1 e2y + e2y y e2y e2y 2 1 v(y) = c1 e2y y . 2 Now we solve for u. u(y) = dy = dx c1 e2y y c1 e2y y 1 2 1 2
1/2

.
1/2

994

This equation is separable. dx = x + c2 = dy c1 e2y y 1 c1 e2y y


1 1/2 2 1 1/2 2

dy

Thus we nally have arrived at an implicit solution for y(x). Example 18.4.2 Consider the equation y + y 3 = 0. With the change of variables, u(y) = y , the equation becomes u u + y 3 = 0. This equation is separable. u du = y 3 dy 1 2 1 u = y 4 + c1 2 4 1/2 1 4 u = 2c1 y 2 1 y = 2c1 y 4 2 dy = dx (2c1 1 y 4 )1/2 2 995
1/2

Integrating gives us the implicit solution 1 dy = x + c2 . (2c1 1 y 4 )1/2 2

18.5

*Equidimensional-in-x Equations

Dierential equations that are invariant under the change of variables x = c are said to be equidimensional-in-x. For a familiar example from linear equations, we note that the Euler equation is equidimensional-in-x. Writing the new derivatives under the change of variables, x = c , d 1 d = , dx c d d2 1 d2 = 2 2, dx2 c d ....

Example 18.5.1 Consider the Euler equation 2 3 y + y + 2 y = 0. x x Under the change of variables, x = c , y(x) = u(), this equation becomes 2 1 3 1 u + u + 2 2u = 0 c2 c c c 2 3 u + u + 2 u = 0. Thus this equation is invariant under the change of variables x = c .

996

Example 18.5.2 For a nonlinear example, consider the equation y y + y y + 2 = 0. xy x

With the change of variables x = c , y(x) = u() the equation becomes u u u u + 3 + 3 2 =0 2 c c c u c u u u u + + 2 = 0. u We see that this equation is also equidimensional-in-x. You may recall that the change of variables x = et reduces an Euler equation to a constant coecient equation. To generalize this result to nonlinear equations we will see that the same change of variables reduces an equidimensional-in-x equation to an autonomous equation. Writing the derivatives with respect to x in terms of t, x = et , d dt d d = = et dx dx dt dt x x2 d2 d =x dx2 dx x d d = dx dt d dx x d d2 d = 2 . dx dt dt

Example 18.5.3 Consider the equation in Example 18.5.2 y y + y y + 2 = 0. xy x 997

Applying the change of variables x = et , y(x) = u(t) yields an autonomous equation for u(t). x2 y x y + (u u )u + x2 y + xy = 0 y u u +u =0 u

Result 18.5.1 A dierential equation that is invariant under the change of variables x = c is equidimensional-in-x. Such an equation can be reduced to autonomous equation of the same order with the change of variables, x = et .

18.6

*Equidimensional-in-y Equations

A dierential equation is said to be equidimensional-in-y if it is invariant under the change of variables y(x) = c v(x). Note that all linear homogeneous equations are equidimensional-in-y. Example 18.6.1 Consider the linear equation y + p(x)y + q(x)y = 0. With the change of variables y(x) = cv(x) the equation becomes cv + p(x)cv + q(x)cv = 0 v + p(x)v + q(x)v = 0 Thus we see that the equation is invariant under the change of variables.

998

Example 18.6.2 For a nonlinear example, consider the equation y y + (y )2 y 2 = 0. Under the change of variables y(x) = cv(x) the equation becomes. cv cv + (cv )2 (cv)2 = 0 v v + (v )2 v 2 = 0. Thus we see that this equation is also equidimensional-in-y. The change of variables y(x) = eu(x) reduces an nth order equidimensional-in-y equation to an equation of order n 1 for u . Writing the derivatives of eu(x) , d u e = u eu dx d2 u e = (u + (u )2 ) eu dx2 d3 u e = (u + 3u u + (u )3 ) eu . 3 dx Example 18.6.3 Consider the linear equation in Example 18.6.1 y + p(x)y + q(x)y = 0. Under the change of variables y(x) = eu(x) the equation becomes (u + (u )2 ) eu +p(x)u eu +q(x) eu = 0 u + (u )2 + p(x)u + q(x) = 0. Thus we have a Riccati equation for u . This transformation might seem rather useless since linear equations are usually easier to work with than nonlinear equations, but it is often useful in determining the asymptotic behavior of the equation. 999

Example 18.6.4 From Example 18.6.2 we have the equation y y + (y )2 y 2 = 0. The change of variables y(x) = eu(x) yields (u + (u )2 ) eu eu +(u eu )2 (eu )2 = 0 u + 2(u )2 1 = 0 u = 2(u )2 + 1 Now we have a Riccati equation for u . We make the substitution u =
v . 2v

(v )2 (v )2 v = 2 2 + 1 2v 2v 2 4v v 2v = 0 v = c1 e 2x +c2 e 2x c1 e 2x c2 e 2x u =2 2 c1 e 2x +c2 e 2x c1 2 e 2x c2 2 e 2x u=2 dx + c3 c1 e 2x +c2 e 2x


u = 2 log c1 e

2x

+c2 e

2x 2

+ c3 e c3

y = c1 e The constants are redundant, the general solution is

2x

+c2 e

2x

y = c1 e

2x

+c2 e

2x

1000

Result 18.6.1 A dierential equation is equidimensional-in-y if it is invariant under the change of variables y(x) = cv(x). An nth order equidimensional-in-y equation can be reduced to an equation of order n 1 in u with the change of variables y(x) = eu(x) .

18.7

*Scale-Invariant Equations

Result 18.7.1 An equation is scale invariant if it is invariant under the change of variables, x = c, y(x) = c v(), for some value of . A scale-invariant equation can be transformed to an equidimensional-in-x equation with the change of variables, y(x) = x u(x).
Example 18.7.1 Consider the equation y + x2 y 2 = 0. Under the change of variables x = c, y(x) = c v() this equation becomes c v () + c2 x2 c2 v 2 () = 0. 2 c Equating powers of c in the two terms yields = 4. Introducing the change of variables y(x) = x4 u(x) yields d2 4 x u(x) + x2 (x4 u(x))2 = 0 dx2 x4 u 8x5 u + 20x6 u + x6 u2 = 0 x2 u 8xu + 20u + u2 = 0. We see that the equation for u is equidimensional-in-x.

1001

18.8

Exercises

Exercise 18.1 1. Find the general solution and the singular solution of the Clairaut equation, y = xp + p2 . 2. Show that the singular solution is the envelope of the general solution. Hint, Solution

Bernoulli Equations
Exercise 18.2 (mathematica/ode/techniques nonlinear/bernoulli.nb) Consider the Bernoulli equation dy + p(t)y = q(t)y . dt 1. Solve the Bernoulli equation for = 1. 2. Show that for = 1 the substitution u = y 1 reduces Bernoullis equation to a linear equation. 3. Find the general solution to the following equations. t2 (a) dy + 2xy + y 2 = 0 dx (b) Hint, Solution 1002 dy + 2ty y 3 = 0, t > 0 dt

Exercise 18.3 Consider a population, y. Let the birth rate of the population be proportional to y with constant of proportionality 1. Let the death rate of the population be proportional to y 2 with constant of proportionality 1/1000. Assume that the population is large enough so that you can consider y to be continuous. What is the population as a function of time if the initial population is y0 ? Hint, Solution Exercise 18.4 Show that the transformation u = y 1n reduces the equation to a linear rst order equation. Solve the equations 1. t2 2. dy + 2ty y 3 = 0 t > 0 dt

dy = ( cos t + T ) y y 3 , and T are real constants. (From a uid ow stability problem.) dt

Hint, Solution

Riccati Equations
Exercise 18.5 1. Consider the Ricatti equation, dy = a(x)y 2 + b(x)y + c(x). dx Substitute y = yp (x) + 1 u(x)

into the Ricatti equation, where yp is some particular solution to obtain a rst order linear dierential equation for u. 2. Consider a Ricatti equation, y = 1 + x2 2xy + y 2 . 1003

Verify that yp (x) = x is a particular solution. Make the substitution y = yp + 1/u to nd the general solution. What would happen if you continued this method, taking the general solution for yp ? Would you be able to nd a more general solution? 3. The substitution u au gives us the second order, linear, homogeneous dierential equation, y= u a + b u + acu = 0. a

The general solution for u has two constants of integration. However, the solution for y should only have one constant of integration as it satises a rst order equation. Write y in terms of the solution for u and verify tha y has only one constant of integration. Hint, Solution

Exchanging the Dependent and Independent Variables


Exercise 18.6 Solve the dierential equation y = Hint, Solution

y . xy + y

Autonomous Equations *Equidimensional-in-x Equations *Equidimensional-in-y Equations *Scale-Invariant Equations

1004

18.9
Hint 18.1

Hints

Bernoulli Equations
Hint 18.2 Hint 18.3 The dierential equation governing the population is dy y2 =y , dt 1000 This is a Bernoulli equation. Hint 18.4 y(0) = y0 .

Riccati Equations
Hint 18.5

Exchanging the Dependent and Independent Variables


Hint 18.6 Exchange the dependent and independent variables.

Autonomous Equations *Equidimensional-in-x Equations


1005

*Equidimensional-in-y Equations *Scale-Invariant Equations

1006

18.10

Solutions
y = xp + p2 . (18.2)

Solution 18.1 We consider the Clairaut equation,

1. We dierentiate Equation 18.2 with respect to x to obtain a second order dierential equation. y = y + xy + 2y y y (2y + x) = 0 Equating the rst or second factor to zero will lead us to two distinct solutions. y = 0 or y = x 2

If y = 0 then y p is a constant, (say y = c). From Equation 18.2 we see that the general solution is, y(x) = cx + c2 . Recall that the general solution of a rst order dierential equation has one constant of integration. If y = x/2 then y = x2 /4 + const. We determine the constant by substituting the expression into Equation 18.2. x2 x x 2 +c=x + 4 2 2 Thus we see that a singular solution of the Clairaut equation is 1 y(x) = x2 . 4 (18.4) (18.3)

Recall that a singular solution of a rst order nonlinear dierential equation has no constant of integration. 1007

-4

-2 -2 -4

Figure 18.1: The Envelope of y = cx + c2 . 2. Equating the general and singular solutions, y(x), and their derivatives, y (x), gives us the system of equations, 1 1 cx + c2 = x2 , c = x. 4 2 Since the rst equation is satised for c = x/2, we see that the solution y = cx + c2 is tangent to the solution y = x2 /4 at the point (2c, |c|). The solution y = cx + c2 is plotted for c = . . . , 1/4, 0, 1/4, . . . in Figure 18.1. The envelope of a one-parameter family F (x, y, c) = 0 is given by the system of equations, F (x, y, c) = 0, Fc (x, y, c) = 0.

For the family of solutions y = cx + c2 these equations are y = cx + c2 , 0 = x + 2c.


2

Substituting the solution of the second equation, c = x/2, into the rst equation gives the envelope, y= 1 1 x x+ x 2 2 1008 1 = x2 . 4

Thus we see that the singular solution is the envelope of the general solution.

Bernoulli Equations
Solution 18.2 1. dy + p(t)y = q(t)y dt dy = (q p) dt y ln y = (q p) dt + c
(qp) dt

y = ce 2. We consider the Bernoulli equation,

dy + p(t)y = q(t)y , dt We divide by y .

= 1.

y y + p(t)y 1 = q(t) This suggests the change of dependent variable u = y 1 , u = (1 )y y . 1 d 1 y + p(t)y 1 = q(t) 1 dt du + (1 )p(t)u = (1 )q(t) dt Thus we obtain a linear equation for u which when solved will give us an implicit solution for y. 1009

3. (a) t2 dy + 2ty y 3 = 0, t > 0 dt 1 y t2 3 + 2t 2 = 1 y y 1 t2 u + 2tu = 1 2 4 2 u u= 2 t t The integrating factor is =e


(4/t) dt

We make the change of variables u = y 2 .

= e4 ln t = t4 .

We multiply by the integrating factor and integrate to obtain the solution. d 4 t u = 2t6 dt 2 u = t1 + ct4 5 2 y 2 = t1 + ct4 5 y= (b) dy + 2xy + y 2 = 0 dx y 2x + = 1 y2 y 1010 1
2 1 t 5

y = + ct4

5t 2 + ct5

We make the change of variables u = y 1 . u 2xu = 1 The integrating factor is =e


(2x) dx

= ex .

We multiply by the integrating factor and integrate to obtain the solution. d 2 2 ex u = ex dx u = ex


2

ex dx + c ex ex ex2 dx + c
2

y= Solution 18.3 The dierential equation governing the population is dy y2 =y , dt 1000

y(0) = y0 .

We recognize this as a Bernoulli equation. The substitution u(t) = 1/y(t) yields du 1 =u , dt 1000 u +u= u= u(0) = 1 . y0

1 1000

t 1 t et e + e d y0 1000 0 1 1 1 et u= + 1000 y0 1000

1011

Solving for y(t), y(t) = 1 + 1000 1 1 y0 1000


1

et

As a check, we see that as t , y(t) 1000, which is an equilibrium solution of the dierential equation. y2 dy =0=y dt 1000 Solution 18.4 1. t2 dy + 2ty y 3 = 0 dt dy + 2t1 y = t2 y 3 dt y = 1000.

We make the change of variables u(t) = y 2 (t). u 4t1 u = 2t2 This gives us a rst order, linear equation. The integrating factor is I(t) = e
4t1 dt

= e4 log t = t4 .

We multiply by the integrating factor and integrate. d 4 t u = 2t6 dt 2 t4 u = t5 + c 5 2 1 u = t + ct4 5 1012

Finally we write the solution in terms of y(t). y(t) = 1


2 1 t 5

+ ct4

y(t) = 2.

5t 2 + ct5

dy ( cos t + T ) y = y 3 dt We make the change of variables u(t) = y 2 (t). u + 2 ( cos t + T ) u = 2 This gives us a rst order, linear equation. The integrating factor is I(t) = e
2( cos t+T ) dt

= e2( sin t+T t)

We multiply by the integrating factor and integrate. d 2( sin t+T t) e u = 2 e2( sin t+T t) dt u = 2 e2( sin t+T t) Finally we write the solution in terms of y(t). y= 2 e sin t+T t e2( sin t+T t) dt + c e2( sin t+T t) dt + c

1013

Riccati Equations
Solution 18.5 We consider the Ricatti equation, dy = a(x)y 2 + b(x)y + c(x). dx 1. We substitute 1 u(x) into the Ricatti equation, where yp is some particular solution. y = yp (x) + yp yp 1 1 u 2 = +a(x) yp + 2 + 2 + b(x) yp + 2 u u u u u 1 yp 1 2 = b(x) + a(x) 2 + 2 u u u u u = (b + 2ayp ) u a We obtain a rst order linear dierential equation for u whose solution will contain one constant of integration. 2. We consider a Ricatti equation, y = 1 + x2 2xy + y 2 . We verify that yp (x) = x is a solution. 1 = 1 + x2 2xx + x2 Substituting y = yp + 1/u into Equation 18.6 yields, u = (2x + 2x) u 1 u = x + c y =x+ 1 cx (18.6) + c(x) (18.5)

1014

1 What would happen if we continued this method? Since y = x + cx is a solution of the Ricatti equation we can make the substitution, 1 1 y =x+ + , (18.7) c x u(x) which will lead to a solution for y which has two constants of integration. Then we could repeat the process, substituting the sum of that solution and 1/u(x) into the Ricatti equation to nd a solution with three constants of integration. We know that the general solution of a rst order, ordinary dierential equation has only one constant of integration. Does this method for Ricatti equations violate this theorem? Theres only one way to nd out. We substitute Equation 18.7 into the Ricatti equation.

u = 2x + 2 x + u =

1 cx

u1

2 u1 cx 2 u + u = 1 cx

The integrating factor is 1 . (c x)2 Upon multiplying by the integrating factor, the equation becomes exact. I(x) = e2/(cx) = e2 log(cx) = 1 1 u = 2 (c x) (c x)2 1 + b(c x)2 u = (c x)2 cx u = x c + b(c x)2 Thus the Ricatti equation has the solution, y =x+ 1 1 + . c x x c + b(c x)2 1015 d dx

It appears that we we have found a solution that has two constants of integration, but appearances can be deceptive. We do a little algebraic simplication of the solution. y =x+ 1 1 + c x (b(c x) 1)(c x) (b(c x) 1) + 1 y =x+ (b(c x) 1)(c x) b y =x+ b(c x) 1 1 y =x+ (c 1/b) x

This is actually a solution, (namely the solution we had before), with one constant of integration, (namely c1/b). Thus we see that repeated applications of the procedure will not produce more general solutions. 3. The substitution u au gives us the second order, linear, homogeneous dierential equation, y= u a + b u + acu = 0. a

The solution to this linear equation is a linear combination of two homogeneous solutions, u1 and u2 . u = c1 u1 (x) + c2 u2 (x) The solution of the Ricatti equation is then y= c1 u1 (x) + c2 u2 (x) . a(x)(c1 u1 (x) + c2 u2 (x)) 1016

Since we can divide the numerator and denominator by either c1 or c2 , this answer has only one constant of integration, (namely c1 /c2 or c2 /c1 ).

Exchanging the Dependent and Independent Variables

Solution 18.6 Exchanging the dependent and independent variables in the dierential equation,

y =

y , xy + y

yields

x (y) = y 1/2 x + y 1/2 . 1017

This is a rst order dierential equation for x(y). x y 1/2 x = y 1/2 d 2y 3/2 2y 3/2 x exp = y 1/2 exp dy 3 3 2y 3/2 2y 3/2 x exp = exp + c1 3 3 2y 3/2 x = 1 + c1 exp 3 3/2 x+1 2y = exp c1 3 x+1 2 log = y 3/2 c1 3 y= y= 3 log 2 c+ x+1 c1
2/3

3 log(x + 1) 2

2/3

Autonomous Equations *Equidimensional-in-x Equations *Equidimensional-in-y Equations *Scale-Invariant Equations

1018

Chapter 19 Transformations and Canonical Forms


Prize intensity more than extent. Excellence resides in quality not in quantity. The best is always few and rare abundance lowers value. Even among men, the giants are usually really dwarfs. Some reckon books by the thickness, as if they were written to exercise the brawn more than the brain. Extent alone never rises above mediocrity; it is the misfortune of universal geniuses that in attempting to be at home everywhere are so nowhere. Intensity gives eminence and rises to the heroic in matters sublime. -Balthasar Gracian

19.1

The Constant Coecient Equation

The solution of any second order linear homogeneous dierential equation can be written in terms of the solutions to either y = 0, or y y = 0 Consider the general equation y + ay + by = 0. 1019

We can solve this dierential equation by making the substitution y = ex . This yields the algebraic equation 2 + a + b = 0. 1 = a a2 4b 2 There are two cases to consider. If a2 = 4b then the solutions are y1 = e(a+ If a2 = 4b then we have y1 = eax/2 , y2 = x eax/2 Note that regardless of the values of a and b the solutions are of the form y = eax/2 u(x) We would like to write the solutions to the general dierential equation in terms of the solutions to simpler dierential equations. We make the substitution y = ex u The derivatives of y are y = ex (u + u) y = ex (u + 2u + 2 u) Substituting these into the dierential equation yields u + (2 + a)u + (2 + a + b)u = 0 In order to get rid of the u term we choose The equation is then u + b There are now two cases to consider. 1020 a2 4 u = 0. a = . 2
a2 4b)x/2

y2 = e(a

a2 4b)x/2

Case 1. If b = a2 /4 then the dierential equation is u =0 which has solutions 1 and x. The general solution for y is then y = eax/2 (c1 + c2 x). Case 2. If b = a2 /4 then the dierential equation is u We make the change variables u(x) = v(), The derivatives in terms of are d d d 1 d = = dx dx d d 2 1 d 1 d 1 d2 d = = 2 2. dx2 d d d The dierential equation for v is 1 v 2 v 2 We choose = a2 b 4 1021 a2 b v =0 4 a2 b v =0 4
1/2

a2 b u = 0. 4 x = .

to obtain v v =0 which has solutions e . The solution for y is y = ex c1 ex/ +c2 ex/ 2 2 y = eax/2 c1 e a /4b x +c2 e a /4b

19.2
19.2.1

Normal Form
Second Order Equations
y + p(x)y + q(x)y = 0. (19.1)

Consider the second order equation Through a change of dependent variable, this equation can be transformed to u + I(x)y = 0. This is known as the normal form of (19.1). The function I(x) is known as the invariant of the equation. Now to nd the change of variables that will accomplish this transformation. We make the substitution y(x) = a(x)u(x) in (19.1). au + 2a u + a u + p(au + a u) + qau = 0 u + 2 To eliminate the u term, a(x) must satisfy a +p=0 a 1 a + pa = 0 2 2 1022 a +p u + a a pa + +q u=0 a a

1 2 For this choice of a, our dierential equation for u becomes a = c exp u + q

p(x) dx .

p2 p 4 2

u = 0.

Two dierential equations having the same normal form are called equivalent.

Result 19.2.1 The change of variables y(x) = exp transforms the dierential equation y + p(x)y + q(x)y = 0 into its normal form u + I(x)u = 0 where the invariant of the equation, I(x), is p2 p I(x) = q . 4 2 19.2.2 Higher Order Dierential Equations
y + p(x)y + q(x)y + r(x)y = 0. 1023

1 2

p(x) dx u(x)

Consider the third order dierential equation

We can eliminate the y term. Making the change of dependent variable y = u exp 1 3 p(x) dx

1 1 y = u pu exp p(x) dx 3 3 1 1 2 y = u pu + (p2 3p )u exp p(x) dx 3 9 3 1 1 1 y = u pu + (p2 3p )u + (9p 9p p3 )u exp 3 27 3 yields the dierential equation

p(x) dx

1 1 u + (3q 3p p2 )u + (27r 9pq 9p + 2p3 )u = 0. 3 27

Result 19.2.2 The change of variables y(x) = exp transforms the dierential equation y (n) + pn1 (x)y (n1) + pn2 (x)y (n2) + + p0 (x)y = 0 into the form u(n) + an2 (x)u(n2) + an3 (x)u(n3) + + a0 (x)u = 0. 1 n pn1 (x) dx u(x)

1024

19.3
19.3.1

Transformations of the Independent Variable


Transformation to the form u + a(x) u = 0

Consider the second order linear dierential equation y + p(x)y + q(x)y = 0. We make the change of independent variable = f (x), The derivatives in terms of are d d d d = =f dx dx d d 2 d d d d2 d =f f = (f )2 2 + f 2 dx d d d d The dierential equation becomes (f )2 u + f u + pf u + qu = 0. In order to eliminate the u term, f must satisfy f + pf = 0 f = exp f= The dierential equation for u is then u + exp p(x) dx p(x) dx dx. u() = y(x).

q u=0 (f )2 1025

u () + q(x) exp 2

p(x) dx u() = 0.

Result 19.3.1 The change of variables = exp p(x) dx dx, u() = y(x)

transforms the dierential equation y + p(x)y + q(x)y = 0 into u () + q(x) exp 2 p(x) dx u() = 0.

19.3.2

Transformation to a Constant Coecient Equation


y + p(x)y + q(x)y = 0.

Consider the second order linear dierential equation

With the change of independent variable = f (x), the dierential equation becomes (f )2 u + (f + pf )u + qu = 0. For this to be a constant coecient equation we must have (f )2 = c1 q, and 1026 f + pf = c2 q, u() = y(x),

for some constants c1 and c2 . Solving the rst condition, f = c q, f =c The second constraint becomes f + pf = const q 1 1/2 cq q + pcq 1/2 2 = const q q + 2pq = const. q 3/2 q(x) dx.

Result 19.3.2 Consider the dierential equation y + p(x)y + q(x)y = 0. If the expression q + 2pq q 3/2 is a constant then the change of variables =c q(x) dx, u() = y(x),

will yield a constant coecient dierential equation. (Here c is an arbitrary constant.)

1027

19.4

Integral Equations
x

Volterras Equations. Volterras integral equation of the rst kind has the form N (x, )f () d = f (x).
a

The Volterra equation of the second kind is


x

y(x) = f (x) +
a

N (x, )y() d.

N (x, ) is known as the kernel of the equation. Fredholms Equations. Fredholms integral equations of the rst and second kinds are
b

N (x, )f () d = f (x),
a b

y(x) = f (x) +
a

N (x, )y() d.

19.4.1

Initial Value Problems

Consider the initial value problem y + p(x)y + q(x)y = f (x), Integrating this equation twice yields
x a a x

y(a) = ,

y (a) = .

y () + p()y () + q()y() d d =
a a

f () d d

1028

(x )[y () + p()y () + q()y()] d =


a a

(x )f () d.

Now we use integration by parts. (x )y ()


x x a x

y () d + (x )p()y()
x

x a

[(x )p () p()]y() d

+
a

(x )q()y() d =
a

(x )f () d.
x

(x a)y (a) + y(x) y(a) (x a)p(a)y(a)


a x x

[(x )p () p()]y() d

+
a

(x )q()y() d =
a

(x )f () d.

We obtain a Volterra integral equation of the second kind for y(x).


x x

y(x) =
a

(x )f () d + (x a)(p(a) + ) + +
a

(x )[p () q()] p() y() d.

Note that the initial conditions for the dierential equation are built into the Volterra equation. Setting x = a in the Volterra equation yields y(a) = . Dierentiating the Volterra equation,
x x

y (x) =
a

f () d + (p(a) + ) p(x)y(x) +
a

[p () q()] p()y() d

and setting x = a yields y (a) = p(a) + p(a) = . (Recall from calculus that d dx
x x

g(x, ) d = g(x, x) + 1029

[g(x, )] d.) x

Result 19.4.1 The initial value problem y + p(x)y + q(x)y = f (x), y(a) = , y (a) = .

is equivalent to the Volterra equation of the second kind


x

y(x) = F (x) +
a

N (x, )y() d

where
x

F (x) =
a

(x )f () d + (x a)(p(a) + ) +

N (x, ) = (x )[p () q()] p(). 19.4.2 Boundary Value Problems


y = f (x), y(a) = , y(b) = . (19.2)

Consider the boundary value problem

To obtain a problem with homogeneous boundary conditions, we make the change of variable y(x) = u(x) + + to obtain the problem u = f (x), u(a) = u(b) = 0. Now we will use Greens functions to write the solution as an integral. First we solve the problem G = (x ), G(a|) = G(b|) = 0. 1030 (x a) ba

The homogeneous solutions of the dierential equation that satisfy the left and right boundary conditions are c1 (x a) and c2 (x b). Thus the Greens function has the form G(x|) = c1 (x a), c2 (x b), for x for x

Imposing continuity of G(x|) at x = and a unit jump of G(x|) at x = , we obtain G(x|) = Thus the solution of the (19.2) is y(x) = + Now consider the boundary value problem y + p(x)y + q(x)y = 0, From the above result we can see that the solution satises y(x) = + (x a) + ba
b (xa)(b) , ba (xb)(a) , ba

for x for x

(x a) + ba

G(x|)f () d.
a

y(a) = ,

y(b) = .

G(x|)[f () p()y () q()y()] d.


a

Using integration by parts, we can write


b

G(x|)p()y () d = G(x|)p()y()
b

b a

+
a

G(x|) p() + G(x|)p () y() d

=
a

G(x|) p() + G(x|)p () y() d. 1031

Substituting this into our expression for y(x), y(x) = + (x a) + ba


b b

G(x|)f () d +
a a

G(x|) p() + G(x|)[p () q()] y() d,

we obtain a Fredholm integral equation of the second kind.

Result 19.4.2 The boundary value problem y + p(x)y + q(x)y = f (x), y(a) = , y(b) = .

is equivalent to the Fredholm equation of the second kind


b

y(x) = F (x) +
a

N (x, )y() d

where F (x) = + (x a) + ba
b b

G(x|)f () d,
a

N (x, ) = G(x|) = H(x|) =

H(x|)y() d,
a (xa)(b) , for x ba (xb)(a) , for x , ba (xa) (xa)(b) [p () q()] ba p() + ba (xb) (xb)(a) [p () q()] ba p() + ba

for x for x .

1032

19.5

Exercises

The Constant Coecient Equation Normal Form


Exercise 19.1 Solve the dierential equation 4 1 y + 2+ x y + 24 + 12x + 4x2 y = 0. 3 9 Hint, Solution

Transformations of the Independent Variable Integral Equations


Exercise 19.2 Show that the solution of the dierential equation y + 2(a + bx)y + (c + dx + ex2 )y = 0 can be written in terms of one of the following canonical forms: v v v v Hint, Solution Exercise 19.3 Show that the solution of the dierential equation y +2 a+ b x y + c+ d e + 2 x x y=0 + ( 2 + A)v = 0 = v +v =0 = 0.

1033

can be written in terms of one of the following canonical forms: A B + 2 v=0 1 A v + + v=0 2 A v + 2v = 0 v + 1+ Hint, Solution Exercise 19.4 Show that the second order Euler equation x2 d2 y dy + a1 x + a0 y = 0 2x d dx

can be transformed to a constant coecient equation. Hint, Solution Exercise 19.5 Solve Bessels equation of order 1/2, 1 1 y + y + 1 2 x 4x Hint, Solution y = 0.

1034

19.6

Hints

The Constant Coecient Equation Normal Form


Hint 19.1 Transform the equation to normal form.

Transformations of the Independent Variable Integral Equations


Hint 19.2 Transform the equation to normal form and then apply the scale transformation x = + . Hint 19.3 Transform the equation to normal form and then apply the scale transformation x = . Hint 19.4 Make the change of variables x = et , y(x) = u(t). Write the derivatives with respect to x in terms of t. x = et dx = et dt d d = et dx dt d d = x dx dt Hint 19.5 Transform the equation to normal form.

1035

19.7

Solutions

The Constant Coecient Equation Normal Form


Solution 19.1 4 1 y + 2+ x y + 24 + 12x + 4x2 y = 0 3 9 To transform the equation to normal form we make the substitution y = exp = exx The invariant of the equation is I(x) = 1 1 24 + 12x + 4x2 9 4 = 1. 4 2+ x 3
2

1 2 u

4 2+ x 3

dx u

2 /3

1 d 2 dx

4 2+ x 3

The normal form of the dierential equation is then u +u=0 which has the general solution u = c1 cos x + c2 sin x Thus the equation for y has the general solution y = c1 exx
2 /3

cos x + c2 exx

2 /3

sin x.

1036

Transformations of the Independent Variable Integral Equations


Solution 19.2 The substitution that will transform the equation to normal form is y = exp = eaxbx The invariant of the equation is 1 1 d I(x) = c + dx + ex2 (2(a + bx))2 (2(a + bx)) 4 2 dx = c b a2 + (d 2ab)x + (e b2 )x2 + x + x2 The normal form of the dierential equation is u + ( + x + x2 )u = 0 We consider the following cases: = 0. = 0. = 0. We immediately have the equation u = 0. = 0. With the change of variables v() = u(x), we obtain v + v = 0. 1037 x = 1/2 , 1 2 u. 2(a + bx) dx u

2 /2

= 0. We have the equation y + ( + x)y = 0. The scale transformation x = + yields v + 2 ( + ( + ))y = 0 v = [3 + 2 ( + )]v. Choosing = ()1/3 , yields the dierential equation v = v. = 0. The scale transformation x = + yields v + 2 [ + ( + ) + ( + )2 ]v = 0 v + 2 [ + + 2 + ( + 2) + 2 2 ]v = 0. Choosing = 1/4 , yields the dierential equation v + ( 2 + A)v = 0 where 1 A = 1/2 3/2 . 4 1038 = 2 =

Solution 19.3 The substitution that will transform the equation to normal form is y = exp 1 b 2 a+ 2 x b ax =x e u. dx u

The invariant of the equation is 1 d d e 1 b 2 a+ I(x) = c + + 2 x x 4 x 2 dx 2 d 2ab e + b b = c ax + + x x2 + + 2. x x The invariant form of the dierential equation is u + + We consider the following cases: = 0. = 0. We immediately have the equation u + = 0. We have the equation u + + 2 x x u = 0. u = 0. x2 + 2 x x u = 0.
2

2 a+

b x

1039

The scale transformation u(x) = v(), x = yields v + Choosing = 1 , we obtain v + = 0. The scale transformation x = yields v + 2 + Choosing = 1/2 , we obtain v + 1+ Solution 19.4 We write the derivatives with respect to x in terms of t. x = et dx = et dt d d = et dx dt d d = x dx dt
d Now we express x2 dx2 in terms of t.
2

+ 2 1 + 2

u = 0.

u = 0.

+ 2

v = 0.

1/2 + 2

v = 0.

x2

d2 d =x 2 dx dx

d dx

d d2 d = 2 dx dt dt

1040

Thus under the change of variables, x = et , y(x) = u(t), the Euler equation becomes u u + a1 u + a0 u = 0 u + (a1 1)u + a0 u = 0. Solution 19.5 The transformation y = exp will put the equation in normal form. The invariant is I(x) = Thus we have the dierential equation u + u = 0, with the solution u = c1 cos x + c2 sin x. The solution of Bessels equation of order 1/2 is y = c1 x1/2 cos x + c2 x1/2 sin x. 1 1 4x2 1 4 1 x2 1 1 = 1. 2 x2 1 2 1 dx x = x1/2 u

1041

Chapter 20 The Dirac Delta Function


I do not know what I appear to the world; but to myself I seem to have been only like a boy playing on a seashore, and diverting myself now and then by nding a smoother pebble or a prettier shell than ordinary, whilst the great ocean of truth lay all undiscovered before me. - Sir Issac Newton

20.1

Derivative of the Heaviside Function


H(x) = 0 1 for x < 0, for x > 0.

The Heaviside function H(x) is dened

The derivative of the Heaviside function is zero for x = 0. At x = 0 the derivative is undened. We will represent the derivative of the Heaviside function by the Dirac delta function, (x). The delta function is zero for x = 0 and innite at the point x = 0. Since the derivative of H(x) is undened, (x) is not a function in the conventional sense of the word. One can derive the properties of the delta function rigorously, but the treatment in this text will be almost entirely heuristic. 1042

The Dirac delta function is dened by the properties 0 for x = 0, for x = 0,

(x) =

and

(x) dx = 1.

The second property comes from the fact that (x) represents the derivative of H(x). The Dirac delta function is conceptually pictured in Figure 20.1.

Figure 20.1: The Dirac Delta Function.

Let f (x) be a continuous function that vanishes at innity. Consider the integral

f (x)(x) dx.

1043

We use integration by parts to evaluate the integral.

f (x)(x) dx = f (x)H(x)

f (x)H(x) dx

=
0

f (x) dx

= [f (x)] 0 = f (0) We assumed that f (x) vanishes at innity in order to use integration by parts to evaluate the integral. However, since the delta function is zero for x = 0, the integrand is nonzero only at x = 0. Thus the behavior of the function at innity should not aect the value of the integral. Thus it is reasonable that f (0) = f (x)(x) dx holds for all continuous functions. By changing variables and noting that (x) is symmetric we can derive a more general formula.

f (0) =

f ()() d f ( + x)() d

f (x) =

f (x) =

f ()( x) d f ()(x ) d

f (x) =

This formula is very important in solving inhomogeneous dierential equations.

20.2

The Delta Function as a Limit


b(x, ) = 0
1

Consider a function b(x, ) dened by for |x| > /2 for |x| < /2. 1044

The graph of b(x, 1/10) is shown in Figure 20.2.

10 5

-1

Figure 20.2: Graph of b(x, 1/10).

The Dirac delta function (x) can be thought of as b(x, ) in the limit as dened satises the properties,

0. Note that the delta function so

(x) =

0 for x = 0 for x = 0

and

(x) dx = 1

Delayed Limiting Process. When the Dirac delta function appears inside an integral, we can think of the delta function as a delayed limiting process.

f (x)(x) dx lim
0

f (x)b(x, ) dx.

1045

Let f (x) be a continuous function and let F (x) = f (x). We compute the integral of f (x)(x).

f (x)(x) dx = lim
0

/2

f (x) dx
/2

1 /2 = lim [F (x)] /2
0

= lim
0

F ( /2) F ( /2)

= F (0) = f (0)

20.3

Higher Dimensions

We can dene a Dirac delta function in n-dimensional Cartesian space, n (x), x Rn . It is dened by the following two properties. n (x) = 0 for x = 0 n (x) dx = 1
Rn

It is easy to verify, that the n-dimensional Dirac delta function can be written as a product of 1-dimensional Dirac delta functions.
n

n (x) =
k=1

(xk )

1046

20.4

Non-Rectangular Coordinate Systems

We can derive Dirac delta functions in non-rectangular coordinate systems by making a change of variables in the relation, n (x) dx = 1
Rn

Where the transformation is non-singular, one merely divides the Dirac delta function by the Jacobian of the transformation to the coordinate system. Example 20.4.1 Consider the Dirac delta function in cylindrical coordinates, (r, , z). The Jacobian is J = r.
0 2 0

3 (x x0 ) r dr d dz = 1 For r0 = 0, the Dirac Delta function is 1 3 (x x0 ) = (r r0 ) ( 0 ) (z z0 ) r since it satises the two dening properties. 1 (r r0 ) ( 0 ) (z z0 ) = 0 for (r, , z) = (r0 , 0 , z0 ) r
0 2 0

1 (r r0 ) ( 0 ) (z z0 ) r dr d dz r
2

=
0

(r r0 ) dr
0

( 0 ) d

(z z0 ) dz = 1

For r0 = 0, we have 3 (x x0 ) =

1 (r) (z z0 ) 2r 1047

since this again satises the two dening properties. 1 (r) (z z0 ) = 0 for (r, z) = (0, z0 ) 2r 2 1 1 (r) (z z0 ) r dr d dz = (r) dr d 2r 2 0 0

2 0

(z z0 ) dz = 1

1048

20.5

Exercises

Exercise 20.1 Let f (x) be a function that is continuous except for a jump discontinuity at x = 0. Using a delayed limiting process, show that f (0 ) + f (0+ ) = f (x)(x) dx. 2 Hint, Solution Exercise 20.2 Show that the Dirac delta function is symmetric. (x) = (x) Hint, Solution Exercise 20.3 Show that (cx) = Hint, Solution Exercise 20.4 We will consider the Dirac delta function with a function as on argument, (y(x)). Assume that y(x) has simple zeros at the points {xn }. y(xn ) = 0, y (xn ) = 0 Further assume that y(x) has no multiple zeros. (If y(x) has multiple zeros (y(x)) is not well-dened in the same sense that 1/0 is not well-dened.) Prove that (y(x)) =
n

(x) . |c|

(x xn ) . |y (xn )|

Hint, Solution 1049

Exercise 20.5 Justify the identity

f (x) (n) (x) dx = (1)n f (n) (0)

From this show that (n) (x) = (1)n (n) (x) and x (n) (x) = n (n1) (x). Hint, Solution Exercise 20.6 Consider x = (x1 , . . . , xn ) Rn and the curvilinear coordinate system = (1 , . . . , n ). Show that (x a) = ( ) |J|

where a and are corresponding points in the two coordinate systems and J is the Jacobian of the transformation from x to . x J Hint, Solution Exercise 20.7 Determine the Dirac delta function in spherical coordinates, (r, , ). x = r cos sin , Hint, Solution y = r sin sin , z = r cos

1050

20.6
Hint 20.1

Hints

Hint 20.2 Verify that (x) satises the two properties of the Dirac delta function. Hint 20.3 Evaluate the integral,

f (x)(cx) dx,

by noting that the Dirac delta function is symmetric and making a change of variables. Hint 20.4 Let the points {m } partition the interval ( . . . ) such that y (x) is monotone on each interval (m . . . m+1 ). Consider some such interval, (a . . . b) (m . . . m+1 ). Show that
b

(y(x)) dx =
a

(y) |y (xn )|

dy

if y(xn ) = 0 for a < xn < b otherwise

for = min(y(a), y(b)) and = max(y(a), y(b)). Now consider the integral on the interval ( . . . ) as the sum of integrals on the intervals {(m . . . m+1 )}. Hint 20.5 Justify the identity,

f (x) (n) (x) dx = (1)n f (n) (0),

with integration by parts.

1051

Hint 20.6 The Dirac delta function is dened by the following two properties. (x a) = 0 for x = a (x a) dx = 1
Rn

Verify that ( )/|J| satises these properties in the coordinate system. Hint 20.7 Consider the special cases 0 = 0, and r0 = 0.

1052

20.7

Solutions

Solution 20.1 Let F (x) = f (x).

f (x)(x) dx = lim
0

1 1 1

f (x)b(x, ) dx
0 /2

= lim
0

f (x)b(x, ) dx +
/2 0

f (x)b(x, ) dx

= lim
0

((F (0) F ( /2)) + (F ( /2) F (0)))

1 F (0) F ( /2) F ( /2) F (0) + 0 2 /2 /2 + F (0 ) + F (0 ) = 2 f (0 ) + f (0+ ) = 2 = lim Solution 20.2 (x) satises the two properties of the Dirac delta function. (x) = 0 for x = 0

(x) dx =

(x) (dx) =

(x) dx = 1

Therefore (x) = (x).

1053

Solution 20.3 We note the the Dirac delta function is symmetric and we make a change of variables to derive the identity.

(cx) dx =

(|c|x) dx (x) dx |c|

(cx) =

(x) |c|

Solution 20.4 Let the points {m } partition the interval ( . . . ) such that y (x) is monotone on each interval (m . . . m+1 ). Consider some such interval, (a . . . b) (m . . . m+1 ). Note that y (x) is either entirely positive or entirely negative in the interval. First consider the case when it is positive. In this case y(a) < y(b).

y(b)

(y(x)) dx =
a y(a) y(b)

(y)

dy dx

dy

=
y(a)

(y) dy y (x) dy for y(xn ) = 0 if y(a) < 0 < y(b) otherwise

y(b) (y) y(a) y (xn )

1054

Now consider the case that y (x) is negative on the interval so y(a) > y(b).

y(b)

(y(x)) dx =
a y(a) y(b)

(y)

dy dx

dy

=
y(a) y(a)

(y) dy y (x) (y) dy y (x) dy for y(xn ) = 0 if y(b) < 0 < y(a) otherwise

=
y(b)

y(a) (y) y(b) y (xn )

We conclude that

(y(x)) dx =
a

(y) |y (xn )|

dy

if y(xn ) = 0 for a < xn < b otherwise

for = min(y(a), y(b)) and = max(y(a), y(b)). 1055

Now we turn to the integral of (y(x)) on ( . . . ). Let m = min(y(m ), y(m )) and m = max(y(m ), y(m )).

m+1

(y(x)) dx =
m m

(y(x)) dx
m+1

=
m xn (m ...m+1 ) m m+1

(y(x)) dx (y) dy |y (xn )|

=
m xn (m ...m+1 ) m

=
n

(y) dy |y (xn )| (y) dy |y (xn )|

=
n

(y(x)) =
n

(x xn ) |y (xn )|

Solution 20.5 To justify the identity,

f (x) (n) (x) dx = (1)n f (n) (0),

1056

we will use integration by parts.


(n) (n1)

f (x)

(x) dx = f (x)

(x)

f (x) (n1) (x) dx

f (x) (n1) (x) dx


n

= (1)

f (n) (x)(x) dx (0)

= (1) f

n (n)

CONTINUE HERE (n) (x) = (1)n (n) (x) and x (n) (x) = n (n1) (x). Solution 20.6 The Dirac delta function is dened by the following two properties. (x a) = 0 for x = a (x a) dx = 1
Rn

We verify that ( )/|J| satises these properties in the coordinate system. ( ) (1 1 ) (n n ) = |J| |J| = 0 for = 1057

( ) |J| d = |J| = = =1

( ) d (1 1 ) (n n ) d (1 1 ) d1 (n n ) dn

We conclude that ( )/|J| is the Dirac delta function in the coordinate system. (x a) = ( ) |J|

Solution 20.7 We consider the Dirac delta function in spherical coordinates, (r, , ). The Jacobian is J = r2 sin().
0 0 2 0

3 (x x0 ) r2 sin() dr d d = 1 For r0 = 0, and 0 = 0, , the Dirac Delta function is 3 (x x0 ) = since it satises the two dening properties. r2
0 0 2 0

r2

1 (r r0 ) ( 0 ) ( 0 ) sin()

1 (r r0 ) ( 0 ) ( 0 ) = 0 for (r, , ) = (r0 , 0 , 0 ) sin()

r2

1 (r r0 ) ( 0 ) ( 0 ) r2 sin() dr d d sin()
2

=
0

(r r0 ) dr
0

( 0 ) d
0

( 0 ) d = 1

1058

For 0 = 0 or 0 = , the Dirac delta function is 3 (x x0 ) = We check that the value of the integral is unity.
0 0 2 0

2r2

1 (r r0 ) ( 0 ) . sin()

2r2

1 (r r0 ) ( 0 ) r2 sin() dr d d sin() 1 = 2 1 (r) 4r2

(r r0 ) dr
0 0

d
0

( 0 ) d = 1

For r0 = 0 the Dirac delta function is 3 (x) = We verify that the value of the integral is unity.
0 0 2 0

1 1 (r r0 ) r2 sin() dr d d = 4r2 4

(r) dr
0 0

d
0

sin() d = 1

1059

Chapter 21 Inhomogeneous Dierential Equations


Feelin stupid? I know I am! -Homer Simpson

21.1

Particular Solutions
L[y] y (n) + pn1 (x)y (n1) + + p1 (x)y + p0 (x)y = 0.

Consider the nth order linear homogeneous equation

Let {y1 , y2 , . . . , yn } be a set of linearly independent homogeneous solutions, L[yk ] = 0. We know that the general solution of the homogeneous equation is a linear combination of the homogeneous solutions.
n

yh =
k=1

ck yk (x)

Now consider the nth order linear inhomogeneous equation L[y] y (n) + pn1 (x)y (n1) + + p1 (x)y + p0 (x)y = f (x). 1060

Any function yp which satises this equation is called a particular solution of the dierential equation. We want to know the general solution of the inhomogeneous equation. Later in this chapter we will cover methods of constructing this solution; now we consider the form of the solution. Let yp be a particular solution. Note that yp + h is a particular solution if h satises the homogeneous equation. L[yp + h] = L[yp ] + L[h] = f + 0 = f Therefore yp + yh satises the homogeneous equation. We show that this is the general solution of the inhomogeneous equation. Let yp and p both be solutions of the inhomogeneous equation L[y] = f . The dierence of yp and p is a homogeneous solution. L[yp p ] = L[yp ] L[p ] = f f = 0 yp and p dier by a linear combination of the homogeneous solutions {yk }. Therefore the general solution of L[y] = f is the sum of any particular solution yp and the general homogeneous solution yh .
n

yp + yh = yp (x) +
k=1

ck yk (x)

Result 21.1.1 The general solution of the nth order linear inhomogeneous equation L[y] = f (x) is y = yp + c1 y1 + c2 y2 + + cn yn , where yp is a particular solution, {y1 , . . . , yn } is a set of linearly independent homogeneous solutions, and the ck s are arbitrary constants.
Example 21.1.1 The dierential equation y + y = sin(2x) has the two homogeneous solutions y1 = cos x, 1061 y2 = sin x,

and a particular solution 1 yp = sin(2x). 3 We can add any combination of the homogeneous solutions to yp and it will still be a particular solution. For example, 1 1 p = sin(2x) sin x 3 3 3x 2 x = sin cos 3 2 2 is a particular solution.

21.2

Method of Undetermined Coecients

The rst method we present for computing particular solutions is the method of undetermined coecients. For some simple dierential equations, (primarily constant coecient equations), and some simple inhomogeneities we are able to guess the form of a particular solution. This form will contain some unknown parameters. We substitute this form into the dierential equation to determine the parameters and thus determine a particular solution. Later in this chapter we will present general methods which work for any linear dierential equation and any inhogeneity. Thus one might wonder why I would present a method that works only for some simple problems. (And why it is called a method if it amounts to no more than guessing.) The answer is that guessing an answer is less grungy than computing it with the formulas we will develop later. Also, the process of this guessing is not random, there is rhyme and reason to it. Consider an nth order constant coecient, inhomogeneous equation. L[y] y (n) + an1 y (n1) + + a1 y + a0 y = f (x) If f (x) is one of a few simple forms, then we can guess the form of a particular solution. Below we enumerate some cases. 1062

f = p(x). If f is an mth order polynomial, f (x) = pm xm + + p1 x + p0 , then guess yp = cm xm + c1 x + c0 . f = p(x) eax . If f is a polynomial times an exponential then guess yp = (cm xm + c1 x + c0 ) eax . f = p(x) eax cos (bx). If f is a cosine or sine times a polynomial and perhaps an exponential, f (x) = p(x) eax cos(bx) or f (x) = p(x) eax sin(bx) then guess yp = (cm xm + c1 x + c0 ) eax cos(bx) + (dm xm + d1 x + d0 ) eax sin(bx). Likewise for hyperbolic sines and hyperbolic cosines. Example 21.2.1 Consider y 2y + y = t2 . The homogeneous solutions are y1 = et and y2 = t et . We guess a particular solution of the form yp = at2 + bt + c. We substitute the expression into the dierential equation and equate coecients of powers of t to determine the parameters. yp 2yp + yp = t2 (2a) 2(2at + b) + (at2 + bt + c) = t2 (a 1)t2 + (b 4a)t + (2a 2b + c) = 0 a 1 = 0, b 4a = 0, 2a 2b + c = 0 a = 1, b = 4, c = 6 A particular solution is yp = t2 + 4t + 6. 1063

If the inhomogeneity is a sum of terms, L[y] = f f1 + +fk , you can solve the problems L[y] = f1 , . . . , L[y] = fk independently and then take the sum of the solutions as a particular solution of L[y] = f . Example 21.2.2 Consider L[y] y 2y + y = t2 + e2t . (21.1) The homogeneous solutions are y1 = et and y2 = t et . We already know a particular solution to L[y] = t2 . We seek a particular solution to L[y] = e2t . We guess a particular solution of the form yp = a e2t . We substitute the expression into the dierential equation to determine the parameter. yp 2yp + yp = e2t 4ae2t 4a e2t +a e2t = e2t a=1 A particular solution of L[y] = e2t is yp = e2t . Thus a particular solution of Equation 21.1 is yp = t2 + 4t + 6 + e2t . The above guesses will not work if the inhomogeneity is a homogeneous solution. In this case, multiply the guess by the lowest power of x such that the guess does not contain homogeneous solutions. Example 21.2.3 Consider L[y] y 2y + y = et . The homogeneous solutions are y1 = et and y2 = t et . Guessing a particular solution of the form yp = a et would not work because L[et ] = 0. We guess a particular solution of the form yp = at2 et 1064

We substitute the expression into the dierential equation and equate coecients of like terms to determine the parameters. yp 2yp + yp = et (at2 + 4at + 2a) et 2(at2 + 2at) et +at2 et = et 2a et = et 1 a= 2 A particular solution is yp = Example 21.2.4 Consider 1 1 y + y + 2 y = x, x > 0. x x The homogeneous solutions are y1 = cos(ln x) and y2 = sin(ln x). We guess a particular solution of the form yp = ax3 We substitute the expression into the dierential equation and equate coecients of like terms to determine the parameter. 1 1 yp + yp + 2 yp = x x x 6ax + 3ax + ax = x 1 a= 10 A particular solution is yp = x3 . 10 t2 t e . 2

1065

21.3

Variation of Parameters

In this section we present a method for computing a particular solution of an inhomogeneous equation given that we know the homogeneous solutions. We will rst consider second order equations and then generalize the result for nth order equations.

21.3.1

Second Order Dierential Equations


L[y] y + p(x)y + q(x)y = f (x),

Consider the second order inhomogeneous equation, on a < x < b.

We assume that the coecient functions in the dierential equation are continuous on [a . . . b]. Let y1 (x) and y2 (x) be two linearly independent solutions to the homogeneous equation. Since the Wronskian, W (x) = exp p(x) dx ,

is non-vanishing, we know that these solutions exist. We seek a particular solution of the form, yp = u1 (x)y1 + u2 (x)y2 . We compute the derivatives of yp . y p = u1 y 1 + u1 y 1 + u 2 y 2 + u 2 y 2 yp = u1 y1 + 2u1 y1 + u1 y1 + u2 y2 + 2u2 y2 + u2 y2 We substitute the expression for yp and its derivatives into the inhomogeneous equation and use the fact that y1 and y2 are homogeneous solutions to simplify the equation. u1 y1 + 2u1 y1 + u1 y1 + u2 y2 + 2u2 y2 + u2 y2 + p(u1 y1 + u1 y1 + u2 y2 + u2 y2 ) + q(u1 y1 + u2 y2 ) = f u1 y1 + 2u1 y1 + u2 y2 + 2u2 y2 + p(u1 y1 + u2 y2 ) = f 1066

This is an ugly equation for u1 and u2 , however, we have an ace up our sleeve. Since u1 and u2 are undetermined functions of x, we are free to impose a constraint. We choose this constraint to simplify the algebra. u1 y 1 + u 2 y 2 = 0 This constraint simplies the derivatives of yp , y p = u 1 y 1 + u 1 y 1 + u2 y 2 + u2 y 2 = u1 y 1 + u2 y 2 y p = u 1 y 1 + u 1 y 1 + u2 y 2 + u 2 y 2 . We substitute the new expressions for yp and its derivatives into the inhomogeneous dierential equation to obtain a much simpler equation than before. u1 y1 + u1 y1 + u2 y2 + u2 y2 + p(u1 y1 + u2 y2 ) + q(u1 y1 + u2 y2 ) = f (x) u1 y1 + u2 y2 + u1 L[y1 ] + u2 L[y2 ] = f (x) u1 y1 + u2 y2 = f (x). With the constraint, we have a system of linear equations for u1 and u2 . u1 y 1 + u 2 y 2 = 0 u1 y1 + u2 y2 = f (x). y1 y2 y1 y2 u1 u2 = 0 f

We solve this system using Kramers rule. (See Appendix S.) u1 = f (x)y2 W (x) 1067 u2 = f (x)y1 W (x)

Here W (x) is the Wronskian. W (x) = y1 y2 y1 y2

We integrate to get u1 and u2 . This gives us a particular solution. yp = y1 f (x)y2 (x) dx + y2 W (x) f (x)y1 (x) dx. W (x)

Result 21.3.1 Let y1 and y2 be linearly independent homogeneous solutions of L[y] = y + p(x)y + q(x)y = f (x). A particular solution is yp = y1 (x) f (x)y2 (x) dx + y2 (x) W (x) f (x)y1 (x) dx, W (x)

where W (x) is the Wronskian of y1 and y2 .


Example 21.3.1 Consider the equation, y + y = cos(2x). The homogeneous solutions are y1 = cos x and y2 = sin x. We compute the Wronskian. W (x) = cos x sin x = cos2 x + sin2 x = 1 sin x cos x 1068

We use variation of parameters to nd a particular solution. yp = cos(x) cos(2x) sin(x) dx + sin(x) cos(2x) cos(x) dx

1 1 = cos(x) sin(3x) sin(x) dx + sin(x) cos(3x) + cos(x) dx 2 2 1 1 1 1 = cos(x) cos(3x) + cos(x) + sin(x) sin(3x) + sin(x) 2 3 2 3 1 1 = sin2 (x) cos2 (x) + cos(3x) cos(x) + sin(3x) sin(x) 2 6 1 1 = cos(2x) + cos(2x) 2 6 1 = cos(2x) 3 The general solution of the inhomogeneous equation is 1 y = cos(2x) + c1 cos(x) + c2 sin(x). 3

21.3.2

Higher Order Dierential Equations


L[y] = y(n) + pn1 (x)y (n1) + + p1 (x)y + p0 (x)y = f (x),

Consider the nth order inhomogeneous equation, on a < x < b.

We assume that the coecient functions in the dierential equation are continuous on [a . . . b]. Let {y1 , . . . , yn } be a set of linearly independent solutions to the homogeneous equation. Since the Wronskian, W (x) = exp pn1 (x) dx ,

1069

is non-vanishing, we know that these solutions exist. We seek a particular solution of the form y p = u1 y 1 + u 2 y 2 + + un y n . Since {u1 , . . . , un } are undetermined functions of x, we are free to impose n1 constraints. We choose these constraints to simplify the algebra. u1 y 1 u1 y 1 . . . +u2 y2 +u2 y2 . + . . + +un yn + +un yn . . + . + . . . =0 =0 =0

(n2) (n2) (n2) u1 y 1 +u2 y2 + +un yn =0

We dierentiate the expression for yp , utilizing our constraints. yp =u1 y1 +u2 y2 + +un yn yp =u1 y1 +u2 y2 + +un yn yp =u1 y1 +u2 y2 + +un yn . . . . . = . . . . + . . + . + . . .
(n) (n) yp =u1 y1 +u2 y2 + +un yn + u1 y1 (n) (n) (n1)

+ u2 y 2

(n1)

(n1) + + un y n

We substitute yp and its derivatives into the inhomogeneous dierential equation and use the fact that the yk are homogeneous solutions.
(n) u1 y 1 + + un y n + u 1 y 1 (n) (n1) (n1) + + un y n + pn1 (u1 y1 (n1) (n1) + + un y n ) + + p0 (u1 y1 + un yn ) = f (n1) (n1) + + un y n =f

u1 L[y1 ] + u2 L[y2 ] + + un L[yn ] + u1 y1 u1 y 1


(n1)

(n1)

+ u2 y 2

+ u2 y 2

(n1)

(n1) + + un y n = f.

1070

With the constraints, we have a system of linear y1 y2 y y2 1 . . . . . . (n1) (n1) y1 y2

equations for {u1 , . . . , un }. yn u1 0 y n u2 . . . . = .. .. . . 0 . . . (n1) un f yn

We solve this system using Kramers rule. (See Appendix S.) uk = (1)n+k+1 Here W is the Wronskian. We integrating to obtain the uk s. uk = (1)n+k+1 W [y1 , . . . , yk1 , yk+1 , . . . , yn ](x) f (x) dx, W [y1 , y2 , . . . , yn ](x) for k = 1, . . . , n W [y1 , . . . , yk1 , yk+1 , . . . , yn ] f, W [y1 , y2 , . . . , yn ] for k = 1, . . . , n,

Result 21.3.2 Let {y1 , . . . , yn } be linearly independent homogeneous solutions of L[y] = y(n) + pn1 (x)y (n1) + + p1 (x)y + p0 (x)y = f (x), A particular solution is y p = u 1 y 1 + u2 y 2 + + u n y n . where uk = (1)n+k+1 W [y1 , . . . , yk1 , yk+1 , . . . , yn ](x) f (x) dx, for k = 1, . . . , n, W [y1 , y2 , . . . , yn ](x) on a < x < b.

and W [y1 , y2 , . . . , yn ](x) is the Wronskian of {y1 (x), . . . , yn (x)}.

1071

21.4

Piecewise Continuous Coecients and Inhomogeneities


y y = e|x| , y() = 0, > 0, = 1.

Example 21.4.1 Consider the problem

The homogeneous solutions of the dierential equation are ex and ex . We use variation of parameters to nd a particular solution for x > 0. y p = ex
x e e e e d + ex d 2 2 x x 1 1 e(+1) d ex e(1) d = ex 2 2 1 1 ex + ex = 2( + 1) 2( 1) x e = 2 , for x > 0 1 x

A particular solution for x < 0 is yp = Thus a particular solution is

ex , 2 1 yp =

for x < 0.

e|x| . 2 1

The general solution is 1 e|x| +c1 ex +c2 ex . 1 Applying the boundary conditions, we see that c1 = c2 = 0. Apparently the solution is y= 2 y= e|x| . 2 1

1072

-4 0.3

-2 -0.05

0.25 -0.1 0.2 -0.15 0.15 -0.2 0.1 -0.25 0.05 -0.3 -4 -2 2 4

Figure 21.1: The Incorrect and Correct Solution to the Dierential Equation. This function is plotted in Figure 21.1. This function satises the dierential equation for positive and negative x. It also satises the boundary conditions. However, this is NOT a solution to the dierential equation. Since the dierential equation has no singular points and the inhomogeneous term is continuous, the solution must be twice continuously dierentiable. Since the derivative of e|x| /(2 1) has a jump discontinuity at x = 0, the second derivative does not exist. Thus this function could not possibly be a solution to the dierential equation. In the next example we examine the right way to solve this problem. Example 21.4.2 Again consider y y = e|x| , y() = 0, 1073 > 0, = 1.

Separating this into two problems for positive and negative x, y y = ex , y () = 0, on < x 0, x y+ y+ = e , y+ () = 0, on 0 x < . In order for the solution over the whole domain to be twice dierentiable, the solution and its rst derivative must be continuous. Thus we impose the additional boundary conditions y (0) = y+ (0), y (0) = y+ (0). ex + c+ ex . 2 1

The solutions that satisfy the two dierential equations and the boundary conditions at innity are y = ex + c ex , 2 1 y+ =

The two additional boundary conditions give us the equations y (0) = y+ (0) y (0) = y+ (0) c = c+ + c = 2 c+ . 21 1 1

We solve these two equations to determine c and c+ . c = c+ = Thus the solution over the whole domain is y=
ex ex 2 1 ex ex 2 1

for x < 0, for x > 0

y=

e|x| e|x| . 2 1

This function is plotted in Figure 21.1. You can verify that this solution is twice continuously dierentiable.

1074

21.5
21.5.1

Inhomogeneous Boundary Conditions


Eliminating Inhomogeneous Boundary Conditions
L[y] = f (x), for a < x < b,

Consider the nth order equation subject to the linear inhomogeneous boundary conditions Bj [y] = j , where the boundary conditions are of the form B[y] 0 y(a) + 1 y (a) + + yn1 y (n1) (a) + 0 y(b) + 1 y (b) + + n1 y (n1) Let g(x) be an n-times continuously dierentiable function that satises the boundary conditions. Substituting y = u+g into the dierential equation and boundary conditions yields L[u] = f (x) L[g], Bj [u] = bj Bj [g] = 0 for j = 1, . . . , n. for j = 1, . . . , n,

Note that the problem for u has homogeneous boundary conditions. Thus a problem with inhomogeneous boundary conditions can be reduced to one with homogeneous boundary conditions. This technique is of limited usefulness for ordinary dierential equations but is important for solving some partial dierential equation problems. Example 21.5.1 Consider the problem y + y = cos 2x, g(x) =
x

y(0) = 1,

y() = 2.

+ 1 satises the boundary conditions. Substituting y = u + g yields u + u = cos 2x x 1, y(0) = y() = 0.

1075

Example 21.5.2 Consider y + y = cos 2x, y (0) = y() = 1. g(x) = sin x cos x satises the inhomogeneous boundary conditions. Substituting y = u + sin x cos x yields u + u = cos 2x, u (0) = u() = 0.

Note that since g(x) satises the homogeneous equation, the inhomogeneous term in the equation for u is the same as that in the equation for y. Example 21.5.3 Consider y + y = cos 2x, g(x) = cos x
1 3

2 y(0) = , 3

4 y() = . 3
1 3

satises the boundary conditions. Substituting y = u + cos x 1 u + u = cos 2x + , 3 u(0) = u() = 0.

yields

Result 21.5.1 The nth order dierential equation with boundary conditions L[y] = f (x), Bj [y] = bj , for j = 1, . . . , n

has the solution y = u + g where u satises L[u] = f (x) L[g], Bj [u] = 0, for j = 1, . . . , n

and g is any n-times continuously dierentiable function that satises the inhomogeneous boundary conditions.

1076

21.5.2

Separating Inhomogeneous Equations and Inhomogeneous Boundary Conditions

Now consider a problem with inhomogeneous boundary conditions L[y] = f (x), B1 [y] = 1 , B2 [y] = 2 .

In order to solve this problem, we solve the two problems L[u] = f (x), L[v] = 0, B1 [u] = B2 [u] = 0, B1 [v] = 1 , and

B2 [v] = 2 .

The solution for the problem with an inhomogeneous equation and inhomogeneous boundary conditions will be the sum of u and v. To verify this, L[u + v] = L[u] + L[v] = f (x) + 0 = f (x), Bi [u + v] = Bi [u] + Bi [v] = 0 + i = i . This will be a useful technique when we develop Green functions.

Result 21.5.2 The solution to L[y] = f (x), is y = u + v where L[u] = f (x), B1 [u] = 0, B2 [u] = 0, and L[v] = 0, B1 [v] = 1 , B2 [v] = 2 . B1 [y] = 1 , B2 [y] = 2 ,

1077

21.5.3

Existence of Solutions of Problems with Inhomogeneous Boundary Conditions

Consider the nth order homogeneous dierential equation L[y] = y (n) + pn1 y (n1) + + p1 y + p0 y = f (x), subject to the n inhomogeneous boundary conditions Bj [y] = j , where each boundary condition is of the form B[y] 0 y(a) + 1 y (a) + + n1 y (n1) (a) + 0 y(b) + 1 y (b) + + n1 y (n1) (b). We assume that the coecients in the dierential equation are continuous on [a, b]. Since the Wronskian of the solutions of the dierential equation, W (x) = exp pn1 (x) dx , for j = 1, . . . , n for a < x < b,

is non-vanishing on [a, b], there are n linearly independent solution on that range. Let {y1 , . . . , yn } be a set of linearly independent solutions of the homogeneous equation. From Result 21.3.2 we know that a particular solution yp exists. The general solution of the dierential equation is y = yp + c1 y1 + c2 y2 + + cn yn . The n boundary conditions impose the B1 [y1 ] B2 [y1 ] . . . Bn [y1 ] matrix equation, B1 [y2 ] B1 [yn ] B2 [y2 ] B2 [yn ] . . ... . . . . Bn [y2 ] Bn [yn ] 1078 c1 1 B1 [yp ] c2 2 B2 [yp ] .= . . . . . cn n Bn [yp ]

This equation has a unique solution if and only if the equation B1 [y1 ] B1 [y2 ] B2 [y1 ] B2 [y2 ] . . . . . . Bn [y1 ] Bn [y2 ] .. . B1 [yn ] B2 [yn ] . . . c1 0 c2 0 . = . . . . . cn 0

Bn [yn ]

has only the trivial solution. (This is the case if and only if the determinant of the matrix is nonzero.) Thus the problem L[y] = y (n) + pn1 y (n1) + + p1 y + p0 y = f (x), subject to the n inhomogeneous boundary conditions Bj [y] = j , has a unique solution if and only if the problem L[y] = y (n) + pn1 y (n1) + + p1 y + p0 y = 0, subject to the n homogeneous boundary conditions Bj [y] = 0, has only the trivial solution. 1079 for j = 1, . . . , n, for a < x < b, for j = 1, . . . , n, for a < x < b,

Result 21.5.3 The problem L[y] = y (n) + pn1 y (n1) + + p1 y + p0 y = f (x), subject to the n inhomogeneous boundary conditions Bj [y] = j , for j = 1, . . . , n, for a < x < b,

has a unique solution if and only if the problem L[y] = y (n) + pn1 y (n1) + + p1 y + p0 y = 0, subject to Bj [y] = 0, has only the trivial solution. for j = 1, . . . , n, for a < x < b,

21.6

Green Functions for First Order Equations


L[y] y + p(x)y = f (x), for x > a, (21.2)

Consider the rst order inhomogeneous equation

subject to a homogeneous initial condition, B[y] y(a) = 0. The Green function G(x|) is dened as the solution to L[G(x|)] = (x ) subject to G(a|) = 0. We can represent the solution to the inhomogeneous problem in Equation 21.2 as an integral involving the Green 1080

function. To show that y(x) =


a

G(x|)f () d

is the solution, we apply the linear operator L to the integral. (Assume that the integral is uniformly convergent.)

L
a

G(x|)f () d =
a

L[G(x|)]f () d (x )f () d
a

= = f (x) The integral also satises the initial condition.


B
a

G(x|)f () d =
a

B[G(x|)]f () d (0)f () d
a

= =0

Now we consider the qualitiative behavior of the Green function. For x = , the Green function is simply a homogeneous solution of the dierential equation, however at x = we expect some singular behavior. G (x|) will have a Dirac delta function type singularity. This means that G(x|) will have a jump discontinuity at x = . We integrate the dierential equation on the vanishing interval ( . . . + ) to determine this jump. G + p(x)G = (x )
+

G( |) G( |) +

p(x)G(x|) dx = 1 (21.3)

G( + |) G( |) = 1 1081

The homogeneous solution of the dierential equation is y h = e


p(x) dx

Since the Green function satises the homogeneous equation for x = , it will be a constant times this homogeneous solution for x < and x > . G(x|) = c 1 e c 2 e
p(x) dx p(x) dx

a<x< <x

In order to satisfy the homogeneous initial condition G(a|) = 0, the Green function must vanish on the interval (a . . . ). G(x|) = 0 c e
p(x) dx

a<x< <x

The jump condition, (Equation 21.3), gives us the constraint G( + |) = 1. This determines the constant in the homogeneous solution for x > . G(x|) = 0 e
x

p(t) dt

a<x< <x

We can use the Heaviside function to write the Green function without using a case statement. G(x|) = e
x

p(t) dt

H(x )

Clearly the Green function is of little value in solving the inhomogeneous dierential equation in Equation 21.2, as we can solve that problem directly. However, we will encounter rst order Green function problems in solving some partial dierential equations. 1082

Result 21.6.1 The rst order inhomogeneous dierential equation with homogeneous initial condition L[y] y + p(x)y = f (x), for a < x, y(a) = 0, has the solution y=
a

G(x|)f () d,

where G(x|) satises the equation L[G(x|)] = (x ), The Green function is G(x|) = e for a < x,
x

G(a|) = 0.

p(t) dt

H(x )

21.7

Green Functions for Second Order Equations

Consider the second order inhomogeneous equation L[y] = y + p(x)y + q(x)y = f (x), subject to the homogeneous boundary conditions B1 [y] = B2 [y] = 0. The Green function G(x|) is dened as the solution to L[G(x|)] = (x ) subject to B1 [G] = B2 [G] = 0. 1083 for a < x < b, (21.4)

The Green function is useful because you can represent the solution to the inhomogeneous problem in Equation 21.4 as an integral involving the Green function. To show that
b

y(x) =
a

G(x|)f () d

is the solution, we apply the linear operator L to the integral. (Assume that the integral is uniformly convergent.)
b b

L
a

G(x|)f () d =
a b

L[G(x|)]f () d (x )f () d
a

= f (x) The integral also satises the boundary conditions.


b b

Bi
a

G(x|)f () d =
a b

Bi [G(x|)]f () d [0]f () d
a

= =0

One of the advantages of using Green functions is that once you nd the Green function for a linear operator and certain homogeneous boundary conditions, L[G] = (x ), you can write the solution for any inhomogeneity, f (x). L[f ] = f (x), B1 [y] = B2 [y] = 0 1084 B1 [G] = B2 [G] = 0,

You do not need to do any extra work to obtain the solution for a dierent inhomogeneous term. Qualitatively, what kind of behavior will the Green function for a second order dierential equation have? Will it have a delta function singularity; will it be continuous? To answer these questions we will rst look at the behavior of integrals and derivatives of (x). The integral of (x) is the Heaviside function, H(x).
x

H(x) =

(t) dt =

0 1

for x < 0 for x > 0

The integral of the Heaviside function is the ramp function, r(x).


x

r(x) =

H(t) dt =

0 x

for x < 0 for x > 0

The derivative of the delta function is zero for x = 0. At x = 0 it goes from 0 up to +, down to and then back up to 0. In Figure 21.2 we see conceptually the behavior of the ramp function, the Heaviside function, the delta function, and the derivative of the delta function. We write the dierential equation for the Green function. G (x|) + p(x)G (x|) + q(x)G(x|) = (x ) we see that only the G (x|) term can have a delta function type singularity. If one of the other terms had a delta function type singularity then G (x|) would be more singular than a delta function and there would be nothing in the right hand side of the equation to match this kind of singularity. Analogous to the progression from a delta function to a Heaviside function to a ramp function, we see that G (x|) will have a jump discontinuity and G(x|) will be continuous. Let y1 and y2 be two linearly independent solutions to the homogeneous equation, L[y] = 0. Since the Green function satises the homogeneous equation for x = , it will be a linear combination of the homogeneous solutions. G(x|) = c1 y1 + c2 y2 d1 y1 + d2 y2 1085 for x < for x >

Figure 21.2: r(x), H(x), (x) and

d (x) dx

We require that G(x|) be continuous. G(x|) = G(x|)

x +

We can write this in terms of the homogeneous solutions.

c1 y1 () + c2 y2 () = d1 y1 () + d2 y2 () We integrate L[G(x|)] = (x ) from to +.


+ +

[G (x|) + p(x)G (x|) + q(x)G(x|)] dx =


(x ) dx.

1086

Since G(x|) is continuous and G (x|) has only a jump discontinuity two of the terms vanish.
+ +

p(x)G (x|) dx = 0
+

and
+

q(x)G(x|) dx = 0 (x ) dx
+

G (x|) dx =

G (x|)

= H(x )

G ( + |) G ( |) = 1 We write this jump condition in terms of the homogeneous solutions. d1 y1 () + d2 y2 () c1 y1 () c2 y2 () = 1 Combined with the two boundary conditions, this gives us a total of four equations to determine our four constants, c1 , c2 , d1 , and d2 .

Result 21.7.1 The second order inhomogeneous dierential equation with homogeneous boundary conditions L[y] = y + p(x)y + q(x)y = f (x), has the solution y=
a b

for a < x < b,

B1 [y] = B2 [y] = 0,

G(x|)f () d,

where G(x|) satises the equation L[G(x|)] = (x ), for a < x < b, B1 [G(x|)] = B2 [G(x|)] = 0.

G(x|) is continuous and G (x|) has a jump discontinuity of height 1 at x = .


1087

Example 21.7.1 Solve the boundary value problem y = f (x), y(0) = y(1) = 0,

using a Green function. A pair of solutions to the homogeneous equation are y1 = 1 and y2 = x. First note that only the trivial solution to the homogeneous equation satises the homogeneous boundary conditions. Thus there is a unique solution to this problem. The Green function satises G (x|) = (x ), The Green function has the form G(x|) = c1 + c2 x d1 + d2 x for x < for x > . G(0|) = G(1|) = 0.

Applying the two boundary conditions, we see that c1 = 0 and d1 = d2 . The Green function now has the form G(x|) = Since the Green function must be continuous, c = d( 1) From the jump condition, d d c (x 1) cx dx 1 dx x= c c=1 1 c = 1. 1088 =1
x=

cx d(x 1)

for x < for x > .

d=c

. 1

0.1 -0.1 -0.2 -0.3 0.5 1

0.1 -0.1 -0.2 -0.3 0.5 1

0.1 -0.1 -0.2 -0.3 0.5 1

0.1 -0.1 -0.2 -0.3 0.5 1

Figure 21.3: Plot of G(x|0.05),G(x|0.25),G(x|0.5) and G(x|0.75). Thus the Green function is G(x|) = ( 1)x (x 1) for x < for x > .

The Green function is plotted in Figure 21.3 for various values of . The solution to y = f (x) is
1

y(x) =
0 x

G(x|)f () d
1

y(x) = (x 1)
0

f () d + x
x

( 1)f () d.

Example 21.7.2 Solve the boundary value problem y = f (x), In Example 21.7.1 we saw that the solution to u = f (x), u(0) = u(1) = 0 1089 y(0) = 1, y(1) = 2.

is u(x) = (x 1)
0

f () d + x
x

( 1)f () d.

Now we have to nd the solution to v = 0, The general solution is v = c1 + c2 x. Applying the boundary conditions yields v = 1 + x. Thus the solution for y is
x 1

v(0) = 1,

u(1) = 2.

y = 1 + x + (x 1)
0

f () d + x
x

( 1)f ( xi) d.

Example 21.7.3 Consider y = x, y(0) = y(1) = 0.

Method 1. Integrating the dierential equation twice yields 1 y = x3 + c1 x + c2 . 6 Applying the boundary conditions, we nd that the solution is 1 y = (x3 x). 6 1090

Method 2. Using the Green function to nd the solution,


x 1

y = (x 1)
0

2 d + x
x

( 1) d

1 = (x 1) x3 + x 3

1 1 1 3 1 2 x + x 3 2 3 2

1 y = (x3 x). 6 Example 21.7.4 Find the solution to the dierential equation y y = sin x, that is bounded for all x. The Green function for this problem satises G (x|) G(x|) = (x ). The homogeneous solutions are y1 = ex , and y2 = ex . The Green function has the form G(x|) = c1 ex +c2 ex d1 ex +d2 ex for x < for x > .

Since the solution must be bounded for all x, the Green function must also be bounded. Thus c2 = d1 = 0. The Green function now has the form c ex for x < G(x|) = d ex for x > . Requiring that G(x|) be continuous gives us the condition c e = d e 1091 d = c e2 .

G(x|) has a jump discontinuity of height 1 at x = . d 2 x ce e dx


x= 2

d x ce dx

=1
x=

c e e

c e = 1 1 c = e 2

The Green function is then G(x|) =


1 2 ex 1 ex+ 2

for x < for x >

1 G(x|) = e|x| . 2 A plot of G(x|0) is given in Figure 21.4. The solution to y y = sin x is y(x) = 1 e|x| sin d 2 x 1 ex d + = sin sin ex+ d 2 x 1 sin x + cos x sin x + cos x + ) = ( 2 2 2 y= 1 sin x. 2

1092

0.6 0.4 0.2 -4 -2 -0.2 -0.4 -0.6 2 4

Figure 21.4: Plot of G(x|0).

21.7.1

Green Functions for Sturm-Liouville Problems

Consider the problem L[y] = (p(x)y ) + q(x)y = f (x), subject to B1 [y] = 1 y(a) + 2 y (a) = 0, B2 [y] = 1 y(b) + 2 y (b) = 0. This is known as a Sturm-Liouville problem. Equations of this type often occur when solving partial dierential equations. The Green function associated with this problem satises L[G(x|)] = (x ), B1 [G(x|)] = B2 [G(x|)] = 0. 1093

Let y1 and y2 be two non-trivial homogeneous solutions that satisfy the left and right boundary conditions, respectively. L[y1 ] = 0, B1 [y1 ] = 0, L[y2 ] = 0, B2 [y2 ] = 0.

The Green function satises the homogeneous equation for x = and satises the homogeneous boundary conditions. Thus it must have the following form. G(x|) = c1 ()y1 (x) c2 ()y2 (x) for a x , for x b,

Here c1 and c2 are unknown functions of . The rst constraint on c1 and c2 comes from the continuity condition. G( |) = G( + |) c1 ()y1 () = c2 ()y2 () We write the inhomogeneous equation in the standard form. G (x|) + p q (x ) G (x|) + G(x|) = p p p

The second constraint on c1 and c2 comes from the jump condition. 1 p() 1 c2 ()y2 () c1 ()y1 () = p() G ( + |) G ( |) = Now we have a system of equations to determine c1 and c2 . c1 ()y1 () c2 ()y2 () = 0 c1 ()y1 () c2 ()y2 () = 1 p()

1094

We solve this system with Kramers rule. c1 () = y2 () , p()(W ()) c2 () = y1 () p()(W ())

Here W (x) is the Wronskian of y1 (x) and y2 (x). The Green function is G(x|) = The solution of the Sturm-Liouville problem is
b y1 (x)y2 () p()W () y2 (x)y1 () p()W ()

for a x , for x b.

y=
a

G(x|)f () d.

Result 21.7.2 The problem L[y] = (p(x)y ) + q(x)y = f (x), subject to B1 [y] = 1 y(a) + 2 y (a) = 0, B2 [y] = 1 y(b) + 2 y (b) = 0. has the Green function G(x|) =
y1 (x)y2 () p()W () y2 (x)y1 () p()W ()

for a x , for x b,

where y1 and y2 are non-trivial homogeneous solutions that satisfy B1 [y1 ] = B2 [y2 ] = 0, and W (x) is the Wronskian of y1 and y2 .

1095

Example 21.7.5 Consider the equation y y = f (x), y(0) = y(1) = 0.

A set of solutions to the homogeneous equation is {ex , ex }. Equivalently, one could use the set {cosh x, sinh x}. Note that sinh x satises the left boundary condition and sinh(x 1) satises the right boundary condition. The Wronskian of these two homogeneous solutions is W (x) = sinh x sinh(x 1) cosh x cosh(x 1)

= sinh x cosh(x 1) cosh x sinh(x 1) 1 1 = [sinh(2x 1) + sinh(1)] [sinh(2x 1) sinh(1)] 2 2 = sinh(1). The Green function for the problem is then G(x|) = The solution to the problem is y= sinh(x 1) sinh(1)
x sinh x sinh(1) sinh(1) sinh(x1) sinh sinh(1)

for 0 x for x 1.

sinh()f () d +
0

sinh(x) sinh(1)

sinh( 1)f () d.
x

21.7.2
Consider

Initial Value Problems


L[y] = y + p(x)y + q(x)y = f (x), 1096 for a < x < b,

subject the the initial conditions y(a) = 1 , The solution is y = u + v where u + p(x)u + q(x)u = f (x), and v + p(x)v + q(x)v = 0, Since the Wronskian W (x) = c exp p(x) dx v(a) = 1 , v (a) = 2 . u(a) = 0, u (a) = 0, y (a) = 2 .

is non-vanishing, the solutions of the dierential equation for v are linearly independent. Thus there is a unique solution for v that satises the initial conditions. The Green function for u satises G (x|) + p(x)G (x|) + q(x)G(x|) = (x ), The continuity and jump conditions are G( |) = G( + |), G ( |) + 1 = G ( + |). G(a|) = 0, G (a|) = 0.

Let u1 and u2 be two linearly independent solutions of the dierential equation. For x < , G(x|) is a linear combination of these solutions. Since the Wronskian is non-vanishing, only the trivial solution satises the homogeneous initial conditions. The Green function must be G(x|) = 0 u (x) for x < for x > ,

where u (x) is the linear combination of u1 and u2 that satises u () = 0, 1097 u () = 1.

Note that the non-vanishing Wronskian ensures a unique solution for u . We can write the Green function in the form G(x|) = H(x )u (x). This is known as the causal solution. The solution for u is
b

u=
a b

G(x|)f () d H(x )u (x)f () d


a x

= =
a

u (x)f () d

Now we have the solution for y,


x

y=v+
a

u (x)f () d.

Result 21.7.3 The solution of the problem y + p(x)y + q(x)y = f (x), is y = yh +


a x

y(a) = 1 ,

y (a) = 2 ,

y (x)f () d

where yh is the combination of the homogeneous solutions of the equation that satisfy the initial conditions and y (x) is the linear combination of homogeneous solutions that satisfy y () = 0, y () = 1.

1098

21.7.3
Consider

Problems with Unmixed Boundary Conditions


L[y] = y + p(x)y + q(x)y = f (x), for a < x < b,

subject the the unmixed boundary conditions 1 y(a) + 2 y (a) = 1 , The solution is y = u + v where u + p(x)u + q(x)u = f (x), and v + p(x)v + q(x)v = 0, 1 v(a) + 2 v (a) = 1 , 1 v(b) + 2 v (b) = 2 . The problem for v may have no solution, a unique solution or an innite number of solutions. We consider only the case that there is a unique solution for v. In this case the homogeneous equation subject to homogeneous boundary conditions has only the trivial solution. The Green function for u satises G (x|) + p(x)G (x|) + q(x)G(x|) = (x ), 1 G(a|) + 2 G (a|) = 0, The continuity and jump conditions are G( |) = G( + |), G ( |) + 1 = G ( + |). 1 G(b|) + 2 G (b|) = 0. 1 u(a) + 2 u (a) = 0, 1 u(b) + 2 u (b) = 0, 1 y(b) + 2 y (b) = 2 .

Let u1 and u2 be two solutions of the homogeneous equation that satisfy the left and right boundary conditions, respectively. The non-vanishing of the Wronskian ensures that these solutions exist. Let W (x) denote the Wronskian 1099

of u1 and u2 . Since the homogeneous equation with homogeneous boundary conditions has only the trivial solution, W (x) is nonzero on [a, b]. The Green function has the form c 1 u1 c 2 u2 for x < , for x > .

G(x|) =

The continuity and jump conditions for Green function gives us the equations c1 u1 () c2 u2 () = 0 c1 u1 () c2 u2 () = 1. Using Kramers rule, the solution is c1 = Thus the Green function is G(x|) = The solution for u is
b u1 (x)u2 () W () u1 ()u2 (x) W ()

u2 () , W ()

c2 =

u1 () . W ()

for x < , for x > .

u=
a

G(x|)f () d.

Thus if there is a unique solution for v, the solution for y is


b

y=v+
a

G(x|)f () d.

1100

Result 21.7.4 Consider the problem y + p(x)y + q(x)y = f (x), 1 y(a) + 2 y (a) = 1 , 1 y(b) + 2 y (b) = 2 .

If the homogeneous dierential equation subject to the inhomogeneous boundary conditions has the unique solution yh , then the problem has the unique solution
b

y = yh +
a

G(x|)f () d

where G(x|) =

u1 (x)u2 () W () u1 ()u2 (x) W ()

for x < , for x > ,

u1 and u2 are solutions of the homogeneous dierential equation that satisfy the left and right boundary conditions, respectively, and W (x) is the Wronskian of u1 and u2 . 21.7.4
Consider L[y] = y + p(x)y + q(x)y = f (x), subject the the mixed boundary conditions B1 [y] = 11 y(a) + 12 y (a) + 11 y(b) + 12 y (b) = 1 , B2 [y] = 21 y(a) + 22 y (a) + 21 y(b) + 22 y (b) = 2 . 1101 for a < x < b,

Problems with Mixed Boundary Conditions

The solution is y = u + v where u + p(x)u + q(x)u = f (x), and v + p(x)v + q(x)v = 0, B1 [v] = 1 , B2 [v] = 2 . B1 [u] = 0, B2 [u] = 0,

The problem for v may have no solution, a unique solution or an innite number of solutions. Again we consider only the case that there is a unique solution for v. In this case the homogeneous equation subject to homogeneous boundary conditions has only the trivial solution. Let y1 and y2 be two solutions of the homogeneous equation that satisfy the boundary conditions B1 [y1 ] = 0 and B2 [y2 ] = 0. Since the completely homogeneous problem has no solutions, we know that B1 [y2 ] and B2 [y1 ] are nonzero. The solution for v has the form v = c1 y1 + c2 y2 . Applying the two boundary conditions yields v= The Green function for u satises G (x|) + p(x)G (x|) + q(x)G(x|) = (x ), The continuity and jump conditions are G( |) = G( + |), G ( |) + 1 = G ( + |). B1 [G] = 0, B2 [G] = 0. 2 1 y1 + y2 . B2 [y1 ] B1 [y2 ]

We write the Green function as the sum of the causal solution and the two homogeneous solutions G(x|) = H(x )y (x) + c1 y1 (x) + c2 y2 (x) 1102

With this form, the continuity and jump conditions are automatically satised. Applying the boundary conditions yields

B1 [G] = B1 [H(x )y ] + c2 B1 [y2 ] = 0, B2 [G] = B2 [H(x )y ] + c1 B2 [y1 ] = 0,

B1 [G] = 11 y (b) + 12 y (b) + c2 B1 [y2 ] = 0, B2 [G] = 21 y (b) + 22 y (b) + c1 B2 [y1 ] = 0,

G(x|) = H(x )y (x)

21 y (b) + 22 y (b) 11 y (b) + 12 y (b) y1 (x) y2 (x). B2 [y1 ] B1 [y2 ]

Note that the Green function is well dened since B2 [y1 ] and B1 [y2 ] are nonzero. The solution for u is
b

u=
a

G(x|)f () d.

Thus if there is a unique solution for v, the solution for y is


b

y=
a

G(x|)f () d +

2 1 y1 + y2 . B2 [y1 ] B1 [y2 ]

1103

Result 21.7.5 Consider the problem y + p(x)y + q(x)y = f (x), B1 [y] = 11 y(a) + 12 y (a) + 11 y(b) + 12 y (b) = 1 , B2 [y] = 21 y(a) + 22 y (a) + 21 y(b) + 22 y (b) = 2 . If the homogeneous dierential equation subject to the homogeneous boundary conditions has no solution, then the problem has the unique solution
b

y=
a

G(x|)f () d +

2 1 y1 + y2 , B2 [y1 ] B1 [y2 ]

where G(x|) = H(x )y (x) 21 y (b) + 22 y (b) y1 (x) B2 [y1 ] 11 y (b) + 12 y (b) y2 (x), B1 [y2 ]

y1 and y2 are solutions of the homogeneous dierential equation that satisfy the rst and second boundary conditions, respectively, and y (x) is the solution of the homogeneous equation that satises y () = 0, y () = 1.

1104

21.8

Green Functions for Higher Order Problems

Consider the nth order dierential equation L[y] = y (n) + pn1 (x)y (n1) + + p1 (x)y + p0 y = f (x) on a < x < b, subject to the n independent boundary conditions Bj [y] = j where the boundary conditions are of the form
n1 n1

B[y]
k=0

k y (k) (a) +
k=0

k y (k) (b).

We assume that the coecient functions in the dierential equation are continuous on [a, b]. The solution is y = u + v where u and v satisfy L[u] = f (x), and L[v] = 0, with Bj [v] = j with Bj [u] = 0,

From Result 21.5.3, we know that if the completely homogeneous problem L[w] = 0, with Bj [w] = 0,

has only the trivial solution, then the solution for y exists and is unique. We will construct this solution using Green functions. 1105

First we consider the problem for v. Let {y1 , . . . , yn } be a set of linearly independent solutions. The solution for v has the form v = c1 y1 + + cn yn where the constants are determined by the matrix equation B1 [y1 ] B1 [y2 ] B1 [yn ] B2 [y1 ] B2 [y2 ] B2 [yn ] . . . .. . . . . . . . Bn [y1 ] Bn [y2 ] Bn [yn ]

c1 1 c2 2 . = . . . . . . cn n

To solve the problem for u we consider the Green function satisfying L[G(x|)] = (x ), with Bj [G] = 0.

Let y (x) be the linear combination of the homogeneous solutions that satisfy the conditions y () = 0 y () = 0 . . =. . . . y
(n2)

() = 0 = 1.

(n1) y ()

The causal solution is then yc (x) = H(x )y (x). The Green function has the form G(x|) = H(x )y (x) + d1 y1 (x) + + dn yn (x) 1106

The constants are determined by B1 [y1 ] B2 [y1 ] . . . Bn [y1 ] The solution for u then is

the matrix equation B1 [y2 ] B1 [yn ] B2 [y2 ] B2 [yn ] . . .. . . . . . Bn [y2 ] Bn [yn ]


b

d1 B1 [H(x )y (x)] d2 B2 [H(x )y (x)] .= . . . . . . dn Bn [H(x )y (x)]

u=
a

G(x|)f () d.

Result 21.8.1 Consider the nth order dierential equation L[y] = y (n) + pn1 (x)y (n1) + + p1 (x)y + p0 y = f (x) on a < x < b, subject to the n independent boundary conditions Bj [y] = j If the homogeneous dierential equation subject to the homogeneous boundary conditions has only the trivial solution, then the problem has the unique solution
b

y=
a

G(x|)f () d + c1 y1 + cn yn

where G(x|) = H(x )y (x) + d1 y1 (x) + + dn yn (x), {y1 , . . . , yn } is a set of solutions of the homogeneous dierential equation, and the constants cj and dj can be determined by solving sets of linear equations.
1107

Example 21.8.1 Consider the problem y y + y y = f (x), y(0) = 1, y (0) = 2, y(1) = 3. The completely homogeneous associated problem is w w + w w = 0, The solution of the dierential equation is w = c1 cos x + c2 sin x + c2 ex . The boundary conditions give us the equation 1 0 1 c1 0 0 c2 = 0 . 1 1 cos 1 sin 1 e c3 0 The determinant of the matrix is e cos 1 sin 1 = 0. Thus the homogeneous problem has only the trivial solution and the inhomogeneous problem has a unique solution. We separate the inhomogeneous problem into the two problems u u + u u = f (x), v v + v v = 0, u(0) = u (0) = u(1) = 0, v (0) = 2, v(1) = 3, w(0) = w (0) = w(1) = 0.

v(0) = 1,

First we solve the problem for v. The solution of the dierential equation is v = c1 cos x + c2 sin x + c2 ex . The boundary conditions yields the equation

1 0 1 c1 1 0 c2 = 2 . 1 1 cos 1 sin 1 e c3 3 1108

The solution for v is v= 1 (e + sin 1 3) cos x + (2e cos 1 3) sin x + (3 cos 1 2 sin 1) ex . e cos 1 sin 1

Now we nd the Green function for the problem in u. The causal solution is H(x )u (x) = H(x ) 1 (sin cos ) cos x (sin + cos ) sin + e ex , 2

1 H(x )u (x) = H(x ) ex cos(x ) sin(x ) . 2 The Green function has the form G(x|) = H(x )u (x) + c1 cos x + c2 sin x + c3 ex . The constants are determined by the three conditions c1 cos x + c2 sin x + c3 ex x=0 = 0, (c1 cos x + c2 sin x + c3 ex ) = 0, x x=0 u (x) + c1 cos x + c2 sin x + c3 ex x=1 = 0. The Green function is 1 cos(1 ) + sin(1 ) e1 G(x|) = H(x ) ex cos(x ) sin(x ) + cos x + sin x ex 2 2(cos 1 + sin 1 e) The solution for v is v=
0 1

G(x|)f () d.

Thus the solution for y is 1109

y=
0

G(x|)f () d +

1 (e + sin 1 3) cos x e cos 1 sin 1 + (2e cos 1 3) sin x + (3 cos 1 2 sin 1) ex .

21.9

Fredholm Alternative Theorem

Orthogonality. Two real vectors, u and v are orthogonal if u v = 0. Consider two functions, u(x) and v(x), dened in [a, b]. The dot product in vector space is analogous to the integral
b

u(x)v(x) dx
a

in function space. Thus two real functions are orthogonal if


b

u(x)v(x) dx = 0.
a

Consider the nth order linear inhomogeneous dierential equation L[y] = f (x) on [a, b], subject to the linear inhomogeneous boundary conditions Bj [y] = 0, for j = 1, 2, . . . n.

The Fredholm alternative theorem tells us if the problem has a unique solution, an innite number of solutions, or no solution. Before presenting the theorem, we will consider a few motivating examples. 1110

No Nontrivial Homogeneous Solutions. In the section on Green functions we showed that if the completely homogeneous problem has only the trivial solution then the inhomogeneous problem has a unique solution.

Nontrivial Homogeneous Solutions Exist. If there are nonzero solutions to the homogeneous problem L[y] = 0 that satisfy the homogeneous boundary conditions Bj [y] = 0 then the inhomogeneous problem L[y] = f (x) subject to the same boundary conditions either has no solution or an innite number of solutions. Suppose there is a particular solution yp that satises the boundary conditions. If there is a solution yh to the homogeneous equation that satises the boundary conditions then there will be an innite number of solutions since yp + cyh is also a particular solution.

The question now remains: Given that there are homogeneous solutions that satisfy the boundary conditions, how do we know if a particular solution that satises the boundary conditions exists? Before we address this question we will consider a few examples. Example 21.9.1 Consider the problem y + y = cos x, y(0) = y() = 0.

The two homogeneous solutions of the dierential equation are y1 = cos x, and y2 = sin x.

y2 = sin x satises the boundary conditions. Thus we know that there are either no solutions or an innite number of 1111

solutions. A particular solution is yp = cos x cos2 x cos x sin x dx + sin x dx 1 1 1 1 1 = cos x sin(2x) dx + sin x + cos(2x) 2 2 2 1 1 1 = cos x cos(2x) + sin x x + sin(2x) 4 2 4 1 1 = x sin x + cos x cos(2x) + sin x sin(2x) 2 4 1 1 = x sin x + cos x 2 4 1 y = x sin x + c1 cos x + c2 sin x. 2 Applying the two boundary conditions yields 1 y = x sin x + c sin x. 2 Thus there are an innite number of solutions. Example 21.9.2 Consider the dierential equation y + y = sin x, The general solution is 1 y = x cos x + c1 cos x + c2 sin x. 2 1112 y(0) = y() = 0.

dx

The general solution is

Applying the boundary conditions,

y(0) = 0 c1 = 0 1 y() = 0 cos() + c2 sin() = 0 2 = 0. 2

Since this equation has no solution, there are no solutions to the inhomogeneous problem.

In both of the above examples there is a homogeneous solution y = sin x that satises the boundary conditions. In Example 21.9.1, the inhomogeneous term is cos x and there are an innite number of solutions. In Example 21.9.2, the inhomogeneity is sin x and there are no solutions. In general, if the inhomogeneous term is orthogonal to all the homogeneous solutions that satisfy the boundary conditions then there are an innite number of solutions. If not, there are no inhomogeneous solutions. 1113

Result 21.9.1 Fredholm Alternative Theorem. Consider the nth order inhomogeneous problem L[y] = f (x) on [a, b] subject to Bj [y] = 0 for j = 1, 2, . . . , n, and the associated homogeneous problem L[y] = 0 on [a, b] subject to Bj [y] = 0 for j = 1, 2, . . . , n. If the homogeneous problem has only the trivial solution then the inhomogeneous problem has a unique solution. If the homogeneous problem has m independent solutions, {y1 , y2 , . . . , ym }, then there are two possibilities: If f (x) is orthogonal to each of the homogeneous solutions then there are an innite number of solutions of the form
m

y = yp +
j=1

cj yj .

If f (x) is not orthogonal to each of the homogeneous solutions then there are no inhomogeneous solutions.
Example 21.9.3 Consider the problem y + y = cos 2x, y(0) = 1, y() = 2.

cos x and sin x are two linearly independent solutions to the homogeneous equation. sin x satises the homogeneous boundary conditions. Thus there are either an innite number of solutions, or no solution. 1114

To transform this problem to one with homogeneous boundary conditions, we note that g(x) = the change of variables y = u + g to obtain u + u = cos 2x x 1, y(0) = 0, y() = 0.

+ 1 and make

x Since cos 2x 1 is not orthogonal to sin x, there is no solution to the inhomogeneous problem. To check this, the general solution is

1 y = cos 2x + c1 cos x + c2 sin x. 3 Applying the boundary conditions, y(0) = 1 y() = 2 c1 = 4 3

1 4 = 2. 3 3

Thus we see that the right boundary condition cannot be satised. Example 21.9.4 Consider y + y = cos 2x, y (0) = y() = 1. There are no solutions to the homogeneous equation that satisfy the homogeneous boundary conditions. To check this, note that all solutions of the homogeneous equation have the form uh = c1 cos x + c2 sin x. uh (0) = 0 uh () = 0 c2 = 0 c1 = 0.

From the Fredholm Alternative Theorem we see that the inhomogeneous problem has a unique solution. To nd the solution, start with 1 y = cos 2x + c1 cos x + c2 sin x. 3 1115

y (0) = 1 y() = 1 Thus the solution is

c2 = 1 1 c1 = 1 3

1 4 y = cos 2x cos x + sin x. 3 3 Example 21.9.5 Consider 2 4 y(0) = , y() = . 3 3 cos x and sin x satisfy the homogeneous dierential equation. sin x satises the homogeneous boundary conditions. Since g(x) = cos x 1/3 satises the boundary conditions, the substitution y = u + g yields y + y = cos 2x, 1 u + u = cos 2x + , 3
1 Now we check if sin x is orthogonal to cos 2x + 3 .

y(0) = 0,

y() = 0.

sin x cos 2x +
0

1 3

dx =

1 1 1 sin 3x sin x + sin x dx 2 3 0 2 1 1 = cos 3x + cos x 6 6 0 =0

Since sin x is orthogonal to the inhomogeneity, there are an innite number of solutions to the problem for u, (and hence the problem for y). As a check, then general solution for y is 1 y = cos 2x + c1 cos x + c2 sin x. 3 1116

Applying the boundary conditions, y(0) = 2 3 4 3 c1 = 1 4 4 = . 3 3

y() =

Thus we see that c2 is arbitrary. There are an innite number of solutions of the form 1 y = cos 2x + cos x + c sin x. 3

1117

21.10

Exercises

Undetermined Coecients
Exercise 21.1 (mathematica/ode/inhomogeneous/undetermined.nb) Find the general solution of the following equations. 1. y + 2y + 5y = 3 sin(2t) 2. 2y + 3y + y = t2 + 3 sin(t) Hint, Solution Exercise 21.2 (mathematica/ode/inhomogeneous/undetermined.nb) Find the solution of each one of the following initial value problems. 1. y 2y + y = t et +4, y(0) = 1, y (0) = 1 2. y + 2y + 5y = 4 et cos(2t), y(0) = 1, y (0) = 0 Hint, Solution

Variation of Parameters
Exercise 21.3 (mathematica/ode/inhomogeneous/variation.nb) Use the method of variation of parameters to nd a particular solution of the given dierential equation. 1. y 5y + 6y = 2 et 2. y + y = tan(t), 0 < t < /2 3. y 5y + 6y = g(t), for a given function g. Hint, Solution 1118

Exercise 21.4 (mathematica/ode/inhomogeneous/variation.nb) Solve y (x) + y(x) = x, y(0) = 1, y (0) = 0. Hint, Solution Exercise 21.5 (mathematica/ode/inhomogeneous/variation.nb) Solve x2 y (x) xy (x) + y(x) = x. Hint, Solution Exercise 21.6 (mathematica/ode/inhomogeneous/variation.nb) 1. Find the general solution of y + y = ex . 2. Solve y + 2 y = sin x, y(0) = y (0) = 0. is an arbitrary real constant. Is there anything special about = 1? Hint, Solution Exercise 21.7 (mathematica/ode/inhomogeneous/variation.nb) Consider the problem of solving the initial value problem y + y = g(t), 1. Show that the general solution of y + y = g(t) is
t t

y(0) = 0,

y (0) = 0.

y(t) =

c1
a

g( ) sin d

cos t + c2 +
b

g( ) cos d

sin t,

where c1 and c2 are arbitrary constants and a and b are any conveniently chosen points. 2. Using the result of part (a) show that the solution satisfying the initial conditions y(0) = 0 and y (0) = 0 is given by
t

y(t) =
0

g( ) sin(t ) d.

1119

Notice that this equation gives a formula for computing the solution of the original initial value problem for any given inhomogeneous term g(t). The integral is referred to as the convolution of g(t) with sin t. 3. Use the result of part (b) to solve the initial value problem, y + y = sin(t), y(0) = 0, y (0) = 0,

where is a real constant. How does the solution for = 1 dier from that for = 1? The = 1 case provides an example of resonant forcing. Plot the solution for resonant and non-resonant forcing. Hint, Solution Exercise 21.8 Find the variation of parameters solution for the third order dierential equation y + p2 (x)y + p1 (x)y + p0 (x)y = f (x). Hint, Solution

Green Functions
Exercise 21.9 Use a Green function to solve y = f (x), y() = y () = 0. Verify the the solution satises the dierential equation. Hint, Solution Exercise 21.10 Solve the initial value problem 1 1 y + y 2 y = x2 , y(0) = 0, y (0) = 1. x x First use variation of parameters, and then solve the problem with a Green function. Hint, Solution 1120

Exercise 21.11 What are the continuity conditions at x = for the Green function for the problem y + p2 (x)y + p1 (x)y + p0 (x)y = f (x). Hint, Solution Exercise 21.12 Use variation of parameters and Green functions to solve x2 y 2xy + 2y = ex , Hint, Solution Exercise 21.13 Find the Green function for y y = f (x), Hint, Solution Exercise 21.14 Find the Green function for y y = f (x), Hint, Solution Exercise 21.15 Find the Green function for each of the following: a) xu + u = f (x), u(0+ ) bounded, u(1) = 0. b) u u = f (x), u(a) = u(a) = 0. c) u u = f (x), u(x) bounded as |x| . 1121 y(0) = y() = 0. y (0) = y(1) = 0. y(1) = 0, y (1) = 1.

d) Show that the Green function for (b) approaches that for (c) as a . Hint, Solution Exercise 21.16 1. For what values of does the problem y + y = f (x), y(0) = y() = 0, (21.5)

have a unique solution? Find the Green functions for these cases. 2. For what values of does the problem y + 9y = 1 + x, have a solution? Find the solution. 3. For = n2 , n Z+ state in general the conditions on f in Equation 21.5 so that a solution will exist. What is the appropriate modied Green function (in terms of eigenfunctions)? Hint, Solution Exercise 21.17 Show that the inhomogeneous boundary value problem: Lu (pu ) + qu = f (x), has the solution: u(x) =
a b

y(0) = y() = 0,

a < x < b,

u(a) = ,

u(b) =

g(x; )f () d p(a)g (x; a) + p(b)g (x; b).

Hint, Solution Exercise 21.18 The Green function for u k 2 u = f (x), 1122 < x <

subject to |u()| < is 1 k|x| e . 2k (We assume that k > 0.) Use the image method to nd the Green function for the same equation on the semi-innite interval 0 < x < satisfying the boundary conditions, G(x; ) = i) u(0) = 0 |u()| < , ii) u (0) = 0 |u()| < .

Express these results in simplied forms without absolute values. Hint, Solution Exercise 21.19 1. Determine the Green function for solving: y a2 y = f (x), y(0) = y (L) = 0.

2. Take the limit as L to nd the Green function on (0, ) for the boundary conditions: y(0) = 0, y () = 0. We assume here that a > 0. Use the limiting Green function to solve: y a2 y = ex , y(0) = 0, y () = 0.

Check that your solution satises all the conditions of the problem. Hint, Solution

1123

21.11

Hints

Undetermined Coecients
Hint 21.1

Hint 21.2

Variation of Parameters
Hint 21.3

Hint 21.4

Hint 21.5

Hint 21.6

Hint 21.7

Hint 21.8 Look for a particular solution of the form y p = u 1 y 1 + u 2 y 2 + u3 y 3 , 1124

where the yj s are homogeneous solutions. Impose the constraints u 1 y 1 + u 2 y 2 + u3 y 3 = 0 u1 y1 + u2 y2 + u3 y3 = 0. To avoid some messy algebra when solving for uj , use Kramers rule.

Green Functions
Hint 21.9

Hint 21.10

Hint 21.11

Hint 21.12

Hint 21.13 cosh(x) and sinh(x 1) are homogeneous solutions that satisfy the left and right boundary conditions, respectively. Hint 21.14 sinh(x) and ex are homogeneous solutions that satisfy the left and right boundary conditions, respectively. Hint 21.15 The Green function for the dierential equation L[y] d (p(x)y ) + q(x)y = f (x), dx 1125

subject to unmixed, homogeneous boundary conditions is G(x|) = y1 (x< )y2 (x> ) , p()W () for a x , for x b,

G(x|) =

y1 (x)y2 () p()W () y1 ()y2 (x) p()W ()

where y1 and y2 are homogeneous solutions that satisfy the left and right boundary conditions, respectively. Recall that if y(x) is a solution of a homogeneous, constant coecient dierential equation then y(x + c) is also a solution. Hint 21.16 The problem has a Green function if and only if the inhomogeneous problem has a unique solution. The inhomogeneous problem has a unique solution if and only if the homogeneous problem has only the trivial solution. Hint 21.17 Show that g (x; a) and g (x; b) are solutions of the homogeneous dierential equation. Determine the value of these solutions at the boundary. Hint 21.18 Hint 21.19

1126

21.12

Solutions

Undetermined Coecients
Solution 21.1 1. We consider y + 2y + 5y = 3 sin(2t). We rst nd the homogeneous solution with the substitition y = et . 2 + 2 + 5 = 0 = 1 2i The homogeneous solution is yh = c1 et cos(2t) + c2 et sin(2t). We guess a particular solution of the form yp = a cos(2t) + b sin(2t). We substitute this into the dierential equation to determine the coecients. yp + 2yp + 5yp = 3 sin(2t) 4a cos(2t) 4b sin(2t) 4a sin(2t) + 4b sin(2t) + 5a cos(2t) + 5b sin(2t) = 3 sin(2t) (a + 4b) cos(2t) + (3 4a + b) sin(2t) = 0 a + 4b = 0, 4a + b = 3 12 3 a= , b= 17 17 A particular solution is yp = 3 (sin(2t) 4 cos(2t)). 17 1127

The general solution of the dierential equation is y = c1 et cos(2t) + c2 et sin(2t) + 2. We consider 2y + 3y + y = t2 + 3 sin(t) We rst nd the homogeneous solution with the substitition y = et . 22 + 3 + 1 = 0 = {1, 1/2} The homogeneous solution is yh = c1 et +c2 et/2 . We guess a particular solution of the form yp = at2 + bt + c + d cos(t) + e sin(t). We substitute this into the dierential equation to determine the coecients. 2yp + 3yp + yp = t2 + 3 sin(t) 2(2a d cos(t) e sin(t)) + 3(2at + b d sin(t) + e cos(t)) + at2 + bt + c + d cos(t) + e sin(t) = t2 + 3 sin(t) (a 1)t2 + (6a + b)t + (4a + 3b + c) + (d + 3e) cos(t) (3 + 3d + e) sin(t) = 0 a 1 = 0, 6a + b = 0, 4a + 3b + c = 0, d + 3e = 0, 3 + 3d + e = 0 9 3 a = 1, b = 6, c = 14, d = , e = 10 10 1128 3 (sin(2t) 4 cos(2t)). 17

A particular solution is yp = t2 6t + 14 The general solution of the dierential equation is y = c1 et +c2 et/2 +t2 6t + 14 Solution 21.2 1. We consider the problem y 2y + y = t et +4, y(0) = 1, y (0) = 1. First we solve the homogeneous equation with the substitution y = et . 2 2 + 1 = 0 ( 1)2 = 0 =1 The homogeneous solution is yh = c1 et +c2 t et . We guess a particular solution of the form yp = at3 et +bt2 et +4. We substitute this into the inhomogeneous dierential equation to determine the coecients. yp 2yp + yp = t et +4 (a(t3 + 6t2 + 6t) + b(t2 + 4t + 2)) et 2(a(t2 + 3t) + b(t + 2)) et at3 et +bt2 et +4 = t et +4 (6a 1)t + 2b = 0 6a 1 = 0, 2b = 0 1 a= , b=0 6 1129 3 (3 cos(t) + sin(t)). 10 3 (3 cos(t) + sin(t)). 10

A particular solution is yp = The general solution of the dierential equation is y = c1 et +c2 t et + t3 t e +4. 6 t3 t e +4. 6

We use the initial conditions to determine the constants of integration. y(0) = 1, y (0) = 1 c1 + 4 = 1, c1 + c2 = 1 c1 = 3, c2 = 4 The solution of the initial value problem is y= 2. We consider the problem y + 2y + 5y = 4 et cos(2t), y(0) = 1, y (0) = 0. t3 + 4t 3 et +4. 6

First we solve the homogeneous equation with the substitution y = et . 2 + 2 + 5 = 0 = 1 1 5 = 1 2 The homogeneous solution is yh = c1 et cos(2t) + c2 et sin(2t). 1130

We guess a particular solution of the form yp = t et (a cos(2t) + b sin(2t)) We substitute this into the inhomogeneous dierential equation to determine the coecients. yp + 2yp + 5yp = 4 et cos(2t) et (((2 + 3t)a + 4(1 t)b) cos(2t) + (4(t 1)a (2 + 3t)b) sin(2t)) + 2 et (((1 t)a + 2tb) cos(2t) + (2ta + (1 t)b) sin(2t)) + 5(et (ta cos(2t) + tb sin(2t))) = 4 et cos(2t) 4(b 1) cos(2t) 4a sin(2t) = 0 a = 0, b = 1 A particular solution is yp = t et sin(2t). The general solution of the dierential equation is y = c1 et cos(2t) + c2 et sin(2t) + t et sin(2t). We use the initial conditions to determine the constants of integration. y(0) = 1, y (0) = 0 c1 = 1, c1 + 2c2 = 0 1 c1 = 1, c2 = 2 The solution of the initial value problem is y= 1 t e (2 cos(2t) + (2t + 1) sin(2t)) . 2 1131

Variation of Parameters
Solution 21.3 1. We consider the equation y 5y + 6y = 2 et . We nd homogeneous solutions with the substitution y = et . 2 5 + 6 = 0 = {2, 3} The homogeneous solutions are y1 = e2t , We compute the Wronskian of these solutions. W (t) = e2t e3t = e5t 2 e2t 3 e3t y2 = e3t .

We nd a particular solution with variation of parameters. yp = e2t = 2 e2t = 2 et et y p = et 2. We consider the equation y + y = tan(t), 1132 0<t< . 2 2 et e3t dt + e3t e5t et dt + 2 e3t 2 et e2t dt e5t e2t dt

We nd homogeneous solutions with the substitution y = et . 2 + 1 = 0 = i The homogeneous solutions are y1 = cos(t), We compute the Wronskian of these solutions. W (t) = cos(t) sin(t) = cos2 (t) + sin2 (t) = 1 sin(t) cos(t) y2 = sin(t).

We nd a particular solution with variation of parameters. yp = cos(t) = cos(t) tan(t) sin(t) dt + sin(t) tan(t) cos(t) dt

sin2 (t) dt + sin(t) sin(t) dt cos(t) cos(t/2) sin(t/2) = cos(t) ln + sin(t) cos(t/2) + sin(t/2) yp = cos(t) ln cos(t/2) sin(t/2) cos(t/2) + sin(t/2)

sin(t) cos(t)

3. We consider the equation y 5y + 6y = g(t). The homogeneous solutions are y1 = e2t , 1133 y2 = e3t .

The Wronskian of these solutions is W (t) = e5t . We nd a particular solution with variation of parameters. yp = e2t yp = e2t Solution 21.4 Solve y (x) + y(x) = x, The solutions of the homogeneous equation are y1 (x) = cos x, The Wronskian of these solutions is W [cos x, sin x] = cos x sin x sin x cos x y2 (x) = sin x. y(0) = 1, y (0) = 0. g(t) e3t dt + e3t 5t e g(t) e2t dt + e3t g(t) e2t dt e5t g(t) e3t dt

= cos2 x + sin2 x = 1. The variation of parameters solution for the particular solution is yp = cos x x sin x dx + sin x x cos x dx sin x dx

= cos x x cos x +

cos x dx + sin x x sin x

= cos x (x cos x + sin x) + sin x (x sin x + cos x) = x cos2 x cos x sin x + x sin2 x + cos x sin x =x 1134

The general solution of the dierential equation is thus y = c1 cos x + c2 sin x + x. Applying the two initial conditions gives us the equations c1 = 1, The solution subject to the initial conditions is y = cos x sin x + x. Solution 21.5 Solve x2 y (x) xy (x) + y(x) = x. The homogeneous equation is x2 y (x) xy (x) + y(x) = 0. Substituting y = x into the homogeneous dierential equation yields x2 ( 1)x2 xx + x = 0 2 2 + 1 = 0 ( 1)2 = 0 = 1. The homogeneous solutions are y1 = x, The Wronskian of the homogeneous solutions is W [x, x log x] = x x log x 1 1 + log x y2 = x log x. c2 + 1 = 0.

= x + x log x x log x = x. 1135

Writing the inhomogeneous equation in the standard form: 1 1 1 y (x) y (x) + 2 y(x) = . x x x Using variation of parameters to nd the particular solution, yp = x log x dx + x log x x 1 dx x

1 = x log2 x + x log x log x 2 1 = x log2 x. 2 Thus the general solution of the inhomogeneous dierential equation is 1 y = c1 x + c2 x log x + x log2 x. 2 Solution 21.6 1. First we nd the homogeneous solutions. We substitute y = ex into the homogeneous dierential equation. y +y =0 2 + 1 = 0 = y = ex , ex We can also write the solutions in terms of real-valued functions. y = {cos x, sin x} 1136

The Wronskian of the homogeneous solutions is W [cos x, sin x] = cos x sin x = cos2 x + sin2 x = 1. sin x cos x

We obtain a particular solution with the variation of parameters formula. yp = cos x ex sin x dx + sin x ex cos x dx

1 1 yp = cos x ex (sin x cos x) + sin x ex (sin x + cos x) 2 2 1 x yp = e 2 The general solution is the particular solution plus a linear combination of the homogeneous solutions. y= 2. y + 2 y = sin x, y(0) = y (0) = 0 Assume that is positive. First we nd the homogeneous solutions by substituting y = ex into the homogeneous dierential equation. y + 2 y = 0 2 + 2 = 0 = y = ex , ex y = {cos(x), sin(x)} 1137 1 x e + cos x + sin x 2

The Wronskian of these homogeneous solution is W [cos(x), sin(x)] = cos(x) sin(x) = cos2 (x) + sin2 (x) = . sin(x) cos(x)

We obtain a particular solution with the variation of parameters formula. yp = cos(x) We evaluate the integrals for = 1. yp = cos(x) cos(x) sin(x) sin x cos(x) cos(x) cos(x) + sin x sin(x) + sin(x) 2 1) ( (2 1) sin x yp = 2 1 sin(x) sin x dx + sin(x) cos(x) sin x dx

The general solution for = 1 is y= sin x + c1 cos(x) + c2 sin(x). 2 1

The initial conditions give us the constraints: c1 = 0, 2 1 + c2 = 0, 1

For = 1, (non-resonant forcing), the solution subject to the initial conditions is y= sin(x) sin(x) . (2 1) 1138

Now consider the case = 1. We obtain a particular solution with the variation of parameters formula. yp = cos(x) sin2 (x) dx + sin(x) cos(x) sin x dx

1 1 yp = cos(x) (x cos(x) sin(x)) + sin(x) cos2 (x) 2 2 1 yp = x cos(x) 2 The general solution for = 1 is 1 y = x cos(x) + c1 cos(x) + c2 sin(x). 2 The initial conditions give us the constraints: c1 = 0 1 + c2 = 0 2 For = 1, (resonant forcing), the solution subject to the initial conditions is 1 y = (sin(x) x cos x). 2 Solution 21.7 1. A set of linearly independent, homogeneous solutions is {cos t, sin t}. The Wronskian of these solutions is W (t) = cos t sin t = cos2 t + sin2 t = 1. sin t cos t

We use variation of parameters to nd a particular solution. yp = cos t g(t) sin t dt + sin t 1139 g(t) cos t dt

The general solution can be written in the form,


t t

y(t) =

c1
a

g( ) sin d

cos t + c2 +
b

g( ) cos d

sin t.

2. Since the initial conditions are given at t = 0 we choose the lower bounds of integration in the general solution to be that point.
t t

y=

c1
0

g( ) sin d

cos t + c2 +
0

g( ) cos d

sin t

The initial condition y(0) = 0 gives the constraint, c1 = 0. The derivative of y(t) is then,
t t

y (t) = g(t) sin t cos t +


0 t

g( ) sin d sin t + g(t) cos t sin t + c2 +


0 t

g( ) cos d cos t.

cos t,

y (t) =
0

g( ) sin d sin t + c2 +
0

g( ) cos d

The initial condition y (0) = 0 gives the constraint c2 = 0. The solution subject to the initial conditions is
t

y=
0

g( )(sin t cos cos t sin ) d


t

y=
0

g( ) sin(t ) d

3. The solution of the initial value problem y + y = sin(t), is y=


0 t

y(0) = 0,

y (0) = 0,

sin( ) sin(t ) d.

1140

Figure 21.5: Non-resonant Forcing

For = 1, this is 1 t cos(t ) cos(t + ) d 2 0 t 1 sin(t ) sin(t + ) + = 2 1+ 1 0 1 sin(t) sin(t) sin(t) + sin(t) + = 2 1+ 1

y=

y=

sin t sin(t) + . 2 1 1 2

(21.6)

The solution is the sum of two periodic functions of period 2 and 2/. This solution is plotted in Figure 21.5 on the interval t [0, 16] for the values = 1/4, 7/8, 5/2. 1141

Figure 21.6: Resonant Forcing For = 1, we have y= 1 t cos(t 2 ) cos(tau) d 2 0 t 1 1 = sin(t 2 ) cos t 2 2 0 y= 1 (sin t t cos t) . 2 (21.7)

The solution has both a periodic and a transient term. This solution is plotted in Figure 21.5 on the interval t [0, 16]. Note that we can derive (21.7) from (21.6) by taking the limit as 0. sin(t) sin t t cos(t) sin t = lim 2 1 1 1 2 1 = (sin t t cos t) 2 lim 1142

Solution 21.8 Let y1 , y2 and y3 be linearly independent homogeneous solutions to the dierential equation L[y] = y + p2 y + p1 y + p0 y = f (x). We will look for a particular solution of the form y p = u 1 y 1 + u 2 y 2 + u3 y 3 . Since the uj s are undetermined functions, we are free to impose two constraints. We choose the constraints to simplify the algebra. u 1 y 1 + u2 y 2 + u3 y 3 = 0 u 1 y 1 + u2 y 2 + u3 y 3 = 0 Dierentiating the expression for yp , y p = u 1 y 1 + u1 y 1 + u2 y 2 + u 2 y 2 + u 3 y 3 + u3 y 3 = u 1 y 1 + u2 y 2 + u3 y 3 y p = u 1 y 1 + u1 y 1 + u 2 y 2 + u 2 y 2 + u 3 y 3 + u 3 y 3 = u 1 y 1 + u2 y 2 + u3 y 3 y p = u 1 y 1 + u1 y 1 + u2 y 2 + u 2 y 2 + u3 y 3 + u3 y 3 Substituting the expressions for yp and its derivatives into the dierential equation, u1 y1 + u1 y1 + u2 y2 + u2 y2 + u3 y3 + u3 y3 + p2 (u1 y1 + u2 y2 + u3 y3 ) + p1 (u1 y1 + u2 y2 + u3 y3 ) + p0 (u1 y1 + u2 y2 + u3 y3 ) = f (x) u1 y1 + u2 y2 + u3 y3 + u1 L[y1 ] + u2 L[y2 ] + u3 L[y3 ] = f (x) u1 y1 + u2 y2 + u3 y3 = f (x). 1143

With the two constraints, we have the system of equations, u 1 y 1 + u2 y 2 + u3 y 3 = 0 u 1 y 1 + u2 y 2 + u3 y 3 = 0 u1 y1 + u2 y2 + u3 y3 = f (x) We solve for the uj using Kramers rule. u1 = (y2 y3 y2 y3 )f (x) , W (x) u2 = (y1 y3 y1 y3 )f (x) , W (x) u3 = (y1 y2 y1 y2 )f (x) W (x)

Here W (x) is the Wronskian of {y1 , y2 , y3 }. Integrating the expressions for uj , the particular solution is yp = y1 (y2 y3 y2 y3 )f (x) dx + y2 W (x) (y3 y1 y3 y1 )f (x) dx + y3 W (x) (y1 y2 y1 y2 )f (x) dx. W (x)

Green Functions
Solution 21.9 We consider the Green function problem G = f (x), G(|) = G (|) = 0.

The homogeneous solution is y = c1 + c2 x. The homogeneous solution that satises the boundary conditions is y = 0. Thus the Green function has the form 0 x < , G(x|) = c1 + c2 x x > . The continuity and jump conditions are then G( + |) = 0, G ( + |) = 1.

1144

Thus the Green function is G(x|) = The solution of the problem y = f (x), is

0 x

x < , = (x )H(x ). x>

y() = y () = 0.

y=

f ()G(x|) d f ()(x )H(x ) d

y=

y=

f ()(x ) d

We dierentiate this solution to verify that it satises the dierential equation. y = [f ()(x )]=x + (f ()(x )) d = x y = [f ()]=x = f (x)
x x

f () d

Solution 21.10 Since we are dealing with an Euler equation, we substitute y = x to nd the homogeneous solutions. ( 1) + 1 = 0 ( 1)( + 1) = 0 1 y1 = x, y2 = x 1145

Variation of Parameters. The Wronskian of the homogeneous solutions is W (x) = A particular solution is 1 x2 x x2 (1/x) dx + dx yp = x 2/x x 2/x x2 1 x4 = x dx + dx 2 x 2 4 4 x x = 6 10 x4 = . 15 The general solution is y= Applying the initial conditions, y(0) = 0 y (0) = 0 Thus we have the solution y= x4 + x. 15 c2 = 0 c1 = 1. x4 1 + c1 x + c2 . 15 x 2 1 1 x 1/x = . 2 = 1 1/x x x x

1146

Green Function. Since this problem has both an inhomogeneous term in the dierential equation and inhomogeneous boundary conditions, we separate it into the two problems 1 1 u + u 2 u = x2 , x x 1 1 v + v 2 v = 0, x x u(0) = u (0) = 0, v(0) = 0, v (0) = 1.

First we solve the inhomogeneous dierential equation with the homogeneous boundary conditions. The Green function for this problem satises L[G(x|)] = (x ), G(0|) = G (0|) = 0.

Since the Green function must satisfy the homogeneous boundary conditions, it has the form 0 cx + d/x for x < for x > .

G(x|) =

From the continuity condition, 0 = c + d/. The jump condition yields c d/ 2 = 1. Solving these two equations, we obtain 0
1 x 2

G(x|) =

2 2x

for x < for x >

1147

Thus the solution is

u(x) =
0 x

G(x|) 2 d 2 d

1 2 x 2 2x 0 1 1 = x4 x4 6 10 x4 = . 15 =

Now to solve the homogeneous dierential equation with inhomogeneous boundary conditions. The general solution for v is v = cx + d/x. Applying the two boundary conditions gives v = x. Thus the solution for y is y =x+ Solution 21.11 The Green function satises G (x|) + p2 (x)G (x|) + p1 (x)G (x|) + p0 (x)G(x|) = (x ). First note that only the G (x|) term can have a delta function singularity. If a lower derivative had a delta function type singularity, then G (x|) would be more singular than a delta function and there would be no other term in the equation to balance that behavior. Thus we see that G (x|) will have a delta function singularity; G (x|) will have a jump discontinuity; G (x|) will be continuous at x = . Integrating the dierential equation from to + yields
+ +

x4 . 15

G (x|) dx =

(x ) dx

1148

G ( + |) G ( |) = 1. Thus we have the three continuity conditions: G ( + |) = G ( |) + 1 G ( + |) = G ( |) G( + |) = G( |) Solution 21.12 Variation of Parameters. Consider the problem x2 y 2xy + 2y = ex , Previously we showed that two homogeneous solutions are y1 = x, The Wronskian of these solutions is W (x) = y 2 = x2 . y(1) = 0, y (1) = 1.

x x2 = 2x2 x2 = x2 . 1 2x

In the variation of parameters formula, we will choose 1 as the lower bound of integration. (This will simplify the algebra in applying the initial conditions.) yp = x
x e e 2 d + x2 d 4 4 1 1 x x e e 2 = x d + x d 2 3 1 1 x ex e = x e1 d + x2 x 1 x + x2 1 = x e1 + (1 + x) ex + 2 2 x

ex ex 1 2+ 2x 2x 2 x e d 1

x 1

e d

1149

If you wanted to, you could write the last integral in terms of exponential integral functions. The general solution is y = c1 x + c2 x x e Applying the boundary conditions, y(1) = 0 y (1) = 1 we nd that c1 = 1, c2 = 1. Thus the solution subject to the initial conditions is x2 1 y = (1 + e )x + x + (1 + x) ex + x + 2 2
1 2 x 1 2 1

1 x2 + (1 + x) ex + x + 2 2

x 1

e d

c1 + c2 = 0 c1 + 2c2 = 1,

e d

Green Functions. The solution to the problem is y = u + v where ex 2 2 u u + 2u = 2 , x x x and u(1) = 0, u (1) = 0,

2 2 v v + 2 v = 0, v(1) = 0, x x The problem for v has the solution v = x + x2 . The Green function for u is G(x|) = H(x )u (x) where u () = 0,

v (1) = 1.

and u () = 1. 1150

Thus the Green function is G(x|) = H(x ) x + The solution for u is then u=

x2

e d 2 1 x x2 e = x + d 2 1 1 x2 = x e1 + (1 + x) ex + x + 2 2 G(x|)

x 1

e d.
x 1

Thus we nd the solution for y is x2 1 e1 )x + x2 + (1 + x) ex + x + y = (1 + 2 2 Solution 21.13 The dierential equation for the Green function is G G = (x ), Gx (0|) = G(1|) = 0. Note that cosh(x) and sinh(x 1) are homogeneous solutions that satisfy the left and right boundary conditions, respectively. The Wronskian of these two solutions is W (x) = cosh(x) sinh(x 1) sinh(x) cosh(x 1) e d

= cosh(x) cosh(x 1) sinh(x) sinh(x 1) 1 ex + ex ex1 + ex+1 ex ex ex1 ex+1 = 4 1 1 e + e1 = 2 = cosh(1). 1151

The Green function for the problem is then G(x|) = cosh(x< ) sinh(x> 1) , cosh(1) for 0 x , for x 1.

G(x|) = Solution 21.14 The dierential equation for the Green function is

cosh(x) sinh(1) cosh(1) cosh() sinh(x1) cosh(1)

G G = (x ),

G(0|) = G(|) = 0.

Note that sinh(x) and ex are homogeneous solutions that satisfy the left and right boundary conditions, respectively. The Wronskian of these two solutions is W (x) = sinh(x) ex cosh(x) ex

= sinh(x) ex cosh(x) ex 1 1 = ex ex ex ex + ex ex 2 2 = 1 The Green function for the problem is then G(x|) = sinh(x< ) ex> G(x|) = sinh(x) e sinh() ex for 0 x , for x .

1152

Solution 21.15

a) The Green function problem is xG (x|) + G (x|) = (x ), G(0|) bounded, G(1|) = 0.

First we nd the homogeneous solutions of the dierential equation. xy + y = 0 This is an exact equation. d [xy ] = 0 dx c1 y = x y = c1 log x + c2 The homogeneous solutions y1 = 1 and y2 = log x satisfy the left and right boundary conditions, respectively. The Wronskian of these solutions is 1 1 log x W (x) = = . 0 1/x x The Green function is G(x|) = 1 log x> , (1/)

G(x|) = log x> . b) The Green function problem is G (x|) G(x|) = (x ), 1153 G(a|) = G(a|) = 0.

{ex , ex } and {cosh x, sinh x} are both linearly independent sets of homogeneous solutions. sinh(x + a) and sinh(x a) are homogeneous solutions that satisfy the left and right boundary conditions, respectively. The Wronskian of these two solutions is, W (x) = sinh(x + a) sinh(x a) cosh(x + a) cosh(x a)

= sinh(x + a) cosh(x a) sinh(x a) cosh(x + a) = sinh(2a) The Green function is G(x|) = c) The Green function problem is G (x|) G(x|) = (x ), G(x|) bounded as |x| . sinh(x< + a) sinh(x> a) . sinh(2a)

ex and ex are homogeneous solutions that satisfy the left and right boundary conditions, respectively. The Wronskian of these solutions is ex ex = 2. W (x) = x e ex The Green function is G(x|) = ex< ex> , 2

1 G(x|) = ex< x> . 2 d) The Green function from part (b) is, G(x|) = sinh(x< + a) sinh(x> a) . sinh(2a) 1154

We take the limit as a . (ex< +a ex< a ) (ex> a ex> +a ) sinh(x< + a) sinh(x> a) = lim a a sinh(2a) 2 (e2a e2a ) ex< x> + ex< +x> 2a + ex< x> 2a ex< +x> 4a = lim a 2 2 e4a ex< x> = 2 lim Thus we see that the solution from part (b) approaches the solution from part (c) as a . Solution 21.16 1. The problem, y + y = f (x), y(0) = y() = 0, has a Green function if and only if it has a unique solution. This inhomogeneous problem has a unique solution if and only if the homogeneous problem has only the trivial solution. First consider the case = 0. We nd the general solution of the homogeneous dierential equation. y = c1 + c2 x Only the trivial solution satises the boundary conditions. The problem has a unique solution for = 0. Now consider non-zero . We nd the general solution of the homogeneous dierential equation. y = c1 cos x + c2 sin x .

The solution that satises the left boundary condition is y = c sin 1155 x .

We apply the right boundary condition and nd nontrivial solutions. = 0 sin = n2 , n Z+

Thus the problem has a unique solution for all complex except = n2 , n Z+ . Consider the case = 0. We nd solutions of the homogeneous equation that satisfy the left and right boundary conditions, respectively. y1 = x, y2 = x . We compute the Wronskian of these functions. W (x) = The Green function for this case is x x = . 1 1

x< (x> ) . We consider the case = n2 , = 0. We nd the solutions of the homogeneous equation that satisfy the left and right boundary conditions, respectively. y1 = sin x , y2 = sin (x ) . G(x|) = We compute the Wronskian of these functions. x sin (x ) sin W (x) = cos x cos (x ) The Green function for this case is sin G(x|) = x< sin (x> ) . sin

= sin

1156

2. Now we consider the problem y + 9y = 1 + x, y(0) = y() = 0. The homogeneous solutions of the problem are constant multiples of sin(3x). Thus for each value of , the problem either has no solution or an innite number of solutions. There will be an innite number of solutions if the inhomogeneity 1 + x is orthogonal to the homogeneous solution sin(3x) and no solution otherwise.

(1 + x) sin(3x) dx =
0

+ 2 3

The problem has a solution only for = 2/. For this case the general solution of the inhomogeneous dierential equation is 1 2x y= 1 + c1 cos(3x) + c2 sin(3x). 9 The one-parameter family of solutions that satises the boundary conditions is y= 1 9 1 2x cos(3x) + c sin(3x).

3. For = n2 , n Z+ , y = sin(nx) is a solution of the homogeneous equation that satises the boundary conditions. Equation 21.5 has a (non-unique) solution only if f is orthogonal to sin(nx).

f (x) sin(nx) dx = 0
0

The modied Green function satises G + n2 G = (x ) We expand G in a series of the eigenfunctions.

sin(nx) sin(n) . /2

G(x|) =
k=1

gk sin(kx)

1157

We substitute the expansion into the dierential equation to determine the coecients. This will not determine gn . We choose gn = 0, which is one of the choices that will make the modied Green function symmetric in x and .

gk n k
k=1

2 sin(kx) = 2

sin(kx) sin(k)
k=1 k=n

G(x|) =

k=1 k=n

sin(kx) sin(k) n2 k 2

The solution of the inhomogeneous problem is

y(x) =
0

f ()G(x|) d.

Solution 21.17 We separate the problem for u into the two problems: Lv (pv ) + qv = f (x), a < x < b, v(a) = 0, v(b) = 0 Lw (pw ) + qw = 0, a < x < b, w(a) = , w(b) = and note that the solution for u is u = v + w. The problem for v has the solution,
b

v=
a

g(x; )f () d,
v1 (x)v2 () p()W () v1 ()v2 (x) p()W ()

with the Green function, g(x; ) = v1 (x< )v2 (x> ) p()W () for a x , for x b.

Here v1 and v2 are homogeneous solutions that respectively satisfy the left and right homogeneous boundary conditions. 1158

Since g(x; ) is a solution of the homogeneous equation for x = , g (x; ) is a solution of the homogeneous equation for x = . This is because for x = , L g = L[g] = (x ) = 0.

If is outside of the domain, (a, b), then g(x; ) and g (x; ) are homogeneous solutions on that domain. In particular g (x; a) and g (x; b) are homogeneous solutions, L [g (x; a)] = L [g (x; b)] = 0. Now we use the denition of the Green function and v1 (a) = v2 (b) = 0 to determine simple expressions for these homogeneous solutions. g (x; a) = v1 (a)v2 (x) (p (a)W (a) + p(a)W (a))v1 (a)v2 (x) p(a)W (a) (p(a)W (a))2 v (a)v2 (x) = 1 p(a)W (a) v1 (a)v2 (x) = p(a)(v1 (a)v2 (a) v1 (a)v2 (a)) v (a)v2 (x) = 1 p(a)v1 (a)v2 (a) v2 (x) = p(a)v2 (a)

We note that this solution has the boundary values, g (a; a) = v2 (a) 1 = , p(a)v2 (a) p(a) 1159 g (b; a) = v2 (b) = 0. p(a)v2 (a)

We examine the second solution. g (x; b) = = = = = This solution has the boundary values, g (a; b) = Thus we see that the solution of Lw = (pw ) + qw = 0, is w = p(a)g (x; a) + p(b)g (x; b). Therefore the solution of the problem for u is
b

v1 (x)v2 (b) (p (b)W (b) + p(b)W (b))v1 (x)v2 (b) p(b)W (b) (p(b)W (b))2 v1 (x)v2 (b) p(b)W (b) v1 (x)v2 (b) p(b)(v1 (b)v2 (b) v1 (b)v2 (b)) v1 (x)v2 (b) p(b)v1 (b)v2 (b) v1 (x) p(b)v1 (b)

v1 (a) = 0, p(b)v1 (b)

g (b; b) =

v1 (b) 1 = . p(b)v1 (b) p(b)

a < x < b,

w(a) = ,

w(b) = ,

u=
a

g(x; )f () d p(a)g (x; a) + p(b)g (x; b).

Solution 21.18 Figure 21.7 shows a plot of G(x; 1) and G(x; 1) for k = 1. 1160

-4

-2 -0.1

-0.2

-0.3

-0.4

-0.5

Figure 21.7: G(x; 1) and G(x; 1) First we consider the boundary condition u(0) = 0. Note that the solution of G k 2 G = (x ) (x + ), |G(; )| < , satises the condition G(0; ) = 0. Thus the Green function which satises G(0; ) = 0 is 1 1 G(x; ) = ek|x| + ek|x+| . 2k 2k Since x, > 0 we can write this as 1 1 G(x; ) = ek|x| + ek(x+) 2k 2k 1 k(x) 1 2k e + 2k ek(x+) , for x < = 1 1 2k ek(x) + 2k ek(x+) , for < x =
1 k ek sinh(kx), for x < 1 k ekx sinh(k), for < x

1161

1 G(x; ) = ekx> sinh(kx< ) k Now consider the boundary condition u (0) = 0. Note that the solution of G k 2 G = (x ) + (x + ), |G(; )| < ,

satises the boundary condition G (x; ) = 0. Thus the Green function is G(x; ) = Since x, > 0 we can write this as G(x; ) = = = 1 k|x| 1 k(x+) e e 2k 2k 1 1 k(x) 2k ek(x+) , for x < 2k e 1 1 2k ek(x) 2k ek(x+) , for < x
1 k ek cosh(kx), for x < 1 k ekx cosh(k), for < x

1 k|x| 1 k|x+| e e . 2k 2k

1 G(x; ) = ekx> cosh(kx< ) k The Green functions which satises G(0; ) = 0 and G (0; ) = 0 are shown in Figure 21.8. Solution 21.19 1. The Green function satises g a2 g = (x ), We can write the set of homogeneous solutions as eax , eax or {cosh(ax), sinh(ax)} . 1162 g(0; ) = g (L; ) = 0.

1 -0.1 -0.2 -0.3 -0.4

-0.1 -0.2 -0.3 -0.4 -0.5

Figure 21.8: G(x; 1) and G(x; 1) The solutions that respectively satisfy the left and right boundary conditions are u1 = sinh(ax), The Wronskian of these solutions is W (x) = Thus the Green function is g(x; ) = sinh(ax) cosh(a(L)) a cosh(aL) sinh(a) cosh(a(xL)) a cosh(aL) for x , for x. = sinh(ax< ) cosh(a(x> L)) . a cosh(aL) sinh(ax) cosh(a(x L)) a cosh(ax) a sinh(a(x L)) = a cosh(aL). u2 = cosh(a(x L)).

2. We take the limit as L . g(x; ) = lim sinh(ax< ) cosh(a(x> L)) L a cosh(aL) sinh(ax< ) cosh(ax> ) cosh(aL) sinh(ax> ) sinh(aL) = lim L a cosh(aL) sinh(ax< ) (cosh(ax> ) sinh(ax> )) = a 1163

1 g(x; ) = sinh(ax< ) eax> a The solution of y a2 y = ex , is

y(0) = y () = 0

y=
0

g(x; ) e d

1 a 1 = a =

sinh(ax< ) eax> e d
0 x

sinh(a) eax e d +
0 x

sinh(ax) ea e d

We rst consider the case that a = 1. eax 1 e(a+1)x sinh(ax) a + ex (a cosh(ax) + sinh(ax)) + 21 a a+1 ax x e e = 21 a For a = 1, we have 1 1 e x 1 + 2x + e2x + e2x sinh(x) y= 4 2 1 = x ex . 2 Thus the solution of the problem is = 1 a y=
eax ex a2 1 1 2 x ex

for a = 1, for a = 1.

We note that this solution satises the dierential equation and the boundary conditions.

1164

21.13

Quiz
y y = f (x),

Problem 21.1 Find the general solution of where f (x) is a known function. Solution

1165

21.14

Quiz Solutions
y y = f (x)

Solution 21.1

We substitute y = ex into the homogeneous dierential equation. y y =0 2 ex ex = 0 = 1 The homogeneous solutions are ex and ex . The Wronskian of these solutions is ex ex = 2. ex ex We nd a particular solution with variation of parameters. y p = ex The general solution is y = c1 ex +c2 ex ex ex f (x) dx + ex 2 ex f (x) dx 2 ex f (x) dx. 2

ex f (x) dx + ex 2

1166

Chapter 22 Dierence Equations


Televisions should have a dial to turn up the intelligence. There is a brightness knob, but it doesnt work. -?

22.1

Introduction

Example 22.1.1 Gamblers ruin problem. Consider a gambler that initially has n dollars. He plays a game in which he has a probability p of winning a dollar and q of losing a dollar. (Note that p + q = 1.) The gambler has decided that if he attains N dollars he will stop playing the game. In this case we will say that he has succeeded. Of course if he runs out of money before that happens, we will say that he is ruined. What is the probability of the gamblers ruin? Let us denote this probability by an . We know that if he has no money left, then his ruin is certain, so a0 = 1. If he reaches N dollars he will quit the game, so that aN = 0. If he is somewhere in between ruin and success then the probability of his ruin is equal to p times the probability of his ruin if he had n + 1 dollars plus q times the probability of his ruin if he had n 1 dollars. Writing this in an equation, an = pan+1 + qan1 subject to a0 = 1, 1167 aN = 0.

This is an example of a dierence equation. You will learn how to solve this particular problem in the section on constant coecient equations. Consider the sequence a1 , a2 , a3 , . . . Analogous to a derivative of a continuous function, we can dene a discrete derivative on the sequence Dan = an+1 an . The second discrete derivative is then dened as D2 an = D[an+1 an ] = an+2 2an+1 + an . The discrete integral of an is
n

ai .
i=n0

Corresponding to

df dx = f () f (), dx
1

in the discrete realm we have

D[an ] =
n= n=

(an+1 an ) = a a .

Linear dierence equations have the form Dr an + pr1 (n)Dr1 an + + p1 (n)Dan + p0 (n)an = f (n). From the denition of the discrete derivative an equivalent form is an+r + qr1 (n)anr 1 + + q1 (n)an+1 + q0 (n)an = f (n). Besides being important in their own right, we will need to solve dierence equations in order to develop series solutions of dierential equations. Also, some methods of solving dierential equations numerically are based on approximating them with dierence equations. 1168

There are many similarities between dierential and dierence equations. Like dierential equations, an rth order homogeneous dierence equation has r linearly independent solutions. The general solution to the rth order inhomogeneous equation is the sum of the particular solution and an arbitrary linear combination of the homogeneous solutions. For an rth order dierence equation, the initial condition is given by specifying the values of the rst r an s. Example 22.1.2 Consider the dierence equation an2 an1 an = 0 subject to the initial condition a1 = a2 = 1. Note that although we may not know a closed-form formula for the an we can calculate the an in order by substituting into the dierence equation. The rst few an are 1, 1, 2, 3, 5, 8, 13, 21, . . . We recognize this as the Fibonacci sequence.

22.2

Exact Equations

Consider the sequence a1 , a2 , . . .. Exact dierence equations on this sequence have the form D[F (an , an+1 , . . . , n)] = g(n). We can reduce the order of, (or solve for rst order), this equation by summing from 1 to n 1.
n1 n1

D[F (aj , aj+1 , . . . , j)] =


j=1 j=1

g(j)
n1

F (an , an+1 , . . . , n) F (a1 , a2 , . . . , 1) =


j=1 n1

g(j)

F (an , an+1 , . . . , n) =
j=1

g(j) + F (a1 , a2 , . . . , 1)

1169

Result 22.2.1 We can reduce the order of the exact dierence equation D[F (an , an+1 , . . . , n)] = g(n), by summing both sides of the equation to obtain
n1

for n 1

F (an , an+1 , . . . , n) =
j=1

g(j) + F (a1 , a2 , . . . , 1).

Example 22.2.1 Consider the dierence equation, D[nan ] = 1. Summing both sides of this equation
n1 n1

D[jaj ] =
j=1 j=1

nan a1 = n 1 an = n + a1 1 . n

22.3

Homogeneous First Order

Consider the homogeneous rst order dierence equation for n 1.

an+1 = p(n)an , 1170

We can directly solve for an . an = an a1 an1 an2 an1 an2 a1 an an1 a2 = a1 an1 an2 a1 = a1 p(n 1)p(n 2) p(1)
n1

= a1
j=1

p(j)

Alternatively, we could solve this equation by making it exact. Analogous to an integrating factor for dierential equations, we multiply the equation by the summing factor
n 1

S(n) =
j=1

p(j)

an+1 p(n)an = 0 an an+1 n1 =0 n j=1 p(j) j=1 p(j) D Now we sum from 1 to n 1. an a1 n1 j=1 p(j) n1 an = a1
j=1

an n1 j=1 p(j)

=0

=0

p(j)

1171

Result 22.3.1 The solution of the homogeneous rst order dierence equation an+1 = p(n)an , is an = a1
j=1 n1

for n 1,

p(j).

Example 22.3.1 Consider the equation an+1 = nan with the initial condition a1 = 1.
n1

an = a1
j=1

j = (1)(n 1)! = (n)

Recall that (z) is the generalization of the factorial function. For positive integral values of the argument, (n) = (n 1)!.

22.4

Inhomogeneous First Order


an+1 = p(n)an + q(n) for n 1.

Consider the equation Multiplying by S(n) =


n j=1 1

p(j)

yields an+1 n j=1 p(j) an n1 j=1 p(j) =


n j=1

q(n) . p(j)

The left hand side is a discrete derivative. D an n1 j=1 p(j) =


n j=1

q(n) p(j)

1172

Summing both sides from 1 to n 1,

an a1 = n1 p(j) j=1

n1

q(k)
k j=1

k=1

p(j)

n1

n1

an =
m=1

p(m)
k=1

q(k)
k j=1

p(j)

+ a1 .

Result 22.4.1 The solution of the inhomogeneous rst order dierence equation an+1 = p(n)an + q(n) for n 1 is an =
m=1 n1 n1

p(m)
k=1

q(k)
k j=1 p(j)

+ a1 .

Example 22.4.1 Consider the equation an+1 = nan + 1 for n 1. The summing factor is
1

S(n) =
j=1

1 . n!

1173

Multiplying the dierence equation by the summing factor, an+1 an 1 = n! (n 1)! n! an 1 D = (n 1)! n! an a1 = (n 1)!
n1 n1

k=1

1 k!

an = (n 1)!
k=1

1 + a1 . k!

Example 22.4.2 Consider the equation an+1 = an + ,


n1 n1

for n 0.

From the above result, (with the products and sums starting at zero instead of one), the solution is a0 = =
m=0 n1 n k=0

k j=0

+ a0

k+1

+ a0

1 + a0 1 1 n 1 = n + a0 1 1 n = + a0 n . 1 = n

k=0 n1

1174

22.5

Homogeneous Constant Coecient Equations


an+N + pN 1 an+N 1 + + p1 an+1 + p0 an = 0.

Homogeneous constant coecient equations have the form

The substitution an = rn yields rN + pN 1 rN 1 + + p1 r + p0 = 0 (r r1 )m1 (r rk )mk = 0.


n If r1 is a distinct root then the associated linearly independent solution is r1 . If r1 is a root of multiplicity m > 1 n n n n then the associated solutions are r1 , nr1 , n2 r1 , . . . , nm1 r1 .

Result 22.5.1 Consider the homogeneous constant coecient dierence equation an+N + pN 1 an+N 1 + + p1 an+1 + p0 an = 0. The substitution an = rn yields the equation (r r1 )m1 (r rk )mk = 0. A set of linearly independent solutions is
n n n n n n {r1 , nr1 , . . . , nm1 1 r1 , . . . , rk , nrk , . . . , nmk 1 rk }.

Example 22.5.1 Consider the equation an+2 3an+1 + 2an = 0 with the initial conditions a1 = 1 and a2 = 3. The substitution an = rn yields r2 3r + 2 = (r 1)(r 2) = 0. Thus the general solution is an = c1 1n + c2 2n . 1175

The initial conditions give the two equations, a1 = 1 = c1 + 2c2 a2 = 3 = c1 + 4c2 Since c1 = 1 and c2 = 1, the solution to the dierence equation subject to the initial conditions is an = 2n 1. Example 22.5.2 Consider the gamblers ruin problem that was introduced in Example 22.1.1. The equation for the probability of the gamblers ruin at n dollars is an = pan+1 + qan1 subject to a0 = 1, aN = 0.

We assume that 0 < p < 1. With the substitution an = rn we obtain r = pr2 + q. The roots of this equation are r= 1 4pq 2p 1 1 4p(1 p) = 2p 1 (1 2p)2 = 2p 1 |1 2p| = . 2p 1

We will consider the two cases p = 1/2 and p = 1/2. 1176

p = 1/2. If p < 1/2, the roots are 1 (1 2p) 2p 1p q r1 = = , r2 = 1. p p r= If p > 1/2 the roots are r= r1 = 1, Thus the general solution for p = 1/2 is 1 (2p 1) 2p q p + 1 r2 = = . p p
n

q . an = c 1 + c 2 p The boundary condition a0 = 1 requires that c1 + c2 = 1. From the boundary condition aN = 0 we have q (1 c2 ) + c2 =0 p 1 c2 = 1 + (q/p)N pN . c2 = N p qN Solving for c1 , c1 = 1 pN pN q N q N c1 = N . p qN 1177
N

Thus we have q N pN an = N + N p qN p qN q p
n

p = 1/2. In this case, the two roots of the polynomial are both 1. The general solution is an = c1 + c2 n. The left boundary condition demands that c1 = 1. From the right boundary condition we obtain 1 + c2 N = 0 1 c2 = . N Thus the solution for this case is an = 1 n . N
N/2 N

As a check that this formula makes sense, we see that for n = N/2 the probability of ruin is 1

= 1. 2

22.6

Reduction of Order
(n + 1)(n + 2)an+2 3(n + 1)an+1 + 2an = 0 for n 0 (22.1)

Consider the dierence equation

We see that one solution to this equation is an = 1/n!. Analogous to the reduction of order for dierential equations, the substitution an = bn /n! will reduce the order of the dierence equation. (n + 1)(n + 2)bn+2 3(n + 1)bn+1 2bn + =0 (n + 2)! (n + 1)! n! bn+2 3bn+1 + 2bn = 0 1178

(22.2)

At rst glance it appears that we have not reduced the order of the equation, but writing it in terms of discrete derivatives D2 bn Dbn = 0 shows that we now have a rst order dierence equation for Dbn . The substitution bn = rn in equation 22.2 yields the algebraic equation r2 3r + 2 = (r 1)(r 2) = 0. Thus the solutions are bn = 1 and bn = 2n . Only the bn = 2n solution will give us another linearly independent solution for an . Thus the second solution for an is an = bn /n! = 2n /n!. The general solution to equation 22.1 is then an = c 1 2n 1 + c2 . n! n!

Result 22.6.1 Let an = sn be a homogeneous solution of a linear dierence equation. The substitution an = sn bn will yield a dierence equation for bn that is of order one less than the equation for an .

1179

22.7

Exercises

Exercise 22.1 Find a formula for the nth term in the Fibonacci sequence 1, 1, 2, 3, 5, 8, 13, . . .. Hint, Solution Exercise 22.2 Solve the dierence equation an+2 = Hint, Solution 2 an , n a1 = a2 = 1.

1180

22.8

Hints

Hint 22.1 The dierence equation corresponding to the Fibonacci sequence is an+2 an+1 an = 0, a1 = a2 = 1.

Hint 22.2 Consider this exercise as two rst order dierence equations; one for the even terms, one for the odd terms.

1181

22.9

Solutions

Solution 22.1 We can describe the Fibonacci sequence with the dierence equation an+2 an+1 an = 0, With the substitution an = rn we obtain the equation r2 r 1 = 0. This equation has the two distinct roots 1+ 5 r1 = , 2 Thus the general solution is an = c 1 From the initial conditions we have c1 r1 +c2 r2 = 1 2 2 c1 r1 +c2 r2 = 1. Solving for c2 in the rst equation, c2 = We substitute this into the second equation.
2 c 1 r1 +

a1 = a2 = 1.

1 5 r2 = . 2
n

1+ 5 2

+ c2

1 5 2

1 (1 c1 r1 ). r2

1 2 (1 c1 r1 )r2 = 1 r2 2 c1 (r1 r1 r2 ) = 1 r2 1182

c1 = = =

2 r1

1 5 2 1+ 5 5 2 1+ 5 2 1+ 5 5 2

1 r2 r1 r2

1 = 5 Substitute this result into the equation for c2 . c2 = 1 r2 1 1 r1 5

1 1+ 5 1 5 2 2 1 5 = 1 5 2 5 1 = 5 2 = 1 5 Thus the nth term in the Fibonacci sequence has the formula 1 an = 5 1+ 5 2
n

1 5

1 5 2

It is interesting to note that although the Fibonacci sequence is dened in terms of integers, one cannot express the formula form the nth element in terms of rational numbers. 1183

Solution 22.2 We can consider 2 an , a1 = a2 = 1 n to be a rst order dierence equation. First consider the odd terms. an+2 = a1 = 1 2 a3 = 1 22 a5 = 31 an = For the even terms, a2 = 1 2 a4 = 2 22 a6 = 42 an = Thus an =
2(n1)/2 (n2)(n4)(1) 2(n2)/2 (n2)(n4)(2)

2(n1)/2 (n 2)(n 4) (1)

2(n2)/2 . (n 2)(n 4) (2)

for odd n for even n.

1184

Chapter 23 Series Solutions of Dierential Equations


Skill beats honesty any day. -?

23.1

Ordinary Points

Big O and Little o Notation. The notation O(z n ) means terms no bigger than z n . This gives us a convenient shorthand for manipulating series. For example, z3 sin z = z + O(z 5 ) 6 1 = 1 + O(z) 1z The notation o(z n ) means terms smaller that z n . For example, cos z = 1 + o(1) ez = 1 + z + o(z)

1185

Example 23.1.1 Consider the equation w (z) 3w (z) + 2w(z) = 0. The general solution to this constant coecient equation is w = c1 ez +c2 e2z . The functions ez and e2z are analytic in the nite complex plane. Recall that a function is analytic at a point z0 if and only if the function has a Taylor series about z0 with a nonzero radius of convergence. If we substitute the Taylor series expansions about z = 0 of ez and e2z into the general solution, we obtain

w = c1
n=0

zn 2n z n + c2 . n! n! n=0

Thus we have a series solution of the dierential equation. Alternatively, we could try substituting a Taylor series into the dierential equation and solving for the coecients. Substituting w = an z n into the dierential equation yields n=0 d2 dz 2

d an z 3 dz n=0
n n2

an z + 2
n=0 n=0

an z n = 0

n(n 1)an z
n=2

3
n=1

nan z

n1

+2
n=0

an z n = 0

(n + 2)(n + 1)an+2 z n 3
n=0 n=0

(n + 1)an+1 z n + 2
n=0

an z n = 0

(n + 2)(n + 1)an+2 3(n + 1)an+1 + 2an z n = 0.


n=0

Equating powers of z, we obtain the dierence equation (n + 2)(n + 1)an+2 3(n + 1)an+1 + 2an = 0, 1186 n 0.

We see that an = 1/n! is one solution since n+1 1 13+2 (n + 2)(n + 1) 3 +2 = = 0. (n + 2)! (n + 1)! n! n! We use reduction of order for dierence equations to nd the other solution. Substituting an = bn /n! into the dierence equation yields (n + 2)(n + 1) bn+2 bn+1 bn 3(n + 1) +2 =0 (n + 2)! (n + 1)! n! bn+2 3bn+1 + 2bn = 0.

At rst glance it appears that we have not reduced the order of the dierence equation. However writing this equation in terms of discrete derivatives, D2 bn Dbn = 0 we see that this is a rst order dierence equation for Dbn . Since this is a constant coecient dierence equation we substitute bn = rn into the equation to obtain an algebraic equation for r. r2 3r + 2 = (r 1)(r 2) = 0 Thus the two solutions are bn = 1n b0 and bn = 2n b0 . Only bn = 2n b0 will give us a second independent solution for an . Thus the two solutions for an are a0 2n a0 an = and an = . n! n! Thus we can write the general solution to the dierential equation as

w = c1
n=0

2n z n zn + c2 . n! n! n=0

We recognize these two sums as the Taylor expansions of ez and e2z . Thus we obtain the same result as we did solving the dierential equation directly. 1187

Of course it would be pretty silly to go through all the grunge involved in developing a series expansion of the solution in a problem like Example 23.1.1 since we can solve the problem exactly. However if we could not solve a dierential equation, then having a Taylor series expansion of the solution about a point z0 would be useful in determining the behavior of the solutions near that point. For this method of substituting a Taylor series into the dierential equation to be useful we have to know at what points the solutions are analytic. Lets say we were considering a second order dierential equation whose solutions were 1 w1 = , and w2 = log z. z Trying to nd a Taylor series expansion of the solutions about the point z = 0 would fail because the solutions are not analytic at z = 0. This brings us to two important questions. 1. Can we tell if the solutions to a linear dierential equation are analytic at a point without knowing the solutions? 2. If there are Taylor series expansions of the solutions to a dierential equation, what are the radii of convergence of the series? In order to answer these questions, we will introduce the concept of an ordinary point. Consider the nth order linear homogeneous equation dn w dn1 w dw + pn1 (z) n1 + + p1 (z) + p0 (z)w = 0. n dz dz dz If each of the coecient functions pi (z) are analytic at z = z0 then z0 is an ordinary point of the dierential equation. For reasons of typography we will restrict our attention to second order equations and the point z0 = 0 for a while. The generalization to an nth order equation will be apparent. Considering the point z0 = 0 is only trivially more general as we could introduce the transformation z z0 z to move the point to the origin. In the chapter on rst order dierential equations we showed that the solution is analytic at ordinary points. One would guess that this remains true for higher order equations. Consider the second order equation y + p(z)y + q(z)y = 0, where p and q are analytic at the origin.

p(z) =
n=0

pn z ,

and q(z) =
n=0

qn z n

1188

Assume that one of the solutions is not analytic at the origin and behaves like z at z = 0 where = 0, 1, 2, . . .. That is, we can approximate the solution with w(z) = z + o(z ). Lets substitute w = z + o(z ) into the dierential equation and look at the lowest power of z in each of the terms.

( 1)z

+ o(z

) + z

+ o(z

)
n=0

pn z + z + o(z )
n=0

qn z n = 0.

We see that the solution could not possibly behave like z , = 0, 1, 2, because there is no term on the left to cancel out the z 2 term. The terms on the left side could not add to zero. You could also check that a solution could not possibly behave like log z at the origin. Though we will not prove it, if z0 is an ordinary point of a homogeneous dierential equation, then all the solutions are analytic at the point z0 . Since the solution is analytic at z0 we can expand it in a Taylor series. Now we are prepared to answer our second question. From complex variables, we know that the radius of convergence of the Taylor series expansion of a function is the distance to the nearest singularity of that function. Since the solutions to a dierential equation are analytic at ordinary points of the equation, the series expansion about an ordinary point will have a radius of convergence at least as large as the distance to the nearest singularity of the coecient functions. Example 23.1.2 Consider the equation 1 w + z 2 w = 0. cos z If we expand the solution to the dierential equation in Taylor series about z = 0, the radius of convergence will be at least /2. This is because the coecient functions are analytic at the origin, and the nearest singularities of 1/ cos z are at z = /2. w +

23.1.1

Taylor Series Expansion for a Second Order Dierential Equation


w + p(z)w + q(z)w = 0 1189

Consider the dierential equation

where p(z) and q(z) are analytic in some neighborhood of the origin.

p(z) =
n=0

pn z

and q(z) =
n=0

qn z n

We substitute a Taylor series and its derivatives

w=
n=0

an z n

w =
n=1

nzn z

n1

=
n=0

(n + 1)an+1 z n
n2

w =
n=2

n(n 1)an z

=
n=0

(n + 2)(n + 1)an+2 z n

into the dierential equation to obtain


(n + 2)(n + 1)an+2 z +
n=0 n=0

pn z an z n

n n=0

(n + 1)an+1 z n

+
n=0

qn z n
n=0

=0

(n + 2)(n + 1)an+2 z n +
n=0 n=0 m=0

(m + 1)am+1 pnm z n +
n=0 n m=0

am qnm z n = 0 z n = 0.

(n + 2)(n + 1)an+2 +
n=0 m=0

(m + 1)am+1 pnm + am qnm

1190

Equating coecients of powers of z,


n

(n + 2)(n + 1)an+2 +
m=0

(m + 1)am+1 pnm + am qnm = 0 for n 0.

We see that a0 and a1 are arbitrary and the rest of the coecients are determined by the recurrence relation an+2 = 1 ((m + 1)am+1 pnm + am qnm ) (n + 1)(n + 2) m=0
n

for n 0.

Example 23.1.3 Consider the problem y + 1 y + ex y = 0, cos x

y(0) = y (0) = 1.

Lets expand the solution in a Taylor series about the origin. y(x) =
n=0

an x n

Since y(0) = a0 and y (0) = a1 , we see that a0 = a1 = 1. The Taylor expansions of the coecient functions are 1 = 1 + O(x), cos x Now we can calculate a2 from the recurrence relation. 1 ((m + 1)am+1 p0m + am q0m ) a2 = 1 2 m=0 1 = (1 1 1 + 1 1) 2 = 1 1191
0

and

ex = 1 + O(x).

1.2 1.1 0.2 0.9 0.8 0.7 0.4 0.6 0.8 1 1.2 1.4

Figure 23.1: Plot of the Numerical Solution and the First Three Terms in the Taylor Series. Thus the solution to the problem is y(x) = 1 + x x2 + O(x3 ). In Figure 23.1 the numerical solution is plotted in a solid line and the sum of the rst three terms of the Taylor series is plotted in a dashed line. The general recurrence relation for the an s is useful if you only want to calculate the rst few terms in the Taylor expansion. However, for many problems substituting the Taylor series for the coecient functions into the dierential equation will enable you to nd a simpler form of the solution. We consider the following example to illustrate this point. 1192

Example 23.1.4 Develop a series expansion of the solution to the initial value problem w + (z 2 1 w = 0, + 1) w(0) = 1, w (0) = 0.

Solution using the General Recurrence Relation. The coecient function has the Taylor expansion 1 = 1 + z2

(1)n z 2n .
n=0

From the initial condition we obtain a0 = 1 and a1 = 0. Thus we see that the solution is

w=
n=0

an z n ,

where an+2 and qn = 0 (1)(n/2)

1 am qnm = (n + 1)(n + 2) m=0 for odd n for even n.

Although this formula is ne if you only want to calculate the rst few an s, it is just a tad unwieldy to work with. Lets see if we can get a better expression for the solution. Substitute the Taylor Series into the Dierential Equation. Substituting a Taylor series for w yields d2 dz 2 1 an z + 2 (z + 1) n=0
n

an z n = 0.
n=0

1193

Note that the algebra will be easier if we multiply by z 2 + 1. The polynomial z 2 + 1 has only two terms, but the Taylor series for 1/(z 2 + 1) has an innite number of terms. (z 2 + 1)
n

d2 dz 2

an z n +
n=0 n=0

an z n = 0
n2

n(n 1)an z +
n=2 n=2 n

n(n 1)an z

+
n=0 n

an z n = 0

n(n 1)an z +
n=0 n=0

(n + 2)(n + 1)an+2 z +
n=0

an z n = 0

(n + 2)(n + 1)an+2 + n(n 1)an + an z n = 0


n=0

Equating powers of z gives us the dierence equation an+2 = n2 n + 1 an , (n + 2)(n + 1) for n 0.

From the initial conditions we see that a0 = 1 and a1 = 0. All of the odd terms in the series will be zero. For the even terms, it is easier to reformulate the problem with the change of variables bn = a2n . In terms of bn the dierence equation is (2n)2 2n + 1 bn , b0 = 1. bn+1 = (2n + 2)(2n + 1) This is a rst order dierence equation with the solution
n

bn =
j=0

4j 2 2j + 1 (2j + 2)(2j + 1)
2

Thus we have that an =


n/2 j=0 4j 2j+1 (2j+2)(2j+1)

for even n, for odd n.

0 1194

Note that the nearest singularities of 1/(z 2 + 1) in the complex plane are at z = i. Thus the radius of convergence must be at least 1. Applying the ratio test, the series converges for values of |z| such that an+2 z n+2 lim <1 n an z n n2 n + 1 lim |z|2 < 1 n (n + 2)(n + 1) |z|2 < 1. The radius of convergence is 1. The rst few terms in the Taylor expansion are 1 13 6 1 w = 1 z2 + z4 z + . 2 8 240 In Figure 23.2 the plot of the rst two nonzero terms is shown in a short dashed line, the plot of the rst four nonzero terms is shown in a long dashed line, and the numerical solution is shown in a solid line. In general, if the coecient functions are rational functions, that is they are fractions of polynomials, multiplying the equations by the quotient will reduce the algebra involved in nding the series solution. Example 23.1.5 If we were going to nd the Taylor series expansion about z = 0 of the solution to w + z 1 w + w = 0, 1+z 1 z2

we would rst want to multiply the equation by 1 z 2 to obtain (1 z 2 )w + z(1 z)w + w = 0.

1195

0.2 0.9 0.8 0.7 0.6 0.5 0.4 0.3

0.4

0.6

0.8

1.2

Figure 23.2: Plot of the solution and approximations. Example 23.1.6 Find the series expansions about z = 0 of the fundamental set of solutions for w + z 2 w = 0. Recall that the fundamental set of solutions {w1 , w2 } satisfy w1 (0) = 1 w1 (0) = 0 Thus if w1 =
n=0

w2 (0) = 0 w2 (0) = 1.

an z

and

w2 =
n=0

bn z n ,

1196

then the coecients must satisfy a0 = 1, Substituting the Taylor expansion w =

a1 = 0,
n n=0 cn z

and

b0 = 0,

b1 = 1.

into the dierential equation,


n2

n(n 1)cn z
n=2

+
n=0

cn z n+2 = 0

(n + 2)(n + 1)cn+2 z n +
n=0 n=2

cn2 z n = 0

2c2 + 6c3 z +
n=2

(n + 2)(n + 1)cn+2 + cn2 z n = 0

Equating coecients of powers of z, z 0 : c2 = 0 z 1 : c3 = 0 z n : (n + 2)(n + 1)cn+2 + cn2 = 0, cn cn+4 = (n + 4)(n + 3) For our rst solution we have the dierence equation a0 = 1, a1 = 0, a2 = 0, a3 = 0, For our second solution, b0 = 0, b1 = 1, b2 = 0, b3 = 0, 1197 bn+4 = bn . (n + 4)(n + 3) an+4 = an . (n + 4)(n + 3)

for n 2

1.5

1.5

0.5

0.5

1 -0.5

6 -0.5

-1

-1

Figure 23.3: The graph of approximations and numerical solution of w1 and w2 .

The rst few terms in the fundamental set of solutions are

w1 = 1

1 8 1 4 z + z , 12 672

w2 = z

1 5 1 9 z + z . 20 1440

In Figure 23.3 the ve term approximation is graphed in a coarse dashed line, the ten term approximation is graphed in a ne dashed line, and the numerical solution of w1 is graphed in a solid line. The same is done for w2 . 1198

Result 23.1.1 Consider the nth order linear homogeneous equation dn1 w dw dn w + pn1 (z) n1 + + p1 (z) + p0 (z)w = 0. dz n dz dz If each of the coecient functions pi (z) are analytic at z = z0 then z0 is an ordinary point of the dierential equation. The solution is analytic in some region containing z0 and can be expanded in a Taylor series. The radius of convergence of the series will be at least the distance to the nearest singularity of the coecient functions in the complex plane.

23.2

Regular Singular Points of Second Order Equations


w + p(z) q(z) w + w = 0. z z0 (z z0 )2

Consider the dierential equation

If z = z0 is not an ordinary point but both p(z) and q(z) are analytic at z = z0 then z0 is a regular singular point of the dierential equation. The following equations have a regular singular point at z = 0.
1 w + z w + z2w = 0

w +

1 w sin z

w =0
1 w z sin z

w zw +

=0

Concerning regular singular points of second order linear equations there is good news and bad news. The Good News. We will nd that with the use of the Frobenius method we can always nd series expansions of two linearly independent solutions at a regular singular point. We will illustrate this theory with several examples. 1199

The Bad News. Instead of a tidy little theory like we have for ordinary points, the solutions can be of several dierent forms. Also, for some of the problems the algebra can get pretty ugly. Example 23.2.1 Consider the equation w + 3(1 + z) w = 0. 16z 2
n=0

We wish to nd series solutions about the point z = 0. First we try a Taylor series w = into the dierential equation,

an z n . Substituting this

z2
n=2

n(n 1)an z n2 +
n

3 (1 + z) an z n = 0 16 n=0
n

3 n(n 1)an z + 16 n=0 Equating powers of z, z0 : zn : a0 = 0

3 an z + 16 n=0

an+1 z n = 0.
n=1

n(n 1) + an+1 =

3 3 an + an+1 = 0 16 16

16 n(n 1) + 1 an . 3

This dierence equation has the solution an = 0 for all n. Thus we have obtained only the trivial solution to the dierential equation. We must try an expansion of a more general form. We recall that for regular singular points of rst order equations we can always nd a solution in the form of a Frobenius series w = z an z n , a0 = 0. We n=0 1200

substitute this series into the dierential equation.

2 n=0

( 1) + 2n + n(n 1) an z

n+2

3 + (1 + z)z an z n = 0 16 n=0 an z n + 3 16

( 1) + 2n + n(n 1) an z n +
n=0

3 16

an1 z n = 0
n=1

n=0

Equating the z 0 term to zero yields the equation ( 1) + 3 16 a0 = 0.

Since we have assumed that a0 = 0, the polynomial in must be zero. The two roots of the polynomial are 1 = 1+ 1 3/4 3 = , 2 4 2 = 1 1 3/4 1 = . 2 4

Thus our two series solutions will be of the form


w1 = z

3/4 n=0

an z ,

w2 = z

1/4 n=0

bn z n .

Substituting the rst series into the dierential equation,

n=0

3 3 3 an z n + + 2n + n(n 1) + 16 16 16

an1 z n = 0.
n=1

Equating powers of z, we see that a0 is arbitrary and an = 3 an1 16n(n + 1) 1201 for n 1.

This dierence equation has the solution


n

an = a0
j=1

3 16 3 16

3 16j(j + 1)
n n

= a0 = a0

1 j(j + 1) j=1 1 n!(n + 1)! for n 1.

Substituting the second series into the dierential equation,

n=0

3 3 3 + 2n + n(n 1) + bn z n + 16 16 16

bn1 z n = 0.
n=1

We see that the dierence equation for bn is the same as the equation for an . Thus we can write the general solution to the dierential equation as

w = c1 z

3/4

1+
n=1

3 16
3/4

1 zn n!(n + 1)!
1/4

+ c2 z

1/4

1+
n=1

3 16

1 zn n!(n + 1)!

c1 z

+ c2 z

1+
n=1

3 16

1 zn . n!(n + 1)!

23.2.1

Indicial Equation
p(z) q(z) w + 2 w = 0. z z 1202

Now lets consider the general equation for a regular singular point at z = 0 w +

Since p(z) and q(z) are analytic at z = 0 we can expand them in Taylor series.

p(z) =
n=0

pn z ,

q(z) =
n=0

qn z n

Substituting a Frobenius series w = z equation yields


n

n=0

an z n , a0 = 0 and the Taylor series for p(z) and q(z) into the dierential

( + n)( + n 1) an z +
n=0 n=0

pn z

n n=0

( + n)an z

+
n=0

qn z

n n=0

an z n

=0

( + n)2 ( + n) + p0 ( + n) + q0 an z n
n=0

+
n=1

pn z

n n=0

( + n)an z
n1

+
n=1

qn z

n n=0

an z n
n1

=0

( + n) + (p0 1)(n ) + q0 an z +
n=0 n=1 j=0

( + j)aj pnj

z +
n=1 j=0

aj qnj

zn = 0

Equating powers of z, z0 : z : Let I() = 2 + (p0 1) + q0 = 0. 1203


n

2 + (p0 1) + q0 a0 = 0
n1

( + n) + (p0 1)( + n) + q0 an =
j=0

( + j)pnj + qnj aj .

This is known as the indicial equation. The indicial equation gives us the form of the solutions. The equation for a0 is I()a0 = 0. Since we assumed that a0 is nonzero, I() = 0. Let the two roots of I() be 1 and 2 where (1 ) (2 ). Rewriting the dierence equation for an (),
n1

I( + n)an () =
j=0

( + j)pnj + qnj aj () for n 1.

(23.1)

If the roots are distinct and do not dier by an integer then we can use Equation 23.1 to solve for an (1 ) and an (2 ), which will give us the two solutions

w1 = z

1 n=0

an (1 )z ,

and w2 = z

2 n=0

an (2 )z n .

If the roots are not distinct, 1 = 2 , we will only have one solution and will have to generate another. If the roots dier by an integer, 1 2 = N , there is one solution corresponding to 1 , but when we try to solve Equation 23.1 for an (2 ), we will encounter the equation
N 1

I(2 + N )aN (2 ) = I(1 )aN (2 ) = 0 aN (2 ) =


j=0

( + n)pnj + qnj aj (2 ).

If the right side of the equation is nonzero, then aN (2 ) is undened. On the other hand, if the right side is zero then aN (2 ) is arbitrary. The rest of this section is devoted to considering the cases 1 = 2 and 1 2 = N .

23.2.2

The Case: Double Root

Consider a second order equation L[w] = 0 with a regular singular point at z = 0. Suppose the indicial equation has a double root. I() = ( 1 )2 = 0 1204

One solution has the form w1 = z


1

an z n .
n=0

In order to nd the second solution, we will dierentiate with respect to the parameter, . Let an () satisfy Equation 23.1 Substituting the Frobenius expansion into the dierential equation,

L z

n=0

an ()z n = 0.

Setting = 1 will make the left hand side of the equation zero. Dierentiating this equation with respect to , L z an ()z n = 0. n=0 Interchanging the order of dierentiation,

L log z z

n=0

an ()z + z

n=0

dan () n z = 0. d

Since setting = 1 will make the left hand side of this equation zero, the second linearly independent solution is

w2 = log z z

1 n=0

an (1 )z + z

1 n=0

dan () d

zn
=1

w2 = w1 log z + z 1
n=0

an (1 )z n .

1205

Example 23.2.2 Consider the dierential equation w + 1+z w = 0. 4z 2

There is a regular singular point at z = 0. The indicial equation is ( 1) + One solution will have the form w1 = z 1/2
n=0

1 = 4

1 2

= 0.

an z n ,

a0 = 0.

Substituting the Frobenius expansion z into the dierential equation yields

an ()z n
n=0

n=0

1 z 2 w + (1 + z)w = 0 4 1 1 n+ ( 1) + 2n + n(n 1) an ()z + an ()z n+ + 4 n=0 4

an ()z n++1 = 0.
n=0

Divide by z and adjust the summation indices. 1 [( 1) + 2n + n(n 1)] an ()z + 4 n=0
n

1 an ()z + 4 n=0
n

an1 ()z n = 0
n=1

( 1)a0 +

1 a0 + 4 n=1

( 1) + 2n + n(n 1) +

1 1 an () + an1 () z n = 0 4 4

1206

Equating the coecient of z 0 to zero yields I()a0 = 0. Equating the coecients of z n to zero yields the dierence equation ( 1) + 2n + n(n 1) + an () = The rst few an s are a0 , ( 1) + 9 16 a0 , ( 1) + 25 16 ( 1) + 9 16 a0 , . . . 1 1 an () + an1 () = 0 4 4 1 n(n + 1) ( 1) + + an1 (). 4 4 16

Setting = 1/2, the coecients for the rst solution are a0 , The second solution has the form w2 = w1 log z + z Dierentiating the an (), da0 = 0, d da1 () = (2 1)a0 , d da2 () = (2 1) d ( 1) + 9 16 + ( 1) + 25 16 a0 , ...
1/2 n=0

5 a0 , 16

105 a0 , 16

...

an (1/2)z n .

Setting = 1/2 in this equation yields a0 = 0, Thus the second solution is w2 = w1 log z. 1207 a1 (1/2) = 0, a2 (1/2) = 0, ...

The rst few terms in the general solution are (c1 + c2 log z) 1 5 105 2 z+ z 16 16 .

23.2.3

The Case: Roots Dier by an Integer

Consider the case in which the roots of the indicial equation 1 and 2 dier by an integer. (1 2 = N ) Recall the equation that determines an ()
n1

I( + n)an = ( + n)2 + (p0 1)( + n) + q0 an =


j=0

( + j)pnj + qnj aj .

When = 2 the equation for aN is


N 1

I(2 + N )aN (2 ) = 0 aN (2 ) =
j=0

( + j)pN j + qN j aj .

If the right hand side of this equation is zero, then aN is arbitrary. There will be two solutions of the Frobenius form.

w1 = z

1 n=0

an (1 )z

and w2 = z

2 n=0

an (2 )z n .

If the right hand side of the equation is nonzero then aN (2 ) will be undened. We will have to generate the second solution. Let w(z, ) = z
n=0

an ()z n ,

where an () satises the recurrence formula. Substituting this series into the dierential equation yields L[w(z, )] = 0. 1208

We will multiply by ( 2 ), dierentiate this equation with respect to and then set = 2 . This will generate a linearly independent solution. L[( 2 )w(z, )] = L ( 2 )w(z, ) =L ( 2 )z an ()z n n=0

= L log z z

n=0

( 2 )an ()z + z

n=0

d [( 2 )an ()]z n d

Setting = 2 with make this expression zero, thus


2 n=0 n 2 n=0

log z z

lim {( 2 )an ()} z + z

lim

d [( 2 )an ()] z n d

is a solution. Now lets look at the rst term in this solution


n=0

log z z

lim {( 2 )an ()} z n .

The rst N terms in the sum will be zero. That is because a0 , . . . , aN 1 are nite, so multiplying by ( 2 ) and taking the limit as 2 will make the coecients vanish. The equation for aN () is
N 1

I( + N )aN () =
j=0

( + j)pN j + qN j aj ().

1209

Thus the coecient of the N th term is lim ( 2 )aN () = lim = lim


2 +N 1

( 2 ) I( + N )

N 1

( + j)pN j + qN j aj ()
j=0 N 1

( 2 ) ( + N 1 )( + N 2 )

( + j)pN j + qN j aj ()
j=0

Since 1 = 2 + N , lim2

= 1.
N 1

1 (1 2 )

(2 + j)pN j + qN j aj (2 ).
j=0

Using this you can show that the rst term in the solution can be written d1 log z w1 , where d1 is a constant. Thus the second linearly independent solution is

w2 = d1 log z w1 + z 2
n=0

dn z n ,

where d1 and dn = lim


2

1 1 = a0 (1 2 )

N 1

(2 + j)pN j + qN j aj (2 )
j=0

d ( 2 )an () d

for n 0.

1210

Example 23.2.3 Consider the dierential equation w + 1 2 z w + 2 w = 0. z2

The point z = 0 is a regular singular point. In order to nd series expansions of the solutions, we rst calculate the indicial equation. We can write the coecient functions in the form 1 p(z) = (2 + z), z z Thus the indicial equation is 2 + (2 1) + 2 = 0 ( 1)( 2) = 0. The First Solution. The rst solution will have the Frobenius form

and

q(z) 1 = 2 (2). 2 z z

w1 = z

2 n=0

an (1 )z n .

Substituting a Frobenius series into the dierential equation, z 2 w + (z 2 2z)w + 2w = 0


(n + )(n + 1)z n+ + (z 2 2z)


n=0 n=0

(n + )z n+1 + 2
n=0

an z n = 0

[2 3 + 2]a0 +
n=1

(n + )(n + 1)an + (n + 1)an1 2(n + )an + 2an z n = 0.

Equating powers of z, (n + )(n + 1) 2(n + ) + 2 an = (n + 1)an1 an1 . an = n+2 1211

Setting = 1 = 2, the recurrence relation becomes an (1 ) = an1 (1 ) n (1)n = a0 . n!

The rst solution is

w1 = a0
n=0

(1)n n z = a0 ez . n!

The Second Solution. The equation for a1 (2 ) is 0 a1 (2 ) = 2a0 . Since the right hand side of this equation is not zero, the second solution will have the form

w2 = d1 log z w1 + z

2 n=0

lim

d [( 2 )an ()] z n d

First we will calculate d1 as we dened it previously. d1 = The expression for an () is an () = (1)n a0 . ( + n 2)( + n 1) ( 1) 1212 1 1 a0 = 1. a0 2 1

The rst few an () are

a0 1 a0 a2 () = ( 1) a0 a3 () = . ( + 1)( 1) a1 () =

We would like to calculate

dn = lim

d ( 1)an () d 1213

The rst few dn are d ( 1)a0 d d a0 ( 1) d 1 d a0 d d ( 1) d d a0 d a0 ( 1)

d0 = lim = a0

d1 = lim

= lim =0 d2 = lim = lim

= a0 d3 = lim = lim
1

a0 d ( 1) d ( + 1)( 1) d a0 d ( + 1)

3 = a0 . 4 It will take a little work to nd the general expression for dn . We will need the following relations.
n1

(n) = (n 1)!,

(z) = (z)(z),

(n) = +
k=1

1 . k

1214

See the chapter on the Gamma function for explanations of these equations. dn = lim (1)n a0 1 ( + n 2)( + n 1) ( 1) (1)n a0 = lim 1 ( + n 2)( + n 1) () (1)n a0 () = lim 1 ( + n 1) ()() ()( + n 1) = (1)n a0 lim 1 ( + n 1) ( + n 1) ()[() ( + n 1)] = (1)n a0 lim 1 ( + n 1) (1) (n) = (1)n a0 (n 1)! d d d d d d ( 1) = Thus the second solution is

(1)n+1 a0 (n 1)!

n1

k=0

1 k

w2 = log z w1 + z
n=0

(1)n+1 a0 (n 1)!

n1

k=0

1 k

zn.

The general solution is

w = c1 e

c2 log z e

+c2 z
n=0

(1)n+1 (n 1)!

n1

k=0

1 k

zn.

We see that even in problems that are chosen for their simplicity, the algebra involved in the Frobenius method can be pretty involved. 1215

Example 23.2.4 Consider a series expansion about the origin of the equation w + The indicial equation is 2 1 = 0 = 1. Substituting a Frobenius series into the dierential equation,

1z 1 w 2 w = 0. z z

2 n=0

(n + )(n + 1)an z

n2

+ (z z )
n=0

(n + )an z

n1

n=0

an z n = 0

(n + )(n + 1)an z n +
n=0 n=0

(n + )an z n
n=1

(n + 1)an1 z n
n=0

an z n = 0

( 1) + 1 a0 +
n=1

n + )(n + 1)an + (n + 1)an (n + 1)an1 z n = 0.

Equating powers of z to zero, an () = We know that the rst solution has the form

an1 () . n++1

w1 = z
n=0

an z n .

Setting = 1 in the reccurence formula, an = an1 2a0 = . n+2 (n + 2)! 1216

Thus the rst solution is

w1 = z
n=0

2a0 zn (n + 2)!

1 = 2a0 z = 2a0 z

n=0

z n+2 (n + 2)! zn 1z n!

n=0

2a0 z (e 1 z). = z Now to nd the second solution. Setting = 1 in the reccurence formula, an = a0 an1 = . n n!

We see that in this case there is no trouble in dening a2 (2 ). The second solution is a0 w2 = z Thus we see that the general solution is w= c1 z c2 (e 1 z) + ez z z d1 z 1 e +d2 1 + z z .

n=0

zn a0 z e . = n! z

w=

1217

23.3

Irregular Singular Points

If a point z0 of a dierential equation is not ordinary or regular singular, then it is an irregular singular point. At least one of the solutions at an irregular singular point will not be of the Frobenius form. We will examine how to obtain series expansions about an irregular singular point in the chapter on asymptotic expansions.

23.4

The Point at Innity

If we want to determine the behavior of a function f (z) at innity, we can make the transformation = 1/z and examine the point = 0. Example 23.4.1 Consider the behavior of f (z) = sin z at innity. This is the same as considering the point = 0 of sin(1/), which has the series expansion 1

sin

=
n=0

(1)n . (2n + 1)! 2n+1

Thus we see that the point = 0 is an essential singularity of sin(1/). Hence sin z has an essential singularity at z = . Example 23.4.2 Consider the behavior at innity of z e1/z . We make the transformation = 1/z. 1 1 e = Thus z e1/z has a pole of order 1 at innity. 1218

n=0

n n!

In order to classify the point at innity of a dierential equation in w(z), we apply the transformation = 1/z, u() = w(z). We write the derivatives with respect to z in terms of . z= 1

1 d 2 d d = 2 dz d dz = d d d2 = 2 2 2 dz d d 2 d d = 4 2 + 2 3 d d Now we apply the transformation to the dierential equation. w + p(z)w + q(z)w = 0 u + 2 u + p(1/)( 2 )u + q(1/)u = 0 2 p(1/) q(1/) u + u + u=0 2 4
4 3

Example 23.4.3 Classify the singular points of the dierential equation 1 w + w + 2w = 0. z There is a regular singular point at z = 0. To examine the point at innity we make the transformation = 1/z, 1219

u() = w(z). u + 2 1 2 u + 4u = 0 1 2 u + u + 4u = 0

Thus we see that the dierential equation for w(z) has an irregular singular point at innity.

1220

23.5

Exercises

Exercise 23.1 (mathematica/ode/series/series.nb) f (x) satises the Hermite equation df d2 f 2x + 2f = 0. 2 dx dx Construct two linearly independent solutions of the equation as Taylor series about x = 0. For what values of x do the series converge? Show that for certain values of , called eigenvalues, one of the solutions is a polynomial, called an eigenfunction. Calculate the rst four eigenfunctions H0 (x), H1 (x), H2 (x), H3 (x), ordered by degree. Hint, Solution Exercise 23.2 Consider the Legendre equation (1 x2 )y 2xy + ( + 1)y = 0. 1. Find two linearly independent solutions in the form of power series about x = 0. 2. Compute the radius of convergence of the series. Explain why it is possible to predict the radius of convergence without actually deriving the series. 3. Show that if = 2n, with n an integer and n 0, the series for one of the solutions reduces to an even polynomial of degree 2n. 4. Show that if = 2n + 1, with n an integer and n 0, the series for one of the solutions reduces to an odd polynomial of degree 2n + 1. 5. Show that the rst 4 polynomial solutions Pn (x) (known as Legendre polynomials) ordered by their degree and normalized so that Pn (1) = 1 are P0 = 1 P1 = x 1 1 P2 = (3x2 1) P4 = (5x3 3x) 2 2 1221

6. Show that the Legendre equation can also be written as ((1 x2 )y ) = ( + 1)y. Note that two Legendre polynomials Pn (x) and Pm (x) must satisfy this relation for = n and = m respectively. By multiplying the rst relation by Pm (x) and the second by Pn (x) and integrating by parts show that Legendre polynomials satisfy the orthogonality relation
1

Pn (x)Pm (x) dx = 0 if n = m.
1

If n = m, it can be shown that the value of the integral is 2/(2n + 1). Verify this for the rst three polynomials (but you neednt prove it in general). Hint, Solution Exercise 23.3 Find the forms of two linearly independent series expansions about the point z = 0 for the dierential equation w + 1 1z w + 2 w = 0, sin z z

such that the series are real-valued on the positive real axis. Do not calculate the coecients in the expansions. Hint, Solution Exercise 23.4 Classify the singular points of the equation w + Hint, Solution w + 2w = 0. z1

1222

Exercise 23.5 Find the series expansions about z = 0 for w + Hint, Solution Exercise 23.6 Find the series expansions about z = 0 of the fundamental solutions of w + zw + w = 0. Hint, Solution Exercise 23.7 Find the series expansions about z = 0 of the two linearly independent solutions of w + Hint, Solution Exercise 23.8 Classify the singularity at innity of the dierential equation w + 2 3 + 2 z z w + 1 w = 0. z2 1 1 w + w = 0. 2z z 5 z1 w + w = 0. 4z 8z 2

Find the forms of the series solutions of the dierential equation about innity that are real-valued when z is real-valued and positive. Do not calculate the coecients in the expansions. Hint, Solution 1223

Exercise 23.9 Consider the second order dierential equation x where a, b are real constants. 1. Show that x = 0 is a regular singular point. Determine the location of any additional singular points and classify them. Include the point at innity. 2. Compute the indicial equation for the point x = 0. 3. By solving an appropriate recursion relation, show that one solution has the form y1 (x) = 1 + where the notation (a)n is dened by (a)n = a(a + 1)(a + 2) (a + n 1), (a)0 = 1. ax (a)2 x2 (a)n xn + + + + b (b)2 2! (b)n n! dy d2 y + (b x) ay = 0, 2 dx dx

Assume throughout this problem that b = n where n is a non-negative integer. 4. Show that when a = m, where m is a non-negative integer, that there are polynomial solutions to this equation. Compute the radius of convergence of the series above when a = m. Verify that the result you get is in accord with the Frobenius theory. 5. Show that if b = n + 1 where n = 0, 1, 2, . . ., then the second solution of this equation has logarithmic terms. Indicate the form of the second solution in this case. You need not compute any coecients. Hint, Solution 1224

Exercise 23.10 Consider the equation xy + 2xy + 6 ex y = 0. Find the rst three non-zero terms in each of two linearly independent series solutions about x = 0. Hint, Solution

1225

23.6
Hint 23.1 Hint 23.2 Hint 23.3 Hint 23.4 Hint 23.5 Hint 23.6 Hint 23.7 Hint 23.8 Hint 23.9

Hints

Hint 23.10

1226

23.7

Solutions

Solution 23.1 f (x) is a Taylor series about x = 0.

f (x) =
n=0

an x n nan xn1
n=1

f (x) = =
n=0

nan xn1 n(n 1)an xn2


n=2

f (x) = =
n=0

(n + 2)(n + 1)an+2 xn

We substitute the Taylor series into the dierential equation. f (x) 2xf (x) + 2f = 0

(n + 2)(n + 1)an+2 x 2
n=0 n=0

nan x + 2
n=0

an x n

Equating coecients gives us a dierence equation for an : (n + 2)(n + 1)an+2 2nan + 2an = 0 n an+2 = 2 an . (n + 1)(n + 2) 1227

The rst two coecients, a0 and a1 are arbitrary. The remaining coecients are determined by the recurrence relation. We will nd the fundamental set of solutions at x = 0. That is, for the rst solution we choose a0 = 1 and a1 = 0; for the second solution we choose a0 = 0, a1 = 1. The dierence equation for y1 is n an+2 = 2 an , a0 = 1, a1 = 0, (n + 1)(n + 2) which has the solution a2n = The dierence equation for y2 is an+2 = 2 which has the solution a2n = 0, a2n+1 = n an , (n + 1)(n + 2) 2n a0 = 0, a1 = 1, 2n
n k=0 (2(n

k) ) , (2n)!

a2n+1 = 0.

k) 1 ) . (2n + 1)! A set of linearly independent solutions, (in fact the fundamental set of solutions at x = 0), is

n1 k=0 (2(n

y1 (x) =
n=0

2n

n k=0 (2(n

k) ) 2n x , (2n)!

y2 (x) =
n=0

2n

n1 k=0 (2(n

k) 1 ) 2n+1 x . (2n + 1)!

Since the coecient functions in the dierential equation do not have any singularities in the nite complex plane, the radius of convergence of the series is innite. If = n is a positive even integer, then the rst solution, y1 , is a polynomial of order n. If = n is a positive odd integer, then the second solution, y2 , is a polynomial of order n. For = 0, 1, 2, 3, we have H0 (x) = 1 H1 (x) = x H2 (x) = 1 2x2 2 H3 (x) = x x3 3 1228

Solution 23.2 1. First we write the dierential equation in the standard form. 1 x2 y 2xy + ( + 1)y = 0 (23.2)

2x ( + 1) y + y = 0. (23.3) 2 1x 1 x2 Since the coecients of y and y are analytic in a neighborhood of x = 0, We can nd two Taylor series solutions about that point. We nd the Taylor series for y and its derivatives. y

y=
n=0

an x n nan xn1
n=1

y = y =
n=2

(n 1)nan xn2 (n + 1)(n + 2)an+2 xn


n=0

Here we used index shifting to explicitly write the two forms that we will need for y . Note that we can take the lower bound of summation to be n = 0 for all above sums. The terms added by this operation are zero. We substitute the Taylor series into Equation 23.2.

(n + 1)(n + 2)an+2 x
n=0 n=0

(n 1)nan x 2
n=0

nan x + ( + 1)
n=0

an x n = 0

(n + 1)(n + 2)an+2 (n 1)n + 2n ( + 1) an xn = 0


n=0

1229

We equate coecients of xn to obtain a recurrence relation. (n + 1)(n + 2)an+2 = (n(n + 1) ( + 1))an n(n + 1) ( + 1) an+2 = an , n 0 (n + 1)(n + 2) We can solve this dierence equation to determine the an s. (a0 and a1 are arbitrary.) a0 n2 k(k + 1) ( + 1) , even n, n! k=0 even k an = n2 a1 k(k + 1) ( + 1) , odd n n! k=1
odd k

We will nd the fundamental set of solutions at x = 0, that is the set {y1 , y2 } that satises y1 (0) = 1 y1 (0) = 0 y2 (0) = 0 y2 (0) = 1. For y1 we take a0 = 1 and a1 = 0; for y2 we take a0 = 0 and a1 = 1. The rest of the coecients are determined from the recurrence relation.

y1 =
n=0 even n

1 n! 1 n!

n2

k(k + 1) ( + 1) xn
k=0 even k n2

k(k + 1) ( + 1) xn

y2 =
n=1 odd n

k=1 odd k

1230

2. We determine the radius of convergence of the series solutions with the ratio test. lim an+2 xn+2 <1 an x n <1

lim

n(n+1)(+1) an xn+2 (n+1)(n+2) an x n

lim

n(n + 1) ( + 1) 2 x <1 (n + 1)(n + 2) x2 < 1

Thus we see that the radius of convergence of the series is 1. We knew that the radius of convergence would be at least one, because the nearest singularities of the coecients of (23.3) occur at x = 1, a distance of 1 from the origin. This implies that the solutions of the equation are analytic in the unit circle about x = 0. The radius of convergence of the Taylor series expansion of an analytic function is the distance to the nearest singularity. 3. If = 2n then a2n+2 = 0 in our rst solution. From the recurrence relation, we see that all subsequent coecients are also zero. The solution becomes an even polynomial.
2n

1 m!

m2

k(k + 1) ( + 1) xm

y1 =
m=0 even m

k=0 even k

4. If = 2n + 1 then a2n+3 = 0 in our second solution. From the recurrence relation, we see that all subsequent coecients are also zero. The solution becomes an odd polynomial.
2n+1

1 m!

m2

k(k + 1) ( + 1) xm

y2 =
m=1 odd m

k=1 odd k

1231

Figure 23.4: The First Four Legendre Polynomials 5. From our solutions above, the rst four polynomials are 1 x 1 3x2 5 x x3 3 To obtain the Legendre polynomials we normalize these to have value unity at x = 1 P0 = 1 P1 = x 1 P2 = 3x2 1 2 1 P3 = 5x3 3x 2 These four Legendre polynomials are plotted in Figure 23.4. 1232

6. We note that the rst two terms in the Legendre equation form an exact derivative. Thus the Legendre equation can also be written as (1 x2 )y = ( + 1)y. Pn and Pm are solutions of the Legendre equation. (1 x2 )Pn = n(n + 1)Pn , (1 x2 )Pm = m(m + 1)Pm (23.4)

We multiply the rst relation of Equation 23.4 by Pm and integrate by parts. (1 x2 )Pn Pm = n(n + 1)Pn Pm
1 1

(1 x )Pn Pm dx = n(n + 1)
1 1 1 1 1

Pn Pm dx
1

(1 x2 )Pn Pm
1

1 2

(1 x2 )Pn Pm dx = n(n + 1)
1 1

Pn Pm dx

(1 x )Pn Pm dx = n(n + 1)
1 1

Pn Pm dx

We multiply the secord relation of Equation 23.4 by Pn and integrate by parts. To obtain a dierent expression 1 for 1 (1 x2 )Pm Pn dx.
1 1

(1 x2 )Pm Pn dx = m(m + 1)
1 1 1 (1 1

Pm Pn dx

We equate the two expressions for

x2 )Pm Pn dx. to obtain an orthogonality relation.


1

(n(n + 1) m(m + 1))


1 1

Pn Pm dx = 0

Pn (x)Pm (x) dx = 0 if n = m.
1

1233

We verify that for the rst four polynomials the value of the integral is 2/(2n + 1) for n = m.
1 1

P0 (x)P0 (x) dx =
1 1 1 2 1

1 dx = 2
1

x3 P1 (x)P1 (x) dx = x dx = 3 1 1
1 1

=
1 5

2 3
1

P2 (x)P2 (x) dx =
1 1 1 1

1 1 9x4 6x2 + 1 dx = 4 4

9x 2x3 + x 5
7

=
1 1

2 5 = 2 7

P3 (x)P3 (x) dx =
1 1

1 1 25x6 30x4 + 9x2 dx = 4 4

25x 6x5 + 3x3 7

Solution 23.3 The indicial equation for this problem is 2 + 1 = 0. Since the two roots 1 = i and 2 = i are distinct and do not dier by an integer, there are two solutions in the Frobenius form. w1 = z i
n=0

an z n ,

w1 = z i
n=0

bn z n

However, these series are not real-valued on the positive real axis. Recalling that z i = ei log z = cos(log z) + i sin(log z), and z i = ei log z = cos(log z) i sin(log z),

we can write a new set of solutions that are real-valued on the positive real axis as linear combinations of w1 and w2 . 1 u1 = (w1 + w2 ), 2

u2 =

1 (w1 w2 ) 2i

u1 = cos(log z)
n=0

cn z n ,

u1 = sin(log z)
n=0

dn z n

1234

Solution 23.4 Consider the equation w + w /(z 1) + 2w = 0. We see that there is a regular singular point at z = 1. All other nite values of z are ordinary points of the equation. To examine the point at innity we introduce the transformation z = 1/t, w(z) = u(t). Writing the derivatives with respect to z in terms of t yields d d d2 d2 d = t2 , = t4 2 + 2t3 . 2 dz dt dz dt dt Substituting into the dierential equation gives us t4 u + 2t3 u u + t2 u + 2u = 0 1/t 1 2 1 2 u + 4 u = 0. t t(1 t) t

Since t = 0 is an irregular singular point in the equation for u(t), z = is an irregular singular point in the equation for w(z). Solution 23.5 Find the series expansions about z = 0 for z1 5 w + w = 0. 4z 8z 2 We see that z = 0 is a regular singular point of the equation. The indicial equation is w + 1 1 2 + = 0 4 8 1 1 + = 0. 2 4 Since the roots are distinct and do not dier by an integer, there will be two solutions in the Frobenius form.

w1 = z

1/4 n=0

an (1 )z ,

w2 = z

1/2 n=0

an (2 )z n

1235

We multiply the dierential equation by 8z 2 to put it in a better form. Substituting a Frobenius series into the dierential equation,

8z 2 8

(n + )(n + 1)an z n+2 + 10z


n=0 n=0

(n + )an z n+1 + (z 1)
n=0

an z n+ an z n .

(n + )(n + 1)an z n + 10
n=0 n=0

(n + )an z n +
n=1

an1 z n
n=0

Equating coecients of powers of z, [8(n + )(n + 1) + 10(n + ) 1] an = an1 an1 . an = 2 + 2(n + ) 1 8(n + ) The First Solution. Setting = 1/4 in the recurrence formula, an1 an (1 ) = 2 + 2(n + 1/4) 1 8(n + 1/4) an1 an (1 ) = . 2n(4n + 3) Thus the rst solution is

w1 = z

1/4 n=0

an (1 )z n = a0 z 1/4 1

1 1 2 z+ z + 14 616

The Second Solution. Setting = 1/2 in the recurrence formula, an1 an = 2 + 2(n 1/2) 1 8(n 1/2) an1 an = 2n(4n 3) 1236

Thus the second linearly independent solution is w2 = z 1/2 Solution 23.6 We consider the series solutions of, w + zw + w = 0. We would like to nd the expansions of the fundamental set of solutions about z = 0. Since z = 0 is a regular point, (the coecient functions are analytic there), we expand the solutions in Taylor series. Dierentiating the series expansions for w(z),

1 1 an (2 )z n = a0 z 1/2 1 z + z 2 + 2 40 n=0

w=
n=0

an z n nan z n1
n=1

w = w =
n=2

n(n 1)an z n2 (n + 2)(n + 1)an+2 z n


n=0

We may take the lower limit of summation to be zero without changing the sums. Substituting these expressions into the dierential equation,

(n + 2)(n + 1)an+2 z +
n=0 n=0

nan z +
n=0

an z n = 0

(n + 2)(n + 1)an+2 + (n + 1)an z n = 0.


n=0

1237

Equating the coecient of the z n term gives us (n + 2)(n + 1)an+2 + (n + 1)an = 0, an an+2 = , n 0. n+2 n0

a0 and a1 are arbitrary. We determine the rest of the coecients from the recurrence relation. We consider the cases for even and odd n separately. a2n = a2n2 2n a2n4 = (2n)(2n 2) a0 (2n)(2n 2) 4 2 a0 = (1)n n , n0 m=1 2m = (1)n a2n+1 = a2n1 2n + 1 a2n3 = (2n + 1)(2n 1)

a1 (2n + 1)(2n 1) 5 3 a1 = (1)n n , n0 m=1 (2m + 1) = (1)n If {w1 , w2 } is the fundamental set of solutions, then the initial conditions demand that w1 = 1 + 0 z + and w2 = 0 + z + . We see that w1 will have only even powers of z and w2 will have only odd powers of z.

w1 =
n=0

(1)n 2n z , n m=1 2m

w2 =
n=0

(1)n z 2n+1 n (2m + 1) m=1

1238

Since the coecient functions in the dierential equation are entire, (analytic in the nite complex plane), the radius of convergence of these series solutions is innite. Solution 23.7

w +

1 1 w + w = 0. 2z z

We can nd the indicial equation by substituting w = z + O(z +1 ) into the dierential equation. 1 ( 1)z 2 + z 2 + z 1 = O(z 1 ) 2 Equating the coecient of the z 2 term,

1 ( 1) + = 0 2 1 = 0, . 2 Since the roots are distinct and do not dier by an integer, the solutions are of the form

w1 =
n=0

an z ,

w2 = z

1/2 n=0

bn z n .

1239

Dierentiating the series for the rst solution,

w1 =
n=0

an z n nan z n1
n=1

w1 = =
n=0

(n + 1)an+1 z n n(n + 1)an+1 z n1 .


n=1

w1 = Substituting this series into the dierential equation,

n(n + 1)an+1 z
n=1

n1

1 + 2z

1 (n + 1)an+1 z + z n=0
n

an z n = 0
n=0

n=1

1 1 1 n(n + 1)an+1 + (n + 1)an+1 + an z n1 + a1 + a0 = 0. 2 2z z

Equating powers of z, z 1 : z n1 a1 + a0 = 0 a1 = 2a0 2 1 an : n+ (n + 1)an+1 + an = 0 an+1 = . 2 (n + 1/2)(n + 1)

We can combine the above two equations for an . an+1 = an , (n + 1/2)(n + 1) 1240 for n 0

Solving this dierence equation for an ,


n1

an = a0
j=0

1 (j + 1/2)(j + 1)
n1

(1)n an = a0 n! Now lets nd the second solution. Dierentiating w2 ,

j=0

1 j + 1/2

w2 =
n=0

(n + 1/2)bn z n1/2 (n + 1/2)(n 1/2)bn z n3/2 .


n=0

w2 =

Substituting these expansions into the dierential equation,

(n + 1/2)(n 1/2)bn z n3/2 +


n=0

1 2

(n + 1/2)bn z n3/2 +
n=0 n=1

bn1 z n3/2 = 0.

Equating the coecient of the z 3/2 term, 1 2 1 2 b0 + 11 b0 = 0, 22

we see that b0 is arbitrary. Equating the other coecients of powers of z, 1 (n + 1/2)(n 1/2)bn + (n + 1/2)bn + bn1 = 0 2 bn1 bn = n(n + 1/2) 1241

Calculating the bn s, b0 1 3 2 b0 b2 = 12 3 5 2 2 (1)n 2n b0 bn = n! 3 5 (2n + 1) b1 = Thus the second solution is


1/2 n=0

w2 = b0 z Solution 23.8

(1)n 2n z n . n! 3 5 (2n + 1)

2 3 1 + 2 w + 2 w = 0. z z z In order to analyze the behavior at innity we make the change of variables t = 1/z, u(t) = w(z) and examine the point t = 0. Writing the derivatives with respect to z in terms if t yields 1 z= t 1 dz = 2 dt t d d = t2 dz dt w + d2 d d = t2 t2 2 dz dt dt 2 d d = t4 2 + 2t3 . dt dt 1242

The equation for u is then t4 u + 2t3 u + (2t + 3t2 )(t2 )u + t2 u = 0 1 u + 3u + 2 u = 0 t We see that t = 0 is a regular singular point. To nd the indicial equation, we substitute u = t + O(t+1 ) into the dierential equation. ( 1)t2 3t1 + t2 = O(t1 ) Equating the coecients of the t2 terms, ( 1) + 1 = 0 1i 3 = 2 Since the roots of the indicial equation are distinct and do not dier by an integer, a set of solutions has the form t(1+i Noting that t
(1+i 3)/2 3)/2 n=0

an tn ,

t(1i

3)/2 n=0

bn tn

=t

1/2

exp

i 3 log t , 2

and t

(1i 3)/2

=t

1/2

i 3 exp log t . 2

We can take the sum and dierence of the above solutions to obtain the form 3 3 u1 = t1/2 cos log t log t an tn , u1 = t1/2 sin 2 2 n=0 1243

bn tn .
n=0

Putting the answer in terms of z, we have the form of the two Frobenius expansions about innity. an bn 3 3 1/2 1/2 w1 = z cos log z , w1 = z sin log z . n 2 z 2 zn n=0 n=0 Solution 23.9 1. We write the equation in the standard form. y + a bx y y=0 x x

a Since bx has no worse than a rst order pole and x has no worse than a second order pole at x = 0, that is a x regular singular point. Since the coecient functions have no other singularities in the nite complex plane, all the other points in the nite complex plane are regular points.

Now to examine the point at innity. We make the change of variables u() = y(x), = 1/x. 1 d d u = 2 u = 2 u dx d x d d y = 2 2 u = 4 u + 2 3 u d d y = The dierential equation becomes xy + (b x)y ay 1 4 1 u + 2 3 u + b 2 u au = 0 3 u + (2 b) 2 + u au = 0 2b 1 a u + + 2 3u = 0 Since this equation has an irregular singular point at = 0, the equation for y(x) has an irregular singular point at innity. 1244

2. The coecient functions are

p(x) q(x)

1 x

pn xn =
n=1

1 (b x), x 1 (0 ax). x2

1 x2

qn x n =
n=1

The indicial equation is

2 + (p0 1) + q0 = 0 2 + (b 1) + 0 = 0 ( + b 1) = 0.

3. Since one of the roots of the indicial equation is zero, and the other root is not a negative integer, one of the 1245

solutions of the dierential equation is a Taylor series.

y1 =
k=0

ck xk kck xk1
k=1

y1 = =
k=0

(k + 1)ck+1 xk kck xk1


k=0

= y1 =
k=2

k(k 1)ck xk2 (k + 1)kck+1 xk1


k=1

= =
k=0

(k + 1)kck+1 xk1

We substitute the Taylor series into the dierential equation. xy + (b x)y ay = 0


(k + 1)kck+1 xk + b
k=0 k=0

(k + 1)ck+1 xk
k=0

kck xk a
k=0

ck xk = 0

We equate coecients to determine a recurrence relation for the coecients. (k + 1)kck+1 + b(k + 1)ck+1 kck ack = 0 k+a ck+1 = ck (k + 1)(k + b) 1246

For c0 = 1, the recurrence relation has the solution ck = Thus one solution is

(a)k xk . (b)k k! (a)k k x . (b)k k!

y1 (x) =
k=0

4. If a = m, where m is a non-negative integer, then (a)k = 0 for k > m. This makes y1 a polynomial:
m

y1 (x) =
k=0

(a)k k x . (b)k k!

5. If b = n + 1, where n is a non-negative integer, the indicial equation is ( + n) = 0. For the case n = 0, the indicial equation has a double root at zero. Thus the solutions have the form:
m

y1 (x) =
k=0

(a)k k x , (b)k k!

y2 (x) = y1 (x) log x +


k=0

dk xk

For the case n > 0 the roots of the indicial equation dier by an integer. The solutions have the form:
m

y1 (x) =
k=0

(a)k k x , (b)k k!

y2 (x) = d1 y1 (x) log x + x

n k=0

dk xk

The form of the solution for y2 can be substituted into the equation to determine the coecients dk . 1247

Solution 23.10 We write the equation in the standard form. xy + 2xy + 6 ex y = 0 ex y + 2y + 6 y = 0 x We see that x = 0 is a regular singular point. The indicial equation is 2 = 0 = 0, 1. The rst solution has the Frobenius form. y1 = x + a2 x2 + a3 x3 + O(x4 ) We substitute y1 into the dierential equation and equate coecients of powers of x. xy + 2xy + 6 ex y = 0 x(2a2 + 6a3 x + O(x2 )) + 2x(1 + 2a2 x + 3a3 x2 + O(x3 )) + 6(1 + x + x2 /2 + O(x3 ))(x + a2 x2 + a3 x3 + O(x4 )) = 0 (2a2 x + 6a3 x2 ) + (2x + 4a2 x2 ) + (6x + 6(1 + a2 )x2 ) = O(x3 ) = 0 17 a2 = 4, a3 = 3 17 y1 = x 4x2 + x3 + O(x4 ) 3 Now we see if the second solution has the Frobenius form. There is no a1 x term because y2 is only determined up to an additive constant times y1 . y2 = 1 + O(x2 ) 1248

We substitute y2 into the dierential equation and equate coecients of powers of x. xy + 2xy + 6 ex y = 0 O(x) + O(x) + 6(1 + O(x))(1 + O(x2 )) = 0 6 = O(x) The substitution y2 = 1 + O(x) has yielded a contradiction. Since the second solution is not of the Frobenius form, it has the following form: y2 = y1 ln(x) + a0 + a2 x2 + O(x3 ) The rst three terms in the solution are y2 = a0 + x ln x 4x2 ln x + O(x2 ). We calculate the derivatives of y2 . y2 = ln(x) + O(1) 1 y2 = + O(ln(x)) x We substitute y2 into the dierential equation and equate coecients. xy + 2xy + 6 ex y = 0 (1 + O(x ln x)) + 2 (O(x ln x)) + 6 (a0 + O(x ln x)) = 0 1 + 6a0 = 0 1 y2 = + x ln x 4x2 ln x + O(x2 ) 6

1249

23.8

Quiz
n=1

Problem 23.1 Write the denition of convergence of the series Solution

an .

Problem 23.2 What is the Cauchy convergence criterion for series? Solution Problem 23.3 Dene absolute convergence and uniform convergence. What is the relationship between the two? Solution Problem 23.4 Write the geometric series and the function to which it converges. For what values of the variable does the series converge? Solution Problem 23.5 For what real values of a does the series Solution Problem 23.6 State the ratio and root convergence tests. Solution Problem 23.7 State the integral convergence test. Solution
n=1

na converge?

1250

23.9

Quiz Solutions
N n=1

Solution 23.1 The series an converges if the sequence of partial sums, SN = n=1
N N

an , converges. That is,

lim SN = lim

an = constant.
n=1

Solution 23.2 A series converges if and only if for any > 0 there exists an N such that |Sn Sm | < for all n, m > N . Solution 23.3 The series an converges absolutely if |an | converges. If the rate of convergence of an (z) is indepenn=1 n=1 n=1 dent of z then the series is uniformly convergent. The series is uniformly convergent in a domain if for any given > 0 there exists an N , independent of z, such that
N

|f (z) SN (z)| = f (z)


n=1

an (z) <

for all z in the domain. There is no relationship between absolute convergence and uniform convergence. Solution 23.4 1 = 1z Solution 23.5 The series converges for a < 1. 1251

zn
n=0

for |z| < 1.

Solution 23.6 The series an converges absolutely if n=1


n

lim

an+1 < 1. an

If the limit is greater than unity, then the series diverges. If the limit is unity, the test fails. The series an converges absolutely if n=1
n

lim |an |1/n < 1.

If the limit is greater than unity, then the series diverges. If the limit is unity, the test fails. Solution 23.7 If the coecients an of a series an are monotonically decreasing and can be extended to a monotonically decreasing n=1 function of the continuous variable x: a(x) = an for integer x, then the sum converges or diverges with the integral:

a(x) dx.
1

1252

Chapter 24 Asymptotic Expansions


The more you sweat in practice, the less you bleed in battle. -Navy Seal Saying

24.1
The

Asymptotic Relations
and symbols. First we will introduce two new symbols used in asymptotic relations. f (x) g(x) as x x0 ,

is read, f (x) is much smaller than g(x) as x tends to x0 . This means lim f (x) = 0. g(x)

xx0

The notation f (x) g(x) as x x0 , 1253

is read f (x) is asymptotic to g(x) as x tends to x0 ; which means


xx0

lim

f (x) = 1. g(x)

A few simple examples are ex x as x +

sin x x as x 0 1/x e1/x 1 as x + xn as x 0+ for all n

An equivalent denition of f (x) g(x) as x x0 is f (x) g(x) g(x) as x x0 .

Note that it does not make sense to say that a function f (x) is asymptotic to zero. Using the above denition this would imply f (x) 0 as x x0 . If you encounter an expression like f (x) + g(x) 0, take this to mean f (x) g(x). The Big O and Little o Notation. If |f (x)| m|g(x)| for some constant m in some neighborhood of the point x = x0 , then we say that f (x) = O(g(x)) as x x0 . We read this as f is big O of g as x goes to x0 . If g(x) does not vanish, an equivalent denition is that f (x)/g(x) is bounded as x x0 . If for any given positive there exists a neighborhood of x = x0 in which |f (x)| |g(x)| then f (x) = o(g(x)) as x x0 . This is read, f is little o of g as x goes to x0 . For a few examples of the use of this notation, 1254

ex = o(xn ) as x for any n. sin x = O(x) as x 0. cos x 1 = o(1) as x 0. log x = o(x ) as x + for any positive . Operations on Asymptotic Relations. You can perform the ordinary arithmetic operations on asymptotic relations. Addition, multiplication, and division are valid. You can always integrate an asymptotic relation. Integration is a smoothing operation. However, it is necessary to exercise some care. Example 24.1.1 Consider f (x) This does not imply that 1 x2 as x .

1 as x . x We have forgotten the constant of integration. Integrating the asymptotic relation for f (x) yields f (x) f (x) If c is nonzero then f (x) c as x . It is not always valid to dierentiate an asymptotic relation. Example 24.1.2 Consider f (x) =
1 x

1 + c as x . x

1 x2

sin(x3 ). f (x) 1 x as x .

1255

Dierentiating this relation yields f (x) However, this is not true since f (x) = 1 2 3 sin(x3 ) + 2 cos(x3 ) 2 x x 1 2 as x . x 1 x2 as x .

The Controlling Factor. The controlling factor is the most rapidly varying factor in an asymptotic relation. Consider a function f (x) that is asymptotic to x2 ex as x goes to innity. The controlling factor is ex . For a few examples of this, x log x has the controlling factor x as x . x2 e1/x has the controlling factor e1/x as x 0. x1 sin x has the controlling factor sin x as x . The Leading Behavior. Consider a function that is asymptotic to a sum of terms. f (x) a0 (x) + a1 (x) + a2 (x) + , where a0 (x) a1 (x) a2 (x) , as x x0 . The rst term in the sum is the leading order behavior. For a few examples, For sin x x x3 /6 + x5 /120 as x 0, the leading order behavior is x. For f (x) ex (1 1/x + 1/x2 ) as x , the leading order behavior is ex . 1256 as x x0 .

24.2

Leading Order Behavior of Dierential Equations

It is often useful to know the leading order behavior of the solutions to a dierential equation. If we are considering a regular point or a regular singular point, the approach is straight forward. We simply use a Taylor expansion or the Frobenius method. However, if we are considering an irregular singular point, we will have to be a little more creative. Instead of an all encompassing theory like the Frobenius method which always gives us the solution, we will use a heuristic approach that usually gives us the solution. Example 24.2.1 Consider the Airy equation y = xy. We 1 would like to know how the solutions of this equation behave as x +. First we need to classify the point at innity. The change of variables 1 x= , t yields 1 t4 u + 2t3 u = u t 2 1 u + u 5 u = 0. t t Since the equation for u has an irregular singular point at zero, the equation for y has an irregular singular point at innity.
Using We may be a bit presumptuous on my part. Even if you dont particularly want to know how the solutions behave, I urge you to just play along. This is an interesting section, I promise.
1

y(x) = u(t),

d d = t2 , dx dt

d2 d2 d = t4 2 + 2t3 2 dx dt dt

1257

The Controlling Factor. Since the solutions at irregular singular points often have exponential behavior, we make the substitution y = es(x) into the dierential equation for y. d2 s e = x es dx2 s + (s )2 es = x es s + (s )2 = x

The Dominant Balance. Now we have a dierential equation for s that appears harder to solve than our equation for y. However, we did not introduce the substitution in order to obtain an equation that we could solve exactly. We are looking for an equation that we can solve approximately in the limit as x . If one of the terms in the equation for s is much smaller that the other two as x , then dropping that term and solving the simpler equation may give us an approximate solution. If one of the terms in the equation for s is much smaller than the others then we say that the remaining terms form a dominant balance in the limit as x . Assume that the s term is much smaller that the others, s (s )2 , x as x . This gives us (s )2 x s x 2 s x3/2 as x . 3 Now lets check our assumption that the s term is small. Assuming that we can dierentiate the asymptotic relation s x, we obtain s 1 x1/2 as x . 2 s (s )2 , x x1/2 x as x

Thus we see that the behavior we found for s is consistent with our assumption. The controlling factors for solutions 2 to the Airy equation are exp( 3 x3/2 ) as x . 1258

The Leading Order Behavior of the Decaying Solution. Lets nd the leading order behavior as x goes to 2 innity of the solution with the controlling factor exp( 3 x3/2 ). We substitute 2 s(x) = x3/2 + t(x), 3 into the dierential equation for s. s + (s )2 = x 1 x1/2 + t + (x1/2 + t )2 = x 2 1 t + (t )2 2x1/2 t x1/2 = 0 2 Assume that we can dierentiate t x3/2 to obtain t x1/2 , t x1/2 as x . 2x1/2 t , so we drop the (t )2 term. where t(x) x3/2 as x

1 Since t 2 x1/2 we drop the t term. Also, t This gives us

x1/2 implies that (t )2

1 2x1/2 t x1/2 0 2 1 t x1 4 1 t log x + c 4 1 t log x as x . 4 Checking our assumptions about t, t t x1/2 x1/2 1259 x1 x2 x1/2 x1/2

we see that the behavior of t is consistent with our assumptions. So far we have 2 1 y(x) exp x3/2 log x + u(x) 3 4 where u(x) t(x).

as x ,

log x as x . To continue, we substitute t(x) = 1 log x + u(x) into the dierential equation for 4 1 t + (t )2 2x1/2 t x1/2 = 0 2 2 1 1 2 1 1 x + u + x1 + u 2x1/2 x1 + u x1/2 = 0 4 4 4 2 1 5 u + (u )2 + x1 2x1/2 u + x2 = 0 2 16

Assume that we can dierentiate the asymptotic relation for u to obtain u We know that 1 x1 u 2 x1 , u x2 as x .

2x1/2 u . Using our assumptions, u u x2 x1 u (u )2 5 2 x 0 16 5 2 x 16 5 2 x . 16

Thus we obtain 2x1/2 u + u

5 5/2 x 32 5 u x3/2 + c 48 u c as x . 1260

Since u = c + o(1), eu = ec +o(1). The behavior of y is 2 y x1/4 exp x3/2 (ec +o(1)) as x . 3 Thus the full leading order behavior of the decaying solution is 2 y (const)x1/4 exp x3/2 3 as x .

You can show that the leading behavior of the exponentially growing solution is y (const)x1/4 exp 2 3/2 x 3 as x .

Example 24.2.2 The Modied Bessel Equation. Consider the modied Bessel equation x2 y + xy (x2 + 2 )y = 0. We would like to know how the solutions of this equation behave as x +. First we need to classify the point at innity. The change of variables x = 1 , y(x) = u(t) yields t 1 4 1 (t u + 2t3 u ) + (t2 u ) t2 t 1 1 2 u + u 4+ 2 t t t 1 + 2 u = 0 2 t u=0

Since u(t) has an irregular singular point at t = 0, y(x) has an irregular singular point at innity. 1261

The Controlling Factor. Since the solutions at irregular singular points often have exponential behavior, we make the substitution y = es(x) into the dierential equation for y. x2 (s + (s )2 ) es +xs es (x2 + 2 ) es = 0 2 1 s + (s )2 + s (1 + 2 ) = 0 x x We make the assumption that s (s )2 as x and we know that 2 /x2 two terms from the equation to obtain an approximate equation for s. 1 (s )2 + s 1 0 x This is a quadratic equation for s , so we can solve it exactly. However, let us try to simplify the equation even further. Assume that as x goes to innity one of the three terms is much smaller that the other two. If this is the case, there will be a balance between the two dominant terms and we can neglect the third. Lets check the three possibilities. 1. 1 is small. 1 2.
1 ,0 x2

1 as x . Thus we drop these

1 (s )2 + s 0 x

1 s ,0 x

as x so this balance is inconsistent. 1 s is small. x

(s )2 1 0

s 1

This balance is consistent as 3.

1 x

1 as x . 1 s 10 x

(s )2 is small. This balance is not consistent as x2

s x

1 as x . 1262

The only dominant balance that makes sense leads to s 1 as x . Integrating this relationship, s x + c x as x . Now lets see if our assumption that we made to get the simplied equation for s is valid. Assuming that we can dierentiate s 1, s (s )2 becomes d 1 + o(1) dx 0 + o(1/x) 1 + o(1) 1
2

Thus we see that the behavior we obtained for s is consistent with our initial assumption. We have found two controlling factors, ex and ex . This is a good sign as we know that there must be two linearly independent solutions to the equation. Leading Order Behavior. Now lets nd the full leading behavior of the solution with the controlling factor ex . In order to nd a better approximation for s, we substitute s(x) = x + t(x), where t(x) x as x , into the dierential equation for s. 1 2 s + (s )2 + s 1 + 2 = 0 x x 1 2 t + (1 + t )2 + (1 + t ) 1 + 2 = 0 x x 1 1 2 t + (t )2 + 2 t + =0 x x x2 We know that
1 x

2 and

2 x2

1 x

as x . Dropping these terms from the equation yields t + (t )2 2t 1263 1 0. x

Assuming that we can dierentiate the asymptotic relation for t, we obtain t drop t . Since t vanishes as x goes to innity, (t )2 t . Thus we are left with 2t Integrating this relationship, 1 t log x + c 2 1 log x as x . 2 Checking our assumptions about the behavior of t, t t 1 1 2x 1 2x2 1 1 0, x as x .

1 and t

1 x

as x . We can

1 1 x x we see that the solution is consistent with our assumptions. The leading order behavior to the solution with controlling factor ex is y(x) exp x 1 log x + u(x) 2 = x1/2 ex+u(x) as x ,

where u(x) log x. We substitute t = 1 log x + u(x) into the dierential equation for t in order to nd the 2 asymptotic behavior of u. t + (t )2 + 1 1 +u + +u 2 2x 2x
2

1 2 t x + 1 2 x

1 2 + x x2

=0 1 2 + x x2 =0

1 +u 2x 2 1 u + (u )2 2u + 2 2 = 0 4x x 1264

1 Assuming that we can dierentiate the asymptotic relation for u, u and u x 2 we can neglect the u and (u ) terms. 1 1 2u + 2 0 4 x2

1 x2

as x . Thus we see that

1 1 1 2 2 4 x2 1 1 1 u 2 +c 2 4 x u c as x

Since u = c + o(1), we can expand eu as ec +o(1). Thus we can write the leading order behavior as y x1/2 ex (ec +o(1)). Thus the full leading order behavior is y (const)x1/2 ex as x .

You can verify that the solution with the controlling factor ex has the leading order behavior y (const)x1/2 ex as x .

Two linearly independent solutions to the modied Bessel equation are the modied Bessel functions, I (x) and K (x). These functions have the asymptotic behavior 1 x e , I (x) K (x) ex as x . 2x 2x
In Figure 24.1 K0 (x) is plotted in a solid line and 2x ex is plotted in a dashed line. We see that the leading order behavior of the solution as x goes to innity gives a good approximation to the behavior even for fairly small values of x.

1265

2 1.75 1.5 1.25 1 0.75 0.5 0.25 0 1 2 3 4 5

Figure 24.1: Plot of K0 (x) and its leading order behavior.

24.3

Integration by Parts

Example 24.3.1 The complementary error function

2 erfc(x) = 1266

et dt

is used in statistics for its relation to the normal probability distribution. We would like to nd an approximation to erfc(x) for large x. Using integration by parts,
2 1 2 erfc(x) = 2t et dt 2t x 2 1 t2 2 1 2 t2 e = t e dt 2t x 2 x 1 1 2 2 t2 et dt. = x1 ex x

We examine the residual integral in this expression. 1


x 1 2 2 2t et dt t2 et dt x3 2 x 1 3 x2 = x e . 2

Thus we see that 1 2 x1 ex Therefore, 1 2 erfc(x) x1 ex and we expect that


2 1 x1 ex

t2 et dt as x .

as x ,

would be a good approximation to erfc(x) for large x. In Figure 24.2 log(erfc(x)) is


2

1 graphed in a solid line and log x1 ex is graphed in a dashed line. We see that this rst approximation to the error function gives very good results even for moderate values of x. Table 24.1 gives the error in this rst approximation for various values of x.

1267

2 0.5 -2 -4 -6 -8 -10 1 1.5 2 2.5 3

Figure 24.2: Logarithm of the Approximation to the Complementary Error Function. If we continue integrating by parts, we might get a better approximation to the complementary error function. 1 1 2 2 erfc(x) = x1 ex t2 et dt x 1 1 1 1 3 4 t2 2 2 = x1 ex t3 et + t e dt 2 x 2 x 1 1 1 3 4 t2 2 ex x1 x3 + t e dt = 2 x 2 1 x2 1 1 1 3 3 5 t2 15 6 t2 1 = e x x + t e t e dt 2 4 x 4 x 3 1 1 1 15 6 t2 2 = ex x1 x3 + x5 t e dt 2 4 x 4 1268

x 1 2 3 4 5 6 7 8 9 10

erfc(x) 0.157 0.00468 2.21 105 1.54 108 1.54 1012 2.15 1017 4.18 1023 1.12 1029 4.14 1037 2.09 1045

One Term Relative Error 0.3203 0.1044 0.0507 0.0296 0.0192 0.0135 0.0100 0.0077 0.0061 0.0049 Table 24.1:

Three Term Relative Error 0.6497 0.0182 0.0020 3.9 104 1.1 104 3.7 105 1.5 105 6.9 106 3.4 106 1.8 106

The error in approximating erfc(x) with the rst three terms is given in Table 24.1. We see that for x 2 the three terms give a much better approximation to erfc(x) than just the rst term. At this point you might guess that you could continue this process indenitely. By repeated application of integration by parts, you can obtain the series expansion

2 2 erfc(x) = ex

n=0

(1)n (2n)! . n!(2x)2n+1

1269

This is a Taylor expansion about innity. Lets nd the radius of convergence. an+1 (x) (1)n+1 (2(n + 1))! n!(2x)2n+1 lim < 1 lim <1 n n (n + 1)!(2x)2(n+1)+1 (1)n (2n)! an (x) (2n + 2)(2n + 1) <1 lim n (n + 1)(2x)2 2(2n + 1) lim <1 n (2x)2 1 =0 x Thus we see that our series diverges for all x. Our conventional mathematical sense would tell us that this series is useless, however we will see that this series is very useful as an asymptotic expansion of erfc(x). Say we are working with a convergent series expansion of some function f (x).

f (x) =
n=0

an (x)

For xed x = x0 ,
N

f (x0 )
n=0

an (x0 ) 0 as N .
n=0 bn (x)

For an asymptotic series we have a quite dierent behavior. If g(x) is asymptotic to xed N ,
N

as x x0 then for

g(x)
0

bn (x)

bN (x) as x x0 .

For the complementary error function, 2 2 For xed N , erfc(x) ex


N

n=0

(1)n (2n)! n!(2x)2n+1

x2N 1

as x .

1270

We say that the error function is asymptotic to the series as x goes to innity. 2 2 erfc(x) ex

n=0

(1)n (2n)! n!(2x)2n+1

as x

In Figure 24.3 the logarithm of the dierence between the one term, ten term and twenty term approximations and the complementary error function are graphed in coarse, medium, and ne dashed lines, respectively.

-20

-40

-60

Figure 24.3: log(error in approximation)

*Optimal Asymptotic Series. Of the three approximations, the one term is best for x 2, the ten term is best for 2 x 4, and the twenty term is best for 4 x. This leads us to the concept of an optimal asymptotic 1271

approximation. An optimal asymptotic approximation contains the number of terms in the series that best approximates the true behavior. In Figure 24.4 we see a plot of the number of terms in the approximation versus the logarithm of the error for x = 3. Thus we see that the optimal asymptotic approximation is the rst nine terms. After nine terms the error gets larger. It was inevitable that the error would start to grow after some point as the series diverges for all x.

-12

-14

-16

-18

10

15

20

25

Figure 24.4: The logarithm of the error in using n terms. A good rule of thumb for nding the optimal series is to nd the smallest term in the series and take all of the terms up to but not including the smallest term as the optimal approximation. This makes sense, because the nth term is an approximation of the error incurred by using the rst n 1 terms. In Figure 24.5 there is a plot of n versus the logarithm of the nth term in the asymptotic expansion of erfc(3). We see that the tenth term is the smallest. Thus, in this case, our rule of thumb predicts the actual optimal series.

1272

-12

-14

-16

10

15

20

25

Figure 24.5: The logarithm of the nth term in the expansion for x = 3.

24.4

Asymptotic Series
n=0

A function f (x) has an asymptotic series expansion about x = x0 ,


N

an (x), if for all N.

f (x)
n=0

an (x)

aN (x) as x x0

An asymptotic series may be convergent or divergent. Most of the asymptotic series you encounter will be divergent. If the series is convergent, then we have that
N

f (x)
n=0

an (x) 0 as N for xed x.

1273

Let

n (x)

be some set of gauge functions. The example that we are most familiar with is

n (x)

= xn . If we say that

an n (x)
n=0 n=0

bn n (x),

then this means that an = bn .

24.5
24.5.1

Asymptotic Expansions of Dierential Equations


The Parabolic Cylinder Equation.

Controlling Factor. Let us examine the behavior of the bounded solution of the parabolic cylinder equation as x +. 1 1 y + + x2 y = 0 2 4 This equation has an irregular singular point at innity. With the substitution y = es , the equation becomes s + (s )2 + + We know that + 1 2 1 2 x 4 as x + 1 1 2 x = 0. 2 4

so we drop this term from the equation. Let us make the assumption that s (s )2 as x +. 1274

Thus we are left with the equation (s )2 1 2 x 4 1 s x 2 1 s x2 + c 4 1 s x2 as x + 4

Now lets check if our assumption is consistent. Substituting into s (s )2 yields 1/2 x2 /4 as x + which is true. Since the equation for y is second order, we would expect that there are two dierent behaviors as x +. This is conrmed by the fact that we found two behaviors for s. s x2 /4 corresponds to the solution that is bounded 2 at +. Thus the controlling factor of the leading behavior is ex /4 . Leading Order Behavior. Now we attempt to get a better approximation to s. We make the substitution s = 1 x2 + t(x) into the equation for s where t x2 as x +. 4 1 1 1 1 + t + x2 xt + (t )2 + + x2 = 0 2 4 2 4 t xt + (t )2 + = 0 Since t x2 , we assume that t x and t 1 as x +. Note that this in only an assumption since it is not always valid to dierentiate an asymptotic relation. Thus (t )2 xt and t xt as x +; we drop these terms from the equation. t x t log x + c t log x as x + 1275

Checking our assumptions for the derivatives of t, t x 1 x x t 1 1 x2 1,

we see that they were consistent. Now we wish to rene our approximation for t with the substitution t(x) = log x + u(x). So far we have that y exp x2 x2 + log x + u(x) = x exp + u(x) 4 4 as x +.

We can try and determine u(x) by substituting the expression t(x) = log x + u(x) into the equation for t. 2 2 + u ( + xu ) + 2 + u + (u )2 + = 0 x2 x x

After suitable simplication, this equation becomes u Integrating this asymptotic relation, u
2

2 x3

as x +

2 + c as x +. 2x2

Notice that c as x +; thus this procedure fails to give us the behavior of u(x). Further renements to 2x2 our approximation for s go to a constant value as x +. Thus we have that the leading behavior is y cx exp x2 4 as x +

1276

Asymptotic Expansion Since we have factored o the singular behavior of y, we might expect that what is left over is well behaved enough to be expanded in a Taylor series about innity. Let us assume that we can expand the solution for y in the form y(x) x exp x2 4
x2

(x) = x exp (x),

x2 4

an xn as x +
n=0

where a0 = 1. Dierentiating y = x exp 4

1 2 2 y = x1 x+1 ex /4 (x) + x ex /4 (x) 2 1 1 1 1 2 2 y = ( 1)x2 x ( + 1)x + x+2 ex /4 (x) + 2 x1 x+1 ex /4 (x) 2 2 4 2 + x ex


2 /4

(x).

Substituting this into the dierential equation for y, 1 1 1 1 1 ( 1)x2 ( + ) + x2 (x) + 2 x1 x (x) + (x) + + x2 (x) = 0 2 4 2 2 4 1 2 (x) + (2x x) (x) + ( 1)x = 0 x2 (x) + (2x x3 ) (x) + ( 1)(x) = 0. Dierentiating the expression for (x),

(x) =
n=0

an xn

(x) =
n=1

nan xn1 =
n=1

(n + 2)an+2 xn3

(x) =
n=1

n(n + 1)an xn2 .

1277

Substituting this into the dierential equation for (x),


n(n + 1)an x
n=1

+ 2
n=1

nan x

n=1

(n + 2)an+2 x

+ ( 1)
n=0

an xn = 0.

Equating the coecient of x1 to zero yields a1 x = 0 Equating the coecient of x0 , 2a2 + ( 1)a0 = 0 From the coecient of xn for n > 0, n(n + 1)an 2nan + (n + 2)an+2 + ( 1)an = 0 (n + 2)an+2 = [n(n + 1) 2n + ( 1)]an (n + 2)an+2 = [n2 + n 2n + ( 1)]an (n + 2)an+2 = (n )(n + 1)an . Thus the recursion formula for the an s is an+2 = The rst few terms in (x) are ( 1) 2 ( 1)( 2)( 3) 4 x + x 21 1! 22 2! If we check the radius of convergence of this series (x) 1
n

a1 = 0. 1 a2 = ( 1). 2

(n )(n + 1) an , n+2

a0 = 1,

a1 = 0.

as x +

lim

an+2 xn2 <1 an xn

lim

(n )(n + 1) 2 x <1 n+2

1 =0 x 1278

we see that the radius of convergence is zero. Thus if = 0, 1, 2, . . . our asymptotic expansion for y y x ex
2 /4

( 1) 2 ( 1)( 2)( 3) 4 x + x 21 1! 22 2!

diverges for all x. However this solution is still very useful. If we only use a nite number of terms, we will get a very good numerical approximation for large x. In Figure 24.6 the one term, two term, and three term asymptotic approximations are shown in rough, medium, and ne dashing, respectively. The numerical solution is plotted in a solid line.

1 -2

Figure 24.6: Asymptotic Approximations to the Parabolic Cylinder Function.

1279

Chapter 25 Hilbert Spaces


An expert is a man who has made all the mistakes which can be made, in a narrow eld. - Niels Bohr WARNING: UNDER HEAVY CONSTRUCTION. In this chapter we will introduce Hilbert spaces. We develop the two important examples: l2 , the space of square summable innite vectors and L2 , the space of square integrable functions.

25.1

Linear Spaces

A linear space is a set of elements {x, y, z, . . .} that is closed under addition and scalar multiplication. By closed under addition we mean: if x and y are elements, then z = x + y is an element. The addition is commutative and associative. x+y =y+x (x + y) + z = x + (y + z) 1280

Scalar multiplication is associative and distributive. Let a and b be scalars, a, b C. (ab)x = a(bx) (a + b)x = ax + bx a(x + y) = ax + ay All the linear spaces that we will work with have additional properties: The zero element 0 is the additive identity. x+0=x Multiplication by the scalar 1 is the multiplicative identity. 1x = x Each element x and the additive inverse, x. x + (x) = 0 Consider a set of elements {x1 , x2 , . . .}. Let the ci be scalars. If y = c1 x1 + c2 x2 + then y is a linear combination of the xi . A set of elements {x1 , x2 , . . .} is linearly independent if the equation c1 x1 + c2 x2 + = 0 has only the trivial solution c1 = c2 = = 0. Otherwise the set is linearly dependent. Let {e1 , e2 , } be a linearly independent set of elements. If every element x can be written as a linear combination of the ei then the set {ei } is a basis for the space. The ei are called base elements. x=
i

ci ei

The set {ei } is also called a coordinate system. The scalars ci are the coordinates or components of x. If the set {ei } is a basis, then we say that the set is complete. 1281

25.2

Inner Products

x|y is an inner product of two elements x and y if it satises the properties: 1. Conjugate-commutative. x|y = x|y 2. Linearity in the second argument. x|ay + bz = a x|y + b x|y 3. Positive denite. x|x 0 x|x = 0 if and only if x = 0 From these properties one can derive the properties: 1. Conjugate linearity in the rst argument. ax + by|z = a x|z + b x|z 2. Schwarz Inequality. | x|y |2 x|x y|y One inner product of vectors is the Euclidean inner product.
n

x|y x y =
i=0

xi yi .

One inner product of functions dened on (a . . . b) is


b

u|v =
a

u(x)v(x) dx.

1282

If (x) is a positive-valued function, then we can dene the inner product:


b

u||v =
a

u(x)(x)v(x) dx.

This is called the inner product with respect to the weighting function (x). It is also denoted u|v .

25.3

Norms

A norm is a real-valued function on a space which satises the following properties. 1. Positive. x 0 2. Denite. x = 0 if and only if x = 0 3. Multiplication my a scalar, c C. cx = |c| x 4. Triangle inequality. x+y x + y

Example 25.3.1 Consider a vector space, (nite or innite dimension), with elements x = (x1 , x2 , x3 , . . .). Here are some common norms. Norm generated by the inner product. x = 1283 x|x

The lp norm.
1/p

x There are three common cases of the lp norm. Euclidian norm, or l2 norm.

=
k=1

|xk |

x l1 norm.

=
k=1

|xk |2

x l norm. x

=
k=1

|xk |

= max |xk |
k

Example 25.3.2 Consider a space of functions dened on the interval (a . . . b). Here are some common norms. Norm generated by the inner product. u = The Lp norm.
b 1/p

u|u

=
a

|u(x)| dx

There are three common cases of the Lp norm. Euclidian norm, or L2 norm.
b

=
a

|u(x)|2 dx

1284

L1 norm.
b

u L norm. u

=
a

|u(x)| dx

= lim sup |u(x)|


x(a...b)

Distance. Using the norm, we can dene the distance between elements u and v. d(u, v) u v Note that d(u, v) = 0 does not necessarily imply that u = v. CONTINUE.

25.4 25.5

Linear Independence. Orthogonality


j |k = 0 if j = k

Orthogonality. Orthonormality. j |k = jk Example 25.5.1 Innite vectors. ej has all zeros except for a 1 in the j th position. ej = (0, 0, . . . 0, 1, 0, . . .)

1285

Example 25.5.2 L2 functions on (0 . . . 2). 1 j = ejx , 2 1 0 = , 2 1 (1) j = cos(jx), jZ 1 (1) j = sin(jx), j Z+

25.6

Gramm-Schmidt Orthogonalization

Let {1 (x), . . . , n (x)} be a set of linearly independent functions. Using the Gramm-Schmidt orthogonalization process we can construct a set of orthogonal functions {1 (x), . . . , n (x)} that has the same span as the set of n s with the formulas 1 = 1 1 |2 1 1 2 1 |3 2 |3 3 = 3 1 2 2 1 2 2 2 = 2
n1

n = n
j=1

j |n j . j 2

You could verify that the m are orthogonal with a proof by induction. Example 25.6.1 Suppose we would like a polynomial approximation to cos(x) in the domain [1, 1]. One way to do this is to nd the Taylor expansion of the function about x = 0. Up to terms of order x4 , this is cos(x) = 1 (x)2 (x)4 + + O(x6 ). 2 24 1286

In the rst graph of Figure 25.1 cos(x) and this fourth degree polynomial are plotted. We see that the approximation is very good near x = 0, but deteriorates as we move away from that point. This makes sense because the Taylor expansion only makes use of information about the functions behavior at the point x = 0. As a second approach, we could nd the least squares t of a fourth degree polynomial to cos(x). The set of functions {1, x, x2 , x3 , x4 } is independent, but not orthogonal in the interval [1, 1]. Using Gramm-Schmidt orthogonalization, 0 = 1 1|x =x 1|1 1|x2 x|x2 1 2 = x2 x = x2 1|1 x|x 3 3 3 = x3 x 5 3 6 4 = x4 x2 7 35 1 = x A widely used set of functions in mathematics is the set of Legendre polynomials {P0 (x), P1 (x), . . .}. They dier from the n s that we generated only by constant factors. The rst few are P0 (x) = 1 P1 (x) = x 3x2 1 P2 (x) = 2 5x3 3x P3 (x) = 2 35x4 30x2 + 3 . P4 (x) = 8 1287

Expanding cos(x) in Legendre polynomials


4

cos(x)
n=0

cn Pn (x),

and calculating the generalized Fourier coecients with the formula cn = yields cos(x) 15 45(2 2 21) P2 (x) + P4 (x) 2 4 105 = 4 [(315 30 2 )x4 + (24 2 270)x2 + (27 2 2 )] 8 Pn | cos(x) , Pn |Pn

The cosine and this polynomial are plotted in the second graph in Figure 25.1. The least squares t method uses information about the function on the entire interval. We see that the least squares t does not give as good an approximation close to the point x = 0 as the Taylor expansion. However, the least squares t gives a good approximation on the entire interval. In order to expand a function in a Taylor series, the function must be analytic in some domain. One advantage of using the method of least squares is that the function being approximated does not even have to be continuous.

25.7

Orthonormal Function Expansion

Let {j } be an orthonormal set of functions on the interval (a, b). We expand a function f (x) in the j . f (x) =
j

cj j

1288

1 0.5 -1 -0.5 -0.5 -1 0.5 1 -1 -0.5

1 0.5 0.5 -0.5 -1 1

Figure 25.1: Polynomial Approximations to cos(x).

We choose the coecients to minimize the norm of the error.


2

f
j

cj j

f
j 2

cj j f
j

cj j
j

= f = f

f
j

cj j |cj |2

cj j f

+
j

cj j
j

cj j

cj f |j
j j

cj j |f

f
j

cj j

= f

+
j

|cj |2
j

cj j |f
j

cj j |f

(25.1)

1289

To complete the square, we add the constant f Clearly the unique minimum occurs for

j |f j |f . We see the values of cj which minimize


2

+
j

|cj j |f |2 .

cj = j |f . We substitute this value for cj into the right side of Equation 25.1 and note that this quantity, the squared norm of the error, is non-negative. f
2

+
j

|cj |2
j

|cj |2
j

|cj |2 0

|cj |2

This is known as Bessels Inequality. If the set of {j } is complete then the norm of the error is zero and we obtain Bessels Equality. f 2= |cj |2
j

25.8

Sets Of Functions

Orthogonality. Consider two complex valued functions of a real variable 1 (x) and 2 (x) dened on the interval a x b. The inner product of the two functions is dened
b

1 |2 =
a

1 (x)2 (x) dx. | .

The two functions are orthogonal if 1 |2 = 0. The L2 norm of a function is dened = 1290

Let {1 , 2 , 3 , . . .} be a set of complex valued functions. The set of functions is orthogonal if each pair of functions is orthogonal. That is, n |m = 0 if n = m. If in addition the norm of each function is 1, then the set is orthonormal. That is, n |m = nm = 1 0 if n = m if n = m.

Example 25.8.1 The set of functions 2 sin(x), is orthonormal on the interval [0, ]. To verify this, 2 sin(nx) 2 sin(nx) = 2

2 sin(2x),

2 sin(3x), . . .

sin2 (nx) dx
0

=1 If n = m then 2 sin(nx) 2 sin(mx) 2 = 1 = = 0.

sin(nx) sin(mx) dx
0

(cos[(n m)x] cos[(n + m)x]) dx


0

1291

Example 25.8.2 The set of functions 1 1 1 1 {. . . , ex , , ex , e2x , . . .}, 2 2 2 2 is orthonormal on the interval [, ]. To verify this, 1 1 enx enx 2 2 1 = 2 1 2 = 1. = If n = m then 1 1 enx emx 2 2 1 = 2 1 2 = 0. =

enx enx dx

dx

enx emx dx

e(mn)x dx

Orthogonal with Respect to a Weighting Function. Let (x) be a real-valued, positive function on the interval [a, b]. We introduce the notation
b

n ||m
a

n m dx.

If the set of functions {1 , 2 , 3 , . . .} satisfy n ||m = 0 if n = m 1292

then the functions are orthogonal with respect to the weighting function (x). If the functions satisfy n ||m = nm then the set is orthonormal with respect to (x). Example 25.8.3 We know that the set of functions 2 sin(x), is orthonormal on the interval [0, ]. That is,
0

2 sin(2x),

2 sin(3x), . . .

2 sin(nx)

2 sin(mx) dx = nm .

If we make the change of variables x =


2 0

t in this integral, we obtain 2 sin(n t) 2 sin(m t) dt = nm . 1 sin(3 t), . . .

1 2 t

Thus the set of functions 1 sin( t), is orthonormal with respect to (t) =
1 2 t

1 sin(2 t),

on the interval [0, 2 ].

Orthogonal Series. Suppose that a function f (x) dened on [a, b] can be written as a uniformly convergent sum of functions that are orthogonal with respect to (x).

f (x) =
n=1

cn n (x)

1293

We can solve for the cn by taking the inner product of m (x) and each side of the equation with respect to (x).

m ||f = m ||f =

m
n=1

cn n

cn m ||n
n=1

m ||f = cm m ||m m ||f cm = m ||m

The cm are known as Generalized Fourier coecients. If the functions in the expansion are orthonormal, the formula simplies to

cm = m ||f .

Example 25.8.4 The function f (x) = x( x) has a uniformly convergent series expansion in the domain [0, ] of the form

x( x) =
n=1

cn

2 sin(nx).

1294

The Fourier coecients are

cn = = = = 0 2

2 sin(nx) x( x)

x( x) sin(nx) dx
0

2 2 (1 (1)n ) n3
2 4 n3

for odd n for even n

Thus the expansion is

x( x) =
n=1 oddn

8 sin(nx) for x [0, ]. n3

In the rst graph of Figure 25.2 the rst term in the expansion is plotted in a dashed line and x( x) is plotted in a solid line. The second graph shows the two term approximation. Example 25.8.5 The set {. . . , 1/ 2 ex , 1/ 2, 1/ 2 ex , 1/ 2 e2x , . . .} is orthonormal on the interval [, ]. 1295

2 1

2 1

Figure 25.2: Series Expansions of x( x).

f (x) = sign(x) has the expansion

sign(x)
n=

1 en sign() 2

1 enx 2

= = =

1 2 1 2 1

en sign() d enx
n= 0

en d +
n= 0

en d enx

1 (1)n nx e . n n= 1296

In terms of real functions, this is = 1 1 (1)n (cos(nx) + sin(nx)) n n=


2 =

n=1

1 (1)n sin(nx) n

4 sign(x)

n=1 oddn

1 sin(nx). n

25.9

Least Squares Fit to a Function and Completeness

Let {1 , 2 , 3 , . . .} be a set of real, square integrable functions that are orthonormal with respect to the weighting function (x) on the interval [a, b]. That is, n ||m = nm . Let f (x) be some square integrable function dened on the same interval. We would like to approximate the function f (x) with a nite orthonormal series.
N

f (x)
n=1

n n (x)

f (x) may or may not have a uniformly convergent expansion in the orthonormal functions. We would like to choose the n so that we get the best possible approximation to f (x). The most common measure of how well a series approximates a function is the least squares measure. The error is dened as the integral of the weighting function times the square of the deviation.
b N 2

E=
a

(x) f (x)
n=1

n n (x)

dx

1297

The best t is found by choosing the n that minimize E. Let cn be the Fourier coecients of f (x). cn = n ||f we expand the integral for E.
b N 2

E() =
a

(x) f (x)
n=1 N

n n (x)
N

dx

f
n=1

n n f
n=1 N

n n
N N

= f ||f 2
n=1 N

n n f n n ||f +
n=1 N

+
N n=1 N

n n
n=1

n n

= f ||f 2 = f ||f 2
n=1 N

n m n ||m
n=1 m=1

n cn +
n=1

2 n N 2

= f ||f +
n=1

(n cn )
n=1

c2 n

Each term involving n in non-negative and is minimized for n = cn . The Fourier coecients give the least squares approximation to a function. The least squares t to f (x) is thus
N

f (x)
n=1

n ||f n (x).

1298

Result 25.9.1 If {1 , 2 , 3 , . . .} is a set of real, square integrable functions that are orthogonal with respect to (x) then the least squares t of the rst N orthogonal functions to the square integrable function f (x) is
N

f (x)
n=1

n ||f n (x). n ||n

If the set is orthonormal, this formula reduces to


N

f (x)
n=1

n ||f n (x).

Since the error in the approximation E is a nonnegative number we can obtain on inequality on the sum of the squared coecients.
N

E = f ||f
n=1 N

c2 n

c2 f ||f n
n=1

This equation is known as Bessels Inequality. Since f ||f is just a nonnegative number, independent of N , the sum c2 is convergent and cn 0 as n n=1 n Convergence in the Mean. If the error E goes to zero as N tends to innity
b N N 2

lim

(x) f (x)
a n=1

cn n (x)

dx = 0,

1299

then the sum converges in the mean to f (x) relative to the weighting function (x). This implies that
N N

lim

f ||f
n=1

c2 n

=0

c2 = f ||f . n
n=1

This is known as Parsevals identity.

Completeness. Consider a set of functions {1 , 2 , 3 , . . .} that is orthogonal with respect to the weighting function (x). If every function f (x) that is square integrable with respect to (x) has an orthogonal series expansion

f (x)
n=1

cn n (x)

that converges in the mean to f (x), then the set is complete.

25.10

Closure Relation

Let {1 , 2 , . . .} be an orthonormal, complete set on the domain [a, b]. For any square integrable function f (x) we can write

f (x)
n=1

cn n (x).

1300

Here the cn are the generalized Fourier coecients and the sum converges in the mean to f (x). Substituting the expression for the Fourier coecients into the sum yields

f (x)
n=1

n |f n (x)
b

=
n=1 a

n ()f () d n (x).

Since the sum is not necessarily uniformly convergent, we are not justied in exchanging the order of summation and integration. . . but what the heck, lets do it anyway.
b

=
a b n=1

n ()f ()n (x)

=
a n=1

n ()n (x) f () d

The sum behaves like a Dirac delta function. Recall that (x ) satises the equation
b

f (x) =
a

(x )f () d

for x (a, b).

Thus we could say that the sum is a representation of (x ). Note that a series representation of the delta function could not be convergent, hence the necessity of throwing caution to the wind when we interchanged the summation and integration in deriving the series. The closure relation for an orthonormal, complete set states

n (x)n () (x ).
n=1

1301

Alternatively, you can derive the closure relation by computing the generalized Fourier coecients of the delta function. (x )
n=1

cn n (x)

cn = n |(x )
b

=
a

n (x)(x ) dx

= n ()

(x )
n=1

n (x)n ()

Result 25.10.1 If {1 , 2 , . . .} is an orthogonal, complete set on the domain [a, b], then

n=1

n (x)n () (x ). n 2

If the set is orthonormal, then

n (x)n () (x ).
n=1

Example 25.10.1 The integral of the Dirac delta function is the Heaviside function. On the interval x (, )
x

(t) dt = H(x) =

1 0

for 0 < x < for < x < 0.

1302

Consider the orthonormal, complete set {. . . , 1 ex , 1 , 1 ex , . . .} on the domain [, ]. The delta function 2 2 2 has the series (t) 1 1 1 ent en0 = 2 2 2 n=

ent .
n=

We will nd the series expansion of the Heaviside function rst by expanding directly and then by integrating the expansion for the delta function.

Finding the series expansion of H(x) directly. The generalized Fourier coecients of H(x) are

c0 =

1 H(x) dx 2

1 = 2 = 2

dx
0

cn =

1 enx H(x) dx 2

1 enx dx = 2 0 1 (1)n = . n 2

1303

Thus the Heaviside function has the expansion H(x) 1 1 (1)n 1 nx + e 2 2 n= n 2 2


n=0

1 1 + 2

n=1

1 (1)n sin(nx) n

H(x)

1 2 + 2

n=1 oddn

1 sin(nx). n

Integrating the series for (t). 1 (t) dt 2 =


x x

ent dt
n=

1 1 nt e (x + ) + 2 in n=
n=0

1 1 nx e (1)n (x + ) + 2 n n=
n=0

= =

x 1 1 + + 2 2 2 x 1 1 + + 2 2

n=1

1 nx e enx (1)n + (1)n n

n=1

1 sin(nx) n

1304

Expanding

x 2

in the orthonormal set, x 1 cn enx . 2 n= 2


c0 =

cn =

1 x dx = 0 2 2 x (1)n 1 enx dx = 2 2 n 2

x (1)n 1 nx 1 e = 2 n= n 2 2
n=0

(1)n sin(nx)
n=1

Substituting the series for

x 2

into the expression for the integral of the delta function,


x

(t) dt
x

1 1 + 2

n=1

1 (1)n sin(nx) n

1 2 (t) dt + 2

n=1 oddn

1 sin(nx). n

Thus we see that the series expansions of the Heaviside function and the integral of the delta function are the same.

25.11

Linear Operators

1305

25.12

Exercises

Exercise 25.1 1. Suppose {k (x)} is an orthogonal system on [a, b]. Show that any nite set of the j (x) is a linearly k=0 independent set on [a, b]. That is, if {j1 (x), j2 (x), . . . , jn (x)} is the set and all the j are distinct, then a1 j1 (x) + a2 j2 (x) + + an jn (x) = 0 on a x b is true i: a1 = a2 = = an = 0. 2. Show that the complex functions k (x) ekx/L , k = 0, 1, 2, . . . are orthogonal in the sense that 0, for n = k. Here (x) is the complex conjugate of n (x). n Hint, Solution
L L

k (x) (x) dx = n

1306

25.13
Hint 25.1

Hints

1307

25.14

Solutions

Solution 25.1 1. a1 j1 (x) + a2 j2 (x) + + an jn (x) = 0


n

ak jk (x) = 0
k=1

We take the inner product with j for any = 1, . . . , n. ( ,


n

b a

(x) (x) dx.)

ak jk , j
k=1

=0

We interchange the order of summation and integration.


n

ak jk , j = 0
k=1

jk j = 0 for j = . a j j = 0 j j = 0. a = 0 Thus we see that a1 = a2 = = an = 0. 1308

2. For k = n, k , n = 0.
L

k , n
L L

k (x) (x) dx n ekx/L enx/L dx


L L

= =
L

e(kn)x/L dx
L L

e(kn)x/L = (k n)/L

e(kn) e(kn) = (k n)/L 2L sin((k n)) = (k n) =0

1309

Chapter 26 Self Adjoint Linear Operators


26.1 Adjoint Operators

The adjoint of an operator, L , satises v|Lu L v|u = 0 for all elements u an v. This is known as Greens Identity.

The adjoint of a matrix. For vectors, one can represent linear operators L with matrix multiplication.

Lx Ax 1310

Let B = A be the adjoint of the matrix A. We determine the adjoint of A from Greens Identity. x|Ay Bx|y = 0 x Ay = Bx y xT Ay = Bx y xT Ay = xT B y yT A x = yT BxB = A
T T T T

Thus we see that the adjoint of a matrix is the conjugate transpose of the matrix, A = A . The conjugate transpose is also called the Hermitian transpose and is denoted AH .

The adjoint of a dierential operator. Consider a second order linear dierential operator acting on C 2 functions dened on (a . . . b) which satisfy certain boundary conditions. Lu p2 (x)u + p1 (x)u + p0 (x)u

26.2

Self-Adjoint Operators
T

Matrices. A matrix is self-adjoint if it is equal to its conjugate transpose A = AH A . Such matrices are called Hermitian. For a Hermitian matrix H, Greens identity is y|Hx = Hy|x y Hx = Hy x 1311

The eigenvalues of a Hermitian matrix are real. Let x be an eigenvector with eigenvalue . x|Hx = Hx|x x|x x|x = 0 ( ) x|x = 0 = The eigenvectors corresponding to distinct eigenvalues are distinct. Let x and y be eigenvectors with distinct eigenvalues and . y|Hx = Hy|x y|x y|x = 0 ( ) y|x = 0 ( ) y|x = 0 y|x = 0

Furthermore, all Hermitian matrices are similar to a diagonal matrix and have a complete set of orthogonal eigenvectors.

Trigonometric Series. Consider the problem y = y, y(0) = y(2), 1312 y (0) = y (2).

d We verify that the dierential operator L = dx2 with periodic boundary conditions is self-adjoint.

v|Lu = v| u = [vu ]0 v | u = v |u = vu
2 0 2

v |u

= v |u = Lv|u The eigenvalues and eigenfunctions of this problem are 0 = 0, n = n 2 , (1) = cos(nx), n 0 = 1 (2) = sin(nx), n n Z+

1313

26.3

Exercises

1314

26.4

Hints

1315

26.5

Solutions

1316

Chapter 27 Self-Adjoint Boundary Value Problems


Seize the day and throttle it. -Calvin

27.1

Summary of Adjoint Operators


L[y] = pn dn1 y dn y + pn1 n1 + + p0 y, dxn dx

The adjoint of the operator

dn dn1 (pn y) + (1)n1 n1 (pn1 y) + + p0 y dxn dx If each of the pk is k times continuously dierentiable and u and v are n times continuously dierentiable on some interval, then on that interval Lagranges identity states L [y] = (1)n vL[u] uL [v] = 1317 d B[u, v] dx

is dened

where B[u, v] is the bilinear form


n

B[u, v] =
m=1 j+k=m1 j0,k0

(1)j u(k) (pm v)(j) .

If L is a second order operator then vL[u] uL [v] = u p2 v + u p1 v + u p2 v + (2p2 + p1 )v + (p2 + p1 )v . Integrating Lagranges identity on its interval of validity gives us Greens formula.
b

vL[u] uL [v] dx = v|L[u] L [v]|u = B[u, v]


a

x=b

B[u, v]

x=a

27.2

Formally Self-Adjoint Operators


L[y] = x2 y + 2xy + 3y

Example 27.2.1 The linear operator

has the adjoint operator d2 2 d L [y] = 2 (x y) (2xy) + 3y dx dx = x2 y + 4xy + 2y 2xy 2y + 3y = x2 y + 2xy + 3y.

In Example 27.2.1, the adjoint operator is the same as the operator. If L = L , the operator is said to be formally self-adjoint. 1318

Most of the dierential equations that we study in this book are second order, formally self-adjoint, with real-valued coecient functions. Thus we wish to nd the general form of this operator. Consider the operator L[y] = p2 y + p1 y + p0 y, where the pj s are real-valued functions. The adjoint operator then is L [y] = d2 d (p2 y) (p1 y) + p0 y 2 dx dx = p2 y + 2p2 y + p2 y p1 y p1 y + p0 y = p2 y + (2p2 p1 )y + (p2 p1 + p0 )y.

Equating L and L yields the two equations, 2p2 p1 = p1 , p2 = p1 , p 2 p1 + p0 = p0 p 2 = p1 .

Thus second order, formally self-adjoint operators with real-valued coecient functions have the form L[y] = p2 y + p2 y + p0 y, which is equivalent to the form L[y] = Any linear dierential equation of the form L[y] = y + p1 y + p0 y = f (x), where each pj is j times continuously dierentiable and real-valued, can be written as a formally self adjoint equation. We just multiply by the factor,
x

d (py ) + qy. dx

eP (x) = exp( 1319

p1 () d)

to obtain exp [P (x)] (y + p1 y + p0 y) = exp [P (x)] f (x) d (exp [P (x)] y ) + exp [P (x)] p0 y = exp [P (x)] f (x). dx Example 27.2.2 Consider the equation 1 y + y + y = 0. x Multiplying by the factor exp will make the equation formally self-adjoint. xy + y + xy = 0 d (xy ) + xy = 0 dx
x

1 d

= elog x = x

Result 27.2.1 If L = L then the linear operator L is formally self-adjoint. Second order formally self-adjoint operators have the form L[y] = Any dierential equation of the form L[y] = y + p1 y + p0 y = f (x), where each pj is j times continuously dierentiable and real-valued, can be written as a x formally self adjoint equation by multiplying the equation by the factor exp( p1 () d).
1320

d (py ) + qy. dx

27.3

Self-Adjoint Problems

Consider the nth order formally self-adjoint equation L[y] = 0, on the domain a x b subject to the boundary conditions, Bj [y] = 0 for j = 1, . . . , n. where the boundary conditions can be written
n

Bj [y] =
k=1

jk y (k1) (a) + jk y (k1) (b) = 0.

If the boundary conditions are such that Greens formula reduces to v|L[u] L[v]|u = 0 then the problem is self-adjoint Example 27.3.1 Consider the formally self-adjoint equation y = 0, subject to the boundary conditions y(0) = y() = 0. Greens formula is v| u v |u = [u (v) u(v) ] 0 = [uv u v] 0 = 0.

Thus this problem is self-adjoint.

27.4

Self-Adjoint Eigenvalue Problems


L[y] = 0, subject to Bj [y] = 0, subject to Bj [y] = 0. 1321

Associated with the self-adjoint problem

is the eigenvalue problem L[y] = y,

This is called a self-adjoint eigenvalue problem. The values of for which there exist nontrivial solutions to this problem are called eigenvalues. The functions that satisfy the equation when is an eigenvalue are called eigenfunctions. Example 27.4.1 Consider the self-adjoint eigenvalue problem y = y, subject to y(0) = y() = 0.

First consider the case = 0. The general solution is y = c1 + c2 x. Only the trivial solution satises the boundary conditions. = 0 is not an eigenvalue. Now consider = 0. The general solution is x + c2 sin x . y = c1 cos The solution that satises the left boundary condition is y = c sin For non-trivial solutions, we must have sin = 0, n N. = n2 , Thus the eigenvalues n and eigenfunctions n are n = n 2 , n = sin(nx), for n = 1, 2, 3, . . . x .

Self-adjoint eigenvalue problems have a number a interesting properties. We will devote the rest of this section to developing some of these properties. 1322

Real Eigenvalues. The eigenvalues of a self-adjoint problem are real. Let be an eigenvalue with the eigenfunction . Greens formula states |L[] L[]| = 0 | | = 0 ( ) | = 0 Since 0, | > 0. Thus = and is real. Orthogonal Eigenfunctions. The eigenfunctions corresponding to distinct eigenvalues are orthogonal. Let n and m be distinct eigenvalues with the eigenfunctions n and m . Using Greens formula, n |L[m ] L[n ]|m = 0 n |m m n n |m = 0 (m n ) n |m = 0. Since the eigenvalues are real, (m n ) n |m = 0. Since the two eigenvalues are distinct, n |m = 0 and thus n and m are orthogonal. *Enumerable Set of Eigenvalues. The eigenvalues of a self-adjoint eigenvalue problem form an enumerable set with no nite cluster point. Consider the problem L[y] = y on a x b, subject to Bj [y] = 0.

Let {1 , 2 , . . . , n } be a fundamental set of solutions at x = x0 for some a x0 b. That is, j


(k1)

(x0 ) = jk . 1323

The key to showing that the eigenvalues are enumerable, is that the j are entire functions of . That is, they are analytic functions of for all nite . We will not prove this. The boundary conditions are
n

Bj [y] =
k=1

jk y (k1) (a) + jk y (k1) (b) = 0.


n k=1 ck k

The eigenvalue problem has a solution for a given value of if y = is,


n n

satises the boundary conditions. That

Bj
k=1

ck k =
k=1

ck Bj [k ] = 0 for j = 1, . . . , n.

Dene an n n matrix M such that Mjk = Bk [j ]. Then if c = (c1 , c2 , . . . , cn ), the boundary conditions can be written in terms of the matrix equation M c = 0. This equation has a solution if and only if the determinant of the matrix is zero. Since the j are entire functions of , [M ] is an entire function of . The eigenvalues are real, so [M ] has only real roots. Since [M ] is an entire function, (that is not identically zero), with only real roots, the roots of [M ] can only cluster at innity. Thus the eigenvalues of a self-adjoint problem are enumerable and can only cluster at innity. An example of a function whose roots have a nite cluster point is sin(1/x). This function, (graphed in Figure 27.1), is clearly not analytic at the cluster point x = 0.

Innite Number of Eigenvalues. Though we will not show it, self-adjoint problems have an innite number of eigenvalues. Thus the eigenfunctions form an innite orthogonal set.

Eigenvalues of Second Order Problems. Consider the second order, self-adjoint eigenvalue problem L[y] = (py ) + qy = y, on a x b, 1324 subject to Bj [y] = 0.

-1

Figure 27.1: Graph of sin(1/x). Let n be an eigenvalue with the eigenfunction n . n |L[n ] = n |n n n |(pn ) + qn = n n |n
b

n (pn ) dx + n |q|n = n n |n
a

n pn

b a

n pn dx + n |q|n = n n |n [pn n ]b n |p|n + n |q|n a n |n 1325

n =

Thus we can express each eigenvalue in terms of its eigenfunction. You might think that this formula is just a shade less than worthless. When solving an eigenvalue problem you have to nd the eigenvalues before you determine the eigenfunctions. Thus this formula could not be used to compute the eigenvalues. However, we can often use the formula to obtain information about the eigenvalues before we solve a problem. Example 27.4.2 Consider the self-adjoint eigenvalue problem y = y, The eigenvalues are given by the formula n = n |(1)|n + n |0|n n |n 0 + n |n + 0 = . n |n (1)
b a

y(0) = y() = 0.

We see that n 0. If n = 0 then n |n = 0,which implies that n = const. The only constant that satises the boundary conditions is n = 0 which is not an eigenfunction since it is the trivial solution. Thus the eigenvalues are positive.

27.5

Inhomogeneous Equations
L[y] = 0, Bk [y] = 0,

Let the problem, be self-adjoint. If the inhomogeneous problem, L[y] = f, Bk [y] = 0,

has a solution, then we we can write this solution in terms of the eigenfunction of the associated eigenvalue problem, L[y] = y, Bk [y] = 0.

1326

We denote the eigenvalues as n and the eigenfunctions as n for n Z+ . For the moment we assume that = 0 is not an eigenvalue and that the eigenfunctions are real-valued. We expand the function f (x) in a series of the eigenfunctions. n |f f (x) = fn n (x), fn = n We expand the inhomogeneous solution in a series of eigenfunctions and substitute it into the dierential equation. L[y] = f L yn n (x) = n yn n (x) = yn = The inhomogeneous solution is y(x) = n |f n (x). n n (27.1) fn n fn n (x) fn n (x)

As a special case we consider the Green function problem, L[G] = (x ), We expand the Dirac delta function in an eigenfunction series. (x ) = The Green function is G(x|) = n ()n (x) . n n 1327 n | n (x) = n n ()n (x) n Bk [G] = 0,

We corroborate Equation 27.1 by solving the inhomogeneous equation in terms of the Green function.
b

y=
a b

G(x|)f () d n ()n (x) f () d n n


b a

y=
a

y= y=

n ()f () d n (x) n n n |f n (x) n n

Example 27.5.1 Consider the Green function problem G + G = (x ), First we examine the associated eigenvalue problem. + = , (0) = (1) = 0 + (1 ) = 0, (0) = (1) = 0 n = 1 (n)2 , n = sin(nx), n Z+ We write the Green function as a series of the eigenfunctions.

G(0|) = G(1|) = 0.

G(x|) = 2
n=1

sin(n) sin(nx) 1 (n)2

1328

27.6

Exercises

Exercise 27.1 Show that the operator adjoint to Ly = y (n) + p1 (z)y (n1) + p2 (z)y (n2) + + pn (z)y is given by M y = (1)n u(n) + (1)n1 (p1 (z)u)(n1) + (1)n2 (p2 (z)u)(n2) + + pn (z)u. Hint, Solution

1329

27.7
Hint 27.1

Hints

1330

27.8

Solutions

Solution 27.1 Consider u(x), v(x) C n . (C n is the set of n times continuously dierentiable functions). First we prove the preliminary result n1 d (n) n (n) (1)k u(k) v (nk1) (27.2) uv (1) u v = dx k=0 by simplifying the right side. d dx
n1 n1

(1) u v
k=0

k (k) (nk1)

=
k=0 n1

(1)k u(k) v (nk) + u(k+1) v (nk1)


n1 k (k) (nk)

=
k=0 n1

(1) u v

k=0 n

(1)k+1 u(k+1) v (nk1) (1)k u(k) v (nk)


k=1 n (n) (nn)

=
k=0

(1)k u(k) v (nk)


0 (0) n0

= (1) u v

(1) u

= uv (n) (1)n u(n) v We dene p0 (x) = 1 so that we can write the operators in a nice form.
n n

Ly =
m=0

pm (z)y (nm) ,

Mu =
m=0

(1)m (pm (z)u)(nm)

Now we show that M is the adjoint to L.


n n

uLy yM u = u
m=0 n

pm (z)y

(nm)

y
m=0

(1)m (pm (z)u)(nm)

=
m=0

upm (z)y (nm) (pm (z)u)(nm) y

1331

We use Equation 27.2. d = dz m=0 d uLy yM u = dz


n nm1

(1)k (upm (z))(k) y (nmk1)


k=0 n nm1

(1)k (upm (z))(k) y (nmk1)


m=0 k=0

1332

Chapter 28 Fourier Series


Every time I close my eyes The noise inside me amplies I cant escape I relive every moment of the day Every misstep I have made Finds a way it can invade My every thought And this is why I nd myself awake -Failure -Tom Shear (Assemblage 23)

28.1

An Eigenvalue Problem.
y + y = 0, y() = y(), 1333 y () = y ().

A self adjoint eigenvalue problem. Consider the eigenvalue problem

We rewrite the equation so the eigenvalue is on the right side. L[y] y = y We demonstrate that this eigenvalue problem is self adjoint. v|L[u] L[v]|u = v| u v |u v = [u ] + v |u [ u] v |u v = v()u () + v()u () + v ()u() v ()u() = v()u () + v()u () + v ()u() v ()u() =0 Since Greens Identity reduces to v|L[u] L[v]|u = 0, the problem is self adjoint. This means that the eigenvalues are real and that eigenfunctions corresponding to distinct eigenvalues are orthogonal. We compute the Rayleigh quotient for an eigenvalue with eigenfunction . [ ] + | = | = = () () + () () + | |

() () + () () + | | | = |

We see that the eigenvalues are non-negative. Computing the eigenvalues and eigenfunctions. Now we nd the eigenvalues and eigenfunctions. First we consider the case = 0. The general solution of the dierential equation is y = c1 + c2 x. 1334

The solution that satises the boundary conditions is y = const. Now consider > 0. The general solution of the dierential equation is y = c1 cos x + c2 sin x . We apply the rst boundary condition. y() = y() c1 cos + c2 sin = c1 cos + c2 sin c1 cos c2 sin = c1 cos + c2 sin = 0 c2 sin Then we apply the second boundary condition. y () = y () + c2 cos c1 sin + c2 cos = c1 sin c1 sin + c2 cos = c1 sin + c2 cos c1 sin = 0 To satisify the two boundary conditions either c1 = c2 = 0 or sin = 0. The former yields the trivial solution. The latter gives us the eigenvalues n = n2 , n Z+ . The corresponding solution is yn = c1 cos(nx) + c2 sin(nx). There are two eigenfunctions for each of the positive eigenvalues. We choose the eigenvalues and eigenfunctions. 1 0 = 0, 0 = 2 2 n = n , 2n1 = cos(nx), 2n = sin(nx), 1335

for n = 1, 2, 3, . . .

Orthogonality of Eigenfunctions. We know that the eigenfunctions of distinct eigenvalues are orthogonal. In addition, the two eigenfunctions of each positive eigenvalue are orthogonal. 1 cos(nx) sin(nx) dx = sin2 (nx) 2n

=0

1 Thus the eigenfunctions { 2 , cos(x), sin(x), cos(2x), sin(2x)} are an orthogonal set.

28.2

Fourier Series.

A series of the eigenfunctions 1 0 = , 2 is 1 a0 + an cos(nx) + bn sin(nx) . 2 n=1 This is known as a Fourier series. (We choose 0 = 1 so all of the eigenfunctions have the same norm.) A fairly general 2 class of functions can be expanded in Fourier series. Let f (x) be a function dened on < x < . Assume that f (x) can be expanded in a Fourier series 1 an cos(nx) + bn sin(nx) . f (x) a0 + 2 n=1

(1) = cos(nx), n

(2) = sin(nx), n

for n 1

(28.1)

Here the means has the Fourier series. We have not said if the series converges yet. For now lets assume that the series converges uniformly so we can replace the with an =. 1336

We integrate Equation 28.1 from to to determine a0 . 1 f (x) dx = a0 2


dx +
n=1

an cos(nx) + bn sin(nx) dx

f (x) dx = a0 +
n=1

an

cos(nx) dx + bn

sin(nx) dx

f (x) dx = a0

a0 =

f (x) dx

Multiplying by cos(mx) and integrating will enable us to solve for am . 1 f (x) cos(mx) dx = a0 2 +
n=1

cos(mx) dx

an

cos(nx) cos(mx) dx + bn

sin(nx) cos(mx) dx

All but one of the terms on the right side vanishes due to the orthogonality of the eigenfunctions.

f (x) cos(mx) dx = am

cos(mx) cos(mx) dx 1 + cos(2mx) 2 dx

f (x) cos(mx) dx = am

f (x) cos(mx) dx = am

am =

f (x) cos(mx) dx.

1337

Note that this formula is valid for m = 0, 1, 2, . . .. Similarly, we can multiply by sin(mx) and integrate to solve for bm . The result is bm = 1

f (x) sin(mx) dx.

an and bn are called Fourier coecients. Although we will not show it, Fourier series converge for a fairly general class of functions. Let f (x ) denote the left limit of f (x) and f (x+ ) denote the right limit. Example 28.2.1 For the function dened f (x) = the left and right limits at x = 0 are f (0 ) = 0, f (0+ ) = 1.

0 x+1

for x < 0, for x 0,

Result 28.2.1 Let f (x) be a 2-periodic function for which Fourier coecients 1 an =

|f (x)| dx exists. Dene the

f (x) cos(nx) dx,

1 bn =

f (x) sin(nx) dx.

If x is an interior point of an interval on which f (x) has limited total uctuation, then the Fourier series of f (x) a0 + an cos(nx) + bn sin(nx) , 2 n=1
1 converges to 2 (f (x ) + f (x+ )). If f is continuous at x, then the series converges to f (x).

1338

Periodic Extension of a Function. Let g(x) be a function that is arbitrarily dened on x < . The Fourier series of g(x) will represent the periodic extension of g(x). The periodic extension, f (x), is dened by the two conditions: f (x) = g(x) for x < , f (x + 2) = f (x). The periodic extension of g(x) = x2 is shown in Figure 28.1.

10 8 6 4 2 -5 -2 5 10

Figure 28.1: The Periodic Extension of g(x) = x2 .

Limited Fluctuation. A function that has limited total uctuation can be written f (x) = + (x) (x), where + and are bounded, nondecreasing functions. An example of a function that does not have limited total uctuation 1339

is sin(1/x), whose uctuation is unlimited at the point x = 0. Functions with Jump Discontinuities. Let f (x) be a discontinuous function that has a convergent Fourier series. Note that the series does not necessarily converge to f (x). Instead it converges to f (x) = 1 (f (x ) + f (x+ )). 2 Example 28.2.2 Consider the function dened by f (x) = x 2x for x < 0 for 0 x < .

The Fourier series converges to the function dened by 0 (x) = x f /2 2x The function f (x) is plotted in Figure 28.2. for for for for x = <x<0 x=0 0 < x < .

28.3

Least Squares Fit

Approximating a function with a Fourier series. Suppose we want to approximate a 2-periodic function f (x) with a nite Fourier series. a0 f (x) + (an cos(nx) + bn sin(nx)) 2 n=1 1340
N

3 2 1

-3

-2

-1 -1 -2 -3

Figure 28.2: Graph of f (x). Here the coecients are computed with the familiar formulas. Is this the best approximation to the function? That is, is it possible to choose coecients n and n such that 0 f (x) + (n cos(nx) + n sin(nx)) 2 n=1 would give a better approximation? Least squared error t. The most common criterion for nding the best t to a function is the least squares t. The best approximation to a function is dened as the one that minimizes the integral of the square of the deviation. 1341
N

Thus if f (x) is to be approximated on the interval a x b by a series


N

f (x)
n=1

cn n (x),

(28.2)

the best approximation is found by choosing values of cn that minimize the error E.
b N 2

E
a

f (x)
n=1

cn n (x)

dx

Generalized Fourier coecients. We consider the case that the n are orthogonal. For simplicity, we also assume that the n are real-valued. Then most of the terms will vanish when we interchange the order of integration and summation.
b N N N

E=
a b

f 2f
n=1 N

cn n +
n=1 b

cn n
N m=1 N

cm m
b

dx

E=
a

f 2 dx 2
n=1 b

cn
a N

f n dx +
b n=1 m=1 N

cn cm
a b

n m dx 2 dx n

E=
a b

f 2 dx 2
n=1 N

cn
a

f n dx +
n=1 b

c2 n
a b

E=
a

f 2 dx +
n=1

c2 n
a

2 dx 2cn n
a

f n dx

We complete the square for each term.


b N

E=
a

f 2 dx +
n=1

2 dx cn n

b f n dx a b 2 dx a n

b f n dx a b 2 dx a n

1342

Each term involving cn is non-negative, and is minimized for cn = We call these the generalized Fourier coecients. For such a choice of the cn , the error is
b N b b f n dx a . b 2 n dx a

(28.3)

E=
a

f dx
n=1

c2 n
a

2 dx. n

Since the error is non-negative, we have


b N b

f 2 dx
a n=1

c2 n
a

2 dx. n

This is known as Bessels Inequality. If the series in Equation 28.2 converges in the mean to f (x), lim N E = 0, then we have equality as N .
b b

f 2 dx =
a n=1

c2 n
a

2 dx. n

This is Parsevals equality. Fourier coecients. Previously we showed that if the series, a0 f (x) = + (an cos(nx) + bn sin(nx), 2 n=1 converges uniformly then the coecients in the series are the Fourier coecients, 1 an =

f (x) cos(nx) dx,

1 bn =

f (x) sin(nx) dx.

1343

Now we show that by choosing the coecients to minimize the squared error, we obtain the same result. We apply Equation 28.3 to the Fourier eigenfunctions. a0 = an = bn =
f 1 dx 2 1 dx 4

1 = = = 1 1

f (x) dx

f cos(nx) dx cos2 (nx) dx f sin(nx) dx sin2 (nx) dx

f (x) cos(nx) dx

f (x) sin(nx) dx

28.4

Fourier Series for Functions Dened on Arbitrary Ranges

If f (x) is dened on c d x < c + d and f (x + 2d) = f (x), then f (x) has a Fourier series of the form f (x) Since
c+d

a0 + an cos 2 n=1 n(x + c) d

n(x + c) d
c+d

+ bn sin

n(x + c) d

cos2
cd

dx =
cd

sin2

n(x + c) d

dx = d,

the Fourier coecients are given by the formulas an = bn = 1 d 1 d


c+d

f (x) cos
cd c+d

n(x + c) d n(x + c) d

dx dx.

f (x) sin
cd

1344

Example 28.4.1 Consider the function dened by x + 1 f (x) = x 3 2x

for 1 x < 0 for 0 x < 1 for 1 x < 2.

This function is graphed in Figure 28.3. The Fourier series converges to f (x) = (f (x ) + f (x+ ))/2, 1 for 2 x + 1 for (x) = 1 f for 2 x for 3 2x for f (x) is also graphed in Figure 28.3. The Fourier coecients are an = = 1 3/2 2 3
2

x = 1 1<x<0 x=0 0<x<1 1 x < 2.

f (x) cos
1 5/2

2n(x + 1/2) 3 2nx 3 2nx 3

dx dx

f (x 1/2) cos
1/2 1/2

2 = 3

(x + 1/2) cos
1/2

2 dx + 3 2nx 3

3/2

(x 1/2) cos
1/2

2nx 3

dx

2 + 3 = 1 sin (n)2

5/2

(4 2x) cos
3/2

dx n 3

2n 3

2(1)n n + 9 sin

1345

0.5

0.5

-1

-0.5

0.5

1.5

-1

-0.5

0.5

1.5

-0.5

-0.5

-1

-1

Figure 28.3: A Function Dened on the range 1 x < 2 and the Function to which the Fourier Series Converges. bn = = = 1 3/2 2 3 2 3
2

f (x) sin
1 5/2

2n(x + 1/2) 3 2nx 3 2nx 3

dx dx

f (x 1/2) sin
1/2 1/2

(x + 1/2) sin
1/2

dx + 2nx 3

2 3

3/2

(x 1/2) sin
1/2

2nx 3

dx

+ =

2 3

5/2

(4 2x) sin
3/2

dx n n 3 sin 3 3

n 2 sin2 2 (n) 3

2(1)n n + 4n cos 1346

28.5

Fourier Cosine Series

If f (x) is an even function, (f (x) = f (x)), then there will not be any sine terms in the Fourier series for f (x). The Fourier sine coecient is 1 bn = f (x) sin(nx) dx. Since f (x) is an even function and sin(nx) is odd, f (x) sin(nx) is odd. bn is the integral of an odd function from to and is thus zero. We can rewrite the cosine coecients, an = 1 2 =

f (x) cos(nx) dx

f (x) cos(nx) dx.


0

Example 28.5.1 Consider the function dened on [0, ) by f (x) = x x for 0 x < /2 for /2 x < .

The Fourier cosine coecients for this function are an = = 2


4 8 n2 /2

x cos(nx) dx +
0

( x) cos(nx) dx
/2

cos

n 2

sin2

n 4

for n = 0, for n 1.

In Figure 28.4 the even periodic extension of f (x) is plotted in a dashed line and the sum of the rst ve nonzero terms in the Fourier cosine series are plotted in a solid line.

1347

1.5 1.25 1 0.75 0.5 0.25

-3

-2

-1

Figure 28.4: Fourier Cosine Series.

28.6

Fourier Sine Series

If f (x) is an odd function, (f (x) = f (x)), then there will not be any cosine terms in the Fourier series. Since f (x) cos(nx) is an odd function, the cosine coecients will be zero. Since f (x) sin(nx) is an even function,we can rewrite the sine coecients bn = 2

f (x) sin(nx) dx.


0

1348

Example 28.6.1 Consider the function dened on [0, ) by f (x) = x x for 0 x < /2 for /2 x < .

The Fourier sine coecients for this function are 2 2 /2 x sin(nx) dx + ( x) sin(nx) dx bn = 0 /2 n n 16 = cos sin3 2 n 4 4 In Figure 28.5 the odd periodic extension of f (x) is plotted in a dashed line and the sum of the rst ve nonzero terms in the Fourier sine series are plotted in a solid line.

28.7

Complex Fourier Series and Parsevals Theorem


a0 a0 1 1 + an cos(nx) + bn sin(nx) = + an (enx + enx ) + bn (enx enx ) 2 2 2 2 n=1 n=1 a0 = + 2 n=1

By writing sin(nx) and cos(nx) in terms of enx and enx we can obtain the complex form for a Fourier series.

1 1 (an bn ) enx + (an + bn ) enx 2 2

=
n=

cn enx

where cn =

1 2 (an bn )
a0 12 (a 2 n

+ bn ) 1349

for n 1 for n = 0 for n 1.

1.5 1 0.5

-3

-2

-1 -0.5 -1 -1.5

Figure 28.5: Fourier Sine Series. The functions {. . . , ex , 1, ex , e2x , . . .}, satisfy the relation

enx emx dx =

e(nm)x dx 2 0 for n = m for n = m.

Starting with the complex form of the Fourier series of a function f (x),

f (x)

cn enx ,

1350

we multiply by emx and integrate from to to obtain


f (x) e

mx

dx =

cn enx emx dx f (x) emx dx

cm = If f (x) is real-valued then cm 1 = 2

1 2

f (x) emx dx =

1 2

f (x)(emx ) dx = cm

where z denotes the complex conjugate of z. Assume that f (x) has a uniformly convergent Fourier series.
2

f (x) dx =
m=

cm e cn cn
n= 1

mx n=

cn enx

dx

= 2

= 2
n=

1 a0 a0 1 (an + bn )(an bn ) + + (an bn )(an + bn ) 4 2 2 4 n=1

= 2

a2 1 0 + 4 2

(a2 + b2 ) n n
n=1

This yields a result known as Parsevals theorem which holds even when the Fourier series of f (x) is not uniformly convergent. 1351

Result 28.7.1 Parsevals Theorem. If f (x) has the Fourier series a0 f (x) + (an cos(nx) + bn sin(nx)), 2 n=1 then f (x) dx = a2 + (a2 + b2 ). n n 2 0 n=1
2

28.8

Behavior of Fourier Coecients

Before we jump hip-deep into the grunge involved in determining the behavior of the Fourier coecients, lets take a step back and get some perspective on what we should be looking for. One of the important questions is whether the Fourier series converges uniformly. From Result 12.2.1 we know that a uniformly convergent series represents a continuous function. Thus we know that the Fourier series of a discontinuous function cannot be uniformly convergent. From Section 12.2 we know that a series is uniformly convergent if it can be bounded by a series of positive terms. If the Fourier coecients, an and bn , are O(1/n ) where > 1 then the series can be bounded by (const) 1/n and will thus be uniformly convergent. n=1 Let f (x) be a function that meets the conditions for having a Fourier series and in addition is bounded. Let (, p1 ), (p1 , p2 ), (p2 , p3 ), . . . , (pm , ) be a partition into a nite number of intervals of the domain, (, ) such that on each interval f (x) and all its derivatives are continuous. Let f (p ) denote the left limit of f (p) and f (p+ ) denote the right limit. f (p ) = lim f (p ), +
0

f (p+ ) = lim f (p + ) +
0

1352

Example 28.8.1 The function shown in Figure 28.6 would be partitioned into the intervals (2, 1), (1, 0), (0, 1), (1, 2).

0.5

-2

-1 -0.5

-1

Figure 28.6: A Function that can be Partitioned.

Suppose f (x) has the Fourier series a0 f (x) + an cos(nx) + bn sin(nx). 2 n=1 1353

We can use the integral formula to nd the an s. 1 an = 1 =

f (x) cos(nx) dx
p1 p2

f (x) cos(nx) dx +
p1

f (x) cos(nx) dx + +
pm

f (x) cos(nx) dx

Using integration by parts, = 1 n


p1 p2

f (x) sin(nx) 1 n
p1

+ f (x) sin(nx)
p1 p2

+ + f (x) sin(nx)
pm

f (x) sin(nx) dx +
p1

f (x) sin(nx) dx +
pm

f (x) sin(nx) dx

1 n

f (p ) f (p+ ) sin(np1 ) + + f (p ) f (p+ ) sin(npm ) 1 1 m m 11 n

f (x) sin(nx) dx

= where

1 1 An b n n n 1 An =
m

sin(npj ) f (p ) f (p+ ) j j
j=1

and the bn are the sine coecients of f (x). Since f (x) is bounded, An = O(1). Since f (x) is bounded, bn = 1

f (x) sin(nx) dx = O(1).

Thus an = O(1/n) as n . (Actually, from the Riemann-Lebesgue Lemma, bn = O(1/n).) 1354

Now we repeat this analysis for the sine coecients. 1 bn = f (x) sin(nx) dx p1 p2 1 = f (x) sin(nx) dx + f (x) sin(nx) dx + + f (x) sin(nx) dx p1 pm 1 p1 p = f (x) cos(nx) + f (x) cos(nx) p2 + + f (x) cos(nx) pm 1 n p1 p2 1 + f (x) cos(nx) dx + f (x) cos(nx) dx + f (x) cos(nx) dx n p1 pm 1 1 = Bn + an n n where (1)n 1 Bn = f () f ()
m

cos(npj ) f (p ) f (p+ ) j j
j=1

and the an are the cosine coecients of f (x). Since f (x) and f (x) are bounded, Bn , an = O(1) and thus bn = O(1/n) as n . With integration by parts on the Fourier coecients of f (x) we could nd that an = where An =
1 m j=1

1 1 An b n n n

sin(npj )[f (p ) f (p+ )] and the bn are the sine coecients of f (x), and j j 1 1 bn = Bn + an n n

where Bn =

(1)n [f

1 () f ()]

m j=1

cos(npj )[f (p ) f (p+ )] and the an are the cosine coecients of f (x). j j 1355

Now we can rewrite an and bn as an = 1 1 1 An + 2 Bn 2 an n n n 1 1 1 bn = Bn + 2 An 2 bn . n n n


(j) (j)

Continuing this process we could dene An and Bn so that an = 1 1 1 1 An + 2 Bn 3 An 4 Bn + n n n n 1 1 1 1 bn = Bn + 2 An + 3 Bn 4 An . n n n n

For any bounded function, the Fourier coecients satisfy an , bn = O(1/n) as n . If An and Bn are zero then the Fourier coecients will be O(1/n2 ). A sucient condition for this is that the periodic extension of f (x) is continuous. We see that if the periodic extension of f (x) is continuous then An and Bn will be zero and the Fourier coecients will be O(1/n3 ).

Result 28.8.1 Let f (x) be a bounded function for which there is a partition of the range (, ) into a nite number of intervals such that f (x) and all its derivatives are continuous on each of the intervals. If f (x) is not continuous then the Fourier coecients are O(1/n). If f (x), f (x), . . . , f (k2) (x) are continuous then the Fourier coecients are O(1/nk ).

If the periodic extension of f (x) is continuous, then the Fourier coecients will be O(1/n2 ). The series |an cos(nx) n=1 can be bounded by M 1/n2 where M = max(|an | + |bn |). Thus the Fourier series converges to f (x) uniformly. n=1
n

Result 28.8.2 If the periodic extension of f (x) is continuous then the Fourier series of f (x) will converge uniformly for all x.
If the periodic extension of f (x) is not continuous, we have the following result. 1356

Result 28.8.3 If f (x) is continuous in the interval c < x < d, then the Fourier series is uniformly convergent in the interval c + x d for any > 0.
Example 28.8.2 Dierent Rates of Convergence. A Discontinuous Function. Consider the function dened by f1 (x) = 1 1, for 1 < x < 0 for 0 < x < 1.

This function has jump discontinuities, so we know that the Fourier coecients are O(1/n). Since this function is odd, there will only be sine terms in its Fourier expansion. Furthermore, since the function is symmetric about x = 1/2, there will be only odd sine terms. Computing these terms,
1

bn = 2
0

sin(nx) dx
1

1 cos(nx) =2 n 0 (1)n 1 =2 n n =
4 n

for odd n for even n.

The function and the sum of the rst three terms in the expansion are plotted, in dashed and solid lines respectively, in Figure 28.7. Although the three term sum follows the general shape of the function, it is clearly not a good approximation. 1357

0.4

0.5

0.2

-1

-0.5 -0.5

0.5

-1

-0.5 -0.2

0.5

-1

-0.4

Figure 28.7: Three Term Approximation for a Function with Jump Discontinuities and a Continuous Function.

A Continuous Function. Consider the function dened by

x 1 f2 (x) = x x + 1

for 1 < x < 1/2 for 1/2 < x < 1/2 for 1/2 < x < 1. 1358

1 0.2 1

0.5

0.1

-1

-0.5 -0.1

0.5

0.25

0.1

1 -0.2 0 0.1

Figure 28.8: Three Term Approximation for a Function with Continuous First Derivative and Comparison of the Rates of Convergence. Since this function is continuous, the Fourier coecients will be O(1/n2 ). Also we see that there will only be odd sine terms in the expansion.
1/2 1/2 1

bn =
1

(x 1) sin(nx) dx +
1/2 1/2 1

x sin(nx) dx +
1/2

(x + 1) sin(nx) dx

=2
0

x sin(nx) dx + 2
1/2

(1 x) sin(nx) dx

= =

4 sin(n/2) (n)2
4 (1)(n1)/2 (n)2

for odd n for even n. 1359

The function and the sum of the rst three terms in the expansion are plotted, in dashed and solid lines respectively, in Figure 28.7. We see that the convergence is much better than for the function with jump discontinuities.

A Function with a Continuous First Derivative. Consider the function dened by x(1 + x) x(1 x) for 1 < x < 0 for 0 < x < 1.

f3 (x) =

Since the periodic extension of this function is continuous and has a continuous rst derivative, the Fourier coecients will be O(1/n3 ). We see that the Fourier expansion will contain only odd sine terms.
0 1

bn =
1

x(1 + x) sin(nx) dx +
0 1

x(1 x) sin(nx) dx

=2
0

x(1 x) sin(nx) dx

= =

4(1 (1)n ) (n)3


4 (n)3

for odd n for even n.

The function and the sum of the rst three terms in the expansion are plotted in Figure 28.8. We see that the rst three terms give a very good approximation to the function. The plots of the function, (in a dashed line), and the three term approximation, (in a solid line), are almost indistinguishable. In Figure 28.8 the convergence of the of the rst three terms to f1 (x), f2 (x), and f3 (x) are compared. In the last graph we see a closeup of f3 (x) and its Fourier expansion to show the error.

1360

28.9

Gibbs Phenomenon
1 1

The Fourier expansion of f (x) = is f (x) 4 1 sin(nx). n for 0 x < 1 for 1 x < 0

n=1

For any xed x, the series converges to 1 (f (x ) + f (x+ )). For any > 0, the convergence is uniform in the intervals 2 1 + x and x 1 . How will the nonuniform convergence at integral values of x aect the Fourier series? Finite Fourier series are plotted in Figure 28.9 for 5, 10, 50 and 100 terms. (The plot for 100 terms is closeup of the behavior near x = 0.) Note that at each discontinuous point there is a series of overshoots and undershoots that are pushed closer to the discontinuity by increasing the number of terms, but do not seem to decrease in height. In fact, as the number of terms goes to innity, the height of the overshoots and undershoots does not vanish. This is known as Gibbs phenomenon.

28.10

Integrating and Dierentiating Fourier Series

Integrating Fourier Series. Since integration is a smoothing operation, any convergent Fourier series can be integrated term by term to yield another convergent Fourier series. Example 28.10.1 Consider the step function for 0 x < for x < 0. 1361

f (x) =

1.2

1 0.8

0.1

Figure 28.9: Since this is an odd function, there are no cosine terms in the Fourier series. bn = 2

sin(nx) dx
0

1 = 2 cos(nx) n 0 2 = (1 (1)n ) n 4 for odd n = n 0 for even n.

1362

f (x)
n=1 oddn

4 sin nx n

Integrating this relation,


x x

f (t) dt

n=1 oddn x

4 sin(nt) dt n sin(nt) dt

F (x)
n=1 oddn

4 n

=
n=1 oddn

1 4 cos(nt) n n

=
n=1 oddn

4 ( cos(nx) + (1)n ) n2 1 cos(nx) 4 2 n n2 n=1


oddn

=4
n=1 oddn

Since this series converges uniformly,

4
n=1 oddn

1 cos(nx) 4 = F (x) = 2 n n2 n=1


oddn 0

x x

for x < 0 for 0 x < .

The value of the constant term is 4

n=1 oddn

2 1 = 2 n

1 F (x) dx = .

1363

Thus

1 cos(nx) 4 = n2 n=1
oddn

x x

for x < 0 for 0 x < .

Dierentiating Fourier Series. Recall that in general, a series can only be dierentiated if it is uniformly convergent. The necessary and sucient condition that a Fourier series be uniformly convergent is that the periodic extension of the function is continuous.

Result 28.10.1 The Fourier series of a function f (x) can be dierentiated only if the periodic extension of f (x) is continuous.
Example 28.10.2 Consider the function dened by f (x) = f (x) has the Fourier series

for 0 x < for x < 0. 4 sin nx. n

f (x)
n=1 oddn

The function has a derivative except at the points x = n. Dierentiating the Fourier series yields

f (x) 4
n=1 oddn

cos(nx).

For x = n, this implies 0=4

cos(nx),
n=1 oddn

1364

which is false. The series does not converge. This is as we expected since the Fourier series for f (x) is not uniformly convergent.

1365

28.11

Exercises

Exercise 28.1 1. Consider a 2 periodic function f (x) expressed as a Fourier series with partial sums SN (x) = a0 + an cos(nx) + bn sin(nt). 2 n=1
N

Assuming that the Fourier series converges in the mean, i.e.


N

lim

(f (x) SN (x))2 dx = 0,

show

1 a2 0 + a2 + b 2 = n n 2 n=1 This is called Parsevals equation.

f (x)2 dx.

2. Find the Fourier series for f (x) = x on x < (and repeating periodically). Use this to show

n=1

1 2 = . n2 6

3. Similarly, by choosing appropriate functions f (x), use Parsevals equation to determine

n=1

1 n4

and
n=1

1 . n6

Exercise 28.2 Consider the Fourier series of f (x) = x on x < as found above. Investigate the convergence at the points of discontinuity. 1366

1. Let SN be the sum of the rst N terms in the Fourier series. Show that cos N + dSN = 1 (1)N dx cos x 2 2. Now use this to show that
x 1 ( 2 2 1 2

x SN =
0

sin

N+ sin

d.

3. Finally investigate the maxima of this dierence around x = and provide an estimate (good to two decimal places) of the overshoot in the limit N . Exercise 28.3 Consider the boundary value problem on the interval 0 < x < 1 y + 2y = 1 y(0) = y(1) = 0. 1. Choose an appropriate periodic extension and nd a Fourier series solution. 2. Solve directly and nd the Fourier series of the solution (using the same extension). Compare the result to the previous step and verify the series agree. Exercise 28.4 Consider the boundary value problem on 0 < x < y + 2y = sin x y (0) = y () = 0. 1. Find a Fourier series solution. 2. Suppose the ODE is slightly modied: y + 4y = sin x with the same boundary conditions. Attempt to nd a Fourier series solution and discuss in as much detail as possible what goes wrong. 1367

Exercise 28.5 Find the Fourier cosine and sine series for f (x) = x2 on 0 x < . Are the series dierentiable? Exercise 28.6 Find the Fourier series of cosn (x). Exercise 28.7 For what values of x does the Fourier series 2 (1)n +4 cos nx = x2 2 3 n n=1 converge? What is the value of the above Fourier series for all x? From this relation show that

n=1

1 2 = n2 6

n=1

(1)n+1 2 = n2 12

Exercise 28.8 1. Compute the Fourier sine series for the function f (x) = cos x 1 + 2. How fast do the Fourier coecients an where

2x ,

0 x .

f (x) =
n=1

an sin nx

decrease with increasing n? Explain this rate of decrease. 1368

Exercise 28.9 Determine the cosine and sine series of f (x) = x sin x, (0 < x < ).

Estimate before doing the calculation the rate of decrease of Fourier coecients, an , bn , for large n. Exercise 28.10 Determine the Fourier cosine series of the function f (x) = cos(x), 0 x ,

where is an arbitrary real number. From this series deduce the following identities for non-integer . 1 = + (1)n sin() n=1 1 cot() = + n=1

1 1 + n +n

1 1 + n +n

Integrate the last formula from = 0 to = , (0 < < 1), to show that sin() 2 = 1 2 n n=1 Exercise 28.11 1. Show that x ln cos 2

= ln 2
n=1

(1)n cos(nx), n

< x < .

Use properties of Fourier series to conclude that x (1)n ln cos = ln 2 cos(nx), 2 n n=1 1369

x = (2k + 1), k Z.

Hint: use the identity Log(1 z) =

n=1

zn n

for |z| 1, z = 1.

2. From this series deduce that


0

ln cos 3. Show that 1 sin((x + )/2) ln = 2 sin((x )/2) Exercise 28.12 Solve the problem y + y = f (x),

x 2

dx = ln 2.

n=1

sin(nx) sin(n) , n

x = + 2k.

y(a) = y(b) = 0,

with an eigenfunction expansion. Assume that = n/(b a), n N. Exercise 28.13 Solve the problem y + y = f (x), y(a) = A, y(b) = B, with an eigenfunction expansion. Assume that = n/(b a), n N. Exercise 28.14 Find the trigonometric series and the simple closed form expressions for A(r, x) and B(r, x) where z = r ex and |r| < 1. 1 = 1 + z2 + z4 + 1 z2 1 1 b) A + B log(1 + z) = z z 2 + z 3 2 3 Find An and Bn , and the trigonometric sum for them where: a) A + B c) An + Bn = 1 + z + z 2 + + z n . 1370

Exercise 28.15 1. Is the trigonometric system {1, sin x, cos x, sin 2x, cos 2x, . . .} orthogonal on the interval [0, ]? Is the system orthogonal on any interval of length ? Why, in each case? 2. Show that each of the systems {1, cos x, cos 2x, . . .}, are orthogonal on [0, ]. Make them orthonormal too. Exercise 28.16 Let SN (x) be the N th partial sum of the Fourier series for f (x) |x| on < x < . Find N such that |f (x) SN (x)| < 101 on |x| < . Exercise 28.17 The set {sin(nx)} is orthogonal and complete on [0, ]. n=1 1. Find the Fourier sine series for f (x) 1 on 0 x . 2. Find a convergent series for g(x) = x on 0 x by integrating the series for part (a). 3. Apply Parsevals relation to the series in (a) to nd:

and {sin x, sin 2x, . . .}

n=1

1 (2n 1)2

Check this result by evaluating the series in (b) at x = . Exercise 28.18 1. Show that the Fourier cosine series expansion on [0, ] of: 1, 0 x < , 2 f (x) 1 , x = , 2 2 0, 2 < x , 1371

is 1 2 S(x) = + 2

n=0

(1)n cos((2n + 1)x). 2n + 1

2. Show that the N th partial sum of the series in (a) is 1 1 SN (x) = 2 ( Hint: Consider the dierence of
2N +1 y n n=1 (e ) n 2(N +1) x/2 0

sin((2(N + 1)t) dt. sin t where y = x /2.)

and

N 2y n ) , n=1 (e

3. Show that dSN (x)/dx = 0 at x = xn =

for n = 0, 1, . . . , N, N + 2, . . . , 2N + 2.

4. Show that at x = xN , the maximum of SN (x) nearest to /2 in (0, /2) is 1 1 SN (xN ) = + 2 Clearly xN /2 as N . 5. Show that also in this limit, SN (xN ) 1 1 + 2
0
N 2(N +1)

sin(2(N + 1)t) dt. sin t

sin t dt 1.0895. t

How does this compare with f (/2 0)? This overshoot is the Gibbs phenomenon that occurs at each discontinuity. It is a manifestation of the non-uniform convergence of the Fourier series for f (x) on [0, ]. Exercise 28.19 Prove the Isoperimetric Inequality: L2 4A where L is the length of the perimeter and A the area of any piecewise smooth plane gure. Show that equality is attained only for the circle. (Hints: The closed curve is represented parametrically as x = x(s), y = y(s), 0 s L 1372

where s is the arclength. In terms of t = 2s/L we have dx dt


2

dy dt

L 2

Integrate this relation over [0, 2]. The area is given by


2

A=
0

dy dt. dt

Express x(t) and y(t) as Fourier series and use the completeness and orthogonality relations to show that L2 4A 0.) Exercise 28.20 1. Find the Fourier sine series expansion and the Fourier cosine series expansion of g(x) = x(1 x), on 0 x 1. Which is better and why over the indicated interval? 2. Use these expansions to show that:

i)
k=1

1 2 = , k2 6

ii)
k=1

(1)k 2 = , k2 12

iii)
k=1

(1)k 3 = . (2k 1)2 32

Note: Some useful integration by parts formulas are: x sin(nx) = x 1 x 1 sin(nx) cos(nx); x cos(nx) = 2 cos(nx) + sin(nx) 2 n n n n 2 2 2x n x 2 x2 sin(nx) = 2 sin(nx) cos(nx) n n3 2x n2 x2 2 x2 cos(nx) = 2 cos(nx) + sin(nx) n n3

1373

28.12
Hint 28.1

Hints

Hint 28.2

Hint 28.3

Hint 28.4

Hint 28.5

Hint 28.6 Expand cosn (x) = Using Newtons binomial formula. Hint 28.7 1 x (e + ex ) 2
n

Hint 28.8

Hint 28.9

1374

Hint 28.10 Hint 28.11 Hint 28.12 Hint 28.13 Hint 28.14 Hint 28.15 Hint 28.16 Hint 28.17 Hint 28.18 Hint 28.19 Hint 28.20

1375

28.13

Solutions

Solution 28.1 1. We start by assuming that the Fourier series converges in the mean.
2

a0 f (x) (an cos(nx) + bn sin(nx)) 2 n=1

=0

We interchange the order of integration and summation.


(f (x)) dx a0

f (x) dx 2
n=1

an

f (x) cos(nx) dx + bn

f (x) sin(nx)

a2 0 2 +

+ a0
n=1

(an cos(nx) + bn sin(nx)) dx (an cos(nx) + bn sin(nx))(am cos(mx) + bm sin(mx)) dx = 0

n=1 m=1

Most of the terms vanish because the eigenfunctions are orthogonal.


(f (x)) dx a0

f (x) dx 2
n=1

an

f (x) cos(nx) dx + bn

f (x) sin(nx)

a2 0 + 2 n=1

(a2 cos2 (nx) + b2 sin2 (nx)) dx = 0 n n

1376

We use the denition of the Fourier coecients to evaluate the integrals in the last sum.

(f (x)) dx

a2 0

2
n=1

a2 n

b2 n

a2 a2 + b 2 = 0 + 0 + n n 2 n=1

a2 1 0 + a2 + b 2 = n n 2 n=1

f (x)2 dx

2. We determine the Fourier coecients for f (x) = x. Since f (x) is odd, all of the an are zero. b0 = 1 x sin(nx) dx 1 1 = x cos(nx) + n 2(1) n
n+1

1 cos(nx) dx n

= The Fourier series is x=

n=1

2(1)n+1 sin(nx) for x ( . . . ). n


1 n=1 n2 .

We apply Parsevals theorem for this series to nd the value of

n=1

4 1 = n2

x2 dx

n=1

4 2 2 = n2 3 1 2 = n2 6

n=1

1377

3. Consider f (x) = x2 . Since the function is even, there are no sine terms in the Fourier series. The coecients in the cosine series are 2 2 x dx 0 2 2 = 3 2 2 an = x cos(nx) dx 0 4(1)n = . n2 a0 = Thus the Fourier series is 2 (1)n x = +4 cos(nx) for x ( . . . ). 3 n2 n=1
2

We apply Parsevals theorem for this series to nd the value of 2 4 1 1 + 16 = 9 n4 n=1


1 n=1 n4 .

x4 dx

2 4 1 2 4 + 16 = 9 n4 5 n=1

n=1

1 4 = n4 90

1378

Now we integrate the series for f (x) = x2 .

2
0 3

2 3
2

d = 4
n=1

(1)n n2

cos(n) d
0

x (1)n x=4 sin(nx) 3 3 n3 n=1

We apply Parsevals theorem for this series to nd the value of

1 n=1 n6 .

16
n=1

1 1 = n6

x3 2 x 3 3

dx

16
n=1

1 16 6 = n6 945

n=1

1 6 = n6 945

1379

Solution 28.2 1. We dierentiate the partial sum of the Fourier series and evaluate the sum.

SN =
n=1 N

2(1)n+1 sin(nx) n (1)n+1 cos(nx)

SN = 2
n=1

SN = 2
n=1

(1)n+1 enx

1 (1)N +2 e(N +1)x 1 + ex 1 + ex (1)N e(N +1)x (1)N eN x SN = 1 + cos(x) cos((N + 1)x) + cos(N x) SN = 1 (1)N 1 + cos(x) cos N + 1 x cos x 2 2 SN = 1 (1)N cos2 x 2 SN = 2 cos N + dSN = 1 (1)N dx cos x 2 1380
1 2

2. We integrate SN .
x

SN (x) SN (0) = x
0 x

(1)N cos cos N+ sin


1 ( 2 2

N+
2

1 2

x SN =
0

sin

3. We nd the extrema of the overshoot E = x SN with the rst derivative test. E = We look for extrema in the range ( . . . ). N+ x= 1 The closest of these extrema to x = is x= 1 1 N + 1/2
1 ( 2 2

sin

N+ sin

1 (x 2 x 2

=0

1 2

(x ) = n , n [1 . . . 2N ]

n N + 1/2

Let E0 be the overshoot at this point. We approximate E0 for large N .


(11/(N +1/2))

E0 =
0

sin

N+ sin

We shift the limits of integration.

E0 =
/(N +1/2)

sin

N+ sin
2

1 2

1381

We add and subtract an integral over [0 . . . /(N + 1/2)].

E0 =
0

sin

N+ sin
2

1 2

/(N +1/2)

d
0

sin

N+ sin
2

1 2

We can evaluate the rst integral with contour integration on the unit circle C.

sin

N+ sin
2

1 2

d =
0

sin ((2N + 1) ) d sin ()

sin ((2N + 1) ) d sin () z 2N +1 dz 1 = 2 C (z 1/z)/(2) z z 2N +1 dz = 2 C (z 1) z 2N +1 , 1 + Res = Res (z + 1)(z 1) 12N +1 (1)2N +1 + = 2 2 = = 1382

1 2

z 2N +1 , 1 (z + 1)(z 1)

We approximate the second integral.


/(N +1/2) 0

sin

N+ sin
2

1 2

d =

2 2N + 1
0

sin(x) dx sin 2Nx+1

2 =2
0

sin(x) dx x 1 x

n=0

(1)n x2n+1 dx (2n + 1)!

=2
n=0 0

(1)n x2n dx (2n + 1)!

=2
n=0

(1)n 2n+1 dx (2n + 1)(2n + 1)!

3.70387 In the limit as N , the overshoot is | 3.70387| 0.56. Solution 28.3 1. The eigenfunctions of the self-adjoint problem y = y, are n = sin(nx), 1383 n Z+ y(0) = y(1) = 0,

We nd the series expansion of the inhomogeneity f (x) = 1.

1=
n=1

fn sin(nx)
1

fn = 2
0

sin(nx) dx cos(nx) n
1 0

fn = 2

2 fn = (1 (1)n ) n 4 for odd n n fn = 0 for even n We expand the solution in a series of the eigenfunctions.

y=
n=1

an sin(nx)

We substitute the series into the dierential equation. y + 2y = 1


n=1

an n sin(nx) + 2
n=1

2 2

an sin(nx) =
n=1 odd n

4 sin(nx) n

an =

4 n(2 2 n2 )

for odd n for even n

y=
n=1 odd n

4 sin(nx) n(2 2 n2 )

1384

2. Now we solve the boundary value problem directly. y + 2y = 1 y(0) = y(1) = 0 The general solution of the dierential equation is 1 y = c1 cos 2x + c2 sin 2x + . 2 We apply the boundary conditions to nd the solution. 1 1 c1 + = 0, c1 cos 2 + c2 sin 2 + =0 2 2 cos 2 1 1 c 1 = , c2 = 2 2 sin 2 cos 2 1 1 1 cos 2x + y= sin 2x 2 sin 2 We nd the Fourier sine series of the solution.

y=
n=1 1

an sin(nx)

an =
0

y(x) sin(nx) dx cos 2 1 1 cos 2x + sin 2x sin 2


0

an = 2

sin(nx) dx

an = an =

2(1 (1)2 n(2 2 n2 ) for odd n for even n

4 n(2 2 n2 )

1385

We obtain the same series as in the rst part. Solution 28.4 1. The eigenfunctions of the self-adjoint problem y = y, are y (0) = y () = 0,

1 0 = , n = cos(nx), n Z+ 2 We nd the series expansion of the inhomogeneity f (x) = sin(x). f0 + fn cos(nx) f (x) = 2 n=1 2 f0 =

sin(x) dx
0

f0 = fn = 2
0

sin(x) cos(nx) dx 2(1 + (1)n ) (1 n2 ) for even n for odd n

fn = fn =

4 (1n2 )

We expand the solution in a series of the eigenfunctions. a0 + y= an cos(nx) 2 n=1 1386

We substitute the series into the dierential equation. y + 2y = sin(x) 2 an n cos(nx) + a0 + 2 an cos(nx) = + n=1 n=1
2

n=2 even n

4 cos(nx) (1 n2 )

1 y= +

4 (1 n2 )(2 n2 )

cos(nx)

n=2 even n

2. We expand the solution in a series of the eigenfunctions. y= a0 + an cos(nx) 2 n=1

We substitute the series into the dierential equation. y + 4y = sin(x) 2 an n cos(nx) + 2a0 + 4 an cos(nx) = + n=1 n=1
2

n=2 even n

4 cos(nx) (1 n2 )

It is not possible to solve for the a2 coecient. That equation is (0)a2 = 4 . 3

This problem is to be expected, as this boundary value problem does not have a solution. The solution of the dierential equation is 1 y = c1 cos(2x) + c2 sin(2x) + sin(x) 3 1387

The boundary conditions give us an inconsistent set of constraints. y (0) = 0, 1 c2 + = 0, 3 Thus the problem has no solution. Solution 28.5 Cosine Series. The coecients in the cosine series are 2 2 x dx 0 2 2 = 3 2 2 an = x cos(nx) dx 0 4(1)n = . n2 a0 = Thus the Fourier cosine series is 4(1)n 2 + cos(nx). f (x) = 3 n2 n=1 In Figure 28.10 the even periodic extension of f (x) is plotted in a dashed line and the sum of the rst ve terms in the Fourier series is plotted in a solid line. Since the even periodic extension is continuous, the cosine series is dierentiable. 1388

y () = 0 1 c2 = 0 3

10

10

8 5 6 -3 -2 -1 1 2 3

-5

-3

-2

-1

-10

Figure 28.10: The Fourier Cosine and Sine Series of f (x) = x2 .

Sine Series. The coecients in the sine series are

bn =

x2 sin(nx) dx
0

2(1)n 4(1 (1)n ) = n n3 n 2(1) for even n n = 2(1)n 8 for odd n. n n3 1389

Thus the Fourier sine series is

f (x)
n=1

2(1)n 4(1 (1)n ) + n n3

sin(nx).

In Figure 28.10 the odd periodic extension of f (x) and the sum of the rst ve terms in the sine series are plotted. Since the odd periodic extension of f (x) is not continuous, the series is not dierentiable. Solution 28.6 We could nd the expansion by integrating to nd the Fourier coecients, but it is easier to expand cosn (x) directly. cosn (x) = 1 x (e + ex ) 2 n nx 1 n nx n (n2)x n e(n2)x + e e + e + + = n 1 n1 n 2 0
n

If n is odd, cosn (x) = 1 2n n n (enx + enx ) + (e(n2)x + e(n2)x ) + 0 1 + = = = 1 2n 1 2n1 1 2n1 n (ex + ex ) (n 1)/2

n n n 2 cos(x) 2 cos(nx) + 2 cos((n 2)x) + + 0 1 (n 1)/2


(n1)/2

m=0 n

n cos((n 2m)x) m n cos(kx). (n k)/2

k=1 odd k

1390

If n is even,

cosn (x) =

1 2n

n n (e(n2)x + e(n2)x ) + (enx + enx ) + 0 1 + n n (e2x + ei2x ) + n/2 1 n/2

1 2n

n n n n 2 cos(nx) + 2 cos((n 2)x) + + 2 cos(2x) + 0 1 n/2 1 n/2


(n2)/2

n 1 1 + n1 = n 2 n/2 2 = n 1 1 + n1 2n n/2 2

m=0 n

n cos((n 2m)x) m n cos(kx). (n k)/2

k=2 even k

We may denote, a0 cos (x) = 2


n n

ak cos(kx),
k=1

where ak = 1 + (1)nk 1 n . 2 2n1 (n k)/2

1391

Solution 28.7 We expand f (x) in a cosine series. The coecients in the cosine series are 2 2 x dx 0 2 2 = 3 2 2 an = x cos(nx) dx 0 4(1)n = . n2 a0 = Thus the Fourier cosine series is (1)n 2 f (x) = +4 cos(nx). 3 n2 n=1 The Fourier series converges to the even periodic extension of f (x) = x2 which is f (x) = x 2 x+ 2
2

for 0 < x < ,

( denotes the oor or greatest integer function.) This periodic extension is a continuous function. Since x2 is an even function, we have 2 (1)n +4 cos nx = x2 3 n2 n=1 1392

for x .

We substitute x = into the Fourier series. 2 (1)n cos(n) = 2 +4 2 3 n n=1


n=1

1 2 = n2 6

We substitute x = 0 into the Fourier series. 2 (1)n +4 =0 3 n2 n=1


n=1

(1)n+1 2 = n2 12

Solution 28.8 1. We compute the Fourier sine coecients. an = 2 f (x) sin(nx) dx 0 2 2x = cos x 1 + 0 n 2(1 + (1) ) = (n3 n)
4 (n3 n)

sin(nx) dx

an =

for even n for odd n

1393

2. From our work in the previous part, we see that the Fourier coecients decay as 1/n3 . The Fourier sine series converges to the odd periodic extension of the function, f (x). We can determine the rate of decay of the Fourier (x). For < x < , the odd periodic extension of f (x) is dened coecients from the smoothness of f f (x) = Since f (0+ ) = f (0 ) = 0 and f () = f () = 0 f (x) is continuous, C 0 . Since 2 f (0+ ) = f (0 ) = 2 and f () = f () = f (x) = cos(x) 1 + 2x f (x) = cos(x) + 1 + 0 x < , x < 0.

2x

f (x) is continuously dierentiable, C 1 . However, since f (0+ ) = 1, and f (0 ) = 1

f (x) is not C 2 . Since f (x) is C 1 we know that the Fourier coecients decay as 1/n3 . Solution 28.9 Cosine Series. The even periodic extension of f (x) is a C 0 , continuous, function (See Figure 28.11. Thus the coecients in the cosine series will decay as 1/n2 . The Fourier cosine coecients are a0 = 2 =2 2 1 2 1394

x sin x dx
0

a1 =

x sin x cos x dx
0

an =

2 x sin x cos(nx) dx 0 2(1)n+1 = , for n 2 n2 1

The Fourier cosine series is 1 2(1)n f (x) = 1 cos x 2 cos(nx). 2 n2 1 n=2

-5

Figure 28.11: The even periodic extension of x sin x. Sine Series. The odd periodic extension of f (x) is a C 1 , continuously dierentiable, function (See Figure 28.12. Thus the coecients in the cosine series will decay as 1/n3 . The Fourier sine coecients are a1 = 1 = 2

x sin x sin x dx
0

1395

an =

x sin x sin(nx) dx
0

= The Fourier sine series is

4(1 + (1)n )n , (n2 1)2

for n 2

4 f (x) = sin x 2

n=2

(1 + (1)n )n cos(nx). (n2 1)2

1 -5 5

Figure 28.12: The odd periodic extension of x sin x.

Solution 28.10 If = n is an integer, then the Fourier cosine series is the single term cos(|n|x). We assume that = n. We note that the even periodic extension of cos(x) is C 0 so that the series converges to cos(x) for x 1396

and the coecients decay as 1/n2 . We compute the Fourier cosine coecients. a0 = 2 cos(x) dx 0 2 sin() = cos(x) cos(nx) dx
0

an =

= (1)n The Fourier cosine series is

1 1 + n +n

sin()

sin() cos(x) = + (1)n n=1 We substitute x = 0 into the Fourier cosine series. sin() 1= + (1)n n=1

1 1 + n +n

sin() cos(nx).

1 1 + n +n

sin()

1 = + (1)n sin n=1 Next we substitute x = into the Fourier cosine series. sin() cos() = + (1)n n=1 cot = 1 + n=1

1 1 + n +n

1 1 + n +n 1 1 + n +n

sin()(1)n

1397

Note that neither cot() nor 1/ is integrable at = 0. We write the last formula so each side is integrable. 1 cot = We integrate from = 0 to = < 1. ln ln sin()

n=1

1 1 + n +n

=
0 n=1

[ln(n )] + [ln(n + )] 0 0 ln
n=1

sin() ln ln

ln = sin() sin() = = ln

n n ln 1

+ ln 2 n2 2 n2

n+ n

n=1

1
n=1

sin() 2 = 1 2 n n=1 Solution 28.11 1. We will consider the principal branch of the logarithm, < (Log z) . For < x < , cos(x/2) is positive so that ln(cos(x/2)) is well-dened. At x = , ln(cos(x/2)) is singular. However, the function is integrable so it has a Fourier series which converges except at x = (2k + 1), k Z. ln cos x = ln 2 ex/2 + ex/2 2 = ln 2 + ln ex/2 (1 + ex ) x = ln 2 + Log (1 + ex ) 2 1398

Since | ex | 1 and ex = 1 for that domain.

(x) 0, x = (2k + 1), we can expand the last term in a Taylor series in

x (1)n x n = ln 2 (e ) 2 n=1 n

= ln 2
n=1

(1)n cos(nx) n

(1)n x + sin(nx) 2 n=1 n

For < x < , ln(cos(x/2)) is real-valued. We equate the real parts of the equation on this domain to obtain the desired Fourier series. ln cos x 2

= ln 2
n=1

(1)n cos(nx), n

< x < .

The domain of convergence for this series is periodic extension of the function.

(x) = 0, x = (2k + 1). The Fourier series converges to the

ln cos

(1)n x = ln 2 cos(nx), 2 n n=1

x = (2k + 1), k Z

2. Now we integrate the function from 0 to .


0

x ln cos 2

dx =
0

ln 2
n=1

(1)n cos(nx) n

dx

= ln 2
n=1

(1)n n

cos(nx) dx
0 0

= ln 2
n=1

(1)n sin(nx) n n

1399

ln cos
0

x 2

dx = ln 2

3. We expand the logorithm. 1 sin((x + )/2) 1 1 = ln |sin((x + )/2)| ln |sin((x )/2)| ln 2 sin((x )/2) 2 2 Consider the function ln | sin(y/2)|. Since sin(x) = cos(x /2), we can use the result of part (a) to obtain, ln sin y 2 = ln cos = ln 2
n=1

y 2

(1)n cos(n(y )) n 1 cos(ny), n for y = 2k, k Z.

= ln 2
n=1

We return to the original function: sin((x + )/2) 1 1 ln = 2 sin((x )/2) 2 for x = 2k, k Z. 1 sin((x + )/2) ln = 2 sin((x )/2)

ln 2
n=1

1 1 cos(n(x + )) + ln 2 + cos(n(x )) , n n n=1

n=1

sin(nx) sin(n) , n

x = + 2k

Solution 28.12 The eigenfunction problem associated with this problem is + 2 = 0, (a) = (b) = 0, 1400

which has the solutions, n = n , ba n = sin n(x a) ba , n N.

We expand the solution and the inhomogeneity in the eigenfunctions.

y(x) =
n=1

yn sin

n(x a) ba
b

f (x) =
n=1

fn sin

n(x a) ba

2 fn = ba

f (x) sin
a

n(x a) ba

dx

Since the solution y(x) satises the same homogeneous boundary conditions as the eigenfunctions, we can dierentiate the series. We substitute the series expansions into the dierential equation. y + y = f (x)

yn 2 n
n=1

+ sin (n x) =
n=1

fn sin (n x)

yn =

fn 2 n

Thus the solution of the problem has the series representation,

y(x) =
n=1

2 sin n

n(x a) ba

Solution 28.13 The eigenfunction problem associated with this problem is + 2 = 0, (a) = (b) = 0, 1401

which has the solutions, n n(x a) , n = sin ba ba We expand the solution and the inhomogeneity in the eigenfunctions. n =

n N.

y(x) =
n=1

yn sin , fn =

n(x a) ba 2 ba
b

f (x) =
n=1

fn sin

n(x a) ba

f (x) sin
a

n(x a) ba

dx

Since the solution y(x) does not satisfy the same homogeneous boundary conditions as the eigenfunctions, we can dierentiate the series. We multiply the dierential equation by an eigenfunction and integrate from a to b. We use integration by parts to move derivatives from y to the eigenfunction. y + y = f (x)
b b b

y (x) sin(m x) dx +
a a

y(x) sin(m x) dx =
a

f (x) sin(m x) dx

ba ba ym = fm 2 2 a b ba ba [ym cos(m x)]b y2 sin(m x) dx + ym = fm m a 2 2 a ba ba Bm (1)m + Am (1)m+1 2 ym + ym = fm m 2 2 fm + (1)m m (A + B) ym = 2 m Thus the solution of the problem has the series representation, [y
b sin(m x)]a

y m cos(m x) dx +

y(x) =
n=1

fm + (1)m m (A + B) sin 2 m

n(x a) ba

1402

Solution 28.14 1. A + B = =
n=0

1 1 z2

z 2n r2n e2nx
n=0

= =
n=0

2n

cos(2nx) +
n=1

r2n sin(2nx)

A=
n=0

r2n cos(2nx),

B=
n=1

r2n sin(2nx)

A + B =

1 1 z2 1 = 1 r2 e2x = r2

1 1 cos(2x) r2 sin(2x) 1 r2 cos(2x) + r2 sin(2x) = (1 r2 cos(2x))2 + (r2 sin(2x))2 A= 1 r2 cos(2x) , 1 2r2 cos(2x) + r4 B= r2 sin(2x) 1 2r2 cos(2x) + r4

1403

2. We consider the principal branch of the logarithm. A + B = log(1 + z)

=
n=1

(1)n+1 n z n (1)n+1 n nx r e n (1)n+1 n r cos(nx) + sin(nx) n

=
n=1

=
n=1

A=
n=1

(1)n+1 n r cos(nx), n

B=
n=1

(1)n+1 n r sin(nx) n

A + B = log(1 + z) = log (1 + r ex ) = log (1 + r cos x + r sin x) = log |1 + r cos x + r sin x| + arg (1 + r cos x + r sin x) = log (1 + r cos x)2 + (r sin x)2 + arctan (1 + r cos x, r sin x)

A=

1 log 1 + 2r cos x + r2 , 2 1404

B = arctan (1 + r cos x, r sin x)

3.
n

An + Bn =
k=1

zk

1 z n+1 1z 1 rn+1 e(n+1)x = 1 r ex 1 r ex rn+1 e(n+1)x +rn+2 enx = 1 2r cos x + r2 = An = 1 r cos x rn+1 cos((n + 1)x) + rn+2 cos(nx) 1 2r cos x + r2 r sin x rn+1 sin((n + 1)x) + rn+2 sin(nx) 1 2r cos x + r2
n

Bn =

An + Bn =
k=1 n

zk rk ekx
k=1 n

=
n

An =
k=1

r cos(kx),

Bn =
k=1

rk sin(kx)

Solution 28.15 1.

1 sin x dx = [ cos x] = 2 0
0

1405

Thus the system is not orthogonal on the interval [0, ]. Consider the interval [a, a + ].
a+

1 sin x dx = [ cos x]a+ = 2 cos a a


a a+

1 cos x dx = [sin x]a+ = 2 sin a a


a

Since there is no value of a for which both cos a and sin a vanish, the system is not orthogonal for any interval of length . 2. First note that
0

cos nx dx = 0 for n N. If n = m, n 1 and m 0 then

cos nx cos mx dx =
0

1 2

cos((n m)x) + cos((n + m)x) dx = 0


0

Thus the set {1, cos x, cos 2x, . . .} is orthogonal on [0, ]. Since

dx =
0

cos2 (nx) dx =
0

, 2

the set 1 , is orthonormal on [0, ]. 1406 2 cos x, 2 cos 2x, . . .

If n = m, n 1 and m 1 then

sin nx sin mx dx =
0

1 2

cos((n m)x) cos((n + m)x) dx = 0


0

Thus the set {sin x, sin 2x, . . .} is orthogonal on [0, ]. Since

sin2 (nx) dx =
0

, 2

the set 2 sin x, is orthonormal on [0, ]. Solution 28.16 Since the periodic extension of |x| in [, ] is an even function its Fourier series is a cosine series. Because of the anti-symmetry about x = /2 we see that except for the constant term, there will only be odd cosine terms. Since the periodic extension is a continuous function, but has a discontinuous rst derivative, the Fourier coecients will decay as 1/n2 .

2 sin 2x, . . .

|x| =
n=0

an cos(nx),

for x [, ]

1 a0 =

1 x2 x dx = 2 1407

=
0

an =

2 x cos(nx) dx 0 2 sin(nx) 2 x = n 0 2 cos(nx) = n2 0 2 = 2 (cos(n) 1) n 2(1 (1)n ) = n2 4 + 2

sin(nx) dx n

|x| =

n=1 odd n

1 cos(nx) for x [, ] n2

Dene RN (x) = f (x) SN (x). We seek an upper bound on |RN (x)|. 4 |RN (x)| = 4 1 cos(nx) n2 n=N +1
odd n

1 n2 n=N +1
odd n

4 = Since

n=1 odd n

1 4 2 n

n=1 odd n

1 n2

n=1 odd n

1 2 = n2 8

1408

We can bound the error with, 4 |RN (x)| 2


N

n=1 odd n

1 . n2

N = 7 is the smallest number for which our error bound is less than 101 . N 7 is sucient to make the error less that 0.1. 4 1 1 1 1+ + + 0.079 |R7 (x)| 2 9 25 49 N 7 is also necessary because. 4 1 |RN (0)| = . n=N +1 n2
odd n

Solution 28.17 1.

1
n=1

an sin(nx),

0x

Since the odd periodic extension of the function is discontinuous, the Fourier coecients will decay as 1/n. Because of the symmetry about x = /2, there will be only odd sine terms. an = 2 1 sin(nx) dx 0 2 = ( cos(n) + cos(0)) n 2 = (1 (1)n ) n 4 1

n=1 odd n

sin(nx) n

1409

2. Its always OK to integrate a Fourier series term by term. We integrate the series in part (a).
x a

4 1 dx

n=1 odd n

sin(n) dx n

4 xa

n=1 odd n

cos(na) cos(nx) n2

Since the series converges uniformly, we can replace the with =. xa= 4

n=1 odd n

cos(na) 4 n2

n=1 odd n

cos(nx) n2

Now we have a Fourier cosine series. The rst sum on the right is the constant term. If we choose a = /2 this sum vanishes since cos(n/2) = 0 for odd integer n. 4 x= 2

n=1 odd n

cos(nx) n2

3. If f (x) has the Fourier series f (x) then Parsevals theorem states that f (x) dx = a2 + (a2 + b2 ). n n 2 0 n=1
2

a0 + (an cos(nx) + bn sin(nx)), 2 n=1

1410

We apply this to the Fourier sine series from part (a).


f (x) dx =
0 n=1 odd n

4 n

(1) dx +
0

16 (1) dx =
2

n=1

1 (2n 1)2

n=1

1 2 = (2n 1)2 8

We substitute x = in the series from part (b) to corroborate the result. 4 x= 2 = 4 2


n=1

cos((2n 1)x) (2n 1)2 cos((2n 1)) (2n 1)2

n=1

n=1

1 2 = (2n 1)2 8

Solution 28.18 1.

f (x) a0 +
n=1

an cos(nx)

Since the periodic extension of the function is discontinuous, the Fourier coecients will decay like 1/n. Because of the anti-symmetry about x = /2, there will be only odd cosine terms. a0 = 1

f (x) dx =
0

1 2

1411

an =

2 f (x) cos(nx) dx 0 2 /2 = cos(nx) dx 0 2 = sin(n/2) n 2 (1)(n1)/2 , for odd n n = 0 for even n

The Fourier cosine series of f (x) is

1 2 f (x) + 2

n=0

(1)n cos((2n + 1)x). 2n + 1

2. The N th partial sum is 1 2 + 2


N

SN (x) =

n=0

(1)n cos((2n + 1)x). 2n + 1

We wish to evaluate the sum from part (a). First we make the change of variables y = x /2 to get rid of the 1412

(1)n factor.

n=0

(1)n cos((2n + 1)x) 2n + 1


N

=
n=0 N

(1)n cos((2n + 1)(y + /2)) 2n + 1 (1)n (1)n+1 sin((2n + 1)y) 2n + 1 1 sin((2n + 1)y) 2n + 1

= =

n=0 N

n=0

1413

We write the summand as an integral and interchange the order of summation and integration to get rid of the 1/(2n + 1) factor.
N y

=
n=0 0 y N

cos((2n + 1)t) dt

=
0 y

cos((2n + 1)t) dt
n=0 2N +1 N

=
0 y

cos(nt)
n=1 2N +1

cos(2nt) e

dt

=
0 y

ent
n=1 et

n=1 N 2nt

dt

e2t e2(N +1)t dt 1 et 1 e2t 0 y (et e2(N +1)t )(1 e2t ) (e2t e2(N +1)t )(1 et ) = (1 et )(1 e2t ) 0 y et e2t + e(2N +4)t e(2N +3)t dt = (1 et )(1 e2t ) 0 y et e(2N +3)t = dt 1 e2t 0 y e(2N +2)t 1 = dt et et 0 y e2(N +1)t + = dt 2 sin t 0 1 y sin(2(N + 1)t) dt = 2 0 sin t 1 x/2 sin(2(N + 1)t) = dt 2 0 sin t 1414 =

n=1 (2N +2)t e

dt

Now we have a tidy representation of the partial sum. SN (x) = 3. We solve


dSN (x) dx

1 1 2

x/2 0

sin(2(N + 1)t) dt sin t

= 0 to nd the relative extrema of SN (x). SN (x) = 0 1 sin(2(N + 1)(x /2)) =0 sin(x /2) (1)N +1 sin(2(N + 1)x) =0 cos(x) sin(2(N + 1)x) =0 cos(x) n x = xn = , n = 0, 1, . . . , N, N + 2, . . . , 2N + 2 2(N + 1)

Note that xN +1 = /2 is not a solution as the denominator vanishes there. The function has a removable singularity at x = /2 with limiting value (1)N . 4. 1 1 SN (xN ) = 2 We note that the integrand is even.
N /2 2(N +1) N /2 2(N +1)

sin(2(N + 1)t) dt sin t


2(N +1)

2(N +1)

=
0 0

=
0

SN (xN ) =

1 1 + 2

2(N +1)

sin(2(N + 1)t) dt sin t

1415

5. We make the change of variables 2(N + 1)t t. 1 1 SN (xN ) = + 2 Note that lim
0 0

sin(t) dt 2(N + 1) sin(t/(2(N + 1))) t cos( t) =t 0 1

sin( t)
0

= lim

SN (xN )

1 1 + 2

sin(t) dt 1.0895 as N t

This is not equal to the limiting value of f (x), f (/2 0) = 1. Solution 28.19 With the parametrization in t, x(t) and y(t) are continuous functions on the range [0, 2]. Since the curve is closed, we have x(0) = x(2) and y(0) = y(2). This means that the periodic extensions of x(t) and y(t) are continuous functions. Thus we can dierentiate their Fourier series. First we dene formal Fourier series for x(t) and y(t). x(t) = a0 + an cos(nt) + bn sin(nt) 2 n=1

c0 y(t) = + cn cos(nt) + dn sin(nt) 2 n=1

x (t) =
n=1

nbn cos(nt) nan sin(nt) ndn cos(nt) ncn sin(nt)


n=1

y (t) =

In this problem we will be dealing with integrals on [0, 2] of products of Fourier series. We derive a general formula 1416

for later use.


2 2

xy dt =
0 0 2

a0 + an cos(nt) + bn sin(nt) 2 n=1

c0 + cn cos(nt) + dn sin(nt) 2 n=1 dt

dt

=
0

a0 c 0 + an cn cos2 (nt) + bn dn sin2 (nt) 4 n=1

1 a0 c 0 + (an cn + bn dn ) 2 n=1

In the arclength parametrization we have dx ds In terms of t = 2s/L this is dx dt We integrate this identity on [0, 2]. L2 = 2 =
n=1 2 0 2 2

dy ds
2

= 1.
2

dy dt
2

L 2

dx dt

+
2

dy dt

dt
2

(nbn ) + (nan )

+
n=1

(ndn )2 + (ncn )2

=
n=1

n2 (a2 + b2 + c2 + d2 ) n n n n

L2 = 2 2
n=1

n2 (a2 + b2 + c2 + d2 ) n n n n

1417

We assume that the curve is parametrized so that the area is positive. (Reversing the orientation changes the sign of the area as dened above.) The area is
2

A=
0 2

dy dt dt a0 + an cos(nt) + bn sin(nt) 2 n=1


=
0

ndn cos(nt) ncn sin(nt)


n=1

dt

=
n=1

n(an dn bn cn )

Now we nd an upper bound on the area. We will use the inequality |ab| 1 |a2 + b2 |, which follows from expanding 2 (a b)2 0. A 2 We can express this in terms of the perimeter. = L2 4 L2 4A Now we determine the curves for which L2 = 4A. To do this we nd conditions for which A is equal to the upper bound we obtained for it above. First note that

n a2 + b2 + c2 + d2 n n n n
n=1

n2 a2 + b2 + c2 + d2 n n n n
n=1

n
n=1

a2 n

b2 n

c2 n

d2 n

=
n=1

n2 a2 + b2 + c2 + d2 n n n n

1418

implies that all the coecients except a0 , c0 , a1 , b1 , c1 and d1 are zero. The constraint,

n=1

n(an dn bn cn ) =

n a2 + b2 + c2 + d2 n n n n
n=1

then becomes a1 d1 b1 c1 = a2 + b2 + c2 + d2 . 1 1 1 1 This implies that d1 = a1 and c1 = b1 . a0 and c0 are arbitrary. Thus curves for which L2 = 4A have the parametrization a0 c0 x(t) = + a1 cos t + b1 sin t, y(t) = b1 cos t + a1 sin t. 2 2 Note that c0 2 a0 2 x(t) + y(t) = a2 + b 2 . 1 1 2 2 The curve is a circle of radius a2 + b2 and center (a0 /2, c0 /2). 1 1 Solution 28.20 1. The Fourier sine series has the form

x(1 x) =
n=1

an sin(nx).

The norm of the eigenfunctions is


1 0

1 sin2 (nx) dx = . 2

The coecients in the expansion are


1

an = 2
0

x(1 x) sin(nx) dx 2 (2 2 cos(n) n sin(n)) (1 (1)n ).

= =

3 n3 4 3 n3

1419

Thus the Fourier sine series is 8 x(1 x) = 3

n=1 odd n

sin(nx) 8 = 3 3 n

n=1

sin((2n 1)x) . (2n 1)3

The Fourier cosine series has the form

x(1 x) =
n=0

an cos(nx).

The norm of the eigenfunctions is


1 1

12 dx = 1,
0 0

1 cos2 (nx) dx = . 2

The coecients in the expansion are


1

a0 =
0 1

1 x(1 x) dx = , 6

an = 2
0

x(1 x) cos(nx) dx 2 2 n2 2 2 n2 + 4 sin(n) n cos(n) 3 n3

= = Thus the Fourier cosine series is

(1 + (1)n )

1 4 x(1 x) = 2 6

n=1 even n

cos(nx) 1 1 = 2 2 n 6

n=1

cos(2nx) . n2

1420

0.2 0.1 -1 -0.5 -0.1 -0.2 0.5 1 -1

0.2 0.1 -0.5 -0.1 -0.2 0.5 1

Figure 28.13: The odd and even periodic extension of x(1 x), 0 x 1.

The Fourier sine series converges to the odd periodic extension of the function. Since this function is C 1 , continuously dierentiable, we know that the Fourier coecients must decay as 1/n3 . The Fourier cosine series converges to the even periodic extension of the function. Since this function is only C 0 , continuous, the Fourier coecients must decay as 1/n2 . The odd and even periodic extensions are shown in Figure 28.13. The sine series is better because of the faster convergence of the series.

2. (a) We substitute x = 0 into the cosine series.

1 1 0= 2 6

n=1

1 n2

n=1

1 2 = n2 6

1421

(b) We substitute x = 1/2 into the cosine series. 1 1 1 = 2 4 6


n=1

cos(n) n2

n=1

(1)n 2 = n2 12

(c) We substitute x = 1/2 into the sine series. 1 8 = 3 4


n=1

sin((2n 1)/2) (2n 1)3

n=1

3 (1)n = (2n 1)3 32

1422

Chapter 29 Regular Sturm-Liouville Problems


I learned there are troubles Of more than one kind. Some come from ahead And some come from behind. But Ive bought a big bat. Im all ready, you see. Now my troubles are going To have troubles with me! -I Had Trouble in Getting to Solla Sollew -Theodor S. Geisel, (Dr. Suess)

29.1

Derivation of the Sturm-Liouville Form


p2 (x)y + p1 (x)y + p0 (x)y = y, 1423

Consider the eigenvalue problem on the nite interval [a . . . b],

subject to the homogeneous unmixed boundary conditions 1 y(a) + 2 y (a) = 0, 1 y(b) + 2 y (b) = 0.

Here the coecient functions pj are real and continuous and p2 > 0 on the interval [a . . . b]. (Note that if p2 were negative we could multiply the equation by (1) and replace by .) The parameters j and j are real. We would like to write this problem in a form that can be used to obtain qualitative information about the problem. First we will write the operator in self-adjoint form. We divide by p2 since it is non-vanishing. y + We multiply by an integrating factor. p1 dx eP (x) p2 p p eP (x) y + 1 y + 0 y = eP (x) y p2 p2 p2 p0 eP (x) y + eP (x) y = eP (x) y p2 p2 I = exp For notational convenience, we dene new coecient functions and parameters. p = eP (x) , q = eP (x) p0 , p2 = eP (x) 1 , p2 = . p1 p0 y + y = y. p2 p2 p2

Since the pj are continuous and p2 is positive, p, q, and are continuous. p and are positive functions. The problem now has the form, (py ) + qy + y = 0, subject to the same boundary conditions, 1 y(a) + 2 y (a) = 0, 1424 1 y(b) + 2 y (b) = 0.

This is known as a Regular Sturm-Liouville problem. We will devote much of this chapter to studying the properties of this problem. We will encounter many results that are analogous to the properties of self-adjoint eigenvalue problems. Example 29.1.1 d dx ln x dy dx + xy = 0, y(1) = y(2) = 0

is not a regular Sturm-Liouville problem since ln x vanishes at x = 1.

Result 29.1.1 Any eigenvalue problem of the form p2 y + p1 y + p0 y = y, for a x b, 1 y(a) + 2 y (a) = 0, 1 y(b) + 2 y (b) = 0, where the pj are real and continuous and p2 > 0 on [a, b], and the j and j are real can be written in the form of a regular Sturm-Liouville problem, (py ) + qy + y = 0, on a x b, 1 y(a) + 2 y (a) = 0, 1 y(b) + 2 y (b) = 0.

29.2

Properties of Regular Sturm-Liouville Problems

Self-Adjoint. Consider the Regular Sturm-Liouville equation. L[y] (py ) + qy = y. 1425

We see that the operator is formally self-adjoint. Now we determine if the problem is self-adjoint. v|L[u] L[v]|u = v|(pu ) + qu (pv ) + qv|u = [vpu ]b v |pu + v|qu [pv u]b + pv |u qv|u a a = [vpu ]b [pv u]b a a = p(b) v(b)u (b) v (b)u(b) + p(a) v(a)u (a) v (a)u(a) 1 1 = p(b) v(b) u(b) v(b)u(b) 2 2 1 1 + p(a) v(a) u(a) v(a)u(a) 2 2 =0 Above we used the fact that the i and i are real. 1 2 = 1 2 , 1 2 = 1 2

Thus L[y] subject to the boundary conditions is self-adjoint. Real Eigenvalues. Let be an eigenvalue with the eigenfunction . We start with Greens formula. |L[] L[]| = 0 | | = 0 || + || = 0 ( ) || = 0 Since || > 0, = 0. Thus the eigenvalues are real. 1426

Innite Number of Eigenvalues. There are an innite of eigenvalues which have no nite cluster point. This result is analogous to the result that we derived for self-adjoint eigenvalue problems. When we cover the Rayleigh quotient, we will nd that there is a least eigenvalue. Since the eigenvalues are distinct and have no nite cluster point, n as n . Thus the eigenvalues form an ordered sequence, 1 < 2 < 3 < . Orthogonal Eigenfunctions. Let and be two distinct eigenvalues with the eigenfunctions and . Greens formula states |L[] L[]| = 0. | | = 0 || + || = 0 ( ) || = 0 Since the eigenvalues are distinct, || = 0. Thus eigenfunctions corresponding to distinct eigenvalues are orthogonal with respect to . Unique Eigenfunctions. Let be an eigenvalue. Suppose and are two independent eigenfunctions corresponding to . L[] + = 0, L[] + = 0 We take the dierence of times the rst equation and times the second equation. L[] L[] = 0 (p ) (p ) = 0 (p( )) = 0 p( ) = const In order to satisfy the boundary conditions, the constant must be zero. p( ) = 0 1427

Since p > 0 the second factor vanishes. = 0 2 =0 d =0 dx = const and are not independent. Thus each eigenvalue has a unique, (to within a multiplicative constant), eigenfunction. Real Eigenfunctions. If is an eigenvalue with eigenfunction , then (p ) + q + = 0. We take the complex conjugate of this equation. p + q + = 0.

Thus is also an eigenfunction corresponding to . Are and independent functions, or do they just dier by a multiplicative constant? (For example, ex and ex are independent functions, but x and x are dependent.) From our argument on unique eigenfunctions, we see that = (const). Since and only dier by a multiplicative constant, the eigenfunctions can be chosen so that they are real-valued functions. 1428

Rayleighs Quotient. Let be an eigenvalue with the eigenfunction . |L[] = | |(p ) + q = || p


b a

|p| + |q| = || p
b a

+ |p| |q| ||

This is known as Rayleighs quotient. It is useful for obtaining qualitative information about the eigenvalues. Minimum Property of Rayleighs Quotient. Note that since p, q, and are bounded functions, the Rayleigh quotient is bounded below. Thus there is a least eigenvalue. If we restrict u to be a real continuous function that satises the boundary conditions, then 1 = min
u

[puu ]b + u |p|u u|q|u a , u||u

where 1 is the least eigenvalue. This form allows us to get upper and lower bounds on 1 . To derive this formula, we rst write it in terms of the operator L. 1 = min
u

u|L[u] u||u

Since u is continuous and satises the boundary conditions, we can expand u in a series of the eigenfunctions. u|L[u] = u||u =
n=1 cn n L [ m=1 cm m ] n=1 cn n m=1 cm m n=1 cn n m=1 cm m m n=1 cn n m=1 cm m

1429

We assume that we can interchange summation and integration. = =


m=1 cn cm n m ||n n=1 m=1 cn cm m ||n n=1 2 n=1 |cn | n n ||n 2 n=1 |cn | n ||n 2 n=1 |cn | n ||n 1 2 n=1 |cn | n ||n

= 1 We see that the minimum value of Rayleighs quotient is 1 . The minimum is attained when cn = 0 for all n 2, that is, when u = c1 1 . Completeness. The set of the eigenfunctions of a regular Sturm-Liouville problem is complete. That is, any piecewise continuous function dened on [a, b] can be expanded in a series of the eigenfunctions,

f (x)
n=1

cn n (x),

where the cn are the generalized Fourier coecients, cn = n ||f . n ||n

Here the sum is convergent in the mean. For any xed x, the sum converges to 1 (f (x )+f (x+ )). If f (x) is continuous 2 and satises the boundary conditions, then the convergence is uniform.

1430

Result 29.2.1 Properties of regular Sturm-Liouville problems. The eigenvalues are real. There are an innite number of eigenvalues 1 < 2 < 3 < . There is a least eigenvalue 1 but there is no greatest eigenvalue, (n as n ). For each eigenvalue, there is one unique, (to within a multiplicative constant), eigenfunction n . The eigenfunctions can be chosen to be real-valued. (Assume the n following are real-valued.) The eigenfunction n has exactly n 1 zeros in the open interval a < x < b. The eigenfunctions are orthogonal with respect to the weighting function (x).
b

n (x)m (x)(x) dx = 0 if n = m.
a

The eigenfunctions are complete. Any piecewise continuous function f (x) dened on a x b can be expanded in a series of eigenfunctions

f (x)
n=1

cn n (x),

where cn =

b a f (x)n (x)(x) dx . b 2 n (x)(x) dx a

1 The sum converges to 2 (f (x ) + f (x+ )).

1431

The eigenvalues can be related to the eigenfunctions with a formula known as the Rayleigh quotient.
b pn dn dx a b

+
a

dn dx

q2 n

dx

n =

b 2 a n dx

Example 29.2.1 A simple example of a Sturm-Liouville problem is d dx dy dx + y = 0, y(0) = y() = 0.

Bounding The Least Eigenvalue. The Rayleigh quotient for the rst eigenvalue is 1 =
(1 )2 dx 0 . 2 1 dx 0

Immediately we see that the eigenvalues are non-negative. If 0 (1 )2 dx = 0 then = (const). The only constant that satises the boundary conditions is = 0. Since the trivial solution is not an eigenfunction, = 0 is not an eigenvalue. Thus all the eigenvalues are positive. Now we get an upper bound for the rst eigenvalue. 1 = min
u (u )2 dx 0 2 u dx 0

where u is continuous and satises the boundary conditions. We choose u = x(x ) as a trial function. 1 = =
(u )2 dx 0 2 u dx 0 (2x )2 dx 0 (x2 x)2 dx 0 3

/3 5 /30 10 = 2 1.013

1432

Finding the Eigenvalues and Eigenfunctions. We consider the cases of negative, zero, and positive eigenvalues to check our results above. < 0. The general solution is y = ce
x

+d e

The only solution that satises the boundary conditions is the trivial solution, y = 0. Thus there are no negative eigenvalues. = 0. The general solution is y = c + dx. Again only the trivial solution satises the boundary conditions, so = 0 is not an eigenvalue. > 0. The general solution is We apply the boundary conditions. y(0) = 0 y() = 0 The nontrivial solutions are = n = 1, 2, 3, . . . Thus the eigenvalues and eigenfunctions are n = n 2 , n = sin(nx), for n = 1, 2, 3, . . . y = d sin(n). c=0 d sin( ) = 0 y = c cos( x) + d sin( x).

We can verify that this example satises all the properties listed in Result 29.2.1. Note that there are an innite number of eigenvalues. There is a least eigenvalue 1 = 1 but there is no greatest eigenvalue. For each eigenvalue, there is one eigenfunction. The nth eigenfunction sin(nx) has n 1 zeroes in the interval 0 < x < . 1433

Since a series of the eigenfunctions is the familiar Fourier sine series, we know that the eigenfunctions are orthogonal and complete. We check Rayleighs quotient. pn dn dx n =

+
0 0 0

dn 2 dx

q2 dx n
2

2 dx n

sin(nx) d(sin(nx)) dx = =
0 0

+
0 0

d(sin(nx)) dx

dx

sin2 (nx)dx

n2 cos2 (nx) dx /2

= n2 Example 29.2.2 Consider the eigenvalue problem x2 y + xy + y = y, y(1) = y(2) = 0.

Since x2 > 0 on [1 . . . 2], we can write this problem in terms of a regular Sturm-Liouville eigenvalue problem. We divide by x2 . 1 1 y + y + 2 (1 )y = 0 x x 1 We multiply by the integrating factor exp( x dx) = exp(ln x) = x and make the substitution, = 1 to obtain the Sturm-Liouville form. 1 xy + y + y = 0 x 1 (xy ) + y = 0 x We see that the eigenfunctions will be orthogonal with respect to the weighting function = 1/x. 1434

The Rayleigh quotient is = = p


b a

+ |x|

1 | x |

|x| . 1 | x |

If = 0, then only the trivial solution, = 0, satises the boundary conditions. Thus the eigenvalues are positive. Returning to the original problem, we see that the eigenvalues, , satisfy < 1. Since this is an Euler equation, we can nd solutions with the substitution y = x . ( 1) + + 1 = 0 2 + 1 = 0 Note that < 1. = 1 The general solution is y = c1 x y = c 1 e An equivalent form is y = c1 cos( 1 ln x) + c2 sin( We apply the boundary conditions. y(1) = 0 y(2) = 0 c1 = 0 sin( 1 ln 2) = 0 for n = 1, 2, . . . 1 ln 2 = n, 1435 1 ln x).
1

+ c2 x

We know that the eigenfunctions can be written as real functions. We rewrite the solution.
1 ln x

+c2 e

1 ln x

Thus the eigenvalues and eigenfunctions are n = 1 n ln 2


2

n = sin n

ln x ln 2

for n = 1, 2, . . .

29.3

Solving Dierential Equations With Eigenfunction Expansions


Ax = x.

Linear Algebra. Consider the eigenvalue problem,

If the matrix A has a complete, orthonormal set of eigenvectors { k } with eigenvalues {k } then we can represent any vector as a linear combination of the eigenvectors.
n

y=
k=1

ak k ,
n

ak = k y ( k y) k

y=
k=1

This property allows us to solve the inhomogeneous equation Ax x = b. (29.1)

Before we try to solve this equation, we should consider the existence/uniqueness of the solution. If is not an eigenvalue, then the range of L A is Rn . The problem has a unique solution. If is an eigenvalue, then the null space of L is the span of the eigenvectors of . That is, if = i , then nullspace(L) = span( i1 , i2 , . . . , im ). ({ i1 , i2 , . . . , im } are the eigenvalues of i .) If b is orthogonal to nullspace(L) then Equation 29.1 has a solution, but it is not unique. If y is a solution then we can add any linear combination of { ij } to obtain another solution. Thus the solutions have the form m x=y+
j=1

cj ij .

1436

If b is not orthogonal to nullspace(L) then Equation 29.1 has no solution. Now we solve Equation 29.1. We assume that is not an eigenvalue. We expand the solution x and the inhomogeneity in the orthonormal eigenvectors.
n n

x=
k=1

ak k ,

b=
k=1

bk k

We substitute the expansions into Equation 29.1.


n n n

A
k=1 n

ak k
k=1 n

ak k =
k=1 n

bk k bk k
k=1

ak k k
k=1 k=1

ak k =

bk ak = k The solution is x=
k=1 n

bk . k k

Inhomogeneous Boundary Value Problems. Consider the self-adjoint eigenvalue problem, Ly = y, a < x < b, B1 [y] = B2 [y] = 0. If the problem has a complete, orthonormal set of eigenfunctions {k } with eigenvalues {k } then we can represent any square-integrable function as a linear combination of the eigenfunctions.
b

f=
k

fk k ,

fk = k |f =
a

k (x)f (x) dx

f=
k

k |f k

1437

This property allows us to solve the inhomogeneous dierential equation Ly y = f, a < x < b, B1 [y] = B2 [y] = 0. (29.2)

Before we try to solve this equation, we should consider the existence/uniqueness of the solution. If is not an eigenvalue, then the range of L is the space of square-integrable functions. The problem has a unique solution. If is an eigenvalue, then the null space of L is the span of the eigenfunctions of . That is, if = i , then nullspace(L) = span(i1 , i2 , . . . , im ). ({i1 , i2 , . . . , im } are the eigenvalues of i .) If f is orthogonal to nullspace(L ) then Equation 29.2 has a solution, but it is not unique. If u is a solution then we can add any linear combination of {ij } to obtain another solution. Thus the solutions have the form
m

y =u+
j=1

cj ij .

If f is not orthogonal to nullspace(L ) then Equation 29.2 has no solution. Now we solve Equation 29.2. We assume that is not an eigenvalue. We expand the solution y and the inhomogeneity in the orthonormal eigenfunctions. y=
k

yk k ,

f=
k

fk k

It would be handy if we could substitute the expansions into Equation 29.2. However, the expansion of a function is not necessarily dierentiable. Thus we demonstrate that since y is C 2 (a . . . b) and satises the boundary conditions B1 [y] = B2 [y] = 0, we are justied in substituting it into the dierential equation. In particular, we will show that L[y] = L
k

yk k =
k

yk L [k ] =
k

yk k k .

To do this we will use Greens identity. If u and v are C 2 (a . . . b) and satisfy the boundary conditions B1 [y] = B2 [y] = 0 then u|L[v] = L[u]|v . 1438

First we assume that we can dierentiate y term-by-term. L[y] =


k

yk k k

Now we directly expand L[y] and show that we get the same result. L[y] =
k

ck k

ck = k |L[y] = L[k ]|y = k k |y = k k |y = k y k L[y] =


k

yk k

The series representation of y may not be dierentiable, but we are justied in applying L term-by-term. Now we substitute the expansions into Equation 29.2. L
k

yk k
k

yk k =
k

fk k fk k
k

k yk k
k k

yk k =

fk yk = k The solution is y=
k

fk k k 1439

Consider a second order, inhomogeneous problem. L[y] = f (x), We will expand the solution in an orthogonal basis. y=
n

B1 [y] = b1 ,

B2 [y] = b2

an n

We would like to substitute the series into the dierential equation, but in general we are not allowed to dierentiate such series. To get around this, we use integration by parts to move derivatives from the solution y, to the n . Example 29.3.1 Consider the problem, y + y = f (x), y(0) = a, y() = b,

where = n2 , n Z+ . We expand the solution in a cosine series. y0 y(x) = + yn n=1 We also expand the inhomogeneous term. f0 f (x) = + fn n=1

2 cos(nx)

2 cos(nx)

We multiply the dierential equation by the orthonormal functions and integrate over the interval. We neglect the 1440

special case 0 = 1/ for now.


0

2 cos(nx)y dx + 2 cos(nx)y (x)

2 cos(nx)y dx =

2 f (x) dx

+
0 0

2 n sin(nx)y (x) dx + yn = fn

2 ((1)n y () y (0)) +

2 n sin(nx)y(x)

0 0

2 2 n cos(nx)y(x) dx + yn = fn

2 ((1)n y () y (0)) n2 yn + yn = fn

Unfortunately we dont know the values of y (0) and y (). CONTINUE HERE

1441

29.4

Exercises

Exercise 29.1 Find the eigenvalues and eigenfunctions of y + 2y + y = 0, y(a) = y(b) = 0,

where a < b. Write the problem in Sturm Liouville form. Verify that the eigenvalues and eigenfunctions satisfy the properties of regular Sturm-Liouville problems. Find the coecients in the expansion of an arbitrary function f (x) in a series of the eigenfunctions. Hint, Solution Exercise 29.2 Find the eigenvalues and eigenfunctions of the boundary value problem y + y=0 (x + 1)2

on the interval 1 x 2 with boundary conditions y(1) = y(2) = 0. Discuss how the results satisfy the properties of Sturm-Liouville problems. Hint, Solution Exercise 29.3 Find the eigenvalues and eigenfunctions of y + 2 + 1 y + 2 y = 0, x x y(a) = y(b) = 0,

where 0 < a < b. Write the problem in Sturm Liouville form. Verify that the eigenvalues and eigenfunctions satisfy the properties of regular Sturm-Liouville problems. Find the coecients in the expansion of an arbitrary function f (x) in a series of the eigenfunctions. Hint, Solution 1442

Exercise 29.4 Find the eigenvalues and eigenfunctions of y y + y = 0, y(0) = y(1) = 0.

Find the coecients in the expansion of an arbitrary, f (x), in a series of the eigenfunctions. Hint, Solution Exercise 29.5 Consider y + y = f (x), The associated eigenvalue problem is y + y = y y(0) = 0 y(1) + y (1) = 0. y(0) = 0, y(1) + y (1) = 0. (29.3)

Find the eigenfunctions for this problem and the equation which the eigenvalues must satisfy. To do this, consider the eigenvalues and eigenfunctions for, y + y = 0, y(0) = 0, y(1) + y (1) = 0.

Show that the transcendental equation for has innitely many roots 1 < 2 < 3 < . Find the limit of n as n . How is this limit approached? Give the general solution of Equation 29.3 in terms of the eigenfunctions. Hint, Solution Exercise 29.6 Consider y + y = f (x) y(0) = 0 y(1) + y (1) = 0. Find the eigenfunctions for this problem and the equation which the eigenvalues satisfy. Give the general solution in terms of these eigenfunctions. Hint, Solution 1443

Exercise 29.7 Show that the eigenvalue problem, y + y = 0, y(0) = 0, y (0) y(1) = 0,

(note the mixed boundary condition), has only one real eigenvalue. Find it and the corresponding eigenfunction. Show that this problem is not self-adjoint. Thus the proof, valid for unmixed, homogeneous boundary conditions, that all eigenvalues are real fails in this case. Hint, Solution Exercise 29.8 Determine the Rayleigh quotient, R[] for, 1 y + y + y = 0, x |y(0)| < , y(1) = 0.

Use the trial function = 1 x in R[] to deduce that the smallest zero of J0 (x), the Bessel function of the rst kind and order zero, is less than 6. Hint, Solution Exercise 29.9 Discuss the eigenvalues of the equation y + q(z)y = 0, where q(z) = a > 0, 0 z l b > 0, l < z . y(0) = y() = 0

This is an example that indicates that the results we obtained in class for eigenfunctions and eigenvalues with q(z) continuous and bounded also hold if q(z) is simply integrable; that is

|q(z)| dz
0

1444

is nite. Hint, Solution Exercise 29.10 1. Find conditions on the smooth real functions p(x), q(x), r(x) and s(x) so that the eigenvalues, , of: Lv (p(x)v (x)) (q(x)v (x)) + r(x)v(x) = s(x)v(x), v(a) = v (a) = 0 v (b) = 0, p(b)v (b) q(b)v (b) = 0 are positive. Prove the assertion. 2. Show that for any smooth p(x), q(x), r(x) and s(x) the eigenfunctions belonging to distinct eigenvalues are orthogonal relative to the weight s(x). That is:
b

a<x<b

vm (x)vk (x)s(x) dx = 0 if k = m .
a

3. Find the eigenvalues and eigenfunctions for: d4 = , dx4 Hint, Solution (0) = (0) = 0, (1) = (1) = 0.

1445

29.5
Hint 29.1 Hint 29.2 Hint 29.3

Hints

Hint 29.4 Write the problem in Sturm-Liouville form to show that the eigenfunctions are orthogonal with respect to the weighting function = ex . Hint 29.5 Note that the solution is a regular Sturm-Liouville problem and thus the eigenvalues are real. Use the Rayleigh quotient to show that there are only positive eigenvalues. Informally show that there are an innite number of eigenvalues with a graph. Hint 29.6 Hint 29.7 Find the solution for = 0, < 0 and > 0. A problem is self-adjoint if it satises Greens identity. Hint 29.8 Write the equation in self-adjoint form. The Bessel equation of the rst kind and order zero satises the problem, 1 y + y + y = 0, x |y(0)| < , y(r) = 0,

where r is a positive root of J0 (x). Make the change of variables = x/r, u() = y(x). 1446

Hint 29.9 Hint 29.10

1447

29.6

Solutions

Solution 29.1 Recall that constant coecient equations are shift invariant. If u(x) is a solution, then so is u(x c). We substitute y = ex into the constant coecient equation. y + 2y + y = 0 2 + 2 + = 0 = 2 First we consider the case = 2 . A set of solutions of the dierential equation is ex , x ex The homogeneous solution that satises the left boundary condition y(a) = 0 is y = c(x a) ex . Since only the trivial solution with c = 0 satises the right boundary condition, = 2 is not an eigenvalue. Next we consider the case = 2 . We write = 2 . Note that ( 2 ) 0. A set of solutions of the dierential equation is e(
2 )x

By taking the sum and dierence of these solutions we obtain a new set of linearly independent solutions. ex cos 2 x , ex sin 2 x The solution which satises the left boundary condition is y = c ex sin 2 (x a) . 1448

For nontrivial solutions, the right boundary condition y(b) = 0 imposes the constraint eb sin 2 (b a) = 0 2 (b a) = n, n Z We have the eigenvalues n = + with the eigenfunctions n = ex sin n xa ba .
2 2

n ba

nZ

To write the problem in Sturm-Liouville form, we multiply by the integrating factor e e2x y
2 dx

= e2x . y(a) = y(b) = 0

+ e2x y = 0,

Now we verify that the Sturm-Liouville properties are satised. The eigenvalues n = 2 + are real. There are an innite number of eigenvalues 1 < 2 < 3 < , 2 + ba
2

n ba

nZ

< 2 +

2 ba 1449

< 2 +

3 ba

< .

There is a least eigenvalue 1 = +


2

ba

but there is no greatest eigenvalue, (n as n ). For each eigenvalue, we found one unique, (to within a multiplicative constant), eigenfunction n . We were able to choose the eigenfunctions to be real-valued. The eigenfunction n = ex sin n has exactly n 1 zeros in the open interval a < x < b. The eigenfunctions are orthogonal with respect to the weighting function (x) = e2ax .
b a

xa ba

n (x)m (x)(x) dx =

x a 2ax x a x e e sin m dx ba ba a b xa xa = sin n sin m dx ba ba a ba = sin(nx) sin(mx) dx 0 ba = (cos((n m)x) cos((n + m)x)) dx 2 0 = 0 if n = m ex sin n

The eigenfunctions are complete. Any piecewise continuous function f (x) dened on a x b can be expanded in a series of eigenfunctions

f (x)
n=1

cn n (x),

1450

where cn =
b a

f (x)n (x)(x) dx
b a

2 (x)(x) dx n

The sum converges to 1 (f (x ) + f (x+ )). (We do not prove this property.) 2 The eigenvalues can be related to the eigenfunctions with the Rayleigh quotient. pn dn dx
b a b a

n =

b a b a

dn 2 dx

q2 dx n
2

2 dx n cos n xa sin n xa ba ba
2 2x e

e2x ex
b a

n ba

dx

ex sin n xa ba
b a

dx dx

b a

n 2 ba

n cos2 n xa 2 ba cos n xa sin n xa + 2 sin2 n xa ba ba ba ba

sin2 n xa dx ba

n 2 ba

n cos2 (x) 2 ba cos(x) sin(x) + 2 sin2 (x) dx 0

sin2 (x) dx

= 2 +

n ba

Now we expand a function f (x) in a series of the eigenfunctions.

f (x)
n=1

cn ex sin n

xa ba

1451

where cn = =
b a

f (x)n (x)(x) dx
b a

2 (x)(x) dx n
b

2n ba

f (x) ex sin n
a

xa ba

dx

Solution 29.2 This is an Euler equation. We substitute y = (x + 1) into the equation. y + y=0 (x + 1)2 ( 1) + = 0 1 1 4 = 2

First consider the case = 1/4. A set of solutions is Another set of solutions is x + 1, x + 1 ln(x + 1) .

x + 1,

x + 1 ln

x+1 2

The solution which satises the boundary condition y(1) = 0 is y = c x + 1 ln x+1 2 .

Since only the trivial solution satises the y(2) = 0, = 1/4 is not an eigenvalue. 1452

Now consider the case = 1/4. A set of solutions is (x + 1)(1+


14)/2

, (x + 1)(1

14)/2

We can write this in terms of the exponential and the logarithm. 4 1 4 1 ln(x + 1) x + 1 exp ln(x + 1) , x + 1 exp 2 2 Note that

x+1 x+1 4 1 4 1 ln . x + 1 exp ln , x + 1 exp 2 2 2 2 is also a set of solutions. The new factor of 2 in the logarithm just multiplies the solutions by a constant. We write the solution in terms of the cosine and sine. x+1 x+1 4 1 4 1 ln . x + 1 cos ln , x + 1 sin 2 2 2 2 The solution of the dierential equation which satises the boundary condition y(1) = 0 is 1 4 x+1 y = c x + 1 sin ln . 2 2 Now we use the second boundary condition to nd the eigenvalues. y(2) = 0 =0 nZ , nZ

4 1 3 ln sin 2 2 4 1 3 ln = n, 2 2 = 1 4 1+ 2n ln(3/2)
2

1453

n = 0 gives us a trivial solution, so we discard it. Discarding duplicate solutions, The eigenvalues and eigenfunctions are 2 n 1 ln((x + 1)/2) n = + , yn = x + 1 sin n , n Z+ . 4 ln(3/2) ln(3/2) Now we verify that the eigenvalues and eigenfunctions satisfy the properties of regular Sturm-Liouville problems. The eigenvalues are real. There are an innite number of eigenvalues 1 < 2 < 3 < 1 + 4 ln(3/2)
2 2 2

1 < + 4

2 ln(3/2) 1 + 4

1 < + 4
2

3 ln(3/2)

<

There is a least least eigenvalue 1 = but there is no greatest eigenvalue. The eigenfunctions are orthogonal with respect to the weighting function (x) = 1/(x + 1)2 . Let n = m.
2

ln(3/2)

yn (x)ym (x)(x) dx
1

ln((x + 1)/2) ln((x + 1)/2) x + 1 sin m ln(3/2) ln(3/2) 1 ln(3/2) = sin(nx) sin(mx) dx 0 ln(3/2) = (cos((n m)x) cos((n + m)x)) dx 2 0 =0
2

x + 1 sin n

1 dx (x + 1)2

1454

The eigenfunctions are complete. A function f (x) dened on (1 . . . 2) has the series representation

f (x)
n=1

cn yn (x) =

ln((x + 1)/2) cn x + 1 sin n ln(3/2) n=1


2

where

yn |1/(x + 1)2 |f 2 cn = = yn |1/(x + 1)2 |yn ln(3/2)

sin n
1

ln((x + 1)/2) ln(3/2)

1 f (x) dx (x + 1)3/2

Solution 29.3 Recall that Euler equations are scale invariant. If u(x) is a solution, then so is u(cx) for any nonzero constant c. We substitute y = x into the Euler equation. 2 + 1 y + 2y = 0 x x ( 1) + (2 + 1) + = 0 2 + 2 + = 0 = 2 y + First we consider the case = 2 . A set of solutions of the dierential equation is x , x ln x The homogeneous solution that satises the left boundary condition y(a) = 0 is y = cx (ln x ln a) = cx ln x . a

Since only the trivial solution with c = 0 satises the right boundary condition, = 2 is not an eigenvalue. Next we consider the case = 2 . We write = 2 . 1455

Note that

( 2 ) 0. A set of solutions of the dierential equation is x x e


2

2 ln x

By taking the sum and dierence of these solutions we obtain a new set of linearly independent solutions. x cos 2 ln x , x sin 2 ln x , The solution which satises the left boundary condition is x y = cx sin 2 ln a

For nontrivial solutions, the right boundary condition y(b) = 0 imposes the constraint b sin 2 ln 2 ln b a = n, b a nZ

We have the eigenvalues n = + with the eigenfunctions n = x sin n ln(x/a) ln(b/a) .


2

n ln(b/a)

nZ

To write the problem in Sturm-Liouville form, we multiply by the integrating factor e


(2+1)/x dx

= e(2+1) ln x = x2+1 . 1456

x2+1 y

+ x21 y = 0,

y(a) = y(b) = 0

Now we verify that the Sturm-Liouville properties are satised. The eigenvalues n = 2 + are real. There are an innite number of eigenvalues 1 < 2 < 3 < , + There is a least eigenvalue 1 = +
2 2

n ln(b/a)

nZ

ln(b/a)

< +

2 ln(b/a)

< +

3 ln(b/a)

<

ln(b/a)

but there is no greatest eigenvalue, (n as n ). For each eigenvalue, we found one unique, (to within a multiplicative constant), eigenfunction n . We were able to choose the eigenfunctions to be real-valued. The eigenfunction n = x sin n has exactly n 1 zeros in the open interval a < x < b. 1457 ln(x/a) ln(b/a) .

The eigenfunctions are orthogonal with respect to the weighting function (x) = x21 .
b a b

n (x)m (x)(x) dx =

ln(x/a) ln(x/a) x sin m x21 dx ln(b/a) ln(b/a) a b ln(x/a) ln(x/a) 1 = sin m dx sin n ln(b/a) ln(b/a) x a ln(b/a) = sin(nx) sin(mx) dx 0 ln(b/a) = (cos((n m)x) cos((n + m)x)) dx 2 0 = 0 if n = m x sin n

The eigenfunctions are complete. Any piecewise continuous function f (x) dened on a x b can be expanded in a series of eigenfunctions

f (x)
n=1

cn n (x),

where cn =
b a

f (x)n (x)(x) dx
b a

2 (x)(x) dx n

The sum converges to 1 (f (x ) + f (x+ )). (We do not prove this property.) 2 1458

The eigenvalues can be related to the eigenfunctions with the Rayleigh quotient. n = pn dn dx
b a

b a b a

dn 2 dx

q2 dx n
2

2 dx n
n ln(b/a) b a

b a

x2+1 x1

cos n ln(x/a) sin n ln(x/a) ln(b/a) ln(b/a)


2

dx

= x sin n ln(x/a) ln(b/a)


b a 2

x21 dx

n ln(b/a)

n cos2 () 2 ln(b/a) cos () sin () + 2 sin2 () x1 dx b a

=
0 n ln(b/a) 2

sin2 n ln(x/a) x1 dx ln(b/a) dx

n cos2 (x) 2 ln(b/a) cos(x) sin(x) + 2 sin2 (x) 0

= = +
2

sin2 (x) dx

n ln(b/a)

Now we expand a function f (x) in a series of the eigenfunctions.

f (x)
n=1

cn x sin n

ln(x/a) ln(b/a)

where cn = =
b a

f (x)n (x)(x) dx
b a

2 (x)(x) dx n
b

2n ln(b/a)

f (x)x1 sin n
a

ln(x/a) ln(b/a)

dx

1459

Solution 29.4 y y + y = 0,
x

y(0) = y(1) = 0.

The factor that will put this equation in Sturm-Liouville form is F (x) = exp The dierential equation becomes d x e y + ex y = 0. dx Thus we see that the eigenfunctions will be orthogonal with respect to the weighting function = ex . Substituting y = ex into the dierential equation yields 2 + = 0 1 1 4 = 2 1 = 1/4 . 2 If < 1/4 then the solutions to the dierential equation are exponential and only the trivial solution satises the boundary conditions. If = 1/4 then the solution is y = c1 ex/2 +c2 x ex/2 and again only the trivial solution satises the boundary conditions. Now consider the case that > 1/4. 1 = 1/4 2 The solutions are ex/2 cos( 1/4 x), ex/2 sin( 1/4 x). The left boundary condition gives us y = c ex/2 sin( 1/4 x). 1 dx = ex .

1460

The right boundary condition demands that 1/4 = n, Thus we see that the eigenvalues and eigenfunctions are n = 1 + (n)2 , 4 yn = ex/2 sin(nx). n = 1, 2, . . .

If f (x) is a piecewise continuous function then we can expand it in a series of the eigenfunctions.

f (x) =
n=1

an ex/2 sin(nx)

The coecients are an = = =2 Solution 29.5 Consider the eigenvalue problem y + y = 0 y(0) = 0 y(1) + y (1) = 0. Since this is a Sturm-Liouville problem, there are only real eigenvalues. By the Rayleigh quotient, the eigenvalues are d dx
1 1 0

f (x) ex ex/2 sin(nx) dx

1 x x/2 e (e sin(nx))2 dx 0 1 f (x) ex/2 sin(nx) dx 0 1 sin2 (nx) dx 0 1 x/2

f (x) e

sin(nx) dx.

+
0 1 0

1 0

d 2 dx

dx ,

2 dx

1461

2 (1) + =

1 0 1 0

d 2 dx

dx .

2 dx

This demonstrates that there are only positive eigenvalues. The general solution of the dierential equation for positive, real is y = c1 cos x + c2 sin x . The solution that satises the left boundary condition is y = c sin For nontrivial solutions we must have sin x .

+ cos =0 = tan .

x . In Figure 29.1 The positive solutions of this equation are eigenvalues with corresponding eigenfunctions sin we plot the functions x and tan(x) and draw vertical lines at x = (n 1/2), n N. From this we see that there are an innite number of eigenvalues, 1 < 2 < 3 < . In the limit as n , n (n 1/2). The limit is approached from above. Now consider the eigenvalue problem y + y = y From above we see that the eigenvalues satisfy 1 = tan 1 y(0) = 0 y(1) + y (1) = 0.

and that there are an innite number of eigenvalues. For large n, n 1 (n 1/2). The eigenfunctions are n = sin 1 n x . 1462

Figure 29.1: x and tan(x). To solve the inhomogeneous problem, we expand the solution and the inhomogeneity in a series of the eigenfunctions.

f=
n=1

fn n , y=

fn =

1 0

f (x)n (x) dx
1 0

2 (x) dx n

yn n
n=1

We substitite the expansions into the dierential equation to determine the coecients. y +y =f

n yn n =
n=1 n=1

fn n 1 n x

y=
n=1

fn sin n

1463

Solution 29.6 Consider the eigenvalue problem y + y = y y(0) = 0 y(1) + y (1) = 0. From Exercise 29.5 we see that the eigenvalues satisfy 1 = tan 1

and that there are an innite number of eigenvalues. For large n, n 1 (n 1/2). The eigenfunctions are n = sin 1 n x .

To solve the inhomogeneous problem, we expand the solution and the inhomogeneity in a series of the eigenfunctions.

f=
n=1

fn n , y=

fn =

1 0

f (x)n (x) dx
1 0

2 (x) dx n

yn n
n=1

We substitite the expansions into the dierential equation to determine the coecients. y +y =f

n yn n =
n=1 n=1

fn n 1 n x

y=
n=1

fn sin n

Solution 29.7 First consider = 0. The general solution is y = c1 + c2 x. 1464

y = cx satises the boundary conditions. Thus = 0 is an eigenvalue. Now consider negative real . The general solution is y = c1 cosh x + c2 sinh x .

The solution that satises the left boundary condition is y = c sinh x .

For nontrivial solutions of the boundary value problem, there must be negative real solutions of sinh = 0.

Since x = sinh x has no nonzero real solutions, this equation has no solutions for negative real . There are no negative real eigenvalues. Finally consider positive real . The general solution is y = c1 cos x + c2 sin The solution that satises the left boundary condition is y = c sin x . For nontrivial solutions of the boundary value problem, there must be positive real solutions of sin = 0. x .

Since x = sin x has no nonzero real solutions, this equation has no solutions for positive real . There are no positive real eigenvalues. There is only one real eigenvalue, = 0, with corresponding eigenfunction = x. 1465

The diculty with the boundary conditions, y(0) = 0, y (0) y(1) = 0 is that the problem is not self-adjoint. We demonstrate this by showing that the problem does not satisfy Greens identity. Let u and v be two functions that satisfy the boundary conditions, but not necessarily the dierential equation. u, L[v] L[u], v = u, v
1 ]0

u ,v
1

= [uv u , v u , v [u v]0 + u , v u , v = u(1)v (1) u (1)v(1) Greens identity is not satised, u, L[v] L[u], v = 0; The problem is not self-adjoint. Solution 29.8 First we write the equation in formally self-adjoint form, L[y] (xy ) = xy, |y(0)| < , y(1) = 0.

Let be an eigenvalue with corresponding eigenfunction . We derive the Rayleigh quotient for . , L[] = , x , (x ) = , x [x ]0 , x = , x We apply the boundary conditions and solve for . = , x , x
1

The Bessel equation of the rst kind and order zero satises the problem, 1 y + y + y = 0, x |y(0)| < , 1466 y(r) = 0,

where r is a positive root of J0 (x). We make the change of variables = x/r, u() = y(x) to obtain the problem 1 1 1 u + u + u = 0, 2 r r r |u(0)| < , u(1) = 0,

1 u + u + r2 u = 0, |u(0)| < , u(1) = 0. Now r2 is the eigenvalue of the problem for u(). From the Rayleigh quotient, the minimum eigenvalue obeys the inequality , x r2 , , x where is any test function that satises the boundary conditions. Taking = 1 x we obtain, = 6, x)x(1 x) dx r 6 Thus the smallest zero of J0 (x) is less than or equal to 6 2.4494. (The smallest zero of J0 (x) is approximately 2.40483.)
1 (1 0

r2

1 (1)x(1) dx 0

Solution 29.9 We assume that 0 < l < . Recall that the solution of a second order dierential equation with piecewise continuous coecient functions is piecewise C 2 . This means that the solution is C 2 except for a nite number of points where it is C 1 . First consider the case = 0. A set of linearly independent solutions of the dierential equation is {1, z}. The solution which satises y(0) = 0 is y1 = c1 z. The solution which satises y() = 0 is y2 = c2 ( z). There is a solution for the problem if there are there are values of c1 and c2 such that y1 and y2 have the same position and slope at z = l. y1 (l) = y2 (l), y1 (l) = y2 (l) c1 l = c2 ( l), c1 = c2 1467

Since there is only the trivial solution, c1 = c2 = 0, = 0 is not an eigenvalue. Now consider = 0. For 0 z l a set of linearly independent solutions is cos( az), sin( az) . The solution which satises y(0) = 0 is y1 = c1 sin( az).

For l < z a set of linearly independent solutions is cos( bz), sin( bz) . The solution which satises y() = 0 is y2 = c2 sin( b( z)).

= 0 is an eigenvalue if there are nontrivial solutions of y1 (l) = y2 (l), y1 (l) = y2 (l) c1 sin( al) = c2 sin( b( l)), c1 a cos( al) = c2 b cos( b( l)) We divide the second equation by () since = 0 and write this as a linear algebra problem. sin( al) sin( b( l)) c1 0 = c2 0 a cos( al) b sin( b( l))

This system of equations has nontrivial solutions if and only if the determinant of the matrix is zero. b sin( al) sin( b( l)) + a cos( al) sin( b( l)) = 0 We can use trigonometric identities to write this equation as ( b a) sin (l a ( l) b) + ( b + a) sin (l a + ( l) b) = 0

1468

Clearly this equation has an innite number of solutions for real, positive . However, it is not clear that this equation does not have non-real solutions. In order to prove that, we will show that the problem is self-adjoint. Before going on to that we note that the eigenfunctions have the form

n (z) =

sin an z sin bn ( z)

0zl l < z .

Now we prove that the problem is self-adjoint. We consider the class of functions which are C 2 in (0 . . . ) except at the interior point x = l where they are C 1 and which satisfy the boundary conditions y(0) = y() = 0. Note that the dierential operator is not dened at the point x = l. Thus Greens identity,

u|q|Lv = Lu|q|v

is not well-dened. To remedy this we must dene a new inner product. We choose

u|v
0

uv dx +
l

uv dx.

This new inner product does not require dierentiability at the point x = l. The problem is self-adjoint if Greens indentity is satised. Let u and v be elements of our class of functions. In 1469

addition to the boundary conditions, we will use the fact that u and v satisfy y(l ) = y(l+ ) and y (l ) = y (l+ ).
l

v|Lu =
0

vu dx +
l l l 0 l

vu dx

= [vu ]0 = v(l)u (l)

v u dx + [vu ]l
l

v u dx

v u dx v(l)u (l)
0 l

v u dx

=
0

v u dx
l l l 0 l

v u dx

= [v u]0 + = v (l)u(l) +

v u dx [v u]l +
l

v u dx

v u dx + v (l)u(l) +
0 l

v u dx

=
0

v u dx +
l

v u dx

= Lv|Lu The problem is self-adjoint. Hence the eigenvalues are real. There are an innite number of positive, real eigenvalues n . Solution 29.10 1. Let v be an eigenfunction with the eigenvalue . We start with the dierential equation and then take the inner product with v. (pv ) (qv ) + rv = sv v, (pv ) (qv ) + rv = v, sv 1470

We use integration by parts and utilize the homogeneous boundary conditions. [v(pv ) ]a v , (pv ) [vqv ]a + v , qv + v, rv = v, sv [v pv ]a + v , pv + v , qv + v, rv = v, sv v , pv + v , qv + v, rv = v, sv We see that if p, q, r, s 0 then the eigenvalues will be positive. (Of course we assume that p and s are not identically zero.) 2. First we prove that this problem is self-adjoint. Let u and v be functions that satisfy the boundary conditions, but do not necessarily satsify the dierential equation. v, L[u] L[v], u = v, (pu ) (qu ) + ru (pv ) (qv ) + rv, u Following our work in part (a) we use integration by parts to move the derivatives. = ( v , pu =0 This problem satises Greens identity, v, L[u] L[v], u = 0, and is thus self-adjoint. Let vk and vm be eigenfunctions corresponding to the distinct eigenvalues k and m . We start with Greens identity. vk , L[vm ] L[vk ], vm vk , m svm k svk , vm (m k ) vk , svm vk , svm =0 =0 =0 =0 + v , qu + v, ru ) ( pv , u + qv , u + rv, u )
b b b

The eigenfunctions are orthogonal with respect to the weighting function s. 1471

3. From part (a) we know that there are only positive eigenvalues. The general solution of the dierential equation is = c1 cos(1/4 x) + c2 cosh(1/4 x) + c3 sin(1/4 x) + c4 sinh(1/4 x). Applying the condition (0) = 0 we obtain = c1 (cos(1/4 x) cosh(1/4 x)) + c2 sin(1/4 x) + c3 sinh(1/4 x). The condition (0) = 0 reduces this to = c1 sin(1/4 x) + c2 sinh(1/4 x). We substitute the solution into the two right boundary conditions. c1 sin(1/4 ) + c2 sinh(1/4 ) = 0 c1 1/2 sin(1/4 ) + c2 1/2 sinh(1/4 ) = 0 We see that sin(1/4 ) = 0. The eigenvalues and eigenfunctions are n = (n)4 , n = sin(nx), n N.

1472

Chapter 30 Integrals and Convergence


Never try to teach a pig to sing. It wastes your time and annoys the pig. -?

30.1

Uniform Convergence of Integrals

Consider the improper integral


c

f (x, t) dt. The integral is convergent to S(x) if, given any > 0, there exists T (x, ) such that

f (x, t) dt S(x) <


c

for all > T (x, ).

The sum is uniformly convergent if T is independent of x. Similar to the Weierstrass M-test for innite sums we have a uniform convergence test for integrals. If there exists a continuous function M (t) such that |f (x, t)| M (t) and c M (t) dt is convergent, then c f (x, t) dt is uniformly convergent. 1473

If

f (x, t) dt is uniformly convergent, we have the following properties:

If f (x, t) is continuous for x [a, b] and t [c, ) then for a < x0 < b,
xx0

lim

f (x, t) dt =
c c

xx0

lim f (x, t)

dt.

If a x1 < x2 b then we can interchange the order of integration.


x2 x1 c x2

f (x, t) dt If
f x

dx =
c x1

f (x, t) dx

dt

is continuous, then d dx

f (x, t) dt =
c c

f (x, t) dt. x

30.2

The Riemann-Lebesgue Lemma


b a |f (x)| dx

Result 30.2.1 If

exists, then
b

f (x) sin(x) dx 0 as .
a

Before we try to justify the Riemann-Lebesgue lemma, we will need a preliminary result. Let be a positive constant.
b

sin(x) dx =
a

1 cos(x) 2 .

b a

1474

We will prove the Riemann-Lebesgue lemma for the case when f (x) has limited total uctuation on the interval (a, b). We can express f (x) as the dierence of two functions f (x) = + (x) (x), where + and are positive, increasing, bounded functions. From the mean value theorem for positive, increasing functions, there exists an x0 , a x0 b, such that
b b

+ (x) sin(x) dx = + (b)


a x0

sin(x) dx

2 |+ (b)| . Similarly,
b a

2 (x) sin(x) dx | (b)| .

Thus
b a

f (x) sin(x) dx

2 (|+ (b)| + | (b)|) 0 as .

30.3
30.3.1

Cauchy Principal Value


Integrals on an Innite Domain

The improper integral

f (x) dx is dened
0 b

f (x) dx = lim

f (x) dx + lim
a

f (x) dx,
0

1475

when these limits exist. The Cauchy principal value of the integral is dened
a

PV

f (x) dx = lim

f (x) dx.
a

The principal value may exist when the integral diverges. Example 30.3.1

x dx diverges, but
a

PV

x dx = lim

x dx = lim (0) = 0.
a a

If the improper integral converges, then the Cauchy principal value exists and is equal to the value of the integral. The principal value of the integral of an odd function is zero. If the principal value of the integral of an even function exists, then the integral converges.

30.3.2

Singular Functions
b a b

Let f (x) have a singularity at x = 0. Let a and b satisfy a < 0 < b. The integral of f (x) is dened
1

f (x) dx = lim
1 0

f (x) dx + lim+
2 0 2

f (x) dx,

when the limits exist. The Cauchy principal value of the integral is dened
b b

PV
a

f (x) dx = lim +
0

f (x) dx +
a

f (x) dx ,

when the limit exists. Example 30.3.2 The integral


2 1

1 dx x

1476

diverges, but the principal value exists.


2

PV
1

1 dx = lim 0+ x
0

= lim + = 1 dx 1 x = log 2
2

1 dx + x 1 1 dx + x

1 dx x 2 1 dx x

1477

Chapter 31 The Laplace Transform


31.1 The Laplace Transform

The Laplace transform of the function f (t) is dened L[f (t)] =


0

est f (t) dt,

for all values of s for which the integral exists. The Laplace transform of f (t) is a function of s which we will denote f (s). 1 A function f (t) is of exponential order if there exist constants t0 and M such that |f (t)| < M et , for all t > t0 . (s) > .

t If 0 0 f (t) dt exists and f (t) is of exponential order then the Laplace transform f (s) exists for Here are a few examples of these concepts.

sin t is of exponential order 0.


1

Denoting the Laplace transform of f (t) as F (s) is also common.

1478

t e2t is of exponential order for any > 2. et is not of exponential order for any . tn is of exponential order for any > 0. t2 does not have a Laplace transform as the integral diverges. Example 31.1.1 Consider the Laplace transform of f (t) = 1. Since f (t) = 1 is of exponential order for any > 0, the Laplace transform integral converges for (s) > 0.

f (s) =
0

est dt
0

1 = est s 1 = s

Example 31.1.2 The function f (t) = t et is of exponential order for any > 1. We compute the Laplace transform of this function.

f (s) =
0

est t et dt t e(1s)t dt
0

1 1 e(1s)t dt = t e(1s)t 1s 1s 0 0 1 e(1s)t = (1 s)2 0 1 for (s) > 1. = (1 s)2

1479

Example 31.1.3 Consider the Laplace transform of the Heaviside function, H(t c) = where c > 0.

0 1

for t < c for t > c,

L[H(t c)] =
0

est H(t c) dt est dt


c

= =

est s

ecs = s Example 31.1.4 Next consider H(t c)f (t c).

for

(s) > 0

L[H(t c)f (t c)] =


0

est H(t c)f (t c) dt est f (t c) dt


c

= = =e
0 cs

es(t+c) f (t) dt f (s)

31.2

The Inverse Laplace Transform


f (t) = L1 [f (s)]. 1480

The inverse Laplace transform in denoted

We compute the inverse Laplace transform with the Mellin inversion formula. f (t) = 1 2
+

est f (s) ds

Here is a real constant that is to the right of the singularities of f (s). To see why the Mellin inversion formula is correct, we take the Laplace transform of it. Assume that f (t) is of exponential order . Then will be to the right of the singularities of f (s). L[L1 [f (s)]] = L =
0

1 2

ezt f (z) dz

est

1 2

ezt f (z) dz dt

We interchange the order of integration. 1 = 2 Since (z) = , the integral in t exists for (s) > . 1 = 2
+ +

f (z)
0

e(zs)t dt dz

f (z) dz sz

We would like to evaluate this integral by closing the path of integration with a semi-circle of radius R in the right half plane and applying the residue theorem. However, in order for the integral along the semi-circle to vanish as R , f (z) must vanish as |z| . If f (z) vanishes we can use the maximum modulus bound to show that the integral along the semi-circle vanishes. This we assume that f (z) vanishes at innity. Consider the integral, f (z) 1 dz, 2 C s z 1481

Im(z) +iR s Re(z)

-iR
Figure 31.1: The Laplace Transform Pair Contour.

where C is the contour that starts at R, goes straight up to + R, and then follows a semi-circle back down to R. This contour is shown in Figure 31.1. If s is inside the contour then 1 f (z) dz = f (s). 2 C s z Note that the contour is traversed in the negative direction. Since f (z) decays as |z| , the semicircular contribution to the integral will vanish as R . Thus 1 2 Therefore, we have shown than L[L1 [f (s)]] = f (s). f (t) and f (s) are known as Laplace transform pairs. 1482
+

f (z) dz = f (s). sz

31.2.1

f (s) with Poles

Example 31.2.1 Consider the inverse Laplace transform of 1/s2 . s = 1 is to the right of the singularity of 1/s2 .
1+

L1

1 1 = 2 s 2

est
1

1 ds s2

Let BR be the contour starting at 1 R and following a straight line to 1 + R; let CR be the contour starting at 1 + R and following a semicircular path down to 1 R. Let C be the combination of BR and CR . This contour is shown in Figure 31.2.

Im(s) +iR CR BR Re(s)

-iR
Figure 31.2: The Path of Integration for the Inverse Laplace Transform.

1483

Consider the line integral on C for R > 1. 1 2 est


C

1 1 ds = Res est 2 , 0 2 s s d st e = ds s=0 =t

If t 0, the integral along CR vanishes as R . We parameterize s. s = 1 + R e , est = et(1+R e


)

3 2 2 = et etR cos et

est
CR

1 ds s2

1 ds s2 CR 1 R et (R 1)2 0 as R est

Thus the inverse Laplace transform of 1/s2 is L1 1 = t, s2 for t 0.

Let f (s) be analytic except for isolated poles at s1 , s2 , . . . , sN and let be to the right of these poles. Also, let (s) 0 as |s| . Dene BR to be the straight line from R to + R and CR to be the semicircular path f 1484

from + R to R. If R is large enough to enclose all the poles, then 1 2 1 2


N

e f (s) ds =
st BR +CR N n=1

Res(est f (s), sn ) est f (s) ds.


CR

est f (s) ds =
BR

1 Res(est f (s), sn ) 2 n=1

Now lets examine the integral along CR . Let the maximum of |f (s)| on CR be MR . We can parameterize the contour with s = + R e , /2 < < 3/2.
3/2

e f (s) ds =
st CR /2 3/2

et(+R e

i )

f ( + R e )R e d

/2

et etR cos RMR d

= RMR et
0

etR sin d

If t 0 we can use Jordans Lemma to obtain, . < RMR et tR = MR et t We use that MR 0 as R . 0 as R 1485

Thus we have an expression for the inverse Laplace transform of f (s). 1 2


+ N

e f (s) ds =
st N n=1

Res(est f (s), sn )

L1 [f (s)] =
n=1

Res(est f (s), sn )

Result 31.2.1 If f (s) is analytic except for poles at s1 , s2 , . . . , sN and f (s) 0 as |s| then the inverse Laplace transform of f (s) is
N

f (t) = L [f (s)] =
1 n=1

Res(est f (s), sn ),

for t > 0.

1 Example 31.2.2 Consider the inverse Laplace transform of s3 s2 . First we factor the denominator. 1 1 1 = 2 . 3 s2 s s s1

Taking the inverse Laplace transform, L1 1 1 1 1 1 = Res est 2 , 0 + Res est 2 ,1 3 s s s1 s s1 d est = + et ds s 1 s=0 t 1 + + et = 2 (1) 1 1486

s3

Thus we have that L1 1 = et t 1, s3 s2 for t > 0.

Example 31.2.3 Consider the inverse Laplace transform of s2 + s 1 . s3 2s2 + s 2 We factor the denominator. s2 + s 1 . (s 2)(s )(s + ) Then we take the inverse Laplace transform. L
1

s2 + s 1 s2 + s 1 s2 + s 1 est est = Res , 2 + Res , s3 2s2 + s 2 (s 2)(s )(s + ) (s 2)(s )(s + ) s2 + s 1 , + Res est (s 2)(s )(s + ) 1 1 = e2t + et + et 2 2

Thus we have L1 s2 + s 1 = sin t + e2t , s3 2s2 + s 2 for t > 0.

1487

31.2.2

f (s) with Branch Points


1 . s

Example 31.2.4 Consider the inverse Laplace transform of branch cut from s = 0 to s = and e/2 1 = , s r

s denotes the principal branch of s1/2 . There is a

for < < .


1 s

Let be any positive number. The inverse Laplace transform of 1 2


+

is

f (t) =

1 est ds. s

We will evaluate the integral by deforming it to wrap around the branch cut. Consider the integral on the contour + shown in Figure 31.3. CR and CR are circular arcs of radius R. B is the vertical line at (s) = joining the two arcs. C is a semi-circle in the right half plane joining and . L+ and L are lines joining the circular arcs at (s) = . Since there are no residues inside the contour, we have 1 2 +
B
+ CR

+
L+

+
C

+
L

+
CR

1 est ds = 0. s

We will evaluate the inverse Laplace transform for t > 0. + First we will show that the integral along CR vanishes as R .
/2

ds =
+ CR

d +
/2 /2

d.

The rst integral vanishes by the maximum modulus bound. Note that the length of the path of integration is less than 1488

+ CR

B
+ L

/2 C /2+

LCR

Figure 31.3: Path of Integration for 1/ s

2.

/2

d
/2

1 max est + s sCR

(2)

1 = et (2) R 0 as R 1489

+ The second integral vanishes by Jordans Lemma. A parameterization of CR is s = R e .

eR e
/2

1 R e

d
/2

eR e

1 R e

1 R

eR cos()t d
/2 /2

1 eRt sin() d R 0 1 < R 2Rt 0 as R


We could show that the integral along CR vanishes by the same method. Now we have

1 2

+
B L+

+
C

+
L

1 est ds = 0. s

We can show that the integral along C vanishes as 0 with the maximum modulus bound. 1 est ds s 1 max est sC s 1 <et 0 as 0 Now we can express the inverse Laplace transform in terms of the integrals along L+ and L . 1 f (t) 2
+

( )

1 1 est ds = 2 s 1490

1 1 est ds 2 s L+

1 est ds s L

On L+ , s = r e , ds = e dr = dr; on L , s = r e , ds = e dr = dr. We can combine the integrals along the top and bottom of the branch cut.
0 1 1 ert (1) dr f (t) = 2 2 r 1 2 ert dr = 2 0 r 0

ert (1) dr r

We make the change of variables x = rt. 1 = t We recognize this integral as (1/2). 1 = (1/2) t 1 = t Thus the inverse Laplace transform of
1 s 0

1 ex dx x

is 1 f (t) = , t for t > 0.

31.2.3

Asymptotic Behavior of f (s)

Consider the behavior of f (s) =


0

est f (t) dt

1491

as s +. Assume that f (t) is analytic in a neighborhood of t = 0. Only the behavior of the integrand near t = 0 will make a signicant contribution to the value of the integral. As you move away from t = 0, the est term dominates. Thus we could approximate the value of f (s) by replacing f (t) with the rst few terms in its Taylor series expansion about the origin. t2 est f (0) + tf (0) + f (0) + dt as s + f (s) 2 0 Using n! L [tn ] = n+1 s we obtain f (0) f (0) f (0) f (s) + 2 + 3 + as s +. s s s Example 31.2.5 The Taylor series expansion of sin t about the origin is sin t = t t3 + O(t5 ). 6

Thus the Laplace transform of sin t has the behavior 1 1 L[sin t] 2 4 + O(s6 ) as s +. s s We corroborate this by expanding L[sin t]. L[sin t] = 1 s2 + 1 s2 = 1 + s2

= s2
n=0

(1)n s2n

1 1 = 2 4 + O(s6 ) s s 1492

31.3

Properties of the Laplace Transform

In this section we will list several useful properties of the Laplace transform. If a result is not derived, it is shown in the Problems section. Unless otherwise stated, assume that f (t) and g(t) are piecewise continuous and of exponential order . L[af (t) + bg(t)] = aL[f (t)] + bL[g(t)] L[ect f (t)] = f (s c) for s > c +
dn L[tn f (t)] = (1)n dsn [f (s)] for n = 1, 2, . . .

If

f (t) t 0

dt exists for positive then f (t) L = t

f () d.
s

L L L

t 0

f ( ) d =

f (s) s

d f (t) dt d2 f (t) dt2

= sf (s) f (0) = s2 f (s) sf (0) f (0) d f (t) = dt

To derive these formulas, L est f (t) dt


0 0

= est f (t)

s est f (t) dt

= f (0) + sf (s) L d2 f (t) = sL[f (t)] f (0) dt2 = s2 f (s) sf (0) f (0) 1493

Let f (t) and g(t) be continuous. The convolution of f (t) and g(t) is dened
t t

h(t) = (f g) =
0

f ( )g(t ) d =
0

f (t )g( ) d

The convolution theorem states g h(s) = f (s)(s). To show this,


t

h(s) =
0

est
0

f ( )g(t ) d dt est f ( )g(t ) dt d

=
0

=
0

es f ( )

es(t ) g(t ) dt d

=
0

es f ( ) d
0

es g() d

g = f (s)(s) If f (t) is periodic with period T then L[f (t)] =


T 0

est f (t) dt . 1 esT First we factor the denominator.

Example 31.3.1 Consider the inverse Laplace transform of

1 . s3 s2

s3

1 1 1 = 2 2 s s s1 1494

We know the inverse Laplace transforms of each term. L1 1 = t, s2 L1 1 = et s1

We apply the convolution theorem. L1 1 1 = s2 s 1


t

et d
0 t 0 t

= et e

et
0

e d

= t 1 + et

L1

1 1 = et t 1. s2 s 1

Example 31.3.2 We can nd the inverse Laplace transform of s2 + s 1 s3 2s2 + s 2 with the aid of a table of Laplace transform pairs. We factor the denominator. s2 + s 1 (s 2)(s )(s + ) 1495

We expand the function in partial fractions and then invert each term. s2 + s 1 1 /2 /2 = + (s 2)(s )(s + ) s2 s s+ 2 s +s1 1 1 = + 2 (s 2)(s )(s + ) s2 s +1 L1 1 1 + 2 = e2t + sin t s2 s +1

31.4

Constant Coecient Dierential Equations


y + y = cos t, for t > 0, y(0) = 1.

Example 31.4.1 Consider the dierential equation

We take the Laplace transform of this equation. s +1 1 s + y (s) = 2 + 1) (s + 1)(s s+1 1/2 1 s+1 + 2 y (s) = s+1 2s +1 s(s) y(0) + y (s) = y s2 Now we invert y (s). y(t) = 1 1 t 1 e + cos t + sin t, 2 2 2 for t > 0

Notice that the initial condition was included when we took the Laplace transform. 1496

One can see from this example that taking the Laplace transform of a constant coecient dierential equation reduces the dierential equation for y(t) to an algebraic equation for y (s). Example 31.4.2 Consider the dierential equation y + y = cos(2t), We take the Laplace transform of this equation. s2 y (s) sy(0) y (0) + y (s) = y (s) = (s2 s2 s +4 for t > 0, y(0) = 1, y (0) = 0.

s s + 2 2 + 4) + 1)(s s +1

From the table of Laplace transform pairs we know L1 s = cos t, s2 + 1 L1 1 1 = sin(2t). s2 + 4 2

We use the convolution theorem to nd the inverse Laplace transform of y (s).


t

y(t) =
0

1 sin(2 ) cos(t ) d + cos t 2

1 t sin(t + ) + sin(3 t) d + cos t = 4 0 t 1 1 = cos(t + ) cos(3 t) + cos t 4 3 0 1 1 1 cos(2t) + cos t cos(2t) + cos(t) + cos t = 4 3 3 1 4 = cos(2t) + cos(t) 3 3 1497

Alternatively, we can nd the inverse Laplace transform of y (s) by rst nding its partial fraction expansion. y (s) = s/3 s/3 s 2 + 2 +1 s +4 s +1 s/3 4s/3 = 2 + 2 s +4 s +1 1 4 y(t) = cos(2t) + cos(t) 3 3 s2

Example 31.4.3 Consider the initial value problem y + 5y + 2y = 0, Without taking a Laplace transform, we know that since y(t) = 1 + 2t + O(t2 ) the Laplace transform has the behavior y (s) 2 1 + 2 + O(s3 ), s s as s +. y(0) = 1, y (0) = 2.

31.5

Systems of Constant Coecient Dierential Equations

The Laplace transform can be used to transform a system of constant coecient dierential equations into a system of algebraic equations. This should not be surprising, as a system of dierential equations can be written as a single dierential equation, and vice versa. Example 31.5.1 Consider the set of dierential equations y1 = y2 y2 = y3 y3 = y3 y2 y1 + t3 1498

with the initial conditions y1 (0) = y2 (0) = y3 (0) = 0. We take the Laplace transform of this system. s1 y1 (0) = y2 y s2 y2 (0) = y3 y s3 y3 (0) = 3 y2 y1 + y y The rst two equations can be written as y3 s2 y3 y2 = . s y1 = We substitute this into the third equation. s3 = 3 y y y3 y3 6 2+ 4 s s s 6 (s3 + s2 + s + 1)3 = 2 y s 6 y3 = 2 3 . s (s + s2 + s + 1) 6 s4

We solve for y1 . 6 s4 (s3 + s2 + s + 1) 1 1 1 1s y1 = 4 3 + + s s 2(s + 1) 2(s2 + 1) y1 = 1499

We then take the inverse Laplace transform of y1 . y1 = t3 t2 1 t 1 1 + e + sin t cos t. 6 2 2 2 2

We can nd y2 and y3 by dierentiating the expression for y1 . 1 1 1 t2 t et + cos t + sin t 2 2 2 2 1 t 1 1 y3 = t 1 + e sin t + cos t 2 2 2 y2 =

1500

31.6

Exercises

Exercise 31.1 Find the Laplace transform of the following functions: 1. f (t) = eat 2. f (t) = sin(at) 3. f (t) = cos(at) 4. f (t) = sinh(at) 5. f (t) = cosh(at) 6. f (t) = 7. f (t) =
0

sin(at) t
t

sin(au) du u

8. f (t) =

1, 0 t < 0, t < 2 and f (t + 2) = f (t) for t > 0. That is, f (t) is periodic for t > 0.

Hint, Solution Exercise 31.2 Show that L[af (t) + bg(t)] = aL[f (t)] + bL[g(t)]. Hint, Solution Exercise 31.3 Show that if f (t) is of exponential order , L[ect f (t)] = f (s c) for s > c + . 1501

Hint, Solution Exercise 31.4 Show that L[tn f (t)] = (1)n Hint, Solution Exercise 31.5 Show that if 0
f (t) t

dn [f (s)] for n = 1, 2, . . . dsn

dt exists for positive then L f (t) = t

f () d.
s

Hint, Solution Exercise 31.6 Show that


t

L
0

f ( ) d =

f (s) . s

Hint, Solution Exercise 31.7 Show that if f (t) is periodic with period T then L[f (t)] = Hint, Solution
T 0

est f (t) dt . 1 esT

1502

Exercise 31.8 The function f (t) t 0, is periodic with period 2T ; i.e. f (t + 2T ) f (t), and is also odd with period T ; i.e. f (t + T ) = f (t). Further,
T

f (t) est dt = g (s).


0

Show that the Laplace transform of f (t) is f (s) = g (s)/(1 + esT ). Find f (t) such that f (s) = s1 tanh(sT /2). Hint, Solution Exercise 31.9 Find the Laplace transform of t , > 1 by two methods. 1. Assume that s is complex-valued. Make the change of variables z = st and use integration in the complex plane. 2. Show that the Laplace transform of t is an analytic function for (s) > 0. Assume that s is real-valued. Make the change of variables x = st and evaluate the integral. Then use analytic continuation to extend the result to complex-valued s. Hint, Solution Exercise 31.10 (mathematica/ode/laplace/laplace.nb) Show that the Laplace transform of f (t) = ln t is Log s f (s) = , s s [ = 0.5772 . . . is known as Eulers constant.] Hint, Solution Exercise 31.11 Find the Laplace transform of t ln t. Write the answer in terms of the digamma function, () = ()/(). What is the answer for = 0? Hint, Solution 1503

where =
0

et ln t dt.

Exercise 31.12 Find the inverse Laplace transform of f (s) = with the following methods. 1. Expand f (s) using partial fractions and then use the table of Laplace transforms. 2. Factor the denominator into (s 2)(s2 + 1) and then use the convolution theorem. 3. Use Result 31.2.1. Hint, Solution Exercise 31.13 Solve the dierential equation y + y + y = sin t, y(0) = y (0) = 0, 0< 1 1 s3 2s2 +s2

using the Laplace transform. This equation represents a weakly damped, driven, linear oscillator. Hint, Solution Exercise 31.14 Solve the problem, y ty + y = 0, with the Laplace transform. Hint, Solution Exercise 31.15 Prove the following relation between the inverse Laplace transform and the inverse Fourier transform, L1 [f (s)] = 1 ct 1 e F [f (c + )], 2 1504 y(0) = 0, y (0) = 1,

where c is to the right of the singularities of f (s). Hint, Solution Exercise 31.16 (mathematica/ode/laplace/laplace.nb) Show by evaluating the Laplace inversion integral that if f (s) = s
1/2

e2(as)

1/2

s1/2 =

s for s > 0,

then f (t) = ea/t / t. Hint: cut the s-plane along the negative real axis and deform the contour onto the cut. 2 2 Remember that 0 eax cos(bx) dx = /4a eb /4a . Hint, Solution Exercise 31.17 (mathematica/ode/laplace/laplace.nb) Use Laplace transforms to solve the initial value problem d4 y y = t, dt4 Hint, Solution Exercise 31.18 (mathematica/ode/laplace/laplace.nb) Solve, by Laplace transforms, t dy = sin t + y( ) cos(t ) d, dt 0 Hint, Solution Exercise 31.19 (mathematica/ode/laplace/laplace.nb) Suppose u(t) satises the dierence-dierential equation du + u(t) u(t 1) = 0, dt 1505 t 0, y(0) = y (0) = y (0) = y (0) = 0.

y(0) = 0.

and the initial condition u(t) = u0 (t), 1 t 0, where u0 (t) is given. Show that the Laplace transform u(s) of u(t) satises es u0 (0) u(s) = + 1 + s es 1 + s es Find u(t), t 0, when u0 (t) = 1. Check the result. Hint, Solution Exercise 31.20 Let the function f (t) be dened by f (t) = 1 0t< 0 t < 2,
0

est u0 (t) dt.


1

and for all positive values of t so that f (t + 2) = f (t). That is, f (t) is periodic with period 2. Find the solution of the intial value problem d2 y y = f (t); dt2 y(0) = 1, y (0) = 0.

Examine the continuity of the solution at t = n, where n is a positive integer, and verify that the solution is continuous and has a continuous derivative at these points. Hint, Solution Exercise 31.21 Use Laplace transforms to solve dy + dt Hint, Solution
t

y( ) d = et ,
0

y(0) = 1.

1506

Exercise 31.22 An electric circuit gives rise to the system L di1 + Ri1 + q/C = E0 dt di2 L + Ri2 q/C = 0 dt dq = i 1 i2 dt q(0) = 0.

with initial conditions

E0 , 2R Solve the system by Laplace transform methods and show that i1 (0) = i2 (0) = i1 = where = Hint, Solution Exercise 31.23 Solve the initial value problem, y + 4y + 4y = 4 et , Hint, Solution

E0 E0 t e sin(t) + 2R 2L and 2 = 2 2 . LC

R 2L

y(0) = 2, y (0) = 3.

1507

31.7

Hints

Hint 31.1 Use the dierentiation and integration properties of the Laplace transform where appropriate. Hint 31.2

Hint 31.3

Hint 31.4 If the integral is uniformly convergent and

g s

is continuous then
b b

d ds Hint 31.5

g(s, t) dt =
a a

g(s, t) dt s

etx dt =
s

1 sx e x

Hint 31.6 Use integration by parts. Hint 31.7


(n+1)T

e
0

st

f (t) dt =
n=0 nT

est f (t) dt

1508

The sum can be put in the form of a geometric series.

n =
n=0

1 , 1

for || < 1

Hint 31.8

Hint 31.9 Write the answer in terms of the Gamma function. Hint 31.10

Hint 31.11

Hint 31.12

Hint 31.13

Hint 31.14

Hint 31.15

Hint 31.16

1509

Hint 31.17 Hint 31.18 Hint 31.19 Hint 31.20 Hint 31.21 Hint 31.22 Hint 31.23

1510

31.8

Solutions

Solution 31.1 1.

L eat =
0

est eat dt e(sa)t dt


0

e(sa)t = sa L eat = 2.

for
0

(s) > (a)

1 sa

for

(s) > (a)

L[sin(at)] =
0

est sin(at) dt

1 (s+a)t e = e(sa)t dt 2 0 1 e(s+a)t e(sa)t = + , for 2 s a s + a 0 1 1 1 = 2 s a s + a L[sin(at)] = a + a2 for (s) > 0

(s) > 0

s2

1511

3. L[cos(at)] = L d sin(at) dt a sin(at) = sL sin(0) a s2 s + a2 for (s) > 0

L[cos(at)] = 4.

L[sinh(at)] =
0

est sinh(at) dt

1 (s+a)t e = e(sa)t dt 2 0 1 e(s+a)t e(sa)t = + for 2 sa s+a 0 1 1 1 = 2 sa s+a L[sinh(at)] = 5. L[cosh(at)] = L s2 a a2 for

(s) > | (a)|

(s) > | (a)|

d sinh(at) dt a sinh(at) sinh(0) = sL a s2 s a2 for (s) > | (a)|

L[cosh(at)] =

1512

6. First note that

sin(at) L (s) = L[sin(at)]() d. t s Now we use the Laplace transform of sin(at) to compute the Laplace transform of sin(at)/t.

sin(at) = t

a d + a2 s d 1 = 2+1 a (/a) s = arctan a s s = arctan 2 a 2

L 7. L
0 t

a sin(at) = arctan t s sin(a ) 1 sin(at) d = L s t sin(a ) 1 a d = arctan s s


2 0

L
0

8. est f (t) dt L[f (t)] = 1 e2s st e dt = 0 2s 1e 1 es = s(1 e2s ) 1513

L[f (t)] = Solution 31.2

1 s(1 + es )

L[af (t) + bg(t)] =


0

est af (t) + bg(t) dt


=a
0

st

f (t) dt + b
0

est g(t) dt

= aL[f (t)] + bL[g(t)]

Solution 31.3 If f (t) is of exponential order , then ect f (t) is of exponential order c + .

L[ect f (t)] =
0

est ect f (t) dt e(sc)t f (t) dt


0

= f (s c) for s > c +

Solution 31.4 First consider the Laplace transform of t0 f (t). L[t0 f (t)] = f (s) 1514

Now consider the Laplace transform of tn f (t) for n 1.

L[t f (t)] =
0

est tn f (t) dt

d st n1 e t f (t) dt ds 0 d = L[tn1 f (t)] ds = Thus we have a dierence equation for the Laplace transform of tn f (t) with the solution dn L[t f (t)] = (1) L[t0 f (t)] for n Z0+ , n ds
n n

L[tn f (t)] = (1)n

dn f (s) for n Z0+ . n ds

Solution 31.5 If 0 f (t) dt exists for positive and f (t) is of exponential order then the Laplace transform of f (t)/t is dened for t s > . L f (t) = t =
0 s 0

1 est f (t) dt t

et d f (t) dt et f (t) dt d
s 0

= =
s

f () d

1515

Solution 31.6

L
0

f ( ) d =
0

st 0 t

f ( ) d dx

est = f ( ) d s 0 1 st e f (t) dt = s 0 1 = f (s) s

0 0

est d s dt

f ( ) d dt
0

1516

Solution 31.7 f (t) is periodic with period T .

L[f (t)] =
0 T

est f (t) dt
2T

=
0

est f (t) dt +
T (n+1)T

est f (t) dt +

=
n=0 nT T

est f (t) dt es(t+nT ) f (t + nT ) dt


n=0 0 T

= =
n=0 T

snT 0

est f (t) dt

=
0

est f (t) dt
n=0 T 0

esnT

f (t) dt 1 esT

est

1517

Solution 31.8

f (s) =
0 n

est f (t) dt
(n+1)T

=
0 n nT T

est f (t) dt es(t+nT ) f (t + nT ) dt


0 n 0 T

= =
0 T

esnT
0

est (1)n f (t) dt


n

=
0

st

f (t) dt
0

(1)n esT

f (s) = Consider f (s) = s1 tanh(sT /2).

g (s) , 1 + esT

for

(s) > 0

s1 tanh(sT /2) = s1

esT /2 esT /2 esT /2 + esT /2 1 esT = s1 1 + esT

We have g (s)
0

f (t) est dt =

1 est . s

1518

By inspection we see that this is satised for f (t) = 1 for 0 < t < T . We conclude: 1 for t [2nT . . . (2n + 1)T ), 1 for t [(2n + 1)T . . . (2n + 2)T ),

f (t) =

where n Z. Solution 31.9 The Laplace transform of t , > 1 is

f (s) =
0

est t dt. (s) > 0 and > 1.

Assume s is complex-valued. The integral converges for

Method 1. We make the change of variables z = st. f (s) =


C

ez

z s
C

1 dz s

= s(+1)

ez z dz

C is the path from 0 to along arg(z) = arg(s). (Shown in Figure 31.4). Since the integrand is analytic in the domain < r < R, 0 < < arg(s), the integral along the boundary of this domain vanishes.
R R e arg(s) e arg(s)

+
R

+
R e arg(s)

+
e arg(s)

ez z dz = 0

We show that the integral along CR , the circular arc of radius R, vanishes as R with the maximum modulus 1519

Im(z)

arg(s)

Re(z)

Figure 31.4: The Path of Integration. integral bound. ez z dz R| arg(s)| max ez z


CR zCR

= R| arg(s)| eR cos(arg(s)) R 0 as R . The integral along C , the circular arc of radius , vanishes as 0. We demonstrate this with the maximum modulus integral bound. ez z dz | arg(s)| max ez z
C zC

= | arg(s)| e cos(arg(s)) 0 as 0. 1520

Taking the limit as 0 and R , we see that the integral along C is equal to the integral along the real axis.

ez z dz =
C 0

ez z dz

We can evaluate the Laplace transform of t in terms of this integral.

L [t ] = s

(+1) 0

et t dt

L [t ] =

( + 1) s+1

In the case that is a non-negative integer = n > 1 we can write this in terms of the factorial. L [tn ] = Method 2. First note that the integral f (s) =
0

n! sn+1

est t dt

exists for (s) > 0. It converges uniformly for interchange dierentiation and integration.

(s) c > 0. On this domain of uniform convergence we can

d df = ds ds

est t dt
0

=
0

st e t dt s t est t dt

=
0

=
0

est t+1 dt

1521

Since f (s) is dened for (s) > 0, f (s) is analytic for (s) > 0. Let be real and positive. We make the change of variables x = t. x 1 ex f () = dx 0

= = Note that the function

(+1) 0

ex x dx

( + 1) +1

( + 1) f (s) = s+1 is the analytic continuation of f (). Thus we can dene the Laplace transform for all complex s in the right half plane. ( + 1) f (s) = s+1 Solution 31.10 Note that f (s) is an analytic function for (s) > 0. Consider real-valued s > 0. By denition, f (s) is

f (s) =
0

est ln t dt.

We make the change of variables x = st.

f (s) =
0

ex ln

x s

dx s

1 = s

ex (ln x ln s) dx
0

ln |s| x 1 x e dx + e ln x dx = s s 0 0 ln s = , for real s > 0 s s 1522

The analytic continuation of f (s) into the right half-plane is

Log s f (s) = . s s

Solution 31.11 Dene

f (s) = L[t ln t] =
0

est t ln t dt.

This integral denes f (s) for (s) > 0. Note that the integral converges uniformly for we can interchange dierentiation and integration.

(s) c > 0. On this domain

f (s) =
0

st e t ln t dt = s

t est t Log t dt
0

Since f (s) also exists for

(s) > 0, f (s) is analytic in that domain. 1523

Let be real and positive. We make the change of variables x = t. f () = L [t ln t]

=
0

et t ln t dt ex
0

= = = = = = = = Note that the function 1

ln

x1 dx

ex x (ln x ln ) dx +1 0 1 ex x ln x dx ln ex x dx +1 0 0 x 1 e x dx ln ( + 1) +1 0 1 d ex x dx ln ( + 1) +1 d 0 1 d ( + 1) ln ( + 1) +1 d 1 ( + 1) ( + 1) ln +1 ( + 1) 1 ( + 1) (( + 1) ln ) +1 f (s) = 1 s+1 ( + 1) (( + 1) ln s)

is an analytic continuation of f (). Thus we can dene the Laplace transform for all s in the right half plane. L[t ln t] = 1 s+1 ( + 1) (( + 1) ln s) for (s) > 0.

1524

For the case = 0, we have L[ln t] = 1 (1) ((1) ln s) s1 ln s L[ln t] = , s

where is Eulers constant =


0

ex ln x dx = 0.5772156629 . . .

Solution 31.12 Method 1. We factor the denominator. f (s) = 1 1 = 2 + 1) (s 2)(s (s 2)(s )(s + )

We expand the function in partial fractions and simplify the result. 1 1/5 (1 2)/10 (1 + 2)/10 = (s 2)(s )(s + ) s2 s s+ 1 s+2 1 1 2 f (s) = 5s2 5s +1 We use a table of Laplace transforms to do the inversion. L[e2t ] = 1 , s2 L[cos t] = s2 s , +1 L[sin t] = s2 1 +1

f (t) = Method 2. We factor the denominator.

1 2t e cos t 2 sin t 5 1 1 2+1 s2s 1525

f (s) =

From a table of Laplace transforms we note L[e2t ] = We apply the convolution theorem.
t

1 , s2

L[sin t] =

s2

1 . +1

f (t) =
0

sin e2(t ) d

1 2t e cos t 2 sin t f (t) = 5 Method 3. We factor the denominator. f (s) = 1 (s 2)(s )(s + )

f (s) is analytic except for poles and vanishes at innity. f (t) = = Res
sn =2,, e2t

est , sn (s 2)(s )(s + ) +

et et + (2 )(2 + ) ( 2)(2) ( 2)(2) e2t (1 + 2) et (1 2) et = + + 5 10 10 2t t t t t e e +e e e = + + 5 10 5 f (t) = 1 2t e cos t 2 sin t 5 1526

Solution 31.13 y + y + y = sin t, We take the Laplace transform of this equation. (s2 y (s) sy(0) y (0)) + (s(s) y(0)) + y (s) = L[sin(t)] y 2 (s + s + 1)(s) = L[sin(t)] y 1 y (s) = 2 L[sin(t)] s + s+1 1 y (s) = 2 L[sin(t)] (s + 2 )2 + 1 4 We use a table of Laplace transforms to nd the inverse Laplace transform of the rst term. L1 1 (s + 2 )2 + 1 = 1 1
2

y(0) = y (0) = 0,

0<

e t/2 sin
4

We dene
2

4 to get rid of some clutter. Now we apply the convolution theorem to invert
t

y s.

y(t) =
0

1 /2 e sin ( ) sin(t ) d 1 cos (t) + 1 1 sin (t) cos t 2

y(t) = e t/2

The solution is plotted in Figure 31.5 for = 0.05.


2

Evaluate the convolution integral by inspection.

1527

15 10 5 20 -5 -10 -15 40 60 80 100

Figure 31.5: The Weakly Damped, Driven Oscillator Solution 31.14 We consider the solutions of y ty + y = 0, y(0) = 0, y (0) = 1 which are of exponential order for any > 0. We take the Laplace transform of the dierential equation. s2 y 1 + d (s) + y = 0 y ds 2 1 y + s+ y= s s 2 /2 es 1 y (s) = 2 + c 2 s s 1528

We use that y (s) to conclude that c = 0.

y(0) y (0) + 2 + s s 1 s2 y(t) = t

y (s) =

Solution 31.15 1 L1 [f (s)] = 2 First we make the change of variable s = c + . 1 ct e L1 [f (s)] = 2 Then we make the change of variable = . L1 [f (s)] = 1 ct t e e f (c + ) d 2 1 ct 1 e F [f (c + )] L1 [f (s)] = 2
c+

est f (s) ds
c

et f (c + ) d

Solution 31.16 We assume that (a) 0. We are considering the principal branch of the square root: s1/2 = s. There is a branch cut on the negative real axis. f (s) is singular at s = 0 and along the negative real axis. Let be any positive number. 1/2 2(as)1/2 e The inverse Laplace transform of is s 1 f (t) = 2
+

est

1/2

e2(as)

1/2

ds.

1529

We will evaluate the integral by deforming it to wrap around the branch cut. Consider the integral on the contour + shown in Figure 31.6. CR and CR are circular arcs of radius R. B is the vertical line at (s) = joining the two arcs. C is a semi-circle in the right half plane joining and . L+ and L are lines joining the circular arcs at (s) = .

+ CR

B
+ L

/2 C /2+

LCR

Figure 31.6: Path of Integration Since there are no residues inside the contour, we have 1 2 +
B
+ CR

+
L+

+
C

+
L

+
CR

est

1/2

e2(as)

1/2

ds = 0.

We will evaluate the inverse Laplace transform for t > 0. + First we will show that the integral along CR vanishes as R . We parametrize the path of integration with + s = R e and write the integral along CR as the sum of two integrals.
/2

ds =
+ CR

d +
/2 /2

1530

The rst integral vanishes by the maximum modulus bound. Note that the length of the path of integration is less than 2.
/2

d
/2 t

max
[/2.../2]

est

1/2

e2(as)

1/2

(2)

= e (2) R 0 as R

The second integral vanishes by Jordans Lemma.


R e t

e
/2

2aR e e d R e

e
/2

R e t

R cos()t e d R /2 /2 Rt sin() e d R 0 < R 2Rt 0 as R

2aR e/2 e d R e

We could show that the integral along CR vanishes by the same method.

Now we have 1 2 +
B L+

+
C

+
L

est

1/2

e2(as)

1/2

ds = 0.

1531

We show that the integral along C vanishes as 0 with the maximum modulus bound. s
1/2

est
C

e2(as)

1/2

ds

max est sC s et 0 as 0.

1/2

e2(as)

1/2

( )

Now we can express the inverse Laplace transform in terms of the integrals along L+ and L
+ 1/2 2(as)1/2 1 e est ds 2 s 1 1/2 2(as)1/2 1 est e = ds 2 L+ s 2

f (t)

est
L

1/2

e2(as)

1/2

ds.

On L+ , s = r e , ds = e dr = dr; on L , s = r e , ds = e dr = dr. We can combine the integrals along the top and bottom of the branch cut. 2ar 1 rt e e e e2 a r ( dr) ( dr) 2 0 r r 1 ert e2 a r + e2 a r dr r 0 1 ert 2 cos 2 a r dr r 0
0 rt

1 f (t) = 2 1 = 2 1 = 2

1532

We make the change of variables x = = = = =

r.

1 1 tx2 e cos 2 ax 2x dx 0 x 2 2 etx cos 2 ax dx 0 4a/(4t) 2 e 4t ea/t t

Thus the inverse Laplace transform is ea/t f (t) = t Solution 31.17 We consider the problem d4 y y = t, y(0) = y (0) = y (0) = y (0) = 0. dt4 We take the Laplace transform of the dierential equation. s4 y (s) s3 y(0) s2 y (0) sy (0) y (0) y (s) = s4 y (s) y (s) = y (s) = s2 (s4 1 s2 1 s2

1 1)

There are several ways in which we could carry out the inverse Laplace transform to nd y(t). We could expand the right side in partial fractions and then use a table of Laplace transforms. Since the function is analytic except for 1533

isolated singularities and vanishes as s we could use the result,


N

L [f (s)] =
1 n=1

Res est f (s), sn ,

where {sk }n are the singularities of f (s). Since we can write the function as a product of simpler terms we could k=1 also apply the convolution theorem. We will rst do the inverse Laplace transform by expanding the function in partial fractions to obtain simpler rational functions. s2 (s4 1 1 = 2 1) s (s 1)(s + 1)(s )(s + ) a b c d e f = 2+ + + + + s s s1 s+1 s s+ 1 = 1 1 s=0 d 1 =0 ds s4 1 s=0 1 2 (s + 1)(s )(s + ) s 1 2 (s 1)(s )(s + ) s 1 2 (s 1)(s + 1)(s + ) s 1 2 (s 1)(s + 1)(s ) s s4 1534

a= b= c= d= e= f=

=
s=1

1 4

1 4 s=1 1 = 4 s= 1 = 4 s= =

Now we have simple functions that we can look up in a table. 1 1/4 1/4 1/2 y (s) = 2 + + 2 s s1 s+1 s +1 1 1 1 y(t) = t + et et + sin t H(t) 4 4 2 y(t) = t + 1 (sinh t + sin t) H(t) 2

We can also do the inversion with the convolution theorem. 1 1 1 1 = 2 2 2 (s4 1) 21 s s s +1s From a table of Laplace transforms we know, 1 L1 2 = t, s 1 L1 2 = sin t, s +1 1 = sinh t. L1 2 s 1 Now we use the convolution theorem to nd the solution for t > 0. t 1 L1 4 = sinh( ) sin(t ) d s 1 0 1 = (sinh t sin t) 2 L1 1 = 2 (s4 1) s 1 (sinh sin ) (t ) d 0 2 1 = t + (sinh t + sin t) 2 1535
t

Solution 31.18

dy = sin t + dt s(s) y(0) = y

y( ) cos(t ) d
0

1 s + y (s) 2 s2 + 1 s +1 3 (s + s)(s) s(s) = 1 y y 1 y (s) = 3 s t2 y(t) = 2

Solution 31.19 The Laplace transform of u(t 1) is

L[u(t 1)] =
0

est u(t 1) dt es(t+1) u(t) dt


1 0 s 1 0

= =e

st

u(t) dt + e

s 0

est u(t) dt

= es
1

est u0 (t) dt + es u(s).

1536

We take the Laplace transform of the dierence-dierential equation.


0

s(s) u(0) + u(s) es u


1

est u0 (t) dt + es u(s) = 0


0

(1 + s es )(s) = u0 (0) + es u
1

est u0 (t) dt
0

u0 (0) u(s) = + 1 + s es 1 + s es Consider the case u0 (t) = 1.

es

est u0 (t) dt
1

0 es 1 est dt + 1 + s es 1 + s es 1 es 1 1 1 u(s) = + + es 1 + s es 1 + s es s s s 1/s + 1 e /s u(s) = 1 + s es 1 u(s) = s u(t) = 1

u(s) =

Clearly this solution satises the dierence-dierential equation. Solution 31.20 We consider the problem, d2 y y = f (t), y(0) = 1, dt2 where f (t) is periodic with period 2 and is dened by, f (t) = y (0) = 0,

1 0 t < , 0 t < 2. 1537

We take the Laplace transform of the dierential equation. s2 y (s) sy(0) y (0) y (s) = f (s) s2 y (s) s y (s) = f (s) y (s) = s f (s) + 2 s2 1 s 1

By inspection, (of a table of Laplace transforms), we see that L1 L1 Now we use the convolution theorem. L
1

s2

s = cosh(t)H(t), 1 1 = sinh(t)H(t). s2 1
t

f (s) = s2 1

f ( ) sinh(t ) d
0

The solution for positive t is


t

y(t) = cosh(t) +
0

f ( ) sinh(t ) d.

Clearly the solution is continuous because the integral of a bounded function is continuous. The rst derivative of the solution is
t

y (t) = sinh t + f (t) sinh(0) +


0 t

f ( ) cosh(t ) d

y (t) = sinh t +
0

f ( ) cosh(t ) d

We see that the rst derivative is also continuous. 1538

Solution 31.21 We consider the problem


t dy + y( ) d = et , dt 0 We take the Laplace transform of the equation and solve for y .

y(0) = 1.

s y(0) + y

1 y = s s+1 s(s + 2) y= (s + 1)(s2 + 1)

We expand the right side in partial fractions. y= 1 1 + 3s + 2(s + 1) 2(s2 + 1)

We use a table of Laplace transforms to do the inversion. 1 1 y = et + (sin(t) + 3 cos(t)) 2 2 Solution 31.22 We consider the problem di1 + Ri1 + q/C = E0 dt di2 L + Ri2 q/C = 0 dt dq = i 1 i2 dt E0 , q(0) = 0. i1 (0) = i2 (0) = 2R L 1539

We take the Laplace transform of the system of dierential equations. E0 q E0 L s1 i + R1 + = i 2R C s E0 q L s2 i + R2 = 0 i 2R C s = 1 2 q i i We solve for 1 , 2 and q . i i 1 = E0 i 2 2 = E0 i 2 CE0 q= 2 We factor the polynomials in the denominators. 1 = E0 i 2 2 = E0 i 2 CE0 q= 2 Here we have dened = R 2L and 2 = 1540 2 2 . LC 1 1/L + Rs (s + )(s + + ) 1 1/L Rs (s + )(s + + ) 1 s + 2 s (s + )(s + + ) 1 1/L + 2 Rs s + Rs/L + 2/(CL) 1 1/L 2 Rs s + Rs/L + 2/(CL) 1 s + R/L 2 s s + Rs/L + 2/(CL)

We expand the functions in partial fractions. 1 1 1 1 = E0 i + 2 Rs 2L s + + s + 1 1 1 2 = E0 i 2 Rs 2L s + + s + + CE0 1 q= + 2 s 2 s + s + + Now we can do the inversion with a table of Laplace transforms. E0 1 e()t e(+)t + 2 R 2L E0 1 e()t e(+)t i2 = 2 R 2L CE0 q= 1+ ( + ) e(+)t ( ) e()t 2 2 i1 = We simplify the expressions to obtain the solutions. E0 1 + 2 R E0 1 i2 = 2 R CE0 q= 1 et 2 i1 = Solution 31.23 We consider the problem y + 4y + 4y = 4 et , y(0) = 2, y (0) = 3 1 t e sin(t) L 1 t e sin(t) L cos(t) + sin(t)

1541

We take the Laplace transform of the dierential equation and solve for y (s). s2 y sy(0) y (0) + 4s 4y(0) + 4 = y y s2 y 2s + 3 + 4s 8 + 4 = y y 4 s+1 4 2s + 5 y= + (s + 1)(s + 2)2 (s + 2)2 2 3 4 y= s + 1 s + 2 (s + 2)2 4 s+1

We take the inverse Laplace transform to determine the solution. y = 4 et (2 + 3t) e2t

1542

Chapter 32 The Fourier Transform


32.1 Derivation from a Fourier Series

Consider the eigenvalue problem y + y = 0, The eigenvalues and eigenfunctions are n 2 for n Z0+ L n = enx/L , for n Z L n = The eigenfunctions form an orthogonal set. A piecewise continuous function dened on [L . . . L] can be expanded in a series of the eigenfunctions. f (x) cn enx/L L n= 1543 y(L) = y(L), y (L) = y (L).

The Fourier coecients are cn =


L L

enx/L f (x)
L

enx/L
L

enx/L

1 = 2

enx/L f (x) dx.


L

We substitute the expression for cn into the series for f (x).

f (x)
n=

1 2L

en/L f () d enx/L .
L

We let n = n/L and = /L.

f (x)
n =

1 2

en f () d en x .
L

In the limit as L , (and thus 0), the sum becomes an integral.

f (x)

1 2

e f () d ex d.

Thus the expansion of f (x) for nite L

f (x)
n=

cn 1 2
L

nx/L e L

cn =

enx/L f (x) dx
L

1544

in the limit as L becomes

f (x)

f () ex d

1 f () = 2

f (x) ex dx.

Of course this derivation is only heuristic. In the next section we will explore these formulas more carefully.

32.2

The Fourier Transform


L L

Let f (x) be piecewise continuous and let I(x, L) = 1 2

|f (x)| dx exist. We dene the function I(x, L).

f () e d ex d.

Since the integral in parentheses is uniformly convergent, we can interchange the order of integration. = = 1 2
L

f () e(x) d
L L

1 2 1 = 2 1 = 1 =

e(x) d ( x) L 1 eL(x) eL(x) d f () ( x) sin(L( x)) f () d x sin(L) f ( + x) d. f () 1545

In Example 32.3.3 we will show that


0

sin(L) d = . 2

Continuous Functions. Suppose that f (x) is continuous. sin(L) d 1 f (x + ) f (x) sin(L) d. I(x, L) f (x) = f (x) = f (x) If f (x) has a left and right derivative at x then f (x+)f (x) is bounded and Riemann-Lebesgue lemma to show that the integral vanishes as L . 1 Now we have an identity for f (x). f (x) = 1 2
f (x+)f (x)

d < . We use the

f (x + ) f (x) sin(L) d 0 as L .

f () e d ex d.

Piecewise Continuous Functions. Now consider the case that f (x) is only piecewise continuous. f (x+ ) 1 = 2 1 f (x ) = 2 sin(L) d 0 0 sin(L) f (x ) d f (x+ ) 1546

f (x+ ) + f (x ) I(x, L) = 2

f (x + ) f (x ) sin(L) d f (x + ) f (x+ ) sin(L) d 0

If f (x) has a left and right derivative at x, then

f (x + ) f (x ) is bounded for 0, and f (x + ) f (x+ ) is bounded for 0.

Again using the Riemann-Lebesgue lemma we see that

1 f (x+ ) + f (x ) = 2 2

f () e d ex d.

1547

Result 32.2.1 Let f (x) be piecewise continuous with transform of f (x) is dened 1 f () = F[f (x)] = 2

|f (x)| dx

< . The Fourier

f (x) ex dx.

We see that the integral is uniformly convergent. The inverse Fourier transform is dened f (x+ ) + f (x ) = F 1 [f ()] = 2 If f (x) is continuous then this reduces to

f () ex d.

f (x) = F

[f ()] =

f () ex d.

32.2.1

A Word of Caution

Other texts may dene the Fourier transform dierently. The important relation is

f (x) =

1 2

f () e

d ex d.

1 Multiplying the right side of this equation by 1 = yields

1 f (x) =

f () e

d ex d.

1548

Setting =

2 and choosing sign in the exponentials gives us the Fourier transform pair 1 f () = 2 1 f (x) = 2

f (x) ex dx

f () ex d.

Other equally valid pairs are

f () =

f (x) ex dx

1 f (x) = 2 and

f () ex d,

f () =

f (x) ex dx

1 f (x) = 2

f () ex d.

Be aware of the dierent denitions when reading other texts or consulting tables of Fourier transforms.

32.3
32.3.1

Evaluating Fourier Integrals


Integrals that Converge
1 F[f (x)] = 2

If the Fourier integral

f (x) ex dx,

1549

converges for real , then nding the transform of a function is just a matter of direct integration. We will consider several examples of such garden variety functions in this subsection. Later on we will consider the more interesting cases when the integral does not converge for real . Example 32.3.1 Consider the Fourier transform of ea|x| , where a > 0. Since the integral of ea|x| is absolutely convergent, we know that the Fourier transform integral converges for real . We write out the integral. 1 2 1 = 2 1 = 2

F ea|x| =

ea|x| ex dx
0

e
0

axx

1 dx + 2

eaxx dx
0

e(a

()+ ())x

dx +

1 2

e(a
0

()+ ())x

dx

The integral converges for | ()| < a. This domain is shown in Figure 32.1.

Im(z)

Re(z)

Figure 32.1: The Domain of Convergence 1550

Now We do the integration. F ea|x| = 1 2


0

e(a)x dx +

1 2

e(a+)x dx
0

e(a+)x 1 1 + = 2 a 2 a + 1 1 1 = + 2 a a + 1 a = , for | ()| < a ( 2 + a2 ) e(a)x We can extend the domain of the Fourier transform with analytic continuation. a F ea|x| = , for = a 2 + a2 ) ( Example 32.3.2 Consider the Fourier transform of f (x) = 1 1 F = x 2 The integral converges for
1 , x

> 0.

1 ex dx x

() = 0. We will evaluate the integral for positive and negative real values of .

For > 0, we will close the path of integration in the lower half-plane. Let CR be the contour from x = R to x = R following a semicircular path in the lower half-plane. The integral along CR vanishes as R by Jordans Lemma. 1 ex dx 0 as R . x CR Since the integrand is analytic in the lower half-plane the integral vanishes. F 1 =0 x 1551

For < 0, we will close the path of integration in the upper half-plane. Let CR denote the semicircular contour from x = R to x = R in the upper half-plane. The integral along CR vanishes as R goes to innity by Jordans Lemma. We evaluate the Fourier transform integral with the Residue Theorem. 1 1 F = 2i Res x 2 = e We combine the results for positive and negative values of . F 1 = x 0 e for > 0, for < 0 ex , i x i

32.3.2

Cauchy Principal Value and Integrals that are Not Absolutely Convergent.

That the integral of f (x) is absolutely convergent is a sucient but not a necessary condition that the Fourier transform of f (x) exists. The integral f (x) ex dx may converge even if |f (x)| dx does not. Furthermore, if the Fourier transform integral diverges, its principal value may exist. We will say that the Fourier transform of f (x) exists if the principal value of the integral exists.

F[f (x)] =

f (x) ex dx

Example 32.3.3 Consider the Fourier transform of f (x) = 1/x.


1 1 x e dx f () = 2 x

If > 0, we can close the contour in the lower half-plane. The integral along the semi-circle vanishes due to Jordans Lemma. 1 x e dx = 0 lim R C x R 1552

We can evaluate the Fourier transform with the Residue Theorem. 1 1 1 x e f () = (2i) Res ,0 2 2 x f () = , for > 0. 2 The factor of 1/2 in the above derivation arises because the path of integration is in the negative, (clockwise), direction and the path of integration crosses through the rst order pole at x = 0. The path of integration is shown in Figure 32.2.

Im(z) Re(z)

Figure 32.2: The Path of Integration If < 0, we can close the contour in the upper half plane to obtain f () = , for < 0. 2 For = 0 the integral vanishes because
1 x

is an odd function.
(0) = 1 = 1 dx = 0 f 2 x

1553

We collect the results in one formula.

f () = sign() 2 We write the integrand for > 0 as the sum of an odd and and even function. 1 1 x e dx = 2 x 2 1 cos(x) dx + sin(x) dx = x x

The principal value of the integral of any odd function is zero.

1 sin(x) dx = x

If the principal value of the integral of an even function exists, then the integral converges.
0

1 sin(x) dx = x 1 sin(x) dx = x 2

Thus we have evaluated an integral that we used in deriving the Fourier transform.

32.3.3

Analytic Continuation

Consider the Fourier transform of f (x) = 1. The Fourier integral is not convergent, and its principal value does not exist. Thus we will have to be a little creative in order to dene the Fourier transform. Dene the two functions 1 0 for x > 0 for x > 0 f+ (x) = 1/2 for x = 0 , f (x) = 1/2 for x = 0 . 0 for x < 0 1 for x < 0 1554

Note that 1 = f (x) + f+ (x). The Fourier transform of f+ (x) converges for () < 0.

1 ex dx F[f+ (x)] = 2 0 1 e( ()+ ())x dx. = 2 0 1 ex = 2 0 = for () < 0 2 Using analytic continuation, we can dene the Fourier transform of f+ (x) for all except the point = 0. F[f+ (x)] = 2

We follow the same procedure for f (x). The integral converges for F[f (x)] = 1 2 1 = 2 =
0

() > 0.

ex dx
0

e(
ex 0

()+ ())x

dx

1 2 = . 2 Using analytic continuation we can dene the transform for all nonzero . F[f (x)] = 2 1555

Now we are prepared to dene the Fourier transform of f (x) = 1. F[1] = F[f (x)] + F[f+ (x)] = + 2 2 = 0, for = 0 When = 0 the integral diverges. When we consider the closure relation for the Fourier transform we will see that F[1] = ().

32.4

Properties of the Fourier Transform

In this section we will explore various properties of the Fourier Transform. I would like to avoid stating assumptions on various functions at the beginning of each subsection. Unless otherwise indicated, assume that the integrals converge.

32.4.1

Closure Relation.

Recall the closure relation for an orthonormal set of functions {1 , 2 , . . .},

n (x)n () (x ).
n=1

There is a similar closure relation for Fourier integrals. We compute the Fourier transform of (x ). F[(x )] =
1 (x ) ex dx 2 1 e = 2

1556

Next we take the inverse Fourier transform.

(x )

1 x e e d 2

1 (x ) 2

e(x) d.

Note that the integral is divergent, but it would be impossible to represent (x ) with a convergent integral.

32.4.2

Fourier Transform of a Derivative.


1 y (x) ex dx 2 1 1 = y(x) ex 2 2 1 y(x) ex dx = 2 = F[y(x)]

Consider the Fourier transform of y (x). F[y (x)] =

()y(x) ex dx

Next consider y (x). F[y (x)] = F d (y (x)) dx = F[y (x)] = ()2 F[y(x)] = 2 F[y(x)] 1557

In general, F y (n) (x) = ()n F[y(x)].

Example 32.4.1 The Dirac delta function can be expressed as the derivative of the Heaviside function.

H(x c) =

0 1

for x < c, for x > c

Thus we can express the Fourier transform of H(x c) in terms of the Fourier transform of the delta function.

F[(x c)] = F[H(x c)] 1 2

(x c) ex dx = F[H(x c)]

1 c e = F[H(x c)] 2

F[H(x c)] =

1 c e 2

1558

32.4.3

Fourier Convolution Theorem.

Consider the Fourier transform of a product of two functions.

F[f (x)g(x)] =

1 2 1 = 2 1 = 2

f (x)g(x) ex dx

f () ex d g(x) ex dx

f ()g(x) e()x dx

d d

= =

f ()

1 2

g(x) e()x dx

f ()G( ) d

The convolution of two functions is dened

f g(x) =

f ()g(x ) d.

Thus

F[f (x)g(x)] = f g () =

g f ()( ) d.

1559

Now consider the inverse Fourier Transform of a product of two functions.

g F 1 [f ()()] =

g f ()() ex d 1 2

f () e d g () ex d

1 = 2 1 = 2 1 = 2 Thus

f ()() e(x) d g

d d

f ()

g () e(x) d

f ()g(x ) d

1 1 g F 1 [f ()()] = f g(x) = 2 2

f ()g(x ) d,

g F[f g(x)] = 2 f ()(). These relations are known as the Fourier convolution theorem. Example 32.4.2 Using the convolution theorem and the table of Fourier transform pairs in the appendix, we can nd the Fourier transform of 1 f (x) = 4 . x + 5x2 + 4 We factor the fraction. 1 f (x) = 2 (x + 1)(x2 + 4) From the table, we know that 2c F 2 = ec|| for c > 0. 2 x +c 1560

We apply the convolution theorem. F[f (x)] = F 1 8 1 = 8 = First consider the case > 0.
0 1 2+3 e e2+ d + F[f (x)] = d + 8 0 1 1 2 1 e = + e e2 + e 8 3 3 1 1 = e e2 6 12

1 2 4 2 + 1 x2 + 4 8x

e|| e2|| d
0

e e

2||

d +
0

e e2|| d

e23 d

Now consider the case < 0.


0 1 2+3 e e2 d + d + F[f (x)] = 8 1 1 1 e e2 + e + e2 = 8 3 3 1 1 = e e2 6 12

e23 d
0

We collect the result for positive and negative . F[f (x)] = 1 || 1 2|| e e 6 12 1561

A better way to nd the Fourier transform of f (x) = is to rst expand the function in partial fractions. f (x) = 1/3 1/3 2 x2 + 1 x + 4 1 x4 + 5x2 + 4

1 2 4 1 F[f (x)] = F 2 F 2 6 x +1 12 x +4 1 || 1 2|| = e e 6 12

32.4.4

Parsevals Theorem.
nx n= cn e

Recall Parsevals theorem for Fourier series. If f (x) is a complex valued function with the Fourier series then

2
n=

|cn |2 =

|f (x)|2 dx.

Analogous to this result is Parsevals theorem for Fourier transforms. Let f (x) be a complex valued function that is both absolutely integrable and square integrable.

|f (x)| dx < and


|f (x)|2 dx <

1562

The Fourier transform of f (x) is f (). 1 2

F f (x) =

f (x) ex dx

1 = f (x) ex dx 2 1 f (x) ex dx = 2

= f () We apply the convolution theorem.


1

[2 f ()f ()] =

f ()f ((x )) d

2 f ()f () ex d =

f ()f ( x) d

We set x = 0.

f ()f () d =

f ()f () d |f (x)|2 dx

|f ()|2 d =

This is known as Parsevals theorem. 1563

32.4.5

Shift Property.
1 F[f (x + c)] = 2 1 = 2

The Fourier transform of f (x + c) is f (x + c) ex dx


f (x) e(xc) dx

F[f (x + c)] = ec f () The inverse Fourier transform of f ( + c) is

[f ( + c)] =

f ( + c) ex d f () e(c)x d

F 1 [f ( + c)] = ecx f (x)

32.4.6

Fourier Transform of x f(x).


1 2 1 = 2 =

The Fourier transform of xf (x) is F[xf (x)] = xf (x) ex dx


f (x)

x (e ) dx f (x) ex dx

1 2

1564

F[xf (x)] = Similarly, you can show that

f . nf . n

F[xn f (x)] = (i)n

32.5

Solving Dierential Equations with the Fourier Transform

The Fourier transform is useful in solving some dierential equations on the domain ( . . . ) with homogeneous boundary conditions at innity. We take the Fourier transform of the dierential equation L[y] = f and solve for y . We take the inverse transform to determine the solution y. Note that this process is only applicable if the Fourier transform of y exists. Hence the requirement for homogeneous boundary conditions at innity. We will use the table of Fourier transforms in the appendix in solving the examples in this section. Example 32.5.1 Consider the problem y y = e|x| , We take the Fourier transform of this equation. 2 y () y () = We take the inverse Fourier transform to determine the solution. y () = / + 2 )( 2 + 1) 1 1 1 = 2 21 2+1 + 2 1 / 1/ = 2 2 2 + 2 1 +1 ( 2 1565 2 / + 2 y() = 0, > 0, = 1.

y(x) =

e|x| e|x| 2 1

Example 32.5.2 Consider the Green function problem G G = (x ), We take the Fourier transform of this equation. 2 G G = F[(x )] 1 G= 2 F[(x )] +1 We use the Table of Fourier transforms. G = F e|x| F[(x )] We use the convolution theorem to do the inversion. 1 G = 2

y() = 0.

e|x| ( ) d

1 G(x|) = e|x| 2 The inhomogeneous dierential equation y y = f (x), has the solution 1 y= 2

y() = 0, f () e|x| d.

When solving the dierential equation L[y] = f with the Fourier transform, it is quite common to use the convolution theorem. With this approach we have no need to compute the Fourier transform of the right side. We merely denote it as F[f ] until we use f in the convolution integral. 1566

32.6
32.6.1

The Fourier Cosine and Sine Transform


The Fourier Cosine Transform

Suppose f (x) is an even function. In this case the Fourier transform of f (x) coincides with the Fourier cosine transform of f (x). F[f (x)] = 1 2 1 = 2 1 = 2 1 =

f (x) ex dx

f (x)(cos(x) sin(x)) dx

f (x) cos(x) dx

f (x) cos(x) dx
0

The Fourier cosine transform is dened: 1 Fc [f (x)] = fc () =

f (x) cos(x) dx.


0

Note that fc () is an even function. The inverse Fourier cosine transform is


1 Fc [fc ()] =

fc () ex d fc ()(cos(x) + sin(x)) d

= = =2
0

fc () cos(x) d

fc () cos(x) d.

1567

Thus we have the Fourier cosine transform pair


1 f (x) = Fc [fc ()] = 2 0

fc () cos(x) d,

1 fc () = Fc [f (x)] =

f (x) cos(x) dx.


0

32.6.2

The Fourier Sine Transform

Suppose f (x) is an odd function. In this case the Fourier transform of f (x) coincides with the Fourier sine transform of f (x). F[f (x)] = 1 2 1 = 2 =

f (x) ex dx

f (x)(cos(x) sin(x)) dx

f (x) sin(x) dx
0

Note that f () = F[f (x)] is an odd function of . The inverse Fourier transform of f () is

[f ()] =

f () ex d

= 2
0

f () sin(x) d.

Thus we have that

f (x) = 2
0

f (x) sin(x) dx sin(x) d


0

=2
0

f (x) sin(x) dx sin(x) d.

1568

This gives us the Fourier sine transform pair

f (x) =

1 Fs [fs ()]

=2
0

fs () sin(x) d,

1 fs () = Fs [f (x)] =

f (x) sin(x) dx.


0

Result 32.6.1 The Fourier cosine transform pair is dened:

f (x) =

1 Fc [fc ()]

=2
0

fc () cos(x) d f (x) cos(x) dx

1 fc () = Fc [f (x)] = The Fourier sine transform pair is dened:

f (x) =

1 Fs [fs ()]

=2
0

fs () sin(x) d f (x) sin(x) dx

1 fs () = Fs [f (x)] =

32.7
32.7.1

Properties of the Fourier Cosine and Sine Transform


Transforms of Derivatives

Cosine Transform. Using integration by parts we can nd the Fourier cosine transform of derivatives. Let y be a function for which the Fourier cosine transform of y and its rst and second derivatives exists. Further assume that y 1569

and y vanish at innity. We calculate the transforms of the rst and second derivatives.

1 y cos(x) dx 0 1 y sin(x) dx = y cos(x) 0 + 0 1 = yc () y(0) 1 Fc [y ] = y cos(x) dx 0 1 = y cos(x) 0 + y sin(x) dx 0 1 2 = y (0) + y sin(x) 0 y cos(x) dx 0 1 = 2 fc () y (0) Fc [y ] =

Sine Transform. You can show, (see Exercise 32.3), that the Fourier sine transform of the rst and second derivatives are

Fs [y ] = fc () Fs [y ] = 2 yc () + y(0).

1570

32.7.2

Convolution Theorems

Cosine Transform of a Product. Consider the Fourier cosine transform of a product of functions. Let f (x) and g(x) be two functions dened for x 0. Let Fc [f (x)] = fc (), and Fc [g(x)] = gc (). 1 1 = 2 =

Fc [f (x)g(x)] =

f (x)g(x) cos(x) dx
0

2
0 0 0 0

fc () cos(x) d g(x) cos(x) dx fc ()g(x) cos(x) cos(x) dx d

1 We use the identity cos a cos b = 2 (cos(a b) + cos(a + b)).

1 = =
0

0 0

fc ()g(x) cos(( )x) + cos(( + )x) dx d 1 fc ()


0

1 g(x) cos(( )x) dx +

g(x) cos(( + )x) dx d


0

=
0

fc () gc ( ) + gc ( + ) d

gc () is an even function. If we have only dened gc () for positive argument, then gc () = gc (||).

=
0

fc () gc (| |) + gc ( + ) d

1571

Inverse Cosine Transform of a Product. Now consider the inverse Fourier cosine transform of a product of functions. Let Fc [f (x)] = fc (), and Fc [g(x)] = gc ().
1 Fc [fc ()c ()] g

=2
0

fc ()c () cos(x) d g 1
0

=2
0

f () cos() d gc () cos(x) d
0

= = = =

2 1 1 2 1 2

f ()c () cos() cos(x) d d g


f ()c () cos((x )) + cos((x + )) d d g


0 0

f () 2
0 0

gc () cos((x )) d + 2
0

gc () cos((x + )) d

f () g(|x |) + g(x + ) d
0

Sine Transform of a Product. You can show, (see Exercise 32.5), that the Fourier sine transform of a product of functions is

Fs [f (x)g(x)] =
0

fs () gc (| |) gc ( + ) d.

Inverse Sine Transform of a Product. You can also show, (see Exercise 32.6), that the inverse Fourier sine transform of a product of functions is 1 1 Fs [fs ()c ()] = g 2

f () g(|x |) g(x + ) d.
0

1572

Result 32.7.1 The Fourier cosine and sine transform convolution theorems are

Fc [f (x)g(x)] =
0 1 Fc [fc ()c ()] g

fc () gc (| |) + gc ( + ) d

1 = 2
0

f () g(|x |) + g(x + ) d
0

Fs [f (x)g(x)] =
1 Fs [fs ()c ()] g

fs () gc (| |) gc ( + ) d

1 = 2

f () g(|x |) g(x + ) d
0

32.7.3

Cosine and Sine Transform in Terms of the Fourier Transform

We can express the Fourier cosine and sine transform in terms of the Fourier transform. First consider the Fourier cosine transform. Let f (x) be an even function. Fc [f (x)] = 1

f (x) cos(x) dx
0

We extend the domain integration because the integrand is even. 1 = 2 Note that

f (x) cos(x) dx

f (x) sin(x) dx = 0 because the integrand is odd.


1 f (x) ex dx 2 = F[f (x)]

1573

Fc [f (x)] = F[f (x)],

for even f (x).

For general f (x), use the even extension, f (|x|) to write the result. Fc [f (x)] = F[f (|x|)] There is an analogous result for the inverse Fourier cosine transform.
1 Fc f () = F 1 f (||)

For the sine series, we have Fs [f (x)] = F [sign(x)f (|x|)]


1 Fs f () = F 1 sign()f (||)

Result 32.7.2 The results:


1 Fc [f (x)] = F[f (|x|)] Fc f () = F 1 f (||) 1 Fs [f (x)] = F[sign(x)f (|x|)] Fs f () = F 1 sign()f (||)

allow us to evaluate Fourier cosine and sine transforms in terms of the Fourier transform. This enables us to use contour integration methods to do the integrals.

32.8

Solving Dierential Equations with the Fourier Cosine and Sine Transforms
y y = 0, y(0) = 1, 1574 y() = 0.

Example 32.8.1 Consider the problem

Since the initial condition is y(0) = 1 and the sine transform of y is 2 yc () + y(0) we take the Fourier sine transform of both sides of the dierential equation. 2 yc () + y(0) yc () = 0 ( 2 + 1)c () = y yc () = ( 2 + 1) We use the table of Fourier Sine transforms. y = ex Example 32.8.2 Consider the problem y y = e2x , y (0) = 0, y() = 0.

Since the initial condition is y (0) = 0, we take the Fourier cosine transform of the dierential equation. From the table of cosine transforms, Fc [e2x ] = 2/(( 2 + 4)). 2 yc () 2 1 y (0) yc () = 2 + 4) (

yc () =

2 + 4)( 2 + 1) 2 1/3 1/3 2 = 2+1 +4 1 2/ 2 1/ = 2+4 3 3 2 + 1 ( 2 y= 1 2x 2 x e e 3 3

1575

32.9

Exercises
sin(c) .

Exercise 32.1 Show that H(x + c) H(x c) = Hint, Solution Exercise 32.2 Using contour integration, nd the Fourier transform of f (x) = where (c) = 0 Hint, Solution Exercise 32.3 Find the Fourier sine transforms of y (x) and y (x). Hint, Solution Exercise 32.4 Prove the following identities. 1. F[f (x a)] = ea f () 2. F[f (ax)] = Hint, Solution Exercise 32.5 Show that

x2

1 , + c2

1 f |a| a

Fs [f (x)g(x)] =
0

fs () gc (| |) gc ( + ) d. 1576

Hint, Solution Exercise 32.6 Show that


1 g Fs [fs ()c ()]

1 = 2

f () g(|x |) g(x + ) d.
0

Hint, Solution Exercise 32.7 Let fc () = Fc [f (x)], fc () = Fs [f (x)], and assume the cosine and sine transforms of xf (x) exist. Express Fc [xf (x)] and Fs [xf (x)] in terms of fc () and fc (). Hint, Solution Exercise 32.8 Solve the problem y y = e2x , using the Fourier sine transform. Hint, Solution Exercise 32.9 Prove the following relations between the Fourier sine transform and the Fourier transform. Fs [f (x)] = F[sign(x)f (|x|)]
1 Fs f () = F 1 sign()f (||)

y(0) = 1,

y() = 0,

Hint, Solution Exercise 32.10 Let fc () = Fc [f (x)] and fc () = Fs [f (x)]. Show that 1. Fc [xf (x)] =
f () c

1577

2. Fs [xf (x)] = fc ()

3. Fc [f (cx)] = 1 fc c 4. Fs [f (cx)] = 1 fc c Hint, Solution

c c

for c > 0 for c > 0.

Exercise 32.11 Solve the integral equation,


u() ea(x) d = ebx , where a, b > 0, a = b, with the Fourier transform. Hint, Solution Exercise 32.12 Evaluate 1 where is a positive, real number and Hint, Solution (c) > 0.
0

1 cx e sin(x) dx, x

Exercise 32.13 Use the Fourier transform to solve the equation y a2 y = ea|x| on the domain < x < with boundary conditions y() = 0. Hint, Solution 1578

Exercise 32.14 1. Use the cosine transform to solve y a2 y = 0 on x 0 with y (0) = b, y() = 0. 2. Use the cosine transform to show that the Green function for the above with b = 0 is G(x, ) = Hint, Solution Exercise 32.15 1. Use the sine transform to solve y a2 y = 0 on x 0 with y(0) = b, y() = 0. 2. Try using the Laplace transform on this problem. Why isnt it as convenient as the Fourier transform? 3. Use the sine transform to show that the Green function for the above with b = 0 is g(x; ) = Hint, Solution Exercise 32.16 1. Find the Green function which solves the equation y + 2y + ( 2 + 2 )y = (x ), > 0, > 0, 1 a(x) e ea|x+| 2a 1 a|x| 1 a(x) e e . 2a 2a

in the range < x < with boundary conditions y() = y() = 0. 1579

2. Use this Greens function to show that the solution of y + 2y + ( 2 + 2 )y = g(x), with g() = 0 in the limit as 0 is y= 1
x

> 0, > 0,

y() = y() = 0,

g() sin[(x )]d.

You may assume that the interchange of limits is permitted. Hint, Solution Exercise 32.17 Using Fourier transforms, nd the solution u(x) to the integral equation

u() 1 d = 2 2 + a2 ] [(x ) x + b2

0 < a < b.

Hint, Solution Exercise 32.18 The Fourer cosine transform is dened by 1 fc () =

f (x) cos(x) dx.


0

1. From the Fourier theorem show that the inverse cosine transform is given by

f (x) = 2
0

fc () cos(x) d.

2. Show that the cosine transform of f (x) is 2 fc () f (0) .

1580

3. Use the cosine transform to solve the following boundary value problem. y a2 y = 0 on x > 0 with y (0) = b, y() = 0 Hint, Solution Exercise 32.19 The Fourier sine transform is dened by 1 fs () = 1. Show that the inverse sine transform is given by

f (x) sin(x) dx.


0

f (x) = 2
0

fs () sin(x) d.

2. Show that the sine transform of f (x) is

f (0) 2 fs ().

3. Use this property to solve the equation y a2 y = 0 on x > 0 with y(0) = b, y() = 0. 4. Try using the Laplace transform on this problem. Why isnt it as convenient as the Fourier transform? Hint, Solution Exercise 32.20 Show that 1 (Fc [f (x) + f (x)] Fs [f (x) f (x)]) 2 where F, Fc and Fs are respectively the Fourier transform, Fourier cosine transform and Fourier sine transform. Hint, Solution F[f (x)] = 1581

Exercise 32.21 Find u(x) as the solution to the integral equation:


u() 1 d = 2 , 2 + a2 (x ) x + b2

0 < a < b.

Use Fourier transforms and the inverse transform. Justify the choice of any contours used in the complex plane. Hint, Solution

1582

32.10
Hint 32.1

Hints

H(x + c) H(x c) = Hint 32.2 Consider the two cases half plane. Hint 32.3

1 for |x| < c, 0 for |x| > c

() < 0 and

() > 0, closing the path of integration with a semi-circle in the lower or upper

Hint 32.4

Hint 32.5

Hint 32.6

Hint 32.7

Hint 32.8

Hint 32.9

1583

Hint 32.10 Hint 32.11 2 The left side is the convolution of u(x) and eax . Hint 32.12 Hint 32.13 Hint 32.14 Hint 32.15 Hint 32.16 Hint 32.17 Hint 32.18 Hint 32.19 Hint 32.20

1584

Hint 32.21

1585

32.11

Solutions

Solution 32.1
1 (H(x + c) H(x c)) ex dx 2 c 1 ex dx 2 c c 1 ex 2 c ec 1 ec 2

F[H(x + c) H(x c)] = = = =

F[H(x + c) H(x c)] = Solution 32.2

sin(c)

1 1 = x2 + c2 2 1 = 2

1 ex dx x2 + c2 ex dx (x c)(x + c)

If

() < 0 then we close the path of integration with a semi-circle in the upper half plane. F 1 1 = 2i Res 2 + c2 x 2 ex , x = c (x c)(x + c) 1586 = 1 c e 2c

If > 0 then we close the path of integration in the lower half plane. 1 1 = 2i Res 2 + c2 x 2 ex , c (x c)(x + c) 1 c e 2c

Thus we have that F Solution 32.3 x2 1 c|| 1 e = , 2 +c 2c for (c) = 0.

1 y sin(x) dx 0 1 y sin(x) y cos(x) dx = 0 0 = yc () 1 y sin(x) dx Fs [y ] = 0 1 = y sin(x) y cos(x) dx 0 0 2 = y cos(x) y sin(x) dx 0 0 = 2 ys () + y(0). Fs [y ] =

1587

Solution 32.4 1. 1 F[f (x a)] = 2 1 = 2 =e

f (x a) ex dx

f (x) e(x+a) dx

1 2

f (x) ex dx

F[f (x a)] = ea f () 2. If a > 0, then F[f (ax)] =


1 f (ax) ex dx 2 1 1 = f () e/a d 2 a 1 = f . a a 1 f (ax) ex dx 2 1 1 e/a d = 2 a 1 = f . a a

If a < 0, then F[f (ax)] =

Thus F[f (ax)] = 1 f . |a| a

1588

Solution 32.5

Fs [f (x)g(x)] =

1 1 = 2 =

f (x)g(x) sin(x) dx
0

2
0 0 0 0

fs () sin(x) d g(x) sin(x) dx fs ()g(x) sin(x) sin(x) dx d

1 Use the identity, sin a sin b = 2 [cos(a b) cos(a + b)].

= =

1
0

0 0

fs ()g(x) cos(( )x) cos(( + )x) dx d 1 fs ()

g(x) cos(( )x) dx


0

g(x) cos(( + )x) dx d


0

Fs [f (x)g(x)] =
0

fs () Gc (| |) Gc ( + ) d

1589

Solution 32.6
1 Fs [fs ()Gc ()] = 2 0

fs ()Gc () sin(x) d 1
0

=2
0

f () sin() d Gc () sin(x) d
0

2 1 = 1 = 2 1 = 2 =

f ()Gc () sin() sin(x) d d


f ()Gc () cos((x )) cos((x + )) d d


0 0

f () 2
0 0

Gc () cos((x )) d 2
0

Gc () cos((x + )) d) d

f ()[g(x ) g(x + )] d
0

1 Fs [fs ()Gc ()]

1 = 2

f () g(|x |) g(x + ) d
0

Solution 32.7 1 xf (x) cos(x) dx 0 1 = f (x) (sin(x)) dx 0 1 f (x) sin(x) dx = 0 = fs () 1590

Fc [xf (x)] =

Fs [xf (x)] =

1 1 =

xf (x) sin(x) dx
0

f (x)
0

( cos(x)) dx

1 = f (x) cos(x) dx 0 = fc () Solution 32.8 y y = e2x , y(0) = 1, y() = 0

We take the Fourier sine transform of the dierential equation. 2 ys () + ys () = 2/ y(0) ys () = 2 +4

/ / + 2 2 + 1) + 4)( ( + 1) /(3) /(3) / 2 + 2 = 2 +4 +1 +1 1 / 2 / + = 2+1 3 3 2 + 4 ( 2 y= 2 x 1 2x e + e 3 3

Solution 32.9 Consider the Fourier sine transform. Let f (x) be an odd function. Fs [f (x)] = 1

f (x) sin(x) dx
0

1591

Extend the integration because the integrand is even. = Note that


1 2

f (x) sin(x) dx

f (x) cos(x) dx = 0 as the integrand is odd.


1 f (x) ex dx 2 = F[f (x)]

Fs [f (x)] = F[f (x)],

for odd f (x).

For general f (x), use the odd extension, sign(x)f (|x|) to write the result. Fs [f (x)] = F[sign(x)f (|x|)] Now consider the inverse Fourier sine transform. Let f () be an odd function.
1 Fs f () = 2 0

f () sin(x) d

Extend the integration because the integrand is even.

f () sin(x) d

Note that

f () cos(x) d = 0 as the integrand is odd.

f ()(i) ex d

= F 1 f ()

1592

1 Fs f () = F 1 f () ,

for odd f ().

For general f (), use the odd extension, sign()f (||) to write the result.
1 Fs f () = F 1 sign()f (||)

Solution 32.10

Fc [xf (x)] = = = =

1 xf (x) cos(x) dx 0 1 f (x) sin(x) dx 0 1 f (x) sin(x) dx 0 fs ()

Fs [xf (x)] =

1 1 =

xf (x) sin(x) dx
0

f (x)
0

( cos(x)) dx

1 f (x) cos(x) dx = 0 = fc () 1593

Fc [f (cx)] =

1 f (cx) cos(x) dx 0 1 d = f () cos 0 c c 1 = fc c c 1 f (cx) sin(x) dx 0 d 1 = f () sin 0 c c 1 = fs c c

Fs [f (cx)] =

Solution 32.11

u() ea(x) d = ebx We take the Fourier transform and solve for U (). 2U ()F eax 2U ()
2

= F ebx

1 1 2 2 e /(4a) = e /(4b) 4a 4b 1 a 2 (ab)/(4ab) e U () = 2 b

Now we take the inverse Fourier transform. U () = 1 2 a b 4ab/(a b) 4ab/(a b) 1594 e


2 (ab)/(4ab)

u(x) = Solution 32.12

a (a b)

eabx

2 /(ab)

I= = = = = = = Solution 32.13 We consider the dierential equation

1 1 cx e sin(x) dx 0 x 1 ezx dz sin(x) dx 0 c 1 zx e sin(x) dx dz c 0 1 dz 2 + 2 c z z 1 arctan c 1 c arctan 2 1 arctan c

y a2 y = ea|x| on the domain < x < with boundary conditions y() = 0. We take the Fourier transform of the dierential equation and solve for y (). a 2 y a2 y = 2 + a2 ) ( a y () = 2 + a2 )2 ( 1595

We take the inverse Fourier transform to nd the solution of the dierential equation.

y(x) =

( 2

a ex d + a2 )2

Note that since y () is a real-valued, even function, y(x) is a real-valued, even function. Thus we only need to evaluate the integral for positive x. If we replace x by |x| in this expression we will have the solution that is valid for all x. For x 0, we evaluate the integral by closing the path of integration in the upper half plane and using the Residue Theorem and Jordans Lemma. a y(x) =

1 ( a)2 ( a)2

+ 1 a ex , = a = 2 Res 2 ( + a)2 ( a) ex d = 2a lim a d ( + a)2 x ex 2 ex = 2a lim a ( + a)2 ( + a)3 x eax 2 eax = 2a 4a2 8a3 ax (1 + ax) e = 2a2

ex d

The solution of the dierential equation is y(x) = 1 (1 + a|x|) ea|x| . 2a2

1596

Solution 32.14 1. We take the Fourier cosine transform of the dierential equation. 2 y () b a2 y () = 0 b y () = 2 + a2 ) (

Now we take the inverse Fourier cosine transform. We use the fact that y () is an even function.
1 y(x) = Fc

b + a2 ) b = F 1 ( 2 + a2 ) 1 b ex , = a = 2 Res 2 + a2 ex = 2b lim , for x 0 a + a ( 2 b y(x) = eax a

2. The Green function problem is G a2 G = (x ) on x, > 0, We take the Fourier cosine transform and solve for G(; ). 2 G a2 G = Fc [(x )] 1 G(; ) = 2 Fc [(x )] + a2 1597 G (0; ) = 0, G(; ) = 0.

We express the right side as a product of Fourier cosine transforms. G(; ) = Fc [eax ]Fc [(x )] a Now we can apply the Fourier cosine convolution theorem.
1 Fc 1 [Fc [f (x)]Fc [g(x)]] = f (t) g(|x t|) + g(x + t) dt 2 0 1 G(x; ) = (t ) ea|xt| + ea(x+t) dt a 2 0 1 a|x| e G(x; ) = + ea(x+) 2a

Solution 32.15 1. We take the Fourier sine transform of the dierential equation. 2 y () + b a2 y () = 0 b y () = 2 + a2 ) (

Now we take the inverse Fourier sine transform. We use the fact that y () is an odd function.
1 y(x) = Fs

b + a2 ) b = F 1 2 + a2 ) ( b ex , = a = 2 Res 2 + a2 ex = 2b lim a + a ax = be for x 0 ( 2 1598

y(x) = b eax 2. Now we solve the dierential equation with the Laplace transform. y a2 y = 0 s2 y (s) sy(0) y (0) a2 y (s) = 0 We dont know the value of y (0), so we treat it as an unknown constant. bs + y (0) s 2 a2 y (0) y(x) = b cosh(ax) + sinh(ax) a y (s) = In order to satisfy the boundary condition at innity we must choose y (0) = ab. y(x) = b eax We see that solving the dierential equation with the Laplace transform is not as convenient, because the boundary condition at innity is not automatically satised. We had to nd a value of y (0) so that y() = 0. 3. The Green function problem is G a2 G = (x ) on x, > 0, We take the Fourier sine transform and solve for G(; ). 2 G a2 G = Fs [(x )] 1 G(; ) = 2 Fs [(x )] + a2 1599 G(0; ) = 0, G(; ) = 0.

We write the right side as a product of Fourier cosine transforms and sine transforms. G(; ) = Fc [eax ]Fs [(x )] a Now we can apply the Fourier sine convolution theorem.
1 Fs [Fs [f (x)]Fc [g(x)]] =

1 2

f (t) g(|x t|) g(x + t) dt


0

1 G(x; ) = a 2

(t ) ea|xt| ea(x+t) dt 1 a(x) e ea|x+| 2a

G(x; ) =

Solution 32.16 1. We take the Fourier transform of the dierential equation, solve for G and then invert. G + 2G + 2 + 2 G = (x ) e 2 G + 2 G + 2 + 2 G = 2 e G= 2 2 2 2 ) 2 ( e ex G= d 2( 2 2 2 2 ) e(x) 1 G= d 2 ( + )( ) For x > we close the path of integration in the upper half plane and use the Residue theorem. There are two simple poles in the upper half plane. For x < we close the path of integration in the lower half plane. Since 1600

the integrand is analytic there, the integral is zero. G(x; ) = 0 for x < . For x > we have G(x; ) = 1 2 Res 2 e(x) , = + ( + )( ) + Res e(x) , = ( + )( )

G(x; ) =

e(+)(x) e(+)(x) + 2 2 1 G(x; ) = e(x) sin((x )). 1 (x) e sin((x ))H(x ).

Thus the Green function is G(x; ) =

2. We use the Green function to nd the solution of the inhomogeneous equation. y + 2y + 2 + 2 y = g(x),

y() = y() = 0

y(x) =

g()G(x; ) d

y(x) =

1 g() e(x) sin((x ))H(x ) d x 1 y(x) = g() e(x) sin((x )) d 1


x

We take the limit 0. y= g() sin((x )) d

1601

Solution 32.17 First we consider the Fourier transform of f (x) = 1/(x2 + c2 ) where 1 + c2

(c) > 0.

f () = F 1 = 2 1 = 2

x2

1 ex dx + c2 ex dx (x c)(x + c) x2

If < 0 then we close the path of integration with a semi-circle in the upper half plane. ex 1 f () = 2i Res , x = c 2 (x c)(x + c) ec = , for < 0 2c Note that f (x) = 1/(x2 + c2 ) is an even function of x so that f () is an even function of . If f () = g() for < 0 then f () = g(||) for all . Thus F 1 c|| 1 e = . 2 +c 2c

x2

Now we consider the integral equation


u() 1 d = 2 2 + a2 ] [(x ) x + b2 1602

0 < a < b.

We take the Fourier transform, utilizing the convolution theorem.

ea|| eb|| = 2a 2b (ba)|| ae u() = 2b 1 a u(x) = 2(b a) 2 2b x + (b a)2 2() u u(x) = a(b a) b(x2 + (b a)2 )

Solution 32.18 1. Note that fc () is an even function. We compute the inverse Fourier cosine transform.

1 f (x) = Fc fc ()

fc () ex d fc ()(cos(x) + sin(x)) d

= = =2
0

fc () cos(x) d

fc () cos(x) d

1603

2. Fc [y ] = 1 y cos(x) dx 0 1 = [y cos(x)]0 + y sin(x) dx 0 1 2 = y (0) + [y sin(x)] y cos(x) dx 0 0 Fc [y ] = 2 yc () y (0)

3. We take the Fourier cosine transform of the dierential equation. 2 y () b a2 y () = 0 b y () = ( 2 + a2 )

Now we take the inverse Fourier cosine transform. We use the fact that y () is an even function.
1 y(x) = Fc

b + a2 ) b = F 1 2 + a2 ) ( b 1 ex , = a = 2 Res 2 + a2 ex = 2b lim , for x 0 a + a ( 2 b y(x) = eax a 1604

Solution 32.19 1. Suppose f (x) is an odd function. The Fourier transform of f (x) is F[f (x)] = 1 2 1 = 2 =

f (x) ex dx

f (x)(cos(x) sin(x)) dx

f (x) sin(x) dx.


0

Note that f () = F[f (x)] is an odd function of . The inverse Fourier transform of f () is

F 1 [f ()] =

f () ex d

= 2
0

f () sin(x) d.

Thus we have that

f (x) = 2
0

f (x) sin(x) dx sin(x) d


0

=2
0

f (x) sin(x) dx sin(x) d.

This gives us the Fourier sine transform pair

f (x) = 2
0

fs () sin(x) d,

1 fs () =

f (x) sin(x) dx.


0

1605

2. Fs [y ] = 1 y sin(x) dx 0 1 = y sin(x) y cos(x) dx 0 0 2 = y cos(x) y sin(x) dx 0 0 Fs [y ] = 2 ys () + 3. We take the Fourier sine transform of the dierential equation. 2 y () + b a2 y () = 0 b y () = 2 + a2 ) ( y(0)

Now we take the inverse Fourier sine transform. We use the fact that y () is an odd function.
1 y(x) = Fs

b + a2 ) b = F 1 ( 2 + a2 ) b ex , = a = 2 Res 2 + a2 x e = 2b lim a + a ax = be for x 0 ( 2 y(x) = b eax

1606

4. Now we solve the dierential equation with the Laplace transform. y a2 y = 0 s2 y (s) sy(0) y (0) a2 y (s) = 0 We dont know the value of y (0), so we treat it as an unknown constant. bs + y (0) s 2 a2 y (0) y(x) = b cosh(ax) + sinh(ax) a y (s) = In order to satisfy the boundary condition at innity we must choose y (0) = ab. y(x) = b eax We see that solving the dierential equation with the Laplace transform is not as convenient, because the boundary condition at innity is not automatically satised. We had to nd a value of y (0) so that y() = 0. Solution 32.20 The Fourier, Fourier cosine and Fourier sine transforms are dened: 1 2 1 F[f (x)]c = 1 F[f (x)]s = F[f (x)] =

f (x) ex dx,

f (x) cos(x) dx,


0

f (x) sin(x) dx.


0

We start with the right side of the identity and apply the usual tricks of integral calculus to reduce the expression to the left side. 1 (Fc [f (x) + f (x)] Fs [f (x) f (x)]) 2 1607

1 2 1 2 1 2

f (x) cos(x) dx +
0 0

f (x) cos(x) dx
0

f (x) sin(x) dx +
0

f (x) sin(x) dx

f (x) cos(x) dx
0 0

f (x) cos(x) dx
0 0

f (x) sin(x) dx
0 0

f (x) sin(x) dx f (x) sin(x) dx

f (x) cos(x) dx +
0

f (x) cos(x) dx
0

f (x) sin(x) dx

1 2

f (x) cos(x) dx

f (x) sin(x) dx

1 2

f (x) ex dx

F[f (x)] Solution 32.21 We take the Fourier transform of the integral equation, noting that the left side is the convolution of u(x) and 1 1 =F 2 2 +a x + b2

1 . x2 +a2

2()F u

x2

We nd the Fourier transform of f (x) = even, real-valued function. F

1 . x2 +c2

Note that since f (x) is an even, real-valued function, f () is an

1 1 = 2 + c2 x 2

x2

1 ex dx + c2

1608

For x > 0 we close the path of integration in the upper half plane and apply Jordans Lemma to evaluate the integral in terms of the residues. = 1 2 Res 2 ec = 2c 1 c e = 2c ex , x = c (x c)(x + c)

Since f () is an even function, we have F Our equation for u() becomes, 2() u 1 a|| 1 b|| e e = 2a 2b a (ba)|| e u() = . 2b x2 1 c|| 1 e = . 2 +c 2c

We take the inverse Fourier transform using the transform pair we derived above. u(x) = u(x) = a 2(b a) 2 + (b a)2 2b x a(b a) b(x2 + (b a)2 )

1609

Chapter 33 The Gamma Function


33.1 Eulers Formula

For non-negative, integral n the factorial function is n! = n(n 1) (1), with 0! = 1.

We would like to extend the factorial function so it is dened for all complex numbers. Consider the function (z) dened by Eulers formula

(z) =
0

et tz1 dt. (z) 0 then the integrand will

(Here we take the principal value of tz1 .) The integral converges for (z) > 0. If be at least as singular as 1/t at t = 0 and thus the integral will diverge.

1610

Dierence Equation. Using integration by parts,

(z + 1) =
0

et tz dt
t z

= e Since (z) > 0 the rst term vanishes.

0 0

et ztz1 dt.

=z
0

et tz1 dt

= z(z) Thus (z) satises the dierence equation (z + 1) = z(z). For general z it is not possible to express the integral in terms of elementary functions. However, we can evaluate the integral for some z. The value z = 1 looks particularly simple to do.

(1) =
0

et dt = et

= 1.
0

Using the dierence equation we can nd the value of (n) for any positive, integral n. (1) = 1 (2) = 1 (3) = (2)(1) = 2 (4) = (3)(2)(1) = 6 = (n + 1) = n!. 1611

Thus the Gamma function, (z), extends the factorial function to all complex z in the right half-plane. For nonnegative, integral n we have (n + 1) = n!.

Analyticity. The derivative of (z) is

(z) =
0

et tz1 log t dt.

Since this integral converges for

(z) > 0, (z) is analytic in that domain.

33.2

Hankels Formula

We would like to nd the analytic continuation of the Gamma function into the left half-plane. We accomplish this with Hankels formula 1 et tz1 dt. (z) = 2 sin(z) C Here C is the contour starting at below the real axis, enclosing the origin and returning to above the real axis. A graph of this contour is shown in Figure 33.1. Again we use the principle value of tz1 so there is a branch cut on the negative real axis. The integral in Hankels formula converges for all complex z. For non-positive, integral z the integral does not vanish. Thus because of the sine term the Gamma function has simple poles at z = 0, 1, 2, . . .. For positive, integral z, the integrand is entire and thus the integral vanishes. Using LHospitals rule you can show that the points, z = 1, 2, 3, . . . are removable singularities and the Gamma function is analytic at these points. Since the only zeroes of sin(z) occur for integral z, (z) is analytic in the entire plane except for the points, z = 0, 1, 2, . . .. 1612

Figure 33.1: The Hankel Contour. Dierence Equation. Using integration by parts we can derive the dierence equation from Hankels formula. (z + 1) = 1 et tz dt 2 sin((z + 1)) C +0 1 et tz = 2 sin(z) 0 1 et tz1 dt = z 2 sin(z) C = z(z).

et ztz1 dt
C

Evaluating (1), (1) = lim et tz1 dt . 2 sin(z)


C

z1

1613

Both the numerator and denominator vanish. Using LHospitals rule, et tz1 log t dt = lim z1 2 cos(z) t e log t dt = C 2
C

Let Cr be the circle of radius r starting at radians and going to radians. 1 2 1 = 2 1 = 2 =


r

et [log(t) i] dt +
r Cr

et log t dt +
r

et [log(t) + i] dt et log t dt
Cr

et [ log(t) + i] dt +
r

et [log(t) + i] dt +

et 2 dt +
r Cr

et log t dt

The integral on Cr vanishes as r 0. 1 2 2 = 1. =

et dt
0

Thus we obtain the same value as with Eulers formula. It can be shown that Hankels formula is the analytic continuation of the Gamma function into the left half-plane.

33.3

Gauss Formula

Gauss dened the Gamma function as an innite product. This form is useful in deriving some of its properties. We can obtain the product form from Eulers formula. First recall that e
t

= lim

t 1 n

1614

Substituting this into Eulers formula,

(z) =
0

et tz1 dt
n

= lim With the substitution = t/n,

1
0

t n

tz1 dt.

= lim

(1 )n nz1 z1 n d
0 1

= lim nz
n 0

(1 )n z1 d.

Let n be an integer. Using integration by parts we can evaluate the integral.


1 0

(1 )n z1 d =

1 z (1 )n z n(1 )n1 d z z 0 0 n 1 (1 )n1 z d = z 0 n(n 1) 1 (1 )n2 z+1 d = z(z + 1) 0 1 n(n 1) (1) = z+n1 d z(z + 1) (z + n 1) 0

z+n n(n 1) (1) z(z + 1) (z + n 1) z + n n! = z(z + 1) (z + n) = 1615

1 0

Thus we have that (z) = lim nz n! n z(z + 1) (z + n) 1 (1)(2) (n) = lim nz n (z + 1)(z + 2) (z + n) z 1 1 = lim nz z n (1 + z)(1 + z/2) (1 + z/n) 1 2z 3z nz 1 = lim z n (1 + z)(1 + z/2) (1 + z/n) 1z 2z (n 1)z

Since limn

(n+1)z nz

= 1 we can multiply by that factor. = 1 1 2z 3z (n + 1)z lim z n (1 + z)(1 + z/2) (1 + z/n) 1z 2z nz

1 1 (n + 1)z = z n=1 1 + z/n nz Thus we have Gauss formula for the Gamma function 1 (z) = z n=1

1 1+ n

1+

z n

We derived this formula from Eulers formula which is valid only in the left half-plane. However, the product formula is valid for all z except z = 0, 1, 2, . . ..

33.4

Weierstrass Formula
1616

The Euler-Mascheroni Constant. Before deriving Weierstrass product formula for the Gamma function we will need to dene the Euler-Mascheroni constant 1 1 1 + + + 2 3 n

= lim

1+

log n = 0.5772 .

In deriving the Euler product formula, we had the equation n! . z(z + 1) (z + n) z 1 z 1 z 1 z = lim z 1 1 + 1+ 1 + n n 1 2 n 1 z z z z log n e = lim z 1 + 1+ 1 + n (z) 1 2 n z z z z/2 z z/n e e e 1+ 1 + exp = lim z 1 + n 1 2 n (z) = lim nz
n

1+

1 1 + + log n z 2 n

Weierstrass formula for the Gamma function is then 1 = z ez (z) n=1

1+

z z/n e . n

Since the product is uniformly convergent, 1/(z) is an entire function. Since 1/(z) has no singularities, we see that (z) has no zeros. 1617

Result 33.4.1 Eulers formula for the Gamma function is valid for

(z) > 0.

(z) =
0

et tz1 dt

Hankels formula denes the (z) for the entire complex plane except for the points z = 0, 1, 2, . . .. 1 et tz1 dt (z) = 2 sin(z) C Gauss and Weierstrass product formulas are, respectively 1 (z) = z n=1

1 1+ n

1+

z n

and

1 = z ez (z) n=1

1+

z z/n e . n

33.5

Stirlings Approximation

In this section we will try to get an approximation to the Gamma function for large positive argument. Eulers formula is (x) =
0

et tx1 dt.

We could rst try to approximate the integral by only looking at the domain where the integrand is large. In Figure 33.2 the integrand in the formula for (10), et t9 , is plotted. 1618

40000 30000 20000 10000 5 10 15 20 25 30

Figure 33.2: Plot of the integrand for (10)

We see that the important part of the integrand is the hump centered around x = 9. If we nd where the integrand of (x) has its maximum

d t x1 e t =0 dx et tx1 + (x 1) et tx2 = 0 (x 1) t = 0 t = x 1,

we see that the maximum varies with x. This could complicate our analysis. To take care of this problem we introduce 1619

the change of variables t = xs.

(x) =
0

exs (xs)x1 x ds

= xx
0

exs sx s1 ds ex(slog s) s1 ds
0

= xx

The integrands, (ex(slog s) s1 ), for (5) and (20) are plotted in Figure 33.3.
0.007 0.006 0.005 0.004 0.003 0.002 0.001 1 2 3 4
-9

210 1.510

-9 -9

110 510

-10

Figure 33.3: Plot of the integrand for (5) and (20). We see that the important part of the integrand is the hump that seems to be centered about s = 1. Also note that the the hump becomes narrower with increasing x. This makes sense as the ex(slog s) term is the most rapidly varying term. Instead of integrating from zero to innity, we could get a good approximation to the integral by just integrating over some small neighborhood centered at s = 1. Since s log s has a minimum at s = 1, ex(slog s) has a maximum there. Because the important part of the integrand is the small area around s = 1, it makes sense to 1620

approximate s log s with its Taylor series about that point. 1 s log s = 1 + (s 1)2 + O (s 1)3 2 Since the hump becomes increasingly narrow with increasing x, we will approximate the 1/s term in the integrand with its value at s = 1. Substituting these approximations into the integral, we obtain
1+

(x) x

x 1

ex(1+(s1)
1+

2 /2)

ds ds

= xx ex
1

ex(s1)

2 /2

As x both of the integrals


1

ex(s1)

2 /2

ds

and
1+

ex(s1)

2 /2

ds

are exponentially small. Thus instead of integrating from 1 to 1 + we can integrate from to .

(x) xx ex

ex(s1) exs

2 /2

2 /2

ds

= xx ex = xx ex (x)

ds

2 x as x .

2xx1/2 ex

This is known as Stirlings approximation to the Gamma function. In the table below, we see that the approximation is pretty good even for relatively small argument. 1621

n 5 15 25 35 45

(n) 24 8.71783 1010 6.20448 1023 2.95233 1038 2.65827 1054

2xx1/2 ex relative error 23.6038 0.0165 8.66954 1010 0.0055 6.18384 1023 0.0033 2.94531 1038 0.0024 54 2.65335 10 0.0019

In deriving Stirlings approximation to the Gamma function we did a lot of hand waving. However, all of the steps can be justied and better approximations can be obtained by using Laplaces method for nding the asymptotic behavior of integrals.

1622

33.6

Exercises

2

Exercise 33.1 Given that ex dx = deduce the value of (1/2). Now nd the value of (n + 1/2). Exercise 33.2 3 Evaluate 0 ex dx in terms of the gamma function. Exercise 33.3 Show that

ex sin(log x) dx =
0

() + () . 2

1623

33.7

Hints

Hint 33.1 Use the change of variables, = x2 in the integral. To nd the value of (n + 1/2) use the dierence relation. Hint 33.2 Make the change of variable = x3 . Hint 33.3

1624

33.8

Solutions

Solution 33.1

ex dx = e
0 x2

dx =

Make the change of variables = x2 .


0

1 1/2 e d = 2 2 (1/2) =

Recall the dierence relation for the Gamma function (z + 1) = z(z). (n + 1/2) = (n 1/2)(n 1/2) 2n 1 = (n 1/2) 2 (2n 3)(2n 1) = (n 3/2) 22 (1)(3)(5) (2n 1) = (1/2) 2n (n + 1/2) = (1)(3)(5) (2n 1) 2n 1625

Solution 33.2 We make the change of variable = x3 , x = 1/3 , dx = 1 2/3 d. 3


0

ex dx =

1 e 2/3 d 3 0 1 1 = 3 3

Solution 33.3

ex sin(log x) dx =
0 0

ex

1 log x e e log x dx 2

= = = =

1 x e x x dx 2 0 1 ((1 + ) (1 )) 2 1 (() ()()) 2 () + () 2

1626

Chapter 34 Bessel Functions


Ideas are angels. Implementations are a bitch.

34.1

Bessels Equation
1 2 y + y + 1 2 z z

A commonly encountered dierential equation in applied mathematics is Bessels equation y = 0.

For our purposes, we will consider R0+ . This equation arises when solving certain partial dierential equations with the method of separation of variables in cylindrical coordinates. For this reason, the solutions of this equation are sometimes called cylindrical functions. This equation cannot be solved directly. However, we can nd series representations of the solutions. There is a regular singular point at z = 0, so the Frobenius method is applicable there. The point at innity is an irregular singularity, so we will look for asymptotic series about that point. Additionally, we will use Laplaces method to nd denite integral representations of the solutions. 1627

Note that Bessels equation depends only on 2 and not alone. Thus if we nd a solution, (which of course depends on this parameter), y (z) we know that y (z) is also a solution. For this reason, we will consider R0+ . Whether or not y (z) and y (z) are linearly independent, (distinct solutions), remains to be seen. Example 34.1.1 Consider the dierential equation 1 2 y + y + 2y = 0 z z One solution is y (z) = z . Since the equation depends only on 2 , another solution is y (z) = z . For = 0, these two solutions are linearly independent. Now consider the dierential equation y + 2y = 0 One solution is y (z) = cos(z). Therefore, another solution is y (z) = cos(z) = cos(z). However, these two solutions are not linearly independent.

34.2

Frobeneius Series Solution about z = 0

We note that z = 0 is a regular singular point, (the only singular point of Bessels equation in the nite complex plane.) We will use the Frobenius method at that point to analyze the solutions. We assume that 0. The indicial equation is ( 1) + 2 = 0 = . If do not dier by an integer, (that is if is not a half-integer), then there will be two series solutions of the Frobenius form. y1 (z) = z
k=0

ak z k ,

y2 (z) = z
k=0

bk z k

1628

If is a half-integer, the second solution may or may not be in the Frobenius form. In any case, then will always be at least one solution in the Frobenius form. We will determine that series solution. y(z) and it derivatives are

y=
k=0

ak z k+ ,

y =
k=0

(k + )ak z k+1 ,

y =
k=0

(k + )(k + 1)ak z k+2 .

We substitute the Frobenius series into the dierential equation. z 2 y + zy + z 2 2 y = 0


(k + )(k + 1)ak z
k=0

k+

+
k=0

(k + )ak z

k+

+
k=0

ak z

k++2

k=0

2 ak z k+ = 0

k 2 + 2k ak z k +
k=0 k=2

ak2 z k = 0

We equate powers of z to obtain equations that determine the coecients. The coecient of z 0 is the equation 0 a0 = 0. This corroborates that a0 is arbitrary, (but non-zero). The coecient of z 1 is the equation (1 + 2)a1 = 0 a1 = 0 The coecient of z k for k 2 gives us k 2 + 2k ak + ak2 = 0. ak2 ak2 ak = 2 = k + 2k k(k + 2) From the recurrence relation we see that all the odd coecients are zero, a2k+1 = 0. The even coecients are a2k = a2k2 (1)k a0 = 2k 4k(k + ) 2 k!(k + + 1) 1629

Thus we have the series solution y(z) = a0

k=0

(1)k z 2k . 22k k!(k + + 1)

a0 is arbitrary. We choose a0 = 2 . We call this solution the Bessel function of the rst kind and order and denote it with J (z). z 2k+ (1)k J (z) = k!(k + + 1) 2 k=0 Recall that the Gamma function is non-zero and nite for all real arguments except non-positive integers. (x) has singularities at x = 0, 1, 2, . . .. Therefore, J (z) is well-dened when is not a positive integer. Since J (z) z at z = 0, J (z) is clear linearly independent to J (z) for non-integer . In particular we note that there are two solutions of the Frobenius form when is a half odd integer.

J (z) =
k=0

(1)k z k!(k + 1) 2

2k

for Z+

Of course for = 0, J (z) and J (z) are identical. Consider the case that = n is a positive integer. Since (x) + as x 0, 1, 2, . . . we see the the coecients in the series for Jnu (z) vanish for k = 0, . . . , n 1.

Jn (z) =
k=n

(1)k z k!(k n + 1) 2

2kn

Jn (z) =
k=0

(1)k+n z (k + n)!(k + 1) 2
n k=0

2k+n

Jn (z) = (1)

(1)k z k!(k + n)! 2

2k+n

Jn (z) = (1)n Jn (z) Thus we see that Jn (z) and Jn (z) are not linearly independent for integer n. 1630

34.2.1

Behavior at Innity
4 u + 2 3 u + 2 u + 1 2 2 u = 0 1 1 2 u + u + u = 0. 4 2

With the change of variables z = 1/, w(z) = u() Bessels equation becomes

The point = 0 and hence the point z = is an irregular singular point. We will nd the leading order asymptotic behavior of the solutions as z +. Controlling Factor. We starti with Bessels equation for real argument. 1 2 y + y + 1 2 x x We make the substitution y = es(x) . 2 1 s + (s )2 + s + 1 2 = 0 x x 2 1 as x ; we will assume that s (s ) as x . y=0

We know that

2 x2

1 (s )2 + s + 1 0 as x x To simplify the equation further, we will try the possible two-term balances.
1 1 1. (s )2 + x s 0 s x than the other terms.

This balance is not consistent as it violates the assumption that 1 is smaller This balance is consistent. This balance is inconsistent as (s )2 isnt smaller than the other terms. (s )2 .

2. (s )2 + 1 0 3.
1 s x

s s x

+10

Thus the only dominant balance is s . This balance is consistent with our initial assumption that s Thus s x and the controlling factor is ex . 1631

Leading Order Behavior. In order to nd the leading order behavior, we substitute s = x+t(x) where t(x) x as x into the dierential equation for s. We rst consider the case s = x + t(x). We assume that t 1 and t 1/x. 2 1 t + ( + t ) + ( + t ) + 1 2 = 0 x x 1 2 t + 2t + (t )2 + + t 2 = 0 x x x
2

We use our assumptions about the behavior of t and t . 2t + 0 x 1 t 2x

1 t ln x as x . 2 This asymptotic behavior is consistent with our assumptions. Substituting s = x + t(x) will also yield t 1 ln x. Thus the leading order behavior of the solutions is 2 y c ex 2 ln x+u(x) = cx1/2 ex+u(x)
1

as x ,

where u(x) ln x as x . 1 By substituting t = 2 ln x+u(x) into the dierential equation for t, you could show that u(x) const as x . Thus the full leading order behavior of the solutions is y cx1/2 ex+u(x) as x

where u(x) 0 as x . Writing this in terms of sines and cosines yields y1 x1/2 cos(x + u1 (x)), y2 x1/2 sin(x + u2 (x)), 1632 as x ,

where u1 , u2 0 as x .

Result 34.2.1 Bessels equation for real argument is 1 2 y + y + 1 2 x x y = 0.

If is not an integer then the solutions behave as linear combinations of y1 = x , and y2 = x

at x = 0. If is an integer, then the solutions behave as linear combinations of y1 = x , and y2 = x + cx log x

at x = 0. The solutions are asymptotic to a linear combination of y1 = x1/2 sin(x + u1 (x)), as x +, where u1 , u2 0 as x . and y2 = x1/2 cos(x + u2 (x))

34.3

Bessel Functions of the First Kind

Consider the function exp( 1 z(t 1/t)). We can expand this function in a Laurent series in powers of t, 2

e 2 z(t1/t) =
n=

Jn (z)tn ,

1633

where the coecient functions Jn (z) are Jn (z) = 1 2 n1 e 2 z( 1/ ) d.


1

1 Here the path of integration is any positive closed path around the origin. exp( 2 z(t 1/t)) is the generating function for Bessel function of the rst kind.

34.3.1

The Bessel Function Satises Bessels Equation


n1

We would like to expand Jn (z) in powers of z. The rst step in doing this is to make the substitution = 2t/z. 1 Jn (z) = 2 1 = 2 We dierentiate the expression for Jn (z). Jn (z) = 1 nz n1 2 2n 1 z n = 2 2 z 2 z 2 z 2
n

2t z z 2
n

exp tn1 etz

1 z 2
2 /4t

2t z z 2t dt

2 dt z

tn1 etz

2 /4t

dt +

1 2

z 2

tn1

2z 4t

etz

2 /4t

dt

n z n1 tz2 /4t e t dt z 2t n n z n 1 z n z 2 + 2 tn1 etz /4t dt z z 2t z 2t 2t z 2t 2 2 n nz n 1 nz z 2 2 + 2 tn1 etz /4t dt 2 z 2zt z 2t 2zt 4t n(n 1) 2n + 1 z2 2 + 2 tn1 etz /4t dt 2 z 2t 4t 1634

Jn (z) =

1 2 1 = 2 1 = 2

We substitute Jn (z) into Bessels equation. 1 Jn + J n + 1 z 1 z n = 2 2 1 z n = 2 2 1 z n = 2 2 Since tn1 etz


2 /4t

n2 z2

Jn + 1 n2 z2 tn1 etz
2 /4t

n(n 1) 2n + 1 z2 n 1 + 2 + z2 2t 4t z 2 2t 2 n+1 z 2 1 + 2 tn1 etz /4t dt t 4t d n1 tz2 /4t e t dt dt

dt

is analytic in 0 < |t| < when n is an integer, the integral vanishes.

= 0. Thus for integer n, Jn (z) satises Bessels equation. Jn (z) is called the Bessel function of the rst kind. The subscript is the order. Thus J1 (z) is a Bessel function of order 1. J0 (x) and J1 (x) are plotted in the rst graph in Figure 34.1. J5 (x) is plotted in the second graph in Figure 34.1. Note that for non-negative, integer n, Jn (z) behaves as z n at z = 0.

34.3.2

Series Expansion of the Bessel Function


1 2 1 2 z 2 z 2
n

We expand exp(z 2 /4t) in the integral expression for Jn . Jn (z) = = tn1 etz tn1 et
m=0
2 /4t

dt z 2 4t
m

1 m!

dt

1635

1 0.8 0.6 0.4 0.2 5 2 -0.2 -0.4 4 6 8 10 12 14 -0.1 -0.2 10 15 20 0.3 0.2 0.1

Figure 34.1: Plots of J0 (x), J1 (x) and J5 (x).

For the path of integration, we are free to choose any contour that encloses the origin. Consider the circular path on |t| = 1. Since the integral is uniformly convergent, we can interchange the order of integration and summation.

Jn (z) =

1 2

z 2

(1)m z 2m 22m m! m=0 1636

tnm1 et dt

Let n be a non-negative integer. 1 2 tnm1 et dt = lim = We have the series expansion Jn (z) = z (1)m m!(n + m)! 2 m=0
n+2m z0

1 dn+m z (e ) (n + m)! dz n+m

1 (n + m)!

for n 0.

Now consider Jn (z), (n positive). 1 Jn (z) = 2 z 2


n

(1)m z 2m 22m m! m=1


1 (mn)!

tnm1 et dt

For m n, the integrand has a pole of order m n + 1 at the origin. 1 2 The expression for Jn is then (1)m z Jn (z) = m!(m n)! 2 m=n = (1)m+n z (m + n)!m! 2 m=0
n+2m

tnm1 et dt =

for m n for m < n

n+2m

= (1)n Jn (z). Thus we have that Jn (z) = (1)n Jn (z) for integer n.

1637

34.3.3

Bessel Functions of Non-Integer Order

The generalization of the factorial function is the Gamma function. For integer values of n, n! = (n + 1). The Gamma function is dened for all complex-valued arguments. Thus one would guess that if the Bessel function of the rst kind were dened for non-integer order, it would have the denition, J (z) = (1)m z m!( + m + 1) 2 m=0
+2m

The Integrand for Non-Integer . Recall the denition of the Bessel function 1 z 2 J (z) = t1 etz /4t dt. 2 2 When is an integer, the integrand is single valued. Thus if you start at any point and follow any path around the origin, the integrand will return to its original value. This property was the key to Jn satisfying Bessels equation. If is not an integer, then this property does not hold for arbitrary paths around the origin. A New Contour. First, since the integrand is multiple-valued, we need to dene what branch of the function we are talking about. We will take the principal value of the integrand and introduce a branch cut on the negative real axis. Let C be a contour that starts at z = below the branch cut, circles the origin, and returns to the point z = above the branch cut. This contour is shown in Figure 34.2. Thus we dene 1 z 2 t1 etz /4t dt. J (z) = 2 2 C Bessels Equation. Substituting J (z) into Bessels equation yields 1 2 J + J + 1 2 z z
1 etz 2 /4t

J =

1 2

z 2

d 1 tz2 /4t e t dt. dt

Since t is analytic in 0 < |z| < and | arg(z)| < , and it vanishes at z = , the integral is zero. Thus the Bessel function of the rst kind satises Bessels equation for all complex orders. 1638

Figure 34.2: The Contour of Integration. Series Expansion. Because of the et factor in the integrand, the integral dening J converges uniformly. Expanding 2 ez /4t in a Taylor series yields J (z) = Since 1 1 = () 2 we have the series expansion of the Bessel function (1)m z m!( + m + 1) 2 m=0
+2m

1 2

z 2

(1)m z 2m 22m m! m=0

tm1 et dt
C

t1 et dt,
C

J (z) =

1639

Linear Independence. We use Abels formula to compute the Wronskian of Bessels equation.
z

W (z) = exp

1 d

= e log z =

1 z

Thus to within a function of , the Wronskian of any two solutions is 1/z. For any given , there are two linearly independent solutions. Note that Bessels equation is unchanged under the transformation . Thus both J and J satisfy Bessels equation. Now we must determine if they are linearly independent. We have already shown that for integer values of they are not independent. (Jn = (1)n Jn .) Assume that is not an integer. We compute the Wronskian of J and J . W [J , J ] = J J J J

= J J J J We substitute in the expansion for J = (1)m z m!( + m + 1) 2 m=0


m +2m

n=0 +2m

(1)n ( + 2n) z n!( + n + 1)2 2

+2n1

(1) z m!( + m + 1) 2 m=0

n=0

(1)n ( + 2n) z n!( + n + 1)2 2

+2n1

Since the Wronskian is a function of times 1/z the coecients of all of the powers of z except 1/z must vanish. = z( + 1)( + 1) z( + 1)( + 1) 2 = z()(1 ) 1640

Using an identity for the Gamma function simplies this expression. = 2 sin() z

Since the Wronskian is nonzero for non-integer , J and J are independent functions when is not an integer. In this case, the general solution of Bessels equation is aJ + bJ .

34.3.4

Recursion Formulas

In showing that J satises Bessels equation for arbitrary complex , we obtained d tz2 /4t t e dt = 0. dt

Expanding the integral, t +


C

1 2 Since J (z) =
1 (z/2) 2 C

z 2

z 2 2 2 t t1 etz /4t dt = 0. 4 z2 2 t + t2 t1 etz /4t dt = 0. 4

t1 etz

2 /4t

dt, 2 z
1

J1 +

2 z

z2 J+1 J = 0. 4 2 J z

J1 + J+1 =

1641

Dierentiating the integral expression for J ,

J (z) =

1 z 1 2 2 1 z J (z) = z 2 2

t1 etz
C C

2 /4t

dt +

t1 e

tz 2 /4t

z 2 1 z dt 2 2

1 2

t1
C +1 C

z 2 etz /4t dt 2t
2 /4t

t2 etz

dt

J =

J J+1 z

From the two relations we have derived you can show that

1 J = (J1 + J+1 ) 2

and

J = J1 J . z

1642

Result 34.3.1 The Bessel function of the rst kind, J (z), is dened, 1 z J (z) = 2 2 The Bessel function has the expansion, J (z) = z (1)m m!( + m + 1) 2 m=0
+2m C

t1 etz

/4t

dt.

The asymptotic behavior for large argument is J (z) 2 cos z + e| z 2 4


(z)|

O |z|1

as |z| , | arg(z)| < .

The Wronskian of J (z) and J (z) is W (z) = 2 sin(). z

Thus J (z) and J (z) are independent when is not an integer. The Bessel functions satisfy the recursion relations, J1 + J+1 = 2 J z J = J J+1 z J = J1 J . z

1 J = (J1 J+1 ) 2

1643

34.3.5

Bessel Functions of Half-Integer Order

Consider J1/2 (z). Start with the series expansion J1/2 (z) = (1)m z m!(1/2 + m + 1) 2 m=0
1/2+2m

Use the identity (n + 1/2) =

(1)(3)(2n1) . 2n

(1)m 2m+1 m!(1)(3) (2m + 1) m=0

z 2

1/2+2m

(1)m 2m+1 = (2)(4) (2m) (1)(3) (2m + 1) m=0 = 2 z


1/2

1 2

1/2+m

z 1/2+2m

(1)m 2m+1 z (2m + 1)! m=0

We recognize the sum as the Taylor series expansion of sin z. 2 z


1/2

= Using the recurrence relations,

sin z

J+1 = we can nd Jn+1/2 for any integer n.

J J z

and J1 =

J + J , z

1644

Example 34.3.1 To nd J3/2 (z), J3/2 (z) = 1/2 J1/2 (z) J1/2 (z) z 1/2 1/2 2 1 = z 1/2 sin z z 2 2 2
1/2

1/2

3/2

sin z

1/2

z 1/2 cos z

= 21/2 1/2 z 3/2 sin z + 21/2 1/2 z 3/2 sin z 21/2 1/2 cos z = = You can show that J1/2 (z) = 2 z z 3/2 sin z
1/2

1/2

z 1/2 cos z

z 3/2 sin z z 1/2 cos z .


1/2

cos z.

Note that at a rst glance it appears that J3/2 z 1/2 as z 0. However, if you expand the sine and cosine you will see that the z 1/2 and z 1/2 terms vanish and thus J3/2 (z) z 3/2 as z 0 as we showed previously. Recall that we showed the asymptotic behavior as x + of Bessel functions to be linear combinations of x1/2 sin(x + U1 (x)) and x1/2 cos(x + U2 (x)) where U1 , U2 0 as x +.

34.4

Neumann Expansions

Consider expanding an analytic function in a series of Bessel functions of the form f (z) =
n=0

an Jn (z).

1645

If f (z) is analytic in the disk |z| r then we can write f (z) = 1 2 f () d, z

1 where the path of integration is || = r and |z| < r. If we were able to expand the function z in a series of Bessel functions, then we could interchange the order of summation and integration to get a Bessel series expansion of f (z). 1 z

The Expansion of 1/( z). Assume that

has the uniformly convergent expansion

1 = c0 ()J0 (z) + 2 cn ()Jn (z), z n=1 where each cn () is analytic. Note that + z Thus we have + z c 0 J0 + 2
n=1

1 1 1 = + = 0. 2 z ( z) ( z)2

c0 ()J0 (z) + 2
n=1

cn ()Jn (z) = 0

c n Jn + c 0 J0 + 2
n=1

cn Jn = 0.

Using the identity 2Jn = Jn1 Jn+1 ,


c 0 J0 + 2
n=1

cn Jn + c0 (J1 ) +
n=1

cn (Jn1 Jn+1 ) = 0.

1646

Collecting coecients of Jn ,

(c0 + c1 )J0 +
n=1

(2cn + cn+1 cn1 )Jn = 0.

Equating the coecients of Jn , we see that the cn are given by the relations, c1 = c0 , We can evaluate c0 (). Setting z = 0, 1 = c0 ()J0 (0) + 2 cn ()Jn (0) n=1 1 = c0 (). Using the recurrence relations we can calculate the cn s. The rst few are: c1 = 1 1 = 2 2 1 2 1 4 c2 = 2 3 = + 3 1 12 3 24 1 4 = 2 + 4. c3 = 2 2 2

and cn+1 = cn1 2cn .

We see that cn is a polynomial of degree n + 1 in 1/. One can show that 4 n 2n1 n! 1 + 2 + + + 24n(2n2)(2nn) n+1 2(2n2) 24(2n2)(2n4) cn () = 4 n1 2n1 n! 1 + 2 + + + 24(n1)(2n2)(2n(n1)) n+1 2(2n2) 24(2n2)(2n4) 1647

for even n for odd n

Uniform Convergence of the Series. We assumed before that the series expansion of The behavior of cn and Jn are cn () = This gives us 1 cn ()Jn (z) = 2 If
z

1 z

is uniformly convergent.

2n1 n! + O( n ), n+1

Jn (z) =

zn + O(z n+1 ). 2n n!

+O

n+1

= < 1 we can bound the series with the geometric series

n . Thus the series is uniformly convergent.

Neumann Expansion of an Analytic Function. Let f (z) be a function that is analytic in the disk |z| r. Consider |z| < r and the path of integration along || = r. Cauchys integral formula tells us that f (z) = Substituting the expansion for
1 , z

1 2

f () d. z

1 = 2

f () co ()J0 (z) + 2
n=1

cn ()Jn (z)

1 = J0 (z) 2 = J0 (z)f (0) +

f () Jn (z) d + n=1

cn ()f () d

n=1

Jn (z)

cn ()f () d.

1648

Result 34.4.1 let f (z) be analytic in the disk, |z| r. Consider |z| < r and the path of integration along || = r. f (z) has the Bessel function series expansion

f (z) = J0 (z)f (0) +


n=1

Jn (z)

cn ()f () d,

where the cn satisfy

1 = c0 ()J0 (z) + 2 cn ()Jn (z). z n=1

34.5

Bessel Functions of the Second Kind

When is an integer, J and J are not linearly independent. In order to nd an second linearly independent solution, we dene the Bessel function of the second kind, (also called Webers function),

Y =

J (z) cos()J (z) sin() lim J (z) cos()J (z) sin()

when is not an integer when is an integer.

J and Y are linearly independent for all . In Figure 34.3 Y0 and Y1 are plotted in solid and dashed lines, respectively. 1649

0.5 0.25 5 -0.25 -0.5 -0.75 -1 10 15 20

Figure 34.3: Bessel Functions of the Second Kind

Result 34.5.1 The Bessel function of the second kind, Y (z), is dened, Y =
J (z) cos()J (z) sin() J (z) cos()J (z) lim sin()

when is not an integer when is an integer.

The Wronskian of J (z) and Y (z) is W [J , Y ] = 2 . z

Thus J (z) and Y (z) are independent for all . The Bessel functions of the second kind satisfy the recursion relations,
1650

Y1 + Y+1 =

2 Y z

1 Y = (Y1 Y+1 ) 2

Y = Y Y+1 z Y = Y1 Y . z

34.6

Hankel Functions

Another set of solutions to Bessels equation is the Hankel functions,


(1) H (z) = J (z) + Y (z), (2) H (z) = J (z) Y (z)

Result 34.6.1 The Hankel functions are dened


(1) H (z) = J (z) + Y (z), (2) H (z) = J (z) Y (z)

The Wronskian of H (z) and H (z) is


(1) (2) W [H , H ] =

(1)

(2)

4 . z

The Hankel functions are independent for all . The Hankel functions satisfy the same recurrence relations as the other Bessel functions.

34.7

The Modied Bessel Equation


1 2 w + w 1+ 2 z z 1651

The modied Bessel equation is w = 0.

This equation is identical to the Bessel equation except for a sign change in the last term. If we make the change of variables = z, u() = w(z) we obtain the equation 1 2 u u 1 2 u = 0 2 1 u + u + 1 2 u = 0. This is the Bessel equation. Thus J (z) is a solution to the modied Bessel equation. This motivates us to dene the modied Bessel function of the rst kind I (z) = J (z). Since J and J are linearly independent solutions when is not an integer, I and I are linearly independent solutions to the modied Bessel equation when is not an integer. The Taylor series expansion of I (z) about z = 0 is I (z) = J (z) =

(1)m z m!( + m + 1) 2 m=0 (1)m 2m z m!( + m + 1) 2 m=0

+2m

+2m

1 z m!( + m + 1) 2 m=0

+2m

Modied Bessel Functions of the Second Kind. In order to have a second linearly independent solution when is an integer, we dene the modied Bessel function of the second kind K (z) =
I I 2 sin()

when is not an integer, when is an integer.

lim I I 2 sin() 1652

10 8 6 4 2

Figure 34.4: Modied Bessel Functions

I and K are linearly independent for all . In Figure 34.4 I0 and K0 are plotted in solid and dashed lines, respectively. 1653

Result 34.7.1 The modied Bessel functions of the rst and second kind, I (z) and K (z), are dened, I (z) = J (z). K (z) =
I I 2 sin()

when is not an integer,


I I 2 sin()

lim

when is an integer.

The modied Bessel function of the rst kind has the expansion, I (z) = z 1 m!( + m + 1) 2 m=0
+2m

The Wronskian of I (z) and I (z) is W [I , I ] = 2 sin(). z

I (z) and I (z) are linearly independent when is not an integer. The Wronskian of I (z) and K (z) is 1 W [I , K ] = . z I (z) and K (z) are independent for all . The modied Bessel functions satisfy the recursion relations, A1 A+1 = 2 A z A = A+1 + A z A = A1 A . z
1654

1 A = (A1 + A+1 ) 2 where A stands for either I or K.

34.8

Exercises
z 2 y (z) + zy (z) + z 2 2 y = 0

Exercise 34.1 Consider Bessels equation where 0. Find the Frobenius series solution that is asymptotic to t as t 0. By multiplying this solution by a constant, dene the solution (1)k z 2k+ . J (z) = k!(k + + 1) 2 k=1 This is called the Bessel function of the rst kind and order . Clearly J (z) is dened and is linearly independent to J (z) if is not an integer. What happens when is an integer? Exercise 34.2 Consider Bessels equation for integer n, z 2 y + zy + z 2 n2 y = 0. Using the kernel K(z, t) = e 2 z(t t ) , nd two solutions of Bessels equation. (For n = 0 you will nd only one solution.) Are the two solutions linearly independent? Dene the Bessel function of the rst kind and order n, Jn (z) = 1 2 tn1 e 2 z(t1/t) dt,
C
1 1 1

where C is a simple, closed contour about the origin. Verify that

e 2 z(t1/t) =
n=

Jn (z)tn .

This is the generating function for the Bessel functions. 1655

Exercise 34.3 Use the generating function

e to show that Jn satises Bessels equation

1 z(t1/t) 2

=
n=

Jn (z)tn

z 2 y + zy + z 2 n2 y = 0. Exercise 34.4 Using Jn1 + Jn+1 = show that 2n Jn z and Jn = n Jn Jn+1 , z

n 1 Jn = (Jn1 Jn+1 ) and Jn = Jn1 Jn . 2 z

Exercise 34.5 Find the general solution of 1 1 w + w + 1 2 z 4z Exercise 34.6 Show that J (z) and Y (z) are linearly independent for all . Exercise 34.7 Compute W [I , I ] and W [I , K ]. Exercise 34.8 Using the generating function, z exp 2 1 t t
+

w = z.

=
n=

Jn (z)tn ,

1656

verify the following identities: 1. 2n Jn (z) = Jn1 (z) + Jn+1 (z). z This relation is useful for recursively computing the values of the higher order Bessel functions. 1 (Jn1 Jn+1 ) . 2 This relation is useful for computing the derivatives of the Bessel functions once you have the values of the Bessel functions of adjacent order. Jn (z) = d n z Jn (z) = z n Jn+1 (z). dz

2.

3.

Exercise 34.9 Use the Wronskian of J (z) and J (z), W [J (z), J (z)] = to derive the identity J+1 (z)J (z) + J (z)J1 (z) = Exercise 34.10 Show that, using the generating function or otherwise, J0 (z) + 2J2 (z) + 2J4 (z) + 2J6 (z) + = 1 J0 (z) 2J2 (z) + 2J4 (z) 2J6 (z) + = cos z 2J1 (z) 2J3 (z) + 2J5 (z) = sin z 2 2 2 2 J0 (z) + 2J1 (z) + 2J2 (z) + 2J3 (z) + = 1 1657 2 sin . z 2 sin , z

Exercise 34.11 It is often possible to solve certain ordinary dierential equations by converting them into the Bessel equation by means of various transformations. For example, show that the solution of y + xp2 y = 0, can be written in terms of Bessel functions. y(x) = c1 x1/2 J1/p 2 p/2 x p + c2 x1/2 Y1/p 2 p/2 x p

Here c1 and c2 are arbitrary constants. Thus show that the Airy equation, y + xy = 0, can be solved in terms of Bessel functions. Exercise 34.12 The spherical Bessel functions are dened by Jn+1/2 (z), 2z Yn+1/2 (z), 2z Kn+1/2 (z), 2z In+1/2 (z). 2z 1658

jn (z) = yn (z) = kn (z) = in (z) =

Show that sin z cos z , z2 z sinh z i0 (z) = , z k0 (z) = exp(z). 2z j1 (z) = Exercise 34.13 Show that as x , ex Kn (x) x 1+ 4n2 1 (4n2 1)(4n2 9) + + 8x 128x2 .

1659

34.9
Hint 34.2 Hint 34.3

Hints

Hint 34.4 Use the generating function

e to show that Jn satises Bessels equation

1 z(t1/t) 2

=
n=

Jn (z)tn

z 2 y + zy + z 2 n2 y = 0. Hint 34.6 Use variation of parameters and the Wronskian that was derived in the text. Hint 34.7 Compute the Wronskian of J (z) and Y (z). Use the relation W [J , J ] = 2 sin() z

Hint 34.8 Derive W [I , I ] from the value of W [J , J ]. Derive W [I , K ] from the value of W [I , I ]. Hint 34.9 Hint 34.10

1660

Hint 34.11 Hint 34.12 Hint 34.13 Hint 34.14

1661

34.10

Solutions
L[y] z 2 y + zy + z 2 n2 y = 0.

Solution 34.1 Bessels equation is We consider a solution of the form y(z) =


C

e 2 z(t1/t) v(t) dt.

We substitute the form of the solution into Bessels equation. L e 2 z(t1/t) v(t) dt = 0
C
1

z2
C

1 4

t+

1 t

+z

1 2

1 t

+ z 2 n2

e 2 z(t1/t) v(t) dt = 0

(34.1)

By considering d 1 z(t1/t) t e2 = dt d2 2 1 z(t1/t) t e2 = dt2 we see that L e 2 z(t1/t) = Thus Equation 34.1 becomes
1 d2 2 1 z(t1/t) d 1 t e2 3 t e 2 z(t1/t) +(1 n2 ) e 2 z(t1/t) v(t) dt = 0 2 dt dt 1

1 1 x t+ 2 t + x 2t + 1 t

+ 1 e 2 z(t1/t) + 2 e 2 z(t1/t)
1

1 1 2 x t+ 4 t

d2 2 d t 3 t + 1 n2 2 dt dt

e 2 z(t1/t) .

1662

We apply integration by parts to move derivatives from the kernel to v(t). t2 e 2 z(t1/t) v(t)
C
1

t e 2 z(t1/t) v (t)
C

+ 3t e 2 z(t1/t) v(t)
C

+
C

e 2 z(t1/t) t2 v (t) + 3tv(t) + 1 n2 v(t) dt =

1 z(t1/t) 2

(t2 3t)v(t) tv (t)


C

+
C

1 z(t1/t) 2

t2 v (t) + 3tv(t) + (1 n2 )v(t) dt = 0

In order that the integral vanish, v(t) must be a solution of the dierential equation t2 v + 3tv + 1 n2 v = 0. This is an Euler equation with the solutions {tn1 , tn1 } for non-zero n and {t1 , t1 log t} for n = 0. Consider the case of non-zero n. Since e 2 z(t1/t)
1

t2 3t v(t) tv (t)

is single-valued and analytic for t = 0 for the functions v(t) = tn1 and v(t) = tn1 , the boundary term will vanish if C is any closed contour that that does not pass through the origin. Note that the integrand in our solution, e 2 z(t1/t) v(t), is analytic and single-valued except at the origin and innity where it has essential singularities. Consider a simple closed contour that does not enclose the origin. The integral along such a path would vanish and give us y(z) = 0. This is not an interesting solution. Since 1 e 2 z(t1/t) v(t), has non-zero residues for v(t) = tn1 and v(t) = tn1 , choosing any simple, positive, closed contour about the origin will give us a non-trivial solution of Bessels equation. These solutions are y1 (t) =
C
1

tn1 e 2 z(t1/t) dt,

y2 (t) =
C

tn1 e 2 z(t1/t) dt.

Now consider the case n = 0. The two solutions above concide and we have the solution y(t) =
C

t1 e 2 z(t1/t) dt.

1663

Choosing v(t) = t1 log t would make both the boundary terms and the integrand multi-valued. We do not pursue the possibility of a solution of this form. The solution y1 (t) and y2 (t) are not linearly independent. To demonstrate this we make the change of variables t 1/t in the integral representation of y1 (t). y1 (t) =
C

tn1 e 2 z(t1/t) dt (1/t)n1 e 2 z(1/t+t)


C
1 1

= =
C

1 dt t2

(1)n tn1 e 2 z(t1/t) dt

= (1)n y2 (t) Thus we see that a solution of Bessels equation for integer n is y(t) =
C

tn1 e 2 z(t1/t) dt

where C is any simple, closed contour about the origin. Therefore, the Bessel function of the rst kind and order n, Jn (z) = 1 2 tn1 e 2 z(t1/t) dt
C
1 1

is a solution of Bessels equation for integer n. Note that Jn (z) is the coecient of tn in the Laurent series of e 2 z(t1/t) . This establishes the generating function for the Bessel functions.

1 z(t1/t) 2

=
n=

Jn (z)tn

1664

Solution 34.2 The generating function is

z (t1/t) 2

=
n=

Jn (z)tn .

In order to show that Jn satises Bessels equation we seek to show that

z 2 Jn (z) + zJn (z) + (z 2 n2 )Jn (z) tn = 0.


n=

To get the appropriate terms in the sum we will dierentiate the generating function with respect to z and t. First we dierentiate it with respect to z. 1 2 1 4 1 t t t 1 t
2

z (t1/t) 2

=
n=

Jn (z)tn Jn (z)tn
n=

e 2 (t1/t) =

Now we dierentiate with respect to t and multiply by t get the n2 Jn term. z 2 z 2 z 2 z 2 1 1 2 t t 1 t e 1 1+ 2 t t+ 1 t


2

e 2 (t1/t) =
n=

nJn (z)tn1 nJn (z)tn


n=

e 2 (t1/t) = 1 t+ t t+ 1 t
2

z (t1/t) 2

z + 4

e
2

z (t1/t) 2

=
n=

n2 Jn (z)tn1 n2 Jn (z)tn
n=

e 2 (t1/t) +

z 4

e 2 (t1/t) =

1665

Now we can evaluate the desired sum.

z 2 Jn (z) + zJn (z) + z 2 n2 Jn (z) tn


n=

z2 4

1 t t

z + 2

1 t t

z +z 2
2

1 t t

z2 4

1 t+ t

e 2 (t1/t)

z 2 Jn (z) + zJn (z) + z 2 n2 Jn (z) tn = 0


n=

z 2 Jn (z) + zJn (z) + z 2 n2 Jn (z) = 0 Thus Jn satises Bessels equation. Solution 34.3 n Jn Jn+1 z 1 = (Jn1 + Jn+1 ) Jn+1 2 1 = (Jn1 Jn+1 ) 2

Jn =

Jn =

n Jn Jn+1 z n 2n = Jn Jn Jn1 z z n = Jn1 Jn z 1666

Solution 34.4 The linearly independent homogeneous solutions are J1/2 and J1/2 . The Wronskian is 2 2 sin(/2) = . z z

W [J1/2 , J1/2 ] =

Using variation of parameters, a particular solution is


z z J1/2 () J1/2 () d + J1/2 (z) d 2/ 2/ z z 2 J1/2 () d J1/2 (z) 2 J1/2 () d. 2

yp = J1/2 (z) = J1/2 (z) 2

Thus the general solution is


z z

y = c1 J1/2 (z) + c2 J1/2 (z) + J1/2 (z) 2

J1/2 () d J1/2 (z) 2


2

2 J1/2 () d.

We could substitute J1/2 (z) = 2 z


1/2

sin z

and J1/2 =

2 z

1/2

cos z

into the solution, but we cannot evaluate the integrals in terms of elementary functions. (You can write the solution in terms of Fresnel integrals.) 1667

Solution 34.5

W [J , Y ] =

J J cot() J csc() J J cot() J csc()

J J J J csc() J J J J 2 = csc() sin() z 2 = z = cot() Since the Wronskian does not vanish identically, the functions are independent for all values of . Solution 34.6 I (z) = J (z)

W [I , I ] = = =

I I I I J (z) J (z) J (z) J (z)

J (z) J (z) J (z) J (z) 2 sin() = z 2 = sin() z 1668

W [I , K ] =

I csc()(I I ) 2 I csc()(I I ) 2 I I I I = csc() I I I I 2 2 = csc() sin() 2 z 1 = z

Solution 34.7 1. We diferentiate the generating function with respect to t.

e z 2 1+
n

z (t1/t) 2

=
n=

Jn (z)tn

1 1+ 2 t 1 t2

z (t1/t) 2

Jn (z)ntn1
n=

Jn (z)tn =
n=

2 Jn (z)ntn1 z n= 2 = Jn (z)ntn1 z n= 2 Jn (z)ntn1 z n=


Jn (z)t +
n= n=

Jn (z)t

n2

Jn1 (z)tn1 +
n= n=

Jn+1 (z)tn1 =

2 Jn1 (z) + Jn+1 (z) = Jn (z)n z 2n Jn (z) = Jn1 (z) + Jn+1 (z) z 1669

2. We diferentiate the generating function with respect to z.

e 1 2 1 2 1 2 1 2

z (t1/t) 2

=
n=

Jn (z)tn

1 t t 1 t

e 2 (t1/t) =

Jn (z)tn
n=

Jn (z)tn =
n= n=

Jn (z)tn

Jn (z)t
n=

n+1

n=

Jn (z)t

n1

=
n=

Jn (z)tn Jn (z)tn
n=

Jn1 (z)t
n= n=

Jn+1 (z)t

1 (Jn1 (z) Jn+1 (z)) = Jn (z) 2 1 Jn (z) = (Jn1 Jn+1 ) 2 3. d n z Jn (z) = nz n1 Jn (z) + z n Jn (z) dz 1 2n 1 = z n Jn (z) + z n (Jn1 (z) Jn+1 (z)) 2 z 2 1 n 1 = z (Jn+1 (z) + Jn1 (z)) + z n (Jn1 (z) Jn+1 (z)) 2 2 d n z Jn (z) = z n Jn+1 (z) dz 1670

Solution 34.8 For this part we will use the identities J (z) = J (z) J+1 (z), z J (z) = J1 (z) J (z). z

2 sin() J (z) J (z) = J (z) J (z) z 2 sin() J (z) J (z) = J1 (z) z J z J (z) J+1 (z) z 2 sin() J (z) J (z) J (z) J (z) = J1 (z) J+1 (z) z J (z) J (z) z 2 sin() J+1 (z)J (z) J (z)J1 (z) = z 2 J+1 (z)J (z) + J (z)J1 (z) = sin z Solution 34.9 The generating function for the Bessel functions is

1 z(t1/t) 2

=
n=

Jn (z)tn .

(34.2)

1. We substitute t = 1 into the generating function.

Jn (z) = 1
n=

J0 (z) +
n=1

Jn (z) +
n=1

Jn (z) = 1

1671

We use the identity Jn = (1)n Jn .

J0 (z) +
n=1

(1 + (1)n ) Jn (z) = 1

J0 (z) + 2
n=2 even n

Jn (z) = 1

J0 (z) + 2
n=1

J2n (z) = 1

2. We substitute t = into the generating function.

Jn (z)n = ez
n=

J0 (z) +
n=1

Jn (z) +
n=1

Jn (z)n = ez (1)n Jn (z)()n = ez

J0 (z) +
n=1

Jn (z)n +
n=1

J0 (z) + 2
n=1

Jn (z)n = ez

(34.3)

Next we substitute t = into the generating function.

J0 (z) + 2
n=1

(1)n Jn (z)n = ez

(34.4)

1672

Dividing the sum of Equation 34.3 and Equation 34.4 by 2 gives us the desired identity.

J0 (z) +
n=1

(1 + (1)n ) Jn (z)n = cos z

J0 (z) + 2
n=2 even n

Jn (z)n = cos z

J0 (z) + 2

(1)n/2 Jn (z) = cos z


n=2 even n

J0 (z) + 2
n=1

(1)n J2n (z) = cos z

3. Dividing the dierence of Equation 34.3 and Equation 34.4 by 2 gives us the other identity.

n=1

(1 (1)n ) Jn (z)n = sin z

2
n=1 odd n

Jn (z)n1 = sin z

(1)(n1)/2 Jn (z) = sin z


n=1 odd n

2
n=0

(1)n J2n+1 (z) = sin z

1673

4. We substitute t for t in the generating function.

1 2 z(t1/t)

=
n=

Jn (z)(t)n .

(34.5)

We take the product of Equation 34.2 and Equation 34.5 to obtain the nal identity.

Jn (z)t
n=

n m=

Jm (z)(t)m

= e 2 z(t1/t) e 2 z(t1/t) = 1

Note that the coecients of all powers of t except t0 in the product of sums must vanish.

Jn (z)tn Jn (z)(t)n = 1
n= 2 Jn (z) = 1 n= 2 J0 (z) + 2 n=1 2 Jn (z) = 1

Solution 34.10 First we make the change of variables y(x) = x1/2 v(x). We compute the derivatives of y(x). 1 y = x1/2 v + x1/2 v, 2 1 y = x1/2 v + x1/2 v x3/2 v. 4 1674

We substitute these into the dierential equation for y. y + xp2 y = 0 1 x1/2 v + x1/2 v x3/2 v + xp3/2 v = 0 4 1 x2 v + xv + xp v=0 4
2 Then we make the change of variables v(x) = u(), = p xp/2 . We write the derivatives in terms of .

d d d p d d =x = xxp/21 = dx dx d d 2 d 2 d d p2 d d d p d p d p 2 d2 x2 2 + x =x x = = 2 2 + dx dx dx dx 2 d 2 d 4 d 4 d x We write the dierential equation for u(). p2 2 p2 p2 2 1 u + u + u=0 4 4 4 4 1 1 u + u + 1 2 2 u=0 p This is the Bessel equation of order 1/p. We can write the general solution for u in terms of Bessel functions of the rst kind if p = 1. Otherwise, we use a Bessel function of the second kind. u() = c1 J1/p () + c2 J1/p () for p = 0, 1 u() = c1 J1/p () + c2 Y1/p () for p = 0 1675

We write the solution in terms of y(x). y(x) = c1 xJ1/p 2 p/2 x p + c2 xJ1/p + c2 xY1/p 2 p/2 x p 2 p/2 x p for p = 0, 1 for p = 0

y(x) = c1 xJ1/p

2 p/2 x p

The Airy equation y + xy = 0 is the case p = 3. The general solution of the Airy equation is y(x) = c1 xJ1/3 2 3/2 x 3 + c2 xJ1/3 2 3/2 x . 3

Solution 34.11 Consider J1/2 (z). We start with the series expansion. z (1)m J1/2 (z) = m!(1/2 + m + 1) 2 m=0 Use the identity (n + 1/2) =
(1)(3)(2n1) . 2n 1/2+2m

= = =

(1)m 2m+1 m!(1)(3) (2m + 1) m=0

z 2

1/2+2m

(1)m 2m+1 (2)(4) (2m) (1)(3) (2m + 1) m=0 2 z


1/2

1 2

1/2+m

z 1/2+2m

(1)m 2m+1 z (2m + 1)! m=0 1676

We recognize the sum as the Taylor series expansion of sin z. = Using the recurrence relations, J+1 = J J z and J1 = J + J , z 2 z
1/2

sin z

we can nd Jn+1/2 for any integer n. We need J3/2 (z) to determine j1 (z). To nd J3/2 (z), J3/2 (z) = 1/2 J1/2 (z) J1/2 (z) z 1/2 1/2 2 1 = z 1/2 sin z z 2 2 2
1/2

1/2

3/2

sin z

1/2

z 1/2 cos z

= 21/2 1/2 z 3/2 sin z + 21/2 1/2 z 3/2 sin z 21/2 1/2 cos z = = z 3/2 sin z
1/2

1/2

z 1/2 cos z

z 3/2 sin z z 1/2 cos z .

The spherical Bessel function j1 (z) is j1 (z) = The modied Bessel function of the rst kind is I (z) = J (z). 1677 sin z cos z . z2 z

We can determine I1/2 (z) from J1/2 (z). I1/2 (z) = 1/2 = = The spherical Bessel function i0 (z) is i0 (z) = The modied Bessel function of the second kind is K (z) = lim

2 sin(z) z 2 sinh(z) z

2 sinh(z) z sinh z . z I I 2 sin()

Thus K1/2 (z) can be determined in terms of I1/2 (z) and I1/2 (z). I1/2 I1/2 K1/2 (z) = 2 We determine I1/2 with the recursion relation I1 (z) = I (z) + I (z). z I1/2 (z) = I1/2 (z) + = = 1 I1/2 (z) 2z 2 3/2 1 z sinh(z) + 2z 2 1/2 z sinh(z)

2 1/2 1 z cosh(z) 2 2 cosh(z) z

1678

Now we can determine K1/2 (z). K1/2 (z) = = The spherical Bessel function k0 (z) is k0 (z) = z e . 2z 2 2 cosh(z) z z e 2z 2 sinh(z) z

Solution 34.12 The Point at Innity. With the change of variables z = 1/, w(z) = u() the modied Bessel equation becomes 1 n2 w + w 1+ 2 w =0 z z 4 3 2 u + 2 u + u 1 + n2 2 u = 0 1 1 n2 u + u 2 u = 0. 4 The point = 0 and hence the point z = is an irregular singular point. We will nd the leading order asymptotic behavior of the solutions as z +. Controlling Factor. Starting with the modied Bessel equation for real argument 1 n2 y + y 1+ 2 x x we make the substitution y = es(x) to obtain n2 1 s + (s )2 + s 1 2 = 0. x x 1679 y = 0,

We know that

n2 x2

1 as x ; we will assume that s 1 (s )2 + s 1 0 x

(s )2 as x . This gives us as x .

To simplify the equation further, we will try the possible two-term balances.
1 1 1. (s )2 + x s 0 s x than the other terms.

This balance is not consistent as it violates the assumption that 1 is smaller This balance is consistent. This balance is inconsistent as (s )2 isnt smaller than the other terms.

2. (s )2 1 0 3.
1 s x

s 1 s x

10

Thus the only dominant balance is s 1. This balance is consistent with our initial assumption that s (s )2 . x Thus s x and the controlling factor is e . We are interested in the decaying solution, so we will work with the controlling factor ex . Leading Order Behavior. In order to nd the leading order behavior, we substitute s = x+t(x) where t(x) x as x into the dierential equation for s. We assume that t 1 and t 1/x. 1 n2 t + (1 + t )2 + (1 + t ) 1 2 = 0 x x 2 1 1 n t 2t + (t )2 + t 2 = 0 x x x Using our assumptions about the behavior of t and t , 2t 1 0 x 1 t 2x

1 t ln x as x . 2 1680

This asymptotic behavior is consistent with our assumptions. Thus the leading order behavior of the decaying solution is y c ex 2 ln x+u(x) = cx1/2 ex+u(x)
1

as x ,

where u(x) ln x as x . 1 By substituting t = 2 ln x+u(x) into the dierential equation for t, you could show that u(x) const as x . Thus the full leading order behavior of the decaying solution is y cx1/2 ex as x

where u(x) 0 as x . It turns out that the asymptotic behavior of the modied Bessel function of the second kind is x e Kn (x) as x 2x Asymptotic Series. Now we nd the full asymptotic series for Kn (x) as x . We substitute Kn (x) ex x e w(x)Kn (x) 2x x

into the modied Bessel equation, where w(x) is a Taylor series about x = , i.e., Kn (x) First we dierentiate the expression for Kn (x). Kn (x) Kn (x) x 1 e w 1+ w 2x 2x x 1 1 3 e w 2+ w + 1+ + 2 2x x x 4x 1681 x e ak xk , 2x k=0

a0 = 1.

We substitute these expressions into the modied Bessel equation. x2 y + xy x2 + n2 y = 0 3 1 x2 w 2x2 + x w + x2 + x + w + xw x + 4 2 1 n2 w = 0 x2 w 2x2 w + 4 We compute the derivatives of the Taylor series.

w x2 + n2 w = 0

w =
k=1

(k)ak xk1 (k 1)ak+1 xk2


k=0

= w =
k=1

(k)(k 1)ak xk2 (k)(k 1)ak xk2


k=0

We substitute these expression into the dierential equation.


x2

k(k + 1)ak xk2 + 2x2


k=0 k=0

(k + 1)ak+1 xk2 + (k + 1)ak+1 xk +

1 n2 4

ak xk = 0
k=0

k(k + 1)ak xk + 2
k=0 k=0

1 n2 4

ak xk = 0
k=0

1682

We equate coecients of x to obtain a recurrence relation for the coecients. k(k + 1)ak + 2(k + 1)ak+1 + 1 n 2 ak = 0 4 n2 1/4 k(k + 1) ak+1 = ak 2(k + 1) n2 (k + 1/2)2 ak+1 = ak 2(k + 1) 4n2 (2k + 1)2 ak+1 = ak 8(k + 1)
k j=1

We set a0 = 1. We use the recurrence relation to determine the rest of the coecients. (4n2 (2j 1)2 ) 8k k! Now we have the asymptotic expansion of the modied Bessel function of the second kind. ak = Kn (x) x e 2x k=0
k j=1

(4n2 (2j 1)2 ) k x , 8k k!

as x

Convergence. We determine the domain of convergence of the series with the ratio test. The Taylor series about innity will converge outside of some circle. ak+1 (x) <1 k ak (x) ak+1 xk1 <1 lim k ak xk 4n2 (2k + 1)2 lim |x|1 < 1 k 8(k + 1) < |x| lim 1683

The series does not converge for any x in the nite complex plane. However, if we take only a nite number of terms in the series, it gives a good approximation of Kn (x) for large, positive x. At x = 10, the one, two and three term approximations give relative errors of 0.01, 0.0006 and 0.00006, respectively.

1684

Part V Partial Dierential Equations

1685

Chapter 35 Transforming Equations


Im about two beers away from ne. Let {xi } denote rectangular coordinates. Let {ai } be unit basis vectors in the orthogonal coordinate system {i }. The distance metric coecients hi can be dened hi = The gradient, divergence, etc., follow. u= 1 h1 h2 h3 1 2 u= h1 h2 h3 1 v = a2 u a3 u a1 u + + h1 1 h2 2 h3 3 (h2 h3 v1 ) + (h3 h1 v2 ) + (h1 h2 v3 ) 1 2 3 h2 h3 u h3 h1 u h1 h2 u + + h1 1 2 h2 2 3 h3 3 x1 i
2

x2 i

x3 i

1686

35.1

Exercises

Exercise 35.1 Find the Laplacian in cylindrical coordinates (r, , z). x = r cos , Hint, Solution Exercise 35.2 Find the Laplacian in spherical coordinates (r, , ). x = r sin cos , Hint, Solution y = r sin sin , z = r cos y = r sin , z

1687

35.2
Hint 35.1 Hint 35.2

Hints

1688

35.3

Solutions

Solution 35.1

h1 = h2 = h3 = 1 r

(cos )2 + (sin )2 + 0 = 1 (r sin )2 + (r cos )2 + 0 = r 0 + 0 + 12 = 1

u=

u 1 u u + + r r r z z 2 2 1 u 1 u u 2 u= r + 2 2 + 2 r r r r z r

Solution 35.2

h1 = h2 = h3 =

(sin cos )2 + (sin sin )2 + (cos )2 = 1 (r cos cos )2 + (r cos sin )2 + (r sin )2 = r (r sin sin )2 + (r sin cos )2 + 0 = r sin

u=

1 u u 1 u r2 sin + sin + sin r r sin u 1 u 1 2u 1 2 u= 2 r2 + 2 sin + 2 r r r r sin r sin 2 r2

1689

Chapter 36 Classication of Partial Dierential Equations


36.1 Classication of Second Order Quasi-Linear Equations

Consider the general second order quasi-linear partial dierential equation in two variables. a(x, y)uxx + 2b(x, y)uxy + c(x, y)uyy = F (x, y, u, ux , uy ) (36.1)

We classify the equation by the sign of the discriminant. At a given point x0 , y0 , the equation is classied as one of the following types: b2 ac > 0 : hyperbolic 2 b ac = 0 : parabolic 2 b ac < 0 : elliptic If an equation has a particular type for all points x, y in a domain then the equation is said to be of that type in the domain. Each of these types has a canonical form that can be obtained through a change of independent variables. The type of an equation indicates much about the nature of its solution. We seek a change of independent variables, (a dierent coordinate system), such that Equation 36.1 has a simpler form. We will nd that a second order quasi-linear partial dierential equation in two variables can be transformed to 1690

one of the canonical forms: u = G(, , u, u , u ), hyperbolic u = G(, , u, u , u ), parabolic u + u = G(, , u, u , u ), elliptic Consider the change of independent variables = (x, y), We calculate the partial derivatives of u. ux uy uxx uxy uyy = x u + x u = y u + y u 2 2 = x u + 2x x u + x u + xx u + xx u = x y u + (x y + y x )u + x y u + xy u + xy u 2 2 = y u + 2y y u + y u + yy u + yy u = (x, y).

Substituting these into Equation 36.1 yields an equation in and .


2 2 ax + 2bx y + cy u + 2 (ax x + b(x y + y x ) + cy y ) u 2 2 + ax + 2bx y + cy u = H(, , u, u , u )

(, )u + (, )u + (, )u = H(, , u, u , u )

(36.2)

36.1.1

Hyperbolic Equations

We start with a hyperbolic equation, (b2 ac > 0). We seek a change of independent variables that will put Equation 36.1 in the form u = G(, , u, u , u ) (36.3) 1691

We require that the u and u terms vanish. That is = = 0 in Equation 36.2. This gives us two constraints on and .
2 2 ax + 2bx y + cy = 0, x b + b2 ac = , y a b b2 ac x + y = 0, a 2 2 ax + 2bx y + cy = 0 x b b2 ac = y a b + b2 ac x + y = 0 a

(36.4)

Here we chose the signs in the quadratic formulas to get dierent solutions for and . Now we have rst order quasi-linear partial dierential equations for the coordinates and . We solve these equations with the method of characteristics. The characteristic equations for are dy b b2 ac d , (x, y(x)) = 0 = a dx dx Solving the dierential equation for y(x) determines (x, y). We just write the solution for y(x) in the form F (x, y(x)) = const. Since the solution of the dierential equation for is (x, y(x)) = const, we then have = F (x, y). Upon solving for and we divide Equation 36.2 by (, ) to obtain the canonical form. Note that we could have solved for y /x in Equation 36.4. dx y b = = dy x This form is useful if a vanishes. Another canonical form for hyperbolic equations is u u = K(, , u, u , u ). 1692 (36.5) b2 ac c

We can transform Equation 36.3 to this form with the change of variables = + , Equation 36.3 becomes u u = G + , , u, u + u , u u 2 2 . = .

Example 36.1.1 Consider the wave equation with a source. utt c2 uxx = s(x, t) Since 0 (1)(c2 ) > 0, the equation is hyperbolic. We nd the new variables. dx = c, dt dx = c, dt Then we determine t and x in terms of and . t= We calculate the derivatives of and . t = c x = 1 t = c x = 1 Then we calculate the derivatives of u. utt = c2 u 2c2 u + c2 u uxx = u + u 1693 , 2c x= + 2 x = ct + const, x = ct + const, = x + ct = x ct

Finally we transform the equation to canonical form. 2c2 u = s u = 1 s 2c2 + , 2 2c + , 2 2c

If s(x, t) = 0, then the equation is u = 0 we can integrate with respect to and to obtain the solution, u = f () + g(). Here f and g are arbitrary C 2 functions. In terms of t and x, we have u(x, t) = f (x + ct) + g(x ct). To put the wave equation in the form of Equation 36.5 we make a change of variables = + = 2x, = = 2ct utt c2 uxx = s(x, t) 4c2 u 4c2 u = s , 2 2c 1 u u = 2 s , 4c 2 2c Example 36.1.2 Consider y 2 uxx x2 uyy = 0. For x = 0 and y = 0 this equation is hyperbolic. We nd the new variables. y 2 x2 dy x = = , 2 dx y y 2 x2 y dy x = = , dx y2 y y dy = x dx, y dy = x dx, y2 x2 = + const, = y 2 + x2 2 2 2 2 y x = + const, = y 2 x2 2 2

1694

We calculate the derivatives of and . x = 2x y = 2y x = 2x y = 2y Then we calculate the derivatives of u. ux = 2x(u u ) uy = 2y(u + u ) 2 = 4x (u 2u + u ) + 2(u u ) = 4y 2 (u + 2u + u ) + 2(u + u )

uxx uyy

Finally we transform the equation to canonical form. y 2 uxx x2 uyy = 0 8x2 y 2 u 8x2 y 2 u + 2y 2 (u u ) + 2x2 (u + u ) = 0 1 1 16 ( ) ( + )u = 2u 2u 2 2 u u u = 2( 2 2 ) Example 36.1.3 Consider Laplaces equation. uxx + uyy = 0 Since 0 (1)(1) < 0, the equation is elliptic. We will transform this equation to the canical form of Equation 36.3. We nd the new variables. dy = , dx dy = , dx y = x + const, y = x + const, = x + y = x y

1695

We calculate the derivatives of and . x = 1 y = x = 1 y = Then we calculate the derivatives of u. uxx = u + 2u + u uyy = u + 2u u Finally we transform the equation to canonical form. 4u = 0 u = 0 We integrate with respect to and to obtain the solution, u = f () + g(). Here f and g are arbitrary C 2 functions. In terms of x and y, we have u(x, y) = f (x + y) + g(x y). This solution makes a lot of sense, because the real and imaginary parts of an analytic function are harmonic.

36.1.2

Parabolic equations

Now we consider a parabolic equation, (b2 ac = 0). We seek a change of independent variables that will put Equation 36.1 in the form u = G(, , u, u , u ). (36.6) We require that the u and u terms vanish. That is = = 0 in Equation 36.2. This gives us two constraints on and . 2 2 ax x + b(x y + y x ) + cy y = 0, ax + 2bx y + cy = 0 1696

We consider the case a = 0. The latter constraint allows us to solve for x /y . x b b2 ac b = = y a a With this information, the former constraint is trivial. ax x + b(x y + y x ) + cy y = 0 ax (b/a) + b(x + y (b/a)) + cy = 0 (ac b2 )y = 0 0=0 Thus we have a rst order partial dierential equation for the coordinate which we can solve with the method of characteristics. b x + y = 0 a The coordinate is chosen to be anything linearly independent of . The characteristic equations for are dy b d = , (x, y(x)) = 0 dx a dx Solving the dierential equation for y(x) determines (x, y). We just write the solution for y(x) in the form F (x, y(x)) = const. Since the solution of the dierential equation for is (x, y(x)) = const, we then have = F (x, y). Upon solving for and choosing a linearly independent , we divide Equation 36.2 by (, ) to obtain the canonical form. In the case that a = 0, we would instead have the constraint, b x + y = 0. c

36.1.3

Elliptic Equations

We start with an elliptic equation, (b2 ac < 0). We seek a change of independent variables that will put Equation 36.1 in the form u + u = G(, , u, u , u ) (36.7) 1697

If we make the change of variables determined by b + ac b2 x = , y a the equation will have the form

x b ac b2 = , y a

u = G(, , u, u , u ). and are complex-valued. If we then make the change of variables = + , 2 = 2

we will obtain the canonical form of Equation 36.7. Note that since and are complex conjugates, and are real-valued. Example 36.1.4 Consider y 2 uxx + x2 uyy = 0. (36.8) For x = 0 and y = 0 this equation is elliptic. We nd new variables that will put this equation in the form u = G(). From Example 36.1.2 we see that they are y 2 x2 dy x = = , 2 dx y y 2 x2 y dy x = = , 2 dx y y y dy = x dx, y dy = x dx, y2 x2 = + const, = y 2 + x2 2 2 2 2 y x = + const, = y 2 x2 2 2

The variables that will put Equation 36.8 in canonical form are = + = y2, 2 = = x2 2

1698

We calculate the derivatives of and . x = 0 y = 2y x = 2x y = 0 Then we calculate the derivatives of u. ux = 2xu uy = 2yu uxx = 4x2 u + 2u uyy = 4y 2 u + 2u Finally we transform the equation to canonical form. y 2 uxx + x2 uyy = 0 + 2u ) + (4u + 2u ) = 0 1 1 u u 2 2

(4 u

u + u =

36.2

Equilibrium Solutions
ut = uxx , u(x, 0) = x, ux (0, t) = ux (1, t) = 0

Example 36.2.1 Consider the equilibrium solution for the following problem.

Setting ut = 0 we have an ordinary dierential equation. d2 u =0 dx2 1699

This equation has the solution, u = ax + b. Applying the boundary conditions we see that u = b. To determine the constant, we note that the heat energy in the rod is constant in time.
1 1

u(x, t) dx =
0 1 0 1

u(x, 0) dx x dx
0

b dx =
0

Thus the equilibrium solution is 1 u(x) = . 2

1700

36.3

Exercises

Exercise 36.1 Classify and transform the following equation into canonical form. uxx + (1 + y)2 uyy = 0 Hint, Solution Exercise 36.2 Classify as hyperbolic, parabolic, or elliptic in a region R each of the equations: 1. ut = (pux )x 2. utt = c2 uxx u 3. (qux )x + (qut )t = 0 where p(x), c(x, t), q(x, t), and (x) are given functions that take on only positive values in a region R of the (x, t) plane. Hint, Solution Exercise 36.3 Transform each of the following equations for (x, y) into canonical form in appropriate regions 1. xx y 2 yy + x + x2 = 0 2. xx + xyy = 0 The equation in part (b) is known as Tricomis equation and is a model for transonic uid ow in which the ow speed changes from supersonic to subsonic. Hint, Solution

1701

36.4
Hint 36.1 Hint 36.2 Hint 36.3

Hints

1702

36.5

Solutions

Solution 36.1 For y = 1, the equation is parabolic. For this case it is already in the canonical form, uxx = 0. For y = 1, the equation is elliptic. We nd new variables that will put the equation in the form u = G(, , u, u , u ). dy = (1 + y)2 = (1 + y) dx dy = dx 1+y log(1 + y) = x + c 1 + y = c ex (1 + y) ex = c = (1 + y) ex = = (1 + y) ex The variables that will put the equation in canonical form are + = (1 + y) cos x, = = (1 + y) sin x. = 2 2 We calculate the derivatives of and . x = (1 + y) sin x y = cos x x = (1 + y) cos x y = sin x Then we calculate the derivatives of u. ux = (1 + y) sin(x)u + (1 + y) cos(x)u uy = cos(x)u + sin(x)u 2 2 = (1 + y) sin (x)u + (1 + y)2 cos2 (x)u (1 + y) cos(x)u (1 + y) sin(x)u uyy = cos2 (x)u + sin2 (x)u 1703

uxx

We substitute these results into the dierential equation to obtain the canonical form. uxx + (1 + y)2 uyy = 0 (1 + y)2 (u + u ) (1 + y) cos(x)u (1 + y) sin(x)u = 0 2 + 2 (u + u ) u u = 0 u + u = Solution 36.2 1. ut = (pux )x puxx + 0uxt + 0utt + px ux ut = 0 Since 02 p0 = 0, the equation is parabolic. 2. utt = c2 uxx u utt + 0utx c2 uxx + u = 0 Since 02 (1)(c2 ) > 0, the equation is hyperbolic. 3. (qux )x + (qut )t = 0 quxx + 0uxt + qutt + qx ux + qt ut = 0 Since 02 qq < 0, the equation is elliptic. 1704 u + u 2 + 2

Solution 36.3 1. For y = 0, the equation is hyperbolic. We nd the new independent variables. dy = dx dy = dx y2 = y, y = c ex , ex y = c, = ex y 1 y2 = y, y = c ex , ex y = c, = ex y 1

Next we determine x and y in terms of and . = y 2 , = ex We calculate the derivatives of and . x = ex y = y = ex = / x = ex y = y = ex = Then we calculate the derivatives of . = + , x x = + , = y y = + + yy = + 2 + / , ex = y= /, x= 1 log 2

xx = 2 2 + 2 + + ,

1705

Finally we transform the equation to canonical form. xx y 2 yy + x + x2 = 0 4 + + + + log = 1 + log 2 =0

For y = 0 we have the ordinary dierential equation xx + x + x2 = 0. 2. For x < 0, the equation is hyperbolic. We nd the new independent variables. dy = x, dx dy = x, dx 2 2 y = x x + c, = x x y 3 3 2 2 y = x x + c, = x x + y 3 3

Next we determine x and y in terms of and . x= We calculate the derivatives of and . x = x = 3 ( + ) 4


1/6 1/6

3 ( + ) 4

1/3

y=

, ,

y = 1

x =

3 ( + ) 4 1706

y = 1

Then we calculate the derivatives of . x = 3 ( + ) ( + ) 4 y = +


1/6

xx =

3 ( + ) 4

1/3

( + ) + (6( + ))1/3 + (6( + ))2/3 ( + ) yy = 2 +

Finally we transform the equation to canonical form. xx + xyy = 0 (6( + ))


1/3

+ (6( + ))1/3 + (6( + ))2/3 ( + ) = 0 = + 12( + )

For x > 0, the equation is elliptic. The variables we dened before are complex-valued. 2 = x3/2 y, 3 We choose the new real-valued variables. = , We write the derivatives in terms of and . = = = 1707 = ( + ) 2 = x3/2 + y 3

We transform the equation to canonical form. + 12( + ) 2 = 12 = + = 6

1708

Chapter 37 Separation of Variables


37.1 37.2 Eigensolutions of Homogeneous Equations Homogeneous Equations with Homogeneous Boundary Conditions

The method of separation of variables is a useful technique for nding special solutions of partial dierential equations. We can combine these special solutions to solve certain problems. Consider the temperature of a one-dimensional rod of length h 1 . The left end is held at zero temperature, the right end is insulated and the initial temperature distribution is known at time t = 0. To nd the temperature we solve the problem: u 2u = 2, 0 < x < h, t > 0 t x u(0, t) = ux (h, t) = 0 u(x, 0) = f (x)
1

Why h? Because l looks like 1 and we use L to denote linear operators

1709

We look for special solutions of the form, u(x, t) = X(x)T (t). Substituting this into the partial dierential equation yields X(x)T (t) = X (x)T (t) T (t) X (x) = T (t) X(x) Since the left side is only dependent on t, the right side in only dependent on x, and the relation is valid for all t and x, both sides of the equation must be constant. T X = = T X Here is an arbitrary constant. (Youll see later that this form is convenient.) u(x, t) = X(x)T (t) will satisfy the partial dierential equation if X(x) and T (t) satisfy the ordinary dierential equations, T = T and X = X.

Now we see how lucky we are that this problem happens to have homogeneous boundary conditions 2 . If the left boundary condition had been u(0, t) = 1, this would imply X(0)T (t) = 1 which tells us nothing very useful about either X or T . However the boundary condition u(0, t) = X(0)T (t) = 0, tells us that either X(0) = 0 or T (t) = 0. Since the latter case would give us the trivial solution, we must have X(0) = 0. Likewise by looking at the right boundary condition we obtain X (h) = 0. We have a regular Sturm-Liouville problem for X(x). X + X = 0, The eigenvalues and orthonormal eigenfunctions are n =
2

X(0) = X (h) = 0

(2n 1) 2h

Xn =

2 sin h

(2n 1) x , 2h

n Z+ .

Actually luck has nothing to do with it. I planned it that way.

1710

Now we solve the equation for T (t). T = n T T = c en t The eigen-solutions of the partial dierential equation that satisfy the homogeneous boundary conditions are 2 sin n x en t . h We seek a solution of the problem that is a linear combination of these eigen-solutions. un (x, t) =

u(x, t) =
n=1

an

2 sin h

n x en t

We apply the initial condition to nd the coecients in the expansion.

u(x, 0) =
n=1

an 2 h
h

2 sin h sin

n x = f (x)

an =

n x f (x) dx

37.3

Time-Independent Sources and Boundary Conditions

Consider the temperature in a one-dimensional rod of length h. The ends are held at temperatures and , respectively, and the initial temperature is known at time t = 0. Additionally, there is a heat source, s(x), that is independent of time. We nd the temperature by solving the problem, ut = uxx + s(x), u(0, t) = , u(h, t) = , u(x, 0) = f (x). (37.1)

Because of the source term, the equation is not separable, so we cannot directly apply separation of variables. Furthermore, we have the added complication of inhomogeneous boundary conditions. Instead of attacking this problem directly, we seek a transformation that will yield a homogeneous equation and homogeneous boundary conditions. 1711

Consider the equilibrium temperature, (x). It satises the problem, (x) = The Green function for this problem is, G(x; ) = The equilibrium temperature distribution is x 1 xh (x) = + h h h 1 x (x) = + ( ) h h
x h

s(x) = 0,

(0) = ,

(h) = .

x< (x> h) . h

x< (x> h)s() d,


0 h

(x h)
0

s() d + x
x

( h)s() d .

Now we substitute u(x, t) = v(x, t) + (x) into Equation 37.1. 2 (v + (x)) = 2 (v + (x)) + s(x) t x vt = vxx + (x) + s(x) vt = vxx

(37.2)

Since the equilibrium solution satises the inhomogeneous boundary conditions, v(x, t) satises homogeneous boundary conditions. v(0, t) = v(h, t) = 0. The initial value of v is v(x, 0) = f (x) (x). 1712

We seek a solution for v(x, t) that is a linear combination of eigen-solutions of the heat equation. We substitute the separation of variables, v(x, t) = X(x)T (t) into Equation 37.2 T X = = T X This gives us two ordinary dierential equations. X + X = 0, X(0) = X(h) = 0 T = T. The Sturm-Liouville problem for X(x) has the eigenvalues and orthonormal eigenfunctions, n = We solve for T (t). Tn = c e(n/h) t . The eigen-solutions of the partial dierential equation are vn (x, t) = 2 nx (n/h)2 t e sin . h h
2

n h

Xn =

nx 2 sin , h h

n Z+ .

The solution for v(x, t) is a linear combination of these.

v(x, t) =
n=1

an

2 nx (n/h)2 t e sin h h

We determine the coecients in the series with the initial condition.

v(x, 0) =
n=1

an 2 h
h

2 nx sin = f (x) (x) h h sin nx (f (x) (x)) dx h

an =

1713

The temperature of the rod is

u(x, t) = (x) +
n=1

an

2 nx (n/h)2 t e sin h h

37.4

Inhomogeneous Equations with Homogeneous Boundary Conditions


ut = uxx + s(x, t), u(0, t) = u(h, t) = 0, u(x, 0) = f (x). (37.3)

Now consider the heat equation with a time dependent source, s(x, t).

In general we cannot transform the problem to one with a homogeneous dierential equation. Thus we cannot represent the solution in a series of the eigen-solutions of the partial dierential equation. Instead, we will do the next best thing and expand the solution in a series of eigenfunctions in Xn (x) where the coecients depend on time.

u(x, t) =
n=1

un (t)Xn (x)

We will nd these eigenfunctions with the separation of variables, u(x, t) = X(x)T (t) applied to the homogeneous equation, ut = uxx , which yields, 2 nx Xn (x) = sin , n Z+ . h h We expand the heat source in the eigenfunctions.

s(x, t) =
n=1

sn (t)
h

2 nx sin h h nx s(x, t) dx, h

sn (t) =

2 h

sin
0

1714

We substitute the series solution into Equation 37.3.

un (t)
n=1

2 nx n sin = un (t) h h h n=1 un (t) +

2 nx sin + sn (t) h h n=1

2 nx sin h h

n 2 un (t) = sn (t) h Now we have a rst order, ordinary dierential equation for each of the un (t). We obtain initial conditions from the initial condition for u(x, t).

u(x, 0) =
n=1

un (0) 2 h
h

nx 2 sin = f (x) h h nx f (x) dx fn h

un (0) = The temperature is given by

sin
0

u(x, t) =
n=1
2

un (t)
t

2 nx sin , h h e(n/h)
2 (t )

un (t) = fn e(n/h) t +
0

sn ( ) d.

37.5

Inhomogeneous Boundary Conditions

Consider the temperature of a one-dimensional rod of length h. The left end is held at the temperature (t), the heat ow at right end is specied, there is a time-dependent source and the initial temperature distribution is known at time t = 0. To nd the temperature we solve the problem: ut = uxx + s(x, t), 0 < x < h, t > 0 u(0, t) = (t), ux (h, t) = (t) u(x, 0) = f (x) 1715 (37.4)

Transformation to a homogeneous equation. Because of the inhomogeneous boundary conditions, we cannot directly apply the method of separation of variables. However we can transform the problem to an inhomogeneous equation with homogeneous boundary conditions. To do this, we rst nd a function, (x, t) which satises the boundary conditions. We note that (x, t) = (t) + x(t) does the trick. We make the change of variables u(x, t) = v(x, t) + (x, t) in Equation 37.4. vt + t = (vxx + xx ) + s(x, t) vt = vxx + s(x, t) t The boundary and initial conditions become v(0, t) = 0, vx (h, t) = 0, v(x, 0) = f (x) (x, 0).

Thus we have a heat equation with the source s(x, t) t (x, t). We could apply separation of variables to nd a solution of the form 2 (2n 1)x u(x, t) = (x, t) + un (t) sin . h 2h n=1 Direct eigenfunction expansion. Alternatively we could seek a direct eigenfunction expansion of u(x, t).

u(x, t) =
n=1

un (t)

2 sin h

(2n 1)x 2h

Note that the eigenfunctions satisfy the homogeneous boundary conditions while u(x, t) does not. If we choose any xed time t = t0 and form the periodic extension of the function u(x, t0 ) to dene it for x outside the range (0, h), then 1716

this function will have jump discontinuities. This means that our eigenfunction expansion will not converge uniformly. We are not allowed to dierentiate the series with respect to x. We cant just plug the series into the partial dierential equation to determine the coecients. Instead, we will multiply Equation 37.4, by an eigenfunction and integrate from x = 0 to x = h. To avoid dierentiating the series with respect to x, we will use integration by parts to move derivatives from u(x, t) to the eigenfunction. (We will denote n =
(2n1) 2h 2

.)

2 h un (t) un (t)

sin(
0

n x)(ut uxx ) dx =
h

2 h
0 h

sin(
0 h

n x)s(x, t) dx

2 ux sin( h

n x)

+
0

2 n h
0

ux cos( n x) dx = sn (t) + 2 n h
h

2 (1)n ux (h, t) + h un (t)

2 n u cos( n x) h

u sin(
0

n x) dx = sn (t)

2 2 (1)n (t) n u(0, t) + n un (t) = sn (t) h h 2 un (t) + n un (t) = n (t) + (1)n (t) + sn (t) h

Now we have an ordinary dierential equation for each of the un (t). We obtain initial conditions for them using the initial condition for u(x, t).

u(x, 0) =
n=1

un (0) 2 h
h

2 sin( n x) = f (x) h n x)f (x) dx fn

un (0) =

sin(
0

1717

Thus the temperature is given by u(x, t) = 2 h


t

2 h

un (t) sin( n x),


n=1

un (t) = fn en t +

en (t )
0

n ( ) + (1)n ( ) d.

37.6

The Wave Equation

Consider an elastic string with a free end at x = 0 and attached to a massless spring at x = 1. The partial dierential equation that models this problem is utt = uxx ux (1, t) = u(1, t), u(x, 0) = f (x),

ux (0, t) = 0,

ut (x, 0) = g(x).

We make the substitution u(x, t) = (x)(t) to obtain = = . First we consider the problem for . + = 0, (0) = (1) + (1) = 0.

To nd the eigenvalues we consider the following three cases: < 0. The general solution is = a cosh( x) + b sinh( x). 1718

(0) = 0 (1) + (1) = 0

b = 0.

a cosh( ) + a sinh( ) = 0 a = 0.

Since there is only the trivial solution, there are no negative eigenvalues. = 0. The general solution is = ax + b. (0) = 0 (1) + (1) = 0 Thus = 0 is not an eigenvalue. > 0. The general solution is = a cos( x) + b sin( x). (0) (1) + (1) = 0 b = 0. a cos( ) a sin( ) = 0 cos( ) = sin( ) = cot( ) a = 0. b + 0 = 0.

By looking at Figure 37.1, (the plot shows the functions f (x) = x, f (x) = cot x and has lines at x = n), we see that there are an innite number of positive eigenvalues and that n (n)2 as n . The eigenfunctions are n = cos( n x).

1719

10 8 6 4 2

2 -2

10

Figure 37.1: Plot of x and cot x. The solution for is n = an cos( n t) + bn sin( Thus the solution to the dierential equation is

n t).

u(x, t) =
n=1

cos( n x)[an cos( n t) + bn sin(

n t)].

Let

f (x) =
n=1

fn cos( n x) gn cos( n x).


n=1

g(x) =

1720

From the initial value we have


cos( n x)an =
n=1 n=1

fn cos( n x)

an = fn . The initial velocity condition gives us


cos( n x) n bn =
n=1 n=1

gn cos( n x)

gn bn = . n Thus the solution is u(x, t) = gn cos( n x) fn cos( n t) + sin( n n=1

n t) .

37.7

General Method

Here is an outline detailing the method of separation of variables for a linear partial dierential equation for u(x, y, z, . . .). 1. Substitute u(x, y, z, . . .) = X(x)Y (y)Z(z) into the partial dierential equation. Separate the equation into ordinary dierential equations. 2. Translate the boundary conditions for u into boundary conditions for X, Y , Z, . . .. The continuity of u may give additional boundary conditions and boundedness conditions. 3. Solve the dierential equation(s) that determine the eigenvalues. Make sure to consider all cases. The eigenfunctions will be determined up to a multiplicative constant. 1721

4. Solve the rest of the dierential equations subject to the homogeneous boundary conditions. The eigenvalues will be a parameter in the solution. The solutions will be determined up to a multiplicative constant. 5. The eigen-solutions are the product of the solutions of the ordinary dierential equations. n = Xn Yn Zn . The solution of the partial dierential equation is a linear combination of the eigen-solutions. u(x, y, z, . . .) = an n

6. Solve for the coecients, an using the inhomogeneous boundary conditions.

1722

37.8

Exercises

Exercise 37.1 Solve the following problem with separation of variables. ut (uxx + uyy ) = q(x, y, t), 0 < x < a, 0 < y < b u(x, y, 0) = f (x, y), u(0, y, t) = u(a, y, t) = u(x, 0, t) = u(x, b, t) = 0 Hint, Solution Exercise 37.2 Consider a thin half pipe of unit radius laying on the ground. It is heated by radiation from above. We take the initial temperature of the pipe and the temperature of the ground to be zero. We model this problem with a heat equation with a source term. ut = uxx + A sin(x) u(0, t) = u(, t) = 0, u(x, 0) = 0 Hint, Solution Exercise 37.3 Consider Laplaces Equation 2 u = 0 inside the quarter circle of radius 1 (0 , 0 r 1). Write the problem 2 in polar coordinates u = u(r, ) and use separation of variables to nd the solution subject to the following boundary conditions. 1. u (r, 0) = 0, 2

u r,

= 0,

u(1, ) = f () u (1, ) = g() r

2.

u u (r, 0) = 0, r, 2 Under what conditions does this solution exist? 1723

= 0,

Hint, Solution Exercise 37.4 Consider the 2-D heat equation ut = (uxx + uyy ), on a square plate 0 < x < 1, 0 < y < 1 with two sides insulated ux (0, y, t) = 0 ux (1, y, t) = 0, two sides with xed temperature u(x, 0, t) = 0 u(x, 1, t) = 0, and initial temperature u(x, y, 0) = f (x, y). 1. Reduce this to a set of 3 ordinary dierential equations using separation of variables. 2. Find the corresponding set of eigenfunctions and give the solution satisfying the given initial condition. Hint, Solution Exercise 37.5 Solve the 1-D heat equation ut = uxx , on the domain 0 < x < subject to conditions that the ends are insulated (i.e. zero ux) ux (0, t) = 0 ux (, t) = 0, and the initial temperature distribution is u(x, 0) = x. Hint, Solution 1724

Exercise 37.6 Obtain Poissons formula to solve the Dirichlet problem for the circular region 0 r < R, 0 < 2. That is, determine a solution (r, ) to Laplaces equation 2 =0 in polar coordinates given (R, ). Show that (r, ) = Hint, Solution Exercise 37.7 Consider the temperature of a ring of unit radius. Solve the problem ut = u , with separation of variables. Hint, Solution Exercise 37.8 Solve the Laplaces equation by separation of variables. u uxx + uyy = 0, 0 < x < 1, 0 < y < 1, u(x, 0) = f (x), u(x, 1) = 0, u(0, y) = 0, u(1, y) = 0 Here f (x) is an arbitrary function which is known. Hint, Solution Exercise 37.9 Solve Laplaces equation in the unit disk with separation of variables. u = 0, 0 < r < 1 u(1, ) = f () 1725 u(, 0) = f () 1 2
2

(R, )
0

R2 r 2 d R2 + r2 2Rr cos( )

The Laplacian in cirular coordinates is u Hint, Solution Exercise 37.10 Find the normal modes of oscillation of a drum head of unit radius. The drum head obeys the wave equation with zero displacement on the boundary. v Hint, Solution Exercise 37.11 Solve the equation t = a2 xx , 0 < x < l, t>0 with boundary conditions (0, t) = (l, t) = 0, and initial conditions (x, 0) = x, 0 x l/2, l x, l/2 < x l. 1 r r r v r + 1 2v 1 2v = 2 2, r2 2 c t v(1, , t) = 0 2 u 1 u 1 2u + + 2 2. r2 r r r

Comment on the dierentiability ( that is the number of nite derivatives with respect to x ) at time t = 0 and at time t = , where > 0 and 1. Hint, Solution Exercise 37.12 Consider a one-dimensional rod of length L with initial temperature distribution f (x). The temperatures at the left and right ends of the rod are held at T0 and T1 , respectively. To nd the temperature of the rod for t > 0, solve ut = uxx , 0 < x < L, t > 0 u(0, t) = T0 , u(L, t) = T1 , u(x, 0) = f (x), 1726

with separation of variables. Hint, Solution Exercise 37.13 For 0 < x < l solve the problem t = a2 xx + w(x, t) (0, t) = 0, x (l, t) = 0, (x, 0) = f (x) by means of a series expansion involving the eigenfunctions of d2 (x) + (x) = 0, dx2 (0) = (l) = 0. Here w(x, t) and f (x) are prescribed functions. Hint, Solution Exercise 37.14 Solve the heat equation of Exercise 37.13 with the same initial conditions but with the boundary conditions (0, t) = 0, c(l, t) + x (l, t) = 0. (37.5)

Here c > 0 is a constant. Although it is not possible to solve for the eigenvalues in closed form, show that the eigenvalues assume a simple form for large values of . Hint, Solution Exercise 37.15 Use a series expansion technique to solve the problem t = a2 xx + 1, with boundary and initial conditions given by (x, 0) = 0, (0, t) = t, 1727 x (l, t) = c(l, t) t > 0, 0<x<l

where c > 0 is a constant. Hint, Solution Exercise 37.16 Let (x, t) satisfy the equation t = a2 xx for 0 < x < l, t > 0 with initial conditions (x, 0) = 0 for 0 < x < l, with boundary conditions (0, t) = 0 for t > 0, and (l, t) + x (l, t) = 1 for t > 0. Obtain two series solutions for this problem, one which is useful for large t and the other useful for small t. Hint, Solution Exercise 37.17 A rod occupies the portion 1 < x < 2 of the x-axis. The thermal conductivity depends on x in such a manner that the temperature (x, t) satises the equation t = A2 (x2 x )x (37.6) where A is a constant. For (1, t) = (2, t) = 0 for t > 0, with (x, 0) = f (x) for 1 < x < 2, show that the appropriate series expansion involves the eigenfunctions 1 n (x) = sin x n ln x ln 2 .

Work out the series expansion for the given boundary and initial conditions. Hint, Solution Exercise 37.18 Consider a string of length L with a xed left end a free right end. Initially the string is at rest with displacement f (x). Find the motion of the string by solving, utt = c2 uxx , 0 < x < L, t > 0, u(0, t) = 0, ux (L, t) = 0, u(x, 0) = f (x), ut (x, 0) = 0, 1728

with separation of variables. Hint, Solution Exercise 37.19 Consider the equilibrium temperature distribution in a two-dimensional block of width a and height b. There is a heat source given by the function f (x, y). The vertical sides of the block are held at zero temperature; the horizontal sides are insulated. To nd this equilibrium temperature distribution, solve the potential equation, uxx + uyy = f (x, y), 0 < x < a, 0 < y < b, u(0, y) = u(a, y) = 0, uy (x, 0) = uy (x, b) = 0, with separation of variables. Hint, Solution Exercise 37.20 Consider the vibrations of a sti beam of length L. More precisely, consider the transverse vibrations of an unloaded beam, whose weight can be neglected compared to its stiness. The beam is simply supported at x = 0, L. (That is, it is resting on fulcrums there. u(0, t) = 0 means that the beam is resting on the fulcrum; uxx (0, t) = 0 indicates that there is no bending force at that point.) The beam has initial displacement f (x) and velocity g(x). To determine the motion of the beam, solve utt + a2 uxxxx = 0, 0 < x < L, t > 0, u(x, 0) = f (x), ut (x, 0) = g(x), u(0, t) = uxx (0, t) = 0, u(L, t) = uxx (L, t) = 0, with separation of variables. Hint, Solution Exercise 37.21 The temperature along a magnet winding of length L carrying a current I satises, (for some > 0): ut = uxx + I 2 u. 1729

The ends of the winding are kept at zero, i.e., u(0, t) = u(L, t) = 0; and the initial temperature distribution is u(x, 0) = g(x). Find u(x, t) and determine the critical current ICR which is dened as the least current at which the winding begins to heat up exponentially. Suppose that < 0, so that the winding has a negative coecient of resistance with respect to temperature. What can you say about the critical current in this case? Hint, Solution Exercise 37.22 The e-folding time of a decaying function of time is the time interval, e , in which the magnitude of the function 1 is reduced by at least 1 . Thus if u(x, t) = et f (x) + et g(x) with > > 0 then e = . A body with heat e conductivity has its exterior surface maintained at temperature zero. Initially the interior of the body is at the uniform temperature T > 0. Find the e-folding time of the body if it is: a) An innite slab of thickness a. b) An innite cylinder of radius a. c) A sphere of radius a. Note that in (a) the temperature varies only in the z direction and in time; in (b) and (c) the temperature varies only in the radial direction and in time. d) What are the e-folding times if the surfaces are perfectly insulated, (i.e., at the surface)? Hint, Solution 1730
u n

= 0, where n is the exterior normal

Exercise 37.23 Solve the heat equation with a time-dependent diusivity in the rectangle 0 < x < a, 0 < y < b. The top and bottom sides are held at temperature zero; the lateral sides are insulated. We have the initial-boundary value problem: ut = (t) (uxx + uyy ) , 0 < x < a, 0 < y < b, u(x, 0, t) = u(x, b, t) = 0, ux (0, y, t) = ux (a, y, t) = 0, u(x, y, 0) = f (x, y). The diusivity, (t), is a known, positive function. Hint, Solution Exercise 37.24 A semi-circular rod of innite extent is maintained at temperature T = 0 on the at side and at T = 1 on the curved surface: x2 + y 2 = 1, y > 0. Find the steady state temperature in a cross section of the rod using separation of variables. Hint, Solution Exercise 37.25 Use separation of variables to nd the steady state temperature u(x, y) in a slab: x 0, 0 y 1, which has zero temperature on the faces y = 0 and y = 1 and has a given distribution: u(y, 0) = f (y) on the edge x = 0, 0 y 1. Hint, Solution Exercise 37.26 Find the solution of Laplaces equation subject to the boundary conditions. u = 0, 0 < < , a < r < b, u(r, 0) = u(r, ) = 0, u(a, ) = 0, u(b, ) = f (). Hint, Solution 1731 t > 0,

Exercise 37.27 a) A piano string of length L is struck, at time t = 0, by a at hammer of width 2d centered at a point , having velocity v. Find the ensuing motion, u(x, t), of the string for which the wave speed is c. b) Suppose the hammer is curved, rather than at as above, so that the initial velocity distribution is v cos 0
(x) 2d

ut (x, 0) =

, |x | < d |x | > d.

Find the ensuing motion. c) Compare the kinetic energies of each harmonic in the two solutions. Where should the string be struck in order to maximize the energy in the nth harmonic in each case? Hint, Solution Exercise 37.28 If the striking hammer is not perfectly rigid, then its eect must be included as a time dependent forcing term of the form: s(x, t) = v cos 0
(x) 2d

sin

, for |x | < d, otherwise.

0 < t < ,

Find the motion of the string for t > . Discuss the eects of the width of the hammer and duration of the blow with regard to the energy in overtones. Hint, Solution Exercise 37.29 Find the propagating modes in a square waveguide of side L for harmonic signals of frequency when the propagation speed of the medium is c. That is, we seek those solutions of utt c2 u = 0, 1732

where u = u(x, y, z, t) has the form u(x, y, z, t) = v(x, y, z) et , which satisfy the conditions: u(x, y, z, t) = 0 for x = 0, L, y = 0, L, lim |u| = and = 0.
z

z > 0,

Indicate in terms of inequalities involving k = /c and appropriate eigenvalues, n,m say, for which n and m the solutions un,m satisfy the conditions. Hint, Solution Exercise 37.30 Find the modes of oscillation and their frequencies for a rectangular drum head of width a and height b. The modes of oscillation are eigensolutions of utt = c2 u, 0 < x < a, 0 < y < b, u(0, y) = u(a, y) = u(x, 0) = u(x, b) = 0. Hint, Solution Exercise 37.31 Using separation of variables solve the heat equation t = a2 (xx + yy ) in the rectangle 0 < x < lx , 0 < y < ly with initial conditions (x, y, 0) = 1, and boundary conditions (0, y, t) = (lx , y, t) = 0, Hint, Solution 1733 y (x, 0, t) = y (x, ly , t) = 0.

Exercise 37.32 Using polar coordinates and separation of variables solve the heat equation t = a2 in the circle 0 < r < R0 with initial conditions (r, , 0) = V where V is a constant, and boundary conditions (R0 , , t) = 0. 1. Show that for t > 0, (r, , t) = 2V
n=1 2 a2 j0,n t 2 R0 2

exp

J0 (j0,n r/R0 ) , j0,n J1 (j0,n )

where j0,n are the roots of J0 (x): J0 (j0,n ) = 0, Hint: The following identities may be of some help:
R0

n = 1, 2, . . .

rJ0 (j0,n r/R0 ) J0 (j0,m r/R0 ) dr = 0,


0 R0 2 rJ0 (j0,n r/R0 ) dr = 0 r 2 R0 2 J (j0,n ), 2 1

m = n,

rJ0 (r)dr =
0

r J1 (r)

for any .

2. For any xed r, 0 < r < R0 , use the asymptotic approximation for the Jn Bessel functions for large argument (this can be found in any standard math tables) to determine the rate of decay of the terms of the series solution for at time t = 0. Hint, Solution 1734

Exercise 37.33 Consider the solution of the diusion equation in spherical coordinates given by x = r sin cos , y = r sin sin , z = r cos , where r is the radius, is the polar angle, and is the azimuthal angle. We wish to solve the equation on the surface of the sphere given by r = R, 0 < < , and 0 < < 2. The diusion equation for the solution (, , t) in these coordinates on the surface of the sphere becomes a2 = 2 t R where a is a positive constant. 1. Using separation of variables show that a solution can be found in the form (, , t) = T (t)()(), where T ,, obey ordinary dierential equations in t,, and respectively. Derive the ordinary dierential equations for T and , and show that the dierential equation obeyed by is given by d2 c = 0, d2 where c is a constant. 2. Assuming that (, , t) is determined over the full range of the azimuthal angle, 0 < < 2, determine the allowable values of the separation constant c and the corresponding allowable functions . Using these values of c and letting x = cos rewrite in terms of the variable x the dierential equation satised by . What are appropriate boundary conditions for ? The resulting equation is known as the generalized or associated Legendre equation. 1735 1 sin sin + 1 2 sin2 2 . (37.7)

3. Assume next that the initial conditions for are chosen such that (, , t = 0) = f (), where f () is a specied function which is regular at the north and south poles (that is = 0 and = ). Note that the initial condition is independent of the azimuthal angle . Show that in this case the method of separation of variables gives a series solution for of the form

(, t) =
l=0

Al exp(2 t)Pl (cos ), l

where Pl (x) is the lth Legendre polynomial, and determine the constants l as a function of the index l. 4. Solve for (, t), t > 0 given that f () = 2 cos2 1. Useful facts: dPl (x) d (1 x2 ) + l(l + 1)Pl (x) = 0 dx dx P0 (x) = 1 P1 (x) = x 3 2 1 x P2 (x) = 2 2
1

0 dxPl (x)Pm (x) = 2 2l + 1

if l = m if l = m

Hint, Solution Exercise 37.34 Let (x, y) satisfy Laplaces equation xx + yy = 0 1736

in the rectangle 0 < x < 1, 0 < y < 2, with (x, 2) = x(1 x), and with = 0 on the other three sides. Use a series solution to determine inside the rectangle. How many terms are required to give ( 1 , 1) with about 1% (also 0.1%) 2 accuracy; how about x ( 1 , 1)? 2 Hint, Solution Exercise 37.35 Let (r, , ) satisfy Laplaces equation in spherical coordinates in each of the two regions r < a, r > a, with 0 as r . Let
ra+ ra+

lim (r, , ) lim (r, , ) = 0,


ra ra

m lim r (r, , ) lim r (r, , ) = Pn (cos ) sin(m),

where m and n m are integers. Find in r < a and r > a. In electrostatics, this problem corresponds to that of determining the potential of a spherical harmonic type charge distribution over the surface of the sphere. In this way one can determine the potential due to an arbitrary surface charge distribution since any charge distribution can be expressed as a series of spherical harmonics. Hint, Solution Exercise 37.36 Obtain a formula analogous to the Poisson formula to solve the Neumann problem for the circular region 0 r < R, 0 < 2. That is, determine a solution (r, ) to Laplaces equation
2

=0

in polar coordinates given r (R, ). Show that (r, ) = within an arbitrary additive constant. Hint, Solution 1737 R 2
2

r (R, ) ln 1
0

2r r2 cos( ) + 2 R R

Exercise 37.37 Investigate solutions of t = a2 xx obtained by setting the separation constant C = ( + )2 in the equations obtained by assuming = X(x)T (t): T = C, T Hint, Solution X C = 2. X a

1738

37.9
Hint 37.1 Hint 37.2 Hint 37.3 Hint 37.4 Hint 37.5 Hint 37.6

Hints

Hint 37.7 Impose the boundary conditions u(0, t) = u(2, t), u (0, t) = u (2, t). Hint 37.8 Apply the separation of variables u(x, y) = X(x)Y (y). Solve an eigenvalue problem for X(x). Hint 37.9 Hint 37.10

1739

Hint 37.11

Hint 37.12 There are two ways to solve the problem. For the rst method, expand the solution in a series of the form

u(x, t) =
n=1

an (t) sin

nx . L

Because of the inhomogeneous boundary conditions, the convergence of the series will not be uniform. You can dierentiate the series with respect to t, but not with respect to x. Multiply the partial dierential equation by the eigenfunction sin(nx/L) and integrate from x = 0 to x = L. Use integration by parts to move derivatives in x from u to the eigenfunctions. This process will yield a rst order, ordinary dierential equation for each of the an s. For the second method: Make the change of variables v(x, t) = u(x, t) (x), where (x) is the equilibrium temperature distribution to obtain a problem with homogeneous boundary conditions. Hint 37.13

Hint 37.14

Hint 37.15

Hint 37.16

Hint 37.17

1740

Hint 37.18 Use separation of variables to nd eigen-solutions of the partial dierential equation that satisfy the homogeneous boundary conditions. There will be two eigen-solutions for each eigenvalue. Expand u(x, t) in a series of the eigensolutions. Use the two initial conditions to determine the constants. Hint 37.19 Expand the solution in a series of eigenfunctions in x. Determine these eigenfunctions by using separation of variables on the homogeneous partial dierential equation. You will nd that the answer has the form,

u(x, y) =
n=1

un (y) sin

nx . a

Substitute this series into the partial dierential equation to determine ordinary dierential equations for each of the un s. The boundary conditions on u(x, y) will give you boundary conditions for the un s. Solve these ordinary dierential equations with Green functions. Hint 37.20 Solve this problem by expanding the solution in a series of eigen-solutions that satisfy the partial dierential equation and the homogeneous boundary conditions. Use the initial conditions to determine the coecients in the expansion. Hint 37.21 Use separation of variables to nd eigen-solutions that satisfy the partial dierential equation and the homogeneous boundary conditions. The solution is a linear combination of the eigen-solutions. The whole solution will be exponentially decaying if each of the eigen-solutions is exponentially decaying. Hint 37.22 For parts (a), (b) and (c) use separation of variables. For part (b) the eigen-solutions will involve Bessel functions. For part (c) the eigen-solutions will involve spherical Bessel functions. Part (d) is trivial. Hint 37.23 The solution is a linear combination of eigen-solutions of the partial dierential equation that satisfy the homogeneous boundary conditions. Determine the coecients in the expansion with the initial condition. 1741

Hint 37.24 The problem is 1 1 urr + ur + 2 u = 0, 0 < r < 1, 0 < < r r u(r, 0) = u(r, ) = 0, u(0, ) = 0, u(1, ) = 1 The solution is a linear combination of eigen-solutions that satisfy the partial dierential equation and the three homogeneous boundary conditions. Hint 37.25 Hint 37.26 Hint 37.27 Hint 37.28 Hint 37.29 Hint 37.30 Hint 37.31 Hint 37.32

1742

Hint 37.33 Hint 37.34 Hint 37.35 Hint 37.36 Hint 37.37

1743

37.10

Solutions

Solution 37.1 We expand the solution in eigenfunctions in x and y which satify the boundary conditions.

u=
m,n=1

umn (t) sin

ny mx sin a b

We expand the inhomogeneities in the eigenfunctions.

q(x, y, t) =
m,n=1

qmn (t) sin


b

mx ny sin a b mx ny sin a b dy dx

qmn (t) =

4 ab

a 0 0

q(x, y, t) sin

f (x, y) =
m,n=1

fmn sin
b

mx ny sin a b mx ny sin a b dy dx

fmn =

4 ab

a 0 0

f (x, y) sin

1744

We substitute the expansion of the solution into the diusion equation and the initial condition to determine initial value problems for the coecients in the expansion. ut (uxx + uyy ) = q(x, y, t)

umn (t) +
m,n=1

m a

n + b

mx ny umn (t) sin sin a b

=
m,n=1

qmn (t) sin

mx ny sin a b

umn (t) +

m 2 n 2 + umn (t) = qmn (t) a b u(x, y, 0) = f (x, y)

ny mx umn (0) sin sin a b m,n=1

=
m,n=1

fmn sin

ny mx sin a b

umn (0) = fmn We solve the ordinary dierential equations for the coecients umn (t) subject to their initial conditions.
t

umn (t) =
0

exp

m a

n b

(t ) qmn ( ) d + fmn exp

m a

n b

Solution 37.2 After looking at this problem for a minute or two, it seems like the answer would have the form u = sin(x)T (t). This form satises the boundary conditions. We substitute it into the heat equation and the initial condition to determine 1745

T sin(x)T = sin(x)T + A sin(x), T (0) = 0 T + T = A, T (0) = 0 A T = + c et A T = 1 et Now we have the solution of the heat equation. u= Solution 37.3 First we write the Laplacian in polar coordinates. 1 1 urr + ur + 2 u = 0 r r 1. We introduce the separation of variables u(r, ) = R(r)(). 1 1 R + R + 2 R = 0 r r R R +r = = r2 R R We have a regular Sturm-Liouville problem for and a dierential equation for R. + = 0, (0) = (/2) = 0 r2 R + rR R = 0, R is bounded 1746 (37.8) A sin(x) 1 et

First we solve the problem for to determine the eigenvalues and eigenfunctions. The Rayleigh quotient is =
/2 ( )2 d 0 /2 2 d 0

Immediately we see that the eigenvalues are non-negative. If = 0, then the right boundary condition implies that = 0. Thus = 0 is not an eigenvalue. We nd the general solution of Equation 37.8 for positive . = c1 cos + c2 sin The solution that satises the left boundary condition is = c cos . We apply the right boundary condition to determine the eigenvalues. cos n = (2n 1)2 , =0 2 n = cos ((2n 1)) ,

n Z+

Now we solve the dierential equation for R. Since this is an Euler equation, we make the substitition R = r . r2 Rn + rRn (2n 1)2 Rn = 0 ( 1) + (2n 1)2 = 0 = (2n 1) Rn = c1 r2n1 + c2 r12n The solution which is bounded in 0 r 1 is Rn = r2n1 . 1747

The solution of Laplaces equation is a linear combination of the eigensolutions.

u=
n=1

un r2n1 cos ((2n 1))

We use the boundary condition at r = 1 to determine the coecients.

u(1, ) = f () =
n=1

un cos ((2n 1))

un =

/2

f () cos ((2n 1)) d


0

2. We introduce the separation of variables u(r, ) = R(r)(). 1 1 R + R + 2 R = 0 r r R R +r = = r2 R R We have a regular Sturm-Liouville problem for and a dierential equation for R. + = 0, (0) = (/2) = 0 r2 R + rR R = 0, R is bounded (37.9)

First we solve the problem for to determine the eigenvalues and eigenfunctions. We recognize this problem as the generator of the Fourier cosine series. n = (2n)2 , n Z0+ , 1 0 = , n = cos (2n) , n Z+ 2 1748

Now we solve the dierential equation for R. Since this is an Euler equation, we make the substitition R = r . r2 Rn + rRn (2n)2 Rn = 0 ( 1) + (2n)2 = 0 = 2n R0 = c1 + c2 ln(r), Rn = c1 r2n + c2 r2n , The solutions which are bounded in 0 r 1 are Rn = r2n . The solution of Laplaces equation is a linear combination of the eigensolutions. u0 u= + un r2n cos (2n) 2 n=1 We use the boundary condition at r = 1 to determine the coecients.

n Z+

ur (1, ) =
n=1

2nun cos(2n) = g()

Note that the constant term is missing in this cosine series. g() has such a series expansion only if
/2

g() d = 0.
0

This is the condition for the existence of a solution of the problem. If this is satised, we can solve for the coecients in the expansion. u0 is arbitrary. un = 4
/2

g() cos (2n) d,


0

n Z+

1749

Solution 37.4 1. ut = (uxx + uyy ) XY T = (X Y T + XY T ) T X Y = + = T X Y Y X = = X Y We have boundary value problems for X(x) and Y (y) and a dierential equation for T (t). X + X = 0, X (0) = X (1) = 0 Y + ( )Y = 0, Y (0) = Y (1) = 0 T = T 2. The solutions for X(x) form a cosine series. m = m 2 2 , The solutions for Y (y) form a sine series. mn = (m2 + n2 ) 2 , We solve the ordinary dierential equation for T (t). Tmn = e(m
2 +n2 ) 2 t

m Z0+ ,

1 X0 = , 2

Xm = cos(mx)

n Z+ ,

Yn = sin(nx)

We expand the solution of the heat equation in a series of the eigensolutions. 1 u(x, y, t) = 2

u0n sin(ny) e
n=1

n2 2 t

+
m=1 n=1

umn cos(mx) sin(ny) e(m

2 +n2 ) 2 t

1750

We use the initial condition to determine the coecients. 1 u(x, y, 0) = f (x, y) = 2 umn = 4
0 0

u0n sin(ny) +
n=1 1 1 m=1 n=1

umn cos(mx) sin(ny)

f (x, y) cos(mx) sin(ny) dx dy

Solution 37.5 We use the separation of variables u(x, t) = X(x)T (t) to nd eigensolutions of the heat equation that satisfy the boundary conditions at x = 0, . ut = uxx XT = X T T X = = T X The problem for X(x) is X + X = 0, The eigenfunctions form the familiar cosine series. n = n 2 , n Z0+ , 1 X0 = , 2 Xn = cos(nx) X (0) = X () = 0.

Next we solve the dierential equation for T (t). Tn = n2 Tn T0 = 1, Tn = en


2t

We expand the solution of the heat equation in a series of the eigensolutions. 1 2 u(x, t) = u0 + un cos(nx) en t 2 n=1 1751

We use the initial condition to determine the coecients in the series. 1 u(x, 0) = x = u0 + un cos(nx) 2 n=1 2 u0 = 2 un =

x dx =
0

x cos(nx) dx =
0

0 4 n2

even n odd n

u(x, t) = 2 Solution 37.6 We expand the solution in a Fourier series.

n=1 odd n

4 2 cos(nx) en t 2 n

1 = a0 (r) + an (r) cos(n) + bn (r) sin(n) 2 n=1 n=1 We substitute the series into the Laplaces equation to determine ordinary dierential equations for the coecients. r r r + 1 2 =0 r2 2

1 1 1 a0 + a0 = 0, an + an n2 an = 0, bn + bn n2 bn = 0 r r r The solutions that are bounded at r = 0 are, (to within multiplicative constants), a0 (r) = 1, Thus (r, ) has the form 1 (r, ) = c0 + cn rn cos(n) + dn rn sin(n) 2 n=1 n=1 1752

an (r) = rn ,

bn (r) = rn .

We apply the boundary condition at r = R. 1 (R, ) = c0 + cn Rn cos(n) + dn Rn sin(n) 2 n=1 n=1 The coecients are c0 = 1
2

(R, ) d,
0

cn =

1 Rn

(R, ) cos(n) d,
0

dn =

1 Rn

(R, ) sin(n) d.
0

We substitute the coecients into our series solution. 1 (r, ) = 2 1 (r, ) = 2 (r, ) = 1 (r, ) = 2 (r, ) =
2 0 2 0

1 (R, ) d + 1 (R, ) d +
2

n=1 2

r R

n 0

(R, ) cos(n( )) d

(R, )
0 n=1 2

r R
r R

en()

d d d

1 2

(R, ) d +
0 2

(R, )
0

e() r 1 R e()

2 0

1 (R, ) d +
2

(R, )
0

1 R2

1 2

(R, ) d +
0

(R, )
0

Rr cos( ) r d + r2 2Rr cos( )

r () r 2 e R R r r 2 2 R cos( ) + R 2

(r, ) =

1 2

(R, )
0

R2 r 2 d R2 + r2 2Rr cos( )

Solution 37.7 In order that the solution is continuously dierentiable, (which it must be in order to satisfy the dierential equation), we impose the boundary conditions u(0, t) = u(2, t), u (0, t) = u (2, t).

1753

We apply the separation of variables u(, t) = ()T (t). ut = u T = T T = = T We have the self-adjoint eigenvalue problem + = 0, (0) = (2), (0) = (2)

which has the eigenvalues and orthonormal eigenfunctions n = n 2 , 1 n = en , 2 Tn = n2 Tn Tn = en The solution is a linear combination of the eigen-solutions. u(, t) = 1 2 un en en t 2 n=

2 t

n Z.

Now we solve the problems for Tn (t) to obtain eigen-solutions of the heat equation.

We use the initial conditions to determine the coecients. u(, 0) = 1 un en = f () 2 n=


2

1 un = 2

en f () d
0

1754

Solution 37.8 Substituting u(x, y) = X(x)Y (y) into the partial dierential equation yields X Y = = . X Y With the homogeneous boundary conditions, we have the two problems X + X = 0, Y Y = 0, The eigenvalues and orthonormal eigenfunctions for X(x) are n = (n)2 , The general solution for Y is Yn = a cosh(ny) + b sinh(ny). The solution for that satises the right homogeneous boundary condition, (up to a multiplicative constant), is Yn = sinh(n(1 y)) u(x, y) is a linear combination of the eigen-solutions.

X(0) = X(1) = 0, Y (1) = 0.

Xn =

2 sin(nx).

u(x, y) =
n=1

un 2 sin(nx) sinh(n(1 y))

We use the inhomogeneous boundary condition to determine coecients.

u(x, 0) =
n=1

un 2 sin(nx) sinh(n) = f (x)


1

un =

2
0

sin(n)f () d

1755

Solution 37.9 We substitute u(r, ) = R(r)() into the partial dierential equation. 2 u 1 u 1 2u + + 2 2 =0 r2 r r r 1 1 R + R + 2 R = 0 r r R R r2 +r = = R R r2 R + rR R = 0, + = 0 We assume that u is a strong solution of the partial dierential equation and is thus twice continuously dierentiable, (u C 2 ). In particular, this implies that R and are bounded and that is continuous and has a continuous rst derivative along = 0. This gives us a boundary value problem for and a dierential equation for R. + = 0, (0) = (2), (0) = (2) 2 r R + rR R = 0, R is bounded The eigensolutions for form the familiar Fourier series. n = n 2 , n Z0+

1 (1) 0 = , (1) = cos(n), n Z+ n 2 (2) n = sin(n), n Z+ Now we nd the bounded solutions for R. The equation for R is an Euler equation so we use the substitution R = r . r2 Rn + rRn n Rn = 0 ( 1) + n = 0 = n 1756

First we consider the case 0 = 0. The solution is R = a + b ln r. Boundedness demands that b = 0. Thus we have the solution R = 1. Now we consider the case n = n2 > 0. The solution is Rn = arn + brn . Boundedness demands that b = 0. Thus we have the solution Rn = r n . The solution for u is a linear combination of the eigensolutions. u(r, ) = a0 + (an cos(n) + bn sin(n)) rn 2 n=1

The boundary condition at r = 1 determines the coecients in the expansion. a0 u(1, ) = + [an cos(n) + bn sin(n)] = f () 2 n=1 an = 1
2

f () cos(n) d,
0

bn =

f () sin(n) d
0

Solution 37.10 A normal mode of frequency is periodic in time. v(r, , t) = u(r, ) et 1757

We substitute this form into the wave equation to obtain a Helmholtz equation, (also called a reduced wave equation). 1 u 1 2u 2 r + 2 2 = 2 u, r r r r c 2 2 u 1 u 1 u + + 2 2 + k 2 u = 0, 2 r r r r u(1, ) = 0, u(1, ) = 0

Here we have dened k = . We apply the separation of variables u = R(r)() to the Helmholtz equation. c r2 R + rR + R + k 2 r2 R = 0, R R r2 + r + k2 r2 = = 2 R R Now we have an ordinary dierential equation for R(r) and an eigenvalue problem for (). 2 1 R + R + k 2 2 R = 0, R(0) is bounded, R(1) = 0, r r 2 + = 0, () = (), () = (). We compute the eigenvalues and eigenfunctions for . n = n, 1 0 = , (1) = cos(n), n 2 The dierential equations for the Rn are Bessel equations. n2 1 Rn + Rn + k 2 2 r r Rn = 0, n Z0+ (2) = sin(n), n n Z+

Rn (0) is bounded,

Rn (1) = 0

The general solution is a linear combination of order n Bessel functions of the rst and second kind. Rn (r) = c1 Jn (kr) + c2 Yn (kr) 1758

Since the Bessel function of the second kind, Yn (kr), is unbounded at r = 0, the solution has the form Rn (r) = cJn (kr). Applying the second boundary condition gives us the admissable frequencies. Jn (k) = 0 = Jn (jnm r), n Z0+ ,

knm = jnm ,

Rnm

m Z+

Here jnm is the mth positive root of Jn . We combining the above results to obtain the normal modes of oscillation. 1 v0m = J0 (j0m r) ecj0m t , m Z+ 2 vnm = cos(n + )Jnm (jnm r) ecjnm t , n, m Z+ Some normal modes are plotted in Figure 37.2. Note that cos(n + ) represents a linear combination of cos(n) and sin(n). This form is preferrable as it illustrates the circular symmetry of the problem. Solution 37.11 We will expand the solution in a complete, orthogonal set of functions {Xn (x)}, where the coecients are functions of t. = Tn (t)Xn (x)
n

We will use separation of variables to determine a convenient set {Xn }. We substitite = T (t)X(x) into the diusion equation. t = a2 xx XT = a2 X T T X = = a2 T X T = a2 T, X + X = 0 1759

Note that in order to satisfy (0, t) = (l, t) = 0, the Xn must satisfy the same homogeneous boundary conditions, Xn (0) = Xn (l) = 0. This gives us a Sturm-Liouville problem for X(x).

X + X = 0, X(0) = X(l) = 0 n 2 nx n = , Xn = sin , n Z+ l l

Thus we seek a solution of the form

=
n=1

Tn (t) sin

nx . l

(37.10)

This solution automatically satises the boundary conditions. We will assume that we can dierentiate it. We will substitite this form into the diusion equation and the initial condition to determine the coecients in the series, Tn (t). First we substitute Equation 37.10 into the partial dierential equation for to determine ordinary dierential equations for the Tn .

t = a2 xx n nx = a2 Tn (t) sin l l n=1 n=1 Tn = an l


2 2

Tn (t) sin

nx l

Tn

1760

Now we substitute Equation 37.10 into the initial condition for to determine initial conditions for the Tn .

Tn (0) sin
n=1

nx = (x, 0) l
nx (x, 0) dx l l sin2 nx dx l 0

Tn (0) = Tn (0) =

l 0

sin

2 l nx sin (x, 0) dx l 0 l 2 l/2 2 l/2 nx nx Tn (0) = sin x dx + sin (l x) dx l 0 l l 0 l 4l n Tn (0) = 2 2 sin n 2 4l T2n1 (0) = (1)n , T2n (0) = 0, n Z+ (2n 1)2 2 We solve the ordinary dierential equations for Tn subject to the initial conditions. T2n1 (t) = (1)n 4l exp (2n 1)2 2 a(2n 1) l
2

t ,

T2n (t) = 0,

n Z+

This determines the series representation of the solution. = 4 l

(1)n
n=1

l (2n 1)

exp

a(2n 1) l

t sin

(2n 1)x l

From the initial condition, we know that the the solution at t = 0 is C 0 . That is, it is continuous, but not dierentiable. The series representation of the solution at t = 0 is 4 = l

(1)
n=1

l (2n 1) 1761

sin

(2n 1)x l

That the coecients decay as 1/n2 corroborates that (x, 0) is C 0 . The derivatives of with respect to x are 2m1 4(1)m+1 = x2m1 l 4(1) = 2m x l
2m m

(1)n (1)n

n=1

(2n 1) l (2n 1) l

2m3

exp
2m2

a(2n 1) l a(2n 1) l
2

t cos t sin

(2n 1)x l (2n 1)x l

exp
n x

n=1

For any xed t > 0, the coecients in the series for x. Thus for any xed t > 0, is C in x. Solution 37.12

decay exponentially. These series are uniformly convergent in

ut = uxx , 0 < x < L, t > 0 u(0, t) = T0 , u(L, t) = T1 , u(x, 0) = f (x), Method 1. We solve this problem with an eigenfunction expansion in x. To nd an appropriate set of eigenfunctions, we apply the separation of variables, u(x, t) = X(x)T (t) to the partial dierential equation with the homogeneous boundary conditions, u(0, t) = u(L, t) = 0. (XT )t = (XT )xx XT = X T T X = = 2 T X We have the eigenvalue problem, X + 2 X = 0, which has the solutions, n = nx , L X(0) = X(L) = 0, nx , L n N.

Xn = sin 1762

We expand the solution of the partial dierential equation in terms of these eigenfunctions.

u(x, t) =
n=1

an (t) sin

nx L

Because of the inhomogeneous boundary conditions, the convergence of the series will not be uniform. We can dierentiate the series with respect to t, but not with respect to x. We multiply the partial dierential equation by an eigenfunction and integrate from x = 0 to x = L. We use integration by parts to move derivatives from u to the eigenfunction.

ut uxx = 0
L

(ut uxx ) sin


0 L 0

mx L

dx = 0
L

an (t) sin
n=1

nx L

sin

mx L

dx ux sin
L

mx L
2 0 L

+
0

m L

ux cos
0

mx L

dx = 0

L m mx am (t) + u cos 2 L L

+
0 2

m L
L 0

u sin an (t) sin


n=1

mx L nx L

dx = 0 sin mx L dx = 0

L m m am (t) + ((1)m u(L, t) u(0, t)) + 2 L L

m L m 2 L am (t) + ((1)m T1 T0 ) + am (t) = 0 2 L 2 L m 2 2m am (t) + am (t) = 2 (T0 (1)m T1 ) L L 1763

Now we have a rst order dierential equation for each of the an s. We obtain initial conditions for each of the an s from the initial condition for u(x, t). u(x, 0) = f (x)

an (0) sin
n=1

nx = f (x) L nx L dx fn

an (0) =

2 L

f (x) sin
0

By solving the rst order dierential equation for an (t), we obtain an (t) = 2(T0 (1)n T1 ) 2(T0 (1)n T1 ) 2 + e(n/L) t fn n n .

Note that the series does not converge uniformly due to the 1/n term. Method 2. For our second method we transform the problem to one with homogeneous boundary conditions so that we can use the partial dierential equation to determine the time dependence of the eigen-solutions. We make the change of variables v(x, t) = u(x, t) (x) where (x) is some function that satises the inhomogeneous boundary conditions. If possible, we want (x) to satisfy the partial dierential equation as well. For this problem we can choose (x) to be the equilibrium solution which satises (x) = 0, This has the solution (x) = T0 + With the change of variables, v(x, t) = u(x, t) T0 + 1764 T1 T0 x , L (0)T0 , (L) = T1 .

T1 T0 x. L

we obtain the problem vt = vxx , v(0, t) = 0, v(L, t) = 0, 0 < x < L, t>0 T1 T0 x . L

v(x, 0) = f (x) T0 +

Now we substitute the separation of variables v(x, t) = X(x)T (t) into the partial dierential equation. (XT )t = (XT )xx T X = = 2 T X Utilizing the boundary conditions at x = 0, L we obtain the two ordinary dierential equations, T = 2 T, X = 2 X, X(0) = X(L) = 0. The problem for X is a regular Sturm-Liouville problem and has the solutions n = n , L Xn = sin nx , L n N.

The ordinary dierential equation for T becomes, Tn = which, (up to a multiplicative constant), has the solution, Tn = e(n/L) t . Thus the eigenvalues and eigen-solutions of the partial dierential equation are, n = n , L vn = sin nx (n/L)2 t e , L 1765 n N.
2

n L

Tn ,

Let v(x, t) have the series expansion,

v(x, t) =
n=1

an sin

nx (n/L)2 t e . L

We determine the coecients in the expansion from the initial condition,

v(x, 0) =
n=1

an sin

nx T1 T0 = f (x) T0 + x . L L
T1 T0 x L

The coecients in the expansion are the Fourier sine coecients of f (x) T0 + an = 2 L
L 0

f (x) T0 +

nx T1 T0 x sin L L n 2(T0 (1) T1 ) an = fn n

dx

With the coecients dened above, the solution for u(x, t) is T1 T0 2(T0 (1)n T1 ) u(x, t) = T0 + x+ fn L n n=1

sin

nx (n/L)2 t e . L

Since the coecients in the sum decay exponentially for t > 0, we see that the series is uniformly convergent for positive t. It is clear that the two solutions we have obtained are equivalent. Solution 37.13 First we solve the eigenvalue problem for (x), which is the problem we would obtain if we applied separation of variables to the partial dierential equation, t = xx . We have the eigenvalues and orthonormal eigenfunctions n = (2n 1) 2l
2

n (x) =

2 sin l

(2n 1)x 2l

n Z+ .

1766

We expand the solution and inhomogeneity in Equation 37.5 in a series of the eigenvalues.

(x, t) =
n=1

Tn (t)n (x)
l

w(x, t) =
n=1

wn (t)n (x),

wn (t) =
0

n (x)w(x, t) dx

Since satises the same homgeneous boundary conditions as , we substitute the series into Equation 37.5 to determine dierential equations for the Tn (t).

Tn (t)n (x) = a2
n=1 n=1

Tn (t)(n )n (x) +
n=1 2

wn (t)n (x)

Tn (t) = a2

(2n 1) 2l

Tn (t) + wn (t)

Now we substitute the series for into its initial condition to determine initial conditions for the Tn . (x, 0) =
n=1 l

Tn (0)n (x) = f (x) n (x)f (x) dx


0

Tn (0) = We solve for Tn (t) to determine the solution, (x, t). Tn (t) = exp (2n 1)a 2l
2

Tn (0) +
0

wn ( ) exp

(2n 1)a 2l

Solution 37.14 Separation of variables leads to the eigenvalue problem + = 0, (0) = 0, 1767 (l) + c (l) = 0.

First we consider the case = 0. A set of solutions of the dierential equation is {1, x}. The solution that satises the left boundary condition is (x) = x. The right boundary condition imposes the constraint l + c = 0. Since c is positive, this has no solutions. = 0 is not an eigenvalue. Now we consider = 0. A set of solutions of the dierential equation is {cos( x), sin( x)}. The solution that satises the left boundary condition is = sin( x). The right boundary condition imposes the constraint c sin l + tan cos l = l = 0 c

For large , the we can determine approximate solutions. n l n The eigenfunctions are n (x) = sin
l 0

(2n 1) , n Z+ 2 2 (2n 1) , n Z+ 2l

sin2

n x , n Z+ . n x dx

We expand (x, t) and w(x, t) in series of the eigenfunctions.

(x, t) =
n=1

Tn (t)n (x)
l

w(x, t) =
n=1

wn (t)n (x),

wn (t) =
0

n (x)w(x, t) dx

1768

Since satises the same homgeneous boundary conditions as , we substitute the series into Equation 37.5 to determine dierential equations for the Tn (t).

Tn (t)n (x) = a
n=1

2 n=1

Tn (t)(n )n (x) +
n=1

wn (t)n (x)

Tn (t) = a2 n Tn (t) + wn (t) Now we substitute the series for into its initial condition to determine initial conditions for the Tn .

(x, 0) =
n=1

Tn (0)n (x) = f (x)


l

Tn (0) =
0

n (x)f (x) dx

We solve for Tn (t) to determine the solution, (x, t).


t

Tn (t) = exp a2 n t

Tn (0) +
0

wn ( ) exp a2 n d

Solution 37.15 First we seek a function u(x, t) that satises the boundary conditions u(0, t) = t, ux (l, t) = cu(l, t). We try a function of the form u = (ax + b)t. The left boundary condition imposes the constraint b = 1. We then apply the right boundary condition no determine u. at = c(al + 1)t c a= 1 + cl cx u(x, t) = 1 1 + cl 1769

Now we dene to be the dierence of and u. (x, t) = (x, t) u(x, t) satises an inhomogeneous diusion equation with homogeneous boundary conditions. ( + u)t = a2 ( + u)xx + 1 t = a2 xx + 1 + a2 uxx ut cx t = a2 xx + 1 + cl The initial and boundary conditions for are (x, 0) = 0, We solved this system in problem 2. Just take w(x, t) = The solution is

(0, t) = 0,

x (l, t) = c(l, t).

cx , 1 + cl

f (x) = 0.

(x, t) =
n=1 t

Tn (t)n (x),

Tn (t) =
0

wn exp a2 n (t ) d,
l

wn (t) =
0

n (x)

cx dx. 1 + cl

This determines the solution for . 1770

Solution 37.16 First we solve this problem with a series expansion. We transform the problem to one with homogeneous boundary conditions. Note that x u(x) = l+1 satises the boundary conditions. (It is the equilibrium solution.) We make the change of variables = u. The problem for is t = a2 xx , (0, t) = (l, t) + x (l, t) = 0, (x, 0) = x . l+1

This is a particular case of what we solved in Exercise 37.14. We apply the result of that problem. The solution for (x, t) is x + Tn (t)n (x) l + 1 n=1 sin n x , n Z+ l 2 sin n x dx 0 tan l =
l

(x, t) = n (x) =

Tn (t) = Tn (0) exp a2 n t Tn (0) =


0

n (x)

x dx l+1

This expansion is useful for large t because the coecients decay exponentially with increasing t. 1771

Now we solve this problem with the Laplace transform.

t = a2 xx ,

(x, 0) = 0 1 s = a2 xx , (0, s) = 0, (l, s) + x (l, s) = s s 1 xx 2 = 0, (0, s) = 0, (l, s) + x (l, s) = a s

(0, t) = 0,

(l, t) + x (l, t) = 1,

The solution that satises the left boundary condition is

= c sinh

sx a

We apply the right boundary condition to determine the constant.

= s sinh

sinh
sl a

sx a s cosh a

sl a

1772

We expand this in a series of simpler functions of s.

= s exp = exp

2 sinh
sl a

sx a

exp
sx a sl a

sl a

s a

exp 1

sl a

+ exp

sl a

2 sinh s exp

1+

s a

s a

exp 2 asl 1

= exp

sx a

exp
s a

sx a

s 1+

exp

sl a

1 s/a 1+ s/a

exp 2 asl
n

s(xl) a

exp
s a

s(xl) a

s 1+

n=0

1 s/a 1 + s/a

2 sln exp a

1 = s

n=0

(1 s/a)n s((2n + 1)l x) exp a (1 + s/a)n+1

n=0

(1 s/a)n s((2n + 1)l + x) exp a (1 + s/a)n+1

By expanding

(1 s/a)n (1 + s/a)n+1 s
m/23/2

in binomial series all the terms would be of the form exp s((2n 1)l a x) .

1773

Taking the rst term in each series yields a 3/2 s s(l x) exp a exp s(l + x) a , as s . 1.

We take the inverse Laplace transform to obtain an appoximation of the solution for t 2 2 exp (l+x) exp (lx) 4a2 t 4a2 t (x, t) 2a2 t lx l+x erfc Solution 37.17 We apply the separation of variables (x, t) = X(x)T (t). t = A2 x2 x
2 2 x

lx 2a t

erfc

l+x 2a t

for t

XT = T A x X (x2 X ) T = = A2 T X This gives us a regular Sturm-Liouville problem. x2 X + X = 0, X(1) = X(2) = 0

This is an Euler equation. We make the substitution X = x to nd the solutions. x2 X + 2xX + X = 0 ( 1) + 2 + = 0 1 1 4 = 2 1 = 1/4 2 1774 (37.11)

First we consider the case of a double root when = 1/4. The solutions of Equation 37.11 are {x1/2 , x1/2 ln x}. The solution that satises the left boundary condition is X = x1/2 ln x. Since this does not satisfy the right boundary condition, = 1/4 is not an eigenvalue. Now we consider = 1/4. The solutions of Equation 37.11 are 1 cos x 1 1/4 ln x , sin x 1 sin x The right boundary condition imposes the constraint 1/4 ln 2 = n, This gives us the eigenvalues and eigenfunctions. n = We normalize the eigenfunctions.
2 1

1/4 ln x

The solution that satises the left boundary condition is 1/4 ln x .

n Z+ . n ln x ln 2

1 n + 4 ln 2

1 Xn (x) = sin x
1

n Z+ .

1 sin2 x

n ln x ln 2

dx = ln 2
0

sin2 (n) d = , n Z+ .

ln 2 2

Xn (x) =

2 1 sin ln 2 x

n ln x ln 2

From separation of variables, we have dierential equations for the Tn . n 2 1 + Tn 4 ln 2 1 n 2 Tn (t) = exp A2 + t 4 ln 2 Tn = A2 1775

We expand in a series of the eigensolutions.

(x, t) =
n=1

n Xn (x)Tn (t)

We substitute the expansion for into the initial condition to determine the coecients.

(x, 0) =
n=1 2

n Xn (x) = f (x) Xn (x)f (x) dx


1

n = Solution 37.18

utt = c2 uxx , 0 < x < L, t > 0, u(0, t) = 0, ux (L, t) = 0, u(x, 0) = f (x), ut (x, 0) = 0, We substitute the separation of variables u(x, t) = X(x)T (t) into the partial dierential equation. (XT )tt = c2 (XT )xx T X = = 2 2T c X With the boundary conditions at x = 0, L, we have the ordinary dierential equations, T = c2 2 T, X = 2 X, X(0) = X (L) = 0. 1776

The problem for X is a regular Sturm-Liouville eigenvalue problem. From the Rayleigh quotient, =
2

[ ]L + 0
L 0

L ( )2 0

dx

2 dx

L ( )2 dx 0 L 2 dx 0

we see that there are only positive eigenvalues. For 2 > 0 the general solution of the ordinary dierential equation is X = a1 cos(x) + a2 sin(x). The solution that satises the left boundary condition is X = a sin(x). For non-trivial solutions, the right boundary condition imposes the constraint, cos (L) = 0, = The eigenvalues and eigenfunctions are n = (2n 1) , 2L Xn = sin (2n 1)x 2L
2

1 2

n N.

n N.

The dierential equation for T becomes T = c which has the two linearly independent solutions,
(1) Tn = cos 2

(2n 1) 2L

T,

(2n 1)ct 2L

(2) Tn = sin

(2n 1)ct 2L

1777

The eigenvalues and eigen-solutions of the partial dierential equation are, n = u(1) = sin n (2n 1)x 2L

(2n 1) , 2L ,

n N, (2n 1)x 2L sin (2n 1)ct 2L .

cos

(2n 1)ct 2L

u(2) = sin n

We expand u(x, t) in a series of the eigen-solutions. u(x, t) =


n=1

sin

(2n 1)x 2L

an cos

(2n 1)ct 2L

+ bn sin

(2n 1)ct 2L

We impose the initial condition ut (x, 0) = 0,

ut (x, 0) =
n=1

bn

(2n 1)c sin 2L bn = 0.

(2n 1)x 2L

= 0,

The initial condition u(x, 0) = f (x) allows us to determine the remaining coecients,

u(x, 0) =
n=1

an sin
L

(2n 1)x 2L (2n 1)x 2L

= f (x),

an = The series solution for u(x, t) is,

2 L

f (x) sin
0

dx.

u(x, t) =
n=1

an sin

(2n 1)x 2L

cos

(2n 1)ct 2L

1778

Solution 37.19 uxx + uyy = f (x, y), 0 < x < a, 0 < y < b, u(0, y) = u(a, y) = 0, uy (x, 0) = uy (x, b) = 0 We will solve this problem with an eigenfunction expansion in x. To determine a suitable set of eigenfunctions, we substitute the separation of variables u(x, y) = X(x)Y (y) into the homogeneous partial dierential equation. uxx + uyy = 0 (XY )xx + (XY )yy = 0 X Y = = 2 X Y With the boundary conditions at x = 0, a, we have the regular Sturm-Liouville problem, X = 2 X, which has the solutions, n = X(0) = X(a) = 0, n Z+ .

n nx , Xn = sin , a a We expand u(x, y) in a series of the eigenfunctions.

u(x, y) =
n=1

un (y) sin

nx a

We substitute this series into the partial dierential equation and boundary conditions at y = 0, b.

n=1

n a

un (y) sin

nx nx + un (y) sin a a

= f (x)

un (0) sin
n=1

nx = a

un (b) sin
n=1

nx =0 a

1779

We expand f (x, y) in a Fourier sine series.

f (x, y) =
n=1

fn (y) sin
a

nx a nx a dx

2 fn (y) = a

f (x, y) sin
0

We obtain the ordinary dierential equations for the coecients in the expansion. n 2 un (y) = fn (y), un (0) = un (b) = 0, a We will solve these ordinary dierential equations with Green functions. Consider the Green function problem, un (y) gn (y; ) The homogeneous solutions ny n(y b) and cosh a a satisfy the left and right boundary conditions, respectively. We compute the Wronskian of these two solutions. cosh W (y) = cosh(ny/a) cosh(n(y b)/a) n sinh(ny/a) a sinh(n(y b)/a) ny n(y b) ny n cosh sinh sinh cosh = a a a a n nb = sinh a a
n a

n Z+ .

n a

gn (y; ) = (y ),

gn (0; ) = gn (b; ) = 0.

n(y b) a

The Green function is gn (y; ) = a cosh(ny< /a) cosh(n(y> b)/a) . n sinh(nb/a) 1780

The solutions for the coecients in the expansion are


b

un (y) =
0

gn (y; )fn () d.

Solution 37.20 utt + a2 uxxxx = 0, 0 < x < L, t > 0, u(x, 0) = f (x), ut (x, 0) = g(x), u(0, t) = uxx (0, t) = 0, u(L, t) = uxx (L, t) = 0, We will solve this problem by expanding the solution in a series of eigen-solutions that satisfy the partial dierential equation and the homogeneous boundary conditions. We will use the initial conditions to determine the coecients in the expansion. We substitute the separation of variables, u(x, t) = X(x)T (t) into the partial dierential equation. (XT )tt + a2 (XT )xxxx = 0 T X = = 4 2T a X Here we make the assumption that 0 arg() < /2, i.e., lies in the rst quadrant of the complex plane. Note that 4 covers the entire complex plane. We have the ordinary dierential equation, T = a2 4 T, and with the boundary conditions at x = 0, L, the eigenvalue problem, X = 4 X, X(0) = X (0) = X(L) = X (L) = 0.

For = 0, the general solution of the dierential equation is X = c1 + c2 x + c3 x2 + c4 x3 . 1781

Only the trivial solution satises the boundary conditions. = 0 is not an eigenvalue. For = 0, a set of linearly independent solutions is {ex , ex , ex , ex }. Another linearly independent set, (which will be more useful for this problem), is {cos(x), sin(x), cosh(x), sinh(x)}. Both sin(x) and sinh(x) satisfy the left boundary conditions. Consider the linear combination c1 cos(x)+c2 cosh(x). The left boundary conditions impose the two constraints c1 + c2 = 0, c1 c2 = 0. Only the trivial linear combination of cos(x) and cosh(x) can satisfy the left boundary condition. Thus the solution has the form, X = c1 sin(x) + c2 sinh(x). The right boundary conditions impose the constraints, c1 sin(L) + c2 sinh(L) = 0, c1 2 sin(L) + c2 2 sinh(L) = 0 c1 sin(L) + c2 sinh(L) = 0, c1 sin(L) + c2 sinh(L) = 0 This set of equations has a nontrivial solution if and only if the determinant is zero, sin(L) sinh(L) = 2 sin(L) sinh(L) = 0. sin(L) sinh(L) Since sinh(z) is nonzero in 0 arg(z) < /2, z = 0, and sin(z) has the zeros z = n, n N in this domain, the eigenvalues and eigenfunctions are, n = n , L Xn = sin 1782 nx , L n N.

The dierential equation for T becomes, T = a2 which has the solutions, n 2 n t , sin a L L The eigen-solutions of the partial dierential equation are, cos a u(1) = sin n nx n cos a L L
2 2

n L

T,

t ,

u(2) = sin n

nx n sin a L L

t ,

n N.

We expand the solution of the partial dierential equation in a series of the eigen-solutions.

u(x, t) =
n=1

sin

nx L

cn cos a

n L

t + dn sin a

n L

The initial condition for u(x, t) and ut (x, t) allow us to determine the coecients in the expansion.

u(x, 0) =
n=1

cn sin dn a n L
2

nx = f (x) L sin nx = g(x) L

ut (x, 0) =
n=1

cn and dn are coecients in Fourier sine series. cn = 2 L


L

f (x) sin
0 L

nx L nx L

dx

dn =

2L a 2 n2

g(x) sin
0

dx

1783

Solution 37.21 ut = uxx + I 2 u, 0 < x < L, t > 0, u(0, t) = u(L, t) = 0, u(x, 0) = g(x). We will solve this problem with an expansion in eigen-solutions of the partial dierential equation. We substitute the separation of variables u(x, t) = X(x)T (t) into the partial dierential equation. (XT )t = (XT )xx + I 2 XT T I 2 X = = 2 T X Now we have an ordinary dierential equation for T and a Sturm-Liouville eigenvalue problem for X. (Note that we have followed the rule of thumb that the problem will be easier if we move all the parameters out of the eigenvalue problem.) T = 2 I 2 T X = 2 X, X(0) = X(L) = 0 The eigenvalues and eigenfunctions for X are n = The dierential equation for T becomes, Tn = which has the solution, Tn = c exp n L
2

n , L

Xn = sin

nx , L

n N.

n L

I 2 Tn ,

I 2 t .

1784

From this solution, we see that the critical current is ICR = . L

If I is greater that this, then the eigen-solution for n = 1 will be exponentially growing. This would make the whole solution exponentially growing. For I < ICR , each of the Tn is exponentially decaying. The eigen-solutions of the partial dierential equation are, un = exp We expand u(x, t) in its eigen-solutions, un .

n L

I 2 t sin

nx , L

n N.

u(x, t) =
n=1

an exp

n L

I 2 t sin

nx L

We determine the coecients an from the initial condition.

u(x, 0) =
n=1

an sin
L

nx = g(x) L nx L dx.

2 an = L

g(x) sin
0

If < 0, then the solution is exponentially decaying regardless of current. Thus there is no critical current. Solution 37.22

1785

a) The problem is ut (x, y, z, t) = u(x, y, z, t), < x < , < y < , 0 < z < a, u(x, y, z, 0) = T, u(x, y, 0, t) = u(x, y, a, t) = 0. t > 0,

Because of symmetry, the partial dierential equation in four variables is reduced to a problem in two variables, ut (z, t) = uzz (z, t), 0 < z < a, t > 0, u(z, 0) = T, u(0, t) = u(a, t) = 0. We will solve this problem with an expansion in eigen-solutions of the partial dierential equation that satisfy the homogeneous boundary conditions. We substitute the separation of variables u(z, t) = Z(z)T (t) into the partial dierential equation. ZT = Z T T Z = = 2 T Z With the boundary conditions at z = 0, a we have the Sturm-Liouville eigenvalue problem, Z = 2 Z, which has the solutions, n = The problem for T becomes, Tn = with the solution, Tn = exp 1786 n a
2

Z(0) = Z(a) = 0, nz , a
2

n , a

Zn = sin

n N.

n a

Tn ,

t .

The eigen-solutions are un (z, t) = sin nz n exp a a


2

t .

The solution for u is a linear combination of the eigen-solutions. The slowest decaying eigen-solution is u1 (z, t) = sin Thus the e-folding time is e = b) The problem is ut (r, , z, t) = u(r, , z, t), 0 < r < a, 0 < < 2, < z < , u(r, , z, 0) = T, u(0, , z, t) is bounded, u(a, , z, t) = 0. The Laplacian in cylindrical coordinates is 1 1 u = urr + ur + 2 u + uzz . r r Because of symmetry, the solution does not depend on or z. 1 ut (r, t) = urr (r, t) + ur (r, t) , 0 < r < a, t > 0, r u(r, 0) = T, u(0, t) is bounded, u(a, t) = 0. We will solve this problem with an expansion in eigen-solutions of the partial dierential equation that satisfy the homogeneous boundary conditions at r = 0 and r = a. We substitute the separation of variables u(r, t) = 1787 t > 0, a2 . 2 z exp a a
2

t .

R(r)T (t) into the partial dierential equation. 1 RT = R T + R T r R R T = + = 2 T R rR We have the eigenvalue problem, 1 R + R + 2 R = 0, r Recall that the Bessel equation, 1 2 y + y + 2 2 x x y = 0, R(0) is bounded, R(a) = 0.

has the general solution y = c1 J (x) + c2 Y (x). We discard the Bessel function of the second kind, Y , as it is unbounded at the origin. The solution for R(r) is R(r) = J0 (r). Applying the boundary condition at r = a, we see that the eigenvalues and eigenfunctions are n = n , a Rn = J 0 n r a , n N,

where {n } are the positive roots of the Bessel function J0 . The dierential equation for T becomes, Tn = which has the solutions, Tn = exp n a
2

n a

Tn ,

t .

1788

The eigen-solutions of the partial dierential equation for u(r, t) are, un (r, t) = J0 n r a exp n a
2

t .

The solution u(r, t) is a linear combination of the eigen-solutions, un . The slowest decaying eigenfunction is, u1 (r, t) = J0 Thus the e-folding time is e = c) The problem is ut (r, , , t) = u(r, , , t), 0 < r < a, 0 < < 2, 0 < < , u(r, , , 0) = T, u(0, , , t) is bounded, u(a, , , t) = 0. The Laplacian in spherical coordinates is, 2 1 cos 1 u = urr + ur + 2 u + 2 u + 2 2 u . r r r sin r sin Because of symmetry, the solution does not depend on or . 2 ut (r, t) = urr (r, t) + ur (r, t) , 0 < r < a, t > 0, r u(r, 0) = T, u(0, t) is bounded, u(a, t) = 0 1789 t > 0, a2 . 2 1 1 r a exp 1 a
2

t .

We will solve this problem with an expansion in eigen-solutions of the partial dierential equation that satisfy the homogeneous boundary conditions at r = 0 and r = a. We substitute the separation of variables u(r, t) = R(r)T (t) into the partial dierential equation. 2 RT = R T + R T r T R 2R = + = 2 T R rR We have the eigenvalue problem, 2 R + R + 2 R = 0, r Recall that the equation, ( + 1) 2 y + y + 2 x x2 y = 0, R(0) is bounded, R(a) = 0.

has the general solution y = c1 j (x) + c2 y (x), where j and y are the spherical Bessel functions of the rst and second kind. We discard y as it is unbounded at the origin. (The spherical Bessel functions are related to the Bessel functions by j (x) = J+1/2 (x).) 2x The solution for R(r) is Rn = j0 (r). Applying the boundary condition at r = a, we see that the eigenvalues and eigenfunctions are n = The problem for T becomes Tn = 1790 n a
2

n , a

Rn = j0

n r , a Tn ,

n N.

which has the solutions, Tn = exp The eigen-solutions of the partial dierential equation are, un (r, t) = j0 The slowest decaying eigen-solution is, u1 (r, t) = j0 Thus the e-folding time is e = a2 . 2 1 1 r 1 exp a a
2

n a

t .

n r n exp a a

t .

t .

d) If the edges are perfectly insulated, then no heat escapes through the boundary. The temperature is constant for all time. There is no e-folding time. Solution 37.23 We will solve this problem with an eigenfunction expansion. Since the partial dierential equation is homogeneous, we will nd eigenfunctions in both x and y. We substitute the separation of variables u(x, y, t) = X(x)Y (y)T (t) into the partial dierential equation. XY T = (t) (X Y T + XY T ) X Y T = + = 2 (t)T X Y Y X = 2 = 2 X Y 1791

First we have a Sturm-Liouville eigenvalue problem for X, X = 2 X, which has the solutions, m = X (0) = X (a) = 0, m = 0, 1, 2, . . . .

m mx , Xm = cos , a a Now we have a Sturm-Liouville eigenvalue problem for Y , Y = 2 which has the solutions, mn = m a
2

m a

Y,

Y (0) = Y (b) = 0,

n b

Yn = sin

ny , b

m = 0, 1, 2, . . . ,

n = 1, 2, 3, . . . .

A few of the eigenfunctions, cos mx sin ny , are shown in Figure 37.3. a b The dierential equation for T becomes, Tmn = which has the solutions, m 2 n Tmn = exp + a b The eigen-solutions of the partial dierential equation are, umn = cos mx ny sin exp a b m a
2 0

m a

n b

(t)Tmn ,
t

( ) d

n b

2 0

( ) d

The solution of the partial dierential equation is,


u(x, y, t) =
m=0 n=1

cmn cos

mx ny sin exp a b 1792

m a

n b

2 0

( ) d

We determine the coecients from the initial condition.


u(x, y, 0) =
m=0 n=1

cmn cos 2 ab
a 0 0 a 0 b 0 b

mx ny sin a b n b

= f (x, y)

c0n = 4 = ab

f (x, y) sin

dy dx

cmn

f (x, y) cos

n m sin a b

dy dx

Solution 37.24 The steady state temperature satises Laplaces equation, u = 0. The Laplacian in cylindrical coordinates is, 1 1 u(r, , z) = urr + ur + 2 u + uzz . r r Because of the homogeneity in the z direction, we reduce the partial dierential equation to, 1 1 urr + ur + 2 u = 0, r r The boundary conditions are, u(r, 0) = u(r, ) = 0, u(0, ) = 0, u(1, ) = 1. 0 < r < 1, 0 < < .

We will solve this problem with an eigenfunction expansion. We substitute the separation of variables u(r, ) = R(r)T () into the partial dierential equation. 1 1 R T + R T + 2 RT = 0 r r R R T r2 +r = = 2 R R T 1793

We have the regular Sturm-Liouville eigenvalue problem, T = 2 T, which has the solutions, n = n, The problem for R becomes, r2 R + rR n2 R = 0, R(0) = 0. This is an Euler equation. We substitute R = r into the dierential equation to obtain, ( 1) + n2 = 0, = n. The general solution of the dierential equation for R is Rn = c1 rn + c2 rn . The solution that vanishes at r = 0 is Rn = crn . The eigen-solutions of the dierential equation are, un = rn sin(n). The solution of the partial dierential equation is

T (0) = T () = 0, n N.

Tn = sin(n),

u(r, ) =
n=1

an rn sin(n).

We determine the coecients from the boundary condition at r = 1.

u(1, ) =
n=1

an sin(n) = 1 2 (1 (1)n ) n

2 an =

sin(n) d =
0

1794

The solution of the partial dierential equation is u(r, ) = 4

rn sin(n).
n=1 odd n

Solution 37.25 The problem is uxx + uyy = 0, 0 < x, 0 < y < 1, u(x, 0) = u(x, 1) = 0, u(0, y) = f (y). We substitute the separation of variables u(x, y) = X(x)Y (y) into the partial dierential equation. X Y + XY = 0 X Y = = 2 X Y We have the regular Sturm-Liouville problem, Y = 2 Y, which has the solutions, n = n, The problem for X becomes, Xn = (n)2 X, which has the general solution, Xn = c1 enx +c2 enx . The solution that is bounded as x is, Xn = c enx . 1795 Yn = sin(ny), n N. Y (0) = Y (1) = 0,

The eigen-solutions of the partial dierential equation are, un = enx sin(ny), The solution of the partial dierential equation is,

n N.

u(x, y) =
n=1

an enx sin(ny).

We nd the coecients from the boundary condition at x = 0.

u(0, y) =
n=1

an sin(ny) = f (y)

an = 2
0

f (y) sin(ny) dy

Solution 37.26 The Laplacian in polar coordinates is 1 1 u urr + ur + 2 u . r r Since we have homogeneous boundary conditions at = 0 and = , we will solve this problem with an eigenfunction expansion. We substitute the separation of variables u(r, ) = R(r)() into Laplaces equation. 1 1 R + R + 2 R = 0 r r R R +r = = 2 . r2 R R 1796

We have a regular Sturm-Liouville eigenvalue problem for . = 2 , (0) = () = 0 n n n = , n = sin , n Z+ . We have Euler equations for Rn . We solve them with the substitution R = r . r2 Rn + rRn n
2

Rn = 0,

Rn (a) = 0

( 1) +

n 2 =0 n = n/ Rn = c 1 r + c2 rn/ .

The solution, (up to a multiplicative constant), that vanishes at r = a is Rn = rn/ a2n/ rn/ . Thus the series expansion of our solution is,

u(r, ) =
n=1

un rn/ a2n/ rn/ sin

We determine the coecients from the boundary condition at r = b.

u(b, ) =
n=1

un bn/ a2n/ bn/ sin

n n

= f () d

un =

2 (bn/ a2n/ bn/ )

f () sin
0

1797

Solution 37.27 a) The mathematical statement of the problem is utt = c2 uxx , 0 < x < L, t > 0, u(0, t) = u(L, t) = 0, u(x, 0) = 0, ut (x, 0) = v for |x | < d 0 for |x | > d.

Because we are interest in the harmonics of the motion, we will solve this problem with an eigenfunction expansion in x. We substitute the separation of variables u(x, t) = X(x)T (t) into the wave equation. XT = c2 X T X T = = 2 2T c X The eigenvalue problem for X is, X = 2 X, which has the solutions, n , L The ordinary dierential equation for the Tn are, n = X(0) = X(L) = 0, nx , L n N.

Xn = sin

Tn = which have the linearly independent solutions, cos nct L ,

nc L

Tn ,

sin

nct L

1798

The solution for u(x, t) is a linear combination of the eigen-solutions.

u(x, t) =
n=1

sin

nx L

an cos

nct L

+ bn sin

nct L

Since the string initially has zero displacement, each of the an are zero.

u(x, t) =
n=1

bn sin

nx sin L

nct L

Now we use the initial velocity to determine the coecients in the expansion. Because the position is a continuous function of x, and there is a jump discontinuity in the velocity as a function of x, the coecients in the expansion will decay as 1/n2 .

ut (x, 0) =
n=1

nx nc bn sin = L L
L

v for |x | < d 0 for |x | > d. nx L dx

nc 2 bn = L L

ut (x, 0) sin
0

bn =

+d 2 nx v sin dx nc d L 4Lv nd n = 2 2 sin sin n c L L

The solution for u(x, t) is, 4Lv u(x, t) = 2 c

n=1

1 sin n2

nd L

sin

n L

sin

nx sin L

nct L

1799

b) The form of the solution is again,

u(x, t) =
n=1

bn sin

nx sin L

nct L

We determine the coecients in the expansion from the initial velocity.

ut (x, 0) =
n=1

nx nc bn sin = L L nc 2 bn = L L 2 nc
+d L

v cos 0

(x) 2d

for |x | < d for |x | > d.

ut (x, 0) sin
0

nx L

dx

bn = bn = The solution for u(x, t) is, 8dL2 v u(x, t) = 2c u(x, t) = v 2c


v cos
d

(x ) 2d
n L

sin

nx L

dx
L , 2n L 2n

8dL2 v cos nd sin n 2 c(L2 4d2 n2 ) L v 2nd + L sin 2nd n2 2 c L

for d =
n L

sin

for d =

n=1

n(L2

1 cos 4d2 n2 )

nd L

sin

n L n L

sin

nx sin L nx sin L

nct L nct L

for d =

L , 2n L . 2n

n=1

1 n2

2nd + L sin

2nd L

sin

sin

for d =

c) The kinetic energy of the string is E= 1 2


L

(ut (x, t))2 dx,


0

1800

where is the density of the string per unit length. Flat Hammer. The nth harmonic is un = 4Lv sin n2 2 c nd L sin n L sin nx sin L nct L .

The kinetic energy of the nth harmonic is En = This will be maximized if n = 1, L (2m 1) n = , m = 1, . . . , n, L 2 (2m 1)L = , m = 1, . . . , n 2n sin2 We note that the kinetic energies of the nth harmonic decay as 1/n2 . L Curved Hammer. We assume that d = 2n . The nth harmonic is un = 8dL2 v cos n 2 c(L2 4d2 n2 ) nd L sin n L sin nx sin L nct L . 2
L 0

un t

dx =

4Lv 2 sin2 n2 2

nd L

sin2

n L

cos2

nct L

The kinetic energy of the nth harmonic is En = 2


L 0

un t

dx =

16d2 L3 v 2 cos2 2 (L2 4d2 n2 )2 1801

nd L

sin2

n L

cos2

nct L

This will be maximized if sin2 = n L = 1, m = 1, . . . , n

(2m 1)L , 2n

We note that the kinetic energies of the nth harmonic decay as 1/n4 . Solution 37.28 In mathematical notation, the problem is utt c2 uxx = s(x, t), 0 < x < L, u(0, t) = u(L, t) = 0, u(x, 0) = ut (x, 0) = 0. t > 0,

Since this is an inhomogeneous partial dierential equation, we will expand the solution in a series of eigenfunctions in x for which the coecients are functions of t. The solution for u has the form,

u(x, t) =
n=1

un (t) sin

nx . L

Substituting this expression into the inhomogeneous partial dierential equation will give us ordinary dierential equations for each of the un . n 2 nx un + c2 un sin = s(x, t). L L n=1 We expand the right side in a series of the eigenfunctions.

s(x, t) =
n=1

sn (t) sin

nx . L

1802

For 0 < t < we have sn (t) = 2 L 2 = L nx dx L 0 L (x ) t nx v cos sin sin dx 2d L 0 nd n t 8dLv = cos sin sin . 2 4d2 n2 ) (L L L s(x, t) sin
L

For t > , sn (t) = 0. Substituting this into the partial dierential equation yields, nc un + L
2

un =

8dLv (L2 4d2 n2 )

cos

nd L

sin

n L

sin

, for t < , for t > .

Since the initial position and velocity of the string is zero, we have un (0) = un (0) = 0. First we solve the dierential equation on the range 0 < t < . The homogeneous solutions are cos nct L , sin nct L .

Since the right side of the ordinary dierential equation is a constant times sin(t/), which is an eigenfunction of the dierential operator, we can guess the form of a particular solution, pn (t). pn (t) = d sin t

We substitute this into the ordinary dierential equation to determine the multiplicative constant d. pn (t) = 8d 2 L3 v cos 3 (L2 c2 2 n2 )(L2 4d2 n2 ) 1803 nd L sin n L sin t

The general solution for un (t) is un (t) = a cos nct L + b sin nct L 8d 2 L3 v 3 2 cos (L c2 2 n2 )(L2 4d2 n2 ) nd L sin n L sin t .

We use the initial conditions to determine the constants a and b. The solution for 0 < t < is un (t) = 8d 2 L3 v cos 3 (L2 c2 2 n2 )(L2 4d2 n2 ) nd L sin n L L sin cn nct L sin t .

The solution for t > , the solution is a linear combination of the homogeneous solutions. This linear combination is determined by the position and velocity at t = . We use the above solution to determine these quantities. 8d 2 L4 v nd n nc cos sin sin 3 cn(L2 c2 2 n2 )(L2 4d2 n2 ) L L L 2 3 nd n nc 8d L v cos sin 1 + cos un () = 2 (L2 c2 2 n2 )(L2 4d2 n2 ) L L L un () = The fundamental set of solutions at t = is cos nc(t ) L , L sin nc nc(t ) L

From the initial conditions at t = , we see that the solution for t > is

un (t) =

8d 2 L3 v nd n cos sin 3 (L2 c2 2 n2 )(L2 4d2 n2 ) L L L nc nc(t ) nc sin cos + 1 + cos sin cn L L L

nc(t ) L

1804

Width of the Hammer. The nth harmonic has the width dependent factor, d cos L2 4d2 n2 nd L .

Dierentiating this expression and trying to nd zeros to determine extrema would give us an equation with both algebraic and transcendental terms. Thus we dont attempt to nd the maxima exactly. We know that d < L. The cosine factor is large when nd m, m = 1, 2, . . . , n 1, L mL d , m = 1, 2, . . . , n 1. n Substituting d = mL/n into the width dependent factor gives us d (1)m . 4m2 )

L2 (1

Thus we see that the amplitude of the nth harmonic and hence its kinetic energy will be maximized for d L n

The cosine term in the width dependent factor vanishes when d= (2m 1)L , 2n m = 1, 2, . . . , n.

The kinetic energy of the nth harmonic is minimized for these widths. L For the lower harmonics, n , the kinetic energy is proportional to d2 ; for the higher harmonics, n 2d kinetic energy is proportional to 1/d2 . 1805

L , 2d

the

Duration of the Blow. The nth harmonic has the duration dependent factor, 2 L2 n 2 c 2 2 L sin nc nc L cos nc(t ) L L sin nc and 1 + cos + 1 + cos nc L sin nc(t ) L .

If we assume that is small, then nc L nc L . 2 .

Thus the duration dependent factor is about, L2 sin n2 c2 2 nc(t ) L .

L Thus for the lower harmonics, (those satisfying n ), the amplitude is proportional to , which means that the c L kinetic energy is proportional to 2 . For the higher harmonics, (those with n ), the amplitude is proportional to c 2 1/, which means that the kinetic energy is proportional to 1/ .

Solution 37.29 Substituting u(x, y, z, t) = v(x, y, z) et into the wave equation will give us a Helmholtz equation. 2 v et c2 (vxx + vyy + vzz ) et = 0 vxx + vyy + vzz + k 2 v = 0. We nd the propagating modes with separation of variables. We substitute v = X(x)Y (y)Z(z) into the Helmholtz equation. X Y Z + XY Z + XY Z + k 2 XY Z = 0 X Y Z = + + k2 = 2 X Y Z 1806

The eigenvalue problem in x is X = 2 X, which has the solutions, n = We continue with the separation of variables. The eigenvalue problem in y is Y = 2 Y, which has the solutions, m , L Now we have an ordinary dierential equation for Z, n = Z + k2 We dene the eigenvalues, 2 = k 2 n,m If k 2
2 L

X(0) = X(L) = 0, Xn = sin nx . L

n , L

Y Z n = + k2 Y Z L

= 2

Y (0) = Y (L) = 0, Ym = sin my . L

n2 + m2

Z = 0.

n2 + m2 .

(n2 + m2 ) < 0, then the solutions for Z are, exp L


2

(n2 + m2 ) k 2 z

We discard this case, as the solutions are not bounded as z . 1807

If k 2

2 L

(n2 + m2 ) = 0, then the solutions for Z are, {1, z}

The solution Z = 1 satises the boundedness and nonzero condition at innity. This corresponds to a standing wave. 2 If k 2 L (n2 + m2 ) > 0, then the solutions for Z are, en,m z . These satisfy the boundedness and nonzero conditions at innity. For values of n, m satisfying k 2 there are the propagating modes, un,m = sin Solution 37.30 utt = c2 u, 0 < x < a, 0 < y < b, u(0, y) = u(a, y) = u(x, 0) = u(x, b) = 0. We substitute the separation of variables u(x, y, t) = X(x)Y (y)T (t) into Equation 37.12. T X Y = + = 2T c X Y X Y = = X Y This gives us dierential equations for X(x), Y (y) and T (t). X = X, X(0) = X(a) = 0 Y = ( )Y, Y (0) = Y (b) = 0 T = c2 T 1808 (37.12) my (tn,m z) nx e sin . L L
2 L

(n2 + m2 ) 0,

First we solve the problem for X. m = Then we solve the problem for Y . m,n = Finally we determine T . Tm,n = The modes of oscillation are um,n = sin The frequencies are m 2 n 2 + . a b Figure 37.4 shows a few of the modes of oscillation in surface and density plots. m,n = c Solution 37.31 We substitute the separation of variables = X(x)Y (y)T (t) into the dierential equation. t = a2 (xx + yy ) XY T = a2 (X Y T + XY T ) T X Y = + = 2T a X Y X Y T = , = = 2T a X Y 1809 (37.13) ny cos mx sin c sin a b m a
2

m a
2

Xm = sin

mx a ny b

m a

n b

Ym,n = sin

cos c sin

m a

n b

n b

t .

First we solve the eigenvalue problem for X. X + X = 0, m = m lx


2

X(0) = X(lx ) = 0 mx lx , m Z+

Xm (x) = sin

Then we solve the eigenvalue problem for Y . Y + ( m )Y = 0, mn = m + n ly


2

Y (0) = Y (ly ) = 0 ny ly , n Z0+

Ymn (y) = cos

Next we solve the dierential equation for T , (up to a multiplicative constant). T = a2 mn T T (t) = exp a2 mn t The eigensolutions of Equation 37.13 are sin(m x) cos(mn y) exp a2 mn t , m Z+ , n Z0+ .

We choose the eigensolutions mn to be orthonormal on the xy domain at t = 0. m0 (x, y, t) = mn (x, y, t) = 2 sin(m x) exp a2 mn t , lx ly m Z+ m Z+ , n Z+

2 sin(m x) cos(mn y) exp a2 mn t , lx ly

The solution of Equation 37.13 is a linear combination of the eigensolutions. (x, y, t) =


m=1 n=0

cmn mn (x, y, t)

1810

We determine the coecients from the initial condition. (x, y, 0) = 1

cmn mn (x, y, 0) = 1
m=1 n=0 lx ly

cmn =
0 0 lx 0

mn (x, y, 0) dy dx 2 lx ly
ly

cm0 = cm0 = cmn = 2 lx ly

sin(m x) dy dx
0

1 (1)m 2lx ly , m
lx 0 0 ly

m Z+

sin(m x) cos(mn y) dy dx m Z+ , n Z+

cmn = 0, (x, y, t) =

cm0 m0 (x, y, t)
m=1

(x, y, t) =
m=1 odd m

2 2lx ly sin(m x) exp a2 mn t m

Addendum. Note that an equivalent problem to the one specied is t = a2 (xx + yy ) , 0 < x < lx , < y < , (x, y, 0) = 1, (0, y, t) = (ly , y, t) = 0. Here we have done an even periodic continuation of the problem in the y variable. Thus the boundary conditions y (x, 0, t) = y (x, ly , t) = 0 1811

are automatically satised. Note that this problem does not depend on y. Thus we only had to solve t = a2 xx , 0 < x < lx (x, 0) = 1, (0, t) = (ly , t) = 0. Solution 37.32 1. Since the initial and boundary conditions do not depend on , neither does . We apply the separation of variables = u(r)T (t). t = a2 1 t = a2 (rr )r r T 1 = (ru ) = a2 T r We solve the eigenvalue problem for u(r). (ru ) + u = 0, First we write the general solution. u(r) = c1 J0 r + c2 Y0 r u(0) bounded, u(R) = 0 (37.14) (37.15) (37.16)

The Bessel function of the second kind, Y0 , is not bounded at r = 0, so c2 = 0. We use the boundary condition at r = R to determine the eigenvalues. j0,n R
2

n =

un (r) = cJ0

j0,n r R

1812

We choose the constant c so that the eigenfunctions are orthonormal with respect to the weighting function r. J0 un (r) =
R 0 j0,n r R 2 rJ0 j0,n r R

2 J0 RJ1 (j0,n )

j0,n r R

Now we solve the dierential equation for T . T = a2 n T Tn = exp aj0,n R2


2

The eigensolutions of Equation 37.14 are 2 J0 RJ1 (j0,n ) j0,n r R aj0,n R2


2

n (r, t) =

exp

The solution is a linear combination of the eigensolutions. 2 = cn J0 RJ1 (j0,n ) n=1

j0,n r R

exp

aj0,n R2

1813

We determine the coecients from the initial condition. (r, , 0) = V j0,n r 2 cn J0 =V RJ1 (j0,n ) R n=1 R j0,n r 2 J0 dr cn = Vr RJ1 (j0,n ) R 0 2 R cn = V J1 (j0,n ) RJ1 (j0,n ) j0,n /R 2 VR cn = j0,n

J0

(r, , t) = 2V
n=1

j0,n r R

j0,n J1 (j0,n )

exp

aj0,n R2

2. J (r) 2 cos r , r 2 4 1 j,n n + 2 4


2R j0,n r j0,n r R 4 3 4

r +

For large n, the terms in the series solution at t = 0 are J0


j0,n r R

cos

j0,n J1 (j0,n )

j0,n

2 j0,n

cos j0,n

cos (n1/4)r R 4 R . r(n 1/4) cos ((n 1)) 1814

The coecients decay as 1/n. Solution 37.33 1. We substitute the separation of variables = T (t)()() into Equation 37.7 T = 1 1 (sin T ) + T sin sin2 1 R2 T 1 = (sin ) + = 2T a sin sin2 sin (sin ) + sin2 = = 1 (sin ) + = 0, sin sin2 (0) = (2), The eigenvalues and eigenfunctions for are n = n2 , Now we deal with the equation for . x = cos , () = P (x), sin2 = 1 x2 , d 1 d = dx sin d 1 n = en , 2 n Z. (0) = (2). a2 R2

We have dierential equations for each of T , and . T = a2 T, R2 + = 0

2. In order that the solution be continuously dierentiable, we need the periodic boundary conditions

1 1 (sin2 ) + =0 sin sin sin2 n2 1 x2 P + P =0 1 x2 P (x) should be bounded at the endpoints, x = 1 and x = 1. 1815

3. If the solution does not depend on , then the only one of the n that will appear in the solution is 0 = 1/ 2. The equations for T and P become 1 x2 P + P = 0, P (1) bounded, a2 T = 2 T. R

The solutions for P are the Legendre polynomials. l = l(l + 1), We solve the dierential equation for T . a2 T R2 a2 l(l + 1) Tl = exp t R2 T = l(l + 1) The eigensolutions of the partial dierential equation are l = Pl (cos ) exp The solution is a linear combination of the eigensolutions.

Pl (cos ),

l Z0+

a2 l(l + 1) t . R2

=
l=0

Al Pl (cos ) exp

a2 l(l + 1) t R2

1816

4. We determine the coecients in the expansion from the initial condition. (, 0) = 2 cos2 1

Al Pl (cos ) = 2 cos2 1
l=0

A0 + A1 cos + A2

3 1 cos2 + = 2 cos2 1 2 2 4 1 A0 = , A1 = 0, A2 = , A3 = A4 = = 0 3 3 1 4 6a2 (, t) = P0 (cos ) + P2 (cos ) exp 2 t 3 3 R 1 2 (, t) = + 2 cos2 3 3 exp 6a2 t R2

Solution 37.34 Since we have homogeneous boundary conditions at x = 0 and x = 1, we will expand the solution in a series of eigenfunctions in x. We determine a suitable set of eigenfunctions with the separation of variables, = X(x)Y (y). xx + yy = 0 X Y = = X Y We have dierential equations for X and Y . X + X = 0, X(0) = X(1) = 0 Y Y = 0, Y (0) = 0 The eigenvalues and orthonormal eigenfunctions for X are n = (n)2 , Xn (x) = 2 sin(nx), n Z+ . (37.17)

1817

The solutions for Y are, (up to a multiplicative constant), Yn (y) = sinh(ny). The solution of Equation 37.17 is a linear combination of the eigensolutions.

(x, y) =
n=1

an 2 sin(nx) sinh(ny)

We determine the coecients from the boundary condition at y = 2.

x(1 x) =
n=1

an 2 sin(nx) sinh(n2)
1

an sinh(2n) =

2 x(1 x) sin(nx) dx 0 2 2(1 (1)n ) an = 3 3 n sinh(2n)

8 (x, y) = 3

n=1 odd n

1 sinh(ny) sin(nx) 3 n sinh(2n)

The solution at x = 1/2, y = 1 is 8 (1/2, 1) = 3

n=1 odd n

1 sinh(n) . n3 sinh(2n)

1818

Let Rk be the relative error at that point incurred by taking k terms.


1 sinh(n) n=k+2 n3 sinh(2n) odd n Rk = 8 1 sinh(n) 3 n=1 n3 sinh(2n) odd n 1 sinh(n) n=k+2 n3 sinh(2n) odd n Rk = 1 sinh(n) n=1 n3 sinh(2n) odd n

83

Since R1 0.0000693169 we see that one term is sucient for 1% or 0.1% accuracy. Now consider x (1/2, 1). 8 x (x, y) = 2

n=1 odd n

1 sinh(ny) cos(nx) n2 sinh(2n)

x (1/2, 1) = 0 Since all the terms in the series are zero, accuracy is not an issue. Solution 37.35 The solution has the form =
m rn1 Pn (cos ) sin(m), r > a m rn Pn (cos ) sin(m), r < a.

The boundary condition on at r = a gives us the constraint an1 an = 0 = a2n1 . 1819

Then we apply the boundary condition on r at r = a. (n + 1)an2 na2n1 an1 = 1 an+2 = 2n + 1 =


a m 2n+1 rn1 Pn (cos ) sin(m), r > a n+1 m r<a a2n+1 rn Pn (cos ) sin(m),
n+2

Solution 37.36 We expand the solution in a Fourier series. 1 = a0 (r) + an (r) cos(n) + bn (r) sin(n) 2 n=1 n=1 We substitute the series into the Laplaces equation to determine ordinary dierential equations for the coecients. r r r + 1 2 =0 r2 2

1 1 1 a0 + a0 = 0, an + an n2 an = 0, bn + bn n2 bn = 0 r r r The solutions that are bounded at r = 0 are, (to within multiplicative constants), a0 (r) = 1, Thus (r, ) has the form 1 (r, ) = c0 + cn rn cos(n) + dn rn sin(n) 2 n=1 n=1 We apply the boundary condition at r = R.

an (r) = rn ,

bn (r) = rn .

r (R, ) =
n=1

ncn R

n1

cos(n) +
n=1

ndn Rn1 sin(n)

1820

In order that r (R, ) have a Fourier series of this form, it is necessary that
2

r (R, ) d = 0.
0

In that case c0 is arbitrary in our solution. The coecients are cn = 1 nRn1


2

r (R, ) cos(n) d,
0

dn =

1 nRn1

r (R, ) sin(n) d.
0

We substitute the coecients into our series solution to determine it up to the additive constant. (r, ) = R

R (r, ) = R (r, ) = R (r, ) = R (r, ) = (r, ) =

n=1 2 0 2

1 r n R

n 0

r (R, ) cos(n( )) d

r (R, )
n=1

1 r n R
r 0 r

cos(n( )) d en() d d d

r (R, )
0 2 n=1

n1 d Rn

r (R, )
0 2 0

1
r

n=1

n n() e d Rn

r (R, )
0 0 2

1 R e() d 1 R e()

r () e d R 0 R 2 r (r, ) = r (R, ) ln 1 e() d 0 R r (R, ) ln 1 R 2


2

(r, ) =

r (R, ) ln 1 2
0

r r2 cos( ) + 2 R R

1821

Solution 37.37 We will assume that both and are nonzero. The cases of real and pure imaginary have already been covered. We solve the ordinary dierential equations, (up to a multiplicative constant), to nd special solutions of the diusion equation. X ( + )2 = X a2 + T = exp ( + )2 t , X = exp x a T = exp 2 2 t + 2t , X = exp x x a a = exp 2 2 t x + 2t x a a T = ( + )2 , T We take the sum and dierence of these solutions to obtain = exp 2 2 t cos x 2t x sin a a

1822

1823 Figure 37.2: The Normal Modes u01 through u34

m=0, n=1

m=0, n=2

m=0, n=3

m=1, n=1

m=1, n=2

m=1, n=3

m=2, n=1

m=2, n=2

m=2, n=3

Figure 37.3: The eigenfunctions cos

mx a

sin

ny b

1824

m=1,n=1

m=1,n=2

m=1,n=3

m=1,n=1 m=1,n=2 m=1,n=3

m=2,n=1

m=2,n=2

m=2,n=3

m=2,n=1 m=2,n=2 m=2,n=3

m=3,n=1

m=3,n=2

m=3,n=3

m=3,n=1 m=3,n=2 m=3,n=3

Figure 37.4: The modes of oscillation of a rectangular drum head.

1825

Chapter 38 Finite Transforms


Example 38.0.1 Consider the problem 1 2u u 2 2 = (x )(y ) et c t with uy (x, 0, t) = uy (x, b, t) = 0. Substituting u(x, y, t) = v(x, y) et into the partial dierential equation yields the problem v + k 2 v = (x )(y ) on < x < , 0 < y < b, with vy (x, 0) = vy (x, b) = 0. We assume that the solution has the form ny 1 v(x, y) = c0 (x) + cn (x) cos , 2 b n=1 1826

on < x < , 0 < y < b,

(38.1)

and apply a nite cosine transform in the y direction. Integrating from 0 to b yields
b b

vxx + vyy + k v dy =
0 b 0 b 0

(x )(y ) dy,

vy

+
0 b

vxx + k 2 v dy = (x ), vxx + k 2 v dy = (x ).

Substituting in Equation 38.1 and using the orthogonality of the cosines gives us 2 c0 (x) + k 2 c0 (x) = (x ). b Multiplying by cos(ny/b) and integrating form 0 to b yields
b 0

ny vxx + vyy + k v cos b


2

dy =
0

(x )(y ) cos

ny b

dy.

The vyy term becomes


b 0

ny vyy cos b

b ny b n ny dy = vy cos vy sin dy b b b 0 0 b n ny b n 2 ny = v sin v cos dy. b b b b 0 0

The right-hand-side becomes


b

(x )(y ) cos
0

ny b 1827

dy = (x ) cos

n . b

Thus the partial dierential equation becomes


b

vxx
0

n b

v + k 2 v cos

ny b

dy = (x ) cos

n . b

Substituting in Equation 38.1 and using the orthogonality of the cosines gives us cn (x) + k 2 n b
2

2 n cn (x) = (x ) cos . b b

Now we need to solve for the coecients in the expansion of v(x, y). The homogeneous solutions for c0 (x) are ekx . The solution for u(x, y, t) must satisfy the radiation condition. The waves at x = travel to the left and the waves at x = + travel to the right. The two solutions of that will satisfy these conditions are, respectively, y1 = ekx , y2 = ekx .

The Wronskian of these two solutions is 2k. Thus the solution for c0 (x) is c0 (x) = ekx< ekx> bk

We need to consider three cases for the equation for cn . k > n/b Let = k 2 (n/b)2 . The homogeneous solutions that satisfy the radiation condition are y1 = ex , y2 = ex .

The Wronskian of the two solutions is 2. Thus the solution is cn (x) = In the case that cos
n b

ex< ex> n cos . b b

= 0 this reduces to the trivial solution. 1828

k = n/b The homogeneous solutions that are bounded at innity are y1 = 1, y2 = 1.

If the right-hand-side is nonzero there is no way to combine these solutions to satisfy both the continuity and the derivative jump conditions. Thus if cos n = 0 there is no bounded solution. If cos n = 0 then the b b solution is not unique. cn (x) = const. k < n/b Let = (n/b)2 k 2 . The homogeneous solutions that are bounded at innity are y1 = ex , y2 = ex .

The Wronskian of these solutions is 2. Thus the solution is cn (x) = In the case that cos
n b

ex< ex> n cos b b

= 0 this reduces to the trivial solution.

1829

38.1

Exercises

Exercise 38.1 A slab is perfectly insulated at the surface x = 0 and has a specied time varying temperature f (t) at the surface x = L. Initially the temperature is zero. Find the temperature u(x, t) if the heat conductivity in the slab is = 1. Exercise 38.2 Solve uxx + uyy = 0, 0 < x < L, y > 0, u(x, 0) = f (x), u(0, y) = g(y), u(L, y) = h(y), with an eigenfunction expansion.

1830

38.2
Hint 38.1 Hint 38.2

Hints

1831

38.3

Solutions

Solution 38.1 The problem is ut = uxx , 0 < x < L, t > 0, ux (0, t) = 0, u(L, t) = f (t), u(x, 0) = 0. We will solve this problem with an eigenfunction expansion. We nd these eigenfunction by replacing the inhomogeneous boundary condition with the homogeneous one, u(L, t) = 0. We substitute the separation of variables v(x, t) = X(x)T (t) into the homogeneous partial dierential equation. XT = X T X T = = 2 . T X This gives us the regular Sturm-Liouville eigenvalue problem, X = 2 X, which has the solutions, n = (2n 1) , 2L Xn = cos (2n 1)x 2L , n N. X (0) = X(L) = 0,

Our solution for u(x, t) will be an eigenfunction expansion in these eigenfunctions. Since the inhomogeneous boundary condition is a function of t, the coecients will be functions of t.

u(x, t) =
n=1

an (t) cos(n x)

Since u(x, t) does not satisfy the homogeneous boundary conditions of the eigenfunctions, the series is not uniformly convergent and we are not allowed to dierentiate it with respect to x. We substitute the expansion into the partial 1832

dierential equation, multiply by the eigenfunction and integrate from x = 0 to x = L. We use integration by parts to move derivatives from u to the eigenfunctions. ut = uxx
L L

ut cos(m x) dx =
0 L 0 0

uxx cos(m x) dx
L

an (t) cos(n x) cos(m x) dx = [ux cos(m x)]L + 0


n=1 0

ux m sin(m x) dx

L am (t) = [um sin(m x)]L 0 2 L am (t) = m u(L, t) sin(m L) 2 m 2


L 0

u2 cos(m x) dx m
0

an (t) cos(n x) cos(m x) dx


n=1

L L am (t) = m (1)n f (t) 2 am (t) m 2 2 2 n am (t) + m am (t) = (1) m f (t) From the initial condition u(x, 0) = 0 we see that am (0) = 0. Thus we have a rst order dierential equation and an initial condition for each of the am (t). am (t) + 2 am (t) = (1)n m f (t), m This equation has the solution,
t
2

am (0) = 0

am (t) = (1)n m
0

em (t ) f ( ) d.

Solution 38.2 uxx + uyy = 0, 0 < x < L, y > 0, u(x, 0) = f (x), u(0, y) = g(y), u(L, y) = h(y), 1833

We seek a solution of the form,

u(x, y) =
n=1

un (y) sin

nx . L

Since we have inhomogeneous boundary conditions at x = 0, L, we cannot dierentiate the series representation with respect to x. We multiply Laplaces equation by the eigenfunction and integrate from x = 0 to x = L.
L

(uxx + uyy ) sin


0

mx L

dx = 0

We use integration by parts to move derivatives from u to the eigenfunctions. mx L mx L m L ux cos dx + um (y) = 0 L L 0 L 2 0 L mx L m 2 mx L m u cos u sin dx + um (y) = 0 L L L L 2 0 0 m L m 2 L m h(y)(1)m + g(y) um (y) + um (y) = 0 L L 2 L 2 m 2 m um (y) um (y) = 2m ((1) h(y) g(y)) L ux sin Now we have an ordinary dierential equation for the un (y). In order that the solution is bounded, we require that each un (y) is bounded as y . We use the boundary condition u(x, 0) = f (x) to determine boundary conditions for the um (y) at y = 0.

u(x, 0) =
n=1

un (0) sin 2 L
L

nx = f (x) L nx L dx

un (0) = fn

f (x) sin
0

1834

Thus we have the problems, un (y) n L


2

un (y) = 2n ((1)n h(y) g(y)) ,

un (0) = fn ,

un (+) bounded,

for the coecients in the expansion. We will solve these with Green functions. Consider the associated Green function problem n 2 Gn (y; ) = (y ), Gn (0; ) = 0, Gn (+; ) bounded. Gn (y; ) L The homogeneous solutions that satisfy the boundary conditions are sinh respectively. The Wronskian of these solutions is sinh ny L n sinh ny L L Thus the Green function is n 2ny/L eny/L e . n ny/L = L e L ny L and eny/L ,

L sinh ny< eny> /L L Gn (y; ) = . n e2n/L Using the Green function we determine the un (y) and thus the solution of Laplaces equation.

un (y) = fn e

ny/L

+2n
0

Gn (y; ) ((1)n h() g()) d un (y) sin


n=1

u(x, y) =

nx . L

1835

Chapter 39 The Diusion Equation

1836

39.1

Exercises

Exercise 39.1 Is the solution of the Cauchy problem for the heat equation unique? ut uxx = q(x, t), < x < , u(x, 0) = f (x) t>0

Exercise 39.2 Consider the heat equation with a time-independent source term and inhomogeneous boundary conditions. ut = uxx + q(x) u(0, t) = a, u(h, t) = b, u(x, 0) = f (x) Exercise 39.3 Is the Cauchy problem for the backward heat equation ut + uxx = 0, well posed? Exercise 39.4 Derive the heat equation for a general 3 dimensional body, with non-uniform density (x), specic heat c(x), and conductivity k(x). Show that u(x, t) 1 = (k u(x, t)) t c where u is the temperature, and you may assume there are no internal sources or sinks. Exercise 39.5 Verify Duhamels Principal: If u(x, t, ) is the solution of the initial value problem: ut = uxx , u(x, 0, ) = f (x, ), 1837 u(x, 0) = f (x) (39.1)

then the solution of wt = wxx + f (x, t), is w(x, t) =


0 t

w(x, 0) = 0

u(x, t , ) d.

Exercise 39.6 Modify the derivation of the diusion equation t = a2 xx , a2 = k , c (39.2)

so that it is valid for diusion in a non-homogeneous medium for which c and k are functions of x and and so that it is valid for a geometry in which A is a function of x. Show that Equation (39.2) above is in this case replaced by cAt = (kAx )x . Recall that c is the specic heat, k is the thermal conductivity, is the density, is the temperature and A is the cross-sectional area.

1838

39.2
Hint 39.1 Hint 39.2 Hint 39.3 Hint 39.4

Hints

Hint 39.5 Check that the expression for w(x, t) satises the partial dierential equation and initial condition. Recall that x Hint 39.6
x x

h(x, ) d =
a a

hx (x, ) d + h(x, x).

1839

39.3

Solutions

Solution 39.1 Let u and v both be solutions of the Cauchy problem for the heat equation. Let w be the dierence of these solutions. w satises the problem wt wxx = 0, < x < , w(x, 0) = 0. We can solve this problem with the Fourier transform. wt + 2 w = 0, w(, 0) = 0 w=0 w=0 Since u v = 0, we conclude that the solution of the Cauchy problem for the heat equation is unique. Solution 39.2 Let (x) be the equilibrium temperature. It satises an ordinary dierential equation boundary value problem. = q(x) , (0) = a, (h) = b t > 0,

To solve this boundary value problem we nd a particular solution p that satises homogeneous boundary conditions and then add on a homogeneous solution h that satises the inhomogeneous boundary conditions. p = q(x) , p (0) = p (h) = 0 h = 0, h (0) = a, h (h) = b

We nd the particular solution p with the method of Green functions. G = (x ), G(0|) = G(h|) = 0. 1840

We nd homogeneous solutions which respectively satisfy the left and right homogeneous boundary conditions. y1 = x, y2 = h x

Then we compute the Wronskian of these solutions and write down the Green function. x hx = h 1 1 1 G(x|) = x< (h x> ) h The homogeneous solution that satises the inhomogeneous boundary conditions is W = h = a + Now we have the equilibrium temperature.
h ba 1 q() x+ x< (h x> ) d h h 0 h hx x x ba =a+ x+ q() d + (h )q() d h h 0 h x

ba x h

=a+

Let v denote the deviation from the equilibrium temperature. u=+v v satises a heat equation with homogeneous boundary conditions and no source term. vt = vxx , v(0, t) = v(h, t) = 0, v(x, 0) = f (x) (x)

We solve the problem for v with separation of variables. v = X(x)T (t) XT = X T T X = = T X 1841

We have a regular Sturm-Liouville problem for X and a dierential equation for T . X + X = 0, X(0) = X() = 0 n 2 nx n = , Xn = sin , n Z+ h h T = T n 2 t Tn = exp h v is a linear combination of the eigensolutions.

v=
n=1

vn sin

nx n exp h h

The coecients are determined from the initial condition, v(x, 0) = f (x) (x). 2 vn = h
h

(f (x) (x)) sin


0

nx h

dx

We have determined the solution of the original problem in terms of the equilibrium temperature and the deviation from the equilibrium. u = + v. Solution 39.3 A problem is well posed if there exists a unique solution that depends continiously on the nonhomogeneous data. First we nd some solutions of the dierential equation with the separation of variables u = X(x)T (t). ut + uxx = 0, > 0 XT + X T = 0 T X = = T X X + X = 0, T = T u = cos x et , u = sin x et

1842

Note that u = cos satises the Cauchy problem ut + uxx = 0,

x et x

u(x, 0) = cos

Consider 1. The initial condition is small, it satises |u(x, 0)| < . However the solution for any positive time can be made arbitrarily large by choosing a suciently large, positive value of . We can make the solution exceed the value M at time t by choosing a value of such that et > M M 1 ln > t

Thus we see that Equation 39.1 is ill posed because the solution does not depend continuously on the initial data. A small change in the initial condition can produce an arbitrarily large change in the solution for any xed time. Solution 39.4 Consider a Region of material, R. Let u be the temperature and be the heat ux. The amount of heat energy in the region is cu dx.
R

We equate the rate of change of heat energy in the region with the heat ux across the boundary of the region. d dt d dt
R

cu dx =
R R

n ds

We apply the divergence theorem to change the surface integral to a volume integral. cu dx =
R R

dx dx = 0

u + t

1843

Since the region is arbitrary, the integral must vanish identically. c u = t

We apply Fouriers law of heat conduction, = k u, to obtain the heat equation. u 1 = t c (k u)

Solution 39.5 We verify Duhamels principal by showing that the integral expression for w(x, t) satises the partial dierential equation and the initial condition. Clearly the initial condition is satised.
0

w(x, 0) =
0

u(x, 0 , ) d = 0

Now we substitute the expression for w(x, t) into the partial dierential equation. t
t 0

2 u(x, t , ) d = 2 x
t 0 t

u(x, t , ) d + f (x, t)
0 t

u(x, t t, t) + f (x, t) +
0 t

ut (x, t , ) d =
0 t

uxx (x, t , ) d + f (x, t) uxx (x, t , ) d + f (x, t)

ut (x, t , ) d =
0

(ut (x, t , ) d uxx (x, t , )) d


0

Since ut (x, t , ) d uxx (x, t , ) = 0, this equation is an identity. 1844

Solution 39.6 We equate the rate of change of thermal energy in the segment ( . . . ) with the heat entering the segment through the endpoints.

t cA dx = k(, ())A()x (, t) k(, ())A()x (, t)


t cA dx = [kAx ]

t cA dx =

(kAx )x dx

cAt (kAx )x dx = 0

Since the domain is arbitrary, we conclude that cAt = (kAx )x .

1845

Chapter 40 Laplaces Equation


40.1 Introduction
u = 0 where 2 2 + + 2 . x2 xn 1 The inhomogeneous analog is called Poissons Equation. = u = f (x) CONTINUE

Laplaces equation in n dimensions is

40.2

Fundamental Solution
G = (x ). 1846

The fundamental solution of Poissons equation in Rn satises

40.2.1

Two Dimensional Space


2 2 + 2 x2 y

If n = 2 then the fundamental solution satises G = (x )(y ).

Since the product of delta functions, (x )(y ) is circularly symmetric about the point (, ), we look for a solution in the form u(x, y) = v(r) where r = ((x )2 + (y )2 ). CONTINUE

1847

40.3

Exercises

Exercise 40.1 Is the solution of the following Dirichlet problem unique? uxx + uyy = q(x, y), < x < , u(x, 0) = f (x) Exercise 40.2 Is the solution of the following Dirichlet problem unique? uxx + uyy = q(x, y), < x < , y > 0 u(x, 0) = f (x), u bounded as x2 + y 2 Exercise 40.3 Not all combinations of boundary conditions/initial conditions lead to so called well-posed problems. Essentially, a well posed problem is one where the solutions depend continuously on the boundary data. Otherwise it is considered ill posed. Consider Laplaces equation on the unit-square uxx + uyy = 0, with u(0, y) = u(1, y) = 0 and u(x, 0) = 0, uy (x, 0) = sin(nx). 1. Show that even as 0, you can nd n so that the solution can attain any nite value for any y > 0. Use this to then show that this problem is ill posed. 2. Contrast this with the case where u(0, y) = u(1, y) = 0 and u(x, 0) = 0, u(x, 1) = sin(nx). Is this well posed? Exercise 40.4 Use the fundamental solutions for the Laplace equation
2

y>0

G = (x ) 1848

in three dimensions G(x|) = to derive the mean value theorem for harmonic functions u(p) = 1 4R2

1 4|x |

u() dA ,
SR

that relates the value of any harmonic function u(x) at the point x = p to the average of its value on the boundary of the sphere of radius R with center at p, (SR ). Exercise 40.5 Use the fundamental solutions for the modied Helmholz equation
2

u u = (x )
1 e |x| , 4|x |

in three dimensions u (x|) = to derive a generalized mean value theorem:

sinh R 1 u(p) = 4R2 R

u(x) dA
S

that relates the value of any solution u(x) at a point P to the average of its value on the sphere of radius R (S) with center at P. Exercise 40.6 Consider the uniqueness of solutions of 2 u(x) = 0 in a two dimensional region R with boundary curve C and a boundary condition n u(x) = a(x)u(x) on C. State a non-trivial condition on the function a(x) on C for which solutions are unique, and justify your answer. 1849

Exercise 40.7 Solve Laplaces equation on the surface of a semi-innite cylinder of unit radius, 0 < < 2, z > 0, where the solution, u(, z) is prescribed at z = 0: u(, 0) = f (). Exercise 40.8 Solve Laplaces equation in a rectangle. wxx + wyy = 0, 0 < x < a, 0 < y < b, w(0, y) = f1 (y), w(a, y) = f2 (y), wy (x, 0) = g1 (x), w(x, b) = g2 (x) Proceed by considering w = u + v where u and v are harmonic and satisfy u(0, y) = u(a, y) = 0, uy (x, 0) = g1 (x), u(x, b) = g2 (x), v(0, y) = f1 (y), v(a, y) = f2 (y), vy (x, 0) = v(x, b) = 0.

1850

40.4
Hint 40.1 Hint 40.2 Hint 40.3 Hint 40.4 Hint 40.5 Hint 40.6 Hint 40.7 Hint 40.8

Hints

1851

40.5

Solutions

Solution 40.1 Let u and v both be solutions of the Dirichlet problem. Let w be the dierence of these solutions. w satises the problem wxx + wyy = 0, < x < , w(x, 0) = 0. y>0

Since w = cy is a solution. We conclude that the solution of the Dirichlet problem is not unique. Solution 40.2 Let u and v both be solutions of the Dirichlet problem. Let w be the dierence of these solutions. w satises the problem wxx + wyy = 0, < x < , y > 0 w(x, 0) = 0, w bounded as x2 + y 2 . We solve this problem with a Fourier transform in x. 2 w + wyy = 0, w= w(, 0) = 0, w bounded as y

c1 cosh y + c2 sinh(y), = 0 c1 + c2 y, =0 w=0 w=0

Since u v = 0, we conclude that the solution of the Dirichlet problem is unique. 1852

Solution 40.3 1. We seek a solution of the form u(x, y) = sin(nx)Y (y). This form satises the boundary conditions at x = 0, 1. uxx + uyy = 0 (n) Y + Y = 0, Y (0) = 0 Y = c sinh(ny)
2

Now we apply the inhomogeneous boundary condition. uy (x, 0) = sin(nx) = cn sin(nx) sin(nx) sinh(ny) n For = 0 the solution is u = 0. Now consider any > 0. For any y > 0 and any nite value M , we can choose a value of n such that the solution along y = 0 takes on all values in the range [M . . . M ]. We merely choose a value of n such that sinh(ny) M . n Since the solution does not depend continuously on boundary data, this problem is ill posed. 2. We seek a solution of the form u(x, y) = c sin(nx) sinh(ny). This form satises the dierential equation and the boundary conditions at x = 0, 1 and at y = 0. We apply the inhomogeneous boundary condition at y = 1. u(x, 1) = sin(nx) = c sin(nx) sinh(n) sinh(ny) u(x, y) = sin(nx) sinh(n) For = 0 the solution is u = 0. Now consider any > 0. Note that |u| for (x, y) [0 . . . 1] [0 . . . 1]. The solution depends continuously on the given boundary data. This problem is well posed. Solution 40.4 The Green function problem for a sphere of radius R centered at the point is G = (x ), G
|x|=R

u(x, y) =

= 0.

(40.1)

1853

We will solve Laplaces equation, u = 0, where the value of u is known on the boundary of the sphere of radius R in terms of this Green function. First we solve for u(x) in terms of the Green function. (uG Gu) d =
S S

u(x ) d = u(x)

(uG Gu) d =
S

u G G n n S G dA = u S n u u
S

dA

u(x) =

G dA n

We are interested in the value of u at the center of the sphere. Let = |p | u(p) =
S

u()

G (p|) dA

We do not need to compute the general solution of Equation 40.1. We only need the Green function at the point x = p. We know that the general solution of the equation G = (x ) is G(x|) = 1 + v(x), 4|x |

where v(x) is an arbitrary harmonic function. The Green function at the point x = p is G(p|) = 1 + const. 4|p | 1854

We add the constraint that the Green function vanishes at = R. This determines the constant. G(p|) = 1 1 + 4|p | 4R 1 1 G(p|) = + 4 4R 1 G (p|) = 42

Now we are prepared to write u(p) in terms of the Green function. u(p) =
S

u() 1 4R2

1 dA 42 u() dA

u(p) =

This is the Mean Value Theorem for harmonic functions. Solution 40.5 The Green function problem for a sphere of radius R centered at the point is G G = (x ), We will solve the modied Helmholtz equation, u u = 0, where the value of u is known on the boundary of the sphere of radius R in terms of this Green function. in terms of this Green function. Let L[u] = u u. (uL[G] GL[u]) d =
S S

|x|=R

= 0.

(40.2)

u(x ) d = u(x)

1855

(uL[G] GL[u]) d =
S S

(uG Gu) d u dA

G u G n n S G = u dA S n =

u(x) =
S

G dA n

We are interested in the value of u at the center of the sphere. Let = |p |

u(p) =
S

u()

G (p|) dA

We do not need to compute the general solution of Equation 40.2. We only need the Green function at the point x = p. We know that the Green function there is a linear combination of the fundamental solutions,

G(p|) = c1

1 1 e |p| +c2 e |p| , 4|p | 4|p |

such that c1 + c2 = 1. The Green function is symmetric with respect to x and . We add the constraint that the Green 1856

function vanishes at = R. This gives us two equations for c1 and c2 . c1 R c2 R e e =0 4R 4R e2 R 1 c1 = , c2 = e2 R 1 e2 R 1 sinh ( R) G(p|) = 4 sinh R ( R) cosh ( R) sinh G (p|) = 42 sinh R 4 sinh R G (p|) ||=R = 4R sinh R c1 + c2 = 1, Now we are prepared to write u(p) in terms of the Green function. u(p) =
S

u()

4 sinh u(x) 4R sinh

dA R dA R

u(p) =
S

Rearranging this formula gives us the generalized mean value theorem. sinh R 1 u(p) = 4R2 R 1857

u(x) dA
S

Solution 40.6 First we think of this problem in terms of the the equilibrium solution of the heat equation. The boundary condition expresses Newtons law of cooling. Where a = 0, the boundary is insulated. Where a > 0, the rate of heat loss is proportional to the temperature. The case a < 0 is non-physical and we do not consider this scenario further. We know that if the boundary is entirely insulated, a = 0, then the equilibrium temperature is a constant that depends on the initial temperature distribution. Thus for a = 0 the solution of Laplaces equation is not unique. If there is any point on the boundary where a is positive then eventually, all of the heat will ow out of the domain. The equilibrium temperature is zero, and the solution of Laplaces equation is unique, u = 0. Therefore the solution of Laplaces equation is unique if a is continuous, non-negative and not identically zero. Now we prove our assertion. First note that if we substitute f = v u in the divergence theorem, f dx =
R R

f n ds,

we obtain the identity, (vu +


R

v u) dx =
R

u ds. n

(40.3)

Let u be a solution of Laplaces equation subject to the Robin boundary condition with our restrictions on a. We take v = u in Equation 40.3. u ( u)2 dx = u ds = au2 ds n R C C Since the rst integral is non-negative and the last is non-positive, the integrals vanish. This implies that u = 0. u is a constant. In order to satisfy the boundary condition where a is non-zero, u must be zero. Thus the unique solution in this scenario is u = 0. Solution 40.7 The mathematical statement of the problem is u u + uzz = 0, 0 < < 2, u(, 0) = f (). 1858 z > 0,

We have the implicit boundary conditions, u(0, z) = u(2, z), and the boundedness condition, u(, +) bounded. We expand the solution in a Fourier series. (This ensures that the boundary conditions at = 0, 2 are satised.)

u (0, z) = u (0, z)

u(, z) =
n=

un (z) en

We substitute the series into the partial dierential equation to obtain ordinary dierential equations for the un . n2 un (z) + un (z) = 0 The general solutions of this equation are un (z) = The bounded solutions are c1 + c2 z, c1 enz +c2 enz for n = 0, for n = 0.

c enz , for n > 0, un (z) = c, for n = 0, = c e|n|z . nz ce , for n < 0,

We substitute the series into the initial condition at z = 0 to determine the multiplicative constants.

u(, 0) = un (0) = 1 2
n= 2

un (0) en = f () f () en d fn
0

1859

Thus the solution is

u(, z) =
n=

fn en e|n|z .
2

Note that u(, z) f0 = as z +. Solution 40.8 The decomposition of the problem is shown in Figure 40.1.
w=g2(x) u=g2(x)

1 2

f () d
0

v=0

w=f1(y)

w=0

w=f2(y)

u=0

u=0

u=0

v=f1(y)

v=0

v=f2(y)

wy=g1(x)

uy=g1(x)

vy=0

Figure 40.1: Decomposition of the problem. First we solve the problem for u. uxx + uyy = 0, 0 < x < a, 0 < y < b, u(0, y) = u(a, y) = 0, uy (x, 0) = g1 (x), u(x, b) = g2 (x) We substitute the separation of variables u(x, y) = X(x)Y (y) into Laplaces equation. X Y = = 2 X Y 1860

We have the eigenvalue problem, X = 2 X, which has the solutions, n = The equation for Y (y) becomes, Yn = which has the solutions, eny/a , eny/a or cosh ny ny , sinh a a . n a
2

X(0) = X(a) = 0,

n , a

Xn = sin

nx , a

n N.

Yn ,

It will be convenient to choose solutions that satisfy the conditions, Y (b) = 0 and Y (0) = 0, respectively. n(b y) a ny a

sinh

, cosh

The solution for u(x, y) has the form,

u(x, y) =
n=1

sin

nx a

n sinh

n(b y) a

+ n cosh

ny a

We determine the coecients from the inhomogeneous boundary conditions. (Here we see how our choice of solutions 1861

for Y (y) is convenient.)

uy (x, 0) =
n=1

n nx n sin cosh a a nb a

nb a

= g1 (x) dx

n =

a sech n

2 a

g1 (x) sin
0

nx a

u(x, y) =
n=1

n sin nb a 2 a

ny nx cosh a a
a

n = sech Now we solve the problem for v.

g2 (x) sin
0

nx a

dx

vxx + vyy = 0, 0 < x < a, 0 < y < b, v(0, y) = f1 (y), v(a, y) = f2 (y), vy (x, 0) = 0, v(x, b) = 0 We substitute the separation of variables u(x, y) = X(x)Y (y) into Laplaces equation. X Y = = 2 X Y We have the eigenvalue problem, Y = 2 Y, which has the solutions, n = The equation for X(y) becomes, Xn = (2n 1) 2b 1862
2

Y (0) = Y (b) = 0, (2n 1)y 2b , n N.

(2n 1) , 2b

Yn = cos

Xn .

We choose solutions that satisfy the conditions, X(a) = 0 and X(0) = 0, respectively. sinh The solution for v(x, y) has the form,

(2n 1)(a x) 2b

, sinh

(2n 1)x 2b

v(x, y) =
n=1

cos

(2n 1)y 2b

n sinh

(2n 1)(a x) 2b

+ n sinh

(2n 1)x 2b

We determine the coecients from the inhomogeneous boundary conditions.

v(0, y) =
n=1

n cos

(2n 1)y 2b 2 b
b

sinh

(2n 1)a 2b

= f1 (y) dy

n = csch

(2n 1)a 2b n cos

f1 (y) cos
0

(2n 1)y 2b

v(a, y) =
n=1

(2n 1)y 2b 2 b
b

sinh

(2n 1)a 2b (2n 1)y 2b

= f2 (y) dy

n = csch

(2n 1)a 2b

f2 (y) cos
0

With u and v determined, the solution of the original problem is w = u + v.

1863

Chapter 41 Waves

1864

41.1

Exercises
utt uxx = 0

Exercise 41.1 Consider the 1-D wave equation on the domain 0 < x < 4 with initial displacement u(x, 0) = 1, 1 < x < 2 0, otherwise,

initial velocity ut (x, 0) = 0, and subject to the following boundary conditions 1. u(0, t) = u(4, t) = 0 2. ux (0, t) = ux (4, t) = 0 In each case plot u(x, t) for t = 1 , 1, 3 , 2 and combine onto a general plot in the x, t plane (up to a suciently large 2 2 time) so the behavior of u is clear for arbitrary x, t. Exercise 41.2 Sketch the solution to the wave equation: 1 1 u(x, t) = (u(x + ct, 0) + u(x ct, 0)) + 2 2c for various values of t corresponding to the initial conditions: 1. u(x, 0) = 0, ut (x, 0) = sin x where is a constant, 1865
x+ct

ut (, 0) d,
xct

2. u(x, 0) = 0,

1 for 0 < x < 1 ut (x, 0) = 1 for 1 < x < 0 0 for |x| > 1.

Exercise 41.3 1. Consider the solution of the wave equation for u(x, t): utt = c2 uxx on the innite interval < x < with initial displacement of the form u(x, 0) = and with initial velocity ut (x, 0) = 0. Show that the solution of the wave equation satisfying these initial conditions also solves the following semi-innite problem: Find u(x, t) satisfying the wave equation utt = c2 uxx in 0 < x < , t > 0, with initial conditions u(x, 0) = h(x), ut (x, 0) = 0, and with the xed end condition u(0, t) = 0. Here h(x) is any given function with h(0) = 0. 2. Use a similar idea to explain how you could use the general solution of the wave equation to solve the nite interval problem (0 < x < l) in which u(0, t) = u(l, t) = 0 for all t, with u(x, 0) = h(x) and ut (x, 0) = 0. Take h(0) = h(l) = 0. Exercise 41.4 The deection u(x, T ) = (x) and velocity ut (x, T ) = (x) for an innite string (governed by utt = c2 uxx ) are measured at time T , and we are asked to determine what the initial displacement and velocity proles u(x, 0) and ut (x, 0) must have been. An alert student suggests that this problem is equivalent to that of determining the solution of the wave equation at time T when initial conditions u(x, 0) = (x), ut (x, 0) = (x) are prescribed. Is she correct? If not, can you rescue her idea? 1866 h(x) for x > 0, h(x) for x < 0,

Exercise 41.5 In obtaining the general solution of the wave equation the interval was chosen to be innite in order to simplify the evaluation of the functions () and () in the general solution u(x, t) = (x + ct) + (x ct). But this general solution is in fact valid for any interval be it innite or nite. We need only choose appropriate functions (), () to satisfy the appropriate initial and boundary conditions. This is not always convenient but there are other situations besides the solution for u(x, t) in an innite domain in which the general solution is of use. Consider the whip-cracking problem, utt = c2 uxx , (with c a constant) in the domain x > 0, t > 0 with initial conditions u(x, 0) = ut (x, 0) = 0 and boundary conditions u(0, t) = (t) prescribed for all t > 0. Here (0) = 0. Find and so as to determine u for x > 0, t > 0. Hint: (From physical considerations conclude that you can take () = 0. Your solution will corroborate this.) Use the initial conditions to determine () and () for > 0. Then use the initial condition to determine () for < 0. Exercise 41.6 Let u(x, t) satisfy the equation utt = c2 uxx ; (with c a constant) in some region of the (x, t) plane. 1. Show that the quantity (ut cux ) is constant along each straight line dened by x ct = constant, and that (ut + cux ) is constant along each straight line of the form x + ct = constant. These straight lines are called characteristics; we will refer to typical members of the two families as C+ and C characteristics, respectively. Thus the line x ct = constant is a C+ characteristic. 1867 x > 0,

2. Let u(x, 0) and ut (x, 0) be prescribed for all values of x in < x < , and let (x0 , t0 ) be some point in the (x, t) plane, with t0 > 0. Draw the C+ and C characteristics through (x0 , t0 ) and let them intersect the x-axis at the points A,B. Use the properties of these curves derived in part (a) to determine ut (x0 , t0 ) in terms of initial data at points A and B. Using a similar technique to obtain ut (x0 , ) with 0 < < t, determine u(x0 , t0 ) by integration with respect to , and compare this with the solution derived in class: u(x, t) = 1 1 (u(x + ct, 0) + u(x ct, 0)) + 2 2c
x+ct

ut (, 0)d.
xct

Observe that this method of characteristics again shows that u(x0 , t0 ) depends only on that part of the initial data between points A and B. Exercise 41.7 The temperature u(x, t) at a depth x below the Earths surface at time t satises ut = uxx . The surface x = 0 is heated by the sun according to the periodic rule: u(0, t) = T cos(t). Seek a solution of the form u(x, t) = A etx . a) Find u(x, t) satisfying u 0 as x +, (i.e. deep into the Earth). b) Find the temperature variation at a xed depth, h, below the surface. c) Find the phase lag (x) such that when the maximum temperature occurs at t0 on the surface, the maximum at depth x occurs at t0 + (x). d) Show that the seasonal, (i.e. yearly), temperature changes and daily temperature changes penetrate to depths in the ratio: xyear = 365, xday where xyear and xday are the depths of same temperature variation caused by the dierent periods of the source. 1868

Exercise 41.8 An innite cylinder of radius a produces an external acoustic pressure eld u satisfying: utt = c2 u, by a pure harmonic oscillation of its surface at r = a. That is, it moves so that u(a, , t) = f () et where f () is a known function. Note that the waves must be outgoing at innity, (radiation condition at innity). Find the solution, u(r, , t). We seek a periodic solution of the form, u(r, , t) = v(r, ) et . Exercise 41.9 Plane waves are incident on a soft cylinder of radius a whose axis is parallel to the plane of the waves. Find the eld scattered by the cylinder. In particular, examine the leading term of the solution when a is much smaller than the wavelength of the incident waves. If v(x, y, t) is the scattered eld it must satisfy: Wave Equation: vtt = c2 v, Soft Cylinder: Scattered: x2 + y 2 > a2 ; 0 < 2;

v(x, y, t) = e(ka cos t) , on r = a, v is outgoing as r .

Here k = /c. Use polar coordinates in the (x, y) plane. Exercise 41.10 Consider the ow of electricity in a transmission line. The current, I(x, t), and the voltage, V (x, t), obey the telegraphers system of equations: Ix = CVt + GV, Vx = LIt + RI, 1869

where C is the capacitance, G is the conductance, L is the inductance and R is the resistance. a) Show that both I and V satisfy a damped wave equation. b) Find the relationship between the physical constants, C, G, L and R such that there exist damped traveling wave solutions of the form: V (x, t) = et (f (x at) + g(x + at)). What is the wave speed?

1870

41.2
Hint 41.1

Hints

Hint 41.2

Hint 41.3

Hint 41.4

Hint 41.5 From physical considerations conclude that you can take () = 0. Your solution will corroborate this. Use the initial conditions to determine () and () for > 0. Then use the initial condition to determine () for < 0. Hint 41.6

Hint 41.7

a) Substitute u(x, t) = (A etx ) into the partial dierential equation and solve for . Assume that has positive real part so that the solution vanishes as x +. Hint 41.8 Seek a periodic solution of the form, u(r, , t) = v(r, ) et . 1871

Solve the Helmholtz equation for v with a Fourier series expansion,

v(r, ) =
n=

vn (r) en .

You will nd that the vn satisfy Bessels equation. Choose the vn so that u satises the boundary condition at r = a and the radiation condition at innity. The Bessel functions have the asymptotic behavior, 2 cos( n/2 /4), 2 sin( n/2 /4), Yn () 2 i(n/2/4) (1) e Hn () , 2 i(n/2/4) (2) e , Hn () Jn () Hint 41.9 Hint 41.10 as , as , as , as .

1872

41.3

Solutions

Solution 41.1 1. The initial position is u(x, 0) = H 1 3 x 2 2 .

We extend the domain of the problem to ( . . . ) and add image sources in the initial condition so that u(x, 0) is odd about x = 0 and x = 4. This enforces the boundary conditions at these two points. utt uxx = 0,

x ( . . . ), H

t (0 . . . ) , ut (x, 0) = 0

u(x, 0) =
n=

3 1 x 8n 2 2

1 13 x 8n 2 2

We use DAlemberts solution to solve this problem. 1 H u(x, t) = 2 n=

1 3 x 8n t 2 2 H

+H

1 3 x 8n + t 2 2 H 1 13 x 8n + t 2 2

1 13 x 8n t 2 2

The solution for several times is plotted in Figure 41.1. Note that the solution is periodic in time with period 8. Figure 41.3 shows the solution in the phase plane for 0 < t < 8. Note the odd reections at the boundaries. 2. The initial position is u(x, 0) = H 3 1 x 2 2 .

We extend the domain of the problem to ( . . . ) and add image sources in the initial condition so that 1873

0.4 0.2 -0.2 -0.4 0.4 0.2 -0.2 -0.4 1 2 3 4 1 2 3 4

0.4 0.2 -0.2 -0.4 0.4 0.2 -0.2 -0.4 1 2 3 4 1 2 3 4

Figure 41.1: The solution at t = 1/2, 1, 3/2, 2 for the boundary conditions u(0, t) = u(4, t) = 0. u(x, 0) is even about x = 0 and x = 4. This enforces the boundary conditions at these two points. utt uxx = 0,

x ( . . . ), +H

t (0 . . . ) , ut (x, 0) = 0

u(x, 0) =
n=

3 1 x 8n 2 2

1 13 x 8n 2 2

We use DAlemberts solution to solve this problem. 1 u(x, t) = H 2 n=

1 3 x 8n t 2 2 +H

+H

1 3 x 8n + t 2 2 +H 1 13 x 8n + t 2 2

1 13 x 8n t 2 2 1874

The solution for several times is plotted in Figure 41.2. Note that the solution is periodic in time with period 8. Figure 41.3 shows the solution in the phase plane for 0 < t < 8. Note the even reections at the boundaries.

1 0.8 0.6 0.4 0.2 1 1 0.8 0.6 0.4 0.2 1 2 3 4 2 3 4

1 0.8 0.6 0.4 0.2 1 1 0.8 0.6 0.4 0.2 1 2 3 4 2 3 4

Figure 41.2: The solution at t = 1/2, 1, 3/2, 2 for the boundary conditions ux (0, t) = ux (4, t) = 0. Solution 41.2 1. u(x, t) = 1 1 (u(x + ct, 0) + u(x ct, 0)) + 2 2c x+ct 1 u(x, t) = sin( ) d 2c xct sin(x) sin(ct) u(x, t) = c
x+ct

ut (, 0) d
xct

Figure 41.4 shows the solution for c = = 1. 1875

t 0 1/2 0 0 0 -1/2 -1/2 0 -1 -1/2 -1/2 0 0 1/2 1/2 0 1 0 x 0 0 1/2 1 0

t 0 1/2 1 0 1 1/2 1/2 0 1 1/2 1/2 0 1 1/2 1/2 0 1 0 x 1 0 1/2 1 0

Figure 41.3: The solution in the phase plane for the boundary conditions u(0, t) = u(4, t) = 0 and ux (0, t) = ux (4, t) = 0.

2. We can write the initial velocity in terms of the Heaviside function. 1 for 0 < x < 1 ut (x, 0) = 1 for 1 < x < 0 0 for |x| > 1. ut (x, 0) = H(x + 1) + 2H(x) H(x 1) 1876

1 0.5 u 0 -0.5 -1 -5 2 0 x 5 0

6 4 t

Figure 41.4: Solution of the wave equation. We integrate the Heaviside function. 0 for b < c b H(x c) dx = b a for a > c a b c otherwise If a < b, we can express this as
b

H(x c) dx = min(b a, max(b c, 0)).


a

1877

Now we nd an expression for the solution. 1 1 x+ct (u(x + ct, 0) + u(x ct, 0)) + ut (, 0) d 2 2c xct 1 x+ct (H( + 1) + 2H( ) H( 1)) d u(x, t) = 2c xct u(x, t) = min(2ct, max(x + ct + 1, 0)) + 2 min(2ct, max(x + ct, 0)) min(2ct, max(x + ct 1, 0)) u(x, t) = Figure 41.5 shows the solution for c = 1.

1 0.5 u 0 -0.5 -1 -4 -2 0 x 2 4 0 1 3 2 t

Figure 41.5: Solution of the wave equation.

1878

Solution 41.3 1. The solution on the interval ( . . . ) is 1 u(x, t) = (h(x + ct) + h(x ct)). 2 Now we solve the problem on (0 . . . ). We dene the odd extension of h(x). h(x) = Note that h(x) for x > 0, = sign(x)h(|x|) h(x) for x < 0, = h (0+ ) = h (0+ ).

d h (0 ) = (h(x)) dx Thus h(x) is piecewise C 2 . Clearly

x0+

1 u(x, t) = (h(x + ct) + h(x ct)) 2 satises the dierential equation on (0 . . . ). We verify that it satises the initial condition and boundary condition. 1 u(x, 0) = (h(x) + h(x)) = h(x) 2 1 1 u(0, t) = (h(ct) + h(ct)) = (h(ct) h(ct)) = 0 2 2 2. First we dene the odd extension of h(x) on the interval (l . . . l). h(x) = sign(x)h(|x|), x (l . . . l)

Then we form the odd periodic extension of h(x) dened on ( . . . ). x+l h(x) = sign x 2l 2l h x 2l x+l 2l , x ( . . . )

1879

We note that h(x) is piecewise C 2 . Also note that h(x) is odd about the points x = nl, n Z. That is, h(nl x) = h(nl + x). Clearly 1 u(x, t) = (h(x + ct) + h(x ct)) 2 satises the dierential equation on (0 . . . l). We verify that it satises the initial condition and boundary conditions. 1 u(x, 0) = (h(x) + h(x)) 2 u(x, 0) = h(x) x+l x+l h x 2l u(x, 0) = sign x 2l 2l 2l u(x, 0) = h(x) 1 1 u(0, t) = (h(ct) + h(ct)) = (h(ct) h(ct)) = 0 2 2 1 1 u(l, t) = (h(l + ct) + h(l ct)) = (h(l + ct) h(l + ct)) = 0 2 2 Solution 41.4 Change of Variables. Let u(x, t) be the solution of the problem with deection u(x, T ) = (x) and velocity ut (x, T ) = (x). Dene v(x, ) = u(x, T ). We note that u(x, 0) = v(x, T ). v( ) satises the wave equation. v = c2 vxx The initial conditions for v are v(x, 0) = u(x, T ) = (x), Thus we see that the student was correct. 1880 v (x, 0) = ut (x, T ) = (x).

Direct Solution. DAlemberts solution is valid for all x and t. We formally substitute t T for t in this solution to solve the problem with deection u(x, T ) = (x) and velocity ut (x, T ) = (x). u(x, t) = 1 1 ((x + c(t T )) + (x c(t T ))) + 2 2c
x+c(tT )

( ) d
xc(tT )

This satises the wave equation, because the equation is shift-invariant. It also satises the initial conditions. u(x, T ) = ut (x, t) = 1 1 ((x) + (x)) + 2 2c
x

( ) d = (x)
x

1 1 (c (x + c(t T )) c (x c(t T ))) + ((x + c(t T )) + (x c(t T ))) 2 2 1 1 ut (x, T ) = (c (x) c (x)) + ((x) + (x)) = (x) 2 2

Solution 41.5 Since the solution is a wave moving to the right, we conclude that we could take () = 0. Our solution will corroborate this. The form of the solution is u(x, t) = (x + ct) + (x ct). We substitute the solution into the initial conditions. u(x, 0) = () + () = 0, > 0 ut (x, 0) = c () c () = 0, > 0 We integrate the second equation to obtain the system () + () = 0, > 0, () () = 2k, > 0, which has the solution () = k, () = k, 1881 > 0.

Now we substitute the solution into the initial condition. u(0, t) = (ct) + (ct) = (t), t > 0 () + () = (/c), > 0 () = (/c) k, < 0 This determines u(x, t) for x > 0 as it depends on () only for > 0. The constant k is arbitrary. Changing k does not change u(x, t). For simplicity, we take k = 0. u(x, t) = (x ct) u(x, t) = 0 for x ct < 0 (t x/c) for x ct > 0

u(x, t) = (t x/c)H(ct x) Solution 41.6 1. We write the value of u along the line x ct = k as a function of t: u(k + ct, t). We dierentiate ut cux with respect to t to see how the quantity varies. d (ut (k + ct, t) cux (k + ct, t)) = cuxt + utt c2 uxx cuxt dt = utt c2 uxx =0 Thus ut cux is constant along the line x ct = k. Now we examine ut + cux along the line x + ct = k. d (ut (k ct, t) + cux (k ct, t)) = cuxt + utt c2 uxx + cuxt dt = utt c2 uxx =0 ut + cux is constant along the line x + ct = k. 1882

2. From part (a) we know ut (x0 , t0 ) cux (x0 , t0 ) = ut (x0 ct0 , 0) cux (x0 ct0 , 0) ut (x0 , t0 ) + cux (x0 , t0 ) = ut (x0 + ct0 , 0) + cux (x0 + ct0 , 0). We add these equations to nd ut (x0 , t0 ). 1 (ut (x0 ct0 , 0) cux (x0 ct0 , 0)ut (x0 + ct0 , 0) + cux (x0 + ct0 , 0)) 2 Since t0 was arbitrary, we have 1 ut (x0 , ) = (ut (x0 c, 0) cux (x0 c, 0)ut (x0 + c, 0) + cux (x0 + c, 0)) 2 for 0 < < t0 . We integrate with respect to to determine u(x0 , t0 ). ut (x0 , t0 ) =
t0

u(x0 , t0 ) = u(x0 , 0) +
0

1 (ut (x0 c, 0) cux (x0 c, 0)ut (x0 + c, 0) + cux (x0 + c, 0)) d 2

1 t0 = u(x0 , 0) + (cux (x0 c, 0) + cux (x0 + c, 0)) d 2 0 1 t0 + (ut (x0 c, 0) + ut (x0 + c, 0)) d 2 0 1 = u(x0 , 0) + (u(x0 ct0 , 0) u(x0 , 0) + u(x0 + ct0 , 0) u(x0 , 0)) 2 x0 ct0 1 1 x0 +ct0 + ut (, 0) d + ut (, 0) d 2c x0 2c x0 1 1 x0 +ct0 ut (, 0) d = (u(x0 ct0 , 0) + u(x0 + ct0 , 0)) + 2 2c x0 ct0 We have DAlemberts solution. u(x, t) = 1 1 (u(x ct, 0) + u(x + ct, 0)) + 2 2c
x+ct

ut (, 0) d
xct

1883

Solution 41.7

a) We substitute u(x, t) = A etx into the partial dierential equation and take the real part as the solution. We assume that has positive real part so the solution vanishes as x +. A etx = 2 A etx = 2 = (1 + ) A solution of the partial dierential equation is, u(x, t) = A exp t (1 + ) x 2 , 2

x cos t 2 Applying the initial condition, u(0, t) = T cos(t), we obtain, u(x, t) = A exp u(x, t) = T exp b) At a xed depth x = h, the temperature is u(h, t) = T exp Thus the temperature variation is T exp h 2 u(h, t) T exp h cos t 2 x cos t 2

x . 2

x . 2

h . 2

h . 2

1884

c) The solution is an exponentially decaying, traveling wave that propagates into the Earth with speed / /(2) = 2. More generally, the wave ebt cos(t ax) travels in the positive direction with speed /a. Figure 41.6 shows such a wave for a sequence of times.

Figure 41.6: An Exponentially Decaying, Traveling Wave The phase lag, (x) is the time that it takes for the wave to reach a depth of x. It satises, (x) x = 0, 2 x . 2

(x) =

d) Let year be the frequency for annual temperature variation, then day = 365year . If xyear is the depth that a particular yearly temperature variation reaches and xday is the depth that this same variation in daily temperature reaches, then year day exp xyear = exp xday , 2 2 1885

year day xyear = xday , 2 2 xyear = 365. xday Solution 41.8 We seek a periodic solution of the form, u(r, , t) = v(r, ) et . Substituting this into the wave equation will give us a Helmholtz equation for v. 2 v = c2 v 1 1 2 vrr + vr + 2 v + 2 v = 0 r r c We have the boundary condition v(a, ) = f () and the radiation condition at innity. We expand v in a Fourier series in in which the coecients are functions of r. You can check that en are the eigenfunctions obtained with separation of variables. v(r, ) =
n=

vn (r) en

We substitute this expression into the Helmholtz equation to obtain ordinary dierential equations for the coecients vn .

n=

1 vn + vn + r

2 n2 2 c2 r

vn en = 0

The dierential equations for the vn are 1 vn + vn + r 2 n2 2 c2 r 1886 vn = 0.

which has as linearly independent solutions the Bessel and Neumann functions, Jn or the Hankel functions,
(1) Hn

r , c

Yn

r , c

r , c

(2) Hn

r . c

The functions have the asymptotic behavior, 2 cos( n/2 /4), 2 Yn () sin( n/2 /4), 2 i(n/2/4) (1) e Hn () , 2 i(n/2/4) (2) e Hn () , Jn () as , as , as , as .

u(r, , t) will be an outgoing wave at innity if it is the sum of terms of the form ei(tconstr) . Thus the vn must have the form (2) r vn (r) = bn Hn c for some constants, bn . The solution for v(r, ) is

v(r, ) =
n=

(2) bn Hn

r n e . c

1887

We determine the constants bn from the boundary condition at r = a.

v(a, ) =
n=

(2) bn Hn

a n e = f () c
2

bn =

1
(2) 2Hn (a/c) 0

f () en d r n e c

u(r, , t) = et
n=

(2) b n Hn

Solution 41.9 We substitute the form v(x, y, t) = u(r, ) et into the wave equation to obtain a Helmholtz equation. c2 u + 2 u = 0 1 1 urr + ur + 2 u + k 2 u = 0 r r We solve the Helmholtz equation with separation of variables. We expand u in a Fourier series.

u(r, ) =
n=

un (r) en

We substitute the sum into the Helmholtz equation to determine ordinary dierential equations for the coecients. 1 n2 un + un + k 2 2 r r un = 0

This is Bessels equation, which has as solutions the Bessel and Neumann functions, {Jn (kr), Yn (kr)} or the Hankel (1) (2) functions, {Hn (kr), Hn (kr)}. 1888

Recall that the solutions of the Bessel equation have the asymptotic behavior, 2 cos( n/2 /4), 2 Yn () sin( n/2 /4), 2 i(n/2/4) (1) e , Hn () 2 i(n/2/4) (2) e Hn () , Jn () as , as , as , as .

From this we see that only the Hankel function of the rst kink will give us outgoing waves as . Our solution for u becomes,

u(r, ) =
n=

(1) bn Hn (kr) en .

We determine the coecients in the expansion from the boundary condition at r = a.

u(a, ) =
n=

(1) bn Hn (ka) en = eka cos

bn = We evaluate the integral with the identities,

1
(1) 2Hn (ka) 0

eka cos en d

2 1 ex cos en d, Jn (x) = n 2i 0 Jn (x) = (1)n Jn (x).

1889

Thus we obtain,

u(r, ) =
n=

()n Jn (ka)
(1) Hn (ka)

(1) Hn (kr) en .

When a

1/k, i.e. ka

1, the Bessel function has the behavior, (ka/2)n Jn (ka) . n!

In this case, the n = 0 terms in the sum are much smaller than the n = 0 term. The approximate solution is, u(r, ) H0 (kr) H0 (ka) v(r, , t) Solution 41.10 a) Ix = CVt + GV, Vx = LIt + RI First we derive a single partial dierential equation for I. We dierentiate the two partial dierential equations with respect to x and t, respectively and then eliminate the Vxt terms. Ixx = CVtx + GVx , Vxt = LItt + RIt Ixx + LCItt + RCIt = GVx 1890 H0 (kr)
(1) H0 (ka) (1) (1) (1)

et .

We use the initial set of equations to write Vx in terms of I. Ixx + LCItt + RCIt + G(LIt + RI) = 0 Itt + RC + GL GR 1 It + I Ixx = 0 LC LC LC

Now we derive a single partial dierential equation for V . We dierentiate the two partial dierential equations with respect to t and x, respectively and then eliminate the Ixt terms. Ixt = CVtt + GVt , Vxx = LItx + RIx Vxx = RIx LCVtt LGVt We use the initial set of equations to write Ix in terms of V . LCVtt + LGVt Vxx + R(CVt + GV ) = 0 Vtt + RG 1 RC + LG Vt + V Vxx = 0. LC LC LC

Thus we see that I and V both satisfy the same damped wave equation. b) We substitute V (x, t) = et (f (x at) + g(x + at)) into the damped wave equation for V . 2 RG RC + LG + LC LC et (f + g) + 2 + RC + LG LC a et (f + g ) + a2 et (f + g ) 1 t e (f + g ) = 0 LC

Since f and g are arbitrary functions, the coecients of et (f + g), et (f + g ) and et (f + g ) must vanish. This gives us three constraints. a2 1 = 0, LC 2 + RC + LG = 0, LC 1891 2 RC + LG RG + =0 LC LC

The rst equation determines the wave speed to be a = 1/ LC. We substitute the value of from the second equation into the third equation. RC + LG RG = , 2 + =0 2LC LC In order for damped waves to propagate, the physical constants must satisfy, RC + LG RG = 0, LC 2LC 4RGLC (RC + LG)2 = 0, (RC LG)2 = 0, RC = LG.
2

1892

Chapter 42 Similarity Methods


Introduction. Consider the partial dierential equation (not necessarily linear) F Say the solution is u(x, t) = x sin t t1/2 x1/2 . u u , , u, t, x t x = 0.

Making the change of variables = x/t, f () = u(x, t), we could rewrite this equation as f () = sin 1/2 . We see now that if we had guessed that the solution of this partial dierential equation was only dependent on powers of x/t we could have changed variables to and f and instead solved the ordinary dierential equation G df , f, d = 0.

By using similarity methods one can reduce the number of independent variables in some PDEs. 1893

Example 42.0.1 Consider the partial dierential equation x u u +t u = 0. t x

One way to nd a similarity variable is to introduce a transformation to the temporary variables u , t , x , and the parameter . u=u t = t m x = x n where n and m are unknown. Rewriting the partial dierential equation in terms of the temporary variables, x n u 1m u 1n + t m u=0 t x u mn u m+n x +t u =0 t x

There is a similarity variable if can be eliminated from the equation. Equating the coecients of the powers of in each term, m + n = m n = 0. This has the solution m = n. The similarity variable, , will be unchanged under the transformation to the temporary variables. One choice is t t n t = . = = x x m x Writing the two partial derivative in terms of , d 1 d = = t t d x d d t d = = 2 x x d x d 1894

The partial dierential equation becomes du du 2 u=0 d d du u = d 1 2 Thus we have reduced the partial dierential equation to an ordinary dierential equation that is much easier to solve. d 1 2 1/2 1/2 u() = exp + d 1 1+ 1 1 u() = exp log(1 ) + log(1 + ) 2 2 u() = exp u() = (1 )1/2 (1 + )1/2 u(x, t) = 1 + t/x 1 t/x
1/2

Thus we have found a similarity solution to the partial dierential equation. Note that the existence of a similarity solution does not mean that all solutions of the dierential equation are similarity solutions. Another Method. Another method is to substitute = x t and determine if there is an that makes a similarity variable. The partial derivatives become d d = = x t t d d d d = = x1 t x x d d 1895

The partial dierential equation becomes x+1 du du + x1 t2 u = 0. d d

If there is a value of such that we can write this equation in terms of , then = x t is a similarity variable. If = 1 then the coecient of the rst term is trivially in terms of . The coecient of the second term then becomes x2 t2 . Thus we see = x1 t is a similarity variable. Example 42.0.2 To see another application of similarity variables, any partial dierential equation of the form F tx, u, is equivalent to the ODE F where = tx. Performing the change of variables, 1 du 1 du du 1 u = = x = x t x t d x d d 1 du 1 du du 1 u = = t = . t x t x d t d d For example the partial dierential equation u which can be rewritten u u x u + + tx2 u = 0 t t x , u, du du , d d =0 ut ux , x t =0

1 u 1 u + + txu = 0, x t t x 1896

is equivalent to u where = tx. du du + + u = 0 d d

1897

42.1

Exercises
ut = uxx

Exercise 42.1 Consider the 1-D heat equation Assume that there exists a function (x, t) such that it is possible to write u(x, t) = F ((x, t)). Re-write the PDE in terms of F (), its derivatives and (partial) derivatives of . By guessing that this transformation takes the form = xt , nd a value of so that this reduces to an ODE for F () (i.e. x and t are explicitly removed). Find the general solution and use this to nd the corresponding solution u(x, t). Is this the general solution of the PDE? Exercise 42.2 With = x t, nd such that for some function f , = f () is a solution of t = a2 xx . Find f () as well.

1898

42.2
Hint 42.1 Hint 42.2

Hints

1899

42.3

Solutions

Solution 42.1 We write the derivatives of u(x, t) in terms of derivatives of F (). ut = xt1 F = F t ux = t F 2 uxx = t2 F = 2 F x We substitite these expressions into the heat equation. 2 F = 2F t x x2 1 F = F t We can write this equation in terms of F and only if = 1/2. We make this substitution and solve the ordinary dierential equation for F (). F = F 2 2 log(F ) = + c 4 2 F = c exp 4 F = c1 We can write F in terms of the error function. F = c1 erf 2 + c2 exp 2 4 d + c2

1900

We write this solution in terms of x and t. u(x, t) = c1 erf x 2 t + c2

This is not the general solution of the heat equation. There are many other solutions. Note that since x and t do not explicitly appear in the heat equation, u(x, t) = c1 erf is a solution. Solution 42.2 We write the derivatives of in terms of f . f = x f = t1 f t x = f = x1 tf x xx = f x1 t + x1 tx1 t f x 2 22 2 2 xx = x t f + ( 1)x tf t = xx = x2 2 2 f + ( 1)f We substitute these expressions into the diusion equation. f = x2 t 2 2 f + ( 1)f In order for this equation to depend only on the variable , we must have = 2. For this choice we obtain an ordinary 1901 x x0 2 (t t0 ) + c2

dierential equation for f (). f = 4 2 f + 6f 1 3 f = 2 f 4 2 1 3 log(f ) = log + c 4 2 f = c1 3/2 e1/(4)

f () = c1 f () = c1

t3/2 e1/(4t) dt + c2
1/(2 )

et dt + c2 1 2 + c2

f () = c1 erf

1902

Chapter 43 Method of Characteristics


43.1 First Order Linear Equations
ut + cux = 0 (43.1)

Consider the following rst order wave equation.

Let x(t) be some path in the phase plane. Perhaps x(t) describes the position of an observer who is noting the value of the solution u(x(t), t) at their current location. We dierentiate with respect to t to see how the solution varies for the observer. d u(x(t), t) = ut + x (t)ux (43.2) dt We note that if the observer is moving with velocity c, x (t) = c, then the solution at their current location does not change because ut + cux = 0. We will examine this more carefully. By comparing Equations 43.1 and 43.2 we obtain ordinary dierential equations representing the position of an observer and the value of the solution at that position. dx = c, dt du =0 dt

1903

Let the observer start at the position x0 . Then we have an initial value problem for x(t). dx = c, x(0) = x0 dt x(t) = x0 + ct These lines x(t) are called characteristics of Equation 43.1. Let the initial condition be u(x, 0) = f (x). We have an initial value problem for u(x(t), t). du = 0, u(0) = f (x0 ) dt u(x(t), t) = f (x0 ) Again we see that the solution is constant along the characteristics. We substitute the equation for the characteristics into this expression. u(x0 + ct, t) = f (x0 ) u(x, t) = f (x ct) Now we see that the solution of Equation 43.1 is a wave moving with velocity c. The solution at time t is the initial condition translated a distance of ct.

43.2

First Order Quasi-Linear Equations


ut + a(x, t, u)ux = 0 (43.3)

Consider the following quasi-linear equation.

We will solve this equation with the method of characteristics. We dierentiate the solution along a path x(t). d u(x(t), t) = ut + x (t)ux dt 1904 (43.4)

By comparing Equations 43.3 and 43.4 we obtain ordinary dierential equations for the characteristics x(t) and the solution along the characteristics u(x(t), t). dx = a(x, t, u), dt du =0 dt

Suppose an initial condition is specied, u(x, 0) = f (x). Then we have ordinary dierential equation, initial value problems. dx = a(x, t, u), x(0) = x0 dt du = 0, u(0) = f (x0 ) dt We see that the solution is constant along the characteristics. The solution of Equation 43.3 is a wave moving with velocity a(x, t, u). Example 43.2.1 Consider the inviscid Burger equation, ut + uux = 0, u(x, 0) = f (x).

We write down the dierential equations for the solution along a characteristic. dx = u, x(0) = x0 dt du = 0, u(0) = f (x0 ) dt First we solve the equation for u. u = f (x0 ). Then we solve for x. x = x0 + f (x0 )t. This gives us an implicit solution of the Burger equation. u(x0 + f (x0 )t, t) = f (x0 )

1905

43.3

The Method of Characteristics and the Wave Equation


utt = c2 uxx .

Consider the one dimensional wave equation,

We make the change of variables, a = ux , b = ut , to obtain a coupled system of rst order equations. at b x = 0 b t c 2 ax = 0 We write this as a matrix equation. a b +
t

0 1 c2 0

a b

=0
x

The eigenvalues and eigenvectors of the matrix are 1 = c, 2 = c, 1 = 1 , c 2 = 1 . c

The matrix is diagonalized by a similarity transformation. c 0 0 c = 1 1 c c


1

0 1 c2 0

1 1 c c

We make a change of variables to diagonalize the system. a b 1 1 c c +


t

1 1 c c

1 1 c c =0
x

0 1 c2 0

1906

Now we left multiply by the inverse of the matrix of eigenvectors to obtain an uncoupled system that we can solve directly. +
t

c 0 0 c

= 0.
x

(x, t) = p(x + ct),

(x, t) = q(x ct),

Here p, q C 2 are arbitrary functions. We change variables back to a and b. a(x, t) = p(x + ct) + q(x ct), b(x, t) = cp(x + ct) cq(x ct)

We could integrate either a = ux or b = ut to obtain the solution of the wave equation. u = F (x ct) + G(x + ct) Here F, G C 2 are arbitrary functions. We see that u(x, t) is the sum of a waves moving to the right and left with speed c. This is the general solution of the one-dimensional wave equation. Note that for any given problem, F and G are only determined to whithin an additive constant. For any constant k, adding k to F and subtracting it from G does not change the solution. u = (F (x ct) + k) + (G(x ct) k)

43.4

The Wave Equation for an Innite Domain


utt = c2 uxx , < x < , t > 0 u(x, 0) = f (x), ut (x, 0) = g(x)

Consider the Cauchy problem for the wave equation on < x < .

We know that the solution is the sum of right-moving and left-moving waves. u(x, t) = F (x ct) + G(x + ct) 1907 (43.5)

The initial conditions give us two constraints on F and G. F (x) + G(x) = f (x), We integrate the second equation. F (x) + G(x) = Here Q(x) = 1 c g(x) dx + const cF (x) + cG (x) = g(x).

q(x) dx. We solve the system of equations for F and G. 1 1 F (x) = f (x) 2 2c g(x) dx k, 1 1 G(x) = f (x) + 2 2c g(x) dx + k

Note that the value of the constant k does not aect the solution, u(x, t). For simplicity we take k = 0. We substitute F and G into Equation 43.5 to determine the solution.
x+ct xct 1 1 u(x, t) = (f (x ct) + f (x + ct)) + g(x) dx g(x) dx 2 2c 1 x+ct 1 g() d u(x, t) = (f (x ct) + f (x + ct)) + 2 2c xct

u(x, t) =

1 1 (u(x ct, 0) + u(x + ct, 0)) + 2 2c

x+ct

ut (, 0) d
xct

43.5

The Wave Equation for a Semi-Innite Domain


utt = c2 uxx , 0 < x < , t > 0 u(x, 0) = f (x), ut (x, 0) = g(x), u(0, t) = h(t) 1908

Consider the wave equation for a semi-innite domain.

Again the solution is the sum of a right-moving and a left-moving wave. u(x, t) = F (x ct) + G(x + ct) For x > ct, the boundary condition at x = 0 does not aect the solution. Thus we know the solution in this domain from our work on the wave equation in the innite domain. u(x, t) = 1 1 (f (x ct) + f (x + ct)) + 2 2c 1 F () = f () 2 1 G() = f () + 2 1 2c 1 2c
x+ct

g() d,
xct

x > ct

From this, F () and G() are determined for > 0. g() d, g() d, >0 >0

In order to determine the solution u(x, t) for x, t > 0 we also need to determine F () for < 0. To do this, we substitute the form of the solution into the boundary condition at x = 0. u(0, t) = h(t), t > 0 F (ct) + G(ct) = h(t), t > 0 F () = G() + h(/c), < 0 1 1 F () = f () g() d + h(/c), 2 2c We determine the solution of the wave equation for x < ct. u(x, t) = F (x ct) + G(x + ct) 1 1 x+ct 1 1 x+ct u(x, t) = f (x + ct) g() d + h(t x/c) + f (x + ct) + g() d, 2 2c 2 2c 1 1 x+ct u(x, t) = (f (x + ct) + f (x + ct)) + g() d + h(t x/c), x < ct 2 2c x+ct 1909 x < ct <0

Finally, we collect the solutions in the two domains. u(x, t) =


1 2 1 2 1 (f (x ct) + f (x + ct)) + 2c xct g() d, x > ct x+ct 1 (f (x + ct) + f (x + ct)) + 2c x+ct g() d + h(t x/c), x < ct x+ct

43.6

The Wave Equation for a Finite Domain


utt = c2 uxx , < x < , t > 0 u(x, 0) = f (x), ut (x, 0) = g(x)

Consider the wave equation for the innite domain.

If f (x) and g(x) are odd about x = 0, (f (x) = f (x), g(x) = g(x)), then u(x, t) is also odd about x = 0. We can demonstrate this with DAlemberts solution. u(x, t) = 1 1 (f (x ct) + f (x + ct)) + 2 2c
x+ct

g() d
xct

1 1 x+ct g() d u(x, t) = (f (x ct) + f (x + ct)) 2 2c xct 1 xct 1 = (f (x + ct) + f (x ct)) g() (d) 2 2c x+ct 1 x+ct 1 = (f (x ct) + f (x + ct)) + g() d 2 2c xct = u(x, t) Thus if the initial conditions f (x) and g(x) are odd about a point then the solution of the wave equation u(x, t) is also odd about that point. The analogous result holds if the initial conditions are even about a point. These results are useful in solving the wave equation on a nite domain. 1910

Consider a string of length L with xed ends. utt = c2 uxx , 0 < x < L, t > 0 u(x, 0) = f (x), ut (x, 0) = g(x), u(0, t) = u(L, t) = 0 We extend the domain of the problem to x ( . . . ). We form the odd periodic extensions f and g which are odd about the points x = 0, L. If a function h(x) is dened for positive x, then sign(x)h(|x|) is the odd extension of the function. If h(x) is dened for x (L . . . L) then its periodic extension is h x 2L x+L 2L .

We combine these two formulas to form odd periodic extensions. x+L f (x) = sign x 2L 2L x+L g (x) = sign x 2L 2L f g x+L 2L x+L x 2L 2L x 2L

Now we can write the solution for the vibrations of a string with xed ends. u(x, t) = 1 1 f (x ct) + f (x + ct) + 2 2c
x+ct

g () d
xct

43.7

Envelopes of Curves

Consider the tangent lines to the parabola y = x2 . The slope of the tangent at the point (x, x2 ) is 2x. The set of tangents form a one parameter family of lines, f (x, t) = t2 + (x t)2t = 2tx t2 . 1911

-1

-1

Figure 43.1: A parabola and its tangents. The parabola and some of its tangents are plotted in Figure 43.1. The parabola is the envelope of the family of tangent lines. Each point on the parabola is tangent to one of the lines. Given a curve, we can generate a family of lines that envelope the curve. We can also do the opposite, given a family of lines, we can determine the curve that they envelope. More generally, given a family of curves, we can determine the curve that they envelope. Let the one parameter family of curves be given by the equation F (x, y, t) = 0. For the example of the tangents to the parabola this equation would be y 2tx + t2 = 0. Let y(x) be the envelope of F (x, y, t) = 0. Then the points on y(x) must lie on the family of curves. Thus y(x) must satisfy the equation F (x, y, t) = 0. The points that lie on the envelope have the property, F (x, y, t) = 0. t We can solve this equation for t in terms of x and y, t = t(x, y). The equation for the envelope is then F (x, y, t(x, y)) = 0. 1912

Consider the example of the tangents to the parabola. The equation of the one-parameter family of curves is F (x, y, t) y 2tx + t2 = 0. The condition Ft (x, y, t) = 0 gives us the constraint, 2x + 2t = 0. Solving this for t gives us t(x, y) = x. The equation for the envelope is then, y 2xx + x2 = 0, y = x2 . Example 43.7.1 Consider the one parameter family of curves, (x t)2 + (y t)2 1 = 0. These are circles of unit radius and center (t, t). To determine the envelope of the family, we rst use the constraint Ft (x, y, t) to solve for t(x, y). Ft (x, y, t) = 2(x t) 2(y t) = 0 x+y t(x, y) = 2 Now we substitute this into the equation F (x, y, t) = 0 to determine the envelope. F x+y x, y, 2 = x+y x 2
2 2

x+y + y 2
2

1=0

xy 2

yx 2

1=0

(x y)2 = 2 y =x 2 The one parameter family of curves and its envelope is shown in Figure 43.2.

1913

-3

-2

-1

-1

-2

-3

Figure 43.2: The envelope of (x t)2 + (y t)2 1 = 0.

43.8

Exercises

Exercise 43.1 Consider the small transverse vibrations of a composite string of innite extent, made up of two homogeneous strings of dierent densities joined at x = 0. In each region 1) x < 0, 2) x > 0 we have utt c2 uxx = 0 j = 1, 2 c1 = c2 , j and we require continuity of u and ux at x = 0. Suppose for t < 0 a wave approaches the junction x = 0 from the left, i.e. as t approaches 0 from negative values: u(x, t) = F (x c1 t) x < 0, t 0 0 x > 0, t 0

As t increases further, the wave reaches x = 0 and gives rise to reected and transmitted waves. 1914

1. Formulate the appropriate initial values for u at t = 0. 2. Solve the initial-value problem for < x < , t > 0. 3. Identify the incident, reected and transmitted waves in your solution and determine the reection and transmission coecients for the junction in terms of c1 and c2 . Comment also on their values in the limit c1 c2 . Exercise 43.2 Consider a semi-innite string, x > 0. For all time the end of the string is displaced according to u(0, t) = f (t). Find the motion of the string, u(x, t) with the method of characteristics and then with a Fourier transform in time. The wave speed is c. Exercise 43.3 Solve using characteristics: uux + uy = 1, Exercise 43.4 Solve using characteristics: (y + u)ux + yuy = x y, u
y=1

x=y

x . 2

= 1 + x.

1915

43.9
Hint 43.1

Hints

Hint 43.2 1. Because the left end of the string is being displaced, there will only be right-moving waves. Assume a solution of the form u(x, t) = F (x ct). 2. Take a Fourier transform in time. Use that there are only outgoing waves. Hint 43.3 Hint 43.4

1916

43.10

Solutions

Solution 43.1 1. u(x, 0) = ut (x, 0) = F (x), x < 0 0, x>0 c1 F (x), x < 0 0, x>0

2. Regardless of the initial condition, the solution has the following form. u(x, t) = f1 (x c1 t) + g1 (x + c1 t), x < 0 f2 (x c2 t) + g1 (x + c2 t), x > 0

For x < 0, the right-moving wave is F (x c1 t) and the left-moving wave is zero for x < c1 t. For x > 0, there is no left-moving wave and the right-moving wave is zero for x > c2 t. We apply these restrictions to the solution. u(x, t) = F (x c1 t) + g(x + c1 t), x < 0 f (x c2 t), x>0

We use the continuity of u and ux at x = 0 to solve for f and g. F (c1 t) + g(c1 t) = f (c2 t) F (c1 t) + g (c1 t) = f (c2 t) We integrate the second equation. F (t) + g(t) = f (c2 t/c1 ) c1 F (t) + g(t) = f (c2 t/c1 ) + a c2 1917

We solve for f for x < c2 t and for g for x > c1 t. f (c2 t/c1 ) = 2c2 F (t) + b, c1 + c2 g(t) = c2 c1 F (t) + b c1 + c2

By considering the case that the solution is continuous, F (0) = 0, we conclude that b = 0 since f (0) = g(0) = 0. f (t) = 2c2 F (c1 t/c2 ), c1 + c2 g(t) = c2 c1 F (t) c1 + c2

Now we can write the solution for u(x, t) for t > 0. u(x, t) = F (x c1 t) +
2c2 F c1 +c2 c1 (x c2 c2 c1 F (x c1 +c2

c1 t)H(x + c1 t), x < 0 x>0

c2 t) H(c2 t x),

3. The incident, reected and transmitted waves are, respectively, F (x c1 t), c2 c1 F (x c1 t)H(x + c1 t), c1 + c2 2c2 F c1 + c2 c1 (x c2 t) H(c2 t x). c2

The reection and transmission coecients are, respectively, c1 c2 , c1 + c2 2c2 . c1 + c2

In the limit as c1 c2 , the reection coecient vanishes and the transmission coecient tends to unity. Solution 43.2 1. Method of characteristics. The problem is utt c2 uxx = 0, x > 0, < t < , u(0, t) = f (t). 1918

Because the left end of the string is being displaced, there will only be right-moving waves. The solution has the form u(x, t) = F (x ct). We substitute this into the boundary condition. F (ct) = f (t) F () = f c u(x, t) = f (t x/c) 2. Fourier transform. We take the Fourier transform in time of the wave equation and the boundary condition. u(0, t) = f (t) 2 u = c2 uxx , u(0, ) = f () 2 uxx + 2 u = 0, u(0, ) = f () c The general solution of this ordinary dierential equation is u(x, ) = a() ex/c +b() ex/c . The radiation condition, (u(x, t) must be a wave traveling in the positive direction), and the boundary condition at x = 0 will determine the constants a and b. Consider the solution u(x, t) we will obtain by taking the inverse Fourier transform of u.

utt = c2 uxx ,

u(x, t) =

a() ex/c +b() ex/c et d a() e(t+x/c) +b() e(tx/c) d

u(x, t) =

1919

The rst and second terms in the integrand are left and right traveling waves, respectively. In order that u is a right traveling wave, it must be a superposition of right traveling waves. We conclude that a() = 0. We apply the boundary condition at x = 0, we solve for u. u(x, ) = f () ex/c Finally we take the inverse Fourier transform.

u(x, t) =

f () e(tx/c) d

u(x, t) = f (t x/c) Solution 43.3 uux + uy = 1, We form


du . dy

x=y

x 2

(43.6)

du dx = ux + uy dy dy We compare this with Equation 43.6 to obtain dierential equations for x and u. dx = u, dy The initial data is x(y = ) = , du = 1. dy u(y = ) = (43.7)

. 2 We solve the dierenial equation for u (43.7) subject to the initial condition (43.8). u(x(y), y) = y 1920 2

(43.8)

The dierential equation for x becomes dx =y . dy 2 We solve this subject to the initial condition (43.8). 1 x(y) = (y 2 + (2 y)) 2 This denes the characteristic starting at the point (, ). We solve for . = We substitute this value for into the solution for u. u(x, y) = y(y 4) + 2x 2(y 2) y 2 2x y2

This solution is dened for y = 2. This is because at (x, y) = (2, 2), the characteristic is parallel to the line x = y. Figure 43.3 has a plot of the solution that shows the singularity at y = 2. Solution 43.4 (y + u)ux + yuy = x y, We dierentiate u with respect to s. du dx dy = ux + uy ds ds ds We compare this with Equation 43.9 to obtain dierential equations for x, y and u. dx = y + u, ds dy = y, ds 1921 du =xy ds u
y=1

=1+x

(43.9)

x 0 2 10 u 0 -10 -2 0 y

-2

Figure 43.3: The solution u(x, y). We parametrize the initial data in terms of s. x(s = 0) = , y(s = 0) = 1, u(s = 0) = 1 +

We solve the equation for y subject to the inital condition. y(s) = es This gives us a coupled set of dierential equations for x and u. dx = es +u, ds The solutions subject to the initial conditions are x(s) = ( + 1) es es , 1922 u(s) = es + es . du = x es ds

We substitute y(s) = es into these solutions. 1 x(s) = ( + 1)y , y u(s) = y + 1 y

We solve the rst equation for and substitute it into the second equation to obtain the solution. u(x, y) = 2 + xy y 2 y

This solution is valid for y > 0. The characteristic passing through (, 1) is x(s) = ( + 1) es es , y(s) = es .

Hence we see that the characteristics satisfy y(s) 0 for all real s. Figure 43.4 shows some characteristics in the (x, y) plane with starting points from (5, 1) to (5, 1) and a plot of the solution.

2 1.75 1.5 1.25 1 0.75 0.5 0.25 -10-7.5 -5 -2.5 2.5 5 7.5 10

-2 -1 x 0 1 2 15 10 u 5 0 0.5 1 y 1.5 2

Figure 43.4: Some characteristics and the solution u(x, y).

1923

Chapter 44 Transform Methods


44.1 Fourier Transform for Partial Dierential Equations

Solve Laplaces equation in the upper half plane u=0 u(x, 0) = f (x)
2

< x < , y > 0 <x<

Taking the Fourier transform in the x variable of the equation and the boundary condition, 2u 2u + = 0, F [u(x, 0)] = F [f (x)] x2 y 2 2 U (, 0) = F (). 2 U (, y) + 2 U (, y) = 0, y F The general solution to the equation is U (, y) = a ey +b ey . 1924

Remember that in solving the dierential equation here we consider to be a parameter. Requiring that the solution be bounded for y [0, ) yields U (, y) = a e||y . Applying the boundary condition, U (, y) = F () e||y . The inverse Fourier transform of e||y is F 1 e||y = Thus 2y + y2 2y F [u(x, y)] = F [f (x)] F 2 . x + y2 U (, y) = F () F x2 Recall that the convolution theorem is 1 F 2

2y . x2 + y 2

f (x )g() d = F ()G().

Applying the convolution theorem to the equation for U , u(x, y) = 1 2 y


f (x )2y d 2 + y2 f (x ) d. 2 + y2

u(x, y) =

1925

44.2

The Fourier Sine Transform


ut = uxx , u(0, t) = 0, x > 0, t > 0 u(x, 0) = f (x)

Consider the problem

Since we are given the position at x = 0 we apply the Fourier sine transform. ut = 2 u + 2 u(0, t) ut = 2 u
2t

u(, t) = c() e The initial condition is u(, 0) = f (). We solve the rst order dierential equation to determine u.

2 u(, t) = f () e t 1 2 ex /(4t) u(, t) = f ()Fc 4t

We take the inverse sine transform with the convolution theorem. u(x, t) = 4 3/2 1

f () e|x|
0

2 /(4t)

e(x+)

2 /(4t)

44.3

Fourier Transform
u u + u = 0, t x < x < , 1926 t > 0,

Consider the problem

u(x, 0) = f (x). Taking the Fourier Transform of the partial dierential equation and the initial condition yields U U + U = 0, t 1 U (, 0) = F () = 2

f (x) ex dx.

Now we have a rst order dierential equation for U (, t) with the solution U (, t) = F () e(1+)t . Now we apply the inverse Fourier transform.

u(x, t) =

F () e(1+)t ex d

u(x, t) = et

F () e(x+t) d

u(x, t) = et f (x + t)

1927

44.4

Exercises

Exercise 44.1 Find an integral representation of the solution u(x, y), of uxx + uyy = 0 in < x < , 0 < y < , subject to the boundary conditions: u(x, 0) = f (x), < x < ; u(x, y) 0 as x2 + y 2 . Exercise 44.2 Solve the Cauchy problem for the one-dimensional heat equation in the domain < x < , t > 0, ut = uxx , with the Fourier transform. Exercise 44.3 Solve the Cauchy problem for the one-dimensional heat equation in the domain < x < , t > 0, ut = uxx , with the Laplace transform. Exercise 44.4 1. In Exercise ?? above, let f (x) = f (x) for all x and verify that (x, t) so obtained is the solution, for x > 0, of the following problem: nd (x, t) satisfying t = a2 xx in 0 < x < , t > 0, with boundary condition (0, t) = 0 and initial condition (x, 0) = f (x). This technique, in which the solution for a semi-innite interval is obtained from that for an innite interval, is an example of what is called the method of images. 1928 u(x, 0) = f (x), u(x, 0) = f (x),

2. How would you modify the result of part (a) if the boundary condition (0, t) = 0 was replaced by x (0, t) = 0? Exercise 44.5 Solve the Cauchy problem for the one-dimensional wave equation in the domain < x < , t > 0, utt = c2 uxx , with the Fourier transform. Exercise 44.6 Solve the Cauchy problem for the one-dimensional wave equation in the domain < x < , t > 0, utt = c2 uxx , with the Laplace transform. Exercise 44.7 Consider the problem of determining (x, t) in the region 0 < x < , 0 < t < , such that t = a2 xx , with initial and boundary conditions (x, 0) = 0 for all x > 0, (0, t) = f (t) for all t > 0, where f (t) is a given function. 1. Obtain the formula for the Laplace transform of (x, t), (x, s) and use the convolution theorem for Laplace transforms to show that t x2 x 1 d. (x, t) = f (t ) 3/2 exp 2 4a 2a 0 1929 (44.1) u(x, 0) = f (x), ut (x, 0) = g(x), u(x, 0) = f (x), ut (x, 0) = g(x),

2. Discuss the special case obtained by setting f (t) = 1 and also that in which f (t) = 1 for 0 < t < T , with f (t) = 0 for t > T . Here T is some positive constant. Exercise 44.8 Solve the radiating half space problem: ut = uxx , x > 0, t > 0, ux (0, t) u(0, t) = 0, u(x, 0) = f (x). To do this, dene v(x, t) = ux (x, t) u(x, t) and nd the half space problem that v satises. Solve this problem and then show that

u(x, t) =
x

e(x) v(, t) d.

Exercise 44.9 Show that

e
0

c 2

x x2 /(4c) sin(x) d = 3/2 e . 4c

Use the sine transform to solve: ut = uxx , x > 0, t > 0, u(0, t) = g(t), u(x, 0) = 0. Exercise 44.10 Use the Fourier sine transform to nd the steady state temperature u(x, y) in a slab: x 0, 0 y 1, which has zero temperature on the faces y = 0 and y = 1 and has a given distribution: u(y, 0) = f (y) on the edge x = 0, 0 y 1. Exercise 44.11 Find a harmonic function u(x, y) in the upper half plane which takes on the value g(x) on the x-axis. Assume that u and ux vanish as |x| . Use the Fourier transform with respect to x. Express the solution as a single integral by using the convolution formula. 1930

Exercise 44.12 Find the bounded solution of ut = uxx a2 u, 0 < x < , t > 0, ux (0, t) = f (t), u(x, 0) = 0. Exercise 44.13 The left end of a taut string of length L is displaced according to u(0, t) = f (t). The right end is xed, u(L, t) = 0. Initially the string is at rest with no displacement. If c is the wave speed for the string, nd its motion for all t > 0. Exercise 44.14 Let 2 = 0 in the (x, y)-plane region dened by 0 < y < l, < x < , with (x, 0) = (x ), (x, l) = 0, and 0 as |x| . Solve for using Fourier transforms. You may leave your answer in the form of an integral but in fact it is possible to use techniques of contour integration to show that (x, y|) = sin(y/l) 1 . 2l cosh[(x )/l] cos(y/l)

Note that as l we recover the result derived in class: which clearly approaches (x ) as y 0. y 1 , (x )2 + y 2

1931

44.5

Hints

Hint 44.1 The desired solution form is: u(x, y) = K(x , y)f () d. You must nd the correct K. Take the Fourier transform with respect to x and solve for u(, y) recalling that uxx = 2 u. By uxx we denote the Fourier transform with respect to x of uxx (x, y). Hint 44.2 Use the Fourier convolution theorem and the table of Fourier transforms in the appendix. Hint 44.3 Hint 44.4 Hint 44.5 Use the Fourier convolution theorem. The transform pairs, F[((x + ) + (x ))] = cos( ), sin( ) F[(H(x + ) H(x ))] = , will be useful. Hint 44.6 Hint 44.7 Hint 44.8 v(x, t) satises the same partial dierential equation. You can solve the problem for v(x, t) with the Fourier sine 1932

transform. Use the convolution theorem to invert the transform. To show that e(x) v(, t) d, u(x, t) =
x

nd the solution of ux u = v that is bounded as x . Hint 44.9 Note that

e
0

c 2

sin(x) d = x

ec cos(x) d.

Write the integral as a Fourier transform. Take the Fourier sine transform of the heat equation to obtain a rst order, ordinary dierential equation for u(, t). Solve the dierential equation and do the inversion with the convolution theorem. Hint 44.10 Hint 44.11 Hint 44.12 Hint 44.13 Hint 44.14

1933

44.6

Solutions

Solution 44.1 1. We take the Fourier transform of the integral equation, noting that the left side is the convolution of u(x) and 1 . x2 +a2 1 1 2()F 2 u =F 2 x + a2 x + b2
1 We nd the Fourier transform of f (x) = x2 +c2 . Note that since f (x) is an even, real-valued function, f () is an even, real-valued function. 1 1 1 ex dx F 2 = 2 2 + c2 x +c 2 x

For x > 0 we close the path of integration in the upper half plane and apply Jordans Lemma to evaluate the integral in terms of the residues. 1 2 Res 2 ec = 2c 1 c e = 2c = Since f () is an even function, we have F Our equation for u() becomes, 2() u 1 a|| 1 b|| e e = 2a 2b a (ba)|omega| e u() = . 2b 1934 x2 1 c|| 1 e = . 2 +c 2c ex , x = c (x c)(x + c)

We take the inverse Fourier transform using the transform pair we derived above. u(x) = u(x) = a 2(b a) 2 + (b a)2 2b x a(b a) b(x2 + (b a)2 )

2. We take the Fourier transform of the partial dierential equation and the boundary condtion. uxx + uyy = 0, u(, y) + uyy (, y) = 0,
2

u(x, 0) = f (x) u(, 0) = f ()

This is an ordinary dierential equation for u in which is a parameter. The general solution is u = c1 ey +c2 ey . We apply the boundary conditions that u(, 0) = f () and u 0 and y . u(, y) = f () ey We take the inverse transform using the convolution theorem. 1 u(x, y) = 2 Solution 44.2 ut = uxx , u(x, 0) = f (x),

e(x)y f () d

We take the Fourier transform of the heat equation and the initial condition. ut = 2 u, u(, 0) = f () 1935

This is a rst order ordinary dierential equation which has the solution,
2 u(, t) = f () e t .

Using a table of Fourier transforms we can write this in a form that is conducive to applying the convolution theorem. u(, t) = f ()F 1 u(x, t) = 2 t Solution 44.3 We take the Laplace transform of the heat equation. ut = uxx s u(x, 0) = xx u u f (x) s uxx u = The Green function problem for Equation 44.2 is s G G = (x ), G(; ) is bounded.

x2 /(4t) e t e(x)
2 /(4t)

f () d

(44.2)

The homogeneous solutions that satisfy the left and right boundary conditions are, respectively, sa sa , exp . exp x x We compute the Wronskian of these solutions.

exp W =
s a

exp

s x a sa x

exp

s a

s x a

exp

sa x

= 2

1936

The Green function is

G(x; ) =

exp

s x <

exp
s

s x >

G(x; ) = exp 2 s

s |x | .

Now we solve Equation 44.2 using the Green function.

f () G(x; ) d s 1 u(x, s) = f () exp |x | 2 s u(x, s) =

Finally we take the inverse Laplace transform to obtain the solution of the heat equation.

1 u(x, t) = 2 t

f () exp

(x )2 4t

Solution 44.4 1. Clearly the solution satises the dierential equation. We must verify that it satises the boundary condition, 1937

(0, t) = 0. d 2a t 0 1 (x )2 1 (x )2 (x, t) = f () exp d + f () exp d 4a2 t 4a2 t 2a t 2a t 0 (x + )2 1 (x )2 1 (x, t) = f () exp d + f () exp d 4a2 t 4a2 t 2a t 0 2a t 0 1 (x + )2 1 (x )2 (x, t) = f () exp d + f () exp d 4a2 t 4a2 t 2a t 0 2a t 0 (x )2 (x + )2 1 f () exp exp d (x, t) = 4a2 t 4a2 t 2a t 0 x2 + 2 x x 1 f () exp exp exp 2 d (x, t) = 2t 2t 4a 2a 2a t 2a t 0 (x, t) = 1 (x, t) = a t

f () exp

(x )2 4a2 t

f () exp
0

x2 + 2 4a2 t

sinh

x 2a2 t

Since the integrand is zero for x = 0, the solution satises the boundary condition there. 2. For the boundary condition x (0, t) = 0 we would choose f (x) to be even. f (x) = f (x). The solution is 1 (x, t) = a t The derivative with respect to x is x (x, t) = 1 3/2 2a3 t

f () exp
0

x2 + 2 4a2 t

cosh

x 2a2 t

f () exp
0

x2 + 2 4a2 t

sinh

x 2a2 t

x cosh

x 2a2 t

d.

Since the integrand is zero for x = 0, the solution satises the boundary condition there. 1938

Solution 44.5 utt = c2 uxx , With the change of variables = ct, the problem becomes 1 v (x, 0) = g(x). c (This change of variables isnt necessary, it just gives us fewer constants to carry around.) We take the Fourier transform in x of the equation and the initial conditions, (we consider to be a parameter). v = vxx , v(x, 0) = f (x), v (, ) = 2 v (, ), v (, ) = f (), 1 v (, ) = g () c t 1 = = , t c t v(x, ) = u(x, t), u(x, 0) = f (x), ut (x, 0) = g(x),

Now we have an ordinary dierential equation for v (, ), (now we consider to be a parameter). The general solution of this constant coecient dierential equation is, v (, ) = a() cos( ) + b() sin( ), where a and b are constants that depend on the parameter . We applying the initial conditions to obtain v (, ). 1 v (, ) = f () cos( ) + g () sin( ) c With the Fourier transform pairs F[((x + ) + (x ))] = cos( ), sin( ) F[(H(x + ) H(x ))] = , we can write v (, ) in a form that is conducive to applying the Fourier convolution theorem. 1 v (, ) = F[f (x)]F[((x + ) + (x ))] + F[g(x)]F[(H(x + ) H(x ))] c 1939

v(x, ) =

1 2

f ()((x + ) + (x )) d

1 1 c 2

g()(H(x + ) H(x )) d
x+

1 1 v(x, ) = (f (x + ) + f (x )) + 2 2c

g() d
x

Finally we make the change of variables t = /c, u(x, t) = v(x, ) to obtain DAlemberts solution of the wave equation, 1 1 u(x, t) = (f (x ct) + f (x + ct)) + 2 2c Solution 44.6 With the change of variables = ct, the problem becomes 1 v (x, 0) = g(x). c We take the Laplace transform in of the equation, (we consider x to be a parameter), v = vxx , v(x, 0) = f (x), s2 V (x, s) sv(x, 0) v (x, 0) = Vxx (x, s), 1 Vxx (x, s) s2 V (x, s) = sf (x) g(x), c Now we have an ordinary dierential equation for V (x, s), (now we consider s to be a parameter). We impose the boundary conditions that the solution is bounded at x = . Consider the Greens function problem gxx (x; ) s2 g(x; ) = (x ), 1940 g(; ) bounded. t 1 = = , t c t v(x, ) = u(x, t),
x+ct

g() d.
xct

esx is a homogeneous solution that is bounded at x = . esx is a homogeneous solution that is bounded at x = +. The Wronskian of these solutions is W (x) = Thus the Greens function is g(x; ) = The solution for V (x, s) is V (x, s) = V (x, s) = V (x, s) = 1 2 1 2
1 2s esx es 1 2s es esx

esx esx = 2s. s esx s esx

for x < , 1 = es|x| . 2s for x > ,

1 2s

1 es|x| (sf () g()) d, c 1 2cs 1 2c

es|x| f () d +

es|x| g()) d,

es|| f (x ) d +

es|| g(x )) d. s

Now we take the inverse Laplace transform and interchange the order of integration. 1 v(x, ) = L1 2 v(x, ) = 1 2

es|| f (x ) d +

1 1 L 2c 1 2c 1 2c

es|| g(x )) d s es|| g(x )) d s

L1 es|| f (x ) d +

L1

v(x, ) =

1 2

( ||)f (x ) d +

H( ||)g(x )) d g(x ) d

1 1 v(x, ) = (f (x ) + f (x + )) + 2 2c 1941

1 1 v(x, ) = (f (x ) + f (x + )) + 2 2c 1 1 v(x, ) = (f (x ) + f (x + )) + 2 2c

x+

g() d
x x+

g() d
x

Now we write make the change of variables t = /c, u(x, t) = v(x, ) to obtain DAlemberts solution of the wave equation, 1 1 u(x, t) = (f (x ct) + f (x + ct)) + 2 2c Solution 44.7 1. We take the Laplace transform of Equation 44.1. s (x, 0) = a2 xx s xx 2 = 0 a
x+ct

g() d.
xct

(44.3)

We take the Laplace transform of the initial condition, (0, t) = f (t), and use that (x, s) vanishes as x to obtain boundary conditions for (x, s). (0, s) = f (s), The solutions of Equation 44.3 are exp The solution that satises the boundary conditions is s (x, s) = f (s) exp x . a 1942 (, s) = 0

s x . a

We write this as the product of two Laplace transforms. (x, s) = f (s)L x2 x 3/2 exp 2 4a t 2a t

We invert using the convolution theorem. x 2a


t

(x, t) =

f (t )
0

1 3/2

exp

x2 4a2

d.

2. Consider the case f (t) = 1.


t 1 x x2 exp 2 4a 2a 0 3/2 x x = , d = 4a 3/2 2a x/(2a t)

(x, t) =

2 (x, t) =

e d

(x, t) = erfc

x 2a t

Now consider the case in which f (t) = 1 for 0 < t < T , with f (t) = 0 for t > T . For t < T , is the same as before. (x, t) = erfc x 2a t 1943 , for 0 < t < T

Consider t > T . (x, t) = x 2a


t tT

1 3/2

exp

2 (x, t) = (x, t) = erf Solution 44.8

x/(2a t)

x2 4a2
2

e d erf x 2a t

x/(2a tT )

x 2a t T

ut = uxx , x > 0, t > 0, ux (0, t) u(0, t) = 0, u(x, 0) = f (x). First we nd the partial dierential equation that v satises. We start with the partial dierential equation for u, ut = uxx . Dierentiating this equation with respect to x yields, utx = uxxx . Subtracting times the former equation from the latter yields, utx ut = uxxx uxx , 2 (ux u) = 2 (ux u) , t x vt = vxx . 1944

Thus v satises the same partial dierential equation as u. This is because the equation for u is linear and homogeneous and v is a linear combination of u and its derivatives. The problem for v is, vt = vxx , x > 0, t > 0, v(0, t) = 0, v(x, 0) = f (x) f (x). With this new boundary condition, we can solve the problem with the Fourier sine transform. We take the sine transform of the partial dierential equation and the initial condition. vt (, t) = 2 v (, t) + 1 v(0, t) , v (, 0) = Fs [f (x) f (x)] vt (, t) = 2 v (, t) v (, 0) = Fs [f (x) f (x)] Now we have a rst order, ordinary dierential equation for v . The general solution is, v (, t) = c e t . The solution subject to the initial condition is, v (, t) = Fs [f (x) f (x)] e t . Now we take the inverse sine transform to nd v. We utilize the Fourier cosine transform pair,
1 Fc e
2t 2 2

x2 /(4t) e , t

to write v in a form that is suitable for the convolution theorem. v (, t) = Fs [f (x) f (x)] Fc 1945 x2 /(4t) e t

Recall that the Fourier sine convolution theorem is, Fs 1 2

f () (g(|x |) g(x + )) d = Fs [f (x)]Fc [g(x)].


0

Thus v(x, t) is 1 v(x, t) = 2 t

(f () f ()) e|x|
0

2 /(4t)

e(x+)

2 /(4t)

d.

With v determined, we have a rst order, ordinary dierential equation for u, ux u = v. We solve this equation by multiplying by the integrating factor and integrating. x e u = ex v x
x

ex u =
x

e v(x, t) d + c(t)

u= The solution that vanishes as x is

e(x) v(x, t) d + ex c(t)

u(x, t) =
x

e(x) v(, t) d.

1946

Solution 44.9
0

ec sin(x) d =

2 ec cos(x) d x 0 1 2 = ec cos(x) d 2 x 1 2 = ec +x d 2 x 1 2 2 = ec(+x/(2c)) ex /(4c) d 2 x 1 x2 /(4c) c2 e = e d 2 x 1 x2 /(4c) e = 2 c x x x2 /(4c) = 3/2 e 4c

ut = uxx , x > 0, t > 0, u(0, t) = g(t), u(x, 0) = 0. We take the Fourier sine transform of the partial dierential equation and the initial condition. ut (, t) = 2 u(, t) + g(t), u(, 0) = 0 Now we have a rst order, ordinary dierential equation for u(, t). 2 2 e t ut (, t) = g(t) e t t 2 t t 2 2 u(, t) = e g( ) e d + c() e t 0 1947

The initial condition is satised for c() = 0. u(, t) = We take the inverse sine transform to nd u.
1 u(x, t) = Fs t

g( ) e
0

2 (t )

g( ) e
0

2 (t )

u(x, t) =
0 t

1 g( )Fs

2 (t ) e d

u(x, t) =
0

x 2 ex /(4(t )) d g( ) 3/2 2 (t )
t 0

x u(x, t) = 2 Solution 44.10 The problem is

ex /(4(t )) g( ) d (t )3/2

uxx + uyy = 0, 0 < x, 0 < y < 1, u(x, 0) = u(x, 1) = 0, u(0, y) = f (y). We take the Fourier sine transform of the partial dierential equation and the boundary conditions. k u(0, y) + uyy (, y) = 0 k uyy (, y) 2 u(, y) = f (y), u(, 0) = u(, 1) = 0 2 u(, y) + 1948

This is an inhomogeneous, ordinary dierential equation that we can solve with Green functions. The homogeneous solutions are {cosh(y), sinh(y)}. The homogeneous solutions that satisfy the left and right boundary conditions are y1 = sinh(y), The Wronskian of these two solutions is, W (x) = sinh(y) sinh((y 1)) cosh(y) cosh((y 1)) y2 = sinh((y 1)).

= (sinh(y) cosh((y 1)) cosh(y) sinh((y 1))) = sinh(). The Green function is G(y|) = sinh(y< ) sinh((y> 1)) . sinh()

The solution of the ordinary dierential equation for u(, y) is u(, y) = 1 =


1

f ()G(y|) d
0 y

f ()
0

sinh() sinh((y 1)) 1 d sinh()

f ()
y

sinh(y) sinh(( 1)) d. sinh()

With some uninteresting grunge, you can show that,

2
0

sinh() sinh((y 1)) sin() sin(y) sin(x) d = 2 . sinh() (cosh(x) cos((y )))(cosh(x) cos((y + ))) 1949

Taking the inverse Fourier sine transform of u(, y) and interchanging the order of integration yields, u(x, y) = 2
y

f ()
0

sin() sin(y) d (cosh(x) cos((y )))(cosh(x) cos((y + ))) 2 1 sin(y) sin() d. + f () y (cosh(x) cos(( y)))(cosh(x) cos(( + y))) 2
1

u(x, y) = Solution 44.11 The problem for u(x, y) is,

f ()
0

sin() sin(y) d (cosh(x) cos((y )))(cosh(x) cos((y + )))

uxx + uyy = 0, < x < , y > 0, u(x, 0) = g(x). We take the Fourier transform of the partial dierential equation and the boundary condition. 2 u(, y) + uyy (, y) = 0, u(, 0) = g ().

This is an ordinary dierential equation for u(, y). So far we only have one boundary condition. In order that u is bounded we impose the second boundary condition u(, y) is bounded as y . The general solution of the dierential equation is c1 () ey +c2 () ey , for = 0, u(, y) = c1 () + c2 ()y, for = 0. Note that ey is the bounded solution for < 0, 1 is the bounded solution for = 0 and ey is the bounded solution for > 0. Thus the bounded solution is u(, y) = c() e||y . 1950

The boundary condition at y = 0 determines the constant of integration. u(, y) = g () e||y Now we take the inverse Fourier transform to obtain the solution for u(x, y). To do this we use the Fourier transform pair, 2c = ec|| , F 2 2 x +c and the convolution theorem, F 1 2

g f ()g(x ) d = f ()().

1 u(x, y) = 2

g()

2y d. (x )2 + y 2

Solution 44.12 Since the derivative of u is specied at x = 0, we take the cosine transform of the partial dierential equation and the initial condition. ut (, t) = 2 u(, t) 1 ux (0, t) a2 u(, t), u(, 0) = 0 ut + 2 + a2 u = f (t), u(, 0) = 0

This rst order, ordinary dierential equation for u(, t) has the solution, u(, t) =
t

e(
0

2 +a2 )(t )

f ( ) d.

1951

We take the inverse Fourier cosine transform to nd the solution u(x, t). u(x, t) = u(x, t) = t u(x, t) = 0 u(x, t) = Solution 44.13 Mathematically stated we have utt = c2 uxx , 0 < x < L, t > 0, u(x, 0) = ut (x, 0) = 0, u(0, t) = f (t), u(L, t) = 0. We take the Laplace transform of the partial dierential equation and the boundary conditions. s2 u(x, s) su(x, 0) ut (x, 0) = c2 uxx (x, s) 2 s uxx = 2 u, u(0, s) = f (s), u(L, s) = 0 c Now we have an ordinary dierential equation. A set of solutions is sx sx cosh , sinh . c c The solution that satises the right boundary condition is u = a sinh s(L x) c . 1 F c
t 1 Fc e 0 t

e(
0

2 +a2 )(t )

f ( ) d f ( ) d

2 (t )

ea

2 (t )

2 2 ex /(4(t )) ea (t ) f ( ) d (t )
t 0

ex

2 /(4(t ))a2 (t )

f ( ) d

1952

The left boundary condition determines the multiplicative constant. u(x, s) = f (s) If we can nd the inverse Laplace transform of u(x, s) = sinh(s(L x)/c) sinh(sL/c) sinh(s(L x)/c) sinh(sL/c)

then we can use the convolution theorem to write u in terms of a single integral. We proceed by expanding this function in a sum. es(Lx)/c es(Lx)/c sinh(s(L x)/c) = esL/c esL/c sinh(sL/c) esx/c es(2Lx)/c = 1 e2sL/c

= e =

sx/c

s(2Lx)/c n=0

e2nsL/c es(2(n+1)Lx)/c

es(2nL+x)/c
n=0 n=0

=
n=0

es(2nL+x)/c
n=1

es(2nLx)/c

Now we use the Laplace transform pair: L[(x a)] = esa . L


1

sinh(s(L x)/c) = sinh(sL/c)

(t (2nL + x)/c)
n=0 n=1

(t (2nL x)/c)

1953

We write u in the form,


u(x, s) = L[f (t)]L


n=0

(t (2nL + x)/c)
n=1

(t (2nL x)/c) .

By the convolution theorem we have


t

u(x, t) =
0

f ( )
n=0

(t (2nL + x)/c)
n=1

(t (2nL x)/c)

d.

We can simplify this a bit. First we determine which Dirac delta functions have their singularities in the range (0..t). For the rst sum, this condition is 0 < t (2nL + x)/c < t. The right inequality is always satised. The left inequality becomes (2nL + x)/c < t, ct x n< . 2L For the second sum, the condition is 0 < t (2nL x)/c < t. Again the right inequality is always satised. The left inequality becomes n< ct + x . 2L

We change the index range to reect the nonzero contributions and do the integration. ctx ct+x
t
2L 2L

u(x, t) =
0

f ( )
n=0

(t (2nL + x)/c)
n=1

(t (2nL x)/c) d.

1954

ctx 2L

ct+x 2L

u(x, t) =
n=0

f (t (2nL + x)/c)
n=1

f (t (2nL x)/c)

Solution 44.14 We take the Fourier transform of the partial dierential equation and the boundary conditions. 2 + yy = 0, We solve this boundary value problem. (, y) = c1 cosh((l y)) + c2 sinh((l y)) 1 sinh((l y)) e (, y) = 2 sinh(l) We take the inverse Fourier transform to obtain an expression for the solution. (x, y) = 1 2

1 e (, 0) = , 2

(, l) = 0

e(x)

sinh((l y)) d sinh(l)

1955

Chapter 45 Green Functions


45.1 Inhomogeneous Equations and Homogeneous Boundary Conditions
L[u(x)] = f (x) for x , For example, L[u] might be L[u] = ut u, and B[u] might be u = 0, or u n = 0. or L[u] = utt c2 u. B[u(x)] = 0 for x (45.1)

Consider a linear dierential equation on the domain subject to homogeneous boundary conditions.

If we nd a Green function G(x; ) that satises L[G(x; )] = (x ), then the solution to Equation 45.1 is u(x) =

B[G(x; )] = 0

G(x; )f () d.

1956

We verify that this solution satises the equation and boundary condition. L[u(x)] =

L[G(x; )]f () d (x )f () d

= f (x) B[u(x)] =

B[G(x; )]f () d 0 f () d

= =0

45.2

Homogeneous Equations and Inhomogeneous Boundary Conditions

Consider a homogeneous linear dierential equation on the domain subject to inhomogeneous boundary conditions, L[u(x)] = 0 for x , If we nd a Green function g(x; ) that satises L[g(x; )] = 0, then the solution to Equation 45.2 is u(x) =

B[u(x)] = h(x) for x .

(45.2)

B[g(x; )] = (x )

g(x; )h() d.

1957

We verify that this solution satises the equation and boundary condition. L[u(x)] =

L[g(x; )]h() d 0 h() d

= =0 B[u(x)] =

B[g(x; )]h() d (x )h() d

= h(x)

Example 45.2.1 Consider the Cauchy problem for the homogeneous heat equation. ut = uxx , < x < , t > 0 u(x, 0) = h(x), u(, t) = 0 We nd a Green function that satises gt = gxx , < x < , t > 0 g(x, 0; ) = (x ), g(, t; ) = 0. Then we write the solution u(x, t) =

g(x, t; )h() d.

To nd the Green function for this problem, we apply a Fourier transform to the equation and boundary condition 1958

for g. gt = 2 g , g (, 0; ) = F[(x )]
2t

g (, t; ) = F[(x )] e g (, t; ) = F[(x )]F We invert using the convolution theorem. g(x, t; ) = 1 2

x2 exp t 4t

(x )2 exp t 4t 2 1 (x ) = exp 4t 4t ( )

The solution of the heat equation is u(x, t) = 1 4t

exp

(x )2 4t

h() d.

45.3

Eigenfunction Expansions for Elliptic Equations


L[G] = (x ), x B[G] = 0, x (45.3)

Consider a Green function problem for an elliptic equation on a nite domain.

Let the set of functions {n } be orthonormal and complete on . (Here n is the multi-index n = n1 , . . . , nd .) n (x)m (x) dx = nm

1959

In addition, let the n be eigenfunctions of L subject to the homogeneous boundary conditions. L [n ] = n n , We expand the Green function in the eigenfunctions. G=
n

B [n ] = 0

gn n (x)

Then we expand the Dirac Delta function. (x ) =


n

dn n (x)

dn =

n (x)(x ) dx dn = n ()

We substitute the series expansions for the Green function and the Dirac Delta function into Equation 45.3. gn n n (x) =
n n

n ()n (x)

We equate coecients to solve for the gn and hence determine the Green function. n () n n ()n (x) G(x; ) = n n gn = Example 45.3.1 Consider the Green function for the reduced wave equation, u k 2 u in the rectangle, 0 x a, 0 y b, and vanishing on the sides. 1960

First we nd the eigenfunctions of the operator L = k 2 = 0. Note that = X(x)Y (y) is an eigenfunction of 2 2 L if X is an eigenfunction of x2 and Y is an eigenfunction of y2 . Thus we consider the two regular Sturm-Liouville eigenvalue problems: X = X, Y = Y, This leads us to the eigenfunctions mn = sin We use the orthogonality relation
2

X(0) = X(a) = 0 Y (0) = Y (b) = 0 ny mx sin . a b a dx = mn 2

sin
0

nx mx sin a a

to make the eigenfunctions orthonormal. ny mx 2 sin , mn = sin a b ab The mn are eigenfunctions of L. L [mn ] = m a
2

m, n Z+

n b

+ k 2 mn

By expanding the Green function and the Dirac Delta function in the mn and substituting into the dierential equation we obtain the solution.

G=
m,n=1

2 ab

sin

m a

sin

n b

2 ab

sin

mx a

sin

ny b

m 2 a mx a

n 2 b

+ k2
n b

G(x, y; , ) = 4ab
m,n=1

sin

sin m sin ny sin a b (mb)2 + (na)2 + (kab)2

1961

Example 45.3.2 Consider the Green function for Laplaces equation, u = 0 in the disk, |r| < a, and vanishing at r = a. First we nd the eigenfunctions of the operator = 2 1 1 2 + + 2 2. r2 r r r
d2 d2

We will look for eigenfunctions of the form = ()R(r). We choose the to be eigenfunctions of the periodic boundary conditions in . = , (0) = (2), n = e
in

subject to

(0) = (2)

nZ

We determine R(r) by requiring that be an eigenfunction of . = 1 1 (n R)rr + (n R)r + 2 (n R) = n R r r 1 1 n R + n R + 2 (n2 )n R = R r r For notational convenience, we denote = 2 . 1 n2 R + R + 2 2 r r The general solution for R is R = c1 Jn (r) + c2 Yn (r). The left boundary condition demands that c2 = 0. The right boundary condition determines the eigenvalues. Rnm = Jn jn,m r a 1962 , nm = jn,m a R = 0, R(0) bounded, R(a) = 0

Here jn,m is the mth positive root of Jn . This leads us to the eigenfunctions nm = ein Jn We use the orthogonality relations
2

jn,m r a

eim ein d = 2mn ,


0 1

rJ (j,m r)J (j,n r) dr =


0

1 2 (J (j,n )) mn 2

to make the eigenfunctions orthonormal. nm = The nm are eigenfunctions of L. nm = jn,m a


2

1 a|J n (jn,m )|

ein Jn

jn,m r a

n Z,

m Z+

nm

By expanding the Green function and the Dirac Delta function in the nm and substituting into the dierential equation we obtain the solution.
1 a|J
n (jn,m )|

ein Jn

G=
n= m=1

jn,m a

jn,m a

1 a|J 2
n (jn,m )|

ein Jn

jn,m r a

G(r, ; , ) =
n= m=1

1 (jn,m J n (jn,m ))2

ein() Jn

jn,m a

Jn

jn,m r a

1963

45.4

The Method of Images


u = f (x, y), < x < , y > 0 u(x, 0) = 0, u(x, y) 0 as x2 + y 2
2

Consider Poissons equation in the upper half plane.

The associated Green function problem is G = (x )(y ), < x < , y > 0 G(x, 0|, ) = 0, G(x, y|, ) 0 as x2 + y 2 . We will solve the Green function problem with the method of images. We expand the domain to include the lower half plane. We place a negative image of the source in the lower half plane. This will make the Green function odd about y = 0, i.e. G(x, 0|, ) = 0.
2 2

G = (x )(y ) (x )(y + ), < x < , G(x, y|, ) 0 as x2 + y 2

y>0

Recall that the innite space Green function which satises F = (x )(y ) is F (x, y|, ) = 1 ln (x )2 + (y )2 . 4

We solve for G by using the innite space Green function. G = F (x, y|, ) F (x, y|, ) 1 1 ln (x )2 + (y )2 ln (x )2 + (y + )2 = 4 4 1 (x )2 + (y )2 = ln 4 (x )2 + (y + )2 1964

We write the solution of Poissons equation using the Green function.


u(x, y) =
0

G(x, y|, )f (, ) d d 1 ln 4 (x )2 + (y )2 (x )2 + (y + )2 f (, ) d d

u(x, y) =
0

1965

45.5

Exercises

Exercise 45.1 Consider the Cauchy problem for the diusion equation with a source. ut uxx = s(x, t), u(x, 0) = f (x), u 0 as x

Find the Green function for this problem and use it to determine the solution. Exercise 45.2 Consider the 2-dimensional wave equation utt c2 (uxx + uyy ) = 0. 1. Determine the fundamental solution for this equation. (i.e. response to source at t = , x = ). You may nd the following information useful about the Bessel function: 1 J0 (x) =

ex cos d,
0

J0 (ax) sin(bx) dx =
0

0,
1 , b2 a2

0<b<a 0<a<b

2. Use the method of descents to recover the 1-D fundamental solution. Exercise 45.3 Consider the linear wave equation utt = c2 uxx , with constant c, on the innite domain < x < . 1. By using the Fourier transform nd the solution of Gtt = c2 Gxx subject to initial conditions G(x, 0) = 0, Gt (x, 0) = (x ). 1966

2. Now use this to nd u in the case where c = 1, u(x, 0) = 0, and ut (x, 0) = 0 |x| > 1 1 |x| |x| < 1

Sketch the solution in x for xed times t < 1 and t > 1 and also indicate on the x, t (t > 0) plane the regions of qualitatively dierent behavior of u. Exercise 45.4 Consider a generalized Laplace equation with non-constant coecients of the form:
2

u + A(x)

u + h(x)u = q(x),

on a region V with u = 0 on the boundary S. Suppose we nd a Green function which satises


2

G + A(x)

G + h(x)G = (x ).

Use the divergence theorem to derive an appropriate generalized Greens identity and show that u() =
V

G(x|)q(x) dx.

What equation should the Green function satisfy? Note: this equation is called the adjoint of the original partial dierential equation. Exercise 45.5 Consider Laplaces equation in the innite three dimensional domain with two sources of equal strength C, opposite sign and separated by a distance . 2 u = C(x + ) C(x ), where = ( 2 , 0, 0). 1. Find the solution in terms of the fundamental solutions. 1967

2. Now consider the limit in which the distance between sources goes to zero ( 0) and the strength increases in such a way that C = D remains xed. Show that the solution can be written u= Dx , 4r3

where r = |x|. This is called the response to a dipole located at the origin, with strength D, and oriented in the positive x direction. 3. Show that in general the response to a unit (D = 1) dipole at an arbitrary point 0 and oriented in the direction of the unit vector a is 1 1 a u(x) = 4 |x | =0 Exercise 45.6 Consider Laplaces equation
2

u = 0,

inside the unit circle with boundary condition u = f (). By using the Green function for the Dirichlet problem on the circle: 1 |x | G(x|) = ln , 2 |||x |
1 where and have the same polar angle and | | = || , show that the solution may be expressed in polar coordinates as 1 r2 2 f () d. u(r, ) = 2 2r cos( ) 2 1+r 0

Exercise 45.7 Consider an alternate derivation of the fundamental solution of Laplaces equation
2

u = (x),

with u 0 as |x| in three dimensions. 1968

1. Convert this equation to spherical coordinates. You may dene a new delta function 3 (r) = (x)(y)(z) such that
B

3 (r) dV =

1 if B contains the origin 0 otherwise

2. Show, by symmetry, that this can be reduced to an ordinary dierential equation. Solve to nd the general solution of the homogeneous equation. Now determine the constants by using the constraint that u 0 as |x| , and by integrating the partial dierential equation over a small ball around the origin (and using Gauss theorem). 3. Now use similar ideas to re-derive the fundamental solution in two dimensions. Can we still say u 0 as |x| ? Use instead the constraint that u = 0 when |x| = 1. 4. Finally derive the 2-D solution from the 3-D one using the method of descent. Consider Laplaces equation in three dimensions with a line source at x = 0, y = 0, < z < , uxx + uyy + uzz = (x)(y). Use the fundamental solution to nd u(r) where r = x2 + y 2 , and without loss of generality we have taken the plane at z = 0. Then evaluate this integral to nd u. (Hint: rst try to compute ur ) Exercise 45.8 Consider the heat equation on the bounded domain 0 < x < L with xed temperature at each end. Use Laplace transforms to determine the Green Function which satises Gt Gxx = (x )(t), G(0, t) = 0 G(L, t) = 0, G(x, 0 ) = 0. 1. First show that L[G(x, t)] = cosh
s (L

x> + x< ) cosh s 2 s sinh L 1969

s (L

x> x< )

2. Show that this can be re-written as L[G(x, t)] = s s 1 1 e |x2kL| e |x+2kL| . 2 s 2 s k=

3. Use this to nd G in terms of fundamental solutions


x2 1 e 4t , f (x, t) = 2 t

and comment on how this Greens function corresponds to real and image sources. Additionally compare this to the alternative expression, G(x, t) = 2 L

e
n=1

n2 2 t L2

sin

nx n sin , L L

and comment on the convergence of the respective formulations for small and large time. Exercise 45.9 Consider the Green function for the 1-D heat equation Gt Gxx = (x )(t ), on the semi-innite domain with insulated end Gx (0, t) = 0, and subject to the initial condition G(x, ) = 0. 1. Solve for G with the Fourier cosine transform. 1970 G 0 as x ,

2. (15 points) Relate this to the fundamental solution on the innite domain, and discuss in terms of responses to real and image sources. Give the solution for x > 0 of ut uxx = q(x, t), ux (0, t) = 0, u 0 as x , u(x, 0) = f (x). Exercise 45.10 Consider the heat equation ut = uxx + (x )(t), on the innite domain < x < , where we assume u 0 as x and initially u(x, 0 ) = 0. 1. First convert this to a problem where there is no forcing, so that ut = uxx with an appropriately modied initial condition. 2. Now use Laplace tranforms to convert this to an ordinary dierential equation in u(x, s), where u(x, s) = L[u(x, t)]. Solve this ordinary dierential equation and show that s 1 |x| u(x, s) = e . 2 s Recall f (s) = L[f (t)] =
st e 0

f (t) dt.

3. Finally use the Laplace inversion formula and Cauchys Theorem on an appropriate contour to compute u(x, t). Recall 1 f (t) = L1 [F (s)] = F (s) est ds, 2 where is the Bromwich contour (s = a + t where t ( . . . ) and a is a non-negative constant such that the contour lies to the right of all poles of f ). 1971

Exercise 45.11 Derive the causal Green function for the one dimensional wave equation on (..). That is, solve Gtt c2 Gxx = (x )(t ), G(x, t; , ) = 0 for t < . Use the Green function to nd the solution of the following wave equation with a source term. utt c2 uxx = q(x, t), u(x, 0) = ut (x, 0) = 0

Exercise 45.12 By reducing the problem to a series of one dimensional Green function problems, determine G(x, ) if
2

G = (x )

(a) on the rectangle 0 < x < L, 0 < y < H and G(0, y; , ) = Gx (L, y; , ) = Gy (x, 0; , ) = Gy (x, H; , ) = 0 (b) on the box 0 < x < L, 0 < y < H, 0 < z < W with G = 0 on the boundary. (c) on the semi-circle 0 < r < a, 0 < < with G = 0 on the boundary. (d) on the quarter-circle 0 < r < a, 0 < < /2 with G = 0 on the straight sides and Gr = 0 at r = a. Exercise 45.13 Using the method of multi-dimensional eigenfunction expansions, determine G(x, x0 ) if
2

G = (x x0 )

and 1972

(a) on the rectangle (0 < x < L, 0 < y < H) G =0 y G =0 y

at x = 0,

G=0 G =0 x

at y = 0,

at x = L,

at y = H,

(b) on the rectangular shaped box (0 < x < L, 0 < y < H, 0 < z < W ) with G = 0 on the six sides. (c) on the semi-circle (0 < r < a, 0 < < ) with G = 0 on the entire boundary. (d) on the quarter-circle (0 < r < a, 0 < < /2) with G = 0 on the straight sides and G/r = 0 at r = a. Exercise 45.14 Using the method of images solve
2

G = (x x0 )

in the rst quadrant (x 0 and y 0) with G = 0 at x = 0 and G/y = 0 at y = 0. Use the Green function to solve in the rst quadrant u=0 u(0, y) = g(y) u (x, 0) = h(x). y 1973
2

Exercise 45.15 Consider the wave equation dened on the half-line x > 0: 2u 2u = c2 2 + Q(x, t), t2 x u(x, 0) = f (x) u (x, 0) = g(x) t u(0, t) = h(t) (a) Determine the appropriate Greens function using the method of images. (b) Solve for u(x, t) if Q(x, t) = 0, f (x) = 0, and g(x) = 0. (c) For what values of t does h(t) inuence u(x1 , t1 ). Interpret this result physically. Exercise 45.16 Derive the Green functions for the one dimensional wave equation on (..) for non-homogeneous initial conditions. Solve the two problems gtt c2 gxx = 0, tt c2 xx = 0, using the Fourier transform. Exercise 45.17 Use the Green functions from Problem 45.11 and Problem 45.16 to solve utt c2 uxx = f (x, t), x > 0, < t < u(x, 0) = p(x), ut (x, 0) = q(x). Use the solution to determine the domain of dependence of the solution. 1974 g(x, 0; , ) = (x ), gt (x, 0; , ) = 0, (x, 0; , ) = 0, t (x, 0; , ) = (x ),

Exercise 45.18 Show that the Green function for the reduced wave equation, u k 2 u = 0 in the rectangle, 0 x a, 0 y b, and vanishing on the sides is: 2 G(x, y; , ) = a where n = k2 + n2 2 . a2

n=1

sinh(n y< ) sinh(n (y> b)) nx sin sin n sinh(n b) a

n a

Exercise 45.19 Find the Green function for the reduced wave equation u k 2 u = 0, in the quarter plane: 0 < x < , 0 < y < subject to the mixed boundary conditions: u(x, 0) = 0, Find two distinct integral representations for G(x, y; , ). Exercise 45.20 Show that in polar coordinates the Green function for u = 0 in the innite sector, 0 < < , 0 < r < , and vanishing on the sides is given by, r cosh ln cos ( ) 1 . G(r, , , ) = ln r 4 cosh ln cos ( + )

ux (0, y) = 0.

Use this to nd the harmonic function u(r, ) in the given sector which takes on the boundary values: u(r, ) = u(r, ) = 0 for r < c 1 for r > c.

1975

Exercise 45.21 The Green function for the initial value problem, ut uxx = 0, on < x < is G(x, t; ) = u(x, 0) = f (x),

1 2 e(x) /(4t) . 4t Use the method of images to nd the corresponding Green function for the mixed initial-boundary problems: 1. ut = uxx , 2. ut = uxx , u(x, 0) = f (x) for x > 0, u(x, 0) = f (x) for x > 0, u(0, t) = 0, ux (0, t) = 0.

Exercise 45.22 Find the Green function (expansion) for the one dimensional wave equation utt c2 uxx = 0 on the interval 0 < x < L, subject to the boundary conditions: a) u(0, t) = ux (L, t) = 0, b) ux (0, t) = ux (L, t) = 0. Write the nal forms in terms showing the propagation properties of the wave equation, i.e., with arguments ((x ) (t )). Exercise 45.23 Solve, using the above determined Green function, utt c2 uxx = 0, 0 < x < 1, t > 0, ux (0, t) = ux (1, t) = 0, u(x, 0) = x2 (1 x)2 , ut (x, 0) = 1. For c = 1, nd u(x, t) at x = 3/4, t = 7/2.

1976

45.6
Hint 45.1

Hints

Hint 45.2

Hint 45.3

Hint 45.4

Hint 45.5

Hint 45.6

Hint 45.7

Hint 45.8

Hint 45.9

Hint 45.10

1977

Hint 45.11 Hint 45.12 Take a Fourier transform in x. This will give you an ordinary dierential equation Green function problem for G. Find the continuity and jump conditions at t = . After solving for G, do the inverse transform with the aid of a table. Hint 45.13 Hint 45.14 Hint 45.15 Hint 45.16 Hint 45.17 Hint 45.18 Use Fourier sine and cosine transforms. Hint 45.19 The the conformal mapping z = w/ to map the sector to the upper half plane. The new problem will be Gxx + Gyy = (x )(y ), G(x, 0, , ) = 0, G(x, y, , ) 0 as x, y . Solve this problem with the image method. 1978 < x < , 0 < y < ,

Hint 45.20 Hint 45.21 Hint 45.22

1979

45.7

Solutions

Solution 45.1 The Green function problem is Gt Gxx = (x )(t ), G(x, t|, ) = 0 for t < , G 0 as x We take the Fourier transform of the dierential equation. Gt + 2 G = F[(x )](t ),

G(, t|, ) = 0 for t < Now we have an ordinary dierential equation Green function problem for G. The homogeneous solution of the ordinary dierential equation is 2 e t The jump condition is G(, 0; , + ) = F[(x )].

We write the solution for G and invert using the convolution theorem. 2 G = F[(x )] e (t ) H(t ) 2 ex /(4(t )) H(t ) (t ) 1 2 ey /(4(t )) dyH(t ) G= (x y ) 2 (t ) G = F[(x )]F G= 1 4(t ) e(x)
2 /(4(t ))

H(t )

We write the solution of the diusion equation using the Green function.

u=
0 t

G(x, t|, )s(, ) d d +


(x)2 /(4(t ))

G(x, t|, 0)f () d

u=
0

1 4(t )

1 s(, ) d d + 4t

e(x)

2 /(4t)

f () d

1980

Solution 45.2 1. We apply Fourier transforms in x and y to the Green function problem.

Gtt c2 (Gxx + Gyy ) = (t )(x )(y ) 1 1 e Gtt + c2 2 + 2 G = (t ) e 2 2 This gives us an ordinary dierential equation Green function problem for G(, , t). We nd the causal solution. That is, the solution that satises G(, , t) = 0 for t < .

G=

sin

2 + 2 c(t ) c 2 + 2

1 (+) e H(t ) 4 2

Now we take inverse Fourier transforms in and .


G=

e((x)+(y)) 4 2 c 2 + 2

sin

2 + 2 c(t ) d dH(t )

We make the change of variables = cos , = sin and do the integration in polar coordinates.
0

G=

1 4 2 c

2 0

e((x) cos +(y) sin ) sin (c(t )) d dH(t ) 1981

Next we introduce polar coordinates for x and y.

x = r cos , G= 1 4 2 c
0 0 0 0 2 2

y = r sin

er(cos cos +sin sin ) d sin (c(t )) dH(t ) er cos() d sin (c(t )) dH(t )

1 G= 2 4 c

1 G= J0 (r) sin (c(t )) dH(t ) 2c 0 1 1 G= H(c(t ) r)H(t ) 2c (c(t ))2 r2

G(x, t|, ) =

H(c(t ) |x |) 2c (c(t ))2 |x |2

2. To nd the 1D Green function, we consider a line source, (x)(t). Without loss of generality, we have taken the 1982

source to be at x = 0, t = 0. We use the 2D Green function and integrate over space and time. gtt c2 g = (x)(t)

H c(t ) 2c

(x )2 + (y )2 ()( ) d d d

g=

(c(t ))2 (x )2 (y )2

1 g= 2c

1 g= 2c 2

H ct

x2 + 2 d

(ct)2 x2 2 1

(ct) x2 (ct)2 x2

dH (ct |x|) (ct)2 x2 2 1 g(x, t|0, 0) = H (ct |x|) 2c 1 g(x, t|, ) = H (c(t ) |x |) 2c

Solution 45.3 1. Gtt = c2 Gxx , G(x, 0) = 0, Gt (x, 0) = (x ) Gtt = c2 2 G, G(, 0) = 0, Gt (, 0) = F[(x )] 1 G = F[(x )] sin(ct) c = F[(x )]F[H(ct |x|)] G c 1 G(x, t) = (x )H(ct ||) d c 2 1 G(x, t) = H(ct |x |) 2c 1983

2. We can write the solution of utt = c2 uxx , u(x, 0) = 0, ut (x, 0) = f (x)

in terms of the Green function we found in the previous part.

u=

G(x, t|)f () d

We consider c = 1 with the initial condition f (x) = (1 |x|)H(1 |x|). u(x, t) = 1 2


x+t

(1 ||)H(1 ||) d
xt

First we consider the case t < 1/2. We will use fact that the solution is symmetric in x. 0, 1 2
1

u(x, t) =

2 1 2 0, 0, 1 (1 + t + x)2 4 (1 + x)t 1 u(x, t) = 2 (2t t2 x2 ) (1 x)t 1 (1 + t x)2 4 0,

x+t (1 ||) d, 1 x+t (1 ||) d, xt 1 (1 ||) d, xt

x + t < 1 x t < 1 < x + t 1 < x t, x + t < 1 xt<1<x+t 1<xt x + t < 1 x t < 1 < x + t 1 < x t, x + t < 0 xt<0<x+t 0 < x t, x + t < 1 xt<1<x+t 1<xt

1984

Next we consider the case 1/2 < t < 1.

u(x, t) =

0, 1 2
1 2 1 2 0,

x+t (1 ||) d, 1 x+t (1 ||) d, xt 1 (1 ||) d, xt

x + t < 1 x t < 1 < x + t 1 < x t, x + t < 1 xt<1<x+t 1<xt x + t < 1 1 < x + t < 0 x t < 1, 0 < x + t 1 < x t, x + t < 1 x t < 0, 1 < x + t 0<xt<1 1<xt

0, 1 (1 + t + x)2 4 1 (1 t2 + 2t(1 x) + x(2 x)) 4 1 u(x, t) = 2 (2t t2 x2 ) 1 (1 t2 + 2t(1 + x) x(2 + x)) 4 1 (1 + t x)2 4 0,

1985

Finally we consider the case 1 < t. 0, 1 2


1 x+t (1 ||) d, 1 1 (1 ||) d, 1 1 (1 ||) d, xt

u(x, t) =

2 1 2 0,

x + t < 1 1 < x + t < 1 x t < 1, 1 < x + t 1 < x t < 1 1<xt

x + t < 1 1 < x + t < 0 0<x+t<1 1 u(x, t) = 2 x t < 1, 1 < x + t 1 (1 (t x 2)(t x)) 1 < x t < 0 4 1 (1 + t x)2 0<xt<1 4 0, 1<xt Figure 45.1 shows the solution at t = 1/2 and t = 2.
0.5 0.4 0.3 0.2 0.1 -2 -1 1 2 -4 -2 0.5 0.4 0.3 0.2 0.1 2 4

0, 1 (1 + t + x)2 4 1 (1 (t + x 2)(t + x)) 4

Figure 45.1: The solution at t = 1/2 and t = 2. Figure 45.2 shows the behavior of the solution in the phase plane. There are lines emanating form x = 1, 0, 1 showing the range of inuence of these points. 1986

u=1

u=0

u=0 x

Figure 45.2: The behavior of the solution in the phase plane. Solution 45.4 We dene L[u]
2

u + a(x)

u + h(x)u.

We use the Divergence Theorem to derive a generalized Greens Theorem. uL[v] dx =


V V

u( v+

v+a

v + hv) dx (au) + huv) dx (u v v u + uva) n dA


V

uL[v] dx =
V V

(u

(uva) v

uL[v] dx =
V V

v(

(au) + hu) dx +

(uL[v] vL [u]) dx =
V V

(u v v u + uva) n dA

We dene the adjoint operator L . L [u] =


2

u 1987

(au) + hu

We substitute the solution u and the adjoint Green function G into the generalized Greens Theorem.

(G L[u] uL [G ]) dx =
V V

(G u u G + vG a) n dA

(G q uL [G ]) dx = 0
V

If the adjoint Green function satises L [G ] = (x) then we can write u as an integral of the adjoint Green function and the inhomegeneity. u() =
V

G (x|)q(x) dx

Thus we see that the adjoint Green function problem is the appropriate one to consider. For L[G] = (x ),

u() =
V

G(x|)q(x) dx

Solution 45.5 1.
2

u = C(x + ) C(x ) C C + u= 4|x + | 4|x | 1988

2. We take c = D/ and consider the limit 0. u = lim


0

D 4 D 4

1 (x /2)2 + y 2 + z 2

1 (x + /2)2 + y 2 + z 2 (x /2)2 + y 2 + z 2

u = lim
0

(x + /2)2 + y 2 + z 2

((x /2)2 + y 2 + z 2 )((x + /2)2 + y 2 + z 2 ) x x D r + 2r + O( 2 ) r 2r + O( 2 ) u = lim 0 r2 + O( ) 4 D x + O( ) r u = lim 0 4 r2 + O( ) Dx u= 4r3

3. Let = 0 a/2.
2

u = lim
0

(x + ) (x )

u=

1 1 lim 4 0

1 1 |x ( 0 + a/2)| |x ( 0 a/2)|

We note that this is the denition of a directional derivative. u(x) = Solution 45.6 The Green function is G(x|) = 1 ln 2 |x | |||x | . 1 4

1 |x |

a
=0

1989

We write this in polar coordinates. Denote x = r e and = e . Let = be the dierence in angle between x and . G(x|) = 1 ln 2 r2 + 2 2r cos r2 + 1/2 2(r/) cos r2 + 2 2r cos r2 2 + 1 2r cos

G(x|) =

1 ln 4

We solve Laplaces equation with the Green function. u(x) = u(r, ) =


0 4

f ()
2

G(x|)

n ds

f ()G (r, |1, ) d

G =

1 r + r(r2 1)(2 + 1) cos 2 (r2 + 2 2r cos )(r2 2 + 1 2r cos ) 1 r2 1 G (r, |1, ) = 2 1 + r2 2r cos 2 1 r2 f () u(r, ) = d 2 2r cos( ) 2 1+r 0

Solution 45.7 1. 1 r2 r G r G = (x )(y )(z ) 1 G 1 2G + 2 sin() + 2 = 3 (r) r sin r sin 2

r2

2. Since the Green function has spherical symmetry, G = G = 0. This reduces the problem to an ordinary dierential equation. 1 G r2 = 3 (r) 2 r r r 1990

We nd the homogeneous solutions. 2 urr + ur = 0 r 2 ln r ur = c e = cr2 c1 u= + c2 r We consider the solution that vanishes at innity. u= c r

Thus we see that G = c/r. We determine the constant by integrating G over a sphere about the origin, R. G dx = 1
R

G n ds = 1
R

Gr ds = 1
R 0 0 2

c 2 r sin() dd = 1 r2 4c = 1 1 c= 4 1 G= 4r 1991

3. We write the Laplacian in circular coordinates.

G = (x )(y ) 1 G 1 2G r + 2 2 = 2 (r) r r r r

Since the Green function has circular symmetry, G = 0. This reduces the problem to an ordinary dierential equation. 1 r r r G r = 2 (r)

We nd the homogeneous solutions.

1 urr + ur = 0 r ur = c e ln r = cr1 u = c1 ln r + c2

There are no solutions that vanishes at innity. Instead we take the solution that vanishes at r = 1.

u = c ln r 1992

Thus we see that G = c ln r. We determine the constant by integrating G over a ball about the origin, R.

G dx = 1
R

G n ds = 1
R

Gr ds = 1
R 2 0

c r d = 1 r 2c = 1 1 ln r 2

G=

1993

4.

u= 1 u= 4

1 ()() d d d 4(r ) ()()

(x )2 + (y )2 + (z )2 1 1 u= d 4 x2 + y 2 + (z )2 1 1 u= d 4 r2 + 2 1 r ur = d 2 + 2 )3/2 4 (r 1 2 ur = 4 r 1 u= ln r 2

d d d

Solution 45.8 1. We take the Laplace transform of the dierential equation and the boundary conditions in x. Gt Gxx = (x )(t ) sG Gxx = (x ) s 1 Gxx G = (x ), G(0, t) = G(L, t) = 0

Now we have an ordinary dierential equation Green function problem. We nd homogeneous solutions which respectively satisfy the left and right boundary conditions and compute their Wronskian. y1 = sinh s x , 1994 y2 = sinh s (L x)

W =

sinh s cosh

s x s x

s sinh (L x) s s cosh (L x)

= 2 = 2

s sinh s sinh

s x cosh s L

s (L x) + cosh

s x sinh

s (L x)

We write the Green function in terms of the homogeneous solutions of the Wronskian. 1 G= 2 1 sinh sinh sinh s x< sinh
s (L s L

G= G= cosh

s L s x <

s (L x> )

sinh 2 s sinh

x> )
s (L

s (L

x> + x< ) cosh s 2 s sinh L

x> x< )

2. We expand 1/ sinh(x) in a series. 1 2 = x e ex sinh(x) 2 ex = 1 e2x

= 2e =2

x n=0

e2nx e(2n+1)x

n=0

1995

We use the expansion of the hyperbolic cosecant in our expression for the Green function. e s/(Lx> +x< ) + e s/(Lx> +x< ) e s/(Lx> x< ) e s/(Lx> x< ) G= s 4 s sinh L 1 e /s(Lx> +x< ) + e /s(Lx> +x< ) G= 2 s e
/s(Lx> x< )

/s(Lx> x< ) n=0

(2n+1)

s/L

1 G= 2 s

s/(x> +x< 2nL)

+
n=0

s/(x> x< 2(n+1)L)

n=0

s/(x> x< 2nL)

n=0

n=0

s/(x> +x< 2(n+1)L)

1 G= 2 s

1 s/(x> +x< 2nL)

+
n=

s/(x> x< +2nL)

n=0

1 s/(x> x< 2nL)

e
s/(x> +x< +2nL)

n=0

n=

1 G= 2 s

e
n=

s/|x< x> 2nL|

n=

s/|x< +x> 2nL|

1 G= 2 s

e
n=

s/|x2nL|

n=

s/|x+2nL|

1996

3. We take the inverse Laplace transform to nd the Green function for the diusion equation. 1 G= 2 t

e
n=

(x2nL)2 /(4t)

n=

e(x+2nL)

2 /(4t)

G=
n=

f (x 2nL, t)
n=

f (x + 2nL, t)

On the interval (L . . . L), there is a real source at x = and a negative image source at x = . This pattern is repeated periodically. The above formula is useful when approximating the solution for small time, t 1. For such small t, the terms decay very quickly away from n = 0. A small number of terms could be used for an accurate approximation. The alternate formula is useful when approximating the solution for large time, t 1. For such large t, the terms in the sine series decay exponentially Again, a small number of terms could be used for an accurate approximation. Solution 45.9 1. We take the Fourier cosine transform of the dierential equation. Gt Gxx = (x )(t ) 1 Gt 2 G Gx (0, t) = Fc [(x )](t ) 2 Gt + G = Fc [(x )](t )
2 G = Fc [(x )] e (t ) H(t ) 2 ex /(4(t )) H(t ) G = Fc [(x )]Fc (t )

1997

We do the inversion with the convolution theorem. 1 G= 2

( )
0

2 2 e|x| /(4(t )) + e(x+) /(4(t )) dH(t ) (t ) 1 e(x)


2 /(4(t ))

G(x, t; , ) =

4(t )

+ e(x+)

2 /(4(t ))

H(t )

2. The fundamental solution on the innite domain is F (x, t; , ) = 1 4(t ) e(x)


2 /(4(t ))

H(t ).

We see that the Green function on the semi-innite domain that we found above is a sum of fundamental solutions. G(x, t; , ) = F (x, t; , ) + F (x, t; , ) Now we solve the inhomogeneous problem.
t

u(x, t) =
0 0

G(x, t; , )q(, ) d d +
0

G(x, t; , 0)f () d

u(x, t) =

1 4

t 0 0

1 2 2 e(x) /(4(t )) + e(x+) /(4(t )) q(, ) d d t 1 2 2 e(x) /(4t) + e(x+) /(4t) f () d + 4t 0

Solution 45.10 1. We integrate the heat equation from t = 0 to t = 0+ to determine an initial condition. ut = uxx + (x )(t) u(x, 0+ ) u(x, 0 ) = (x ) 1998

Now we have an initial value problem with no forcing. ut = uxx , for t > 0, u(x, 0) = (x )

2. We take the Laplace transform of the initial value problem. s u(x, 0) = uxx u 1 s uxx u = (x ), u(, s) = 0 The solutions that satisfy the left and right boundary conditions are, respectively, s/x s/x u1 = e , u2 = e We compute the Wronskian of these solutions and then write the solution for u. s e s/x e s/x = 2 W = s/x s/x s/ e s/ e 1 e s/x< e s/x> u= s 2 1 u = e s/|x| 2 s 3. In Exercise 31.16, we showed that ea/t 2as e = . s t We use this result to do the inverse Laplace transform. L1 1 2 e(x) /(4t) u(x, t) = 2 t

1999

Solution 45.11 Gtt c2 Gxx = (x )(t ), G(x, t; , ) = 0 for t < . We take the Fourier transform in x. Gtt + c2 2 G = F[(x )](t ), G(, 0; , ) = Gt (, 0; , ) = 0 Now we have an ordinary dierential equation Green function problem for G. We have written the causality condition, the Green function is zero for t < , in terms of initial conditions. The homogeneous solutions of the ordinary dierential equation are {cos(ct), sin(ct)}. It will be handy to use the fundamental set of solutions at t = : cos(c(t )), The continuity and jump conditions are G(, 0; , + ) = 0, 1 sin(c(t )) . c

Gt (, 0; , + ) = F[(x )]

We write the solution for G and invert using the convolution theorem. 1 G = F[(x )]H(t ) sin(c(t )) c = H(t )F[(x )]F H(c(t ) |x|) G c 1 (y )H(c(t ) |x y|) dy G = H(t ) c 2 1 G = H(t )H(c(t ) |x |) 2c 1 G = H(c(t ) |x |) 2c 2000

The Green function for = = 0 and c = 1 is plotted in Figure 45.3 on the domain x (1..1), t (0..1). The 1 Green function is a displacement of height 2c that propagates out from the point x = in both directions with speed c. The Green function shows the range of inuence of a disturbance at the point x = and time t = . The disturbance inuences the solution for all ct < x < + ct and t > .
1 0.8 0.6 t 0.4 0.2 0 0.4 0.2 -1 -0.5 0 x 0.5 0 1

Figure 45.3: Green function for the wave equation. Now we solve the wave equation with a source. utt c2 uxx = q(x, t),

u(x, 0) = ut (x, 0) = 0

u=
0

G(x, t|, t)q(, ) d d 1 H(c(t ) |x |)q(, ) d d 2c


t 0 x+c(t )

u=
0

u=

1 2c

q(, ) d d
xc(t )

2001

Solution 45.12 1. We expand the Green function in eigenfunctions in x.

G(x; ) =
n=1

an (y) sin

(2n 1)x 2L

We substitute the expansion into the dierential equation.


2 n=1

an (y)

2 sin L

(2n 1)x 2L

= (x )(y )

an (y)
n=1

(2n 1) 2L

an (y)

2 sin L

(2n 1)x 2L

= (y )
n=1

2 sin L

(2n 1) 2L

2 sin L

(2n 1)x 2L

an (y)

(2n 1) 2L

an (y) =

2 sin L

(2n 1) 2L

(y )

From the boundary conditions at y = 0 and y = H, we obtain boundary conditions for the an (y). an (0) = an (H) = 0. The solutions that satisfy the left and right boundary conditions are an1 = cosh The Wronskian of these solutions is W = (2n 1) sinh 2L 2002 (2n 1) 2 . (2n 1)y 2L , an2 = cosh (2n 1)(H y) 2L .

Thus the solution for an (y) is 2 sin L (2n 1) 2L cosh


(2n1)y< 2L

cosh

an (y) =

(2n1)(Hy> ) 2L (2n1) 2

(2n1) sinh 2L

2 2L an (y) = csch (2n 1)

(2n 1) 2

cosh

(2n 1)y< 2L cosh (2n 1)(H y> ) 2L sin (2n 1) 2L .

This determines the Green function. 2 2L G(x; ) =

n=1

1 csch 2n 1

(2n 1) 2 cosh

cosh

(2n 1)y< 2L sin (2n 1) 2L sin (2n 1)x 2L

(2n 1)(H y> ) 2L

2. We seek a solution of the form

G(x; ) =
m=1 n=1

amn (z)

2 mx ny sin sin . L H LH

We substitute this into the dierential equation.


2 m=1 n=1

amn (z)

2 mx ny sin sin L H LH

= (x )(y )(z )

2003

amn (z)
m=1 n=1

m L

n H

amn (z)

mx 2 ny sin sin L H LH m L sin n H 2 mx ny sin sin L H LH (z )

= (z )
m=1 n=1

2 sin LH

n m m 2 n 2 2 + sin sin amn (z) = L H L H LH From the boundary conditions on G, we obtain boundary conditions for the amn . amn (z) amn (0) = amn (W ) = 0 The solutions that satisfy the left and right boundary conditions are amn1 = sinh m L
2

n H

amn2 = sinh

m L

n H

(W z) .

The Wronskian of these solutions is W = Thus the solution for amn (z) is amn (z) = 2 sin LH m L sin n H sinh
m 2 L

m L

n H

sinh

m L

n H

n 2 H

z< sinh sinh

m 2 L m 2 L

n 2 H n 2 H

(W z> )

m 2 L

n 2 H

2004

amn (z) =

mn LH

csch (mn W ) sin

m L

sin

n H sinh (mn z< ) sinh (mn (W z> )) ,

where mn = This determines the Green function. 4 G(x; ) = LH

m L m L

n H

1 mn

csch (mn W ) sin

sin

m=1 n=1

mx L sin ny sinh (mn z< ) sinh (mn (W z> )) H

sin 3. First we write the problem in circular coordinates.

n H

G = (x ) 1 1 1 Grr + Gr + 2 G = (r )( ), r r r G(r, 0; , ) = G(r, ; , ) = G(0, ; , ) = G(a, ; , ) = 0 Because the Green function vanishes at = 0 and = we expand it in a series of the form

G=
n=1

gn (r) sin(n).

We substitute the series into the dierential equation.

n=1

1 n2 1 2 gn (r) + gn (r) 2 gn (r) sin(n) = (r ) sin(n) sin(n) r r r n=1 1 n2 2 sin(n)(r ) gn (r) + gn (r) 2 gn (r) = r r r 2005

From the boundary conditions on G, we obtain boundary conditions for the gn . gn (0) = gn (a) = 0 The solutions that satisfy the left and right boundary conditions are a r n gn1 = rn , gn2 = a r The Wronskian of these solutions is 2nan W = . r Thus the solution for gn (r) is gn (r) = 2 sin(n)
n r< r> n a n

a r>

2nan n

r< 1 sin(n) gn (r) = n a This determines the solution.

r> a

a r>

G=
n=1

1 n

r< a

r> a

a r>

sin(n) sin(n)

4. First we write the problem in circular coordinates. 1 1 1 Grr + Gr + 2 G = (r )( ), r r r G(r, 0; , ) = G(r, /2; , ) = G(0, ; , ) = Gr (a, ; , ) = 0 Because the Green function vanishes at = 0 and = /2 we expand it in a series of the form

G=
n=1

gn (r) sin(2n).

2006

We substitute the series into the dierential equation.

n=1

1 4n2 1 4 sin(2n) sin(2n) gn (r) + gn (r) 2 gn (r) sin(2n) = (r ) r r r n=1 1 4n2 4 gn (r) + gn (r) 2 gn (r) = sin(2n)(r ) r r r

From the boundary conditions on G, we obtain boundary conditions for the gn . gn (0) = gn (a) = 0 The solutions that satisfy the left and right boundary conditions are gn1 = r2n , The Wronskian of these solutions is W = Thus the solution for gn (r) is 4 sin(2n)
2n r< r> 2n a

gn2 =

r a

2n

a r

2n

4na2n . r
2n

+
2n

a r>

gn (r) = gn (r) = This determines the solution.

4na
2n

1 r< sin(2n) n a

r> a

2n

a r>

2n

G=
n=1

1 r< n a

2n

r> a

2n

a r>

2n

sin(2n) sin(2n)

2007

Solution 45.13 1. The set {Xn } =


2

sin

(2m 1)x 2L

m=1

are eigenfunctions of

and satisfy the boundary conditions Xn (0) = Xn (L) = 0. The set {Yn } = cos ny H
n=0

are eigenfunctions of

and satisfy the boundary conditions Yn (0) = Yn (H) = 0. The set (2m 1)x 2L ny cos H
m=1,n=0

sin

are eigenfunctions of 2 and satisfy the boundary conditions of this problem. We expand the Green function in a series of these eigenfunctions.

G=
m=1

gm0

2 sin LH

(2m 1)x 2L

+
m=1 n=1

gmn

2 sin LH

(2m 1)x 2L

cos

ny H

We substitute the series into the Green function dierential equation. G = (x )(y ) 2008

m=1

gm0

(2m 1) 2L

2 sin LH (2m 1) 2L

(2m 1)x 2L
2

m=1 n=1

gmn

ny H

2 sin LH 2 sin LH

(2m 1)x 2L (2m 1)x 2L 2 sin LH

cos

ny H

=
m=1

2 sin LH 2 sin LH

(2m 1) 2L (2m 1) 2L

+
m=1 n=1

cos

n H

(2m 1)x 2L

cos

ny H

We equate terms and solve for the coecients gmn . gm0 = gmn = 2 LH 2 2 LH 1
2m1 2 2L

2L (2m 1) +
n 2 H

sin sin

(2m 1) 2L (2m 1) cos 2L

n H

This determines the Green function. 2. Note that 8 sin LHW kx L , sin my nz , sin H W : k, m, n Z+
2

is orthonormal and complete on (0 . . . L) (0 . . . H) (0 . . . W ). The functions are eigenfunctions of expand the Green function in a series of these eigenfunctions.

. We

G=
k,m,n=1

gkmn

8 sin LHW 2009

kx L

sin

my nz sin H W

We substitute the series into the Green function dierential equation. G = (x )(y )(z )

k,m,n=1

gkmn

k L

m H

n W

8 sin LHW k L sin

kx L m H

sin

my nz sin H W n W kx L sin my nz sin H W

=
k,m,n=1

8 sin LHW

sin

8 sin LHW We equate terms and solve for the coecients gkmn . gkmn = This determines the Green function. 3. The Green function problem is
8 LHW

sin

k L

sin
m 2 H

m H

sin
n 2 W

n W

k 2 L

1 1 1 G Grr + Gr + 2 G = (r )( ). r r r We seek a set of functions {n ()Rnm (r)} which are orthogonal and complete on (0 . . . a) (0 . . . ) and which 2 are eigenfunctions of the laplacian. For the n we choose eigenfunctions of 2 . = 2 , (0) = () = 0 n = n, n = sin(n), n Z+

2010

Now we look for eigenfunctions of the laplacian. 1 1 (Rn )rr + (Rn )r + 2 (Rn ) = 2 Rn r r 1 n2 R n + R n 2 Rn = 2 Rn r r 1 n2 R + R + 2 2 R = 0, R(0) = R(a) = 0 r r The general solution for R is R = c1 Jn (r) + c2 Yn (r). the solution that satises the left boundary condition is R = cJn (r). We use the right boundary condition to determine the eigenvalues. jn,m r jn,m , Rnm = Jn , m, n Z+ m = a a here jn,m is the mth root of Jn . Note that jn,m r : m, n Z+ a is orthogonal and complete on (r, ) (0 . . . a) (0 . . . ). We use the identities sin(n)Jn

sin2 (n) d =
0

, 2

1 0

1 2 2 rJn (jn,m r) dr = Jn+1 (jn,m ) 2 jn,m r a

to make the functions orthonormal. 2 a|Jn+1 (jn,m )|

sin(n)Jn

: m, n Z+

We expand the Green function in a series of these eigenfunctions. G=


n,m=1

gnm

2 a|Jn+1 (jn,m )| 2011

Jn

jn,m r a

sin(n)

We substitute the series into the Green function dierential equation. 1 1 1 Grr + Gr + 2 G = (r )( ) r r r
2

n,m=1

jn,m a

gnm

2 a|Jn+1 (jn,m )|

Jn 2

jn,m r a Jn

sin(n) jn,m a sin(n) 2 a|Jn+1 (jn,m )| Jn jn,m r a sin(n)

=
n,m=1

a|Jn+1 (jn,m )|

We equate terms and solve for the coecients gmn . gnm = This determines the green function. 4. The Green function problem is 1 1 1 G Grr + Gr + 2 G = (r )( ). r r r We seek a set of functions {n ()Rnm (r)} which are orthogonal and complete on (0 . . . a) (0 . . . /2) and 2 which are eigenfunctions of the laplacian. For the n we choose eigenfunctions of 2 . = 2 , (0) = (/2) = 0 n = 2n, n = sin(2n), n Z+ a jn,m
2

2 a|Jn+1 (jn,m )|

Jn

jn,m a

sin(n)

2012

Now we look for eigenfunctions of the laplacian. 1 1 (Rn )rr + (Rn )r + 2 (Rn ) = 2 Rn r r 1 (2n)2 R n + R n 2 Rn = 2 Rn r r 1 (2n)2 R + R + 2 2 R = 0, R(0) = R(a) = 0 r r The general solution for R is R = c1 J2n (r) + c2 Y2n (r). the solution that satises the left boundary condition is R = cJ2n (r). We use the right boundary condition to determine the eigenvalues. m = here j n,m is the mth root of J n . Note that j 2n,m r : m, n Z+ a is orthogonal and complete on (r, ) (0 . . . a) (0 . . . /2). We use the identities sin(2n)J
2n

j 2n,m , a

Rnm = J2n

j 2n,m r a

m, n Z+

mn , 2 0 1 j 2 2 ,n rJ (j ,m r)J (j ,n r) dr = J (j ,n ) 2j 2 0 ,n sin(m) sin(n) d = to make the functions orthonormal. 2j 2n,m sin(2n)J2n a j 2 2 |J (j 2n 2n,m )| 2n,m 4n 2013

mn

j 2n,m r a

: m, n Z+

We expand the Green function in a series of these eigenfunctions.

G=
n,m=1

gnm

2j 2n,m a
2 j2 2n,m 4n |J2n (j 2n,m )|

J2n

j 2n,m r a

sin(2n)

We substitute the series into the Green function dierential equation. 1 1 1 Grr + Gr + 2 G = (r )( ) r r r

n,m=1

j 2n,m a

gnm =

2j 2n,m
2 a j 2 2n,m 4n |J2n (j 2n,m )|

J2n

j 2n,m r a J2n

sin(2n)

2j 2n,m a
2 j2 2n,m 4n |J2n (j 2n,m )|

n,m=1

j 2n,m a

sin(2n) j 2n,m r a

2j 2n,m We equate terms and solve for the coecients gmn . gnm = a j 2n,m
2

2 2n,m

J2n
2n (j 2n,m )|

sin(2n)

4n2 |J

2j 2n,m a j
2 2n,m

J2n
2n (j 2n,m )|

4n2 |J

j 2n,m a

sin(2n)

This determines the green function. Solution 45.14 We start with the equation
2

G = (x )(y ). 2014

We do an odd reection across the y axis so that G(0, y; , ) = 0.


2

G = (x )(y ) (x + )(y )

Then we do an even reection across the x axis so that Gy (x, 0; , ) = 0.


2

G = (x )(y ) (x + )(y ) + (x )(y + ) (x + )(y + )

We solve this problem using the innite space Green function. G= 1 1 ln (x )2 + (y )2 ln (x + )2 + (y )2 4 4 1 1 ln (x )2 + (y + )2 ln (x + )2 + (y + )2 + 4 4 G= 1 ln 4 ((x )2 + (y )2 ) ((x )2 + (y + )2 ) ((x + )2 + (y )2 ) ((x + )2 + (y + )2 )

Now we solve the boundary value problem. u(, ) =


S 0

u(x, y)

G u(x, y) G n n
0

dS +
V

Gu dV

u(, ) =

u(0, y)(Gx (0, y; , )) dy +

G(x, 0; , )(uy (x, 0)) dx

u(, ) =
0

g(y)Gx (0, y; , ) dy +
0

G(x, 0; , )h(x) dx (x )2 + 2 h(x) dx (x + )2 + 2 0 (x )2 + y 2 ln h() d (x + )2 + y 2 0 ln

x u(x, y) = u(, ) =

0 0

1 1 + 2 2 + (y )2 + (y + )2 1 1 + 2 2 + (y )2 x x + (y + )2 2015

g(y) dy +

1 2 1 g() d + 2

Solution 45.15 First we nd the innite space Green function. Gtt c2 Gxx = (x )(t ), We solve this problem with the Fourier transform. Gtt + c2 2 G = F[(x )](t ) 1 G = F[(x )]H(t ) sin(c(t )) c G = H(t )F[(x )]F H(c(t ) |x|) c 1 G = H(t ) (y )H(c(t ) |x y|) dy c 2 1 G = H(t )H(c(t ) |x |) 2c 1 G = H(c(t ) |x |) 2c 1. So that the Green function vanishes at x = 0 we do an odd reection about that point. Gtt c2 Gxx = (x )(t ) (x + )(t ) 1 1 G = H(c(t ) |x |) H(c(t ) |x + |) 2c 2c 2. Note that the Green function satises the symmetry relation G(x, t; , ) = G(, ; x, t). This implies that Gxx = G , 2016 Gtt = G . G = Gt = 0 for t <

We write the Green function problem and the inhomogeneous dierential equation for u in terms of and . G c2 G = (x )(t ) u c2 u = Q(, ) (45.4) (45.5)

We take the dierence of u times Equation 45.4 and G times Equation 45.5 and integrate this over the domain (0, ) (0, t+ ).
t+ 0 0 t+ t+ 0 0 t+

(u(x )(t ) GQ) d d =


0 0

uG u G c2 (uG u G) d d (uG u G) c2 (uG u G)


t+

u(x, t) =
0 0 t+ 0

GQ d d +
0

d d

u(x, t) =
0 t+ 0 0

GQ d d + GQ d d
0

[uG

u G]t+ 0

d c

2 0

[uG u G] d 0
t+

u(x, t) =

[uG u G] =0 d + c

2 0

[uG ]=0 d

We consider the case Q(x, t) = f (x) = g(x) = 0.


t+

u(x, t) = c We calculate G . G= G =

2 0

h( )G (x, t; 0, ) d

1 (H(c(t ) |x |) H(c(t ) |x + |)) 2c

1 ((c(t ) |x |)(1) sign(x )(1) (c(t ) |x + |)(1) sign(x + )) 2c 1 G (x, t; 0, ) = (c(t ) |x|) sign(x) c 2017

We are interested in x > 0. 1 G (x, t; 0, ) = (c(t ) x) c Now we can calculate the solution u.
t+

u(x, t) = c u(x, t) =

2 0 t+ 0

1 h( ) (c(t ) x) d c x c d

h( ) (t ) u(x, t) = h t x c

3. The boundary condition inuences the solution u(x1 , t1 ) only at the point t = t1 x1 /c. The contribution from the boundary condition u(0, t) = h(t) is a wave moving to the right with speed c. Solution 45.16

gtt c2 gxx = 0, g(x, 0; , ) = (x ), gt (x, 0; , ) = 0 gtt + c2 2 gxx = 0, g (x, 0; , ) = F[(x )], gt (x, 0; , ) = 0 g = F[(x )] cos(ct) g = F[(x )]F[((x + ct) + (x ct))] 1 ( )((x + ct) + (x ct)) d g= 2 1 g(x, t; ) = ((x + ct) + (x ct)) 2 2018

tt c2 xx = 0, (x, 0; , ) = 0, t (x, 0; , ) = (x ) tt + c2 2 xx = 0, (x, 0; , ) = 0, t (x, 0; , ) = F[(x )] 1 = F[(x )] sin(ct) c = F[(x )]F (H(x + ct) + H(x ct)) c 1 ( ) (H(x + ct) + H(x ct)) d = 2 c (x, t; ) = Solution 45.17

1 (H(x + ct) + H(x ct)) 2c

u(x, t) =
0

G(x, t; , )f (, ) d d +

g(x, t; )p() d +

(x, t; )q() d

u(x, t) =

1 2c

H(t )(H(x + c(t )) H(x c(t )))f (, ) d d + 1 2


((x + ct) + (x ct))p() d +

1 2c

(H(x + ct) + H(x ct))q() d

u(x, t) =

1 2c

t 0

(H(x + c(t )) H(x c(t )))f (, ) d d

1 1 + (p(x + ct) + p(x ct)) + 2 2c u(x, t) = 1 2c


t 0 x+c(t ) xc(t )

x+ct

q() d
xct

1 1 f (, ) d d + (p(x + ct) + p(x ct)) + 2 2c 2019

x+ct

q() d
xct

This solution demonstrates the domain of dependence of the solution. The rst term is an integral over the triangle domain {(, ) : 0 < < t, x c < < x + c }. The second term involves only the points (x ct, 0). The third term is an integral on the line segment {(, 0) : x ct < < x + ct}. In totallity, this is just the triangle domain. This is shown graphically in Figure 45.4.

x,t

Domain of Dependence x-ct x+ct

Figure 45.4: Domain of dependence for the wave equation. Solution 45.18 Single Sum Representation. First we nd the eigenfunctions of the homogeneous problem u k 2 u = 0. We substitute the separation of variables, u(x, y) = X(x)Y (y) into the partial dierential equation. X Y + XY k 2 XY = 0 X Y = k2 = 2 X Y We have the regular Sturm-Liouville eigenvalue problem, X = 2 X, X(0) = X(a) = 0, 2020

n nx , , Xn = sin a a We expand the solution u in a series of these eigenfunctions. n =

which has the solutions,

n N.

G(x, y; , ) =
n=1

cn (y) sin

nx a

We substitute this series into the partial dierential equation to nd equations for the cn (y).

n=1

n a

cn (y) + cn (y) k 2 cn (y) sin

nx = (x )(y ) a

The series expansion of the right side is,

(x )(y ) =
n=1

dn (y) sin

nx a nx a dx

dn (y) =

2 a

(x )(y ) sin
0

dn (y) = The the equations for the cn (y) are cn (y) k 2 + n a


2

2 sin a

n a

(y ).

cn (y) =

2 sin a

n a

(y ),

cn (0) = cn (b) = 0.

The homogeneous solutions are {cosh(n y), sinh(n y)}, where n = k 2 (n/a)2 . The solutions that satisfy the boundary conditions at y = 0 and y = b are, sinh(n y) and sinh(n (y b)), respectively. The Wronskian of these 2021

solutions is, W (y) = sinh(n y) sinh(n (y b)) n cosh(n y) n cosh(n (y b))

= n (sinh(n y) cosh(n (y b)) sinh(n (y b)) cosh(n y)) = n sinh(n b). The solution for cn (y) is n sinh(n y< ) sinh(n (y> b)) 2 sin . a a n sinh(n b) The Green function for the partial dierential equation is cn (y) = 2 G(x, y; , ) = a

n=1

sinh(n y< ) sinh(n (y> b)) nx sin sin n sinh(n b) a

n a

Solution 45.19 We take the Fourier cosine transform in x of the partial dierential equation and the boundary condition along y = 0. Gxx + Gyy k 2 G = (x )(y ) 1 1 2 G(, y) Gx (0, y) + Gyy (, y) k 2 G(, y) = cos()(y ) 1 Gyy (, y) (k 2 + 2 )G(, y) == cos()(y ), G(, 0) = 0 Then we take the Fourier sine transform in y. 1 2 G(, ) + G(, 0) (k 2 + 2 )G(, ) = 2 cos() sin() cos() sin() G= 2 2 (k + 2 + 2 ) 2022

We take two inverse transforms to nd the solution. For one integral representation of the Green function we take the inverse sine transform followed by the inverse cosine transform. sin() 1 G = cos() 2 + 2 + 2 ) (k 1 2 2 e k + y G = cos()Fs [(y )]Fc 2 + 2 k 1 1 G(, y) = cos() (z ) exp k 2 + 2 |y z| exp k 2 + 2 (y + z) 2 0 k 2 + 2 cos() exp k 2 + 2 |y | exp k 2 + 2 (y + ) G(, y) = 2 k 2 + 2 1 cos() G(x, y; , ) = exp k 2 + 2 |y | exp k 2 + 2 (y + ) d 0 k 2 + 2

dz

For another integral representation of the Green function, we take the inverse cosine transform followed by the inverse sine transform. cos() 1 G(, ) = sin() 2 + 2 + 2 ) (k 1 2 2 e k + x G(, ) = sin()Fc [(x )]Fc k2 + 2 1 1 2 2 2 2 e k + |xz| + e k + (x+z) dz G(x, ) = sin() (z ) 2 0 k2 + 2 1 1 2 2 2 2 e k + |x| + e k + (x+) G(x, ) = sin() 2 k 2 + 2 1 sin(y) sin() k2 + 2 |x| k2 + 2 (x+) e G(x, y; , ) = +e d 0 k2 + 2

2023

Solution 45.20 The problem is: 1 1 (r )( ) Grr + Gr + 2 G = , r r r G(r, 0, , ) = G(r, , , ) = 0, G(0, , , ) = 0 G(r, , , ) 0 as r . 0 < r < , 0 < < ,

Let w = r ei and z = x + iy. We use the conformal mapping, z = w/ to map the sector to the upper half z plane. The problem is (x, y) space is Gxx + Gyy = (x )(y ), G(x, 0, , ) = 0, G(x, y, , ) 0 as x, y . < x < , 0 < y < ,

We will solve this problem with the method of images. Note that the solution of, Gxx + Gyy = (x )(y ) (x )(y + ), < x < , G(x, y, , ) 0 as x, y , < y < ,

satises the condition, G(x, 0, , ) = 0. Since the innite space Green function for the Laplacian in two dimensions is 1 ln (x )2 + (y )2 , 4 the solution of this problem is, G(x, y, , ) = 1 1 ln (x )2 + (y )2 ln (x )2 + (y + )2 4 4 1 (x )2 + (y )2 ln . = 4 (x )2 + (y + )2 2024

Now we solve for x and y in the conformal mapping. z = w/ = (r ei )/ x + iy = r/ (cos(/) + i sin(/)) x = r/ cos(/), y = r/ sin(/)

We substitute these expressions into G(x, y, , ) to obtain G(r, , , ). G(r, , , ) = 1 4 1 = 4 1 = 4 1 = 4 ln ln ln ln (r/ cos(/) / cos(/))2 + (r/ sin(/) / sin(/))2 (r/ cos(/) / cos(/))2 + (r/ sin(/) + / sin(/))2 r2/ + 2/ 2r/ / cos(( )/) r2/ + 2/ 2r/ / cos(( + )/) (r/)/ /2 + (/r)/ /2 cos(( )/) (r/)/ /2 + (/r)/ /2 cos(( + )/) e ln(r/)/ /2 + e ln(/r)/ /2 cos(( )/) e ln(r/)/ /2 + e ln(/r)/ /2 cos(( + )/) G(r, , , ) = Now recall that the solution of u = f (x), subject to the boundary condition, u(x) = g(x), is u(x) = f ()G(x; ) dA + 2025 g()
G(x; )

1 ln 4 cosh

cosh

/ r ln / r ln

cos(( )/) cos(( + )/)

n ds .

The normal directions along the lower and upper edges of the sector are and , respectively. The gradient in polar coordinates is + .

We only need to compute the component of the gradient of G. This is

sin(( )/) 1 sin(( )/) G= + r r 4 cosh ln cos(( )/) 4 cosh ln cos(( + )/)

Along = 0, this is 1 sin(/) G (r, , , 0) = . r 2 cosh ln cos(/)

Along = , this is 1 sin(/) G (r, , , ) = . r 2 cosh ln + cos(/) 2026

The solution of our problem is


c

u(r, ) =

2 cosh

sin(/)
r ln + cos(/)

d +
c

2 cosh

sin(/)
r ln cos(/)

u(r, ) =
c

sin(/) 2 cosh
r ln cos(/) c

+ 2 cosh 1 cosh
2 r

sin(/)
r ln + cos(/)

1 u(r, ) = sin

cos

ln
x

cos2

1 u(r, ) = sin 2 u(r, ) = sin Solution 45.21 First consider the Green function for

cos cos

ln(c/r) ln(c/r)

cosh

1 cos2 1 cos

dx dx

cosh

2x

ut uxx = 0, The dierential equation and initial condition is Gt = Gxx ,

u(x, 0) = f (x).

G(x, 0; ) = (x ).

The Green function is a solution of the homogeneous heat equation for the initial condition of a unit amount of heat concentrated at the point x = . You can verify that the Green function is a solution of the heat equation for t > 0 and that it has the property:

G(x, t; ) dx = 1,

for t > 0.

This property demonstrates that the total amount of heat is the constant 1. At time t = 0 the heat is concentrated at the point x = . As time increases, the heat diuses out from this point. 2027

The solution for u(x, t) is the linear combination of the Green functions that satises the initial condition u(x, 0) = f (x). This linear combination is

u(x, t) =

G(x, t; )f () d.

G(x, t; 1) and G(x, t; 1) are plotted in Figure 45.5 for the domain t [1/100..1/4], x [2..2] and = 1.

2 1 0 0 0.1 0.2 -2 -1 1

Figure 45.5: G(x, t; 1) and G(x, t; 1) Now we consider the problem ut = uxx , u(x, 0) = f (x) for x > 0, 2028 u(0, t) = 0.

Note that the solution of Gt = Gxx , x > 0, t > 0, G(x, 0; ) = (x ) (x + ), satises the boundary condition G(0, t; ) = 0. We write the solution as the dierence of innite space Green functions. G(x, t; ) = 1 1 2 2 e(x) /(4t) e(x+) /(4t) 4t 4t 1 2 /(4t) 2 e(x) = e(x+) /(4t) 4t 1 2 2 e(x + )/(4t) sinh 4t x 2t

G(x, t; ) = Next we consider the problem ut = uxx , Note that the solution of

u(x, 0) = f (x) for x > 0,

ux (0, t) = 0.

Gt = Gxx , x > 0, t > 0, G(x, 0; ) = (x ) + (x + ), satises the boundary condition Gx (0, t; ) = 0. We write the solution as the sum of innite space Green functions. G(x, t; ) = 1 1 2 2 e(x) /(4t) + e(x+) /(4t) 4t 4t 1 2 2 e(x + )/(4t) cosh 4t x 2t

G(x, t; ) =

The Green functions for the two boundary conditions are shown in Figure 45.6. 2029

2 1 1 0.8 0 0.6 0.4 0.05 0.1 0.2 0.15 0.2 0 0.25

2 1 1 0.8 0 0.6 0.4 0.05 0.1 0.2 0.15 0.2 0 0.25

Figure 45.6: Green functions for the boundary conditions u(0, t) = 0 and ux (0, t) = 0. Solution 45.22 a) The Green function problem is Gtt c2 Gxx = (t )(x ), 0 < x < L, G(0, t; , ) = Gx (L, t; , ) = 0, G(x, t; , ) = 0 for t < . t > 0,

The condition that G is zero for t < makes this a causal Green function. We solve this problem by expanding G in a series of eigenfunctions of the x variable. The coecients in the expansion will be functions of t. First we nd the eigenfunctions of x in the homogeneous problem. We substitute the separation of variables u = X(x)T (t) into the 2030

homogeneous partial dierential equation. XT = c2 X T T X = = 2 2T c X The eigenvalue problem is X = 2 X, which has the solutions, (2n 1) , Xn = sin 2L The series expansion of the Green function has the form, n =

X(0) = X (L) = 0, (2n 1)x 2L , n N.

G(x, t; , ) =
n=1

gn (t) sin

(2n 1)x 2L

We determine the coecients by substituting the expansion into the Green function dierential equation. Gtt c2 Gxx = (x )(t )
2

gn (t) +
n=1

(2n 1)c 2L

gn (t) sin

(2n 1)x 2L

= (x )(t )

We need to expand the right side of the equation in the sine series

(x )(t ) =
n=1

dn (t) sin

(2n 1)x 2L (2n 1)x 2L (t ) dx

dn (t) =

2 L

(x )(t ) sin
0

dn (t) =

2 sin L

(2n 1) 2L 2031

By equating coecients in the sine series, we obtain ordinary dierential equation Green function problems for the gn s. gn (t; ) + (2n 1)c 2L
2

gn (t; ) =

2 sin L

(2n 1) 2L

(t )

From the causality condition for G, we have the causality conditions for the gn s, gn (t; ) = gn (t; ) = 0 for t < . The continuity and jump conditions for the gn are gn ( + ; ) = 0, gn ( + ; ) = 2 sin L (2n 1) 2L .

A set of homogeneous solutions of the ordinary dierential equation are cos (2n 1)ct 2L , sin (2n 1)ct 2L

Since the continuity and jump conditions are given at the point t = , a handy set of solutions to use for this problem is the fundamental set of solutions at that point: cos (2n 1)c(t ) 2L , 2L sin (2n 1)c (2n 1)c(t ) 2L

The solution that satises the causality condition and the continuity and jump conditions is, gn (t; ) = 4 sin (2n 1)c

(2n 1) 2L

sin

(2n 1)c(t ) 2L

H(t ).

Substituting this into the sum yields, 4 1 sin G(x, t; , ) = H(t ) c 2n 1 n=1 (2n 1) 2L sin (2n 1)c(t ) 2L sin (2n 1)x 2L .

We use trigonometric identities to write this in terms of traveling waves. 2032

1 1 G(x, t; , ) = H(t ) c 2n 1 n=1 + sin (2n 1)((x ) + c(t )) 2L

sin sin

(2n 1)((x ) c(t )) 2L (2n 1)((x + ) c(t )) 2L

sin

(2n 1)((x + ) + c(t )) 2L

b) Now we consider the Green function with the boundary conditions, ux (0, t) = ux (L, t) = 0. First we nd the eigenfunctions in x of the homogeneous problem. The eigenvalue problem is X = 2 X, which has the solutions, X0 = 1, nx Xn = cos , n = 1, 2, . . . . L 0 = 0, X (0) = X (L) = 0,

n =

n , L

The series expansion of the Green function for t > has the form, 1 nx G(x, t; , ) = g0 (t) + . gn (t) cos 2 L n=1 (Note the factor of 1/2 in front of g0 (t). With this, the integral formulas for all the coecients are the same.) We 2033

determine the coecients by substituting the expansion into the partial dierential equation. Gtt c2 Gxx = (x )(t ) 1 nc g0 (t) + gn (t) + 2 L n=1
2

gn (t) cos

nx = (x )(t ) L

We expand the right side of the equation in the cosine series. 1 nx (x )(t ) = d0 (t) + dn (t) cos 2 L n=1 dn (t) = 2 L
L

(x )(t ) cos
0

nx L

dx

dn (t) =

2 cos L

n L

(t )

By equating coecients in the cosine series, we obtain ordinary dierential equations for the gn . gn (t; ) + nc L
2

gn (t; ) =

2 cos L

n L

(t ),

n = 0, 1, 2, . . .

From the causality condition for G, we have the causality condiions for the gn , gn (t; ) = gn (t; ) = 0 for t < . The continuity and jump conditions for the gn are gn ( + ; ) = 0, gn ( + ; ) = 2 cos L n L .

The homogeneous solutions of the ordinary dierential equation for n = 0 and n > 0 are respectively, {1, t}, cos nct L 2034 , sin nct L .

Since the continuity and jump conditions are given at the point t = , a handy set of solutions to use for this problem is the fundamental set of solutions at that point: {1, t }, cos nc(t ) L , L sin nc nc(t ) L .

The solutions that satisfy the causality condition and the continuity and jump conditions are, g0 (t) = gn (t) = Substituting this into the sum yields, G(x, t; , ) = H(t ) t 2 + L c

2 cos nc

2 (t )H(t ), L n nc(t ) sin L L

H(t ).

n=1

1 cos n

n L

sin

nc(t ) L

cos

nx L

We can write this as the sum of traveling waves.

G(x, t; , ) = + sin

t 1 1 H(t ) + H(t ) L 2c n n=1 n((x ) + c(t )) 2L sin

sin

n((x ) c(t )) 2L

n((x + ) c(t )) 2L + sin n((x + ) + c(t )) 2L

2035

Solution 45.23 First we derive Greens identity for this problem. We consider the integral of uL[v] L[u]v on the domain 0 < x < 1, 0 < t < T.
T 0 T 0 T 0 0 1 0 1 0 1

(uL[v] L[u]v) dx dt u(vtt c2 vxx (utt c2 uxx )v dx dt , x t c2 (uvx ux v), uvt ut v dx dt

Now we can use the divergence theorem to write this as an integral along the boundary of the domain. c2 (uvx ux v), uvt ut v n ds

The domain and the outward normal vectors are shown in Figure 45.7. Writing out the boundary integrals, Greens identity for this problem is,
T 0 0 0 1 1

u(vtt c2 vxx ) (utt c2 uxx )v dx dt =


0

(uvt ut v)t=0 dx
T 1

+
1

(uvt ut v)t=T dx c2
0

(uvx ux v)x=1 dt + c2
T

(uvx ux v)x=0 dt

The Green function problem is Gtt c2 Gxx = (x )(t ), 0 < x, < 1, t, > 0, Gx (0, t; , ) = Gx (1, t; , ) = 0, t > 0, G(x, t; , ) = 0 for t < . If we consider G as a function of (, ) with (x, t) as parameters, then it satises: G c2 G = (x )(t ), G (x, t; 0, ) = G (x, t; 1, ) = 0, > 0, G(x, t; , ) = 0 for > t. 2036

n=(0,1) t=T n=(-1,0) t=0 x=1 n=(1,0)

x=0 n=(0,-1)

Figure 45.7: Outward normal vectors of the domain. Now we apply Greens identity for u = u(, ), (the solution of the wave equation), and v = G(x, t; , ), (the Green function), and integrate in the (, ) variables. The left side of Greens identity becomes:
T 0 1

u(G c2 G ) (u c2 u )G d d
0 T 0 0 1

(u((x )(t )) (0)G) d d u(x, t). Since the normal derivative of u and G vanish on the sides of the domain, the integrals along = 0 and = 1 in Greens identity vanish. If we take T > t, then G is zero for = T and the integral along = T vanishes. The one remaining integral is
1

(u(, 0)G (x, t; , 0) u (, 0)G(x, t; , 0) d. 2037

Thus Greens identity allows us to write the solution of the inhomogeneous problem.
1

u(x, t) =
0

(u (, 0)G(x, t; , 0) u(, 0)G (x, t; , 0)) d.

With the specied initial conditions this becomes


1

u(x, t) =
0

(G(x, t; , 0) 2 (1 )2 G (x, t; , 0)) d.

Now we substitute in the Green function that we found in the previous exercise. The Green function and its derivative are,

G(x, t; , 0) = t +
n=1

2 cos(n) sin(nct) cos(nx), nc

G (x, t; , 0) = 1 2
n=1

cos(n) cos(nct) cos(nx).

The integral of the rst term is,


1

t+
0 n=1

2 cos(n) sin(nct) cos(nx) nc

d = t.

The integral of the second term is


1

2 (1 )2
0

1+2
n=1

cos(n) cos(nct) cos(nx)

d =

1 1 3 cos(2nx) cos(2nct). 30 n4 4 n=1

Thus the solution is 1 1 u(x, t) = +t3 cos(2nx) cos(2nct). 44 30 n n=1 2038

For c = 1, the solution at x = 3/4, t = 7/2 is, 1 7 1 cos(3n/2) cos(7n). + 3 u(3/4, 7/2) = 44 30 2 n n=1 Note that the summand is nonzero only for even terms. 53 3 u(3/4, 7/2) = 15 16 4 = 53 3 15 16 4

n=1

1 cos(3n) cos(14n) n4 (1)n n4

n=1

53 3 7 4 = 15 16 4 720 u(3/4, 7/2) = 12727 3840

2039

Chapter 46 Conformal Mapping

2040

46.1

Exercises

Exercise 46.1 Use an appropriate conformal map to nd a non-trivial solution to Laplaces equation uxx + uyy = 0, on the wedge bounded by the x-axis and the line y = x with boundary conditions: 1. u = 0 on both sides. 2. du = 0 on both sides (where n is the inward normal to the boundary). dn

Exercise 46.2 Consider uxx + uyy = (x )(y ), on the quarter plane x, y > 0 with u(x, 0) = u(0, y) = 0 (and , > 0). 1. Use image sources to nd u(x, y; , ). 2. Compare this to the solution which would be obtained using conformal maps and the Green function for the upper half plane. 3. Finally use this idea and conformal mapping to discover how image sources are arrayed when the domain is now the wedge bounded by the x-axis and the line y = x (with u = 0 on both sides). Exercise 46.3 = + is an analytic function of z, = (z). We assume that (z) is nonzero on the domain of interest. u(x, y) is an arbitrary smooth function of x and y. When expressed in terms of and , u(x, y) = (, ). In Exercise 8.15 we showed that 2 2 2 d 2u 2u + 2 = + . 2 dz x2 y 2 2041

1. Show that if u satises Laplaces equation in the z-plane, uxx + uyy = 0, then satises Laplaces equation in the -plane, + = 0, 2. Show that if u satises Helmholtzs equation in the z-plane, uxx + uyy = u, then in the -plane satises + dz = d
2

3. Show that if u satises Poissons equation in the z-plane, uxx + uyy = f (x, y), then satises Poissons equation in the -plane, + where (, ) = f (x, y). 4. Show that if in the z-plane, u satises the Green function problem, uxx + uyy = (x x0 )(y y0 ), then in the -plane, satises the Green function problem, + = ( 0 )( 0 ). 2042 dz = d
2

(, ),

Exercise 46.4 A semi-circular rod of innite extent is maintained at temperature T = 0 on the at side and at T = 1 on the curved surface: x2 + y 2 = 1, y > 0. Use the conformal mapping 1+z , z = x + y, 1z to formulate the problem in terms of and . Solve the problem in terms of these variables. This problem is solved with an eigenfunction expansion in Exercise ??. Verify that the two solutions agree. w = + = Exercise 46.5 Consider Laplaces equation on the domain < x < , 0 < y < , subject to the mixed boundary conditions, u = 1 on y = 0, x > 0, u = 0 on y = , x > 0, uy = 0 on y = 0, y = , x < 0. Because of the mixed boundary conditions, (u and uy are given on separate parts of the same boundary), this problem cannot be solved with separation of variables. Verify that the conformal map, = cosh1 (ez ), with z = x + y, = + maps the innite interval into the semi-innite interval, > 0, 0 < < . Solve Laplaces equation with the appropriate boundary conditions in the plane by inspection. Write the solution u in terms of x and y.

2043

46.2
Hint 46.1 Hint 46.2 Hint 46.3

Hints

Hint 46.4 Show that w = (1 + z)/(1 z) maps the semi-disc, 0 < r < 1, 0 < < to the rst quadrant of the w plane. Solve the problem for v(, ) by taking Fourier sine transforms in and . To show that the solution for v(, ) is equivalent to the series expression for u(r, ), rst nd an analytic function g(w) of which v(, ) is the imaginary part. Change variables to z to obtain the analytic function f (z) = g(w). Expand f (z) in a Taylor series and take the imaginary part to show the equivalence of the solutions. Hint 46.5 To see how the boundary is mapped, consider the map, z = log(cosh ). The problem in the plane is, v + v = 0, > 0, 0 < < , v (0, ) = 0, v(, 0) = 1, v(, ) = 0. To solve this, nd a plane that satises the boundary conditions.

2044

46.3

Solutions

Solution 46.1 We map the wedge to the upper half plane with the conformal transformation = z 4 . 1. We map the wedge to the upper half plane with the conformal transformation = z 4 . The new problem is u + u = 0, u(, 0) = 0.

This has the solution u = . We transform this problem back to the wedge. u(x, y) = u(x, y) = z4

x4 + 4x3 y 6x2 y 2 4xy 3 + y 4 u(x, y) = 4x3 y 4xy 3 u(x, y) = 4xy x2 y 2

2. We dont need to use conformal mapping to solve the problem with Neumman boundary conditions. u = c is a solution to du uxx + uyy = 0, =0 dn on any domain. Solution 46.2 1. We add image sources to satisfy the boundary conditions. uxx + uyy = (x )(y ) (x + )(y ) (x )(y + ) + (x + )(y + ) 1 ln 2

u=

(x )2 + (y )2 ln

(x + )2 + (y )2 ln (x )2 + (y + )2 + ln (x + )2 + (y + )2

2045

u=

1 ln 4

((x )2 + (y )2 ) ((x + )2 + (y + )2 ) ((x + )2 + (y )2 ) ((x )2 + (y + )2 )

2. The Green function for the upper half plane is G= We use the conformal map, c = z 2 , c = a + b. a = x2 y 2 , b = 2xy We compute the Jacobian of the mapping. J= ax ay 2x 2y = = 4 x2 + y 2 bx by 2y 2x 1 ln 4 ((x )2 + (y )2 ) ((x )2 + (y + )2 )

We transform the problem to the upper half plane, solve the problem there, and then transform back to the rst quadrant. uxx + uyy = (x )(y ) dc (uaa + ubb ) dz
2

= 4 x2 + y 2 (a )(b )

(uaa + ubb ) |2z|2 = 4 x2 + y 2 (a )(b ) uaa + ubb = (a )(b ) 1 ((a )2 + (b )2 ) u= ln 4 ((a )2 + (b + )2 ) ((x2 y 2 2 + 2 )2 + (2xy 2)2 ) 1 u= ln 4 ((x2 y 2 2 + 2 )2 + (2xy + 2)2 ) u= 1 ln 4 ((x )2 + (y )2 ) ((x + )2 + (y + )2 ) ((x + )2 + (y )2 ) ((x )2 + (y + )2 )

2046

We obtain the some solution as before. 3. First consider u = (x )(y ), u(x, 0) = u(x, x) = 0. Enforcing the boundary conditions will require 7 image sources obtained from 4 odd reections. Refer to Figure 46.1 to see the reections pictorially. First we do a negative reection across the line y = x, which adds a negative image source at the point (, ) This enforces the boundary condition along y = x. u = (x )(y ) (x )(y ), u(x, 0) = u(x, x) = 0

Now we take the negative image of the reection of these two sources across the line y = 0 to enforce the boundary condition there. u = (x )(y ) (x )(y ) (x )(y + ) + (x )(y + ) The point sources are no longer odd symmetric about y = x. We add two more image sources to enforce that boundary condition. u = (x )(y ) (x )(y ) (x )(y + ) + (x )(y + ) + (x + )(y ) (x + )(y ) Now sources are no longer odd symmetric about y = 0. Finally we add two more image sources to enforce that boundary condition. Now the sources are odd symmetric about both y = x and y = 0. u = (x )(y ) (x )(y ) (x )(y + ) + (x )(y + ) + (x + )(y ) (x + )(y ) + (x + )(y + ) (x + )(y + )

Solution 46.3 2 2 d + 2 = 2 dz
2

2u 2u + x2 y 2

2047

Figure 46.1: Odd reections to enforce the boundary conditions.

1.

uxx + uyy = 0 d dz
2

( + ) = 0 + = 0 2048

2. uxx + uyy = u d dz
2

( + ) = dz = d
2

+ 3.

uxx + uyy = f (x, y) d dz


2

( + ) = (, ) dz d
2

+ = 4. The Jacobian of the mapping is J=

(, )

x y 2 = x y x y = x2 + y . x y

Thus the Dirac delta function on the right side gets mapped to x2 1 ( 0 )( 0 ). 2 + y

Next we show that |dz/d|2 has the same value as the Jacobian. dz d
2 2 = (x + y )(x y ) = x2 + y

2049

Now we transform the Green function problem. uxx + uyy = (x x0 )(y y0 ) d dz


2

( + ) =

x2

1 ( 0 )( 0 ) 2 + y

+ = ( 0 )( 0 ) Solution 46.4 The mapping, w= 1+z , 1z

maps the unit semi-disc to the rst quadrant of the complex plane. We write the mapping in terms of r and . + = 1 + r e 1 r2 + 2r sin = 1 r e 1 + r2 2r cos 1 r2 1 + r2 2r cos 2r sin = 1 + r2 2r cos =
2r Consider a semi-circle of radius r. The image of this under the conformal mapping is a semi-circle of radius 1r2 and 2 center 1+r2 in the rst quadrant of the w plane. This semi-circle intersects the axis at 1r and 1+r . As r ranges 1r 1+r 1r from zero to one, these semi-circles cover the rst quadrant of the w plane. (See Figure 46.2.) We also note how the boundary of the semi-disc is mapped to the boundary of the rst quadrant of the w plane. The line segment = 0 is mapped to the real axis > 1. The line segment = is mapped to the real axis 0 < < 1. Finally, the semi-circle r = 1 is mapped to the positive imaginary axis.

2050

5 4 3 2 1

-1

Figure 46.2: The conformal map, w = The problem for v(, ) is, v + v = 0, > 0, > 0, v(, 0) = 0, v(0, ) = 1.

1+z . 1z

We will solve this problem with the Fourier sine transform. We take the Fourier sine transform of the partial dierential equation, rst in and then in . 2 v (, ) + v(0, ) + v (, ) = 0, v (, 0) = 0 2 v (, ) + + v (, ) = 0, v (, 0) = 0 2 v (, ) + 2 2 v (, ) + v (, 0) = 0 v (, ) = 2 (2 + 2 ) Now we utilize the Fourier sine transform pair, Fs ecx = 2 / , + c2

2051

to take the inverse sine transform in . v (, ) = With the Fourier sine transform pair,

1 e

1 x = ec , c we take the inverse sine transform in to obtain the solution. Fs 2 arctan v(, ) = 2 arctan

Since v is harmonic, it is the imaginary part of an analytic function g(w). By inspection, we see that this function is g(w) = We change variables to z, f (z) = g(w). f (z) = We expand f (z) in a Taylor series about z = 0, 4 f (z) = and write the result in terms of r and , z = r e . 4 f (z) =

2 log(w). 1+z 1z

2 log

n=1 oddn

zn , n

n=1 oddn

rn e n

2052

u(r, ) is the imaginary part of f (z). 4 u(r, ) =

n=1 oddn

1 n r sin(n) n

This demonstrates that the solutions obtained with conformal mapping and with an eigenfunction expansion in Exercise ?? agree. Solution 46.5 Instead of working with the conformal map from the z plane to the plane, = cosh1 (ez ), it will be more convenient to work with the inverse map, z = log(cosh ), which maps the semi-innite strip to the innite one. We determine how the boundary of the domain is mapped so that we know the appropriate boundary conditions for the semi-innite strip domain. A { : > 0, = 0} B { : > 0, = } C { : = 0, 0 < < /2} D { : = 0, /2 < < } {log(cosh()) : > 0} = {z : x > 0, {log( cosh()) : > 0} = {z : x > 0, {log(cos()) : 0 < < /2} = {z : x < 0, {log(cos()) : /2 < < } = {z : x < 0, y = 0} y = } y = 0} y = }

From the mapping of the boundary, we see that the solution v(, ) = u(x, y), is 1 on the bottom of the semi-innite strip, 0 on the top. The normal derivative of v vanishes on the vertical boundary. See Figure 46.3. In the plane, the problem is, v + v = 0, > 0, 0 < < , v (0, ) = 0, v(, 0) = 1, v(, ) = 0. 2053

y B D C A v=0 v=0 v=1 uy=0 u=1 x uy=0 u=0 C y A z=log(cosh( )) x D B

Figure 46.3: The mapping of the boundary conditions. By inspection, we see that the solution of this problem is, v(, ) = 1 The solution in the z plane is u(x, y) = 1 1 cosh1 (ez ) , .

where z = x + y. We will nd the imaginary part of cosh1 (ez ) in order to write this explicitly in terms of x and y. 2054

Recall that we can write the cosh1 in terms of the logarithm. cosh1 (w) = log w + w2 1 cosh1 (ez ) = log ez + e2z 1 = log ez 1 + 1 e2z = z + log 1 + 1 e2z Now we need to nd the imaginary part. Well work from the inside out. First recall, x + y = x2 + y 2 exp tan1 y x =
4

x2 + y 2 exp

y tan1 2 x

so that we can write the innermost factor as, 1 e2z = 1 e2x cos(2y) + e2x sin(2y) = =
4

(1 e2x cos(2y))2 + (e2x sin(2y))2 exp 1 2 e2x cos(2y) + e4x exp tan1 2

e2x sin(2y) tan1 2 1 e2x cos(2y) sin(2y) e2x cos(2y) tan1 2 sin(2y) cos(2y)

We substitute this into the logarithm. log 1 + 1 e2z = log 1 + Now we can write .

1 2 e2x cos(2y) + e4x exp

e2x

= = y + tan1
4

z + log 1 +

1 e2z
1 2

1 2 e2x cos(2y) + e4x sin


4

tan1
1 2

e2x

sin(2y) cos(2y)

1+

2 e2x

cos(2y) +

e4x

cos

tan

sin(2y) e2x cos(2y)

2055

Finally we have the solution, u(x, y). u(x, y) = 1 y 1 tan1 1+


4

1 2 e2x cos(2y) + e4x sin


4

1 2

tan1
1 2

sin(2y) e2x cos(2y) sin(2y) e2x cos(2y)

1 2 e2x cos(2y) + e4x cos

tan1

2056

Chapter 47 Non-Cartesian Coordinates


47.1 Spherical Coordinates

Writing rectangular coordinates in terms of spherical coordinates,

x = r cos sin y = r sin sin z = r cos . 2057

The Jacobian is cos sin r sin sin r cos cos sin sin r cos sin r sin cos cos 0 r sin cos sin sin cos cos = r sin sin sin cos sin cos cos 0 sin
2

= r2 sin ( cos2 sin2 sin2 cos2 cos2 cos2 sin2 sin2 ) = r2 sin (sin2 + cos2 ) = r2 sin . Thus we have that f (x, y, z) dx dy dz =
V V

f (r, , )r2 sin dr d d.

47.2

Laplaces Equation in a Disk


1 2u = 0, r2 2

Consider Laplaces equation in polar coordinates 1 r r subject to the the boundary conditions 1. u(1, ) = f () 2. ur (1, ) = g(). 2058 r u r + 0r1

We separate variables with u(r, ) = R(r)T (). 1 1 (R T + rR T ) + 2 RT = 0 r r R R T r2 +r = = R R T Thus we have the two ordinary dierential equations T + T = 0, T (0) = T (2), T (0) = T (2) 2 r R + rR R = 0, R(0) < . The eigenvalues and eigenfunctions for the equation in T are 0 = 0, n = n 2 ,
(1) Tn

1 2 (2) = cos(n), Tn = sin(n) T0 =

(I chose T0 = 1/2 so that all the eigenfunctions have the same norm.) For = 0 the general solution for R is R = c1 + c2 log r. Requiring that the solution be bounded gives us R0 = 1. For = n2 > 0 the general solution for R is R = c1 rn + c2 rn . Requiring that the solution be bounded gives us Rn = r n . 2059

Thus the general solution for u is a0 u(r, ) = + rn [an cos(n) + bn sin(n)] . 2 n=1 For the boundary condition u(1, ) = f () we have the equation a0 f () = + [an cos(n) + bn sin(n)] . 2 n=1 If f () has a Fourier series then the coecients are 1 1 an = 1 bn = a0 =
2

f () d
0 2

f () cos(n) d
0 2

f () sin(n) d.
0

For the boundary condition ur (1, ) = g() we have the equation

g() =
n=1

n [an cos(n) + bn sin(n)] .

g() has a series of this form only if


2

g() d = 0.
0

2060

The coecients are 1 an = n 1 bn = n


2

g() cos(n) d
0 2

g() sin(n) d.
0

47.3

Laplaces Equation in an Annulus

Consider the problem


2

u=

1 r r

u r

1 2u = 0, r2 2

0 r < a,

< ,

with the boundary condition u(a, ) = 2 . So far this problem only has one boundary condition. By requiring that the solution be nite, we get the boundary condition |u(0, )| < . By specifying that the solution be C 1 , (continuous and continuous rst derivative) we obtain u(r, ) = u(r, ) and u u (r, ) = (r, ).

We will use the method of separation of variables. We seek solutions of the form u(r, ) = R(r)(). 2061

Substituting into the partial dierential equation, 2 u 1 u 1 2u + + 2 2 =0 r2 r r r 1 1 R + R = 2 R r r rR r2 R + = = R R Now we have the boundary value problem for , () + () = 0, subject to () = () and () = () < ,

We consider the following three cases for the eigenvalue, , < 0. No linear combination of the solutions, = exp( ), exp( ), can satisfy the boundary conditions. Thus there are no negative eigenvalues. = 0. The general solution solution is = a + b. By applying the boundary conditions, we get = a. Thus we have the eigenvalue and eigenfunction, 0 = 0, A0 = 1. > 0. The general solution is = a cos( )+b sin( ). Applying the boundary conditions yields the eigenvalues n = n 2 , with the associated eigenfunctions An = cos(n) and Bn = sin(n). 2062 n = 1, 2, 3, . . .

The equation for R is r2 R + rR n R = 0. In the case 0 = 0, this becomes 1 R = R r a R = r R = a log r + b Requiring that the solution be bounded at r = 0 yields (to within a constant multiple) R0 = 1. For n = n2 , n 1, we have r2 R + rR n2 R = 0 Recognizing that this is an Euler equation and making the substitution R = r , ( 1) + n2 = 0 = n R = arn + brn . requiring that the solution be bounded at r = 0 we obtain (to within a constant multiple) Rn = r n The general solution to the partial dierential equation is a linear combination of the eigenfunctions

u(r, ) = c0 +
n=1

[cn rn cos n + dn rn sin n] .

2063

We determine the coecients of the expansion with the boundary condition

u(a, ) = = c0 +
n=1

[cn an cos n + dn an sin n] .

We note that the eigenfunctions 1, cos n, and sin n are orthogonal on . Integrating the boundary condition from to yields

2 d =

c0 d . 3
2

c0 =

Multiplying the boundary condition by cos m and integrating gives


2 cos m d = cm am

cos2 m d

(1)m 8 cm = . m 2 am We multiply by sin m and integrate to get


2 sin m d = dm am

sin2 m d

dm = 0 Thus the solution is u(r, ) = 2 (1)n 8 n + r cos n. 3 n 2 an n=1

2064

Part VI Calculus of Variations

2065

Chapter 48 Calculus of Variations

2066

48.1

Exercises

Exercise 48.1 1 Discuss the problem of minimizing 0 ((y )4 6(y )2 ) dx, y(0) = 0, y() = . Consider both C 1 [0, ] and Cp [0, ], and comment (with reasons) on whether your answers are weak or strong minima. Exercise 48.2 Consider 1. 2.
x1 (a(y )2 x0 x1 (y )3 x0

+ byy + cy 2 ) dx, y(x0 ) = y0 , y(x1 ) = y1 , a = 0,

dx, y(x0 ) = y0 , y(x1 ) = y1 .

Can these functionals have broken extremals, and if so, nd them. Exercise 48.3 Discuss nding a weak extremum for the following: 1. 2. 3. 4.
1 0 1 0

((y )2 2xy) dx,


1 (y )2 2

y(0) = y (0) = 0,

y(1) =

1 120

+ yy + y + y dx y(a) = A, y(b) = B y(1) = 2

b 2 (y a 1 (xy 0

+ 2xyy ) dx,

+ y 2 2y 2 y ) dx,

y(0) = 1,

Exercise 48.4 Find the natural boundary conditions associated with the following functionals: 1. 2.
D D

F (x, y, u, ux , uy ) dx dy p(x, y)(u2 + u2 ) q(x, y)u2 dx dy + x y

(x, y)u2 ds

Here D represents a closed boundary domain with boundary , and ds is the arc-length dierential. p and q are known in D, and is known on . 2067

Exercise 48.5 The equations for water waves with free surface y = h(x, t) and bottom y = 0 are xx + yy = 0 1 1 t + 2 + 2 + gy = 0 x 2 2 y ht + x hx y = 0, y = 0 0 < y < h(x, t), on y = h(x, t), on y = h(x, t), on y = 0,

where the uid motion is described by (x, y, t) and g is the acceleration of gravity. Show that all these equations may be obtained by varying the functions (x, y, t) and h(x, t) in the variational principle
h(x,t)

R 0

1 1 t + 2 + 2 + gy x 2 2 y

dy

dx dt = 0,

where R is an arbitrary region in the (x, t) plane. Exercise 48.6 b Extremize the functional a F (x, y, y ) dx, y(a) = A, y(b) = B given that the admissible curves can not penetrate 10 the interior of a given region R in the (x, y) plane. Apply your results to nd the curves which extremize 0 (y )3 dx, y(0) = 0, y(10) = 0 given that the admissible curves can not penetrate the interior of the circle (x 5)2 + y 2 = 9. Exercise 48.7 Consider the functional y ds where ds is the arc-length dierential (ds = (dx)2 + (dy)2 ). Find the curve or curves from a given vertical line to a given xed point B = (x1 , y1 ) which minimize this functional. Consider both the 1 classes C 1 and Cp . Exercise 48.8 A perfectly exible uniform rope of length L hangs in equilibrium with one end xed at (x1 , y1 ) so that it passes over a frictionless pin at (x2 , y2 ). What is the position of the free end of the rope? 2068

Exercise 48.9 The drag on a supersonic airfoil of chord c and shape y = y(x) is proportional to
c

D=
0

dy dx

dx.

Find the shape for minimum drag if the moment of inertia of the contour with respect to the x-axis is specied; that is, nd the shape for minimum drag if
c

y 2 dx = A,
0

y(0) = y(c) = 0,

(c, A given).

Exercise 48.10 The deection y of a beam executing free (small) vibrations of frequency satises the dierential equation d2 dx2 EI dy dx 2 y = 0,

where EI is the exural rigidity and is the linear mass density. Show that the deection modes are extremals of the problem L EI(y )2 dx 2 0 = 0, (L = length of beam) L y 2 dx 0 when appropriate homogeneous end conditions are prescribed. Show that stationary values of the ratio are the squares of the natural frequencies. Exercise 48.11 A boatman wishes to steer his boat so as to minimize the transit time required to cross a river of width l. The path of the boat is given parametrically by x = X(t), y = Y (t), for 0 t T . The river has no cross currents, so the current velocity is directed downstream in the y-direction. v0 is the constant boat speed relative to the surrounding water, and w = w(x, y, t) denotes the downstream river current at point (x, y) at time t. Then, X(t) = v0 cos (t), Y (t) = v0 sin (t) + w, 2069

where (t) is the steering angle of the boat at time t. Find the steering control function (t) and the nal time T that will transfer the boat from the initial state (X(0), Y (0)) = (0, 0) to the nal state at X(t) = l in such a way as to minimize T . Exercise 48.12 Two particles of equal mass m are connected by an inextensible string which passes through a hole in a smooth horizontal table. The rst particle is on the table moving with angular velocity = g/ in a circular path, of radius , around the hole. The second particle is suspended vertically and is in equilibrium. At time t = 0, the suspended mass is pulled downward a short distance and released while the rst mass continues to rotate. 1. If x represents the distance of the second mass below its equilibrium at time t and represents the angular position of the rst particle at time t, show that the Lagrangian is given by 1 L = m x2 + ( x)2 2 + gx 2 and obtain the equations of motion. 2. In the case where the displacement of the suspended mass from equilibrium is small, show that the suspended mass performs small vertical oscillations and nd the period of these oscillations. Exercise 48.13 A rocket is propelled vertically upward so as to reach a prescribed height h in minimum time while using a given xed quantity of fuel. The vertical distance x(t) above the surface satises, m = mg + mU (t), x x(0) = 0, (x)(0) = 0,

where U (t) is the acceleration provided by engine thrust. We impose the terminal constraint x(T ) = h, and we wish to nd the particular thrust function U (t) which will minimize T assuming that the total thrust of the rocket engine over the entire thrust time is limited by the condition,
T

U 2 (t) dt = k 2 .
0

Here k is a given positive constant which measures the total amount of fuel available. 2070

Exercise 48.14 A space vehicle moves along a straight path in free space. x(t) is the distance to its docking pad, and a, b are its position and speed at time t = 0. The equation of motion is x = M sin V, x(0) = a, x(0) = b,

where the control function V (t) is related to the rocket acceleration U (t) by U = M sin V , M = const. We wish to dock the vehicle in minimum time; that is, we seek a thrust function U (t) which will minimize the nal time T while bringing the vehicle to rest at the origin with x(T ) = 0, x(T ) = 0. Find U (t), and in the (x, x)-plane plot the corresponding trajectory which transfers the state of the system from (a, b) to (0, 0). Account for all values of a and b. Exercise 48.15 m Find a minimum for the functional I(y) = 0 y + h 1 + (y )2 dx in which h > 0, y(0) = 0, y(m) = M > h. Discuss the nature of the minimum, (i.e., weak, strong, . . . ). Exercise 48.16 Show that for the functional n(x, y) 1 + (y )2 dx, where n(x, y) 0 in some domain D, the Weierstrass E function E(x, y, q, y ) is non-negative for arbitrary nite p and y at any point of D. What is the implication of this for Fermats Principle? Exercise 48.17 2 Consider the integral 1+y2 dx between xed limits. Find the extremals, (hyperbolic sines), and discuss the Jacobi, (y ) Legendre, and Weierstrass conditions and their implications regarding weak and strong extrema. Also consider the value of the integral on any extremal compared with its value on the illustrated strong variation. Comment! Pi Qi are vertical segments, and the lines Qi Pi+1 are tangent to the extremal at Pi+1 . Exercise 48.18 x Consider I = x01 y (1 + x2 y ) dx, y(x0 ) = y0 , y(x1 ) = y1 . Can you nd continuous curves which will minimize I if (i) x0 = 1, y0 = 1, x1 = 2, y1 = 4, (ii) x0 = 1, y0 = 3, x1 = 2, y1 = 5, (iii) x0 = 1, y0 = 1, x1 = 2, y1 = 1. 2071

Exercise 48.19 Starting from (Qx Py ) dx dy =


D

(P dx + Q dy)

prove that (a)


D

xx dx dy =
D

xx dx dy +

(x x ) dy, (y y ) dx,

(b)
D

yy dx dy =
D

yy dx dy xy dx dy
D

(c)
D

xy dx dy =
t1

1 2

(x x ) dx +

1 2

(y y ) dy.

Then, consider I(u) =

(uxx + uyy )2 + 2(1 )(uxx uyy u2 ) dx dy dt. xy


t0 D t1

Show that I =

t1

(
t0 D

u)u dx dy dt +
t0

P (u)u + M (u)

(u) n

ds dt,

where P and M are the expressions we derived in class for the problem of the vibrating plate. Exercise 48.20 For the following functionals use the Rayleigh-Ritz method to nd an approximate solution of the problem of minimizing the functionals and compare your answers with the exact solutions.
0 1

(y )2 y 2 2xy dx, For this problem take an approximate solution of the form

y(0) = 0 = y(1).

y = x(1 x) (a0 + a1 x + + an xn ) , 2072

and carry out the solutions for n = 0 and n = 1.


0 2

(y )2 + y 2 + 2xy dx,
1

y(0) = 0 = y(2).

x(y )2

x2 1 2 y 2x2 y x

dx,

y(1) = 0 = y(2)

Exercise 48.21 Let K(x) belong to L1 (, ) and dene the operator T on L2 (, ) by

T f (x) =

K(x y)f (y) dy.

1. Show that the spectrum of T consists of the range of the Fourier transform K of K, (that is, the set of all values K(y) with < y < ), plus 0 if this is not already in the range. (Note: From the assumption on K it follows that K is continuous and approaches zero at .) 2. For in the spectrum of T , show that is an eigenvalue if and only if K takes on the value on at least some interval of positive length and that every other in the spectrum belongs to the continuous spectrum. 3. Find an explicit representation for (T I)1 f for not in the spectrum, and verify directly that this result agrees with that givenby the Neumann series if is large enough. Exercise 48.22 Let U be the space of twice continuously dierentiable functions f on [1, 1] satisfying f (1) = f (1) = 0, and d2 W = C[1, 1]. Let L : U W be the operator dx2 . Call in the spectrum of L if the following does not occur: There is a bounded linear transformation T : W U such that (L I)T f = f for all f W and T (L I)f = f for all f U . Determine the spectrum of L. 2073

Exercise 48.23 Solve the integral equations


1

1. (x) = x +
0 x

x2 y y 2 (y) dy K(x, y)(y) dy


0

2. (x) = x + where

K(x, y) =

sin(xy) for x 1 and y 1, 0 otherwise

In both cases state for which values of the solution obtained is valid. Exercise 48.24 1. Suppose that K = L1 L2 , where L1 L2 L2 L1 = I. Show that if x is an eigenvector of K corresponding to the eigenvalue , then L1 x is an eigenvector of K corresponding to the eigenvalue 1, and L2 x is an eigenvector corresponding to the eigenvalue + 1.
d 2. Find the eigenvalues and eigenfunctions of the operator K dt + t4 in the space of functions u L2 (, ). 2 t d t d (Hint: L1 = 2 + dt , L2 = 2 dt . et /4 is the eigenfunction corresponding to the eigenvalue 1/2.)
2

Exercise 48.25 Prove that if the value of = 1 is in the residual spectrum of T , then 1 is in the discrete spectrum of T . Exercise 48.26 Solve 1. u (t) +
0 1

sin(k(s t))u(s) ds = f (t),

u(0) = u (0) = 0.

2074

2. u(x) =
0

K(x, s)u(s) ds
x+s 2 xs 2

where K(x, s) = 3. (s) =


0

sin 1 log 2 sin

=
n=1

sin nx sin ns n

1 1 h2 (t) dt, 2 1 2h cos(s t) + h2

|h| < 1

4. (x) =

cosn (x )() d

Exercise 48.27 Let K(x, s) = 2 2 6|x s| + 3(x s)2 . 1. Find the eigenvalues and eigenfunctions of
2

(x) =
0

K(x, s)(s) ds.

(Hint: Try to nd an expansion of the form

K(x, s) =
n=

cn en(xs) .)

2. Do the eigenfunctions form a complete set? If not, show that a complete set may be obtained by adding a suitable set of solutions of
2

K(x, s)(s) ds = 0.
0

2075

3. Find the resolvent kernel (x, s, ). Exercise 48.28 Let K(x, s) be a bounded self-adjoint kernel on the nite interval (a, b), and let T be the integral operator on L2 (a, b) with kernel K(x, s). For a polynomial p(t) = a0 +a1 t+ +an tn we dene the operator p(T ) = a0 I +a1 T + +an T n . Prove that the eigenvalues of p(T ) are exactly the numbers p() with an eigenvalue of T . Exercise 48.29 Show that if f (x) is continuous, the solution of

(x) = f (x) +
0

cos(2xs)(s) ds

is (x) = Exercise 48.30 Consider Lu = 0 in D, where f (x) +

f (s) cos(2xs) ds . 1 2 /4

u = f on C,

Lu uxx + uyy + aux + buy + cu. Here a, b and c are continuous functions of (x, y) on D + C. Show that the adjoint L is given by L v = vxx + vyy avx bvy + (c ax by )v and that (vLu uL v) =
D C

H(u, v),

(48.1)

2076

where H(u, v) (vux uvx + auv) dy (vuy uvy + buv) dx u v x y = v u + auv + buv ds. n n n n Take v in (48.1) to be the harmonic Green function G given by G(x, y; , ) = 1 log 2 1 (x )2 + (y )2 + ,

and show formally, (use Delta functions), that (48.1) becomes u(, )
D

u(L )G dx dy =
C

H(u, G)

(48.2)

where u satises Lu = 0, (G = in D, G = 0 on C). Show that (48.2) can be put into the forms u+
D

(c ax by )G aGx bGy u dx dy = U

(48.3)

and u+
D

(aux + buy + cu)G dx dy = U,

(48.4)

where U is the known harmonic function in D with assumes the boundary values prescribed for u. Finally, rigorously show that the integrodierential equation (48.4) can be solved by successive approximations when the domain D is small enough. Exercise 48.31 Find the eigenvalues and eigenfunctions of the following kernels on the interval [0, 1]. 1. K(x, s) = min(x, s) 2077

2. K(x, s) = emin(x,s) (Hint: + + ex = 0 can be solved in terms of Bessel functions.) Exercise 48.32 Use Hilbert transforms to evaluate

1.

sin(kx) sin(lx) dx x2 z 2 cos(px) cos(qx) dx x2 (x2 ab) sin x + (a + b)x cos x dx x(x2 + a2 )(x2 + b2 )

2.

3.

Exercise 48.33 Show that

(1 t2 )1/2 log(1 + t) dt = x log 2 1 + (1 x2 )1/2 arcsin(x) tx 2

Exercise 48.34 Let C be a simple closed contour. Let g(t) be a given function and consider 1 f (t) dt = g(t0 ) C t t0 (48.5)

Note that the left side can be written as F + (t0 ) + F (t0 ). Dene a function W (z) such that W (z) = F (z) for z inside C and W (z) = F (z) for z outside C. Proceeding in this way, show that the solution of (48.5) is given by f (t0 ) = 1 g(t) dt . C t t0 2078

Exercise 48.35 If C is an arc with endpoints and , evaluate (i) 1 1 d, where 0 < < 1 C ( )1 ( ) ( ) 1 n (ii) d, where 0 < < 1, integer n 0. C

Exercise 48.36 Solve


1

y2

(y) dy = f (x). x2

Exercise 48.37 Solve 1 1 f (t) dt = f (x), 0 t x Exercise 48.38 Show that the general solution of tan(x) 1 f (t) dt = f (x) 0 tx is f (x) = Exercise 48.39 Show that the general solution of f (x) +
C

where 1 < < 1.

Are there any solutions for > 1? (The operator on the left is self-adjoint. Its spectrum is 1 1.)

k sin(x) . (1 x)1x/ xx/

f (t) dt = 1 tx

2079

is given by 1 + k ex , (k is a constant). Here C is a simple closed contour, a constant and f (x) a dierentiable function on C. Generalize the result to the case of an arbitrary function g(x) on the right side, where g(x) is analytic inside C. f (x) = Exercise 48.40 Show that the solution of
C

1 + P (t x) f (t) dt = g(x) tx g( ) 1 1 d 2 2 C t g( )P ( t) d.
C

is given by f (t) =

Here C is a simple closed curve, and P (t) is a given entire function of t. Exercise 48.41 Solve
1

3 f (t) f (t) dt + dt = x tx 2 tx

where this equation is to hold for x in either (0, 1) or (2, 3). Exercise 48.42 Solve
x 0

f (t) dt + A xt

1 x

f (t) dt = 1 tx

where A is a real positive constant. Outline briey the appropriate method of A is a function of x.

2080

48.2
Hint 48.1

Hints

Hint 48.2

Hint 48.3

Hint 48.4

Hint 48.5

Hint 48.6

Hint 48.7

Hint 48.8

Hint 48.9

Hint 48.10

2081

Hint 48.11 Hint 48.12 Hint 48.13 Hint 48.14 Hint 48.15 Hint 48.16 Hint 48.17 Hint 48.18 Hint 48.19 Hint 48.20 Hint 48.21

2082

Hint 48.22 Hint 48.23 Hint 48.24 Hint 48.25 Hint 48.26 Hint 48.27 Hint 48.28 Hint 48.29 Hint 48.30 Hint 48.31 Hint 48.32

2083

Hint 48.33 Hint 48.34 Hint 48.35 Hint 48.36 Hint 48.37 Hint 48.38 Hint 48.39 Hint 48.40 Hint 48.41 Hint 48.42

2084

48.3

Solutions

Solution 48.1 C 1 [0, ] Extremals Admissible Extremal. First we consider continuously dierentiable extremals. Because the Lagrangian is a function of y alone, we know that the extremals are straight lines. Thus the admissible extremal is y= Legendre Condition. Fy y = 12( )2 12 y = 12
2

x.

< 0 for |/| < 1 = 0 for |/| = 1 > 0 for |/| > 1
Thus we see that x may be a minimum for |/| 1 and may be a maximum for |/| 1.

Jacobi Condition. Jacobis accessory equation for this problem is (F,y y h ) = 0 12


2

1 h h =0

=0

The problem h = 0, h(0) = 0, h(c) = 0 has only the trivial solution for c > 0. Thus we see that there are no conjugate points and the admissible extremal satises the strengthened Legendre condition. 2085

A Weak Minimum. For |/| > 1 the admissible extremal x is a solution of the Euler equation, and satises the strengthened Jacobi and Legendre conditions. Thus it is a weak minima. (For |/| < 1 it is a weak maxima for the same reasons.)

Weierstrass Excess Function. The Weierstrass excess function is E(x, y , y , w) = F (w) F ( ) (w y )F,y ( ) y y = w4 6w2 ( )4 + 6( )2 (w y )(4( )3 12 ) y y y y = w 6w
4 2 4 2

+6

(w )(4 +3
4

12 )
2

= w 6w w 4

We can nd the stationary points of the excess function by examining its derivative. (Let = /.) E (w) = 4w3 12w + 4 ()2 3 = 0 1 1 4 2 w3 = + 4 2 2 2 The excess function evaluated at these points is w1 = , w2 = E(w1 ) = 0, 3 E(w2 ) = 34 62 6 3(4 2 )3/2 , 2 3 E(w3 ) = 34 62 6 + 3(4 2 )3/2 . 2 E(w2 ) is negative for 1 < < 3 and E(w3 ) is negative for 3 < < 1. This implies that the weak minimum y = x/ is not a strong local minimum for || < 3|. Since E(w1 ) = 0, cannot use the we Weierstrass excess function to determine if y = x/ is a strong local minima for |/| > 3. 2086

1 Cp [0, ] Extremals

Erdmanns Corner Conditions. Erdmanns corner conditions require that y y F,y = 4( )3 12 and F y F,y = ( )4 6( )2 y (4( )3 12 ) y y y y are continuous at corners. Thus the quantities ( )3 3 y y and ( )4 2( )2 y y

are continuous. Denoting p = y and q = y+ , the rst condition has the solutions 1 q 3 4 q 2 . p = q, p = 2 The second condition has the solutions, p = q, Combining these, we have p = q, p= p = 2 q2

3, q = 3, p = 3, q = 3. Thus we see that there can be a corner only when y = 3 and y+ = 3. Case 1, = 3. Notice the the Lagrangian is minimized point-wise if y = 3. For this case the unique, strong global minimum is y = 3 sign()x. Case 2, || < 3||. For this case there are an innite number of strong minima. Any piecewise linear curve satisfying y (x) = 3 and y+ (x) = 3 and y(0) = 0, y() = is a strong minima. Case 3, || > 3||. First note that the extremal cannot have corners. Thus the unique extremal is y = x. We know that this extremal is a weak local minima. 2087

Solution 48.2 1.
x1

(a(y )2 + byy + cy 2 ) dx,


x0

y(x0 ) = y0 ,

y(x1 ) = y1 ,

a=0

y y Erdmanns First Corner Condition. Fy = 2a + b must be continuous at a corner. This implies that y must be continuous, i.e., there are no corners. The functional cannot have broken extremals. 2.
x1

(y )3 dx,
x0

y(x0 ) = y0 ,

y(x1 ) = y1

Erdmanns First Corner Condition. Fy = 3(y )2 must be continuous at a corner. This implies that y = y+ . Erdmanns Second Corner Condition. F y Fy = ( )3 y 3( )2 = 2( )3 must be continuous at a corner. y y y This implies that y is continuous at a corner, i.e. there are no corners. The functional cannot have broken extremals. Solution 48.3 1.
1

(y )2 2xy dx,
0

y(0) = y (0) = 0,

y(1) =

1 120

Eulers Dierential Equation. We will consider C 4 extremals which satisfy Eulers DE, (F,y ) (F,y ) + F,y = 0. For the given Lagrangian, this is, (2 ) 2x = 0. y 2088

Natural Boundary Condition. The rst variation of the performance index is


1

J =
0

(F,y y + F,y y + Fy y ) dx.

From the given boundary conditions we have y(0) = y (0) = y(1) = 0. Using Eulers DE, we have,
1

J =
0

((Fy (F,y ) ) y + F,y y + Fy y ) dx.

Now we apply integration by parts. J = (Fy (F,y ) )y


1 1 1

+
0 0

((Fy (F,y ) )y + F,y y + Fy y ) dx

=
0

((F,y ) y + Fy y ) dx
1 0

= F,y y

= F,y (1)y (1) In order that the rst variation vanish, we need the natural boundary condition F,y (1) = 0. For the given Lagrangian, this condition is y (1) = 0. The Extremal BVP. The extremal boundary value problem is y = x, y(0) = y (0) = y (1) = 0, y(1) = 1 . 120

The general solution of the dierential equation is y = c0 + c1 x + c2 x2 + c3 x3 + 2089 1 5 x. 120

Applying the boundary conditions, we see that the unique admissible extremal is y= This may be a weak extremum for the problem. Legendres Condition. Since F,y the strengthened Legendre condition is satised. Jacobis Condition. The second variation for F (x, y, y ) is d2 J d2 Jacobis accessory equation is, (2F,y
y b y

x2 3 (x 5x + 5). 120

= 2 > 0,

=
=0 a

F,y

(h )2 + 2F,yy hh + F,yy h2 dx

h + 2F,yy h) + 2F,yy h + 2F,yy h = 0, (h ) = 0

Since the boundary value problem, h = 0, h(0) = h (0) = h(c) = h (c) = 0,

has only the trivial solution for all c > 0 the strengthened Jacobi condition is satised. A Weak Minimum. Since the admissible extremal, y= x2 3 (x 5x + 5), 120

satises the strengthened Legendre and Jacobi conditions, we conclude that it is a weak minimum. 2090

2.
0

1 (y )2 + yy + y + y 2

dx

Boundary Conditions. Since no boundary conditions are specied, we have the Euler boundary conditions, F,y (0) = 0, The derivatives of the integrand are, F,y = y + 1, The Euler boundary conditions are then y (0) + y (0) + 1 = 0, y (1) + y (1) + 1 = 0. F,y = y + y + 1. F,y (1) = 0.

Erdmanns Corner Conditions. Erdmanns rst corner condition species that Fy (x) = y (x) + y (x) + 1 must be continuous at a corner. This implies that y (x) is continuous at corners, which means that there are no corners. Eulers Dierential Equation. Eulers DE is (F,y ) = Fy , y + y = y + 1, y = 1. The general solution is 1 y = c0 + c1 x + x2 . 2 2091

The boundary conditions give us the constraints, c0 + c1 + 1 = 0, 5 c0 + 2c1 + = 0. 2 The extremal that satises the Euler DE and the Euler BCs is y= 1 2 x 3x + 1 . 2

Legendres Condition. Since the strengthened Legendre condition is satised, F,y y (x) = 1 > 0, we conclude that the extremal is a weak local minimum of the problem. Jacobis Condition. Jacobis accessory equation for this problem is, F,y y h F,yy (F,yy ) h = 0, h(0) = h(c) = 0,

(h ) ((1) ) h = 0, h = 0,

h(0) = h(c) = 0,

h(0) = h(c) = 0,

Since this has only trivial solutions for c > 0 we conclude that there are no conjugate points. The extremal satises the strengthened Jacobi condition. The only admissible extremal, y= 1 2 x 3x + 1 , 2

satises the strengthened Legendre and Jacobi conditions and is thus a weak extremum. 2092

3.
a

(y 2 + 2xyy ) dx,

y(a) = A,

y(b) = B

Eulers Dierential Equation. Eulers dierential equation, (F,y ) = Fy , (2xy) = 2y + 2xy , 2y + 2xy = 2y + 2xy , is trivial. Every C 1 function satises the Euler DE. Erdmanns Corner Conditions. The expressions, F,y = 2xy, y F y F,y = y 2 + 2xy y (2xh) = y 2

are continuous at a corner. The conditions are trivial and do not restrict corners in the extremal. Extremal. Any piecewise smooth function that satises the boundary conditions y (a) = A, y (b) = B is an admissible extremal. An Exact Derivative. At this point we note that
b b

(y 2 + 2xyy ) dx =
a a

d (xy 2 ) dx dx
b a

= xy 2

= bB 2 aA2 . The integral has the same value for all piecewise smooth functions y that satisfy the boundary conditions. Since the integral has the same value for all piecewise smooth functions that satisfy the boundary conditions, all such functions are weak extrema.

2093

4.
0

(xy + y 2 2y 2 y ) dx,

y(0) = 1,

y(1) = 2

y Erdmanns Corner Conditions. Erdmanns rst corner condition requires F,y = 22 to be continuous, which is trivial. Erdmanns second corner condition requires that F y F,y = x + y 2 22 y y (22 ) = x + y 2 y y y y is continuous. This condition is also trivial. Thus the extremal may have corners at any point. Eulers Dierential Equation. Eulers DE is (F,y ) = F,y , (2y 2 ) = x + 2y 4yy x y= 2 Extremal. There is no piecewise smooth function that satises Eulers dierential equation on its smooth segments and satises the boundary conditions y(0) = 1, y(1) = 2. We conclude that there is no weak extremum. Solution 48.4 1. We require that the rst variation vanishes Fu h + Fux hx + Fuy hy dx dy = 0.
D

We rewrite the integrand as Fu h + (Fux h)x + (Fuy h)y (Fux )x h (Fuy )y h dx dy = 0,


D

2094

Fu (Fux )x (Fuy )y h dx dy +
D D

(Fux h)x + (Fuy h)y dx dy = 0.

Using the Divergence theorem, we obtain, Fu (Fux )x (Fuy )y h dx dy +


D

(Fux , Fuy ) n h ds = 0.

In order that the line integral vanish we have the natural boundary condition, (Fux , Fuy ) n = 0 for (x, y) . We can also write this as dx dy Fuy = 0 for (x, y) . ds ds The Euler dierential equation for this problem is Fux Fu (Fux )x (Fuy )y = 0. 2. We consider the natural boundary conditions for F (x, y, u, ux , uy ) dx dy +
D

G(x, y, u) ds.

We require that the rst variation vanishes. Fu (Fux )x (Fuy )y h dx dy +


D

(Fux , Fuy ) n h ds +

Gu h ds = 0,

Fu (Fux )x (Fuy )y h dx dy +
D

(Fux , Fuy ) n + Gu h ds = 0,

In order that the line integral vanishes, we have the natural boundary conditions, (Fux , Fuy ) n + Gu = 0 for (x, y) . 2095

For the given integrand this is, (2pux , 2puy ) n + 2u = 0 for (x, y) , p u n + u = 0 for (x, y) . We can also denote this as p Solution 48.5 First we vary .
h(x,t)

u + u = 0 for (x, y) . n

( ) =
R 0

1 1 t + t + (x + x )2 + (y + y )2 + gy 2 2
h(x,t)

dy

dx dt

(0) =
R h(x,t) 0

(t + x x + y y ) dy
h(x,t)

dx dt = 0
h(x,t)

(0) =
R

dy [ht ]y=h(x,t) +
0 h(x,t)

x dy [x hx ]y=h(x,t)
0 0

xx dy

+ [y ]h(x,t) 0

yy dy dx dt = 0
0

Since vanishes on the boundary of R, we have


h(x,t)

(0) =
R

[(ht x hx y )]y=h(x,t) [y ]y=0

(xx + yy ) dy dx dt = 0.
0

From the variations which vanish on y = 0, h(x, t) we have


2

= 0.

2096

This leaves us with (0) =


R

[(ht x hx y )]y=h(x,t) [y ]y=0

dx dt = 0.

By considering variations which vanish on y = 0 we obtain, ht x hx y = 0 on y = h(x, t). Finally we have y = 0 on y = 0. Next we vary h(x, t).
h(x,t)+ (x,t)

( ) =
R 0

1 1 t + 2 + 2 + gy x 2 2 y

dx dt

( )=
R

1 1 t + 2 + 2 + gy x 2 2 y

dx dt = 0
y=h(x,t)

This gives us the boundary condition, 1 1 t + 2 + 2 + gy = 0 on y = h(x, t). x 2 2 y Solution 48.6 The parts of the extremizing curve which lie outside the boundary of the region R must be extremals, (i.e., solutions of Eulers equation) since if we restrict our variations to admissible curves outside of R and its boundary, we immediately obtain Eulers equation. Therefore an extremum can be reached only on curves consisting of arcs of extremals and parts of the boundary of region R. Thus, our problem is to nd the points of transition of the extremal to the boundary of R. Let the boundary of R be given by (x). Consider an extremum that starts at the point (a, A), follows an extremal to the point (x0 , (x0 )), follows the R to (x1 , (x1 )) then follows an extremal to the point (b, B). We seek transversality conditions for the points x0 and x1 . We will extremize the expression,
x0 x1 b

I(y) =
a

F (x, y, y ) dx +
x0

F (x, , ) dx +
x1

F (x, y, y ) dx.

2097

Let c be any point between x0 and x1 . Then extremizing I(y) is equivalent to extremizing the two functionals,
x0 c

I1 (y) =
a x1

F (x, y, y ) dx +
x0 b

F (x, , ) dx,

I2 (y) =
c

F (x, , ) dx +
x1

F (x, y, y ) dx,

I = 0

I1 = I2 = 0.

We will extremize I1 (y) and then use the derived transversality condition on all points where the extremals meet R. The general variation of I1 is, I1 (y) = d x Fy dx + [Fy y]x0 + [(F y Fy )x]a0 a dx a c + [F (x)]c 0 + [(F F )x]x0 = 0 x Fy
x0

Note that x = y = 0 at x = a, c. That is, x = x0 is the only point that varies. Also note that (x) is not independent of x. (x) (x)x. At the point x0 we have y (x)x.
x0

I1 (y) =
a

Fy

d Fy dx

dx + (Fy x)
x0

+ ((F y Fy )x)
x0

(F x)
x0 x0

((F F )x)
x0

=0

I1 (y) =
a

Fy

d Fy dx

dx + ((F (x, y, y ) F (x, , ) + ( y )Fy )x)


x0

=0

Since I1 vanishes for those variations satisfying x0 = 0 we obtain the Euler dierential equation, Fy d Fy = 0. dx 2098

Then we have ((F (x, y, y ) F (x, , ) + ( y )Fy )x)


x0

=0

for all variations x0 . This implies that (F (x, y, y ) F (x, , ) + ( y )Fy )


x0

= 0.

Two solutions of this equation are y (x0 ) = (x0 ) and Fy = 0. Transversality condition. If Fy is not identically zero, the extremal must be tangent to R at the points of contact. Now we apply this result to to nd the curves which extremize 0 (y )3 dx, y(0) = 0, y(10) = 0 given that the admissible curves can not penetrate the interior of the circle (x 5)2 + y 2 = 9. Since the Lagrangian is a function of y alone, the extremals are straight lines. The Erdmann corner conditions require that Fy = 3(y )2 and F y Fy = (y )3 y 3(y )2 = 2(y )3
10

are continuous at corners. This implies that y is continuous. There are no corners. We see that the extrema are 3 4 x, for 0 x 16 , 5 y(x) = 9 (x 5)2 , for 16 x 34 , 5 5 3 34 x, for 5 x 10. 4 Note that the extremizing curves neither minimize nor maximize the integral. 2099

Solution 48.7 C1 Extremals. Without loss of generality, we take the vertical line to be the y axis. We will consider x1 , y1 > 1. With ds = 1 + (y )2 dx we extremize the integral,
x1 0

1 + (y )2 dx.

Since the Lagrangian is independent of x, we know that the Euler dierential equation has a rst integral. d Fy Fy = 0 dx y Fy y + y Fy y Fy = 0 d (y Fy F ) = 0 dx y Fy F = const For the given Lagrangian, this is 1 + (y (y )2 y y(1 + (y )2 ) = const 1 + (y )2 , y = const 1 + (y )2 y = const is one solution. To nd the others we solve for y and then solve the dierential equation. y = a(1 + (y )2 ) y = dx = ya a a dy ya 2100 y y y )2 y 1 + (y )2 = const,

x + b = 2 a(y a) y= The natural boundary condition is Fy


x=0

x2 bx b2 + +a 4a 2a 4a = yy
x=0

1 + (y )2 y (0) = 0

= 0,

The extremal that satises this boundary condition is y= Now we apply y(x1 ) = y1 to obtain a= for y1 x1 . The value of the integral is
x1 0

x2 + a. 4a

1 2

y1

2 y1 x2 1

x2 +a 4a

x 1+ 2a

x1 (x2 + 12a2 ) 1 dx = . 12a3/2

By denoting y1 = cx1 , c 1 we have

1 cx1 x1 c2 1 2 The values of the integral for these two values of a are 2 2 3/2 1 + 3c 3c c 1 2(x1 ) . 3(c c2 1)3/2 a= 2101

The values are equal only when c = 1. These values, (divided by x1 ), are plotted in Figure 48.1 as a function of c. The former and latter are ne and coarse dashed lines, respectively. The extremal with a= 1 2 y1 +
2 y1 x2 1

has the smaller performance index. The value of the integral is


2 x1 (x2 + 3(y1 + y1 x2 )2 1 1 . 2 2 3 3 2(y1 + y1 x1 )

The function y = y1 is an admissible extremal for all x1 . The value of the integral for this extremal is x1 y1 which is larger than the integral of the quadratic we analyzed before for y1 > x1 .

3.5

2.5

1.2

1.4

1.6

1.8

Figure 48.1: Thus we see that y= x2 + a, 4a a= 1 2 y1 +


2 y1 x2 1

2102

is the extremal with the smaller integral and is the minimizing curve in C 1 for y1 x1 . For y1 < x1 the C 1 extremum is, y = y1 . C1 Extremals. Consider the parametric form of the Lagrangian. p
t1

y(t) (x (t))2 + (y (t))2 dt


t0

The Euler dierential equations are d fx fx = 0 and dt d fy fy = 0. dt

If one of the equations is satised, then the other is automatically satised, (or the extremal is straight). With either of these equations we could derive the quadratic extremal and the y = const extremal that we found previously. We will nd one more extremal by considering the rst parametric Euler dierential equation. d fx fx = 0 dt d dt y(t)x (t) (x (t))2 + (y (t))2 y(t)x (t) (x (t))2 + (y (t))2 =0

= const

Note that x(t) = const is a solution. Thus the extremals are of the three forms, x = const, y = const, x2 bx b2 y= + + + a. 4a 2a 4a 2103

The Erdmann corner conditions require that Fy = , 1 + (y )2 y(y )2 2 1 + (y ) = 1 + (y )2 yy

F y Fy =

1 + (y )2

are continuous at corners. There can be corners only if y = 0. 1 Now we piece the three forms together to obtain Cp extremals that satisfy the Erdmann corner conditions. The only possibility that is not C 1 is the extremal that is a horizontal line from (0, 0) to (x1 , 0) and then a vertical line from (x1 , y1 ). The value of the integral for this extremal is
y1 0

2 t dt = (y1 )3/2 . 3

Equating the performance indices of the quadratic extremum and the piecewise smooth extremum,
2 x1 (x2 + 3(y1 + y1 x2 )2 2 1 1 = (y1 )3/2 , 2 2 3 3 3 2(y1 + y1 x1 ) 32 3 y1 = x1 . 3

The only real positive solution is

3+2 3 y1 = x1 1.46789 x1 . 3 The piecewise smooth extremal has the smaller performance index for y1 smaller than this value and the quadratic extremal has the smaller performance index for y1 greater than this value.
1 The Cp extremum is the piecewise smooth extremal for y1 x1 and is the quadratic extremal for y1 x1 3 + 2 3/ 3.

3 + 2 3/ 3

2104

Solution 48.8 The shape of the rope will be a catenary between x1 and x2 and be a vertically hanging segment after that. Let the length of the vertical segment be z. Without loss of generality we take x1 = y2 = 0. The potential energy, (relative to y = 0), of a length of rope ds in 0 x x2 is mgy = gy ds. The total potential energy of the vertically hanging rope is m(center of mass)g = z(z/2)g. Thus we seek to minimize,
x2

g
0

1 y ds gz 2 , 2
x2

y(0) = y1 ,

y(x2 ) = 0,

subject to the isoperimetric constraint, ds z = L.


0

Writing the arc-length dierential as ds =


x2

1 + (y )2 dx we minimize y(0) = y1 , y(x2 ) = 0,

g
0

1 1 + (y )2 ds gz 2 , 2
x2

subject to, 1 + (y )2 dx z = L.
0

Consider the more general problem of nding functions y(x) and numbers z which extremize I a F (x, y, y ) dx+ b f (z) subject to J a G(x, y, y ) dx + g(z) = L. Suppose y(x) and z are the desired solutions and form the comparison families, y(x) + 1 1 (x) + 2 2 (x), z + 1 1 + 2 2 . Then, there exists a constant such that (I + J) 1 (I + J) 2
1 , 2 =0

=0 = 0.

1 , 2 =0

2105

These equations are


b a

d H,y Hy 1 dx + h (z)1 = 0, dx

and

d H,y Hy 2 dx + h (z)2 = 0, dx a where H = F + G and h = f + g. From this we conclude that d H,y Hy = 0, dx with determined by
b

h (z) = 0

J=
a

G(x, y, y ) dx + g(z) = L.

Now we apply these results to our problem. Since f (z) = 1 gz 2 and g(z) = z we have 2 gz = 0, z= . g

It was shown in class that the solution of the Euler dierential equation is a family of catenaries, y= + c1 cosh g x c2 c1 .

One can nd c1 and c2 in terms of by applying the end conditions y(0) = y1 and y(x2 ) = 0. Then the expression for y(x) and z = /g are substituted into the isoperimetric constraint to determine . Consider the special case that (x1 , y1 ) = (0, 0) and (x2 , y2 ) = (1, 0). In this case we can use the fact that y(0) = y(1) to solve for c2 and write y in the form y= + c1 cosh g 2106 x 1/2 c1 .

Applying the condition y(0) = 0 would give us the algebraic-transcendental equation, y(0) = + c1 cosh g 1 2c1 = 0,

which we cant solve in closed form. Since we ran into a dead end in applying the boundary condition, we turn to the isoperimetric constraint.
1

1 + (y )2 dx z = L
0 1

cosh
0

x 1/2 c1 1 2c1

dx z = L z =L

2c1 sinh

With the isoperimetric constraint, the algebraic-transcendental equation and z = /g we now have z = c1 cosh z = 2c1 sinh 1 , 2c1 1 L. 2c1

For any xed L, we can numerically solve for c1 and thus obtain z. You can derive that there are no solutions unless L is greater than about 1.9366. If L is smaller than this, the rope would slip o the pin. For L = 2, c1 has the values 0.4265 and 0.7524. The larger value of c1 gives the smaller potential energy. The position of the end of the rope is z = 0.9248. Solution 48.9 Using the method of Lagrange multipliers, we look for stationary values of
c c ((y )2 0

+ y 2 ) dx,

((y )2 + y 2 ) dx = 0.

2107

The Euler dierential equation is d F( , y ) F,y = 0, dx d (2y ) 2y = 0. dx Together with the homogeneous boundary conditions, we have the problem y y = 0, which has the solutions, n 2 nx , yn = an sin , c c Now we determine the constants an with the moment of inertia constraint. n =
c

y(0) = y(c) = 0,

n Z+ .

a2 sin2 n
0

nx c

dx =

ca2 n =A 2

Thus we have the extremals, yn = The drag for these extremals is 2A D= c


c 0

2A nx sin , c c

n Z+ .

n c

cos

nx c

An2 2 dx = . c2

We see that the drag is minimum for n = 1. The shape for minimum drag is y= 2A nx sin . c c 2108

Solution 48.10 Consider the general problem of determining the stationary values of the quantity 2 given by = The variation of 2 is 2 = JI IJ J2 I 1 = I J J J 1 = I 2 J . J
2 b a b a

F (x, y, y , y ) dx G(x, y, y , y ) dx

I . J

The the values of y and y are specied on the boundary, then the variations of I and J are
b

I =
a

d d2 F F,y + F,y y dx, 2 ,y dx dx


b a d2 H dx2 ,y

J =
a

d2 d G G,y + G,y y dx 2 ,y dx dx

Thus 2 = 0 becomes
d H dx ,y b a

+ H,y y dx = 0,

G dx

where H = F 2 G. A necessary condition for an extremum is d2 d H H,y + H,y = 0 where H F 2 G. 2 ,y dx dx For our problem we have F = EI(y )2 and G = y so that the extremals are solutions of d2 dx2 EI dy dx 2 y = 0,

2109

With homogeneous boundary conditions we have an eigenvalue problem with deections modes yn (x) and corresponding natural frequencies n . Solution 48.11 We assume that v0 > w(x, y, t) so that the problem has a solution for any end point. The crossing time is
l

T =
0

X(t)

1 dx = v0

sec (t) dx.


0

Note that w + v0 sin dy = dx v0 cos w = sec + tan v0 w = sec + sec2 1. v0 We solve this relation for sec . y (y )2 2 w sec v0
2

= sec2 1

w w2 y sec + 2 sec2 = sec2 1 v0 v0

2 2 (v0 w2 ) sec2 + 2v0 wy sec v0 ((y )2 + 1) = 0

sec =

2v0 wy

2 2 2 4v0 w2 (y )2 + 4(v0 w2 )v0 ((y )2 + 1) 2 2(v0 w2 )

sec = v0

wy

2 v0 ((y )2 + 1) w2 2 (v0 w2 )

2110

Since the steering angle satises /2 /2 only the positive solution is relevant. sec = v0 wy +
2 v0 ((y )2 + 1) w2 2 (v0 w2 )

Time Independent Current. If we make the assumption that w = w(x, y) then we can write the crossing time as an integral of a function of x and y.
l

T (y) =
0

wy +

2 v0 ((y )2 + 1) w2 dx 2 (v0 w2 )

A necessary condition for a minimum is T = 0. The Euler dierential equation for this problem is d F,y F,y = 0 dx d dx 1 2 v0 w 2 w +
2 v0 y 2 v0 ((y )2

+ 1)

w2

wy 2 (v0 w2 )2

w(v 2 (1 + 2(y )2 ) w2 )
2 v0 ((y )2

+ 1)

w2

2 y (v0 + w2 )

By solving this second order dierential equation subject to the boundary conditions y(0) = 0, y(l) = y1 we obtain the path of minimum crossing time. Current w = w(x). If the current is only a function of x, then the Euler dierential equation can be integrated to obtain, 2 1 v0 y w + = c0 . 2 2 v0 w 2 v0 ((y )2 + 1) w2 Solving for y , y = Since y(0) = 0, we have
x 2 w + c0 (v0 w2 )

v0

2 1 2c0 w c2 (v0 w2 ) 0

y(x) =
0

2 w() + c0 (v0 (w())2 )

v0

2 1 2c0 w() c2 (v0 (w())2 ) 0

2111

For any given w(x) we can use the condition y(l) = y1 to solve for the constant c0 . Constant Current. If the current is constant then the Lagrangian is a function of y alone. The admissible extremals are straight lines. The solution is then y(x) = y1 x . l

Solution 48.12 1. The kinetic energy of the rst particle is 1 m(( x))2 . Its potential energy, relative to the table top, is zero. 2 1 x The kinetic energy of the second particle is 2 m2 . Its potential energy, relative to its equilibrium position is mgx. The Lagrangian is the dierence of kinetic and potential energy. 1 L = m x2 + ( x)2 2 + gx 2 The Euler dierential equations are the equations of motion. d L,x Lx = 0, dt d L L = 0 dt ,

d d (2mx) + m( x)2 mg = 0, m( x)2 2 = 0 dt dt 2 + ( x)2 g = 0, x ( x)2 2 = const When x = 0, = = g/. This determines the constant in the equation of motion for . = g ( x)2 Now we substitute the expression for into the equation of motion for x. 2 + ( x) x 3 g g =0 ( x)4

2112

2 + x

3 1 g =0 ( x)3 1 1 g =0 (1 x/)3

2 + x 2. For small oscillations,


x

1. Recall the binomial expansion,

(1 + z) =
n=0

a n z , n

for |z| < 1, for |z| 1.

(1 + z)a 1 + az, We make the approximation,

x 1 1+3 , (1 x/)3 to obtain the linearized equation of motion, 3g x = 0. This is the equation of a harmonic oscillator with solution 2 + x x = a sin The period of oscillation is, 3g2(t b) .

T = 2 23g.

Solution 48.13 We write the equation of motion and boundary conditions, x = U (t) g, x(0) = x(0) = 0, 2113 x(T ) = h,

as the rst order system, x = 0, x(0) = 0, x(T ) = h, y = U (t) g, y(0) = 0. We seek to minimize,
T

T =
0

dt,

subject to the constraints, x y = 0, y U (t) + g = 0,


T

U 2 (t) dt = k 2 .
0

Thus we seek extrema of


T T

H dt
0 0

1 + (t)(x y) + (t)(y U (t) + g) + U 2 (t) dt.

Since y is not specied at t = T , we have the natural boundary condition, H ,y


t=T

= 0,

(T ) = 0. The rst Euler dierential equation is d H,x H,x = 0, dt d (t) = 0. dt 2114

We see that (t) = is constant. The next Euler DE is d H,y H,y = 0, dt d (t) + = 0. dt (t) = t + const With the natural boundary condition, (T ) = 0, we have (t) = (T t). The nal Euler DE is, d H H,U = 0, dt ,U (t) 2U (t) = 0. Thus we have (T t) . 2 This is the required thrust function. We use the constraints to nd , and T . T Substituting U (t) = (T t)/(2) into the isoperimetric constraint, 0 U 2 (t) dt = k 2 yields U (t) = 2 T 3 = k2, 12 2 3k U (t) = 3/2 (T t). T The equation of motion for x is x = U (t) g = 3k

T 3/2

(T t).

2115

Integrating and applying the initial conditions x(0) = x(0) = 0 yields, x(t) = Applying the condition x(T ) = h gives us, k 1 T 3/2 gT 2 = h, 2 3 1 2 4 k 3 g T T + ghT 2 + h2 = 0. 4 3 kt2 (3T t) 1 2 gt . 2 2 3T 3/2

If k 4 2/3g 3/2 h then this fourth degree polynomial has positive, real solutions for T . With strict inequality, the minimum time is the smaller of the two positive, real solutions. If k < 4 2/3g 3/2 h then there is not enough fuel to reach the target height. Solution 48.14 We have x = U (t) where U (t) is the acceleration furnished by the thrust of the vehicles engine. In practice, the engine will be designed to operate within certain bounds, say M U (t) M , where M is the maximum forward/backward acceleration. To account for the inequality constraint we write U = M sin V (t) for some suitable V (t). More generally, if we had (t) U (t) (t), we could write this as U (t) = + + sin V (t). 2 2 We write the equation of motion as a rst order system, x = y, x(0) = a, x(T ) = 0, y = M sin V, y(0) = b, y(T ) = 0. Thus we minimize T =
0 T

dt

subject to the constraints, xy =0 y M sin V = 0. 2116

Consider H = 1 + (t)(x y) + (t)(y M sin V ). The Euler dierential equations are d d H,x H,x = 0 (t) = 0 (t) = const dt dt d d H,y H,y = 0 (t) + = 0 (t) = t + const dt dt d H,V H,V = 0 (t)M cos V (t) = 0 V (t) = + n. dt 2 + n = M. 2 Therefore, if the rocket is to be transferred from its initial state to is specied nal state in minimum time with a limited source of thrust, (|U | M ), then the engine should operate at full power at all times except possibly for a nite number of switching times. (Indeed, if some power were not being used, we would expect the transfer would be speeded up by using the additional power suitably.) To see how this bang-bang process works, well look at the phase plane. The problem U (t) = M sin x = y, x(0) = c, y = M, y(0) = d, has the solution x(t) = c + dt M We can eliminate t to get x= These curves are plotted in Figure 48.2. 2117 y2 +c 2M d2 . 2M t2 , 2 Thus we see that

y(t) = d M t.

Figure 48.2: There is only curve in each case which transfers the initial state to the origin. We will denote these curves and , respectively. Only if the initial point (a, b) lies on one of these two curves can we transfer the state of the system to the b2 b2 origin along an extremal without switching. If a = 2M and b < 0 then this is possible using U (t) = M . If a = 2M and b > 0 then this is possible using U (t) = M . Otherwise we follow an extremal that intersects the initial position until this curve intersects or . We then follow or to the origin. Solution 48.15 Since the integrand does not explicitly depend on x, the Euler dierential equation has the rst integral, F y Fy = const. y y+h 2y y + h 1 + (y ) = const 1 + (y )2 y+h = const 1 + (y )2 y + h = c2 (1 + (y )2 ) 1 2118

y + h c2 = c1 y 1 c1 dy y + h c2 1 2c1 = dx

y + h c2 = x c2 1

4c2 (y + h c2 ) = (x c2 )2 1 1 Since the extremal passes through the origin, we have 4c2 (h c2 ) = c2 . 2 1 1 4c2 y = x2 2c2 x 1 (48.6)

Introduce as a parameter the slope of the extremal at the origin; that is, y (0) = . Then dierentiating (48.6) at h 2h x = 0 yields 4c2 = 2c2 . Together with c2 = 4c2 (h c2 ) we obtain c2 = 1+2 and c2 = 1+2 . Thus the equation 1 2 1 1 1 of the pencil (48.6) will have the form 1 + 2 2 y = x + x. (48.7) 4h To nd the envelope of this family we dierentiate ( 48.7) with respect to to obtain 0 = x + 2h x2 and eliminate between this and ( 48.7) to obtain x2 y = h + . 4h See Figure 48.3 for a plot of some extremals and the envelope. All extremals (48.7) lie above the envelope which in ballistics is called the parabola of safety. If (m, M ) lies outside 2 the parabola, M < h + m , then it cannot be joined to (0, 0) by an extremal. If (m, M ) is above the envelope 4h then there are two candidates. Clearly we rule out the one that touches the envelope because of the occurrence of conjugate points. For the other extremal, problem 2 shows that E 0 for all y . Clearly we can embed this extremal in an extremal pencil, so Jacobis test is satised. Therefore the parabola that does not touch the envelope is a strong minimum. 2119

x h 2h

-h

Figure 48.3: Some Extremals and the Envelope.

Solution 48.16

E = F (x, y, y ) F (x, y, p) (y p)Fy (x, y, p) np = n 1 + (y )2 n 1 + p2 (y p) 1 + p2 n = 1 + (y )2 1 + p2 (1 + p2 ) (y p)p 2 1+p n = 1 + (y )2 + p2 + (y )2 p2 2y p + 2y p (1 + py ) 2 1+p n = (1 + py )2 + (y p)2 (1 + py ) 2 1+p 0 2120

The speed of light in an inhomogeneous medium is


(b,B)

ds dt

=
b

1 . n(x,y

The time of transit is then

T =
(a,A)

dt ds = ds

n(x, y) 1 + (y )2 dx.
a

Since E 0, light traveling on extremals follow the time optimal path as long as the extremals do not intersect. Solution 48.17 Extremals. Since the integrand does not depend explicitly on x, the Euler dierential equation has the rst integral, F y F,y = const. 2(1 + y 2 ) 1 + y2 y = const (y )2 (y )3 dy 1 + (y )2 = const dx

arcsinh(y) = c1 x + c2 y = sinh(c1 x + c2 ) Jacobi Test. We can see by inspection that no conjugate points exist. Consider the central eld through (0, 0), sinh(cx), (See Figure 48.4). We can also easily arrive at this conclusion analytically as follows: Solutions u1 and u2 of the Jacobi equation are given by u1 = y = cosh(c1 x + c2 ), c2 y u2 = = x cosh(c1 x + c2 ). c1

Since u2 /u1 = x is monotone for all x there are no conjugate points. 2121

-3

-2

-1 -1

-2

-3

Figure 48.4: sinh(cx) Weierstrass Test. E = F (x, y, y ) F (x, y, p) (y p)F,y (x, y, p) 1 + y2 1 + y2 2(1 + y 2 ) = (y p) (y )2 p2 p3 1 + y 2 p3 p(y )2 + 2(y )3 2p(y )2 = (y )2 p2 p 2 2 (p y ) (p + 2y ) 1+y = (y )2 p2 p For p = p(x, y) bounded away from zero, E is one-signed for values of y close to p. However, since the factor (p + 2y ) can have any sign for arbitrary values of y , the conditions for a strong minimum are not satised. Furthermore, since the extremals are y = sinh(c1 x + c2 ), the slope function p(x, y) will be of one sign only if the range of integration is such that we are on a monotonic piece of the sinh. If we span both an increasing and decreasing section, E changes sign even for weak variations. 2122

Legendre Condition. F,y y = 6(1 + y 2 ) >0 (y )4

Note that F cannot be represented in a Taylor series for arbitrary values of y due to the presence of a discontinuity in F when y = 0. However, F,y y > 0 on an extremal implies a weak minimum is provided by the extremal. 2 Strong Variations. Consider 1+y2 dx on both an extremal and on the special piecewise continuous variation in (y ) the gure. On P Q we have y = with implies that 1+y2 = 0 so that there is no contribution to the integral from (y ) P Q. On QR the value of y is greater than its value along the extremal P R while the value of y on QR is less than the 2 value of y along P R. Thus on QR the quantity 1+y2 is less than it is on the extremal P R. (y ) 1 + y2 dx < (y )2 1 + y2 dx (y )2
2

QR

PR

Thus the weak minimum along the extremal can be weakened by a strong variation. Solution 48.18 The Euler dierential equation is d F,y F,y = 0. dx d (1 + 2x2 y ) = 0 dx 1 + 2x2 y = const 1 y = const 2 x c1 y= + c2 x (i) No continuous extremal exists in 1 x 2 that satises y(1) = 1 and y(2) = 4. 2123

4 (ii) The continuous extremal that satises the boundary conditions is y = 7 x . Since F,y y = 2x2 0 has a Taylor series representation for all y , this extremal provides a strong minimum.

(iii) The continuous extremal that satises the boundary conditions is y = 1. This is a strong minimum. Solution 48.19 For identity (a) we take P = 0 and Q = x x . For identity (b) we take P = y y and Q = 0. For identity (c) we take P = 1 (x x ) and Q = 1 (y y ). 2 2 1 1 (y y )x 2 2 (x x )y dx dy =

1 1 (x x ) dx + (y y ) dy 2 2

1 1 (x y + xy x y xy ) + (y x xy y x xy ) 2 2 1 1 = (x x ) dx + (y y ) dy 2 2 xy dx dy
D

dx dy

xy dx dy =
D

1 2

(x x ) dx +

1 2

(y y ) dy

The variation of I is
t1

I =
t0 D

(2(uxx + uyy )(uxx + uyy ) + 2(1 )(uxx uyy + uyy uxx 2uxy uxy )) dx dy dt.

From (a) we have 2(uxx + uyy )uxx dx dy =


D D

2(uxx + uyy )xx u dx dy +

2((uxx + uyy )ux (uxx + uyy )x u) dy.

2124

From (b) we have 2(uxx + uyy )uyy dx dy =


D D

2(uxx + uyy )yy u dx dy

2((uxx + uyy )uy (uxx + uyy )y u) dy.

From (a) and (b) we get 2(1 )(uxx uyy + uyy uxx ) dx dy
D

=
D

2(1 )(uxxyy + uyyxx )u dx dy +

2(1 )((uxx uy uxxy u) dx + (uyy ux uyyx u) dy).

Using c gives us 2(1 )(2uxy uxy ) dx dy =


D D

2(1 )(2uxyxy u) dx dy +

2(1 )(uxy ux uxyx u) dx 2(1 )(uxy uy uxyy u) dy.

Note that Using the above results, we obtain


t1 t1

u ds = ux dy uy dx. n I = 2
t0 D t1

( + 2(1 )

u)u dx dy dt + 2
t0

u)

u + (

u)

(u) n

ds dt dt.

(uyy ux uxy uy ) dy + (uxy ux uxx uy ) dx


t0

2125

Solution 48.20 1. Exact Solution. The Euler dierential equation is d F,y = F,y dx d [2y ] = 2y 2x dx y + y = x. The general solution is y = c1 cos x + c2 sin x x. Applying the boundary conditions we obtain, y= The value of the integral for this extremal is J sin x 2 x = cot(1) 0.0245741. sin 1 3 sin x x. sin 1

n = 0. We consider an approximate solution of the form y(x) = ax(1x). We substitute this into the functional.
1

J(a) =
0

(y )2 y 2 2xy dx =

3 2 1 a a 10 6

The only stationary point is 1 3 J (a) = a = 0 5 6 5 a= . 18 2126

Since J

5 18

3 > 0, 5

we see that this point is a minimum. The approximate solution is y(x) = 5 x(1 x). 18

This one term approximation and the exact solution are plotted in Figure 48.5. The value of the functional is J = 5 0.0231481. 216

0.07 0.06 0.05 0.04 0.03 0.02 0.01 0.2 0.4 0.6 0.8 1

Figure 48.5: One Term Approximation and Exact Solution. n = 1. We consider an approximate solution of the form y(x) = x(1 x)(a + bx). We substitute this into the functional. 1 1 J(a, b) = 63a2 + 63ab + 26b2 35a 21b (y )2 y 2 2xy dx = 210 0 2127

We nd the stationary points. Ja = 1 (18a + 9b 5) = 0 30 1 Jb = (63a + 52b 21) = 0 210 7 71 a= , b= 369 41 Jaa Jab Jba Jbb =
3 5 3 10 3 10 26 105

Since the Hessian matrix H= is positive denite,

3 41 > 0, det(H) = , 5 700 we see that this point is a minimum. The approximate solution is y(x) = x(1 x) 7 71 + x . 369 41

This two term approximation and the exact solution are plotted in Figure 48.6. The value of the functional is J = 136 0.0245709. 5535

2. Exact Solution. The Euler dierential equation is d F,y = F,y dx d [2y ] = 2y + 2x dx y y = x. 2128

0.07 0.06 0.05 0.04 0.03 0.02 0.01 0.2 0.4 0.6 0.8 1

Figure 48.6: Two Term Approximation and Exact Solution. The general solution is y = c1 cosh x + c2 sinh x x. Applying the boundary conditions, we obtain, y= The value of the integral for this extremal is J = 2(e4 13) 0.517408. 3(e4 1) 2 sinh x x. sinh 2

Polynomial Approximation. Consider an approximate solution of the form y(x) = x(2 x)(a0 + a1 x + an xn ). 2129

The one term approximate solution is y(x) = 5 x(2 x). 14

This one term approximation and the exact solution are plotted in Figure 48.7. The value of the functional is J = 10 0.47619. 21

0.5 -0.05 -0.1 -0.15 -0.2 -0.25 -0.3 -0.35

1.5

Figure 48.7: One Term Approximation and Exact Solution. The two term approximate solution is y(x) = x(2 x) 7 33 x . 161 46

This two term approximation and the exact solution are plotted in Figure 48.8. The value of the functional is J = 416 0.51677. 805 2130

0.5 -0.05 -0.1 -0.15 -0.2 -0.25 -0.3 -0.35

1.5

Figure 48.8: Two Term Approximation and Exact Solution. Sine Series Approximation. Consider an approximate solution of the form y(x) = a1 sin The one term approximate solution is y(x) = x 16 sin . 2 + 4) ( 2 x x + a2 sin (x) + + an sin n . 2 2

This one term approximation and the exact solution are plotted in Figure 48.9. The value of the functional is J = The two term approximate solution is y(x) = 16 x 2 sin + sin(x). 2 + 4) 2 + 1) ( 2 ( 2131 2 ( 2 64 0.467537. + 4)

0.5 -0.05 -0.1 -0.15 -0.2 -0.25 -0.3 -0.35

1.5

Figure 48.9: One Term Sine Series Approximation and Exact Solution. This two term approximation and the exact solution are plotted in Figure 48.10. The value of the functional is J = 4(17 2 + 20) 0.504823. 2 ( 4 + 5 2 + 4)

3. Exact Solution. The Euler dierential equation is d F,y = F,y dx d x2 1 [2xy ] = 2 y 2x2 dx x 1 1 y + y + 1 2 y = x x x The general solution is y = c1 J1 (x) + c2 Y1 (x) x 2132

0.5

1.5

-0.1

-0.2

-0.3

Figure 48.10: Two Term Sine Series Approximation and Exact Solution. Applying the boundary conditions we obtain, y= (Y1 (2) 2Y1 (1))J1 (x) + (2J1 (1) J1 (2))Y1 (x) x J1 (1)Y1 (2) Y1 (1)J1 (2)

The value of the integral for this extremal is J 0.310947 Polynomial Approximation. Consider an approximate solution of the form y(x) = (x 1)(2 x)(a0 + a1 x + an xn ). The one term approximate solution is y(x) = (x 1)(2 x) 23 6(40 log 2 23)

2133

This one term approximation and the exact solution are plotted in Figure 48.11. The one term approximation is a surprisingly close to the exact solution. The value of the functional is

J =

529 0.310935. 360(40 log 2 23)

0.2

0.15

0.1

0.05

1.2

1.4

1.6

1.8

Figure 48.11: One Term Polynomial Approximation and Exact Solution.

Solution 48.21 1. The spectrum of T is the set, { : (T I) is not invertible.} 2134

(T I)f = g

K(x y)f (y) dy f (x) = g

K()f () f () = g () K() f () = g ()

We may not be able to solve for f (), (and hence invert T I), if = K(). Thus all values of K() are in the spectrum. If K() is everywhere nonzero we consider the case = 0. We have the equation,

K(x y)f (y) dy = 0

Since there are an innite number of L2 (, ) functions which satisfy this, (those which are nonzero on a set of measure zero), we cannot invert the equation. Thus = 0 is in the spectrum. The spectrum of T is the range of K() plus zero. 2. Let be a nonzero eigenvalue with eigenfunction . (T I) = 0,

x x

K(x y)(y) dy (x) = 0,

Since K is continuous, T is continuous. This implies that the eigenfunction is continuous. We take the Fourier transform of the above equation. K()() () = 0, K() () = 0,

2135

If (x) is absolutely integrable, then () is continous. Since (x) is not identically zero, () is not identically zero. Continuity implies that () is nonzero on some interval of positive length, (a, b). From the above equation we see that K() = for (a, b). Now assume that K() = in some interval (a, b). Any function () that is nonzero only for (a, b) satises K() () = 0, .

By taking the inverse Fourier transform we obtain an eigenfunction (x) of the eigenvalue . 3. First we use the Fourier transform to nd an explicit representation of u = (T I)1 f . u = (T I)1 f (T I)u = f

K(x y)u(y) dy u = f

2 K u = f u f u= 2 K 1 f u= 1 2 K/ For || > |2 K| we can expand the denominator in a geometric series. 1 u= f n=0 u= 1


2 K

n=0

1 n

Kn (x y)f (y) dy

2136

Here Kn is the nth iterated kernel. Now we form the Neumann series expansion. u = (T I)1 f 1 = = = 1 1 1 I T
1

n=0

1 n T f n 1 n T f n 1 n

1 =

n=0

Kn (x y)f (y) dy

n=0

The Neumann series is the same as the series we derived with the Fourier transform. Solution 48.22 We seek a transformation T such that (L I)T f = f. We denote u = T f to obtain a boundary value problem, u u = f, u(1) = u(1) = 0.

This problem has a unique solution if and only if the homogeneous adjoint problem has only the trivial solution. u u = 0, u(1) = u(1) = 0.

This homogeneous problem has the eigenvalues and eigenfunctions, n = n 2


2

un = sin 2137

n (x + 1) , 2

n N.

The inhomogeneous problem has the unique solution


1

u(x) =
1

G(x, ; )f () d

where
sin( (x< +1)) sin( (1x> )) , < 0, sin(2 ) 1 G(x, ; ) = 2 (x< + 1)(1 x> ), = 0, sinh (x +1) sinh (1x ) < ) ( > ) ( , > 0, sinh(2 )

for = (n/2)2 , n N. We set


1

Tf =
1

G(x, ; )f () d

and note that since the kernel is continuous this is a bounded linear transformation. If f W , then
1

(L I)T f = (L I)
1 1

G(x, ; )f () d

=
1 1

(L I)[G(x, ; )]f () d (x )f () d
1

= f (x). 2138

If f U then
1

T (L I)f =
1

G(x, ; ) f () f () d
1 1 1

= [G(x, ; )f ()]1 = [G (x, ; )f ()]1 +


1 1

G (x, ; )f () d
1 1 1

G(x, ; )f () d
1

G (x, ; )f () d
1 1

G(x, ; )f () d

=
1 1

G (x, ; ) G(x, ; ) f () d (x )f () d
1

= f (x). L has the point spectrum n = (n/2)2 , n N. Solution 48.23 1. We see that the solution is of the form (x) = a + x + bx2 for some constants a and b. We substitute this into the integral equation.
1

(x) = x +
0 1

x2 y y 2 (y) dy x2 y y 2 (a + x + bx2 ) dy
0

a + x + bx2 = x +

(15 + 20a + 12b) + (20 + 30a + 15b)x2 a + bx2 = 60 By equating the coecients of x0 and x2 we solve for a and b. a= ( + 60) , 4(2 + 5 + 60) 2139 b= 5( 60) 6(2 + 5 + 60)

Thus the solution of the integral equation is (x) = x + 5 + 60 5( 24) 2 + 60 x + 6 4 .

2. For x < 1 the integral equation reduces to (x) = x. For x 1 the integral equation becomes,
1

(x) = x +
0

sin(xy)(y) dy.

We could solve this problem by writing down the Neumann series. Instead we will use an eigenfunction expansion. Let {n } and {n } be the eigenvalues and orthonormal eigenfunctions of
1

(x) =
0

sin(xy)(y) dy.

We expand (x) and x in terms of the eigenfunctions.

(x) =
n=1

an n (x) bn n (x),
n=1

x=

bn = x, n (x)

We determine the coecients an by substituting the series expansions into the Fredholm equation and equating 2140

coecients of the eigenfunctions.


1

(x) = x +
0

sin(xy)(y) dy
1

an n (x) =
n=1 n=1

bn n (x) +
0

sin(xy)
n=1

an n (y) dy 1 n (x) n

an n (x) =
n=1 n=1

bn n (x) +
n=1

an

an

1 n

= bn

If is not an eigenvalue then we can solve for the an to obtain the unique solution. an = bn n b n bn = = bn + 1 /n n n

(x) = x +
n=1

bn n (x), n

for x 1.

If = m , and x, m = 0 then there is the one parameter family of solutions,

(x) = x + cm (x) +
n=1 n=m

bn n (x), n

for x 1.

If = m , and x, m = 0 then there is no solution. Solution 48.24 1. Kx = L1 L2 x = x 2141

L1 L2 (L1 x) = L1 (L1 l2 I)x = L1 (x x) = ( 1)(L1 x) L1 L2 (L2 x) = (L2 L1 + I)L2 x = L2 x + L2 x = ( + 1)(L2 x) 2. L1 L2 L 2 L 1 = d t d t d t d t + + + + dt 2 dt 2 dt 2 dt 2 2 d t d 1 t d t d t d 1 t d t2 = + + I + I I+ + I dt 2 dt 2 2 dt 4 dt 2 dt 2 2 dt 4 =I

d 1 t2 1 L1 L2 = + I + I = K + I dt 2 4 2 2 t2 /4 We note that e is an eigenfunction corresponding to the eigenvalue = 1/2. Since L1 et /4 = 0 the result 2 2 of this problem does not produce any negative eigenvalues. However, Ln et /4 is the product of et /4 and a 2 polynomial of degree n in t. Since this function is square integrable it is and eigenfunction. Thus we have the eigenvalues and eigenfunctions, 1 n = n , 2 n = t d 2 dt
n1

et

2 /4

for n N.

Solution 48.25 Since 1 is in the residual spectrum of T , there exists a nonzero y such that (T 1 I)x, y = 0 2142

for all x. Now we apply the denition of the adjoint. x, (T 1 I) y = 0, x, (T 1 I)y = 0, (T 1 I)y = 0 y is an eigenfunction of T corresponding to the eigenvalue 1 . Solution 48.26 1.
1

x x

u (t) +
0

sin(k(s t))u(s) ds = f (t),


1 1

u(0) = u (0) = 0 cos(ks)u(s) ds = f (t)


0

u (t) + cos(kt)
0

sin(ks)u(s) ds sin(kt)

u (t) + c1 cos(kt) c2 sin(kt) = f (t) u (t) = f (t) c1 cos(kt) + c2 sin(kt) The solution of u (t) = g(t), using Green functions is
t

u(0) = u (0) = 0

u(t) =
0

(t )g( ) d.

Thus the solution of our problem has the form,


t t t

u(t) =
0

(t )f ( ) d c1
0 t

(t ) cos(k ) d + c2
0

(t ) sin(k ) d

u(t) =
0

(t )f ( ) d c1

1 cos(kt) kt sin(kt) + c2 k2 k2

2143

We could determine the constants by multiplying in turn by cos(kt) and sin(kt) and integrating from 0 to 1. This would yields a set of two linear equations for c1 and c2 . 2. u(x) =
0 n=1

sin nx sin ns u(s) ds n

We expand u(x) in a sine series.


an sin nx =
n=1 0 n=1

sin nx sin ns n

am sin ms
m=1

ds

an sin nx =
n=1 n=1

sin nx n m=1

am sin ns sin ms ds
0

an sin nx =
n=1 n=1

sin nx am mn n m=1 2

sin nx an sin nx = an 2 n=1 n n=1 The eigenvalues and eigenfunctions are n = 3. () =


0

2n ,

un = sin nx,

n N.

1 1 r2 (t) dt, 2 1 2r cos( t) + r2 () = u(r, ), 2144

|r| < 1

We use Poissons formula.

where u(r, ) is harmonic in the unit disk and satises, u(1, ) = (). For a solution we need = 1 and that u(r, ) is independent of r. In this case u() satises

u () = 0,

u() = ().

The solution is () = c1 + c2 . There is only one eigenvalue and corresponding eigenfunction,

= 1,

= c1 + c2 .

4.

(x) =

cosn (x )() d

We expand the kernel in a Fourier series. We could nd the expansion by integrating to nd the Fourier coecients, but it is easier to expand cosn (x) directly.

1 cos (x) = (ex + ex ) 2 1 n nx n (n2)x n n nx e e(n2)x + e e + + + = n n1 n 2 0 1


n

2145

If n is odd,

cosn (x) =

1 2n

n n (enx + enx ) + (e(n2)x + e(n2)x ) + 0 1 + n (ex + ex ) (n 1)/2

= = =

1 2n 1 2n1 1 2n1

n n n 2 cos(nx) + 2 cos((n 2)x) + + 2 cos(x) 0 1 (n 1)/2


(n1)/2

m=0 n

n cos((n 2m)x) m n cos(kx). (n k)/2

k=1 odd k

2146

If n is even, cosn (x) = 1 2n n n (enx + enx ) + (e(n2)x + e(n2)x ) + 0 1 + = 1 2n n n (ei2x + ei2x ) + n/2 1 n/2

n n n n 2 cos((n 2)x) + + 2 cos(2x) + 2 cos(nx) + n/2 0 1 n/2 1


(n2)/2

1 n 1 = n + n1 2 n/2 2 = 1 n 1 + n1 n n/2 2 2

m=0 n

n cos((n 2m)x) m n cos(kx). (n k)/2

k=2 even k

We will denote, cosn (x ) = where ak = a0 2

ak cos(k(x )),
k=1

1 + (1)nk 1 n . n1 (n k)/2 2 2

We substitute this into the integral equation.

(x) =

a0 2

ak cos(k(x )) () d
k=1

(x) =

a0 2

() d +
k=1

ak cos(kx)

cos(k)() d + sin(kx)

sin(k)() d

2147

1 For even n, substituting (x) = 1 yields = a0 . For n and m both even or odd, substituting (x) = cos(mx) 1 or (x) = sin(mx) yields = am . For even n we have the eigenvalues and eigenvectors,

0 = m = 1 , a2m

1 , a0

0 = 1, m = 1, 2, . . . , n/2.

(1) = cos(2mx), m

(2) = sin(2mx), m

For odd n we have the eigenvalues and eigenvectors, m = 1 a2m1 , (1) = cos((2m 1)x), m (2) = sin((2m 1)x), m m = 1, 2, . . . , (n + 1)/2.

Solution 48.27 1. First we shift the range of integration to rewrite the kernel.
2

(x) =
0

2 2 6|x s| + 3(x s)2 (s) ds


x+2

(x) =
x

2 2 6|y| + 3y 2 (x + y) dy

We expand the kernel in a Fourier series.

K(y) = 2 2 6|y| + 3y 2 =
n=

cn eny
6 , n2

cn =

1 2

x+2

K(y) eny dy =
x

0,

n = 0, n=0

6 ny e = K(y) = n2 n=
n=0

n=1

12 cos(ny) n2

2148

K(x, s) =
n=1

12 cos(n(x s)) = n2

n=1

12 cos(nx) cos(nx) + sin(nx) sin(ns) n2

Now we substitute the Fourier series expression for the kernel into the eigenvalue problem.
2

(x) = 12
0 n=1

1 cos(nx) cos(ns) + sin(nx) sin(ns) n2

(s) ds

From this we obtain the eigenvalues and eigenfunctions, n = n2 , 12 1 (1) = cos(nx), n 1 (2) = sin(nx), n n N.

2. The set of eigenfunctions do not form a complete set. Only those functions with a vanishing integral on [0, 2] can be represented. We consider the equation
2

K(x, s)(s) ds = 0
0 2 0

n=1

12 cos(nx) cos(ns) + sin(nx) sin(ns) n2

(s) ds = 0

This has the solutions = const. The set of eigenfunctions 1 0 = , 2 1 (1) = cos(nx), n 1 (2) = sin(nx), n n N,

is a complete set. We can also write the eigenfunctions as 1 n = enx , 2 n Z.

2149

3. We consider the problem u T u = f. For = , ( not an eigenvalue), we can obtain a unique solution for u.
2

u(x) = f (x) +
0

(x, s, )f (s) ds

Since K(x, s) is self-adjoint and L2 (0, 2), we have (x, s, ) = n (x)n (s) n n=
n=0 1 2

=
n= n=0

enx ens
n2 12

= 6

en(xs) n2 12 n=
n=0

(x, s, ) = 12
n=1

cos(n(x s)) n2 12

Solution 48.28 First assume that is an eigenvalue of T , T = .


n

p(T ) =
k=0 n

an T n an n
k=0

= p() 2150

p() is an eigenvalue of p(T ). Now assume that is an eigenvalues of p(T ), p(T ) = . We assume that T has a complete, orthonormal set of eigenfunctions, {n } corresponding to the set of eigenvalues {n }. We expand in these eigenfunctions. p(T ) = p(T ) cn n = cn n cn n

cn p(n )n = p(n ) = ,

n such that cn = 0

Thus all eigenvalues of p(T ) are of the form p() with an eigenvalue of T . Solution 48.29 The Fourier cosine transform is dened, 1 f () = f (x) = 2
0

f (x) cos(x) dx,


0

f () cos(x) d.

We can write the integral equation in terms of the Fourier cosine transform.

(x) = f (x) +
0

cos(2xs)(s) ds

(x) = f (x) + (2x) 2151

(48.8)

We multiply the integral equation by 1

cos(2xs) and integrate. 1


cos(2xs)(x) dx =
0

cos(2xs)f (x) dx +
0 0 0

cos(2xs)(2x) dx

(2s) = f (2s) + 2

cos(xs)(x) dx

(2s) = f (2s) + (s) 4

4 4 (x) = f (2x) + (2x) We eliminate between (48.8) and (48.9). 1 2 4 (x) = f (x) + f (2x) f (s) cos(2xs) ds 1 2 /4
0

(48.9)

(x) = Solution 48.30

f (x) +

vLu dx dy =
D D

v(uxx + uyy + aux + buy + cu) dx dy (v


D 2

= =
D

u + avux + bvuy + cuv) dx dy v + avux + bvuy + cuv) dx dy +


C

(u (u
D

(v u u v) n ds auv
C

v auvx buvy uvax uvby + cuv) dx dy +

x y + buv n n

ds +
C

u v u n n

2152

Thus we see that (vLu uL v) dx dy =


D C

H(u, v) ds,

where L v = vxx + vyy avx bvy + (c ax by )v and H(u, v) = v u v x y u + auv + buv n n n n .

Let G be the harmonic Green function, which satises, G = in D, Let u satisfy Lu = 0. (GLu uL G) dx dy =
D C

G = 0 on C.

H(u, G) ds H(u, G) ds

uL G dx dy =
C

uG dx dy
D

u(L )G dx dy =
C

H(u, G) ds H(u, G) ds
C

u(x )(y ) dx dy
D

u(L )G dx dy =

u(, )
D

u(L )G dx dy =
C

H(u, G) ds

2153

We expand the operators to obtain the rst form. u+


D

u(aGx bGy + (c ax by )G) dx dy =

u G x y u + auG + buG n n n n C G u + ((c ax by )G aGx bGy )u dx dy = u ds n D C G u+


D

ds

((c ax by )G aGx bGy )u dx dy = U

Here U is the harmonic function that satises U = f on C. We use integration by parts to obtain the second form. u+
D

(cuG ax uG by uG auGx buGy ) dx dy = U auG


C

u+
D

(cuG ax uG by uG + (au)x G + (bu)y G) dx dy u+


D

y x + buG n n

ds = U

(cuG ax uG by uG + ax uG + aux G + by uG + buy G) dx dy = U u+


D

(aux + buy + cu)G dx dy = U

Solution 48.31 1. First we dierentiate to obtain a dierential equation.


1 x 1

(x) =
0

min(x, s)(s) ds =
0 1

es (s) ds +
x 1

ex (s) ds (s) ds
x

(x) = x(x) +
x

(s) ds x(x) (x) = (x) 2154

We note that that (x) satises the constraints,


1

(0) =
0

0 (s) ds = 0,
1

(1) =
1

(s) ds = 0.

Thus we have the problem, + = 0, The general solution of the dierential equation is a + bx for = 0 for > 0 for < 0 (0) = (1) = 0.

(x) =

x + b sin x a cos a cosh x + b sinh x

We see that for = 0 and < 0 only the trivial solution satises the homogeneous boundary conditions. For positive the left boundary condition demands that a = 0. The right boundary condition is then =0 b cos The eigenvalues and eigenfunctions are (2n 1) 2
2

n =

n (x) = sin

(2n 1) x , 2

nN

2155

2. First we dierentiate the integral equation.


x 1

(x) =
0

es (s) ds +
x 1

ex (s) ds (s) ds ex (x)

(x) = ex (x) + ex
x 1

= ex
x

(s) ds
1

(x) = ex
x

(s) ds ex (x)

(x) satises the dierential equation + ex = 0. We note the boundary conditions, (0) (0) = 0, In self-adjoint form, the problem is ex The Rayleigh quotient is = = = [ ex ]0 +
1 0 1 1 x e ( )2 0

(1) = 0.

+ = 0,

(0) (0) = 0,

(1) = 0.

dx

2 dx

1 x e ( )2 dx 0 1 2 dx 0 1 ((0))2 + 0 ex ( )2 dx 1 2 dx 0

(0) (0) +

2156

Thus we see that there are only positive eigenvalues. The dierential equation has the general solution (x) = ex/2 aJ1 2 ex/2 + bY1 2 ex/2 We dene the functions, u(x; ) = ex/2 J1 2 ex/2 , v(x; ) = ex/2 Y1 2 ex/2 .

We write the solution to automatically satisfy the right boundary condition, (1) = 0, (x) = v (1; )u(x; ) u (1; )v(x; ). We determine the eigenvalues from the left boundary condition, (0) (0) = 0. The rst few are 1 2 3 4 5 The eigenfunctions are, n (x) = v (1; n )u(x; n ) u (1; n )v(x; n ). Solution 48.32 1. First note that sin(kx) sin(lx) = sign(kl) sin(ax) sin(bx) where a = max(|k|, |l|), Consider the analytic function, e(ab)x e(a+b) = sin(ax) sin(bx) cos(ax) sin(bx). 2 2157 b = min(|k|, |l|). 0.678298 7.27931 24.9302 54.2593 95.3057

sin(kx) sin(lx) sin(ax) sin(bx) dx = sign(kl) dx 2 z2 x x2 z 2 1 sin(ax) sin(bx) sin(ax) sin(bx) = sign(kl) dx 2z xz x+z 1 = sign(kl) ( cos(az) sin(bz) + cos(az) sin(bz)) 2z

sin(kx) sin(lx) dx = sign(kl) cos(az) sin(bz) x2 z 2 z

2. Consider the analytic function, e|p|x e|q|x cos(|p|x) cos(|q|x) + (sin(|p|x) sin(|q|x)) = . x x

cos(px) cos(qx) cos(|p|x) cos(|q|x) dx = dx 2 x x2 sin(|p|x) sin(|q|x) = lim x0 x

cos(px) cos(qx) dx = (|q| |p|) x2

3. We use the analytic function, (x2 ab) sin x + (a + b)x cos x + ((x2 ab) cos x + (a + b)x sin x) (x a)(x b) ex = (x2 + a2 )(x2 + b2 ) (x2 + a2 )(x2 + b2 )

(x2 ab) sin x + (a + b)x cos x (x2 ab) cos x + (a + b)x sin x = lim x0 x(x2 + a2 )(x2 + b2 ) (x2 + a2 )(x2 + b2 ) ab = 2 2 ab 2158

(x2 ab) sin x + (a + b)x cos x = 2 + a2 )(x2 + b2 ) (x ab

Solution 48.33 We consider the function G(z) = (1 z 2 )1/2 + z log(1 + z). For (1 z 2 )1/2 = (1 z)1/2 (1 + z)1/2 we choose the angles, < arg(1 z) < , 0 < arg(1 + z) < 2,

so that there is a branch cut on the interval (1, 1). With this choice of branch, G(z) vanishes at innity. For the logarithm we choose the principal branch, < arg(1 + z) < . For t (1, 1), 1 t2 + t log(1 + t), G (t) = 1 t2 + t log(1 + t), G+ (t) G (t) = 2 1 t2 log(1 + t), 1 + G (t) + G (t) = t log(1 + t). 2 For t (, 1), G+ (t) = 1 t2 + t (log(t 1) + ) , G (t) = 1 t2 + t (log(t 1) ) , G+ (t) =

2159

G+ (t) G (t) = 2 For x (1, 1) we have G(x) =

t2 1 + t .

1 + G (x) + G (x) 2 = x log(1 + x) 2 1 1 1 2( t 1 + t) = dt + 2 tx 2

1 1

2 1 t2 log(1 + t) dt tx

From this we have


1 1

1 t2 log(1 + t) dt tx

= x log(1 + x) +

t2 1 dt t+x 1 = x log(1 + x) 1 + 1 x2 1 x2 arcsin(x) + x log(2) + x log(1 + x) 2 1 1 t2 log(1 + t) dt = x log x 1 + 1 x2 arcsin(x) tx 2 1 t

Solution 48.34 Let F (z) denote the value of the integral. F (z) = From the Plemelj formula we have, F + (t0 ) + F (t0 ) = 1 f (t) dt, C t t0 f (t0 ) = F + (t0 ) F (t0 ). 2160 1 f (t) dt C t z

With W (z) dened as above, we have

W + (t0 ) + W (t0 ) = F + (t0 ) F (t0 ) = f (t0 ),

and also

1 W + (t) W (t) W (t0 ) + W (t0 ) = dt C t t0 1 F + (t) + F (t) = dt C t t0 1 g(t) dt. = C t t0


+

Thus the solution of the integral equation is

f (t0 ) =

1 g(t) dt. C t t0 2161

Solution 48.35 (i)

G+ () = ( )1 G () = e2 G+ () G( ) = ( )1

+ e2 )( )1 G () + G () = (1 + 2 1 (1 e ) d G+ () + G () = 1 ( ) ( ) C ( ) G+ () G () = (1 e2 )( )1 1 d ( )1 = cot() C ( )1 ( ) ( ) ( )

(ii) Consider the branch of

z z 2162

that tends to unity as z . We nd a series expansion of this function about innity.

z z

= =

1 j

z z

(1)j
j=0 j

(1)k
k=0

k z j

=
j=0 k=0

(1)j

jk

jk k k

Dene the polynomial

Q(z) =
j=0 k=0

(1)j

jk

jk k k

z nj .

Then the function

G(z) =

z z 2163

z n Q(z)

vanishes at innity. G () = n Q() 2 G () = e n Q() G+ () G () = n 1 e2 G+ () + G () = n 1 + e2 2Q() 1 1 n 2 1e d = n 1 + e2 2Q() i C n 1 d = cot() n (1 cot())Q() i C


+

1 i C Solution 48.36

n d = cot()

n Q() Q()

1 1 (y) 1 (y) 1 (y) dy = dy dy 2 x2 y 2x 1 y x 2x 1 y + x 1 1 (y) 1 (y) 1 = dy + dy 2x 1 y x 2x 1 y x 1 (y) + (y) 1 = dy 2x 1 yx

2164

1 1 (y) + (y) dy = f (x) 2x 1 yx 1 1 (y) + (y) 2x dy = f (x) 1 yx 1 1 1 2y k (x) + (x) = f (y) 1 y 2 dy + yx 1 x2 1 1 x2 1 2yf (y) 1 y 2 k 1 (x) + (x) = dy + yx 2 1 x2 1 1 x2

(x) =

1 yf (y) 1 y 2 1 k dy + + g(x) 2 yx 1 x 1 1 x2

Here k is an arbitrary constant and g(x) is an arbitrary odd function. Solution 48.37 We dene
1 1 f (t) dt. 2 0 t z The Plemelj formulas and the integral equation give us,

F (z) =

F + (x) F (x) = f (x) F + (x) + F (x) = f (x). We solve for F + and F . F + (x) = ( + 1)f (x) F (x) = ( 1)f (x) By writing F + (x) +1 = (x) F 1 2165

we seek to determine F to within a multiplicative constant.

+1 1 1+ log F + (x) log F (x) = log + 1 log F + (x) log F (x) = + log F + (x) log F (x) = log

We have left o the additive term of 2n in the above equation, which will introduce factors of z k and (z 1)m in F (z). We will choose these factors so that F (z) has integrable algebraic singularites and vanishes at innity. Note that we have dened to be the real parameter, 1+ 1

= log

By the discontinuity theorem,

log F (z) =

1 + 1 dz 2 0 t z 1 1z = log 2 2 z

= log

z1 z

1/2/(2)

2166

F (z) = F (z) = F (x) = F (x) = Dene f (x) = We apply the Plemelj formulas.

z1 z 1

1/2/(2)

z k (z 1)m z1 z 1x x 1x x
/(2)

z(z 1) e(/(2)) x(1 x) e/2 x(1 x) 1 x(1 x)

/(2)

/(2)

1x x

/(2)

1 1 /2 f (t) e e/2 dt = e/2 + e/2 f (x) 0 tx 1 1 f (t) dt = tanh f (x) 0 t x 2 Thus we see that the eigenfunctions are 1 x(1 x) 1x x
tanh1 ()/

(x) =

for 1 < < 1. The method used in this problem cannot be used to construct eigenfunctions for > 1. For this case we cannot nd an F (z) that has integrable algebraic singularities and vanishes at innity. 2167

Solution 48.38 1 1 f (t) dt = f (x) 0 t x tan(x) We dene the function, F (z) = The Plemelj formula are, F + (x) F (x) = f (x) F + (x) + F (x) = We solve for F + and F . F (x) = From this we see f (x). tan(x)
1 1 f (t) dt. 2 0 t z

1 2

tan(x)

f (x)

F + (x) 1 / tan(x) = = e2x . (x) F 1 / tan(x)

We seek to determine F (z) up to a multiplicative constant. Taking the logarithm of this equation yields log F + (x) log F (x) = 2x + 2n. The 2n term will give us the factors (z 1)k and z m in the solution for F (z). We will choose the integers k and m so that F (z) has only algebraic singularities and vanishes at innity. We drop the 2n term for now. log F (z) = log F (z) = z 1 + log 1 2
1 0

2t dt tz z1 z
z/

1z z 2168

F (z) = e1/

We replace e1/ by a multiplicative constant and multiply by (z 1)1 to give F (z) the desired properties. F (z) = We evaluate F (z) above and below the branch cut. F (x) = c c ex = e(x) (1 x)1x/ xx/ (1 x)1x/ xx/ c (z 1)1z/ z z/

Finally we use the Plemelj formulas to determine f (x). f (x) = F + (x) F (x) = Solution 48.39 Consider the equation, f (z) +
C

k sin(x) (1 x)1x/ xx/

f (t) dt = 1. tz

Since the integral is an analytic function of z o C we know that f (z) is analytic o C. We use Cauchys theorem to evaluate the integral and obtain a dierential equation for f (x). f (x) + f (t) dt = 1 C tx f (x) + f (x) = 1 1 + c ex f (t) dt = g(z). tz

f (x) = Consider the equation, f (z) +

2169

Since the integral and g(z) are analytic functions inside C we know that f (z) is analytic inside C. We use Cauchys theorem to evaluate the integral and obtain a dierential equation for f (x). f (t) dt = g(x) C tx f (x) + f (x) = g(x)
x

f (x) +

f (x) =
z0

e(x) g() d + c ex

Here z0 is any point inside C. Solution 48.40

1 + P (t x) f (t) dt = g(x) tx C f (t) 1 1 1 dt = g(x) P (t x)f (t) dt C t x C We know that if 1 f ( ) d = g() C then f () = g( ) 1 d. C 2170

We apply this theorem to the integral equation. f (x) = 1 g(t) dt + 2 C tx g(t) 1 = 2 dt + C tx g(t) 1 dt = 2 C tx 1 1 P ( t)f ( ) d dt 2 C tx C 1 P ( t) dt f ( ) d 2 C C tx 1 P (t x)f (t) dt C

Now we substitute the non-analytic part of f (t) into the integral. (The analytic part integrates to zero.) g(t) 1 dt 2 C t x 1 g(t) = 2 dt C tx 1 g(t) = 2 dt C tx = f (x) = Solution 48.41 Solution 48.42 1 1 2 1 2 P (t x)
C

1 g( ) d dt 2 C t 1 P (t x) dt g( ) d C t

P ( x)g( ) d
C

1 g(t) 1 dt 2 2 C tx

P (t x)g(t) dt
C

2171

Part VII Nonlinear Dierential Equations

2172

Chapter 49 Nonlinear Ordinary Dierential Equations

2173

49.1

Exercises

Exercise 49.1 A model set of equations to describe an epidemic, in which x(t) is the number infected, y(t) is the number susceptible, is dx dy = rxy x, = rxy + , dt dt where r > 0, 0, 0. Initially x = x0 , y = y0 at t = 0. Directly from the equations, without using the phase plane: 1. Find the solution, x(t), y(t), in the case = = 0. 2. Show for the case = 0, = 0 that x(t) rst decreases or increases according as ry0 < or ry0 > . Show that x(t) 0 as t in both cases. Find x as a function of y. 3. In the phase plane: Find the position of the singular point and its type when > 0, > 0. Exercise 49.2 Find the singular points and their types for the system du = ru + v(1 v)(p v), dx dv = u, dx r > 0, 0 < p < 1,

which comes from one of our nonlinear diusion problems. Note that there is a solution with u = (1 v) for special values of and r. Find v(x) for this special case. 2174

Exercise 49.3 Check that r = 1 is a limit cycle for dx = y + x(1 r2 ) dt dy = x + y(1 r2 ) dt (r = x2 + y 2 ), and that all solution curves spiral into it. Exercise 49.4 Consider y = f (y) x x=y Introduce new coordinates, R, given by x = R cos 1 y = R sin and obtain the exact dierential equations for R(t), (t). Show that R(t) continually increases with t when R = 0. Show that (t) continually decreases when R > 1. Exercise 49.5 One choice of the Lorenz equations is x = 10x + 10y y = Rx y xz 8 z = z + xy 3 Where R is a positive parameter. 2175

1. Invistigate the nature of the sigular point at (0, 0, 0) by nding the eigenvalues and their behavior for all 0 < R < . 2. Find the other singular points when R > 1. 3. Show that the appropriate eigenvalues for these other singular points satisfy the cubic 33 + 412 + 8(10 + R) + 160(R 1) = 0. 4. There is a special value of R, call it Rc , for which the cubic has two pure imaginary roots, say. Find Rc and ; then nd the third root. Exercise 49.6 In polar coordinates (r, ), Einsteins equations lead to the equation d2 v + v = 1 + v2, d2 1 v= , r

for planetary orbits. For Mercury, = 8 108 . When = 0 (Newtonian theory) the orbit is given by v = 1 + A cos , period 2. Introduce = and use perturbation expansions for v() and in powers of to nd the corrections proportional to . [A is not small; is the small parameter]. Exercise 49.7 Consider the problem
2 x + 0 x + x2 = 0,

x = a, x = 0 at t = 0

Use expansions x = a cos + a2 x2 () + a3 x3 () + , = t = 0 + a2 2 + , 2176

to nd a periodic solution and its natural frequency . Note that, with the expansions given, there are no secular term troubles in the determination of x2 (), but x2 () is needed in the subsequent determination of x3 () and . Show that a term a1 in the expansion for would have caused trouble, so 1 would have to be taken equal to zero. Exercise 49.8 Consider the linearized trac problem dpn (t) = [pn1 (t) pn (t)] , n 1, dt pn (0) = 0, n 1, p0 (t) = aet , t > 0. (We take the imaginary part of pn (t) in the nal answers.) 1. Find p1 (t) directly from the equation for n = 1 and note the behavior as t . 2. Find the generating function G(s, t) =
n=1

pn (t)sn .

3. Deduce that pn (t) An et , as t , and nd the expression for An . Find the imaginary part of this pn (t). Exercise 49.9 1. For the equation modied with a reaction time, namely d pn (t + ) = [pn1 (t) pn (t)] n 1, dt nd a solution of the form in 1(c) by direct substitution in the equation. Again take its imaginary part. 2177

2. Find a condition that the disturbance is stable, i.e. pn (t) remains bounded as n . 3. In the stable case show that the disturbance is wave-like and nd the wave velocity.

2178

49.2
Hint 49.1 Hint 49.2 Hint 49.3 Hint 49.4 Hint 49.5 Hint 49.6 Hint 49.7 Hint 49.8 Hint 49.9

Hints

2179

49.3

Solutions

Solution 49.1 1. When = = 0 the equations are dx = rxy, dt Adding these two equations we see that dy = rxy. dt

dx dy = . dt dt

Integrating and applying the initial conditions x(0) = x0 and y(0) = y0 we obtain

x = x0 + y0 y

Substituting this into the dierential equation for y,

dy = r(x0 + y0 y)y dt dy = r(x0 + y0 )y + ry 2 . dt 2180

We recognize this as a Bernoulli equation and make the substitution u = y 1 . y 2 dy = r(x0 + y0 )y 1 r dt du = r(x0 + y0 )u r dt
t

d r(x0 +y0 )t e u = rer(x0 +y0 )t dt u = er(x0 +y0 )t u= y= Applying the initial condition for y, 1 +c = y0 x0 + y0 1 1 c= . y0 x0 + y0 The solution for y is then 1 y= + x0 + y0 1 1 y0 x0 + y0
1 1

rer(x0 +y0 )t dt + cer(x0 +y0 )t 1 + cer(x0 +y0 )t x0 + y0 1 1 + cer(x0 +y0 )t x0 + y0

r(x0 +y0 )t

Since x = x0 + y0 y, the solution to the system of dierential equations is x = x0 + y0 1 1 + 1 er(x0 +y0 )t y0 x0 + y0


1

y=

1 1 + 1 er(x0 +y0 )t y0 x0 + y0

2181

2. For = 0, = 0, the equation for x is x = rxy x. At t = 0, x(0) = x0 (ry0 ). Thus we see that if ry0 < , x is initially decreasing. If ry0 > , x is initially increasing. Now to show that x(t) 0 as t . First note that if the initial conditions satisfy x0 , y0 > 0 then x(t), y(t) > 0 for all t 0 because the axes are a seqaratrix. y(t) is is a strictly decreasing function of time. Thus we see that at some time the quantity x(ry ) will become negative. Since y is decreasing, this quantity will remain negative. Thus after some time, x will become a strictly decreasing quantity. Finally we see that regardless of the initial conditions, (as long as they are positive), x(t) 0 as t . Taking the ratio of the two dierential equations, dx = 1 + . dy ry x = y + ln y + c r Applying the intial condition, x0 = y0 + ln y0 + c r c = x0 + y0 ln y0 . r ln r y y0 .

Thus the solution for x is x = x0 + (y0 y) + 3. When > 0 and > 0 the system of equations is x = rxy x y = rxy + . 2182

The equilibrium solutions occur when x(ry ) = 0 rxy = 0. Thus the singular point is x= , y= . r

Now to classify the point. We make the substitution u = (x ), v = (y ). r u=r u+ v+ u+ r v+ + r

v = r u +

u=

r v + ruv r v = u v ruv

The linearized system is u= r v r v

v = u Finding the eigenvalues of the linearized system,

r r 2 + r = 0 r = + + 2183

2 Since both eigenvalues have negative real part, we see that the singular point is asymptotically stable. A plot of the vector eld for r = = = 1 is attached. We note that there appears to be a stable singular point at x = y = 1 which corroborates the previous results. Solution 49.2 The singular points are u = 0, v = 0, u = 0, v = 1, u = 0, v = p. The point u = 0, v = 0. The linearized system about u = 0, v = 0 is du = ru dx dv = u. dx The eigenvalues are r 0 = 2 r = 0. 1 = 0, r. Since there are positive eigenvalues, this point is a source. The critical point is unstable. The point u = 0, v = 1. Linearizing the system about u = 0, v = 1, we make the substitution w = v 1. du = ru + (w + 1)(w)(p 1 w) dx dw =u dx du = ru + (1 p)w dx dw =u dx 2184

( r )2 4r

r (p 1) = 2 r + p 1 = 0 1 = r r2 4(p 1) 2

Thus we see that this point is a saddle point. The critical point is unstable. The point u = 0, v = p. Linearizing the system about u = 0, v = p, we make the substitution w = v p. du = ru + (w + p)(1 p w)(w) dx dw =u dx

du = ru + p(p 1)w dx dw =u dx r p(1 p) = 2 r + p(1 p) = 0 1 = r r2 4p(1 p) 2

Thus we see that this point is a source. The critical point is unstable. The solution of for special values of and r. Dierentiating u = v(1 v), du = 2v. dv 2185

Taking the ratio of the two dierential equations, du v(1 v)(p v) =r+ dv u v(1 v)(p v) =r+ v(1 v) (p v) =r+ Equating these two expressions, 2v = r + p v .

Equating coecients of v, we see that =

1 . 2

1 = r + 2p 2 Thus we have the solution u =


1 v(1 2

v) when r =

1 2

2p. In this case, the dierential equation for v is

1 dv = v(1 v) dx 2 dv 1 1 v 2 = v 1 + dx 2 2 2186

We make the change of variablles y = v 1 . dy 1 1 = y + dx 2 2 x/ 2 d e ex/ 2 y = dx 2 x x/ 2 e dx + cex/ 2 y = ex/ 2 2 y = 1 + cex/ 2 The solution for v is v(x) = Solution 49.3 We make the change of variables x = r cos y = r sin . Dierentiating these expressions with respect to time, x = r cos r sin y = r sin + r cos . Substituting the new variables into the pair of dierential equations, r cos r sin = r sin + r cos (1 r2 ) r sin + r cos = r cos + r sin (1 r2 ). 2187 1 . 1 + cex/ 2

Multiplying the equations by cos and sin and taking their sum and dierence yields r = r(1 r2 ) r = r. We can integrate the second equation. r = r(1 r2 ) = t + 0 At this point we could note that r > 0 in (0, 1) and r < 0 in (1, ). Thus if r is not initially zero, then the solution tends to r = 1. Alternatively, we can solve the equation for r exactly. r = r r3 1 r = 2 1 3 r r We make the change of variables u = 1/r2 . 1 u=u1 2 u + 2u = 2
t

u = e2t

2e2t dt + ce2t

u = 1 + ce2t 1 r= 1 + ce2t Thus we see that if r is initiall nonzero, the solution tends to 1 as t . 2188

Solution 49.4 The set of dierential equations is y = f (y) x x = y. We make the change of variables x = R cos 1 y = R sin Dierentiating x and y, x = R cos R sin 1 1 y = R sin + R cos . The pair of dierential equations become R sin + R cos = f 1 R sin R cos

1 R cos R sin = R sin .

1 1 R sin + R cos = R cos f 1 R cos R sin = R sin .

1 R sin

2189

Multiplying by cos and sin and taking the sum and dierence of these dierential equations yields 1 R = sin f 1 R sin 1 R sin .

1 1 R = R + cos f Dividing by R in the second equation, 1 R = sin f 1 R sin

1 1 cos f = + R We make the assumptions that 0 < satises |f (y)| 1 for all y. Since sin is odd,

1 R sin .

< 1 and that f (y) is an odd function that is nonnegative for positive y and

sin f

1 R sin

is nonnegative. Thus R(t) continually increases with t when R = 0. If R > 1 then cos f R 1 R sin f 1. Thus the value of , 1 1 cos + f R is always nonpositive. Thus (t) continually decreases with t. 2190 1 R sin , 1 R sin

Solution 49.5 1. Linearizing the Lorentz equations about (0, 0, 0) yields x 10 10 0 x y = R 1 y 0 z 0 0 8/3 z The eigenvalues of the matrix are 8 1 = , 3 11 81 + 40R 2 = 2 11 + 81 + 40R 3 = . 2 There are three cases for the eigenvalues of the linearized system. R < 1. There are three negative, real eigenvalues. In the linearized and also the nonlinear system, the origin is a stable, sink. R = 1. There are two negative, real eigenvalues and one zero eigenvalue. In the linearized system the origin is stable and has a center manifold plane. The linearized system does not tell us if the nonlinear system is stable or unstable. R > 1. There are two negative, real eigenvalues, and one positive, real eigenvalue. The origin is a saddle point. 2. The other singular points when R > 1 are 8 (R 1), 3 8 (R 1), R 1 . 3

2191

3. Linearizing about the point 8 (R 1), 3 yields X Y = Z 10 1


8 (R 3

8 (R 1), R 1 3 10 1
8 (R 3

1)

1)

X 8 (R 1) Y 3 Z 8
3

The characteristic polynomial of the matrix is 3 + 41 2 8(10 + R) 160 + + (R 1). 3 3 3

Thus the eigenvalues of the matrix satisfy the polynomial, 33 + 412 + 8(10 + R) + 160(R 1) = 0. Linearizing about the point yields X Y = Z 10 1
8 (R 3

8 (R 1), 3

8 (R 1), R 1 3 10 1 0

1)

8 (R 3

1)

X 8 (R 1) Y 3 8 Z
3

The characteristic polynomial of the matrix is 3 + 41 2 8(10 + R) 160 + + (R 1). 3 3 3 2192

Thus the eigenvalues of the matrix satisfy the polynomial, 33 + 412 + 8(10 + R) + 160(R 1) = 0. 4. If the characteristic polynomial has two pure imaginary roots and one real root, then it has the form ( r)(2 + 2 ) = 3 r2 + 2 r2 . Equating the 2 and the term with the characteristic polynomial yields r= 41 , 3 = 8 (10 + R). 3

Equating the constant term gives us the equation 41 8 160 (10 + Rc ) = (Rc 1) 3 3 3 which has the solution 470 . 19 For this critical value of R the characteristic polynomial has the roots Rc = 1 = 2 = 41 3 4

2090 4 3 = 2090. 19 19 Solution 49.6 The form of the perturbation expansion is v() = 1 + A cos + u() + O( 2 ) = (1 + 1 + O( 2 )). 2193

Writing the derivatives in terms of , d d = (1 + 1 + ) d d 2 d d2 = (1 + 2 1 + ) 2 . d2 d Substituting these expressions into the dierential equation for v(), 1 + 2 1 + O( 2 ) A cos + u + O( 2 ) + 1 + A cos + u() + O( 2 ) =1+ 1 + 2A cos + A2 cos2 + O( )

u + u 2 1 A cos = + 2 A cos + A2 cos2 + O( 2 ). Equating the coecient of , 1 u + u = 1 + 2 (1 + 1 )A cos + A2 (cos 2 + 1) 2 1 2 1 u + u = (1 + A ) + 2 (1 + 1 )A cos + A2 cos 2. 2 2 To avoid secular terms, we must have 1 = 1. A particular solution for u is 1 1 u = 1 + A2 A2 cos 2. 2 6 The the solution for v is v() = 1 + A cos((1 )) + 1 1 1 + A2 A2 cos(2(1 )) + O( 2 ). 2 6

2194

Solution 49.7 Substituting the expressions for x and into the dierential equations yields
2 a2 0

d2 x2 + x2 d2

2 + cos2 + a3 0

d2 x3 + x3 d2

20 2 cos + 2x2 cos + O(a4 ) = 0

Equating the coecient of a2 gives us the dierential equation d2 x2 + x2 = 2 (1 + cos 2). d2 20 The solution subject to the initial conditions x2 (0) = x2 (0) = 0 is x2 = (3 + 2 cos + cos 2). 2 60

Equating the coecent of a3 gives us the dierential equation


2 0

d2 x3 + x3 d2

2 52 + 2 20 2 + 2 30 60 2 =

2 2 cos + 2 cos 2 + 2 cos 3 = 0. 30 60

To avoid secular terms we must have

52 . 120 Solving the dierential equation for x3 subject to the intial conditions x3 (0) = x3 (0) = 0, x3 = Thus our solution for x(t) is x(t) = a cos + a2 2 (3 + 2 cos + cos 2) + a3 (48 + 29 cos + 16 cos 2 + 3 cos 3) + O(a4 ) 2 4 60 1440 2195 2 (48 + 29 cos + 16 cos 2 + 3 cos 3). 4 1440

5 where = 0 a2 120 t. Now to see why we didnt need an a1 term. Assume that

x = a cos + a2 x2 () + O(a3 ); = 0 + a1 + O(a2 ). Substituting these expressions into the dierential equation for x yields

= t

2 a2 0 (x2 + x2 ) 20 1 cos + cos2 = O(a3 )

x2 + x2 = 2

1 cos 2 (1 + cos 2). 0 20

In order to eliminate secular terms, we need 1 = 0. Solution 49.8 1. The equation for p1 (t) is dp1 (t) = [p0 (t) p1 (t)]. dt dp1 (t) = [aet p1 (t)] dt d t e p1 (t) = aet et dt a t e + cet p1 (t) = + Applying the initial condition, p1 (0) = 0, p1 (t) = a et et + 2196

2. We start with the dierential equation for pn (t). dpn (t) = [pn1 (t) pn (t)] dt Multiply by sn and sum from n = 1 to .

pn (t)s =
n=1 n=1

[pn1 (t) pn (t)]sn

G(s, t) = pn sn+1 G(s, t) t n=0 G(s, t) = sp0 + pn sn+1 G(s, t) t n=1 G(s, t) = aset + sG(s, t) G(s, t) t G(s, t) = aset + (s 1)G(s, t) t (1s)t e G(s, t) = ase(1s)t et t as G(s, t) = et + C(s)e(s1)t (1 s) + The initial condition is G(s, 0) =
n=1

pn (0)sn = 0.

The generating function is then G(s, t) = as et e(s1)t . (1 s) +

2197

3. Assume that |s| < 1. In the limit t we have as et (1 s) + as G(s, t) et 1 + / s as/(1 + /) t e G(s, t) 1 s/(1 + /) G(s, t) aset G(s, t) 1 + / G(s, t) aet
n=1

n=0

s 1 + /

sn (1 + /)n

Thus we have pn (t) a et n (1 + /) as t .

(pn (t))

a et n (1 + /) n 1 / =a [cos(t) + sin(t)] 1 + (/)2 a = [cos(t) [(1 /)n ] + sin(t) [(1 /)n ]] (1 + (/)2 )n
n

a = (1 + (/)2 )n

cos(t)

(1)
j=1 odd j

(j+1)/2

+ sin(t)
j=0 even j

(1)j/2

2198

Solution 49.9 1. Substituting pn = An et into the dierential equation yields

An e(t+ ) = [An1 et An et ] An ( + e ) = An1

We make the substitution An = rn .

rn ( + e ) = rn1 r= + e

Thus we have

pn (t) =

1 1 + e / 2199

et .

Taking the imaginary part,

(pn (t)) = = = =

1 et 1 + e n 1 e cos(t) + sin(t) 1 + (e e ) + ( )2 1 sin( ) cos( ) n cos(t) + sin(t) 1 2 sin( ) + ( )2 n 1 cos(t) 1 sin( ) cos( ) 2 1 2 sin( ) + ( ) n + sin(t) 1 sin( ) cos( ) n 1 1 2 sin( ) + ( )2
n

cos(t)
j=1 odd j n

(1)(j+1)/2

cos( )
j

sin( )

nj

+ sin(t)
j=0 even j

(1)j/2

cos( )

sin( )

nj

2200

2. pn (t) will remain bounded in time as n if 1 1 1 + e 2 1 + e 1 2 1 1 2 sin( ) + 2 sin( ) 3.

2201

Chapter 50 Nonlinear Partial Dierential Equations

2202

50.1

Exercises
ut + uux = 0.

Exercise 50.1 Consider the nonlinear PDE The solution u is constant along lines (characteristics) such that x ut = k for any constant k. Thus the slope of these lines will depend on the initial data u(x, 0) = f (x). 1. In terms of this initial data, write down the equation for the characteristic in the x, t plane which goes through the point (x, t) = (, 0). 2. State a criteria on f such that two characteristics will intersect at some positive time t. Assuming intersections do occur, what is the time of the rst intersection? You may assume that f is everywhere continuous and dierentiable. 3. Apply this to the case where f (x) = 1 ex to indicate where and when a shock will form and sketch (roughly) the solution both before and after this time. Exercise 50.2 Solve the equation t + (1 + x)x + = 0 in with initial condition (x, 0) = f (x). Exercise 50.3 Solve the equation t + x + =0 1+x < x < , t > 0,
2

in the region 0 < x < , t > 0 with initial condition (x, 0) = 0, and boundary condition (0, t) = g(t). [Here is a positive constant.] 2203

Exercise 50.4 Solve the equation t + x + 2 = 0 in < x < , t > 0 with initial condition (x, 0) = f (x). Note that the solution could become innite in nite time. Exercise 50.5 Consider ct + ccx + c = 0, < x < , t > 0. 1. Use the method of characteristics to solve the problem with c = F (x) at t = 0. ( is a positive constant.) 2. Find equations for the envelope of characteristics in the case F (x) < 0. 3. Deduce an inequality relating max |F (x)| and which decides whether breaking does or does not occur. Exercise 50.6 For water waves in a channel the so-called shallow water equations are ht + (hv)x = 0 1 (hv)t + hv 2 + gh2 = 0, g = constant. 2 x (50.1) (50.2)

Investigate whether there are solutions with v = V (h), where V (h) is not posed in advance but is obtained from requiring consistency between the h equation obtained from (1) and the h equation obtained from (2). There will be two possible choices for V (h) depending on a choice of sign. Consider each case separately. In each case x the arbitrary constant that arises in V (h) by stipulating that before the waves arrive, h is equal to the undisturbed depth h0 and V (h0 ) = 0. Find the h equation and the wave speed c(h) in each case. 2204

Exercise 50.7 After a change of variables, the chemical exchange equations can be put in the form + =0 t x = ; t , , = positive constants. (50.3) (50.4)

1. Investigate wave solutions in which = (X), = (X), X = x U t, U = constant, and show that (X) must satisfy an ordinary dierential equation of the form d = quadratic in . dX 2. Discuss ths smooth shock solution as we did for a dierent example in class. In particular nd the expression for U in terms of the values of as X , and nd the sign of d/dX. Check that U= in agreement with the discontinuous theory. Exercise 50.8 Find solitary wave solutions for the following equations: 1. t + x + 6x xxt = 0. (Regularized long wave or B.B.M. equation) 2. utt uxx
3 2 u xx 2

2 1 2 1

uxxxx = 0. (Boussinesq)

3. tt xx + 2x xt + xx t xxxx = 0. (The solitary wave form is for u = x ) 4. ut + 30u2 u1 + 20u1 u2 + 10uu3 + u5 = 0. (Here the subscripts denote x derivatives.)

2205

50.2
Hint 50.1 Hint 50.2 Hint 50.3 Hint 50.4 Hint 50.5 Hint 50.6 Hint 50.7 Hint 50.8

Hints

2206

50.3

Solutions

Solution 50.1 1. x = + u(, 0)t x = + f ()t 2. Consider two points 1 and 2 where 1 < 2 . Suppose that f (1 ) > f (2 ). Then the two characteristics passing through the points (1 , 0) and (2 , 0) will intersect. 1 + f (1 )t = 2 + f (2 )t 2 1 t= f (1 ) f (2 ) We see that the two characteristics intersect at the point (x, t) = 1 + f (1 ) 2 1 2 1 , f (1 ) f (2 ) f (1 ) f (2 ) .

We see that if f (x) is not a non-decreasing function, then there will be a positive time when characteristics intersect. Assume that f (x) is continuously dierentiable and is not a non-decreasing function. That is, there are points where f (x) is negative. We seek the time T of the rst intersection of characteristics. T =
1 <2 f (1 )>f (2 )

min

2 1 f (1 ) f (2 )

(f (2 )f (1 ))/(2 1 ) is the slope of the secant line on f (x) that passes through the points 1 and 2 . Thus we seek the secant line on f (x) with the minimum slope. This occurs for the tangent line where f (x) is minimum. T = 1 min f ()

2207

3. First we nd the time when the characteristics rst intersect. We nd the minima of f (x) with the derivative test. f (x) = 1 ex f (x) = 2x ex
2 2 2

The minimum slope occurs at x = 1/ 2. T =

f (x) = 2 4x2 ex = 0 1 x = 2

e1/2 = 1.16582 2 e1/2 / 2 2 1

Figure 50.1 shows the solution at various times up to the rst collision of characteristics, when a shock forms. After this time, the shock wave moves to the right. Solution 50.2 The method of characteristics gives us the dierential equations x (t) = (1 + x) d = dt Solving the rst dierential equation, x(t) = cet 1, x(0) = t x(t) = ( + 1)e 1 The second dierential equation then becomes (x(t), t) = cet , (, 0) = f (), = (x + 1)et 1 (x, t) = f ((x + 1)et 1)et 2208 x(0) = (, 0) = f ()

1 0.8 0.6 0.4 0.2 -3 -2 -1 1 0.8 0.6 0.4 0.2 -3 -2 -1

1 2 3

1 0.8 0.6 0.4 0.2 -3 -2 -1 1 0.8 0.6 0.4 0.2 -3 -2 -1

1 2 3

1 2 3

1 2 3

Figure 50.1: The solution at t = 0, 1/2, 1, 1.16582. Thus the solution to the partial dierential equation is (x, t) = f ((x + 1)et 1)et . Solution 50.3 d = t + x (t)x = dt 1+x The characteristic curves x(t) satisfy x (t) = 1, so x(t) = t + c. The characteristic curve that separates the region with domain of dependence on the x axis and domain of dependence on the t axis is x(t) = t. Thus we consider the two cases x > t and x < t. x > t. x(t) = t + . 2209

x < t. x(t) = t . Now we solve the dierential equation for in the two domains. x > t. d = , (, 0) = 0, =xt dt 1+x d = dt 1+t+ t 1 dt = c exp t++1 = cexp ( log(t + + 1)) = c(t + + 1) applying the initial condition, we see that =0 x < t. d = , dt 1+x (0, ) = g( ), =tx

d = dt 1+t = c(t + 1 ) = g( )(t + 1 ) = g(t x)(x + 1) 2210

Thus the solution to the partial dierential equation is (x, t) = 0 g(t x)(x + 1) for x > t for x < t.

Solution 50.4 The method of characteristics gives us the dierential equations x (t) = 1 d = 2 dt Solving the rst dierential equation, x(t) = t + . The second dierential equation is then d = 2 , dt (, 0) = f (), 2 d = dt 1 = t + c 1 = tc 1 = t + 1/f () = 1 . t + 1/f (x t) =xt x(0) = (, 0) = f ()

Solution 50.5 1. Taking the total derivative of c with respect to t, dc dx = ct + cx . dt dt 2211

Equating terms with the partial dierential equation, we have the system of dierential equations dx =c dt dc = c. dt subject to the initial conditions x(0) = , We can solve the second ODE directly. c(, t) = c1 et c(, t) = F ()et Substituting this result and solving the rst ODE, dx = F ()et dt F () t x(t) = e + c2 F () x(t) = (1 et ) + . The solution to the problem at the point (x, t) is found by rst solving x= for and then using this value to compute c(x, t) = F ()et . 2212 F () (1 et ) + c(, 0) = F ().

2. The characteristic lines are given by the equation x(t) = F () (1 et ) + .

The points on the envelope of characteristics also satisfy x(t) = 0. Thus the points on the envelope satisfy the system of equations x= F () (1 et ) + F () 0= (1 et ) + 1. 1 et = into the rst equation we can eliminate its t dependence. x= F () + F () F ()

By substituting

Now we can solve the second equation in the system for t. F () 1 t = log 1 + F () et = 1 + 2213

Thus the equations that describe the envelope are x= F () + F () 1 t = log 1 + F ()

3. The second equation for the envelope has a solution for positive t if there is some x that satises 1 < < 0. F (x) This is equivalent to < F (x) < . So in the case that F (x) < 0, there will be breaking i max |F (x)| > . Solution 50.6 With the substitution v = V (h), the two equations become ht + (V + hV )hx = 0 (V + hV )ht + (V 2 + 2hV V + gh)hx = 0. We can rewrite the second equation as V 2 + 2hV V + gh hx = 0. V + hV Requiring that the two equations be consistent gives us a dierential equation for V . ht + V 2 + 2hV V + gh V + hV 2 2 2 V + 2hV V + h (V ) = V 2 + 2hV V + gh g (V )2 = . h V + hV = 2214

There are two choices depending on which sign we choose when taking the square root of the above equation. Positive V . V = g h

V = 2 gh + const We apply the initial condition V (h0 ) = 0. V = 2 g( h The partial dierential equation for h is then ht + (2 g( h h0 )h)x = 0 ht + g(3 h 2 h0 )hx = 0 The wave speed is c(h) = Negative V . V = g h g(3 h 2 h0 ). h0 )

V = 2 gh + const We apply the initial condition V (h0 ) = 0. V = 2 g( h0 h)

2215

The partial dierential equation for h is then ht + The wave speed is c(h) = g(2 h0 3 h)hx = 0. g(2 h0 3 h).

Solution 50.7 1. Making the substitutions, = (X), = (X), X = xU t, the system of partial dierential equations becomes U + = 0 U = . Integrating the rst equation yields U + = c = c + U . Now we substitute the expression for into the second partial dierential equation. U = (c + U ) (c + U ) c c = + + + + U U U Thus (X) satises the ordinary dierential equation = 2 + c c + . U U U

2216

2. Assume that (X) 1 as X + (X) 2 as X (X) 0 as X . Integrating the ordinary dierential equation for ,

X=

d 2 +
c U

c U

We see that the roots of the denominator of the integrand must be 1 and 2 . Thus we can write the ordinary dierential equation for (X) as (X) = ( 1 )( 2 ) = 2 (1 + 2 ) + 1 2 . Equating coecients in the polynomial with the dierential equation for part 1, we obtain the two equations Solving the rst equation for c, U 1 2 . Now we substitute the expression for c into the second equation. c= U 1 2 + = (1 + 2 ) U U 2 1 2 =+ (1 + 2 ) U c = 1 2 , U c + = (1 + 2 ). U U

Thus we see that U is U= . 2 + 2 1 2 (1 + 2 )

2217

Since the quadratic polynomial in the ordinary dierential equation for (X) is convex, it is negative valued between its two roots. Thus we see that d < 0. dX Using the expression for that we obtained in part 1, c + U 2 (c + U 1 ) 2 1 = 2 1 2 1 2 1 =U 2 1 = U. Now lets return to the ordinary dierential equation for (X) (X) = ( 1 )( 2 ) d X= ( 1 )( 2 ) 1 1 1 X= + d (2 1 ) 1 2 1 1 X X0 = ln (2 1 ) 2 1 (2 1 )(X X0 ) = ln 2 1 = exp ((2 1 )(X X0 )) 2 1 = (2 ) exp ((2 1 )(X X0 )) [1 + exp ((2 1 )(X X0 ))] = 1 + 2 exp ((2 1 )(X X0 )) 2218

Thus we obtain a closed form solution for = Solution 50.8 1. t + x + 6x xxt = 0 We make the substitution (x, t) = z(X), X = x U t. (1 U )z + 6zz + U z = 0 (1 U )z + 3z 2 + U z = 0 1 1 (1 U )z 2 + z 3 + U (z )2 = 0 2 2 2 U 1 2 (z )2 = z z3 U U 1 U 1 U 1 z(X) = sech2 X 2 2 U (x, t) = The linearized equation is t + x xxt = 0. Substituting = ex+t into this equation yields 2 = 0 = . 1 2 2219 U 1 sech2 2 1 2 U 1 x U (U 1)U t 1 + 2 exp ((2 1 )(X X0 )) 1 + exp ((2 1 )(X X0 ))

We set 2 = is then = 1 2 (U 1)/U = 1 (U 1)/U ) (U 1)U U (U 1) (U 1)U . U 1 . U

= =

The solution for becomes sech2 2 where = 2. utt uxx We make the substitution u(x, t) = z(X), 2220 X = x U t. 3 2 u 2 uxxxx = 0
xx

x t 2 . 1 2

(U 2 1)z

3 2 z z =0 2 3 2 (U 2 1)z z z =0 2 3 (U 2 1)z z 2 z = 0 2

We multiply by z and integrate. 1 2 1 1 (U 1)z 2 z 3 (z )2 = 0 2 2 2 (z )2 = (U 2 1)z 2 z 3 1 2 z = (U 2 1) sech2 U 1X 2 1 2 u(x, t) = (U 2 1) sech2 U 1x U U 2 1t 2 The linearized equation is utt uxx uxxxx = 0. Substituting u = ex+t into this equation yields 2 2 4 = 0 2 = 2 (2 + 1). We set =

U 2 1.

2221

is then 2 = 2 (2 + 1) = (U 2 1)U 2 = U U 2 1. The solution for u becomes u(x, t) = 2 sech2 where 2 = 2 (2 + 1). 3. tt xx + 2x xt + xx t xxxx We make the substitution (x, t) = z(X), X = x U t. (U 2 1)z 2U z z U z z z = 0 (U 2 1)z 3U z z z = 0 3 (U 2 1)z (z )2 z = 0 2 Multiply by z and integrate. 1 2 1 1 (U 1)(z )2 (z )3 (z )2 = 0 2 2 2 2 2 2 (z ) = (U 1)(z ) (z )3 1 2 z = (U 2 1) sech2 U 1X 2 1 2 x (x, t) = (U 2 1) sech2 U 1x U U 2 1t 2 2222 x t 2

The linearized equation is tt xx xxxx Substituting = ex+t into this equation yields 2 = 2 (2 + 1). The solution for x becomes x = 2 sech2 where 2 = 2 (2 + 1). 4. ut + 30u2 u1 + 20u1 u2 + 10uu3 + u5 = 0 We make the substitution u(x, t) = z(X), X = x U t. x t 2

U z + 30z 2 z + 20z z + 10zz + z (5) = 0 Note that (zz ) = z z + zz . U z + 30z 2 z + 10z z + 10(zz ) + z (5) = 0 U z + 10z 3 + 5(z )2 + 10zz + z (4) = 0 Multiply by z and integrate. 1 5 1 U z 2 + z 4 + 5z(z )2 (z )2 + z z = 0 2 2 2 2223

Assume that (z )2 = P (z). Dierentiating this relation, 2z z = P (z)z 1 z = P (z) 2 1 z = P (z)z 2 1 z z = P (z)P (z). 2 Substituting this expressions into the dierential equation for z, 1 5 11 1 U z 2 + z 4 + 5zP (z) (P (z))2 + P (z)P (z) = 0 2 2 24 2 4U z 2 + 20z 4 + 40zP (z) (P (z))2 + 4P (z)P (z) = 0 Substituting P (z) = az 3 + bz 2 yields (20 + 40a + 15a2 )z 4 + (40b + 20ab)z 3 + (4b2 + 4U )z 2 = 0 This equation is satised by b2 = U , a = 2. Thus we have (z )2 = U z 2 2z 3 U 1 1/4 z= sech2 U X 2 2 U 1 1/4 u(x, t) = sech2 (U x U 5/4 t) 2 2 2224

The linearized equation is ut + u5 = 0. Substituting u = ex+t into this equation yields 5 = 0. We set = U 1/4 . The solution for u(x, t) becomes 2 sech2 2 where = 5 . x t 2

2225

Part VIII Appendices

2226

Appendix A Greek Letters


The following table shows the greek letters, (some of them have two typeset variants), and their corresponding Roman letters. Name alpha beta chi delta epsilon epsilon (variant) phi phi (variant) gamma eta iota kappa lambda mu Roman a b c d e e f f g h i k l m 2227 Lower Upper

nu omicron pi pi (variant) theta theta (variant) rho rho (variant) sigma sigma (variant) tau upsilon omega xi psi zeta

n o p p q q r r s s t u w x y z

2228

Appendix B Notation
C Cn C (x) F[] Fc [] Fs [] () H(x) (1) H (x) (2) H (x) J (x) K (x) L[] class of continuous functions class of n-times continuously dierentiable functions set of complex numbers Dirac delta function Fourier transform Fourier cosine transform Fourier sine transform Eulers constant, = 0 ex Log x dx Gamma function Heaviside function Hankel function of the rst kind and order Hankel function of the second kind and order 1 Bessel function of the rst kind and order Modied Bessel function of the rst kind and order Laplace transform

2229

N N (x) R R+ R o(z) O(z) () (n) () u(n) (x) u(n,m) (x, y) Y (x) Z Z+

set of natural numbers, (positive integers) Modied Bessel function of the second kind and order set of real numbers set of positive real numbers set of negative real numbers terms smaller than z terms no bigger than z principal value of the integral d digamma function, () = d log () dn (n) polygamma function, () = d n ()
nu xn n+m u xn y m

Bessel function of the second kind and order , Neumann function set of integers set of positive integers

2230

Appendix C Formulas from Complex Variables


Analytic Functions. A function f (z) is analytic in a domain if the derivative f (z) exists in that domain. If f (z) = u(x, y) + v(x, y) is dened in some neighborhood of z0 = x0 + y0 and the partial derivatives of u and v are continuous and satisfy the Cauchy-Riemann equations ux = v y , then f (z0 ) exists. Residues. If f (z) has the Laurent expansion

uy = vx ,

f (z) =
n=

an z n ,

then the residue of f (z) at z = z0 is Res(f (z), z0 ) = a1 . 2231

Residue Theorem. Let C be a positively oriented, simple, closed contour. If f (z) is analytic in and on C except for isolated singularities at z1 , z2 , . . . , zN inside C then
N

f (z) dz = 2
C n=1

Res(f (z), zn ).

If in addition f (z) is analytic outside C in the nite complex plane then f (z) dz = 2 Res
C

1 f z2

1 z

,0 .

Residues of a pole of order n. If f (z) has a pole of order n at z = z0 then Res(f (z), z0 ) = lim Jordans Lemma.
zz0

dn1 1 [(z z0 )n f (z)] . n1 (n 1)! dz

. R 0 Let a be a positive constant. If f (z) vanishes as |z| then the integral eR sin d < f (z) eaz dz
C

along the semi-circle of radius R in the upper half plane vanishes as R . Taylor Series. Let f (z) be a function that is analytic and single valued in the disk |z z0 | < R.

f (z) =
n=0

f (n) (z0 ) (z z0 )n n!

The series converges for |z z0 | < R. 2232

Laurent Series. Let f (z) be a function that is analytic and single valued in the annulus r < |z z0 | < R. In this annulus f (z) has the convergent series,

f (z) =
n=

cn (z z0 )n , f (z) dz (z z0 )n+1

where cn =

1 2

and the path of integration is any simple, closed, positive contour around z0 and lying in the annulus. The path of integration is shown in Figure C.1.
Im(z)

Re(z) C

Figure C.1: The Path of Integration.

2233

Appendix D Table of Derivatives


Note: c denotes a constant and denotes dierentiation. d df dg (f g) = g+f dx dx dx f g fg d f = dx g g2 d c f = cf c1 f dx d f (g) = f (g)g dx d2 f (g) = f (g)(g )2 + f g dx2 dn (f g) = dxn n dn f n dn1 f dg n dn2 f d2 g n dn g g+ + + + f 1 dxn1 dx 2 dxn2 dx2 n dxn 0 dxn 2234

d 1 ln x = dx |x| d x c = cx ln c dx df dg d g f = gf g1 + f g ln f dx dx dx d sin x = cos x dx d cos x = sin x dx d tan x = sec2 x dx d csc x = csc x cot x dx d sec x = sec x tan x dx d cot x = csc2 x dx d 1 arcsin x = , dx 1 x2 arcsin x 2 2

2235

d 1 , arccos x = dx 1 x2 1 d arctan x = , dx 1 + x2 d sinh x = cosh x dx d cosh x = sinh x dx d tanh x = sech2 x dx d csch x = csch x coth x dx d sech x = sech x tanh x dx d coth x = csch2 x dx d 1 arcsinh x = dx x2 + 1 d 1 arccosh x = , 21 dx x 1 d arctanh x = , dx 1 x2

0 arccos x arctan x 2 2

x > 1, arccosh x > 0 x2 < 1

2236

d dx d dx d dx

f () d = f (x)
c c

f () d = f (x)
x h h

f (, x) d =
g g

f (, x) d + f (h, x)h f (g, x)g x

2237

Appendix E Table of Integrals


u dv dx = uv dx v du dx dx

f (x) dx = log f (x) f (x) f (x) 2 f (x) x dx = dx = x+1 +1 f (x)

for == 1

1 dx = log x x eax dx = eax a 2238

abx a dx = b log a
bx

for a > 0

log x dx = x log x x 1 1 x dx = arctan 2 +a a a


1 2a 1 2a

x2

1 dx = 2 a2 x

log ax a+x log xa x+a

for x2 < a2 for x2 > a2 for x2 < a2

a2

1 x x dx = arcsin = arccos 2 |a| |a| x

1 dx = log(x + x2 a2 ) x 2 a2 1 x2 1 a2 dx = x 1 sec1 |a| a a+ a2 x 2 x

1 dx = log a x a2 x 2 1 sin(ax) dx = cos(ax) a cos(ax) dx = 1 sin(ax) a

2239

1 tan(ax) dx = log cos(ax) a csc(ax) dx = 1 ax log tan a 2 ax 1 log tan + a 4 2 1 log sin(ax) a 1 cosh(ax) a 1 sinh(ax) a 1 log cosh(ax) a 1 ax log tanh a 2 i log tanh a i ax + 4 2

sec(ax) dx =

cot(ax) dx =

sinh(ax) dx =

cosh(ax) dx =

tanh(ax) dx =

csch(ax) dx =

sech(ax) dx =

coth(ax) dx =

1 log sinh(ax) a

2240

x sin ax dx =

1 x sin ax cos ax 2 a a 2x a2 x 2 2 sin ax cos ax a2 a3 1 x cos ax + sin ax 2 a a 2x cos ax a2 x2 2 + sin ax a2 a3

x2 sin ax dx =

x cos ax dx =

x2 cos ax dx =

2241

Appendix F Denite Integrals


Integrals from to . Let f (z) be analytic except for isolated singularities, none of which lie on the real axis. Let a1 , . . . , am be the singularities of f (z) in the upper half plane; and CR be the semi-circle from R to R in the upper half plane. If
R

lim

R max |f (z)|
zCR m

=0

then

f (x) dx = 2
j=1

Res (f (z), aj ) .

Let b1 , . . . , bn be the singularities of f (z) in the lower half plane. Let CR be the semi-circle from R to R in the lower half plane. If
R

lim

R max |f (z)|
zCR n

=0

then

f (x) dx = 2
j=1

Res (f (z), bj ) .

2242

Integrals from 0 to . Let f (z) be analytic except for isolated singularities, none of which lie on the positive real axis, [0, ). Let z1 , . . . , zn be the singularities of f (z). If f (z) z as z 0 for some > 1 and f (z) z as z for some < 1 then
n

f (x) dx =
0 0 k=1 n

Res (f (z) log z, zk ) .


n 2

1 f (x) log dx = 2

Res f (z) log z, zk +


k=1 k=1

Res (f (z) log z, zk ) z as z for some

Assume that a is not an integer. If z a f (z) < 1 then


0 0 a a

z as z 0 for some > 1 and z a f (z)


n

2 x f (x) dx = 1 e2a
n a

Res (z a f (z), zk ) .
k=1 n

2 x f (x) log x dx = 1 e2a

2a Res (z f (z) log z, zk ) , + 2 sin (a) k=1

Res (z a f (z), zk )
k=1

Fourier Integrals. Let f (z) be analytic except for isolated singularities, none of which lie on the real axis. Suppose that f (z) vanishes as |z| . If is a positive real number then
n

f (x) e

dx = 2
k=1

Res(f (z) ez , zk ),

where z1 , . . . , zn are the singularities of f (z) in the upper half plane. If is a negative real number then
n

f (x) e

dx = 2
k=1

Res(f (z) ez , zk ),

where z1 , . . . , zn are the singularities of f (z) in the lower half plane.

2243

Appendix G Table of Sums

rn =
n=1 N

r , 1r r rN +1 1r

for |r| < 1

rn =
n=1 b

n=
n=a N

(a + b)(b + 1 a) 2 N (N + 1) 2 b(b + 1)(2b + 1) a(a 1)(2a 1) 6

n=
n=1 b

n2 =
n=a

2244

n2 =
n=1

N (N + 1)(2N + 1) 6

n=1

(1)n+1 = log(2) n 1 2 = n2 6 2 (1)n+1 = n2 12 1 = (3) n3 3(3) (1)n+1 = 3 n 4 1 4 = n4 90 (1)n+1 7 4 = n4 720

n=1

n=1

n=1

n=1

n=1

n=1

2245

n=1

1 = (5) n5 (1)n+1 15(5) = 5 n 16 6 1 = n6 945 (1)n+1 31 6 = n6 30240

n=1

n=1

n=1

2246

Appendix H Table of Taylor Series

(1 z)1 =
n=0

zn

|z| < 1

(1 z)

=
n=0

(n + 1)z n

|z| < 1

(1 + z) =
n=0

n z n

|z| < 1

e =
n=0

zn n!

|z| < zn n

log(1 z) =
n=1

|z| < 1

2247

log

1+z 1z

=2
n=1

z 2n1 2n 1

|z| < 1

cos z =
n=0

(1)n z 2n (2n)! (1)n z 2n+1 (2n + 1)! z 3 2z 5 17z 7 + + + 3 15 315

|z| <

sin z =
n=0

|z| <

tan z = z + cos1 z =

|z| <

z3 1 3z 5 1 3 5z 7 z+ + + + 2 23 245 2467 z3 1 3z 5 1 3 5z 7 + + + 23 245 2467 (1)n+1 z 2n1 2n 1

|z| < 1

sin1 z = z +
1

|z| < 1

tan

z=
n=1

|z| < 1

cosh z =
n=0

z 2n (2n)! z 2n+1 (2n + 1)!

|z| <

sinh z =
n=0

|z| <

2248

tanh z = z

z 3 2z 5 17z 7 + + 3 15 315
+2n

|z| <

J (z) =
n=0

z (1)n n!( + n + 1) 2 z 1 n!( + n + 1) 2

|z| <

+2n

I (z) =
n=0

|z| <

2249

Appendix I Table of Laplace Transforms


I.1 Properties of Laplace Transforms

Let f (t) be piecewise continuous and of exponential order . Unless otherwise noted, the transform is dened for s > 0. To reduce clutter, it is understood that the Heaviside function H(t) multiplies the original function in the following two tables.

f (t)
0

est f (t) dt
c+

1 2

ets f (s) ds
c

f (s) af (s) + b(s) g sf (s) f (0)

af (t) + bg(t) d f (t) dt

2250

d2 f (t) dt2 dn f (t) dtn


t

s2 f (s) sf (0) f (0) sn f (s) sn1 f (0) sn2 f (0) f (n1) (0) f (s) s f (s) s2 f (s c) , c>0 t c f (cs) f (cs b) ecs f (s) d f (s) ds dn f (s) dsn s>c+

f ( ) d
0 t 0 0

f (s) ds d ect f (t) 1 f c t c

1 (b/c)t e f c

c>0 c>0

f (t c)H(t c), tf (t) tn f (t) f (t) , t


1 0

(1)n f (t) dt exists t

f (t) dt
s

2251

f ( )g(t ) d,
0

f, g C 0

g f (s)(s)
T 0

f (t),

f (t + T ) = f (t)

est f (t) dt 1 esT est f (t) dt 1 + esT

f (t),

f (t + T ) = f (t)

T 0

I.2

Table of Laplace Transforms

f (t)
0

est f (t) dt
c+

1 2 1 t

ets f (s) ds
c

f (s) 1 s 1 s2 n! sn+1 3/2 s 2 2252

tn , for n = 0, 1, 2, . . . t
1/2

t1/2 t
n1/2

s1/2

nZ

(1)(3)(5) (2n 1) n1/2 s 2n ( + 1) s+1 Log s s

t , Log t

() > 1

t Log t, (t) (n) (t), ect t ect

() > 1

( + 1) (( + 1) Log s) sn+1 1 s>0 s>0 s>c s>c

n Z0+

sn 1 sc 1 (s c)2 1 (s c)n c s2 + c 2 s2 s + c2 2253

tn1 ect , n Z+ (n 1)! sin(ct) cos(ct)

s>c

sinh(ct) cosh(ct) t sin(ct)

c s2 c 2 s2 s c2

s > |c| s > |c|

2cs (s2 + c2 )2 s2 c 2 (s2 + c2 )2 n Z+ n! (s c)n+1 c (s d)2 + c2 sd (s d)2 + c2 0 esc 0 1 for t < c for t > c 1 cs e s cn s + s2 + c 2 > 1 for c < 0 for c > 0 s>d

t cos(ct) tn ect ,

edt sin(ct) edt cos(ct)

s>d

(t c)

H(t c) =

J (ct)

s2 + c 2

2254

I (ct)

s2 c 2

cn s s2 + c 2

(s) > c, > 1

2255

Appendix J Table of Fourier Transforms


f (x)

1 2 F() ex d F()

f (x) ex dx

af (x) + bg(x) f (n) (x) xn f (x) f (x + c) ecx f (x) f (cx) 2256

aF () + bG() ()n F () n F (n) () ec F () F ( + c) |c|1 F (/c)

f (x)g(x) 1 1 f g(x) = 2 2 ecx , ec|x| , 2c , + c2


2

F G() =

F ()G( ) d

f ()g(x ) d

F ()G() 1 2 e /4c 4c

c>0

c>0 c>0

c/ + c2

x2

ec|| 0 e e 0 for > 0 for < 0 for > 0 for < 0

1 , x 1 , x 1 x

>0

<0

sign() 2 0 1 for x < c for x > c 1 c e 2

H(x c) =

2257

ecx H(x), ecx H(x), 1 (x )

(c) > 0

1 2(c + ) 1 2(c ) () 1 e 2 cos() sin() sin(c)

(c) > 0

((x + ) + (x )) ((x + ) (x )) H(c |x|) = 1 0 for |x| < c ,c>0 for |x| > c

2258

Appendix K Table of Fourier Transforms in n Dimensions


f (x) F() ex d
Rn

1 (2)n F()

f (x) ex dx
Rn

af (x) + bg(x) c
n/2

aF () + bG() ec
2

enx

2 /4c

2259

Appendix L Table of Fourier Cosine Transforms


f (x)

1 C() cos (x) d

f (x) cos (x) dx


0

2
0

C() S() 1 f (0) 1 f (0)

f (x) f (x) xf (x) f (cx), c>0

2 C() Fs [f (x)] 1 C c c

2260

x2

2c + c2

ec c/ + c2 1 2 e /(4c) 4c
2

ecx ecx
2

x2 /(4c) e c

ec

2261

Appendix M Table of Fourier Sine Transforms


f (x)

1 S() sin (x) d

f (x) sin (x) dx


0

2
0

S() C() 2 S() + 1 f (0)

f (x) f (x) xf (x) f (cx), c>0

Fc [f (x)]

1 S c c

2262

x2

2x + c2

ec / + c2

ecx 2 arctan 1 cx e x 1 2 x x ecx


2

2 x c

1 c e 1 arctan c 1 1 4c3/2 e
2 2 /(4c)

x x2 /(4c) e 2c3/2

ec

2263

Appendix N Table of Wronskians


W [x a, x b] W eax , ebx W [cos(ax), sin(ax)] W [cosh(ax), sinh(ax)] W [eax cos(bx), eax sin(bx)] W [eax cosh(bx), eax sinh(bx)] W [sin(c(x a)), sin(c(x b))] W [cos(c(x a)), cos(c(x b))] W [sin(c(x a)), cos(c(x b))] 2264 ba (b a) e(a+b)x a a b e2ax b e2ax c sin(c(b a)) c sin(c(b a)) c cos(c(b a))

W [sinh(c(x a)), sinh(c(x b))] W [cosh(c(x a)), cosh(c(x b))] W [sinh(c(x a)), cosh(c(x b))] W edx sin(c(x a)), edx sin(c(x b)) W edx cos(c(x a)), edx cos(c(x b)) W edx sin(c(x a)), edx cos(c(x b)) W edx sinh(c(x a)), edx sinh(c(x b)) W edx cosh(c(x a)), edx cosh(c(x b)) W edx sinh(c(x a)), edx cosh(c(x b)) W [(x a) ecx , (x b) ecx ]

c sinh(c(b a)) c cosh(c(b a)) c cosh(c(b a)) c e2dx sin(c(b a)) c e2dx sin(c(b a)) c e2dx cos(c(b a)) c e2dx sinh(c(b a)) c e2dx sinh(c(b a)) c e2dx cosh(c(b a)) (b a) e2cx

2265

Appendix O Sturm-Liouville Eigenvalue Problems


y + 2 y = 0, y(a) = y(b) = 0 n = n , ba yn = sin n(x a) ba ba 2 , nN

y n , yn = y + 2 y = 0, y(a) = y (b) = 0 n = (2n 1) , 2(b a) yn = sin

(2n 1)(x a) 2(b a) ba 2

nN

y n , yn = y + 2 y = 0, y (a) = y(b) = 0 n = (2n 1) , 2(b a) yn = cos 2266

(2n 1)(x a) 2(b a)

nN

y n , yn = y + 2 y = 0, y (a) = y (b) = 0 n = n , ba yn = cos

ba 2

n(x a) ba y n , yn =

n = 0, 1, 2, . . .

y0 , y0 = b a,

ba for n N 2

2267

Appendix P Green Functions for Ordinary Dierential Equations


G + p(x)G = (x ), G( : ) = 0
x

G(x|) = exp

p(t) dt H(x )

y = 0, y(a) = y(b) = 0 G(x|) = y = 0, y(a) = y (b) = 0 G(x|) = a x< y = 0, y (a) = y(b) = 0 G(x|) = x> b (x< a)(x> b) ba

2268

y c2 y = 0, y(a) = y(b) = 0 G(x|) = y c2 y = 0, y(a) = y (b) = 0 G(x|) = y c2 y = 0, y (a) = y(b) = 0 G(x|) = y + c2 y = 0, y(a) = y(b) = 0, c =
npi , ba

sinh(c(x< a)) sinh(c(x> b)) c sinh(c(b a))

sinh(c(x< a)) cosh(c(x> b)) c cosh(c(b a))

cosh(c(x< a)) sinh(c(x> b)) c cosh(c(b a))

nN sin(c(x< a)) sin(c(x> b)) c sin(c(b a)) nN sin(c(x< a)) cos(c(x> b)) c cos(c(b a))

G(x|) = y + c2 y = 0, y(a) = y (b) = 0, c =


(2n1)pi , 2(ba)

G(x|) = y + c2 y = 0, y (a) = y(b) = 0, c =


(2n1)pi , 2(ba)

nN cos(c(x< a)) sin(c(x> b)) c cos(c(b a))

G(x|) =

2269

y + 2cy + dy = 0, y(a) = y(b) = 0, c2 > d ecx< sinh( c2 d(x< a)) ecx< sinh( c2 d(x> b)) G(x|) = c2 d e2c sinh( c2 d(b a)) y + 2cy + dy = 0, y(a) = y(b) = 0, c2 < d, d c2 =
n , ba

nN

ecx< sin( d c2 (x< a)) ecx< sin( d c2 (x> b)) G(x|) = d c2 e2c sin( d c2 (b a)) y + 2cy + dy = 0, y(a) = y(b) = 0, c2 = d G(x|) = (x< a) ecx< (x> b) ecx< (b a) e2c

2270

Appendix Q Trigonometric Identities


Q.1 Circular Functions
sin2 x + cos2 x = 1, Angle Sum and Dierence Identities sin(x + y) = sin x cos y + cos x sin y sin(x y) = sin x cos y cos x sin y cos(x + y) = cos x cos y sin x sin y cos(x y) = cos x cos y + sin x sin y 1 + tan2 x = sec2 x, 1 + cot2 x = csc2 x

Pythagorean Identities

2271

Function Sum and Dierence Identities 1 1 sin x + sin y = 2 sin (x + y) cos (x y) 2 2 1 1 sin x sin y = 2 cos (x + y) sin (x y) 2 2 1 1 cos x + cos y = 2 cos (x + y) cos (x y) 2 2 1 1 cos x cos y = 2 sin (x + y) sin (x y) 2 2 Double Angle Identities sin 2x = 2 sin x cos x, Half Angle Identities sin2 Function Product Identities 1 1 cos(x y) cos(x + y) 2 2 1 1 cos x cos y = cos(x y) + cos(x + y) 2 2 1 1 sin x cos y = sin(x + y) + sin(x y) 2 2 1 1 cos x sin y = sin(x + y) sin(x y) 2 2 sin x sin y = Exponential Identities ex = cos x + sin x, sin x = ex ex , 2 cos x = ex + ex 2 cos 2x = cos2 x sin2 x

x 1 cos x = , 2 2

cos2

x 1 + cos x = 2 2

2272

Q.2

Hyperbolic Functions
sinh x = ex + ex ex ex , cosh x = 2 2 x x e e sinh x tanh x = = x e + ex cosh x

Exponential Identities

Reciprocal Identities csch x = Pythagorean Identities cosh2 x sinh2 x = 1, Relation to Circular Functions sinh(x) = sin x cosh(x) = cos x tanh(x) = tan x Angle Sum and Dierence Identities sinh(x y) = sinh x cosh y cosh x sinh y cosh(x y) = cosh x cosh y sinh x sinh y tanh x tanh y sinh 2x sinh 2y tanh(x y) = = 1 tanh x tanh y cosh 2x cosh 2y sinh 2x sinh 2y 1 coth x coth y coth(x y) = = coth x coth y cosh 2x cosh 2y 2273 sinh x = sin(x) cosh x = cos(x) tanh x = tan(x) tanh2 x + sech2 x = 1 1 , sinh x sech x = 1 , cosh x coth x = 1 tanh x

Function Sum and Dierence Identities 1 1 sinh x sinh y = 2 sinh (x y) cosh (x y) 2 2 1 1 cosh x + cosh y = 2 cosh (x + y) cosh (x y) 2 2 1 1 cosh x cosh y = 2 sinh (x + y) sinh (x y) 2 2 sinh(x y) tanh x tanh y = cosh x cosh y sinh(x y) coth x coth y = sinh x sinh y

Double Angle Identities sinh 2x = 2 sinh x cosh x, Half Angle Identities sinh2 Function Product Identities 1 1 cosh(x + y) cosh(x y) 2 2 1 1 cosh x cosh y = cosh(x + y) + cosh(x y) 2 2 1 1 sinh x cosh y = sinh(x + y) + sinh(x y) 2 2 sinh x sinh y = See Figure Q.1 for plots of the hyperbolic circular functions. 2274 x cosh x 1 = , 2 2 cosh2 x cosh x + 1 = 2 2 cosh 2x = cosh2 x + sinh2 x

3 2 1 -2 -1 -1 -2 -3 1 2 -2

1 0.5 -1 -0.5 -1 1 2

Figure Q.1: cosh x, sinh x and then tanh x

2275

Appendix R Bessel Functions


R.1 Denite Integrals
1 2 (J (j,n )) mn 2 0 2 1 j ,n 2 2 rJ (j ,m r)J (j ,n r) dr = J (j ,n ) mn 2 2j ,n 0 rJ (j,m r)J (j,n r) dr =
1 1

Let > 1.

rJ (m r)J (n r) dr =
0

1 2 2n

a2 2 + n 2 (J (n ))2 mn b2

Here n is the nth positive root of aJ (r) + brJ (r), where a, b R.

2276

Appendix S Formulas from Linear Algebra


Kramers Rule. Consider the matrix equation Ax = b. This equation has a unique solution if and only if det(A) = 0. If the determinant vanishes then there are either no solutions or an innite number of solutions. If the determinant is nonzero, the solution for each xj can be written xj = det Aj det A

where Aj is the matrix formed by replacing the j th column of A with b. Example S.0.1 The matrix equation 1 2 3 4 has the solution 5 6 x1 = 1 3 2 4 8 = = 4, 2 2 4 2277 1 3 x2 = 1 3 5 6 9 9 = = . 2 2 2 4 x1 x2 = 5 , 6

Appendix T Vector Analysis


Rectangular Coordinates f = f (x, y, z), g = gx i + gy j + gz k

f=

f f f i+ j+ k x y z gx gy gz + + x y z i j
y

g =

k
z

g =

gx gy f =
2

gz

f=

2f 2f 2f + 2 + 2 x2 y z 2278

Spherical Coordinates x = r cos sin , f = f (r, , ), Divergence Theorem. u dx dy = Stokes Theorem. ( u) ds = u dr u n ds y = r sin sin , z = r cos

g = gr r + g + g

2279

Appendix U Partial Fractions


A proper rational function p(x) p(x) = q(x) (x a)n r(x) Can be written in the form p(x) = (x )n r(x) a0 a1 an1 + + + n n1 (x ) (x ) x + ( )

where the ak s are constants and the last ellipses represents the partial fractions expansion of the roots of r(x). The coecients are 1 dk p(x) ak = . k! dxk r(x) x= Example U.0.2 Consider the partial fraction expansion of 1 + x + x2 . (x 1)3 2280

The expansion has the form a0 a1 a2 + + . 3 2 (x 1) (x 1) x1 The coecients are 1 (1 + x + x2 )|x=1 = 3, 0! 1 d a1 = (1 + x + x2 )|x=1 = (1 + 2x)|x=1 = 3, 1! dx 1 1 d2 a2 = (1 + x + x2 )|x=1 = (2)|x=1 = 1. 2 2! dx 2 a0 = Thus we have 1 + x + x2 3 3 1 = + + . 3 3 2 (x 1) (x 1) (x 1) x1 Example U.0.3 Consider the partial fraction expansion of 1 + x + x2 . x2 (x 1)2 The expansion has the form a0 a1 b0 b1 + + + . 2 2 x x (x 1) x1 2281

The coecients are 1 1 + x + x2 = 1, 0! (x 1)2 x=0 1 d 1 + x + x2 a1 = = 1! dx (x 1)2 x=0 1 1 + x + x2 b0 = = 3, 0! x2 x=1 1 d 1 + x + x2 b1 = = 1! dx x2 x=1 a0 = Thus we have

1 + 2x 2(1 + x + x2 ) (x 1)2 (x 1)3

= 3,
x=0

1 + 2x 2(1 + x + x2 ) x2 x3

= 3,
x=1

1 + x + x2 1 3 3 3 = 2+ + . 2 (x 1)2 2 x x x (x 1) x1

If the rational function has real coecients and the denominator has complex roots, then you can reduce the work in nding the partial fraction expansion with the following trick: Let and be complex conjugate pairs of roots of the denominator. p(x) = (x )n (x )n r(x) a0 a1 an1 + + + (x )n (x )n1 x a0 a1 an1 + + + + n n1 (x ) (x ) x

+ ( )

Thus we dont have to calculate the coecients for the root at . We just take the complex conjugate of the coecients for . Example U.0.4 Consider the partial fraction expansion of 1+x . x2 + 1 2282

The expansion has the form a0 a0 + xi x+i The coecients are 1 1+x 1 = (1 i), 0! x + i x=i 2 1 1 a0 = (1 i) = (1 + i) 2 2 a0 = Thus we have 1+x 1i 1+i = + . 2+1 x 2(x i) 2(x + i)

2283

Appendix V Finite Math


Newtons Binomial Formula.
n

(a + b) =
k=0

k nk k a b n n(n 1) n2 2 a b + + nabn1 + bn , 2 n! . k!(n k)!

= an + nan1 b + The binomial coecients are, k n

2284

Appendix W Physics
In order to reduce processing costs, a chicken farmer wished to acquire a plucking machine. Since there was no such machine on the market, he hired a mechanical engineer to design one. After extensive research and testing, the professor concluded that it was impossible to build such a machine with current technology. The farmer was disappointed, but not wanting to abandon his dream of an automatic plucker, he consulted a physicist. After a single afternoon of work, the physicist reported that not only could a plucking machine be built, but that the design was simple. The elated farmer asked him to describe his method. The physicist replied, First, assume a spherical chicken . . . . The problems in this text will implicitly make certain simplifying assumptions about chickens. For example, a problem might assume a perfectly elastic, frictionless, spherical chicken. In two-dimensional problems, we will assume that chickens are circular.

2285

Appendix X Probability
X.1 Independent Events

Once upon a time I was talking with the father of one of my colleagues at Caltech. He was an educated man. I think that he had studied Russian literature and language back when he was in college. We were discussing gambling. He told me that he had a scheme for winning money at the game of 21. I was familiar with counting cards. Being a mathematician, I was not interested in hearing about conditional probability from a literature major, but I said nothing and prepared to hear about his particular technique. I was quite surprised with his method: He said that when he was on a winning streak he would bet more and when he was on a losing streak he would bet less. He conceded that he lost more hands than he won, but since he bet more when he was winning, he made money in the end. I respectfully and thoroughly explained to him the concept of an independent event. Also, if one is not counting cards then each hand in 21 is essentially an independent event. The outcome of the previous hand has no bearing on the current. Throughout the explanation he nodded his head and agreed with my reasoning. When I was nished he replied, Yes, thats true. But you see, I have a method. When Im on my winning streak I bet more and when Im on my losing streak I bet less. I pretended that I understood. I didnt want to be rude. After all, he had taken the time to explain the concept of a winning streak to me. And everyone knows that mathematicians often do not easily understand practical matters, 2286

particularly games of chance. Never explain mathematics to the layperson.

X.2

Playing the Odds

Years ago in a classroom not so far away, your author was being subjected to a presentation of a lengthy proof. About ve minutes into the lecture, the entire class was hopelessly lost. At the forty-ve minute mark the professor had a combinatorial expression that covered most of a chalk board. From his previous queries the professor knew that none of the students had a clue what was going on. This pleased him and he had became more animated as the lecture had progressed. He gestured to the board with a smirk and asked for the value of the expression. Without a moments hesitation, I nonchalantly replied, zero. The professor was taken aback. He was clearly impressed that I was able to evaluate the expression, especially because I had done it in my head and so quickly. He enquired as to my method. Probability, I replied. Professors often present dicult problems that have simple, elegant solutions. Zero is the most elegant of numerical answers and thus most likely to be the correct answer. My second guess would have been one. The professor was not amused. Whenever a professor asks the class a question which has a numeric answer, immediately respond, zero. If you are asked about your method, casually say something vague about symmetry. Speak with condence and give non-verbal cues that you consider the problem to be elementary. This tactic will usually suce. Its quite likely that some kind of symmetry is involved. And if it isnt your response will puzzle the professor. They may continue with the next topic, not wanting to admit that they dont see the symmetry in such an elementary problem. If they press further, start mumbling to yourself. Pretend that you are lost in thought, perhaps considering some generalization of the result. They may be a little irked that you are ignoring them, but its better than divulging your true method.

2287

Appendix Y Economics
There are two important concepts in economics. The rst is Buy low, sell high, which is self-explanitory. The second is opportunity cost, the highest valued alternative that must be sacriced to attain something or otherwise satisfy a want. I discovered this concept as an undergraduate at Caltech. I was never very interested in computer games, but one day I found myself randomly playing tetris. Out of the blue I was struck by a revelation: I could be having sex right now. I havent played a computer game since.

2288

Appendix Z Glossary
Phrases often have dierent meanings in mathematics than in everyday usage. Here I have collected denitions of some mathematical terms which might confuse the novice. beyond the scope of this text: Beyond the comprehension of the author. dicult: Essentially impossible. Note that mathematicians never refer to problems they have solved as being dicult. This would either be boastful, (claiming that you can solve dicult problems), or self-deprecating, (admitting that you found the problem to be dicult). interesting: This word is grossly overused in math and science. It is often used to describe any work that the author has done, regardless of the works signicance or novelty. It may also be used as a synonym for dicult. It has a completely dierent meaning when used by the non-mathematician. When I tell people that I am a mathematician they typically respond with, That must be interesting., which means, I dont know anything about math or what mathematicians do. I typically answer, No. Not really. non-obvious or non-trivial: Real fuckin hard. one can prove that . . . : The one that proved it was a genius like Gauss. The phrase literally means you havent got a chance in hell of proving that . . . 2289

simple: Mathematicians communicate their prowess to colleagues and students by referring to all problems as simple or trivial. If you ever become a math professor, introduce every example as being really quite trivial. 1 Here are some less interesting words and phrases that you are probably already familiar with. corollary: a proposition inferred immediately from a proved proposition with little or no additional proof lemma: an auxiliary proposition used in the demonstration of another proposition theorem: a formula, proposition, or statement in mathematics or logic deduced or to be deduced from other formulas or propositions

For even more fun say it in your best Elmer Fudd accent. This next pwobwem is weawy quite twiviaw.

2290

Index
a + i b form, 184 Abels formula, 911 absolute convergence, 525 adjoint of a dierential operator, 917 of operators, 1317 analytic, 360 Analytic continuation Fourier integrals, 1554 analytic continuation, 436 analytic functions, 2231 anti-derivative, 472 Argand diagram, 184 argument of a complex number, 185 argument theorem, 500 asymptotic expansions, 1253 integration by parts, 1266 asymptotic relations, 1253 autonomous D.E., 993 average value theorem, 498 Bernoulli equations, 985 2291 Bessel functions, 1627 generating function, 1634 of the rst kind, 1633 second kind, 1649 Bessels equation, 1627 Bessels Inequality, 1299 Bessels inequality, 1343 bilinear concomitant, 918 binomial coecients, 2284 binomial formula, 2284 boundary value problems, 1110 branch principal, 6 branch point, 269 branches, 6 calculus of variations, 2066 canonical forms constant coecient equation, 1019 of dierential equations, 1019 cardinality of a set, 3 Cartesian form, 184

Cartesian product of sets, 3 Cauchy convergence, 525 Cauchy principal value, 633, 1552 Cauchys inequality, 496 Cauchy-Riemann equations, 366, 2231 chicken spherical, 2285 clockwise, 241 closed interval, 3 closure relation and Fourier transform, 1556 discrete sets of functions, 1300 codomain, 4 comparison test, 528 completeness of sets of functions, 1300 sets of vectors, 35 complex conjugate, 182, 183 complex derivative, 359, 360 complex innity, 242 complex number, 182 magnitude, 184 modulus, 184 complex numbers, 180 arithmetic, 193 set of, 3 vectors, 193 complex plane, 184 rst order dierential equations, 803 2292

computer games, 2288 connected region, 240 constant coecient dierential equations, 931 continuity, 54 uniform, 56 continuous piecewise, 56 continuous functions, 54, 535, 538 contour, 240 traversal of, 241 contour integral, 464 convergence absolute, 525 Cauchy, 525 comparison test, 528 Gauss test, 535 in the mean, 1299 integral test, 529 of integrals, 1473 Raabes test, 535 ratio test, 531 root test, 533 sequences, 524 series, 525 uniform, 535 convolution theorem and Fourier transform, 1559 for Laplace transforms, 1494 convolutions, 1494 counter-clockwise, 241

curve, 240 closed, 240 continuous, 240 Jordan, 240 piecewise smooth, 240 simple, 240 smooth, 240 denite integral, 122 degree of a dierential equation, 774 del, 157 delta function Kronecker, 35 derivative complex, 360 determinant derivative of, 907 dierence of sets, 4 dierence equations constant coecient equations, 1175 exact equations, 1169 rst order homogeneous, 1170 rst order inhomogeneous, 1172 dierential calculus, 49 dierential equations autonomous, 993 constant coecient, 931 degree, 774

equidimensional-in-x, 996 equidimensional-in-y, 998 Euler, 941 exact, 782, 946 rst order, 773, 791 homogeneous, 774 homogeneous coecient, 786 inhomogeneous, 774 linear, 774 order, 773 ordinary, 773 scale-invariant, 1001 separable, 780 without explicit dep. on y, 947 dierential operator linear, 903 Dirac delta function, 1042, 1301 direction negative, 241 positive, 241 directional derivative, 157 discontinuous functions, 55, 1340 discrete derivative, 1168 discrete integral, 1168 disjoint sets, 4 domain, 4 economics, 2288 eigenfunctions, 1333 eigenvalue problems, 1333

2293

eigenvalues, 1333 elements of a set, 2 empty set, 2 entire, 360 equidimensional dierential equations, 941 equidimensional-in-x D.E., 996 equidimensional-in-y D.E., 998 Euler dierential equations, 941 Eulers formula, 189 Eulers notation i, 182 Eulers theorem, 786 Euler-Mascheroni constant, 1616 exact dierential equations, 946 exact equations, 782 exchanging dep. and indep. var., 991 extended complex plane, 242 extremum modulus theorem, 499 Fibonacci sequence, 1180 uid ow ideal, 382 formally self-adjoint operators, 1318 Fourier coecients, 1294, 1338 behavior of, 1352 Fourier convolution theorem, 1559 Fourier cosine series, 1347 Fourier cosine transform, 1567 of derivatives, 1569

table of, 2260 Fourier series, 1333 and Fourier transform, 1543 uniform convergence, 1356 Fourier Sine series, 1348 Fourier sine series, 1432 Fourier sine transform, 1568 of derivatives, 1569 table of, 2262 Fourier transform alternate denitions, 1548 closure relation, 1556 convolution theorem, 1559 of a derivative, 1557 Parsevals theorem, 1562 shift property, 1564 table of, 2256, 2259 Fredholm alternative theorem, 1110 Fredholm equations, 1028 Frobenius series rst order dierential equation, 809 function bijective, 5 injective, 5 inverse of, 6 multi-valued, 6 single-valued, 4 surjective, 5 function elements, 436 functional equation, 388 2294

fundamental set of solutions of a dierential equation, 914 fundamental theorem of algebra, 498 fundamental theorem of calculus, 125 gamblers ruin problem, 1167, 1176 Gamma function, 1610 dierence equation, 1610 Eulers formula, 1610 Gauss formula, 1614 Hankels formula, 1612 Weierstrass formula, 1616 Gauss test, 535 generating function for Bessel functions, 1633 geometric series, 526 Gibbs phenomenon, 1361 gradient, 157 Gramm-Schmidt orthogonalization, 1286 greatest integer function, 5 Greens formula, 919, 1318 harmonic conjugate, 372 harmonic series, 527, 563 Heaviside function, 797, 1042 holomorphic, 360 homogeneous coecient equations, 786 homogeneous dierential equations, 774 homogeneous functions, 786 homogeneous solution, 793

homogeneous solutions of dierential equations, 903 i Eulers notation, 182 ideal uid ow, 382 identity map, 5 ill-posed problems, 801 linear dierential equations, 912 image of a mapping, 5 imaginary number, 182 imaginary part, 182 improper integrals, 130 indenite integral, 116, 472 indicial equation, 1202 innity complex, 242 rst order dierential equation, 814 point at, 242 inhomogeneous dierential equations, 774 initial conditions, 796 inner product of functions, 1290 integers set of, 2 integral bound maximum modulus, 466 integral calculus, 116 integral equations, 1028

2295

boundary value problems, 1028 initial value problems, 1028 integrals improper, 130 integrating factor, 792 integration techniques of, 127 intermediate value theorem, 56 intersection of sets, 4 interval closed, 3 open, 3 inverse function, 6 inverse image, 5 irregular singular points, 1218 rst order dierential equations, 812 j electrical engineering, 182 Jordan curve, 240 Jordans lemma, 2232 Kramers rule, 2277 Kronecker delta function, 35 LHospitals rule, 76 Lagranges identity, 918, 950, 1317 Laplace transform inverse, 1480 Laplace transform pairs, 1482 Laplace transforms, 1478 2296

convolution theorem, 1494 of derivatives, 1493 Laurent expansions, 626, 2231 Laurent series, 554, 2233 rst order dierential equation, 808 leading order behavior for dierential equations, 1257 least integer function, 5 least squares t Fourier series, 1340 Legendre polynomials, 1287 limit left and right, 51 limits of functions, 49 line integral, 462 complex, 464 linear dierential equations, 774 linear dierential operator, 903 linear space, 1280 Liouvilles theorem, 496 magnitude, 184 maximum modulus integral bound, 466 maximum modulus theorem, 499 Mellin inversion formula, 1481 minimum modulus theorem, 499 modulus, 184 multi-valued function, 6 nabla, 157

natural boundary, 436 Newtons binomial formula, 2284 norm of functions, 1290 normal form of dierential equations, 1022 null vector, 25 one-to-one mapping, 5 open interval, 3 opportunity cost, 2288 optimal asymptotic approximations, 1272 order of a dierential equation, 773 of a set, 3 ordinary points rst order dierential equations, 803 of linear dierential equations, 1185 orthogonal series, 1293 orthogonality weighting functions, 1292 orthonormal, 1291 Parsevals equality, 1343 Parsevals theorem for Fourier transform, 1562 partial derivative, 155 particular solution, 793 of an ODE, 1060 particular solutions

of dierential equations, 904 periodic extension, 1339 piecewise continuous, 56 point at innity, 242 dierential equations, 1218 polar form, 188 potential ow, 382 power series denition of, 539 dierentiation of, 546 integration of, 546 radius of convergence, 540 uniformly convergent, 539 principal argument, 185 principal branch, 6 principal root, 199 principal value, 633, 1552 pure imaginary number, 182 Raabes test, 535 range of a mapping, 5 ratio test, 531 rational numbers set of, 3 Rayleighs quotient, 1429 minimum property, 1429 real numbers set of, 3 real part, 182

2297

rectangular unit vectors, 25 reduction of order, 948 and the adjoint equation, 949 dierence equations, 1178 region connected, 240 multiply-connected, 240 simply-connected, 240 regular, 360 regular singular points rst order dierential equations, 807 regular Sturm-Liouville problems, 1423 properties of, 1431 residuals of series, 526 residue theorem, 630, 2232 principal values, 642 residues, 626, 2231 of a pole of order n, 626, 2232 Riccati equations, 987 Riemann zeta function, 527 Riemann-Lebesgue lemma, 1474 root test, 533 Rouches theorem, 501 scalar eld, 155 scale-invariant D.E., 1001 separable equations, 780 sequences convergence of, 524

series, 524 comparison test, 528 convergence of, 524, 525 Gauss test, 535 geometric, 526 integral test, 529 Raabes test, 535 ratio test, 531 residuals, 526 root test, 533 tail of, 525 set, 2 similarity transformation, 1893 single-valued function, 4 singularity, 376 branch point, 376 spherical chicken, 2285 stereographic projection, 243 Stirlings approximation, 1618 subset, 3 proper, 3 Taylor series, 549, 2232 rst order dierential equations, 805 table of, 2247 transformations of dierential equations, 1019 of independent variable, 1025 to constant coecient equation, 1026 to integral equations, 1028

2298

trigonometric identities, 2271 uniform continuity, 56 uniform convergence, 535 of Fourier series, 1356 of integrals, 1473 union of sets, 4 variation of parameters rst order equation, 795 vector components of, 26 rectangular unit, 25 vector calculus, 154 vector eld, 155 vector-valued functions, 154 Volterra equations, 1028 wave equation DAlemberts solution, 1939 Fourier transform solution, 1939 Laplace transform solution, 1940 Webers function, 1649 Weierstrass M-test, 536 well-posed problems, 801 linear dierential equations, 912 Wronskian, 908 zero vector, 25

2299

Вам также может понравиться